The Cleveland Clinic Cardiology Board Review, 2nd Edition

954 Pages • 654,509 Words • PDF • 68.9 MB
Uploaded at 2021-09-24 17:49

This document was submitted by our user and they confirm that they have the consent to share it. Assuming that you are writer or own the copyright of this document, report to us by using this DMCA report button.


The Cleveland Clinic Cardiology Board Review

SECOND EDITION

The Cleveland Clinic Cardiology Board Review EDITORS

SECOND EDITION

BRIAN P. GRIFFIN, MD, FACC

John and Rosemary Brown Chair in Cardiovascular Medicine

Director, Cardiovascular Disease Training Program Department of Cardiovascular Medicine Heart and Vascular Institute Cleveland Clinic Cleveland, Ohio

SAMIR R. KAP ADIA, MD, FACC

Director, Sones Cardiac Catheterization Laboratory Director, Interventional Cardiology Fellowship

Department of Cardiovascular Medicine Heart and Vascular Institute Cleveland Clinic Cleveland, Ohio

CURTIS M. RIMMERMAN, MD, MBA, FACC

Gus P. Karos Chair, Clinical Cardiovascular Medicine Department of Cardiovascular Medicine Heart and Vascular Institute Cleveland Clinic Cleveland, Ohio

I

. Wolters Kluwer Lippincott Williams & Wilkins Health

Philadelphia Buenos Aires



Baltimore •



Hong Kong

New York •

Sydney

• •

London Tokyo

Acquisitions Editor: Frances DeStefano Product Manager: Leanne Vandetty Production Manager: Alicia Jackson Senior Manufacturing Manager: Benj amin Rivera Marketing Manager: Kimberly Schonberger Design Coordinator: Stephen Druding Production Service: SPi Global Copyright© 2013 by LIPPINCOTT WILLIAMS &: WILKINS, a WOLTERS KLUWER business Two Commerce Square 2001 Market Street Philadelphia, PA 19103 USA LWWcom First Edition © 2007 by LIPPINCOTT WILLIAMS &: WILKINS

All rights reserved. This book is protected by copyright. No part of this book may be reproduced in any form by any means, including photocopying, or utilized by any information storage and retrieval system without written permission from the copyright owner, except for brief quotations embodied in critical articles and reviews. Materials appearing in this book prepared by individuals as part of their official duties as U.S. government employees are not covered by the above-mentioned copyright. Printed in China The Cleveland Clinic cardiology board review I editors, Brian P Griffin, Samir R. Kapadia, Curtis M. Rimmerman. - 2nd ed. p . ; cm. Cardiology board review Includes bibliographical references and index. ISBN 978- 1 -45 1 1 -0537-7 (hardback) - ISBN 1 -45 1 1 -0537-1 (hardback) I. Griffin, Brian P, 1 956- II. Kapadia, Samir R. III. Rimmerman, Curtis M. IV Title: Cardiology board review. [DNLM: 1 . Cardiovascular Diseases-Examination Questions. 2 . Cardiovascular Diseases-Outlines. WG 1 8 . 2 ] 6 1 6 . 1 0076-dc23 2 0 1 1 052966 Library of Congress Cataloging-in-Publication Data

Care has been taken to confirm the accuracy of the information presented and to describe generally accepted practices. However, the authors, editors, and publisher are not responsible for errors or omissions or for any consequences from application of the information in this book and make no warranty, expressed or implied, with respect to the currency, completeness, or accuracy of the contents of the publication. Application of the information in a particular situation remains the professional responsibility of the practitioner. The authors, editors, and publisher have exerted every effort to ensure that drug selection and dosage set forth in this text are in accordance with current recommendations and practice at the time of publica­ tion. However, in view of ongoing research, changes in government regulations, and the constant flow of information relating to drug therapy and drug reactions, the reader is urged to check the package insert for each drug for any change in indications and dosage and for added warnings and precautions. This is par­ ticularly important when the recommended agent is a new or infrequently employed drug. Some drugs and medical devices presented in the publication have Food and Drug Administration (FDA) clearance for limited use in restricted research settings. It is the responsibility of the health care pro­ vider to ascertain the FDA status of each drug or device planned for use in their clinical practice. To purchase additional copies of this book, call our customer service department at (800) 638-3030 or fax orders to (30 1 ) 223-2320. International customers should call (30 1) 223-2300. Visit Lippincott Williams &: Wilkins on the Internet: at LWWcom. Lippincott Williams &: Wilkins customer service representatives are available from 8:30 am to 6 pm, EST. 10 9 8 7 6 5 4 3 2 1

To our families

Contributors xi Preface xvii

1 2 N u clear Cardiac I ma g i n g : A Primer

Richard C. Brunken 1 3 N u clear Stress Testi ng

117

104

137

Richard C. Brunken and Santosh Oommen 1 4 Ca rd iac M R I a n d CT

FUNDAMEN TALS 1

Andrew C. Y To and Milind Y Desai

1 H ow to Pass the Ca rd iovasc u l a r Disease Board Exa m 1

John Rickard and Benico Barzilai

2 Cardiac P hysica l Exa m i nation

24

4

CONGESTIV E HEART FAILURE AND

Craig R. Asher and Cesar Augusto Bonilla Isaza 3 Cardiac Anatomy

Robert E. Hobbs

CARDIOMYOPATHY 155

Fai l u re

34

Miriam Jacob and W H. Wilson Tang

4 Cardiovasc u l a r P hysio logy: Fl ow-Vo l u m e

Loops

Ashley M. Lewis and Michael D. Faulx 5 Basic Ca rd iac Electrophysiology

48

Sergio G. Thal and Patrick ]. Tchou 6 Cardiac Biochemistry

155

1 5 Pathophysiology of Congestive Hea rt

1 6 Medica l Treatment of Heart Fa i l u re

43

182

Andrew Grant and Mazen Hanna

164

Arvind Bhimaraj, Celeste I Williams, and David 0. Taylor

17 Heart Tra n s p l a ntation

Masi K. Bennett and Marc S. Penn 7 C l i n ica l Epidemiology a n d Biostatistics

Michael S. Lauer and Eiran Z. Gorodeski

55

18 Devices for Heart Fa i l u re

200

Sangjin Lee and Maria M. Mountis

19 Myoca rd itis a n d Dilated Ca rd iomyopathy

Andres Schuster and W H. Wilson Tang

20 P u l monary Hypertension

225

213

Matthias Dupont and W H. Wilson Tang

CARDIOVASCULAR IMAGING AND STRESS TESTING 64

8 Ch est Rad iogra p hy for the Cardiovascular Medicine Boa rd s 64

237

2 1 Heart Fa i l u re with Normal Ejection

Andrew 0. Zurich, III and Allan L. Klein Fraction

22 Hypertrophic Ca rdiomyopathy

Anthony ]. Hart and Harry M. Lever

247

Andrei Purysko and Michael A. Bolen

78

9 Funda menta l s of Doppler

Echoca rd iog ra phy

Andrew C. Y To and L. Leonardo Rodriguez

1 0 Electroca rd iographic Stress Testing

98

Marwa A. Sabe and Julie C. Huang 1 1 Stress Echoca rd i o g ra p hy

90

L. Leonardo Rodriguez and Thomas H. Marwick

CONGENITAL HEART DISEASE 264

23 Congenita l Heart Disease i n the Ad u lt

Richard A. Krasuski and David S.Majdalany

264 vii

viii

CONTENTS

24 Esse ntial Ech oca rd iog ra p h ic I m ages i n Ad u lt Con g e n ita l Heart Disease 277

CORONARY ARTERY

Ellen Mayer Sabik

DISEASE AND CARDIAC

532

CATHETERIZATION 532

37 Eva l uation of Ch est Di sco mfo rt

S E C TIO N

Clay A. Cauthen and Donald A. Underwood

CARDIAC ELECTROPHYSIOLOGY 294

25 Twe lve-Lead E l ectroca rd iogra p hy

294

Gregory G. Bashian and Curtis M. Rimmerman

26 E l ectrop hysiologic Testing, i n c l u d i n g H i s B u n d l e a n d Other l ntraca rd iac E l ectrog ra ms 352

Edwin

I

Zishiri and Mina K. Chung

27 Sudden Card iac Death a n d Ventricu l a r Tachyca rd ia 375

29 S u p raventricu l a r Tachyca rd ias

398

4 2 Com p l ications of Myoca rd i a l I nfa rction

Olcay Aksoy and E. Murat Tuzcu

441

425

607

43 Risk Stratification and Post-Myoca rd ial I nfa rction Thera py 627

Willis M. Wu and Samir R. Kapadia

44 Radiation Safety in the Ca rd iac Catheterization La boratory 641

Imran N. Ahmad, Kevin A. Wunderle, and Frederick A. Heupler; Jr.

Khaldoun G. Tarakji and Bruce L. Wilkoff 3 2 Syncope

590

Venu Menon and Christopher M. Huff

30 Wid e-Co m p lex Tachyca rd ia: Ventri c u l a r Tachyca rd ia versu s S u p raventri c u l a r Tachyca rd ia 413 3 1 Pacema kers a n d Defi b r i l lators

575

Kellan E. Ashley and Conrad C. Simpfendorfer

4 1 Acute Myoca rd i a l I nfa rction

Christopher P. Ingelmo and P. Peter Borek

Roy Chung and Walid Saliba

39 Sta ble A n g i n a : Diag n osis, Risk Stratifi cation, Medical Therapy, and Revasc u l a rization Strateg ies 550

Gus Theodos, Anthony A. Bavry, and A. Michael Lincoff

382

john Rickard and Mohamed Kanj

Michael jolly and Leslie Cho

40 U n sta b l e Coro n a ry Syn d romes

Daniel ]. Cantillon and Oussama Wazni

28 Atria l Fibril lation a n d F l utter

38 Coro n a ry Artery Disease: Demog ra p h ics a n d I n cidence 538

45 Hemodyn a m i c Mea s u rements

654

James E. Harvey and Frederick A. Heupler; Jr.

46 Catheterization La bo ratory I m a g i n g a n d Fu nctional Assess ment 671

Fredrick ]. Jaeger

Sachin S. Goel and Samir R. Kapadia

S E C TIO N

47 Percuta neous Coro n a ry I ntervention

VALV ULAR HEART DISEASE 46I

33 Aortic a n d Pu l m o n a ry Va lve Disease

Amar Krishnaswamy and Brian P. Griffin

34 M itra I a n d Tricuspid Va lve Disease

William ]. Stewart

35 I nfective Endoca rd itis

505

461

477

AORTA/PERIPHERAL VASCULAR DISEASE 709

48 Diseases of the Ao rta

522

Peter Zimbwa and Steven M. Gordon 36 Prosthetic Va lvu l a r Disease

Matthew Cavender and Stephen G. Ellis

Lawrence Lazar and James Thomas

684

Gian M. Navaro

709

49 Venous Th ro m boembolism

721

Firas Al Solaiman and john R. Bartholomew

50 Peri phera l Artery Disease

748

735

57 Pe ricardia! Diseases

Siddharth A. Wartak and Heather L. Gornik 51 Ca rotid Disease

Amar Krishnaswamy and Mehdi Shishehbor

844

CONTENTS

Evan Lau and Allan L. Klein

58 Effects of Systemic Di seases on the Heart a n d Ca rd iovasc u l a r System 856

866

jay Sengupta and Curtis M. Rimmerman

59 Ca rd iac Neoplasms CLINICAL AND PREV EN TIV E

Michael Samara and Brian P Griffin

CARDIOLOGY 759

52 H a l l m a rks of Essentia l a n d Seco n d a ry Hypertension 759

Mohamed A. Rafey, Martin ]. Schreiber, Jr., and Joseph V Nally, Jr.

53 The Dys l i pidemias

Adam

W

781

Grasso and Michael B. Rocco

PHARMACOLOGY 876

60 P h a rmacoki n etic a n d Pha rmacodyn a m i c Essentia l s 876

Michael A. Militello

54 Preoperative Eva l uation of Ca rd iac Patients fo r Nonca rdiac S u rgery 806

61 Ca rd iovasc u l a r Medicine-Essentia l Pha rmaceutica l s 881

55 Preg na ncy a n d Heart Disease

62 Ca rd iovasc u l a r Drug I nteractions

816

Matthew C. Bunte and Richard A. Grimm

833

Amanda R. Vest, Anjli Maroa, and Russell E. Raymond 56 Wo men a n d Heart Disease

]oEllyn Moore Abraham and Ellen Mayer Sabik

911

Katherine M. Greenlee and Michael A. Militello

Michael A. Militello

Index 9 1 9

ix

JOELLYN MOORE ABRAHAM, MD Fellow Department of Cardiovascular Medicine Cleveland Clinic Cleveland, Ohio

BENICO BARZILAI, MD Section Head, Clinical Cardiology Department of Cardiovascular Medicine Cleveland Clinic Cleveland, Ohio

IMRAN NAZIR AHMAD, MD Interventional Cardiology Fellow Heart and Vascular Institute Cleveland Clinic Cleveland, Ohio

GREGORY G. BASHIAN, MD Staff Electrophysiologist Centennial Medical Center Nashville, Tennessee

OLCAY AKSOY, MD Interventional Cardiology Fellow Department of Cardiovascular Medicine Heart and Vascular Institute Cleveland Clinic Cleveland, Ohio FIRAS AL SOLAIMAN, MD Cardiology Fellow Department of Cardiovascular Medicine Cleveland Clinic Cleveland, Ohio CRAIG R. ASHER, MD, FACC Cardiology Fellowship Director Department of Cardiovascular Medicine Cleveland Clinic Florida Weston, Florida KELLAN E. ASHLEY, MD Assistant Professor Department of Cardiovascular Medicine/University Heart University of Mississippi Medical Center Jackson, Mississippi JOHN R. BARTHOLOMEW, MD Professor of Medicine Cleveland Clinic Lerner College of Medicine Section Head Vascular Medicine Department of Cardiovascular Medicine Cleveland Clinic Cleveland, Ohio

ANTHONY A. BAVRY, MD, MPH Assistant Professor Department of Medicine University of Florida Interventional Cardiologist Department of Medicine Shands at the University of Florida Gainesville, Florida

RICHARD C. BRUNKEN, MD Professor Department of Radiology Cleveland Clinic Lerner College of Medicine, Case Western Reserve University Director, Nuclear Cardiac Imaging Department of Nuclear Medicine Cleveland Clinic Cleveland, Ohio MATTHEW C. BUNTE, MD Fellow Department of Cardiovascular Medicine Cleveland Clinic Cleveland, Ohio

MOSI KADIN BENNETT, MD, PHD Fellow Department of Cardiovascular Medicine Cleveland Clinic Cleveland, Ohio

DANIEL] CANTILLON, MD Assistant Professor Lerner College of Medicine Staff Physician Cardiac Electrophysiology and Pacing Cleveland Clinic Cleveland, Ohio

ARVIND BHIMARAJ, MD, MPH Faculty, Heart Failure!Transplant Department of Cardiology The Methodist Hospital Methodist DeBakey Heart & Vascular Center Houston, Texas

CLAY CAUTHEN, MD Chief Fellow Cardiovascular Disease Training Program Heart and Vascular Institute Cleveland Clinic Cleveland, Ohio

MICHAEL BOLEN, MD Staff Physician Imaging and Heart and Vascular Institutes Cleveland Clinic Cleveland, Ohio

MATTHEW A. CAVENDER, MD Fellow Department of Cardiovascular Medicine Cleveland Clinic Cleveland, Ohio

CESAR AUGUSTO BONILLA ISAZA, MD Cardiology Fellow Department of Cardiology Cleveland Clinic Florida Weston, Florida PRZEMYSLAW PETER BOREK, MD Electrophysiology Staff Department of Cardiovascular Medicine Cleveland Clinic Cleveland, Ohio

SUNG HEE LESLIE CHO, MD, FACC Staff, Intervention Section Director, Womens Cardiovascular Center Section Head and Medical Director Preventive Cardiology and Rehabilitation Cleveland Clinic Cleveland, Ohio xi

xii

CONTRIBUTORS

MINA K. CHUNG, MD, FACC Associate Professor of Medicine Cleveland Clinic Lerner College of Medicine of Case Western Reserve University Staff Department of Cardiovascular Medicine, Heart & Vascular Institute Department of Molecular Cardiology, Lerner Research Institute Cleveland Clinic Cleveland, Ohio ROY CHUNG, MD Fellow in Cardiovascular Medicine Department of Cardiovascular Medicine Cleveland Clinic Florida Weston, Florida MILIND Y. DESAI, MD, FACC, FAHA, FESC Associate Professor of Medicine Staff Cardiologist Department of Cardiovascular Medicine Heart and Vascular Institute Cleveland Clinic Cleveland, Ohio MATTHIAS DUPONT, MD Heart Failure Fellow Heart and Vascular Institute Cleveland Clinic Cleveland, Ohio STEPHEN G. ELLIS, MD Section Head of Invasive/ Interventional Cardiology Professor of Medicine Department of Cardiovascular Medicine Cleveland Clinic Cleveland, Ohio

STEVEN MACK GORDON, MD Associate Professor of Medicine Cleveland Clinic Lerner College of Medicine of Case Western University Chairman Department of Infectious Disease Cleveland Clinic Cleveland, Ohio HEATHER L GORNIK, MD, MHS Assistant Professor of Medicine Cleveland Clinic Lerner College of Medicine of Case Western Reserve University Staff Physician, Medical Director of Non Invasive Vascular Laboratory Department of Cardiovascular Medicine Cleveland Clinic Cleveland, Ohio EIRAN Z. GORODESKI, MD, MPH Assistant Professor of Medicine Cleveland Clinic Lerner College of Medicine of Case Western Reserve University Associate Staff Department of Cardiovascular Medicine Heart and Vascular Institute Cleveland Clinic Cleveland, Ohio ANDREW D.M. GRANT, MD Clinical Fellow Department of Cardiovascular Imaging Cleveland Clinic Cleveland, Ohio

MICHAEL D. FAULX, MD, FACC Staff Cardiologist and Associate Director, Internal Medicine Residency Heart and Vascular Institute Cleveland Clinic Cleveland, Ohio

ADAM GRASSO, MD Staff Cardiologist Director, Cardiology Consult Service Department of Cardiovascular Medicine Cleveland Clinic Cleveland, Ohio

SACHIN S. GOEL, MD Clinical Associate Department of Cardiovascular Medicine Cleveland Clinic Cleveland, Ohio

KATHERINE M. GREENLEE, PHARM D Cardiology Clinical Specialist Department of Pharmacy Cleveland Clinic Cleveland, Ohio

BRIAN P GRIFFIN, MD, FACC john and Rosemary Brown Chair in Cardiovascular Medicine Director, Cardiovascular Disease Training Program Department of Cardiovascular Medicine Heart and Vascular Institute Cleveland Clinic Cleveland, Ohio RICHARD ALLEN GRIMM, DO, FACC Director, Echocardiography Department of Cardiovascular Medicine Heart & Vascular Institute Cleveland Clinic Cleveland, Ohio MAZEN HANNA, MD Director, Heart Failure Intensive Care Unit Heart and Vascular Institute Cleveland Clinic Cleveland, Ohio ANTHONY]. HART, MD Fellow Department of Cardiovascular Medicine Cleveland Clinic Cleveland, Ohio JAMES E. HARVEY, MD, MSC Chief Interventional Cardiology Fellow Heart and Vascular Institute Cleveland Clinic Cleveland, Ohio FREDERICK A. HEUPLER, JR., MD Director, Diagnostic Section, Sones Cardiac Laboratories Department of Cardiovascular Medicine Cleveland Clinic Cleveland, Ohio ROBERT E. HOBBS, MD, FACC Associate Professor of Medicine Department of Medicine Cleveland Clinic Lerner College of Medicine of Case Western Reserve University Staff Cardiologist Cleveland Clinic Cleveland, Ohio

C ONTRIBUTORS JULIE C. HUANG, MD Staff Department of Cardiovascular Medicine Cleveland Clinic Cleveland, Ohio CHRISTOPHER M. HUFF, MD Fellow Department of Cardiovascular Medicine Cleveland Clinic Cleveland, Ohio

ALLAN L. KLEIN, MD, FRCP (C), FACC, FAHA, FASE Professor of Medicine Cleveland Clinic Lerner College of Medicine of Case Western Reserve University Director of Cardiovascular Imaging Research and the Pericardial Center Department of Cardiovascular Medicine Cleveland Clinic Cleveland, Ohio

CHRISTOPHER P. INGELMO, MD Electrophysiology Fellow Department of Electrophysiology Cleveland Clinic Cleveland, Ohio

RICHARD A. KRASUSKI, MD Director of Adult Congenital Heart Disease Services Department of Cardiovascular Medicine Cleveland Clinic Cleveland, Ohio

MIRIAM S. JACOB, MD Assistant Professor Department of Internal Medicine, Division of Cardiology University of Maryland Baltimore, Maryland

AMAR KRISHNASWAMY, MD Advanced Fellow in Cardiovascular Intervention Heart and Vascular Institute Cleveland Clinic Cleveland, Ohio

FREDRICK ]. JAEGER, DO Staff, Section of Electrophysiology Director, Center for Syncope and Autonomic Disorders Department of Cardiovascular Medicine Cleveland Clinic Cleveland, Ohio

EVAN LAU, MD Advanced Fellow in Cardiovascular Intervention Heart and Vascular Institute Cleveland Clinic Cleveland, Ohio

MICHAEL A. JOLLY, MD Interventional Cardiology Fellow Department of Cardiovascular Medicine Cleveland Clinic Cleveland, Ohio SAMIR R. KAPADIA, MD, FACC Director, Sones Cardiac Catheterization Laboratory Director, Interventional Cardiology Fellowship Department of Cardiovascular Medicine Heart and Vascular Institute Cleveland Clinic Cleveland, Ohio MOHAMED KAN], MD Associate Director, Electrophysiology Labs Cardiac Electrophysiology and Pacing Cleveland Clinic Cleveland, Ohio

MICHAEL S. LAUER, MD Director Division of Cardiovascular Sciences National Heart, Lung, and Blood Institute Bethesda, Maryland LAWRENCE LAZAR, MD Interventional Fellow Department of Cardiology University of California Los Angeles, California SANGJIN LEE, MD, FACC Staff Section of Heart Failure and Cardiac Transplant Medicine Cleveland Clinic Cleveland, Ohio HARRY M. LEVER, MD, FACC Director, Hypertrophic Cardiomyopathy Clinic Heart and Vascular Institute Cleveland Clinic Cleveland, Ohio

xiii

ASHLEY LEWIS, MD Fellow Department of Cardiovascular Medicine Cleveland Clinic Cleveland, Ohio A. MICHAEL LINCOFF, MD Professor of Medicine Cleveland Clinic Lerner College of Medicine of Case Western Reserve University Vice Chairman Department of Cardiovascular Medicine Cleveland Clinic Cleveland, Ohio DAVID S. MAJDALANY, MD, FACC Assistant Professor Cleveland Clinic Lerner College of Medicine of Case Western Reserve University Cleveland Clinic Cleveland, Ohio ANJLI MAROO, MD Staff Cardiologist Fairview Hospital Cleveland, Ohio THOMAS H. MARWICK, MBBS, PHD, MPH Professor of Medicine Cleveland Clinic Lerner College of Medicine of Case Western Reserve University Head, Cardiovascular Imaging Section Heart and Vascular Institute Cleveland Clinic Cleveland, Ohio VENU MENON, MD Director, Coronary Care Unit Cleveland Clinic Cleveland, Ohio MICHAEL A. MILITELLO, PHARM D Cardiovascular Clinical Pharmacist Department of Pharmacy Cleveland Clinic Cleveland, Ohio MARIA M. MOUNTIS, DO, FACC Staff Section of Heart Failure & Transplant Department of Cardiovascular Medicine Cleveland Clinic Cleveland, Ohio

xiv

CONTRIBUTORS

JOSEPH V NALLY, JR, MD Clinical Professor of Medicine Department of Nephrology & Hypertension Cleveland Clinic Lerner College of Medicine of Case Western Reserve University Director, Center for Chronic Kidney Disease Department of Nephrology & Hypertension Cleveland Clinic Cleveland, Ohio GIAN M. NOVARO, MD, MS Director, Echocardiography Department of Cardiovascular Medicine Cleveland Clinic Florida Weston, Florida SANTOSH OOMEN, MD Cardiac Electrophysiology Fellow Department of Cardiovascular Medicine Cleveland Clinic Cleveland, Ohio MARC S. PENN, MD, PhD, FACC Director of Research Summa Cardiovascular Institute Professor Medicine and Integrative Medical Sciences Northeast Medical University Akron, Ohio ANDREI S. PURYSKO, MD Fellow in Thoracic and Abdominal Imaging Department of Radiology Cleveland Clinic Cleveland, Ohio MOHAMMED A. RAFEY, MD Adjunct Staff Nephrologist Department of Nephrology and Hypertension Cleveland Clinic Cleveland, Ohio Consultant Nephrologist Department of Nephrology Apollo Hospitals Hyderabad, India RUSSELL E. RAYMOND, DO Interventional Cardiologist Department of Cardiovascular Medicine Cleveland Clinic Cleveland, Ohio

JOHN RICKARD, MD Electrophysiology Fellow Section of Cardiac Pacing and Electrophysiology Heart and Vascular Institute Cleveland Clinic Cleveland, Ohio CURTIS M. RIMMERMAN, MD, MBA,FACC Gus P Karas Chair, Clinical Cardiovascular Medicine Department of Cardiovascular Medicine Heart and Vascular Institute Cleveland Clinic Cleveland, Ohio MICHAEL B. ROCCO, MD, FACC Assistant Professor Department of Medicine Case Western Reserve University Medical Director, Cardiac Rehab and Stress Testing Heart and Vascular Institute Cleveland Clinic Cleveland, Ohio L. LEONARDO RODRIGUEZ, MD Director, Advanced Imaging Training Program Section of Cardiovascular Imaging Heart and Vascular Institute Cleveland Clinic Cleveland, Ohio MARWA A. SABE, MD Fellow Department of Cardiovascular Medicine Cleveland Clinic Cleveland, Ohio ELLEN MAYER SABIK, MD, FACC, FASE Staff Cardiologist Department of Cardiovascular Medicine Cleveland Clinic Cleveland, Ohio WAUD SALIBA, MD Director, Electrophysiology Labs Department of Cardiovascular Medicine Cleveland Clinic Cleveland, Ohio

MICHAEL A. SAMARA, MD Fellow Advanced Heart Failure and Transplant Cardiology Cleveland Clinic Cleveland, Ohio MARTIN J. SCHREIBER, JR, MD Chairman Department of Nephrology and Hypertension Glickman Urological and Kidney Institute Cleveland Clinic Cleveland, Ohio ANDRES SCHUSTER, MD Fellow Advanced Heart Failure and Transplantation Department of Cardiovascular Medicine Cleveland Clinic Cleveland, Ohio JAY D. SENGUPTA, MD Fellow Cardiac Electrophysiology Cleveland Clinic Cleveland, Ohio MEHDI SHISHEHBOR, DO, MPH, FACC Staff, Interventional Cardiology Director, Endovascular Services Department of Cardiovascular Medicine Cleveland Clinic Cleveland, Ohio CONRAD SIMPFENDORFER, MD Staff Department of Cardiovascular Medicine Cleveland Clinic Cleveland, Ohio WILLIAM]. STEWART, MD Professor of Medicine Director of Cardiovascular Disease Curriculum Cleveland Clinic Lerner College of Medicine of Case Western Reserve University Staff Physician Department of Cardiovascular Medicine Cleveland Clinic Cleveland, Ohio

CONTRIBUTORS W

H. WILSON TANG, MD Associate Professor of Medicine Cleveland Clinic Lerner College of Medicine of Case Western Reserve University Staff Cardiologist Department of Cardiovascular Medicine Cleveland Clinic Cleveland, Ohio KHALDOUN G. TARAKJI, MD, MPH Staff Electrophysiologist Section of Cardiac Pacing and Electrophysiology Heart and Vascular Institute Cleveland Clinic Cleveland, Ohio

DAVID 0. TAYLOR, MD Staff Professor of Medicine Department of Cardiovascular Medicine Cleveland Clinic Cleveland, Ohio PATRICK ]. TCHOU, MD Staff Electrophysiologist Section of Cardiac Electrophysiology Heart and Vascular Institute Cleveland Clinic Cleveland, Ohio SERGIO THAL, MD Director, Cardiac Electrophysiology Southern Arizona VA Health Care System Assistant Professor of Medicine University of Arizona Tucson, Arizona GUS THEODOS, MD Cleveland Clinic Cleveland, Ohio JAMES D. THOMAS, MD, FASE, FACC Professor of Medicine & Biomedical Engineering Cleveland Clinic Lerner College of Medicine Case Western Reserve University Charles and Lorraine Moore Chair of Cardiovascular Imaging Department of Cardiovascular Medicine Cleveland Clinic Cleveland, Ohio

ANDREW C.Y. TO, MBCHB, FRACP Consultant Cardiologist Department of Cardiovascular Medicine North Shore Hospital Takapuna, Auckland, New Zealand E. MURAT TUZCU, MD Professor of Medicine Vice-Chairman, Department of Cardiovascular Medicine Interventional Cardiology Cleveland Clinic Cleveland, Ohio DONALD A. UNDERWOOD, MD Associate Professor of Medicine Cleveland Clinic Lerner College of Medicine of Case Western Reserve University Head, Electrocardiography Section of Clinical Cardiology Robert and Suzanne Tomsich Department of Cardiovascular Medicine Cleveland Clinic Cleveland, Ohio AMANDA R. VEST, MBBS Fellow in Cardiovascular Medicine Cleveland Clinic Cleveland, Ohio SIDDHARTH A. WARTAK, MD, MRCP Clinical Instructor Department of Internal Medicine Montefiore Medical Center Jack D. Weiler Hospital Bronx, New York OUSSAMA WAZNI, MD, FACC Director, Electrophysiology Laboratories Section of Cardiac Pacing and Electrophysiology Heart and Vascular Institute Cleveland Clinic Cleveland, Ohio BRUCE L. WILKOFF, MD Professor of Medicine Department of Cardiovascular Medicine Cleveland Clinic Lerner College of Medicine of Case Western Reserve University Cardiac Pacing & Tachyarrhythmia Devices Department of Cardiovascular Medicine Cleveland Clinic Cleveland, Ohio

xv

CELESTE T. WILLIAMS, MD, MS Assistant Professor Department of Cardiology Wayne State University Medical Director, Circulatory Support Device Program Department of Cardiology Henry Ford Hospital Detroit, Michigan WILLIS M. WU, MD Interventional Cardiology Fellow Department of Cardiovascular Medicine Cleveland Clinic Cleveland, Ohio KEVIN WUNDERLE, MSC, DABR Medical Physicist Department of Radiology Cleveland Clinic Cleveland, Ohio PETER ZIMBWA, MBCHB, MRCP, DPHIL Fellow Department of Cardiovascular Medicine Cleveland Clinic Cleveland, Ohio EDWIN T. ZISHIRI, MD Fellow Clinical Electrophysiology Cleveland Clinic Cleveland, Ohio ANDREW 0. ZURICK III, MD, FACC Medical Director of Cardiovascular Imaging Department of Cardiology St. Thomas Hospital Nashville, Tennessee

T

his book aims to provide a comprehensive overview of cardiovascular medicine in a concise and readable format. As such, it was specifically written to meet the needs of those preparing for certification or recertification examinations in cardiovascular medicine and those seeking a comprehensive update. It is based on a review course that we have organized at Cleveland Clinic over the last 1 2 years. This course has proven popular not only with those pre­ paring for examinations but also with clinical cardiologists, internists, and midlevel providers. This book would not be possible without the input and support of many individuals, specifically the faculty and

fellows in cardiovascular medicine and related disciplines at Cleveland Clinic who wrote the individual chapters and ques­ tions. We also wish to thank the Cardiovascular Graphics group at Cleveland Clinic who provided inestimable assistance to this book and to the course over the years. As with all of our endeavors, this book would also have been impossible without the support of our families, to whom this book is dedicated. We hope that you enjoy the book and that you find it useful. Brian P Griffin Samir R. Kapadia Curtis M. Rimmerman

xvi i

HoW" to Pass the C ardiovascular Disease Board Exain ] ohn Rickard and Benico Barzilai

G E N E R A L I N F O R M AT I O N The American Board of Internal Medicine (ABIM) Certifying Examination in Cardiovascular Disease takes place each year in the fall. Applicants with special conditions who require longer testing durations generally take the exam over a 4-day period starting approximately 2 weeks after the gen­ eral exam. Registration for the board examination begins in early March and lasts until early May. There is a late registra­ tion period, which incurs a late fee (generally around $400) , which lasts from early May until early June. After the late registration fee deadline has passed, registration is no longer possible. Once registered, the opportunity to cancel registra­ tion (for an 85% refund) lasts until early September. After this deadline, cancellation is still possible up until 1 day prior to the exam at a 50% refund. At the Web site, www. abim.org, applicants can register for the exam online. Other valuable information such as coding sheets and a simulated computer question format for the exam are also found at this site. As registration test centers often fill up rapidly, early registration is key to assure the ability to take the exam at a nearby test center. Test results are typically first available on the ABIM Web site in early February. For those recertifying, the test is offered twice annually, once in early April and again in early November. The deadlines to register for each test administration are in mid-February and mid-August, respectively. Board certification in internal medicine is not needed to recertify for the cardiology boards. In addition to passing the board exam, those seeking recer­ tification must also have a valid and unrestricted license to practice medicine and obtain 100 points of self-evaluation via modules available on the ABIM Web site. F O R M AT The cardiovascular diseases board exam is taken over the course of 2 days for first-time takers and 1 day for those tak­ ing it to recertify (those recertifying are exempt from the ECG and imaging sections) . The first day is a full day consisting

of four 2-hour blocks consisting of 200 multiple choice questions. The second day is a half-day consisting of an ECG section of 35 to 40 tracings lasting 2 hours 1 5 minutes and an imaging section lasting 2 hours consisting of 35 to 40 video images that include echocardiograms, ventriculo­ grams, aortograms, and angiograms. Table 1 . 1 delineates the weighted subject content for the exam. Many cardiology trainees do not have sufficient exposure to peripheral vascu­ lar disease, pharmacology, and congenital heart disease dur­ ing their training and must overcome this deficiency during their preparation for the examination. For the ECG section, a brief one- or two-line clinical vignette is provided with each ECG tracing. The test taker then must code relevant findings using a coding sheet, available in advance for review at the ABIM Web site. Similarly, for the imaging section, coding sheets are provided to capture the various findings. Of note, the coding sheet for the imaging section was updated for the 20 1 1 exam. Test takers must make sure they review the updated coding sheet prior to the test. For the 20 1 1 exam, coding sheets could be found at http://www. abim.org/pdf/ cert-related/cvd_sample_ cases. pdf. H OW YO U A R E S CO R E D For scoring, traditionally the multiple choice section and the imaging section have been combined. Starting in 20 1 1 , the imaging section was joined with the ECG section to form one component, while the multiple choice section comprises a separate component. For a passing score, both compo­ nents need to be passed. While the imaging section can be challenging, due to the combination of scoring with the ECG section, a poor performance on the imaging section can be balanced out by a stronger score on the ECG section. While there is no penalty for guessing on the multiple choice sec­ tion, there is on the ECG and imaging sections such that overcoding leads to point deductions. First-time taker num­ bers and pass rates for the exam from 2006 to 20 1 0 are listed in Table 1 .2 . 1

2

SECTION I • FUNDAMENTALS

Breakdown i n Content of the Ca rdiology Board Exa m

Medical Content Category

Arrhythmias Congestive heart failure Coronary artery disease Acute coronary syndromes/acute myocardial infarction Valvular disorders Aorta/peripheral vascular disease Hypertension/pulmonary disorders Physiology/biochemistry Pharmacology Congenital disorders Pericardial disease Miscellaneous

TI PS A 3-month study period prior to the test is a reasonable amount of time to prepare for the exam. Reviewing informa­ tion in a scheduled way over this time period is important. The examination encompasses a large quantity of information making a last-minute approach ill advised. Fellows who are enrolled in busy advanced fellowship programs such as elec­ trophysiology and interventional must be realistic about the need to study for the examination. Too often, these trainees do not allow themselves sufficient time to prepare. They may consider signing up for a dedicated course, which would force them to focus on the material covered on the examination. The multiple choice section of the boards is structured such that a clinical vignette is presented with up to five answer choices provided. While the clinical vignettes are often long, each block of 50 questions is allotted 2 hours of time (2.4 minutes/question) . Very few questions on the boards simply ask a question on a medical fact. The large

First-Ti me Ta ker Pass Rates

Yea r

N u m ber of Exa m inees/ Percentage Passing

2006 2007 2008 2009 20 1 0

767/86% 783/88% 759187% 86 119 1 % 820/90%

Relative Percentage/N u m ber of Questions

1 3 .0%/22-25 1 3 .0%/22-26 1 2 . 5 %/20-23 1 2 . 0%/2 1-25 1 2 . 0%/2 1-23 9 .0%/2 1-23 7. 0%1 1 2-14 6 .0%110-1 1 5 .0%/8-10 5 .0%/8-10 4.0%/6-8 1 . 5 %/2-3

maj ority of questions make the test taker read through a patient scenario complete with a past medical history, cur­ rent symptomatology, in-depth physical examination find­ ings, and imaging and laboratory data prior to asking how to proceed with the patient's management. In addition, the exam will often challenge the test taker to determine the most likely condition from the physical examination and then determine the treatment options based on other information given. Therefore, knowing the physical exami­ nation findings of common cardiovascular conditions is imperative . The board exam will not ask questions on any areas that are controversial or not supported by evidence. The maj ority of questions will focus on information obtained from guidelines-most notably class I and III recommendations. In preparing, it is important to focus on common thera­ peutic and diagnostic conditions rather than rare condi­ tions. Anticipate questions regarding common conditions structured in complex ways. In addition, the results of major, practice-changing clinical trials are favorite board topics. Some board questions may strike the test taker as strange or potentially even unfair. It is important not to get stressed out by such questions as the board pilots new ques­ tions every year. These new questions will not be included in the final score. Lastly, the imaging section of the boards can be difficult due to variable image quality One should ensure not to waste time overcoding but simply code the major findings that are clearly identifiable. It is also critical to make sure that all the available images have been viewed. It is vitally important not to underestimate the ECG sec­ tion. The maj ority of patients who failed the boards in the past have done so by failing this section. Knowing the cod­ ing sheet cold prior to sitting for the test is vital (the coding

CHAPTER 1 • HOW TO PASS THE CARD IOVASCULAR DISEASE B OARD EXAM sheet is available online from the ABIM) . Many test takers run into time issues with this section. Searching for the cor­ rect codes on the sheet can waste a significant amount of time and may cause some examinees not to finish. Secondly, the board exam commonly tests clinical syndromes on the ECG section. The test taker should be very familiar with the clinical syndromes on the code sheet and be able to identify such conditions rapidly. While electronic calipers are pro­ vided, they are rarely required to obtain the correct answer. Overuse of calipers can waste valuable time. It is also impor­ tant not to overcode the ECG portion of the test. The board examiners want to ensure that the examinee can identify the maj or findings on each tracing. Taking time to code small, somewhat questionable ECG findings will waste time and possibly cause point deductions. Lastly, it is important to get a good night sleep prior to the exam as the test is lengthy and can be very fatigu­ ing, especially toward the end of the examination session. Taking the exam at the first opportunity after completion of your fellowship is strongly advised as the material learned in training will be the freshest at that time. Finally, and as men­ tioned previously, early registration is important to secure a nearby test location. Having to travel large distances or

3

staying in a hotel prior to the test will only cause unneeded stress and distraction. T E N P I T FA L L S TO AVO I D 1 . Underestimating the ECG and imaging sections 2. Not being familiar with the coding sheets for the ECG and

imaging sections prior to the test 3. Not being able to identify common cardiovascular conditions 4. 5.

6.

based on physical exam findings Overcoding the ECG and imaging sections Spending a disproportionate time on one or two questions at the expense of other easier questions Getting upset by what appears to be very strange, "out of left field" -type questions that are probably pilot questions that are not factored into the final score

7. Wasting too much time with the electronic calipers on the

ECG section 8. Cramming for the test at the last minute 9. Registering late forcing the test to be administered a distance

away from home 10. Not reviewing the sample questions on the ABIM Web site

C ardiac Phy sical Exainination Craig R. Asher and C esar Augusto Bonilla Isaza

I N T R O D U CT I O N TO P H YS I C A L E X A M I N AT I O N Over the years, the bedside skills o f the cardiologist have diminished, due in part to the readily available access to echocardiography However, the cardiology boards expect a high level of understanding of physical diagnosis. Most of the testing of physical diagnosis is indirect. Many of the questions are structured with a brief history and physical exam that provide clues about the diagnosis or answer. Often these are subtle hints that will not be appreciated by the unprepared. This chapter provides many of the pearls of physical diagnosis that are important for taking the boards.

I N S P E CT I O N Basic principles (these descriptors may correlate with specific diagnoses) : • General appearance: Distress, diaphoresis, tachypnea, cyanosis, pallor • Posture: Orthopnea, platypnea/orthodeoxia (dyspnea and 0 2 desaturation in the upright position such as seen in patients wi.th patent foramen ovale (PFO) and atrial septal defect (ASD) wi.th R-to-L shunt) , trepopnea (dyspnea lying on one side but not the other such as wi.th large pleural effusions) • Stature: Tall (Marfan syndrome, Acromegaly) , short (Turner and Noonan syndrome, Down syndrome) , dwarfism (Ellis-van Creveld syndrome associated wi.th ASD) • Nutritional status: Obese (sleep apnea, metabolic syndrome) , cachexia (end-stage systolic heart failure, chronic disease, malignancy) , athletic or muscular (anabolic steroid use) • Abnormal movements: Chorea (Sydenham chorea as seen wi.th rheumatic fever) , ataxia (Friedrich ataxia associated wi.th hyper­ trophic cardiomyopathy [HCM] or tertiary syphilis associated wi.th aortic aneurysms) , head bobbing (aortic regurgitation [AR] or tricuspid regurgitation [TR] ) , Cheyne-Stokes respirations

4

See Table 2 . 1 for additional associated conditions and specific diseases found with various skin, head and neck, eye, chest and abdomen, extremity findings. A RT E R I A L P U L S E

Basic Principles • Described by upstroke, magnitude, and contour • Composed of percussion (ej ection, mid to later portion) and tidal waves (reflected wave from periphery, midlater por­ tion) • Graded 0 to 4. Grade 0 is absent; Grade 1 is barely palpable; Grade 2 is easily palpable; Grade 3 is normal; and Grade 4 is bounding. • Normal pulse pressure approximately 30 to 40 mm Hg (systolic minus diastolic blood pressure) • Anacrotic notch is present at the systolic upstroke in the arterial pulse (ascending limb). • Dicrotic notch is present in the diastolic downstroke in the arte­ rial pulse (descending limb) at aortic valve closure.

Disease States See Figure 2 . 1 . Pulsus Alternans • Alternating beat to beat strong and weak pulsations in sinus rhythm • Reflects myocardial dysfunction due to alterations in preload, afterload, and contractility wi.th each beat

Pulsus Paradoxus • Exaggeration of normal inspiratory fall of systolic blood pressure (SBP) > 10 mm Hg • Causes include cardiac tamponade, chronic lung disease/ acute asthma, pulmonary embolism (PE) , right ventricular infarction, congestive heart failure, tension pneumothorax,

CHAPTER 2 • CARD IAC PHYSICAL EXAMINATION

Physica l Exami nation Findings with Associated Conditions and Disease States Physica l Finding

Associated Cond itions (Specific Diseases)

Eyes

Roth spots, conjuctival petechiae Xanthelasmas Blue sclerae Icteric sclerae Ectopia lentis (upward displacement) Ectopia lentis (downward displacement) Conjuctivitis Corneal opacities Arcus senelis Retinal occlusion

Endocarditis Dyslipidemia (Familial hypercholesterolemia) (Osteogenesis imperfecta with aortic disease, AR, MVP) Cardiac cirrhosis (Marfan syndrome) (Homocystinuria and premature CAD, stroke, PVD) (Reiter syndrome with aortic disease, AR) (Fabry disease with HCM) (Coronary artery disease) Embolic disease

Chest and abdomen

Pectum excavatum (funnel chest) Pectum carinatum (pigeon chest) Straight back syndrome lntercostal arteries collaterals

(Marfan syndrome with aortic aneurysm, MVP) (Marfan syndrome; Noonan syndrome) (MVP; Ankylosing spondylitis with AR) (Coarctation of the aorta and bicuspid aortic valve)

Extremities

Rudimentary or absent thumbs Osler nodes, Janeway lesions, splinter hemorrhage Hyperextensible j oints Raynaud phenomenon

(Holt-Oram syndrome with ASD) Endocarditis (Ehlers-Danlos syndrome) Connective tissue disorder; Vasculitis

Skin

Jaundice Xanthomas Central Cyanosis Differential Cyanosis Peripheral Cyanosis Telangiectasias (dilated blood vessels) Lentigines (brown skin lesions) Lupus pemio (purple skin lesion) , erythema nodosum Angiofibromas (shiny papules-on face adenoma sebaceum) Striae atrophicae Bronze pigmentation Hyperextensible skin, bruising, fragile Malar rash Erythema marginatum Erythema migrans

Right heart failure; Hemolysis Dyslipidemia (Familial hypercholesterolemia) Right-to-left shunts (Eisenmenger syndrome) ; Metahemoglobinemia Right-to-left shunt with (PDA) Cardiogenic shock, severe peripheral vascular disease (Hereditary hemorrhagic telangiectasias with pulmonary AV fistula; Scleroderma) (LEOPARD syndrome; Carney syndrome with atrial myxomas) (Sarcoidosis with pulmonary HTN , arrhythmias, myopathic disease) (Tuberous sclerosis with rhabdomyomas) (Marfan syndrome with aortic aneurysm, MVP) (Hemochromatosis with supraventricular arrhythmias, cardiomyopathy) (Ehlers-Danlos syndrome with aortic aneurysm) (Lupus erythematosus with endo-, myo-, pericarditis) (Rheumatic fever with valvulitis) (Lyme disease with heart block)

Head and N eek

Elfin facies Moon facies Broad nasal bridge, narrow palpebral fissures, and micrognathia Webbed neck, hypertelorism, low set ears, short stature Bifid uvula Macroglossia

(William syndrome with supravalvular AS , branch PS) (Down syndrome with AV canal defects, ASD , VSD) (DiGeorge syndrome with Tetralogy of Fallot, Truncus arteriosus, VSD) (Noonan syndrome with HCM, PS; Turner syndrome with BAY and coarctation) (Loeys-Dietz syndrome with aortic dissection) (Amyloidosis; Down syndrome, Myxedema)

5

6

SECTION I • FUNDAMENTALS

J\ I I I I I

S 1 S2 S 1 S2 S 1 S 2 Pulsus Alternans (CH F) D

I

I

S1 S2 Hyperdynamic state

S1 S2 Parvus!Tardus with Anacrotic pulse (AS)

B

c

A

lW\_

_/\

S1 S2 Pulsus Bisferiens (AR) E

S1 S2 Dicrotic pulse F

S1 S2 Spike and Dome (HCM) G

FIGURE 2.1 Carotid pu lse findings in normal and disease states. A: The normal carotid pulse.There is a

rapid ascending and descending limb.The descending limb is slower than the ascending limb and has a dicrotic notch that occu rs during aortic valve closure.The dicrotic notch is generally not palpable on examination. B: Hyperdynamic pulse.There is a rapid, high volume ascending and descending limb. C: Parvus/tard us pu lse with anacrotic notch refers to a small-amplitude pulse with a delayed systolic pea k associated with AS.The anacrotic notch on the ascending l i m b may be appreciated on examination. D: Pulsus alternans is the beat-to-beat variation in the arterial pulse am plitude that is seen with left ventricu­ lar dysfu nction and low stroke volu me. E: Pulsus bisferiens is cha racterized by two systolic peaks during systole.The amplitude of the pu lse is high.The i nitia l pea k is due to the ejection or percussion wave, and the second peak is due to a reflected or tidal wave in the peri phery.This type of pulse is most often seen with isolated AR or combi ned AR and stenosis. F: Dicrotic pu lse is another form of double-pea ked pu lse where the dicrotic notch is present i n diastole just after S2• The dicrotic pu lse usually occu rs i n patients with hypotension due to low CO or low SVR. G: Spike and dome pu lse is another form of double-peaked pu lse that occurs with HOCM. There is an initial delayed systolic peak followed by a lower-amplitude systolic peak.

pregnancy, obesity, and rarely constrictive pericarditis (only effusive form)

• Maj or mechanisms include (a) I venous return to the right heart during inspiration with shift of the septum to the left resulting in -1 left ventricle (LV) stroke volume and therefore -1 SEP and (b) I pulmonary venous reservoir with inspiration resulting in -1 left-sided filling (lower pulmonary vein to left ventricular gradient) . • Cardiac tamponade may occur without pulsus paradoxus due to loss of interventricular dependence with high LV end­ diastolic pressure (AR or LV dysfunction) , ASD (volume of shunted blood exceeds volume of blood between inspiration and expiration) , or right ventricular hypertrophy (RVH) and pulmonary hypertension (PH) .

• The paradox is that heart sounds can be heard during inspira­ tion, while the pulse weakens and may not be palpable. • Reversed pulsus paradoxus may occur with HCM or in mechan­ ically ventilated patients.

Double-Peaked Pulse

• I amplitude pulse with two systolic peaks

• Results from accentuated percussion wave and tidal wave • Most common cause is severe AR (bisferiens) with or without aortic stenosis (AS) , though may also occur with hypertrophic obstructive cardiomyopathy (HOCM, bifid or "spike and dome") and hyperdynamic states (patent ductus arteriosus [PDA] , arteriovenous malformations) .

CHAPTER 2 • CARD IAC PHYSICAL EXAMINATION

Pulsus Ta rdus and Parvus • Tardus (slow upstroke) and parvus (low amplitude) • Caused by AS, though may be absent even in the setting of severe AS in elderly with noncompliant carotid vessels • Associated with an anacrotic pulse

Anacrotic Pulse • Notch on the upstroke of the carotid pulse (anacrotic notch) may be palpable. • Two distinct waves can be seen (slow initial upstroke and delayed peak, which is close to S 2) . • Present i n AS

7

Pressure/Pulse Difference i n Two Arms (> 1 0 m m Hg Systolic) • Due to obstruction involving the aorta, innominate and subclavian arteries due to the following etiologies: congenital, arteriosclerosis, embolism, arteritis, dissection, postsurgical (subclavian flap repair for coarctation) or external obstruction (thoracic outlet syndrome).

Historical signs of severe AR due to high stroke volume detected by pulse abnormalities include the following:

H i l l Sign • Extreme augmentation of systolic B P i n the femoral artery compared with the brachia! artery (>40 mm Hg) • Seen with severe AR

Dicrotic Pulse

• Results from a summation of waves traveling distally in the aorta

• Accentuated upstroke with second peak after dicrotic notch in diastole (after S 2)

Mayen Sign

• Second peak in diastole differentiates the dicrotic pulse from a bisferiens pulse.

• J, i n diastolic B P with arm elevation of > 1 5 m m H g

• Occurs in patients with low cardiac output (CO) and high systemic vascular resistance (SVR) or high CO and low SVR (in both cases the systolic pressure is low)

Tra ube Sign "Pistol shot"

Other miscellaneous signs/findings related to arterial pulse include the following:

Corrigan Pulse: "Water-Ham mer" Pulse

Osler Sign • Obliteration of brachia! pulse b y B P cuff with sustained palpable and rigid radial artery • Invasive BP measurements may not correlate with cuff pressures and pseudohypertension may be present.

• Loud systolic sound heard over the femoral artery

• Large-amplitude upstroke and collapse of the carotid artery pulse due to high CO and low resistance

Duroziez Sign • Systolic and diastolic bruit heard over the femoral artery with gentle compression

• Due to atherosclerotic, calcified blood vessels

J U G U LA R V E N O U S P U LS E

Pulse Deficit

Basic Principles

• Difference in the heart rate by direct cardiac auscultation and the distal arterial pulse rate when in atrial fibrillation (AF)

• Pressure and waveforms should be evaluated.

• Due to short diastoles with short RR interval, the contraction may not be strong enough to generate enough stroke volume to the periphery and thus the peripheral pulse may underestimate the heart rate.

• Adjust level of head/torso until pulsations optimally visualized. Generally around 45 degrees. • Internal jugular preferable to external jugular and right internal jugular preferable to left

• Jugular venous pulse (JVP) J, with inspiration in normal patients

Radial-to-Femoral Delay

Jugular Venous Pressu re

• Generally radial and femoral pulse occur at nearly the same time (femoral slightly earlier) .

• Measured as the vertical height above the sternal angle or angle of Louis Gunction of manubrium and sternum) , which is con­ sidered to be 5 cm above the right atrium (RA) in all positions

• Due to obstruction of arterial flow due to coarctation, the femoral pulse may be delayed.

• Confirmed by J, in lower-extremity pressure compared to upper-extremity pressure in the supine position

Asymmetric right greater than left pulses and pressures: • Supravalvular AS: The pool of blood is directed toward the right side of the aorta in greater proportion than to the left (due to the Coanda effect) resulting in a disparity in pulses and pressures, including inequality of carotid pulses.

• 9-cm H2 0 is considered elevated. • Conversion: 1 . 36 cm H 2 0 1 mm Hg =

• Abdominojugular reflux (previously referred to as the hepa­ tojugular) can be performed to confirm or determine elevated venous pressure. Application of pressure > 1 0 to 30 seconds over the right upper quadrant (RUQ) results in sustained ele­ vation of jugular pressure 2':4 cm above the sternal angle for > 1 0 seconds following release of pressure. Straining (Valsalva maneuver) must be avoided since it will cause a false reading.

8

SECTION I • FUNDAMENTALS

j\c � \:) � A

I • •

• • •

A wave: RA filling durig RA systole

C wave: U pward motion tricuspid valve in systole I carotid artery deflection

I

I

X descent: RA relaxation (during RV systole)

FIGURE 2.2 I nterna l j u g u l a r p u l sations

V wave: RA filling d u ring RV systole

Y descent: Fall i n RA pressu re when tricuspid valve opens (RV diastolic fill i ng)

S1

S2

Atrial Fibrillation

• ASD -prominent and equal "a" and V' waves

Jugular Venous Waveforms See Figure 2 . 2 .

• Constrictive pericarditis-prominent "y" descent (predomi­ nant filling during early diastole) and sometimes prominent "x" descent giving "w" shape waveform along with elevated jugular venous pressure and Kussmaul sign

Disease States See Figure 2 . 3 .

• Restrictive cardiomyopathy-prominent "x" and "y" descent may also be present similar to constrictive pericarditis.

• AF-loss o f "a" wave resulting in just one maj or positive wave

• Cardiac tamponade-prominent "x" wave and loss of the "y" descent representing loss of filling in diastole along with elevated jugular venous pressure

• Complete heart block or atrioventricular (AV) dissociation­ cannon "a" wave due to contraction against a closed tricuspid valve • Tricuspid stenosis (TS) , RVH, PH, severe left ventricular hyper­ trophy (LVH)-giant "a" waves

• Superior vena cava (SVC) obstruction-elevated but nonpul­ satile JVP

• Severe TR-large "v" wave and rapid "y'' descent

v

A Large "V" wave (TR) A

Loss or blunted "Y" descent (Cardiac Tamponade) D

i n normal i n d ivid u a l s a n d d u ri n g AF. The physiology attributed to each wave is noted. Typical ly, there a re two positive waves ("a" and "v" waves) a n d two negative waves ("x" and "y" descents) in normal i n d ividuals.The "a" wave is lost with AF. The "c" wave is not appreciable on physical exa m i nation. RA, right atri um; RV, right ventricle.

f\i!_

A

A

~

Large "A" wave (TS)

Cannon "A" wave (AV dissociation)

B

c

Prominent "X" and "Y" descent (Constrictive Pericarditis or Restrictive CM) E

A SD F

FIGURE 2.3 I nternal j u g u l a r p u l sations d u ri n g va rious disease states. A: Large ''v'' or "cv" wave cha racteristic of TR along with a ra pid "y'' descent. B: Large "a" wave as seen with obstruction to right ventricular fi l l i n g with TS. The "y" descent is slow when TS is present. A large "a" wave without a promi nent "y" descent may occur with RVH o r PH. C: Ca nnon "a" waves a re present with AV dissociation and describe the presence of i nterm it­

tent promi nent "a" waves that occur d u ri n g contraction agai nst a closed AV va lve d u ri n g ventricular systole. It should not be confused with a promi nent "v" wave. D: Loss or b l u nting of the "y" descent is an i m porta nt feature of cardiac tamponade that corresponds with i m pa i rment of d iastolic fi l l i ng. E: A promi nent "x" and "y" descent is present with either constrictive pericarditis or restrictive cardiomyopathy. The ra pid "y'' descent is a ma rker of early ra pid filling due to a n abnormal ity of com plia nce that is seen with both of these con d itions. F: The "x" descent and "y" descent with a n ASD a re equal i n a m p l itude.

CHAPTER 2 • CARD IAC PHYSICAL EXAMINATION

Other M iscellaneous Sig ns/Findings

I ntensity

• Kussmaul sign-paradoxical rise in JVP during inspiration due to increased resistance of RA filling during inspiration. The opposite of the normal fall in JVP with inspiration.

• Mitral closure is generally louder than tricuspid closure.

• Classical finding in constrictive pericarditis. May also occur with RV infarct, severe TR or TS, PE, and restrictive cardiomyo­ pathy but is absent with cardiac tamponade except for the effu­ sive constrictive form.

5 1 (particularly M1) is 1' with:

P R E CO R D I A L M OT I O N

• MS with mobile leaflets

Basic Principles • The normal apex moves toward the chest wall in early systole and is best palpated in the fourth or the fifth left intercostal space just medial to the midclavicular line.

• S 1 is generally louder than S 2 at the apex and the left sternal border and softer than S 2 at the left and the right second inter­ spaces.

• Short PR interval (due to wide separation of leaflets at onset of ventricular systole)

• Hyperdynamic LV function or i transvalvular flow due to shunts Ci force of leaflet closure) • TS or ASD (T 1 i)

• It is 1 to 2 cm in size and lasts less than one-third of systole.

5 1 is J with:

• The apical pulsation is not always the point of maximal impulse (PMI) (e.g. , in rheumatic mitral stenosis (MS) , the PMI may be produced by the right ventricle) .

• MS with immobile or calcified leaflets

Hypertrophy • LVH results in an apical impulse that is sustained and not diffuse. • RVH or PH results in a left parasternal heave or lift that is sustained and not diffuse.

• Long PR interval (leaflets close together at onset of ventricular systole) • Severe AR (due to mitral preclosure from the jet hitting the mitral valve and high left ventricular end diastolic pressure [LVEDPJ) • Severe LV dysfunction with poor CO ( 1 force of leaflet closure) • MR due to prolapse or flail (poor coaptation of leaflets)

5 1 is variable with:

Dilation

• Atrial fibrillation

• LV enlargement results in a diffuse, laterally displaced apical impulse.

• Complete heart block and AV dissociation

• RV enlargement results in a diffuse impulse occurring in the parasternal region.

Disease States • LV aneurysms may produce diffuse outward bulging and a rocking effect. • Constrictive pericarditis may be characterized by systolic retrac­ tion of the chest instead of outward motion (Broadbent sign) . • Hyperactive precordium occurs in volume overload (severe aor­ tic and mitral regurgitation [MR] , large left-to-right shunt) . • HCM causes a double systolic outward motion. This is due to a palpable "a" wave (increased atrial filling) and a sustained outward movement of the apex. In some patients, there are two systolic motions as well as the motion during atrial systole resulting in a triple apical impulse.

9

Splitting • Split S 1 must be differentiated from an S4 gallop heard best at the apex with the bell of the stethoscope and an ej ection sound (ES) (pulmonic or aortic) heard at the base of the heart.

Persistent splitting: • Late T1 closure due to severe TS , ASD or right bundle branch block (RBBB) • Late T 1 closure due to Ebstein anomaly (S2 also split) with asso­ ciated multiple systolic and diastolic clicks "sail-like sounds" • Early M1 closure due to LV preexcitation

Reverse splitting (rare): • Late M 1 closure due to severe MS (usually associated with TR) , left bundle branch block (LBBB) , RV pacing

F I R S T H E A RT S O U N D

Basic Principles • Ventricular systole begins with closure of the mitral (first) and tricuspid (second) valves. • S1 is best heard with the diaphragm of the stethoscope at the apex for the mitral and the left sternal border for the tricuspid valve. • Opening sounds of the mitral and tricuspid valves are patho­ logic sounds.

S E CO N D H E A RT S O U N D

Basic Principles • Ventricular systole ends with closure of the aortic (first) and pulmonic (second) valves. • S2 closure sounds are heard best with the diaphragm of the stethoscope in the second left and right intercostal spaces near the sternum.

10

SE CTI ON I • FUNDAMENTALS I nspiration

I ntensity • Aortic closure heard best at the second right intercostal space adja­ cent to the sternum is generally louder than pulmonic closure heard best at the second left intercostal space adjacent to the sternum. • S 2 (A2 ) is i with hypertension (HTN) , dilated aorta. • S 2 (A2 ) is J- with AS. • S 2 (P2 ) is i with pulmonary HTN , dilated pulmonary artery (PA) . S2 (P2) is J, with pulmonary stenosis (PS) .

Expi ration

Normal (physiological)

Persistent

Fixed

Single S2

• A2 is absent with severe AS. • P2 is absent with chronic obstructive pulmonary disease (COPD) and obesity (inaudible sound due respiratory noise) or PS, pul­ monary atresia, right ventricular outflow tract (RVOT) obstruc­ tion, and Tetralogy of Fallot. • A2 -P2 occur together with aging due to decreased inspiratory delay of P 2 .

Splitting Normally A2 and P2 separate during inspiration and come together during expiration (physiologic splitting) (Fig. 2.4). This occurs due to J, pulmonary vascular impedance and relatively longer RV ejection period relative to LV ejection period. • Splitting of the S 2 may be physiologic or pathologic.

Pathologic splitting: a.

Fixed splitting-wide and persistent splitting that remains unchanged throughout the respiratory cycle Conditions-ASD ( 70% secundum ASD when hemo­ dynamically significant) , RV failure (most common cause in adults) , PS, Partial anomalous pulmonary venous return (usu­ ally with sinus venosus ASD) , ventricular septal defect (VSD) with left-to-right shunt (A2 closure is early) b. Persistent splitting-splitting occurs with both inspiration and expiration but is not fixed with a further widening occurring with inspiration. • Conditions: 1 . P2 delayed-RBBB, pulmonary HTN , RV dysfunction, PS, dilated PA 2 . A2 early-severe MR, VSD , Wolf-Parkinson-White (WPW) (LV pre-excitation) c. Paradoxical splitting-the normal sequence of A2 followed by P 2 closure is reversed so that so that with expiration P 2 precedes A2 and with inspiration the sounds come together. • Conditions: 1 . A2 delayed-LBBB or RV pacing, AS , LV dysfunction, HCM, Dilated aorta or Ischemia 2. P2 early-WPW (RV preexcitation) -

Paradoxical

FIGURE 2.4 I l l ustration of normal 52 (physiolog ic) splitting and pathologic 52 splitting (persistent, fixed, pa radoxica l) with the changes that occur as a result of the respi ratory cycle. With normal physiologic spl itting, P2 closure occurs later than A2 closure during inspiration with associated increased preload and a longer right ventricu lar ejection period. During expiration, a single 52 sound is heard. With persistent spl itting, A2 and P2 a re heard throughout the respiratory cycle but separated by a wider distance d u ring inspiration.This is due either to a delay in the closure of P2 or a n early closure o f A 2 • Fixed splitti ng may occur with hemodynami­ cally sig nificant A5Ds and describes the equal and persistent

separation of A2 and P 2 d u ring the respiratory cycle. Paradoxical splitting is the opposite of normal splitting (P2 precedes A2 ) d u r­ ing expiration, and a single sound is heard d uring inspiration.This is due to either a delay in A2 closure or an early P 2 closure.

• Best heard with light pressure of the bell of stethoscope (low frequency) in the left lateral decubitus position at the apex

• Right-sided S3 can be heard at left sternal border and may i with inspiration.

• Most commonly heard in conditions of high flow across an AV valves

• S3 follows an opening snap (OS) and pericardia! knock (PK) in timing. • S3 corresponds with the "y" descent of the central venous or atrial waveform or the Doppler E wave on an echocardiogram. • An S 3 is not expected with severe MS.

F O U RT H H E A RT S O U N D

Basic Principles • S4 is usually pathologic (atrial gallop) .

T H I R D H E A RT S O U N D

Basic Principles • Physiologic sound in young adults though may disappear with standing. Almost all adults lose S3 after 40 years old. • It is normal during the third trimester of pregnancy

• S4 is heard best with the bell of the stethoscope and occurs just before S 1 , after the P wave on the EKG and is equivalent to the Doppler A wave on an echocardiogram. • A left-sided S4 is heard best in the left lateral decubitus position at the apex during expiration and a right-sided S4 is heard at the left sternal border to midsternum best with inspiration.

CHAPTER 2 • CARD IAC PHYSI CAL EXAMINATION • Common pathologic states associated with a left-sided S4 include-AS, HTN , HCM, and Ischemic heart disease. A right­ sided S4 is heard with PH and PS.

Systole

11

Diastole

• S4 gallop is not heard with AF • When S3 and S4 are heard simultaneously such as may occur with tachycardia and prolonged PR intervals, a "summation gallop" (SG) is present. • A quadruple rhythm with a distinct S3 and S4 may be heard with tachycardia.

E X T R A H E A RT S O U N D S

Diastole See Figure 2 . 5 .

MC

LC

FIGURE 2.6 The relative timing of hea rt sounds heard d u ring systole is shown. An ES is the earl iest systolic sound audible and is heard just after S, but occu rs before the ca rotid p u l sation. Nonejection clicks a re usua l ly midsystolic or late systo lic and a re most com m o n ly caused by MVP. MC, midsystolic click; LC, l ate systolic click.

Ope n i n g S n a p • Pathologic sound generated b y abrupt movement of the body of the mitral leaflets in early diastole due to MS or tricuspid stenosis (TS) • OS is a high-pitched sound best heard medial to the apex with the diaphragm of the stethoscope. • If the valve is not mobile or MR is present, an OS may not occur. • An interval of the murmur becomes longer and may be louder.

• With aortic valve prostheses, any decrescendo AR murmurs should be considered abnormal.

Perica rd ia! Frictio n Ru bs

• With mitral valve prostheses, any holosystolic MR murmurs should be considered abnormal.

• Pericardia! rubs are high-pitched, dynamic, and scratchy sounds.

Pacemaker Sounds

• They are best heard with the patient leaning forward (or on elbows and knees) following forced held expiration or deep held inspiration.

• High-frequency click sound heard in patients with either endo­ cardial or epicardial pacemakers thought due to stimulation of skeletal muscle contraction (intercostal or pectoral muscles)

• Three components may be heard, (a) atrial systole, (b) ventricu­ lar systole, and (c) rapid ventricular filling.

• A pacemaker sound is a presystolic click occurring immediately after the pacemaker stimulus and therefore may be confused with an atrial gallop or a loud S 1 sound.

• Generally only one or two components will be heard. When one component is heard, it is generally the systolic component that can be confused with systolic murmurs.

H E A RT M U R M U R S

• The presence of a pericardia! rub does not correlate well with the volume of pericardia! effusion. Pericardia! rubs may occur with large pericardial effusions (several mechanisms contribute to generating the sound) .

• Heart murmurs are due to turbulence of blood flow either due to structural abnormalities or increased blood flow velocity.

• A mediastinal crunch (Hamman sign) is due to air in the pericardium or mediastinum (as may occur after cardiac sur­ gery) and may be associated with subcutaneous emphysema.

• Heart murmurs are characterized in many ways including ( 1 ) timing (systolic, diastolic, or continuous) and (2) clinical significance (benign or pathologic) .

• Pleural rubs are accentuated during inspiration.

• Systolic murmurs may be further classified based on timing of onset and termination as holosystolic, midsystolic, early systolic, and late systolic.

Prosthetic Heart Sounds • The intensity of the opening and closing sounds varies according to the type and design of the prosthetic valve. • With ball-cage valves (Starr-Edwards) , the opening click (OC) is louder than the closing click (CC) for both aortic and mitral prostheses. • With bileaflet or tilting disc valves, the CC is louder than the OC for both aortic and mitral prostheses. • A decrease in the intensity of the OC or CC or a change in the relative intensity of the clicks for a given prosthesis should be considered abnormal.

Basic Principles

• Diastolic murmurs may be further classified based on timing of onset as early diastolic, middiastolic, and late diastolic. • Heart murmurs are also described based on location heard, shape (e.g. , crescendo-decrescendo, plateau) , intensity (I-VI), pitch or frequency (e .g. , high-pitched sounds like AR due to high-pressure gradient versus low-pitched sounds like MS due to low-pressure gradients) , quality (e .g. , musical, harsh) , radiation, accompanying sounds , and response to maneuvers. • Systolic murmurs are further characterized as ( 1 ) ej ection and (2) regurgitant.

CHAPTER 2 • CARD IAC PHYSI CAL EXAMINATION •

• Ej ection systolic murmurs are diamond shaped, low or medium frequency, begin after S1 and end before S2 , and increase in intensity after a long cycle length or PVC.



• Regurgitant systolic murmurs are often holosystolic, high frequency, begin with S 1 and or extend to and touch S 2 , and do not change in intensity after a long cycle length or PVC. • Ej ection murmurs usually result from blood flow through a semilunar valve and regurgitant murmurs result from blood flow through an atrioventricular valve or a ventricular defect.

SYSTO L I C M U R M U R S

Systol ic Murmurs: Ejection Type See Figures 2.8 and 2 . 9 . •

1 . Aortic valvular stenosis: •



• •



Location: heard best with the diaphragm at the aortic area Description: mainly harsh, medium pitch with a crescendo/ decrescendo configuration. In elderly, there is a high-pitched musical murmur that may be heard radiating to the apex (Gallavardin murmur) . This may mimic an MR murmur. Radiation: into the neck and great vessels though it may be toward the apex in elderly, but not beyond the apex Intensity: related to stroke volume and severity and therefore may or may not reflect the severity of stenosis (e.g. , mild AS with high stroke volume may be loud whereas severe AS with low stroke volume may be soft) Severity: severe AS is characterized based on an i in ejection time (longer duration and delayed peaking) .

Maneuvers: AS murmur may ..[, following Valsalva and i post PVC. Associated findings: - Prominent "a" waves ( ..[, RV compliance because of septa! hypertrophy-Bernheim effect) - "Parvus (reduced) and tardus (slow)" carotid upstroke with anacrotic pulse. Not always present in the elderly with stiff vessels. - Thrill over the carotid pulse (shudder) - Precordial thrill - Apical impulse is sustained, nondisplaced. - Early ES heard with congenital stenosis - A2 intensity ..[, or absent with severe AS - Second heart sound is single (P2) or may be paradoxically split. - Palpable and audible S4 - Reduced pulse pressure Variations: Congenital supravalvular AS is heard best at the first or the second right interspace and is associated with radiation toward the right carotid artery with relatively ..[, left-sided pulses. A2 may be increased with this form of AS (Table 2 2)

2. Aortic sclerosis: •

• •





Location: right upper sternal border, heard best with the diaphragm Description: soft Radiation: does not radiate widely Intensity and Severity: related to flow, early peaking Associated findings: no associated findings of AS , normal S 2 , and no radiation to the carotids

S1 Mild AS

Moderate Congen ital AS

13

Moderate AS

Severe AS

Mild Congenital PS

Severe PS

FIGURE 2.8 The systolic ejection m u r m u rs due to AS and p u l monic stenosis (PS). The severity of stenosis is associated with the time to pea k a n d the d u ration of the m u r m u r as wel l as the associated fi n d i ngs. With severe AS, an S4 gallop and paradoxica l S 2 spl itti ng may be present. With severe PS, a rig ht-sided S4 gallop a n d persistent S 2 spl itti ng may be present. With congenital AS a n d PS, a n ES may come before the m u r m u r. With increasi ng severity of PS and AS, the correspond ing P 2 and A2 com po­ nent of the second hea rt sound gets fa i nter.

14

SE CTI ON I • FUNDAMENTALS • •



84 8 1

82

HCM with M R



ASD F I G U R E 2 . 9 Systolic ejection m u rm u rs due t o H O C M and ASD. The m u r m u r of HCM has a crescendo-decrescendo pattern. I n some patients, LVOT obstruction res u lting from systolic anterior motion of the m itra l leaflet ca uses MR.This second systolic m u r­ m u r is difficult to disti nguish from the ejection-type sound. It has the q u a l ities of a reg u rgitant murmur extending to the 5 2 sound and extending to axi lla.The murmur of a n ASD typica lly is due to a n ejection pulmonary outflow sound related to increased stroke vol u me.There a lso may be a diastolic rumble across the tricuspid va lve related to increased flow enteri ng the right ventricle. 3 . Hypertrophic cardiomyopathy: •



Location: left ventricular outflow tract (LVOT) obstruction murmur is heard best along the mid and the lower left sternal edges. Description: harsh

4.

Radiation: LVOT obstruction murmur may be widely trans­ mitted, although not usually heard at the neck. Intensity and Severity: related to the degree of obstruction Maneuvers: hemodynamic changes that affect LV volume, contractility, and vascular resistance help differentiate HOCM from AS: - Standing J. AS and i HOCM - Valsalva (straining phase) i the murmur of HOCM and J. or does not change the murmur of AS - Amyl nitrite i the murmur of HOCM and AS. - Post PVC, the murmur of HOCM and AS is i . Associated findings: - i "a" wave (Bernheim effect) - Murmur of MR occurring in midlate systole may be present due to systolic anterior motion of the mitral valve. - Murmur of RVOT obstruction may be present at the left upper sternal border in rare circumstances. - Brisk carotid upstrokes sometimes bifid, "spike and dome. " If carotid upstroke i s reduced, contemplate a n alternative diagnosis. - Sustained LV apical impulse, double or triple thrust - S2 paradoxical split - S4 gallop

Pulmonic stenosis: Location: heard best in the pulmonary area • Description: harsh, medium pitch, crescendo/decrescendo • Radiation: directed to the left shoulder, back, lung fields, and neck • Intensity: depends on stroke volume and severity • Severity: characterized by the duration of murmur and time to peak • Maneuvers: The murmur i with inspiration. • Associated findings: - i "a" wave •

Distinguishing Featu res between Left Heart Obstructive Conditions Featu re Pulses

Carotid pulse Pulse pressure after PVC

Va lvu lar

Suprava lvular

S u bvalvular

HOCM

Normal to J.. J.. i

Asymmetric i

Normal to J.. J.. i

Brisk, Spike, and Dome J.. (Brockenbrough Sign)

Common in severe disease Common Common with bicuspid valve without calcification

Uncommon

Uncommon

Common

No No

Rare No

Common No

J..

J..

J..

i

Common

Rare

Sometimes

Rare

Heart sounds

Four heart sound Paradoxical splitting Ejection click

Murmurs

Valsalva effect on murmur Murmur of AR

CHAPTER 2 • CARD IAC PHYSI CAL EXAMINATION

5.

6.

- Sustained sternal lift or heave - Normal S 1 followed by ejection click (EC) that may not be present in dysplastic leaflets - Absent or J, P2 - Widely persistent split S 2 - Early pulmonic ES that J, with inspiration - Right-sided S4 (atrial gallop t with inspiration) - Murmur of TR - Elevated ]VP

Innocent murmur in children: (Still murmur) : • Location: left lower sternal border or apex • Description: low-medium frequency, vibratory or buzzing, short midsystolic • Radiation: usually none • Intensity and Severity: related to stroke volume but usually soft • Maneuvers: may change in intensity or disappear with differ­ ent positions, such as standing Innocent murmur in children to young adults: (Pulmonary ejection murmur) : • Location: pulmonary area • Description: high frequency, early to midsystolic crescendo­ decrescendo • Radiation: usually none • Intensity and Severity: related to stroke volume but usually soft

Systolic Murmurs: Regurgitant Type See Figure 2 . 1 0 .

• •



• •



Maneuvers: may t with expiration and during isometric handgrip Variations: - MR due to posterior prolapse may be anteriorly directed toward the left sternal border and neck - MR may not be holosystolic, following a click it may be mid or late systolic and it may be early systolic with acute MR (rapid equalization of pressures) Associated findings: - Laterally displaced apical impulse - J, S1 - Mid to late systolic click, and late systolic murmur in patients with MVP - S3 - S2 (P2 ) may be t when PH occurs.

2. Tricuspid regurgitation: •



• •

• • •

1 . Mitra! regurgitation: •

15

Location: usually heard best with the diaphragm at the apex Description: blowing, high pitched Radiation: typically into the left axilla unlike with AS Intensity and Severity: variable related to BP, loading conditions, mechanism and acuity

Location: heard best along the lower sternal border but also along right sternal border Description: blowing, high pitched Radiation: to the right side, not beyond the axilla as with MR Intensity: may t with inspiration (Carvallo sign) , though sometimes even severe TR is not loud and may not t with inspiration (RV failure when RV volume does not change) Severity: may not be related to intensity though always with elevated ]VP Variations-if RV is severely dilated occupying the left precor­ dium, then TR may be heard toward the apex. Associated findings: - Left parasternal lift (due to RV hypertrophy) - Elevated JVP with large "v'' or "cv" wave with rapid "y" descent with obliterated "x" descent. - Right -sided S3 - Diastolic rumble at the left sternal border, narrow split S2 , and t P2 if it is due to PH - Pulsatile liver - Right heart failure signs

3. Ventricular septal defect: •

• •



M1

A2

MR without MVP

A2

M1 C

MR with MVP

A2

M1

Acute Severe M R







lschemic MR

TR

VSD

FIGURE 2.1 0 Reg u rg itant-type m u r m u rs.The ti ming of the m u r­ m u rs is shown with most reg u rg itant m u r m u rs, M R, TR, and V5D extending from 51 to 5 2• However, some reg u rg itant m u r m u rs a re not holosysto lic. Exa m ples include acute severe MR where there is ra pid equalization of the left atrial a n d ventricu l a r pressu res

resulting in an early systolic m u r m u r, the click m u r m u r of MVP and ischemic MR associated with pa p i l l a ry m uscle dysfu nction.



Location: around the lower sternum Description: harsh and high pitched Radiation: toward the sternum and not to the axilla Intensity: generally loud but depends on the size of the shunt Severity: usually accompanying thrill though the intensity of the murmur is not proportional to the degree of shunt (a loud, restrictive murmur is generally small, and a soft non­ restrictive murmur is generally a large shunt) Maneuvers: does not t with inspiration as does TR Variations: - Depends of the relative compliance of the LY/aorta and RV/ PA-may be early systolic when PH is present - If heard best in the first and second left intercostal spaces and radiating to the left clavicle, suspect supracristal defect or PDA. Associated findings: - Thrill - A2 is usually normal. - P 2 is normal or t. - A diastolic rumble may be present due to increased flow across the mitral valve.

16

SE CTI ON I • FUNDAMENTALS

D I A STO L I C M U R M U R S See Figures 2 . 1 1 and 2 . 1 2 . 1 . Mitral stenosis: •

• •





• •

Location: localized around the apex, heard best in left lateral decubitus position Description: low-pitched diastolic rumble heard best with the bell and crescendo in late diastole Radiation: none Severity: related to duration of the murmur, not to the inten­ sity. Ai-OS interval related to severity Maneuvers: i with amyl nitrite and exercise due to tachycardia Variations: early to mid diastolic rumble may be heard with­ out stenosis due to i flow (i. e . , large VSD , PDA) . Associated findings: - S 1 may be i if pliable leaflets. - OS present with .,[, OS to Ai interval - i Pi and left parasternal lift if PH - AF is common - TR or MR murmurs may be present. - TS murmur may be present. - Elevated ]VP with large "v" waves may be present with pulmonary HTN and associated TR.

2. Aortic regurgitation: •







Location: left or right sternal border Description: blowing, high-pitched decrescendo, heard best with the diaphragm, begins with A2 and heard best sitting, leaning forward in expiration Radiation: if heard best with radiation to the right sternal border, suspect aortic root disease, and if heard best with radiation to the left sternal border or apex, suspect leaflet abnormalities. Intensity: related to the severity and acuity of the lesion dependent on the difference between the aortic and the LV diastolic pressure gradient



S1

PR with Pul monary Hypertension

A2 P 2

A2 Chronic AR

S1

A2 0S Severe MS

·· · .

··· ··

··

.. . . . . · ..

·

j

.

· ·' ·

FIGURE 2.1 2 Diasto l i c m u r m u rs of AR. The m u r m u r of c h ronic AR is a decrescendo murmur beg i n n i n g after Si. There often is a n associated loud systolic ejection m u r m u r due to high stroke vol u me.The m u rm u r of acute severe AR is decrescendo i n confi g u ration b u t is brief i n d u ration due t o ra pid eq ual ization between aortic diastolic and left ventricu l a r diastolic pressu re. The systolic ejection m u r m u r in acute severe AR is genera l ly less intense compared to chronic AR since the stroke vol u m e is not able to increase acutely. The Austi n Flint m u rm u r is a diastolic flow m u rm u r that occurs in patients with AR and m i m ics the diastolic rumble of MS. It beg i n s after a n S3 gal lop. Associated sounds (OS, S, intensity) and maneuvers aid to differentiate a n Austi n F l i nt m u r m u r from a n MS m u r m u r.



• •

S1

FIGURE 2.1 1 Diastolic murmurs. The diastolic decrescendo PR

murmur associated with PH is cal led the Graham Steel l murmur. The murmur follows a loud Pi sound. In contrast, the PR m u rm u r unrelated t o PH starts after t h e P i sound. T h e m u r m u r o f MS usu­ a l ly occurs after a loud S, and may be decreased in intensity prior to presystolic accentuation due to atrial contraction. The severity of MS is determined largely by the duration of the Si-OS interva l and the d u ration of the murmur.

. .···

A2 S1 Acute Severe AR

Austin Flint Murmur of Chronic AR

(Graham Steel l Murmur)

Mild MS

f

..

PR without Pulmonary Hypertension

A2 OS

S1

·: : : : : :>

Severity: in chronic AR, the duration of the murmur is associated with severity In acute AR, a brief and soft early diastolic murmur may be present. The associated findings are important in determining severity. Variations: leaflet perforation may cause a "cooing" or musical sound. Associated findings: - Aortic systolic ejection murmur - Austin Flint murmur-a low-pitched rumbling apical diastolic murmur with presystolic accentuation that may mimic MS - Soft S1 (premature closure) . - Paradoxically split S2 - S3 - Laterally displaced hyperdynamic apical impulse - Wide pulse pressure with .,[, diastolic pressure - Large volume pulses - Bisferiens carotid pulse - Multiple peripheral signs may be present including those discussed in the arterial pulse section. - Diastolic MR may occur due to annular dilation.

3 . Pulmonic regurgitation: • •



Location: pulmonary area Description: high pitched and blowing, early diastolic decre­ scendo and generally brief beginning with Pi if due to PH (Graham Steell) and lower pitched in the absence of PH beginning after Pi Radiation: very localized

CHAPTER 2 • CARD IAC PHYSI CAL EXAMINATION •

• •



4.

Intensity: i with inspiration Severity: "to-and-fro" murmur with severe PR and associated findings Maneuvers: the murmur gets louder with inspiration. Associated findings: - Loud P 2 - Persistent split S2 - Elevated JVP with a prominent "a" wave that may be masked by a large V' wave if TR is also present - TR murmur - Parasternal lift due to RVH may be present.

Tricuspid stenosis: Location: localized at the lower left sternal border or xiphoid area and best heard in the right lateral decubitus position • Character: not as low pitched as MS • Radiation: none • Intensity: i with inspiration • Severity: related to the associated findings • Variations: a short, early to mid diastolic rumble may be heard without stenosis due to i flow such as with an ASD . • Associated findings: - Large "a" wave and slow "y" descent - Tricuspid OS may be heard. - Splitting of S1 and loud S/T1 may occur. - Right heart failure signs may occur.



CO N T I N U O U S H E A RT S O U N D S • They start in systole and encompass part or all of the sys­ tole and must extend through S 2 into diastole without discontinuation. • Usually continuous murmurs peak near to or at S 2 but are not required to encompass all of systole and diastole. • A holosystolic and holodiastolic murmur ("to and fro") together is not a continuous murmur since it does not go through the second heart sound. • Continuous murmurs occur because of a continuous gradient between chambers or vascular structures (aorta-PA, artery­ artery, artery-vein, vein-vein) , during both systole and diastole. • Benign continuous sounds include a venous hum and mam­ mary souffle. • A venous hum is heard mostly in children in the right supra­ clavicular area. It may have a "humming" quality. The inten­ sity is variable depending on position (loudest sitting and with the head rotated contralaterally) and may be diminished by compression. • A mammary souffle is present in late pregnancy or during lacta­ tion. It may be primarily systolic heard in the third to fourth interspace on either or both sides. It may be abolished by com­ pression. • Pathologic continuous murmurs include PDA, coronary fis­ tula, pulmonary arteriovenous fistulas , and coarctation of the aorta. • Continuous murmurs radiated to the back are usually patho­ logic, and coarctation of the aorta, and pulmonary A-V fistulas should be suspected in first instance; rarely, the murmur of PDA is heard in the back.

17

PDA with Moderate Pulmonary HTN

• PDA with Severe Pulmonary HTN

PDA with Eisenmenger Syndrome and Right to Left Shunt

FIGURE 2.1 3 PDA m u rm u r. The m u rm u r of a PDA is a contin uous

murmur cha racterized by an increasing intensity in systole, exten­ sion through the S2 sound, and then decreasing in diastole. Engu lf­ ing the S2 heart sound is essential to distinguish a continuous murmur from a "to-a nd-fro" m u rm u r. As the PA pressure goes u p, the diastolic component of the murmur shortens and may disa p­ pea r. With fu rther increases in PA pressu re, the systolic component diminishes and may also disappear. When Eisen menger synd rome occu rs with a rig ht-to-left shunt, the continuous m u rm u r will be a ltered and a short systolic murmur is a l l that remains.

Patent Ductus Arteriosus See Figure 2 1 3 .

.

• Heard best in the left second interspace near the sternum with radiation to the left clavicle • Harsh, loud, machinery-like quality, sometimes associated with a thrill • i with peak intensity around S 2 and then gradually wanes and may not encompass all of diastole • When PH develops, the diastolic portion gets shorter and softer. With severe pulmonary systolic HTN, the systolic component may also diminish and be absent. • With large left to right shunt, an apical diastolic rumble may be heard. • Associated findings: • Differential cyanosis when right to left shunting occurs (upper extremities with normal oxygenation and lower extremities with cyanosis) • Tachycardia • Bounding peripheral pulses and wide pulse pressure • Apical impulse displaced, diffuse

DY N A M I C AU S C U LTAT I O N See Figures 2 . 14 and 2 . 1 5 .

Respiration

• In general, right-sided murmurs and sounds i with inspiration and left-sided murmurs and sounds J-. • Exceptions include • The ES of PS J- with inspiration. • The click of MVP moves closer to S 1 and the murmur may be longer and accentuated with inspiration.

18

SE CTI ON I • FUNDAMENTALS Systol ic M u rm u rs Maneuvers

I

I

I n spiration

� L

i

TR

PS MVP/MR HOCM

--

s0

I

I

I

i

i

i

AS

MVPIMR HOCM

I

I

Squatting

Standing

MR TR PS

MR TR PS

VSD

VSD

i

I

i

i

CJ tJEJ M

I

Hand grip

D

Va Isa Iva

i

MVP/MR HOC M

I

L

AS

PS MR vso TR

FIGURE 2. 1 4 An a l gorithm demonstrati ng the effect of va rious maneuvers on systolic m u r m u rs. TR, tricuspid reg u rg itation; PS, p u l monary stenosis; MVP/MR, mitra l va lve prola pse/m itral reg u rgitation; HOCM, hypertrophic obstructive cardiomyopathy; M R, mitra l reg u rg itation; AS, aortic stenosis; VSD, ventricu l a r septa I defect. 'The effect of squatting on AS may be va ria ble (decreased, no change, or even increased) depending on the re lative alteration of preload a n d afterload.

Valsalva • Obtained by performing an inspiration followed by forced exhalation against a closed glottis

• Phase II during straining is detected at the bedside-there is a j, in venous return and BP and reflex tachycardia. • The opposite happens during phase IV where an i in stroke vol­ ume results in an i in BP, and a J, in HR. This phase has no utility in clinical practice but may lead to a diagnostic error (the overshoot). • During the strain phase, the only murmurs that i are those of HOCM and the MR murmur associated with MVP gets longer and may i in intensity. • Right-sided murmurs return to baseline levels within 2 to 3 beats after the Valsalva release.

Hemodynamic Maneuvers •

Raising legs while supine augments venous return and aug­ ments most right-sided heart sounds (after a few beats) and

D i astol i c M u rm u rs M a n e uvers

I

i

Va l s a l va

I

!

S q u a tt i n g

I

i

I

!

left-sided heart sounds (after 4 to 6 beats) . The murmur of HOCM is J.. . • Squatting results in i venous return and systemic resistance. Most right- and left-sided murmurs i such as AR, MR, and VSD . The murmur of HOCM is J... • Hand grips i BP and HR. AS murmur is unchanged or may J.. , most other left-sided murmurs i . The HOCM murmur J, and click and murmur of MVP are delayed and usually J, in intensity.

Pharmacologic Agents • Amyl nitrite results in marked transient preload and afterload (BP) reduction and subsequent i in heart rate. • This maneuver is best for distinguishing: 1) AS C i ) versus MR ( j,) 2) MS ( i ) versus Austin Flint ( j,)

• Innocent systolic murmurs are i .

3) MVP click murmur gets longer.

• The intensity of the murmur of A R J...

Post PVC

I

i

Hand g rip

I

!

DW 0D 0D FIGURE 2.1 S An algorithm demonstrating the effect of various

maneuvers on diastolic murmurs. AR, aortic reg u rg itation; PR, pul­ monary reg u rgitation; MS m itra l stenosis;TS, tricuspid stenosis.

• The murmurs of HCM, AS , and PS i .

• There i s n o change i n the murmurs of M R and TR. • The carotid upstroke i in AS, and J.. , or remains unchanged in the HCM. • The pulse pressure with HCM j, (Brockenbrough phenomenon) and that of AS l

TYP ICAL F I N D I N G S O F S P E C I F I C M E D I C A L CO N D I T I O N S

Acute Myocardial I nfarction • Bradycardia or tachycardia • Normotensive or hypotensive

CHAPTER 2 • CARD IAC PHYSI CAL EXAMINATION • S1 soft (associated with MR)

compression of the left lower lobe by a large pericardial effusion)

• S2 paradoxically split • S4 gallop ( .J., LV compliance during ischemia)

• Pulmonary rales

• Early systolic murmur (decrescendo) of acute severe MR (such as papillary muscle rupture)

• Kussmaul sign

• S3 gallop

• Late systolic murmur (crescendo) of MR (such as due to papil­ lary muscle dysfunction)

RV Infarction

• t ]VP with t "a" and "v'' wave • Kussmaul sign

19

Constrictive Pericarditis

• t ]VP with rapid "x" and "y" descent (Friedreich sign)

• Apical impulse may not be palpable. • Systolic retraction of the apical impulse (Broadbent sign) • Quiet heart sounds • Pericardial knock

• Hypotension

• Right-sided S3 or S4 gallop (t with inspiration)

• Right heart failure signs (hepatosplenomegaly, pulsatile liver, ascites, edema)

• Systolic murmur of TR

• Pulsus paradoxus may be present with effusive-constrictive pericarditis.

• Clear lungs

• MR or TR murmurs

Dilated Cardiomyopathy

Pulmonary Hypertension

• t ]VP with t "a" and "v'' wave

• Low pulse amplitude, narrow pulse pressure, and pulsus altemans

• Left parasternal systolic lift

• Diffuse apical impulse displaced laterally and downward

• Loud P2 (may be palpable)

• S2 (P2) t with pulmonary HTN .

• S2 paradoxically split (often due to LBBB)

• S4, S3, or SG with tachycardia • MR or TR murmurs

Restrictive CM • Macroglossia, purpura, bruising (amyloidosis)

• Elevated ]VP with prominent "a" waves

• S2 persistently split • Right-sided S4 or S3 gallop • Pulmonic ES •

Pulmonic regurgitation (Graham Steell murmur)

• TR murmur • Right heart failure signs (hepatosplenomegaly, pulsatile liver, ascites, edema)

• Cachexia • Tachycardia

Type A Aortic Dissection

• Hypotension (including orthostatic hypotension)

• t ]VP with rapid "x" and "y" descent

• Normotensive, hypertensive, or hypotensive

• Kussmaul sign

• Unequal or absent pulses

• Narrow pulse pressure with decreased pulse amplitude

• New high-pitched diastolic decrescendo murmur (short dura­ tion and may be soft)

• S3 gallop (less common S4 gallop) • MR or TR murmurs

• Unequal upper-extremity BP

• Systolic ejection murmur (associated with severe AR and t stroke volume)

• Right heart failure signs (hepatosplenomegaly, pulsatile liver, ascites, edema)

• S3 gallop

Cardiac Tam ponade

• Shock or cardiac tamponade

• Pericardial rub (if rupture into pericardial sac)

• Hypotension (and signs of hypoperfusion) • Tachycardia • Elevated ]VP with prominent "x" descent and reduced or absent "y" descent

• Absence of peripheral signs seen with chronic AR • Neurologic findings (including Horner syndrome and stroke)

• Pulsus paradoxus

• Beck triad: (t JVP, quiet heart sounds, and hypotension) • Quiet heart sounds

• Ewart sign with dullness to percussion and bronchial lung sounds on the left side below the left scapula (due to

Atrial Septal Defect

• Elevated ]VP with equal size "a" and

• RV systolic heave • P2 t

• Palpable pulsation of the PA

"v''

waves

20

SE CTI ON I • FUNDAMENTALS • i S 1 and S/P2

• S 2 fixed split

• Laterally displace apical impulse

• Midsystolic ejection murmur (increased flow across the pulmonary outflow tract)

• Splitting of S1

• Early low-pitched diastolic rumble (due to flow across the tricuspid valve) • MR can be heard with an ostium primum ASD (with associated mitral valve cleft)

• Persistent splitting of S2 • Grade 1 to 2 ejection murmur (due to i blood flow through the pulmonary outflow tract) • S3 gallop

• Association with the Holt-Oram syndrome (upper limb deformities)

• Peripheral edema

Ventricu lar Septal Defect

• Mammary souffle

• Normal or i intensity S2 • Persistent split S2 • S 3 gallop

• Diastolic rumble (due to i flow across the mitral valve)

• Regurgitant holosystolic high-pitched murmur heard best along the left side of the sternum • Palpable thrill • With a supracristal VSD , the murmur is associated with AR. • With a muscular VSD, the murmur may be decrescendo in shape (the defect becomes smaller as the muscle contracts) .

Expected "Normal" Findings during Pregnancy • Normotensive with a tendency toward .J., diastolic BP

• Tachycardia

• Mildly i or normal JVP with prominent "a" and "v" waves

• Brisk carotid upstrokes

• Venous hum

S U G G ESTE D READ I NGS Bates B . The heart, pressures and pulses. In: Bates B , ed. A Guide to Physical Examination. 3rd ed. Philadelphia: Lippincott; 1983. Braunwald E, Perloff J K . Physical examination of the heart and circulation. In: Braunwald E, ed. Braunwald's Heart Disease: A Textbook of Cardiovascular Medicine. 7th ed. Philadelphia: Elsevier Saunders; 2005 . Carabello BA, Crawford FA. Valvular heart disease. N Engl ] Med. 1 997;337:32. Chatterj ee K . Physical examination. In: Topol EJ , ed. Text­ book of Cardiovascular Medicine. 3rd ed. Philadelphia: Lippincott Williams and Wilkins; 2007. Criley JM, Criley DG, Zalace C . The Physiological Origins of Heart Sounds and Murmurs. Philadelphia: Lippincott Williams and Wilkins; 1997. Heger J W, Niemann JT, Criley J M . Cardiology. 5th ed. Phila­ delphia: Lippincott Williams and Wilkins; 2004. Hurst JW. The examination of the heart: the importance of initial screening. Emory Univ ] Med. July-September 1 99 1 ; 1 3 5 .

QU ESTIONS AND ANSWERS Questions

1 . A 20-yea r-old man is referred for a cardiology con­ sult due to a n episode of nonexertional syncope associated with pa lpitations (his mother who is a n u rse took his a pica l pulse and it was >200 bpm and i rreg u l a r d u ring the event). An electroca rd io­ g ra m is reported as abnormal but not availa ble. An echocardiogra m was norma l. Which of the fol l ow­ ing exa m i nations suggests a cause for syncope i n this patient? a. Normal S 1 and physiologic split S 2 b. Normal S 1 and persistent split S 2 c. Physiologic split S 1 and normal S 2 d. Normal S 1 and fixed split S 2 2. A 3 1 -yea r-old woman is 6 months preg nant. She has no sig n ificant medical or cardiac h istory. Her preg nancy has been u ncompl icated except that

now she is experiencing dyspnea with exertion. Which of the fol l owing exa m inations would be considered patholog ic? a. Normal S 1 and S 2 with a 2/6 early systolic ejec­ tion m u rm u r and 1 /6 d iastolic ru m ble. No extra sounds. b. Normal S 1 and S 2 with a S 3 g a l lop c. Normal S 1 and S 2 • No extra sounds. Conti nuous murmur heard under the left breast. d. Normal S 1 and decreased S 2 . An ejection sound (ES) decreases with inspiration . 3. A 25-year-old woman h a s a history o f congenita l heart disease. She was reco m mended to have a "corrective i nterventional proced u re" several years earlier due to a continuous m u rm u r, but she did not do it because she felt wel l . She now has developed the onset of dyspnea with a moderate level

CHAPTER 2 • CARD IAC PHYSI CAL EXAMINATION

of exertion. Which of the fol lowi ng exa m i nations wou l d you expect to be present? a. Normal S,, soft S/P 2, and continuous systolic and d iastolic m u r m u r b . N o r m a l S , , n o r m a l S/P 2, and conti nuous systolic m u rm u r but short diastolic m u r m u r c . Norma l S, and loud S/P 2 with predominant sys­ tolic m u rm u r. Pink hands and blue feet. d. Normal S, and S 2 with only a d iastolic m u r m u r. P i n k hands and blue feet. 4. A 32-year-old woman has just retu rned from a year i n I ndia. She has a sore th roat, fever, rash, a rthralgias and is dyspneic. On exa m i nation, there is a soft S , and l o u d S 2 .There is a n S3 gallop. She h a s 2 murmurs, a 3/6 apical holosystolic reg u rgita nt systolic murmur and a 2/6 apical, short m id-diastolic rumble. What is the most l i kely explanation for her sym ptoms? a. Pul monary stenosis (PS) and pul monary reg u rgitation (PR) b. Aortic stenosis (AS) and aortic reg u rgitation (AR) c. Tricuspid stenosis (TS) and reg u rg itation d. M itral stenosis (MS) and m itra l reg u rg itation (MR) 5. A 26-year-old woman is referred to cardiology clin ic. She was told she had MVP 1 0 yea rs earlier. A 2/6 reg u rg ita nt m u r m u r is hea rd at the LSB with a n associated midsysto l ic click. What is expected to happen when the patient changes from a standing to squatti ng position? a. The click moves toward S2 and the murmur decreases. b. The click moves toward S 2 and the m u rm u r i ntensifies. c. The click moves towa rd S, and the m u rm u r decreases. d. The click moves towa rd S, and the m u rm u r intensifies. 6. A 70-yea r-old man is seen i n the hospita l for preop­ erative cardiac cleara nce prior to h i p s u rgery that is plan ned for today. Prior to fa l l i n g and i nj u ring his h i p, he had no symptoms with >4 M ETS exertion. On exa m i n ation, the i ntern hears a systolic m u rm u r that is 3/6 i n i ntensity. He is concerned that the patient has AS and would l i ke to cancel the surgery and obta i n an echoca rdiogram. Which of the fol­ lowing findings is not consistent with AS? a. Fol l owing a PVC, the i ntensity of the m u r m u r is u ncha nged. b. The m u r m u r sta rts after S, and ends before S 2 • c. The m u r m u r is i ntensified after a long cycle length i n atrial fibril lation (AF).

21

d. The m u r m u r is typica l ly crescendo/decrescendo i n shape. 7. A 70-yea r-o ld man is day # 3 post i nferior M l . He develops sudden hypotension. On exa m i n ation, S, is decreased, S 2 (P 2 ) is increased. There is a decre­ scendo systolic m u r m u r that begins with S, and ends i n midsystole. There is a thrill along the left sternal border. What is the most l i kely diag nosis? a. Severe TR. b. Systolic anterior motion of the m itra l va lve with LVOT obstruction c. Pseudoaneurysm d. lschemic MR 8. A 68-year-old man presents for eval uation of right heart fai l u re. He has a history of coronary a rtery bypass g raft (CABG) 1 0 yea rs previously. He devel­ oped dyspnea with m i n i m a l exertion, orthopnea, pa lpitations, and lower-extrem ity edema in the past year. On exa m i n ation, the jugular venous pressu re is elevated to 1 5 cm H 2 0 with a promi nent "x" descent and a "y" descent that is blu nted. Kussmaul sign is a bsent and pulsus pa radoxus is present.The heart sounds a re soft. A PK is a bsent. Which of the follow­ i n g conditions is suspected? a. Restrictive cardiomyopathy b. "Classica l" form constrictive pericard itis c. Effusive-constrictive pericarditis 9. A 47-yea r-o l d wom a n presents with dyspnea o n exertion a n d episodes o r p resyncope. S h e h a s a h i story o f rheu matic MS req u i ri n g a b i o p ros­ thetic MVR 1 0 yea rs p revio u s l y. O n exa m i n ation, JVP i s 1 2 c m Hp. S, is normal a n d S 2 is persis­ tently split and accentuated. Th e re is a n e a rly, l o n g d u ration 3/6 b l owi n g d iasto l i c decresce n d o m u r m u r a n d a 2/6 e a r l y pea k i n g S E M at t h e l eft u pper ste r n a l bord e r. With expi ration, both m u r­ m u rs decrease i n seve rity. What i s the most l i kely d i a g n osis? a. Pul monary reg u rg itation and stenosis b. PR with pul monary hypertension (PH) c. AR and stenosis d. Prosthetic m itra l valve stenosis 1 0. A 32-year-old man has a history of a heart murmur since childhood. He is asymptomatic but was told he could not play sports as a chi ld. On exami nation, there are equal pulses. S, and S2 a re normal without a split.There is a 3/6 SEM.There is no ES.There is a 2/6 diastolic decrescendo m u rm u r. Fol lowing a PVC, the murmur increased in i ntensity as d id the pulse (Con tin ued)

22

SE CTI ON I • FUNDAMENTALS

pressu re. Which is the correct diag nosis for this patient? a. Subvalvu lar AS due to a su baortic mem brane b. Suprava lvu lar AS c. Hypertrophic cardiomyopathy d. Bicuspid AS Answers 1 . Answer B: The patient has Wo lf-Pa rkinson-Wh ite

(WPW) synd rome with associated ra pid AF that led to nonexertional syncope. With a m a n ifest accessory pathway, left- a n d rig ht-sided WPW may be detected on a u scu ltation d u e to a b n o rm a l ities in the spl itti ng of the second heart sou nd (S 2 ). With a left-sided pathway, the A 2 component of S 2 wou l d occ u r early since the atrioventricu l a r (AV) node is bypassed a n d electrica l activation of the left heart and therefore co m p l etion of ejectio n wou l d lead to a n earlier A 2 closure sound. I n expi ration, A 2 a n d P 2 wo u l d be sepa rated, and in i n s p i ration, the separation wou l d be i n c reased. This is referred to as persistent spl itting of S 2 . When a rig ht-sided pathway is present, the P2 co m ponent of S 2 occu rs early d u ri n g expi ration (P 2 before A 2 ) with a s i n g l e sou nd d u ri n g i n s p i ration. This is referred to as paradoxica l spl itting of S 2 . Physiologic spl itti ng of S, or S 2 may be a normal va riant a n d n ot associated with electrica l or structura l heart d i sease. A fixed split S 2 m ay be associated with a hemodyn a m ica l ly s i g n ificant ASD, thoug h this wou l d not be a n expected cause of syncope. 2. Answer D: The physiologic changes d u ring preg­ nancy including increased H R, stroke vol u m e and decreased systemic and pulmonary vascular resista nce lead to expected variations from normal on card iac physica l exa m i nation. The i ntensity of the fi rst heart sound (S, ) and S2 may be increased and spl itti ng of S, and S 2 may occur. An S 3 is com mon as is a n early peak­ ing, short systolic ejection m u rm u r (�2/6 i ntensity). These fi ndings a re a l l related to increased tota l volume and card iac output (CO). Simila rly, some women will have a very soft d iastolic flow m u r m u r related to increased flow across the AV valves. Th is may be physi­ ologic if there a re no other associated abnormal sounds (e.g., a n opening snap [OS] of MS). Continuous m u rm u rs such as a venous h u m or m a m m a ry souffle may be heard. All other sounds including ( 1 ) red uced i ntensity S, or S 2, (2) pa radoxica l or fixed S2 spl itting, (3) 2':3/6 systolic ejection m u r m u r especi a l ly if mid or late peak­ ing and long i n d u ration, (4) 2':2/6 reg u rg itant m u r m u r, (5) conti nuous heart sounds other than the mammary souffle and venous h u m, (6) S4 gallop, or (7) ESs or extra sounds are considered patholog ic. I n this patient, a decreased S2 sound and a n ES that decreases with

inspiration is consistent with PS. The other exa m ples are acceptable i n a normal preg nant woman. 3. Answer B: The clinica l history and answers a re consistent with a patent d uctus a rteriosus (PDA). Each choice suggests a different size of the shunt and level of pul monary vascular resistance relative to systemic vascular resista nce (SVR). See Fig. 2. 1 3. Choice (A) is con­ sistent with a small hemodynamically insignificant PDA cha racterized by a systol ic and d iastolic com ponent that envelope the second heart sound. Choice (B) is consis­ tent with at least a moderate-sized PDA with associated elevation of PA pressure. This resu lts in shortening of the diasto lic com ponent of the m u rm u r as the systemic and pul monary vascular resistances i n d iastole beg i n t o eq ualize. Remember that a continuous m u rm u r i s defined pri m a ri ly b y its extension through t h e second heart sounds and is not req u i red to extend throughout a l l of systole and diastole. Choice (C) is consistent with a right-to-left s h u nt due to PH and resu ltant Eisen menger physiology. In the setting of a P DA and Eisen menger physiology, the u pper extremities remain wel l oxygen­ ated and pink si nce they do not receive shu nted blood whereas the lower extremities do not-a nd therefore are bl ue. Choice (D) is an u n l i kely exa m ination for a PDA. 4. Answer D: The history and clin ical findings suggest

acute rheumatic fever, which is man ifest as a pancard itis. The endoca rdial lesion is a n active va lvul itis that most com m o n ly i nvolves the m itra l valve followed by the aor­ tic va lve i n freq uency. I n the acute phase of m itral va lvu­ l itis due to rheu matic fever, there is both a fu nctional MS and regu rg itation due to inflam mation of the leaflets. The diastolic m u r m u r is known as the Carey Coom bs m u rm u r and disti n g u ishable from chronic rheu matic MS due to the a bsence of a n OS, loud S l , and presystolic accentuation of the diastolic m u rm u r. The associated M R may b e severe. Aortic va lve i nvolvement is manifest a s A R . The exa m i nation descri bed is most consistent with a predominant M R m u r m u r with m i l d fu nctional MS. S. Answer A: The m itral va lve click and associated

m itral reg u rg itant m u r m u r is dynamically affected by changes in preload and afterload. Since the leaflets and chordae are redundant and elongated, there is effec­ tively a mismatch between the leaflets and the left ven­ tricular cavity size. When the cavity is smaller (as with decreased preload or afterload), the leaflets prola pse earlier in systole.This resu lts in the m itra l click movi ng closer to S , ; therefore, the d u ration and i ntensity of the reg u rg itant m u r m u r a re increased. I n contrast, when the cavity is larger (as with increased preload or after­ load), the leaflets prola pse later in systole. This resu lts in the m itral click movi ng away from S, and closer to S 2 ; therefore, the d u ration and intensity of the reg u rg itant m u rm u r a re decreased (see Fig. 2.7).

CHAPTER 2 • CARD IAC PHYSI CAL EXAMINATION

6 . Answer A : Distinguishing between a systolic

reg u rg itant and a n ejection-type murmur is i mporta nt to characterize the etiology of a m u r m u r. Reg urg itant m u rm u rs result from abnormalities of the atrioven­ tricular valves and shu nts (MR and TR, ventricular septa I defect [VSD]). Ejection m u r m u rs result from a bnormali­ ties of flow through the semi l u n a r va lves or outflow tract (aortic and PS, HOCM). Reg u rg itant m u r m u rs typica l ly sta rt with S, or at least extend to S2, or both. They are typica l ly high pitched and do not increase with a long diastole of AF or post PVCs since the relative g radient between the u pstream and the downstream cham bers do not change significantly. Ejection-type m u r m u rs typica l ly sta rt after S, and end before S 2 and a re genera l ly med i u m pitched and crescendo/decre­ scendo in shape. With a long diastole of AF or post PVC, ejection m u rm u rs increase in i ntensity. 7. Answer D: Acute ischemic MR fol lowing myocardial i nfa rction may be due to orga nic or functional etiolo­ gies. The timing and i ntensity of the m u r m u r a re related in part to the mechanism and the severity as wel l as loadi n g conditions. In the setting of organic MR due to papi llary m uscle rupture, the m u r m u r is typica l ly early systolic and then fades i n later systole. Th is occu rs beca use of rapid eq ual ization of the left atrial and ven­ tricular pressu res i n systole. I n contrast, the fu nctional etiologies of M R including papi l l a ry m uscle dysfunction tend to occur later i n systole. 8. Answer C: Constrictive perica rd itis should be sus­

pected i n any patient with prior coronary a rtery bypass g raft (CABG) and signs or sym ptoms of right heart fa i l u re especi a l ly if left ventricular fu nction is preserved. The c l i n ica l fi ndings i n the patient a re most nota ble for elevated JVP with a blu nted "y" descent and pulsus para­ doxus. The presence of a blu nted "y" descent and pulsus paradoxus is not typical of classical form of constrictive pericarditis where there is typica lly a promi nent "x" and "y" descent and the presence of Kussmaul sig n . However, this constel lation of fi n d i ngs is present with the effusive form of constrictive pericarditis. Effusive-constrictive

23

perica rditis is due to a n elastic perica rdia I viscera l restraint with a tense perica rd ia I effusion resu lting i n t h e clinica l and hemodynamic features consistent with ca rdiac tam ponade that su bseq uently revert to that of constrictive pericarditis fol lowi ng pericard iocentesis. Most patients with effusive-constrictive perica rditis ultimately req u i re perica rd iectomy. A PK is not expected in effusive-constrictive perica rd itis and Kussmaul sign is genera l ly a bsent. 9. An swer B: The patient has a G ra h a m Steell

m u r m u r that is d u e to P R i n the setting of P H . (See Fig. 2 . 1 1 .) It is usua l ly the result of longsta n d i n g MS. It d i ffers fro m the fi nd i n g s seen with P R u n related to PH where S 2 is not accentuated a n d the m u r m u r is not hea rd i m m ediately after S 2 • I ntensification of the m u r m u r with i n s p i ration a n d red uction of the m u r­ m u r with expiration cha racterize this as a rig ht-sided m u r m u r, thus exc l u d i n g AR and prosthetic MS. PS is a ra re fi n d i n g i n rheumatic heart d i sease a n d wou l d be associated with a red uced S 2 • 1 0. Answer A: The patient has a subaortic mem brane resu lting i n subvalvu lar AS and AR. Subvalvular mem branes are usually due to fibrinous mem bra nes or tunnels that result i n fixed stenosis. Severa l mecha­ nisms lead to associated AR including the effects of a jet lesion on the aortic leaflets and retraction of the leaflets due to the membra ne. Supravalvular AS is ra rely associated with AR and typica l ly has unequal pulses with right > left due to the strea ming of the flow jet along the right side of the aorta. Bicuspid valvu lar AS is u n l i kely since the S 2 sou nd is preserved and there is no ejection click (EC). Finally, with HCM, the m u r m u r post PVC is increased due to decreased preload resu lting i n increased systolic a nterior motion o f t h e m itra l va lve and increased obstruction. I n addition, the pulse pres­ sure (difference in pressu re between the systolic and d iasto l ic blood pressu re) post PVC with HCM is decreased. With all other forms of AS (valvu lar, su bvalvu lar, and supravalvu lar), the pu lse pres­ sure post PVC is usually increased.

C ardiac Anatoiny Robert E . Hobbs

T

he heart is a muscular organ, pyramidal in shape, con­ sisting of two parallel-valved pumps, located within the middle mediastinum, two-thirds to the left of the centerline. The base of the heart is oriented superiorly, where­ as the apex points leftward, anteriorly, and slightly inferiorly The cardiac apex is located at the fifth intercostal space near the midclavicular line. The heart is enclosed by the fibrous pericardium, which is bordered by the diaphragm inferiorly; the sternum and ribs anteriorly; the pleurae laterally; and the esophagus, descending aorta, and vertebrae posteriorly The average adult heart measures 1 2 cm from base to apex, 8 to 9 cm in width, and 6 cm in depth, approximating the size of a clenched fist. The heart weighs approximately 325 ± 75 g in men and 275 ± 75 g in women, accounting for 0.45% of body weight in males and 0 .40% in females. Viewed from the front, visible structures include the superior and the inferior vena cavae draining into the right atrium, the right ventricle (the most anterior chamber of the heart) , the main pulmonary artery that courses superiorly and posteriorly before bifurcating, the left atrial appendage, a small portion of the left ventricle visible to the left of the left anterior descending coronary artery, and the ascending aorta (Fig. 3 . 1). The right heart forms the largest part of the anterior surface, whereas the left heart is largely posterior. The epicardial surface of the heart usually is covered with fat, proportional to age and the amount of body fat. Beneath the epicardial fat, the interventricular grooves separate the right and left ventricles and contain arteries, veins, nerves, and lymphatics. The atrioventricular (AV) grooves, which separate the atria from the ventricles, are located at the base of the heart. The interatrial grooves mark the borders between the atria. Posteriorly, the crux ("cross") is the inter­ section of the AV, interatrial, and interventricular grooves. The acute cardiac margin is the junction of the inferior and anterior walls of the right ventricle. The obtuse margin is the rounded lateral wall of the left ventricle. The heart is composed of four chambers. The right and the left atria are weakly contractile reservoirs that receive blood from the body and the lungs. They are positioned 24

above the ventricles and are separated by the AV (tricuspid and mitral) valves. The right and left ventricles are muscular pumping chambers separated from each other by the inter­ ventricular septum. The ventricles eject blood through the semilunar (pulmonic and aortic) valves to the pulmonary artery and aorta, respectively The shape of the heart and the position of the valves are maintained by an internal fibrous skeleton. The cardiac skeleton consists of four valve annuli (or rings) , the mem­ branous septum, and the right and left fibrous trigones. The right fibrous trigone, also known as the central fibrous body, is located between the aortic, mitral, and tricuspid valves. It houses the His bundle and is the strongest component of the cardiac skeleton. TH E PE RICARD I U M The pericardium is a fibrous sac surrounding the heart and great vessels and containing 1 0 to 50 ml of pericardial fluid (Fig. 3.2). It maintains the position of the heart within the mediastinum, lubricates the cardiac surfaces, and provides a barrier against infection. Inferiorly, the pericardium is anchored to the central tendon of the diaphragm. Anteriorly, it is attached by ligamentous connections to the posterior sternum. Superiorly, the pericardium extends to the level of the second intercostal space, and laterally it is attached to the pleurae. The pericardium encloses the heart, portions of the vena cavae, most of the ascending aorta, the main pulmonary artery, and the four pulmonary veins. The pericardium con­ sists of two layers. The inelastic fibrous pericardium is the outermost layer. The serous pericardium forms a thin meso­ thelial layer on the cardiac surface (the visceral pericardium or the epicardium) and lines the inferior surface of the fibrous pericardium (the parietal layer) . The visceral pericardium covers the heart and contains the coronary arteries and veins, autonomic nerves, lymphatics, and fat. Posteriorly, the peri­ cardium folds upon itself to create several distinct sinuses. The oblique sinus is a pericardial reflection along the vena cavae and the pulmonary veins. The transverse sinus is a pericardial

CHAPTER 3 • CARD IAC ANATOMY

25

Right Right !nom i n ate artery Superior vena cava

inominate Left subclavian artery

Superio r

Right atrial a ppendage

Right

atrium

artery

Pulmonary artery

r

vena cava

appendage Left anterior descending artery

appendage Tricuspid valve

C o ro na ry sinus

vena cava

reflection located between the aorta, pulmonary artery, and atria. The ligament of Marshall is a pericardial fold containing the remnant of the embryonic left superior vena cava. C A R D I AC C H A M B E R S

Right Atriu m The right atrium is a low-pressure capacitance chamber that receives blood from the superior vena cava, inferior vena

FIGURE 3.2 Fronta l view of the pericard i u m .

Pulmonary

Left atrial

Left ventricle

FIGURE 3.1 Fronta l view of the heart a n d g reat vesse ls.

Aorta

artery

Pulmo nary vein Left atrial

Left subclavian artery

descending

Left anterior

Diagonal artery

muscle

Papillary

FIGURE 3.3 Fronta l cutaway view of the right atri u m a n d right ventricle.

cava, and coronary sinus (Fig. 3.3). The right atrial volume is approximately 75 to 80 ml, and its free-wall thickness is 1 to 3 mm. The superior vena cava enters the superior aspect of the right atrium and directs its blood flow toward the tricus­ pid valve. The inferior vena cava returns blood from the lower body; and its eustachian valve directs blood flow toward the foramen ovale or the fossa ovalis. The coronary sinus returns most of the blood from the heart itself through an orifice par­ tially guarded by the thebesian valve (valve of the coronary sinus) . When the eustachian or thebesian valves are large and fenestrated, it is described as a Chiari net. Thebesian veins drain cardiac blood into the right atrium via multiple small orifices. The fossa ovalis, representing a closed foramen ovale, forms a 1 .5- to 2 .0-cm depression on the interatrial septum. A patent foramen ovale is found in up to one-third of adults. The crista terminalis is a C-shaped muscular ridge on the right atrial free wall that separates the smooth posterior portion of the right atrium from the muscular anterior portion. The pec­ tinate ("comb") muscles arise from the crista terminalis and course as bands anteriorly on the right atrial free wall. The right atrial appendage is a large triangular structure that over­ lies the right coronary artery and contains pectinate muscles. In the lower medial portion of the right atrium, Koch triangle overlies the AV node and the proximal His bundle. The ten­ don of Todaro is a fibrous band located between the valves of the inferior vena cava and the coronary sinus.

Right Ventricle The right ventricle is the most anterior chamber of the heart (Fig. 3 .4). It is the smaller of the two ventricular chambers, separated from the left ventricle by the interventricular sep­ tum, which bulges into the right ventricle. It is triangular shaped when viewed from the right, and crescent shaped when viewed in cross section from the left. The right ventricular free wall is approximately 3 to 4 mm thick, or about one-third the

26

SE CTI ON I • FUNDAMENTALS

Superior vena cava

Tricuspid valve

Aorta lntercostal arteries Left atrium Pulmonary vein

Left pulmonary arteries

M i t r a ! valve

Aortic valve

Pulmonic valve

Pulmonary veins Inferior

Pulmonary veins

v en a ca va

Middle card iac

vein

Left ventricle

Posterior vein

cardiac

Posterior descending artery

FIGURE 3.4 Coronal section of the heart and g reat vessels. FIGURE 3.5 Posterior view of the hea rt and g reat vessels.

thickness of the left ventricle. The right ventricle consists of an inlet portion, a trabeculated apical portion, and a smooth right ventricular outflow tract (infundibulum or conus por­ tion) . The walls of the right ventricle contain a latticework of muscle fibers called trabeculae carneae. The right ventricu­ lar apex is heavily trabeculated, more so than the left ven­ tricle. The infundibulum (outflow tract or conus) portion of the right ventricle is smooth walled to the pulmonic valve. The right ventricle contains three papillary muscles, although the septal papillary muscle occasionally may be absent. Chordae tendineae (fibrous cords) extend upward from the papillary muscles and attach to the leaflet edges and to the ventricular side of the tricuspid valve. Chordae from one papillary mus­ cle often attach to more than one tricuspid leaflet, and some chordae arise from the septum. The crista supraventricularis (supraventricular crest) separates the inflow and outflow por­ tions of the right ventricle. It consists of a septal band on the ventricular septum and a parietal band on the right ventricu­ lar free wall. The moderator band is an intracavity muscular bridge that connects the distal septum with the right ven­ tricular free wall at the anterior papillary muscle. Blood enters the right ventricle via the tricuspid valve, turns upward at a 45- to 60-degree angle, and passes through the pulmonic valve into the main pulmonary artery

Left Atrium The left atrium is the left upper posterior chamber of the heart (Fig. 3.5). It is cuboidal shaped, smaller than the right atrium (volume, 55 to 65 ml) , but with thicker walls (3 mm) and higher pressure. It receives oxygenated blood from the lungs via four pulmonary veins (two from each lung) . Unlike the right atrium, the left atrium has smooth interior walls and does not have bands of pectinate muscles except in the left atrial appendage. The left atrial muscle extends a variable distance within the pulmonary veins to prevent

reflux during atrial contraction. The left atrial appendage, overlying the left circumflex coronary artery, is smaller, longer, more tortuous, and less triangular than the right atrial appendage, often containing two or more lobes. Left atrial contraction generates a stroke volume of 20 to 30 ml.

Left Ventricle The left ventricle is a high-pressure, muscular chamber, 2 . 5 to 3 times thicker than the right ventricle (see Fig. 3 .4) . It is ellipsoid, or cone shaped when viewed from the right, and ring or doughnut shaped when viewed in cross section from the left. It is longer and narrower than the right ventri­ cle, measuring approximately 7 . 5 cm in length and 4 . 5 cm in width. Structurally, the left ventricle consists of the inflow tract, the apical zone, and the left ventricular outflow tract. The anterior mitral leaflet separates the left ventricle into the posterior inflow tract and the anterior outflow tract. The septum consists of a large inferior muscular portion and a small superior membranous portion. The septum is thick­ est at the midportion and thinnest at the membranous por­ tion near the aortic valve. The membranous septum has AV and intraventricular portions divided by the septal tricus­ pid leaflet. The His bundle is located within the interven­ tricular portion of the membranous septum. The left ventricular free wall measures approximately 8 to 12 mm and is thicker at the base than at the apex. It is composed of three layers: the endocardium, the myocar­ dium, and the epicardium (or visceral pericardium) . The outer two-thirds of the myocardium contains compact layers of muscle that twist and spiral inward from apex to base dur­ ing contraction. The inner third of the myocardium consists of a latticework of trabeculae carneae that are more intricate than right ventricular trabeculations. The septal surface of the left ventricle is smooth.

CHAPTER 3 • CARD IAC ANATOMY Two papillary muscles, the larger anterolateral and the smaller posteromedial, arise from the free wall and have a vari­ able number of heads. They anchor the chordae tendineae of the mitral valve, which are thicker than tricuspid valve chor­ dae. Chordae tendineae restrict valve excursion during ven­ tricular systole, thereby preventing the mitral valve leaflets from prolapsing into the left atrium. Most chordae arise from the heads of the papillary muscles, but some arise from the free wall. Chordae from one papillary muscle may diverge and attach to both mitral leaflets. False chordae occur in half of normal hearts, and may connect walls, papillary muscles, and the septum, but are not attached to the mitral leaflets. They often cross the left ventricular outflow tract and can be identified by echocardiography. Many false chordae contain extensions of left ventricular conducting fibers. Blood enters the left ventricle via the mitral valve and is ejected at a 90- to 1 20-degree angle through the aortic valve. The ejection phase is shorter in the left ventricle, but the pres­ sure is greater compared with right ventricular contraction. C A R D I AC VA LV E S

Tricuspid Valve The tricuspid valve is the largest of the heart valves and maintains forward flow of blood through the right heart (Fig. 3 . 6) . The functional components of the tricuspid valve include the three leaflets, commissures, annulus, chordae tendineae, papillary muscles, and the right ventricle. The leaflets are named for their anatomic position: anterior, pos­ terior, and septal. The anterior leaflet, which is the largest and the most mobile, partially separates the right ventric­ ular inflow and outflow tracts. The posterior leaflet is the smallest, whereas the septal leaflet is the least mobile and is occasionally absent. The valve leaflets are attached to a discontinuous fibrous annulus that has a "D" shape. Chordae tendineae (tendinous cords) are attached to the edges and undersurface of each leaflet and are anchored by the papil­ lary muscles and the interventricular septum.

27

Pulmonic Va lve The pulmonic valve is the most anterior valve of the heart, located between the right ventricular outflow tract and the main pulmonary artery (see Fig. 3 . 6). It is the mirror image of the aortic valve, containing right, left, and anterior cusps (or leaflets) that are thinner than those of the aortic valve. The pulmonary sinuses are partially embedded within the right ventricular infundibulum. During systole, the valve opens to form a rounded, triangular-shaped central orifice. Mitra I Valve The mitral valve is named after the miter, a tall ornamental hat worn by bishops and abbots (Figs. 3 . 6 and 3 . 7) . The valve, located between the left atrium and the left ventri­ cle, maintains the forward flow of blood in the left heart. The mitral valve has six components: leaflets, commissures, annulus, chordae tendineae, papillary muscles, and left ventricle. When viewed from the side, the valve is funnel shaped, with the leaflets forming an apex protruding into the left ventricle. There are two mitral leaflets, the anterior and the posterior, which have similar surface areas but dif­ ferent shapes. The anterior leaflet is semicircular or oval shaped, broader but narrow transversely. It partially sepa­ rates the left ventricular inflow tract from the left ventricular outflow tract. The posterior leaflet is crescent shaped, longer and narrower, half the height but twice the length of the anterior leaflet. It attaches over two-thirds of the posterior valve circumference. The posterior leaflet has two or more indentations forming three scallops (the middle usually is the largest) . During atrial contraction, the valve forms an ellipsoid orifice. During ventricular contraction, the atrial side of the leaflets coapt, preventing regurgitation of blood into the atrium. Two commissures, the anterolateral and the posteromedial, separate the two leaflets. The chordae tendineae prevent the mitral valve from prolapsing into the left atrium during ventricular systole. Approximately 1 00 primary, secondary, and tertiary chordae attach to the free

Anterior pu lmonlc cusp

_.::::����-=,,,--- noncoronary Posterior or

aortic cusp

Anterior tricuspid lea net

Septa! tricuspid leanet

FIGURE 3.6 Cross-sectional view of the cardiac va lves.

FIGURE 3.7 Surgical view of the atrial su rface of the mitra l va lve.

28

SE CTI ON I • FUNDAMENTALS free wall, where its blood flow is directed by the eustachian valve toward the fossa ovalis.

FIGURE 3.8 Side and top views of the aortic va lve.

edge and underside of the valve and are anchored by two papillary muscles in the left ventricle. Some of the poste­ rior leaflet chordae arise from the left ventricular free wall. Unlike the tricuspid chordae, mitral chordae do not have insertions into the septum. The mitral valve is surrounded by a saddle-shaped fibrous ring, the mitral annulus, which anchors the valve. It is connected to the tricuspid annulus by the right fibrous trigone (central fibrous body) , forming part of the fibrous skeleton of the heart.

Aortic Valve The aortic valve, located between the left ventricle and the aorta, is thicker and stronger than the pulmonic valve (Figs. 3 . 6 and 3 . 8) . It consists of three semilunar (half-moon) cusps located within the sinuses of Valsalva, three commis­ sures, and an annulus. The three semilunar cusps, left, right, and noncoronary (or posterior) , are pocket-like structures. The valve has a triangular-shaped central orifice when fully opened during systole. In diastole, blood fills the pocket-like cusps, causing the valve to close by coapting on the ventric­ ular surfaces of the cusps. The nodules of Arantius are small fibrous mounds at the center of the free edge of each cusp. Three commissures radiate from the center of the valve, giv­ ing the appearance of a "peace sign." Approximately 1 % to 2 % of aortic valves are bicuspid. G R E AT V E S S E L S

Vena Cavae These large veins return blood from the body to the right atrium (see Fig. 3 . 1). The superior vena cava is formed by the juncture of the left and right innominate veins. The azygos vein enters the superior vena cava in the midthorax. Half of the superior vena cava is contained within the pericardium. The superior vena cava enters the upper portion of the right atrium, where its blood flow is directed toward the tricuspid valve. The inferior vena cava is larger than the superior vena cava. It receives blood from the lower body and from the abdominal viscera via the hepatic veins. Only 1 to 2 cm of the inferior vena cava is enclosed by the pericardium. The inferior vena cava enters the right atrium on the lower lateral

Pulmonary Arteries The main pulmonary artery is the most anterior cardiac ves­ sel, located entirely within the pericardium (see Fig. 3 . 1) . It arises from the base of the right ventricle, courses superiorly and posteriorly below the aortic arch, where it bifurcates into the left and right pulmonary arteries. The right pulmo­ nary artery is slightly larger and longer than the left, dividing into a superior ascending branch and an inferior descend­ ing branch. The left pulmonary artery passes over the left mainstem bronchus and subdivides into a variable number of branches that parallel bronchial bifurcations. The left pul­ monary artery is connected to the descending thoracic aorta by the ligamentum arteriosum (ductal artery ligament) , a remnant of the ductus arteriosus. Pulmonary Veins Four pulmonary veins, the right and left, superior and infe­ rior, return oxygenated blood from the lungs to the left atrium (see Fig. 3 . 5) . Occasionally, five or six pulmonary veins may be found. Atrial muscle extends for 1 to 3 cm within the pulmonary veins and functions as a sphincter to prevent reflux of blood during atrial systole. The Aorta The aorta arises from the aortic fibrous ring and passes supe­ riorly and to the right as the ascending aorta (Fig. 3.9). The proximal aorta (aortic root) is dilated and contains the aortic valve and the sinuses of Valsalva. The coronary arteries, the first two branches of the aorta, arise from the left and the right sinus of Valsalva, respectively: The aortic root measures approximately 3 cm at the annulus. The sinotubular junction, at the top of the sinuses of Valsalva, separates the aortic root

Right !nominate -----.... artery

Left �---- subclavian artery

Proximal descend ing aorta

Sinotubular junction

Root ------"

FIGURE 3.9 Aortic va lve, thoracic aorta, and a rch vessels.

CHAPTER 3 • CARD IAC ANATOMY (sinus portion) from the tubular ascending aorta. The proxi­ mal two-thirds of the ascending aorta is located within the fibrous pericardium. In the upper thorax, the aorta courses to the left and posteriorly, forming the transverse aortic arch. Three large vessels arise from the transverse aortic arch: the right innominate (brachiocephalic) artery, the left carotid artery, and the left subclavian artery. The aorta passes over the left pulmonary artery and then descends through the posterior mediastinum to the left of the midline. The liga­ mentum arteriosum (ductal artery ligament) is the remnant of the ductus arteriosus that is connected to the left pulmo­ nary artery In the thorax, the aorta gives rise to 1 2 pairs of intercostal arteries, the anterior spinal artery, and several bronchial arteries. It passes through the diaphragm, where it narrows to approximately 1 . 75 cm, and bifurcates into the iliac arteries at the level of the fourth lumbar vertebra.

Coronary Arteries The coronary arteries are the first branches of the aorta, located on the surface of the heart in the AV and interven­ tricular grooves between the cardiac chambers. They are often covered with fat, which is proportional to body fat and aging. The coronary arteries deliver oxygenated blood to the underlying heart muscle. The right coronary artery is located in the right AV groove. It forms a (-shaped structure when viewed from the left, and an L-shaped structure when viewed from the right (Figs. 3 . 1 0 and 3 . 1 1). The right coronary artery bifurcates at the crux of the heart into a posterior descending branch and an AV branch. The posterior descending artery (PDA) follows the posterior interventricular groove and provides blood supply to the inferior (diaphragmatic) portion of interventricular septum. The AV branch passes beyond the crux of the heart, where it gives off branches that perfuse the posterolateral left ventricular myocardium. A dominant right coronary artery provides a posterior descending branch, an AV branch, and posterior ventricular or posterolateral

Right coronary artery

Right

Posterior

ventricu lar branch

atrial

marg inal branches

Posterior

descendin g artery

FIGURE 3.1 0 Right coronary a rtery, left a nterior oblique view.

29

Conus branch

Branch to sinoatria l node

Posterior

Rog ht ooronary --­ artery

---"4----1

Rog hl atrial

marg lnal

branches

ventricular branch

Septal per1ora tors

Posterior descending

artery

FIGURE 3.1 1 Right coronary a rtery, right a nterior oblique view.

branches. A codominant (balanced) right coronary artery provides a posterior descending branch but does not perfuse the posterior left ventricular myocardium. A nondominant right coronary artery is a small vessel that does not reach the crux of the heart and does not have posterior descend­ ing, AV, or posterior ventricular branches. The size of the right coronary artery is inversely proportional to the size of the circumflex branch. The first branch of the right coronary artery is the conus branch, which perfuses the right ventric­ ular outflow tract. It has a "hook" or "question mark" shape when viewed from the right. Fifty percent of hearts have a separate origin of the conus branch from within the right sinus of Valsalva. The second branch of the right coronary artery is the branch to the sinoatrial (SA) node. This thin vessel courses superiorly and posteriorly, supplying blood to the right atrium and the SA node. It has the appearance of a "tree branch" or an "antler" when viewed angiographi­ cally The right coronary artery has a variable number of right atrial marginal branches and right ventricular marginal branches that arise perpendicularly from the main vessel. The AV nodal artery arises from the AV branch at the crux of the heart and courses superiorly The left main trunk varies from 3 to 1 0 mm in diam­ eter and from 1 to 4 cm in length (Figs. 3 . 1 2 and 3 . 1 3) . Occasionally the left main trunk i s absent, whereby the left anterior descending branch and the left circumflex branch arise from separate but adjacent orifices within the left sinus of Valsalva. The left main trunk trifurcates in 30% of hearts into the left anterior descending, a ramus branch, and the left circumflex branch. The left anterior descending branch, located in the ante­ rior interventricular groove, supplies blood to the anterolat­ eral wall of the left ventricle and most of the interventricular septum. It reaches the cardiac apex in 80% of hearts. The left anterior descending provides four to seven septal perfo­ rators, which supply blood to the anterior interventricular septum, and two to three diagonal branches, which perfuse the anterolateral wall of the heart.

30

SE CTI ON I • FUNDAMENTALS

S u perior vena cava

H ig h la te ra l

branch

circumflex L e ft anterior descend ing

ventricular b ra n ch Posterior

Left circu mflex bra n ch

A n teri or

cardiac veins

Coronary sinus

Oblique vein of the left atrium

Small cardiac vein

Inferior

Lateral circu mflex

branch

branches Diagonal

Middle card i a c ---­ vein

Posterior cardiac vein

FIGURE 3 . 1 2 Left coronary a rtery, left a nterior oblique view.

FIGURE 3.1 4 Frontal view of the cardiac vei n s.

The left circumflex coronary artery arises from the left main trunk and supplies blood to the lateral wall of the heart and to the left atrium. The branches of the circumflex some­ times are referred to as obtuse marginal branches. A different classification system describes these branches in relation to their position on the left ventricle: high lateral, lateral, pos­ terolateral, posterior ventricular, and posterior descending. The size of the left circumflex is inversely proportional to the size of the right coronary artery

groove. It drains upward from the apex toward the base and then passes leftward and posteriorly, paralleling the left cir­ cumflex artery, and entering the coronary sinus at its origin. The posterior vein of the left ventricle drains into the distal end of the coronary sinus. The middle cardiac vein, located in the posterior interventricular groove adjacent to the pos­ terior descending coronary artery, drains into the distal coronary sinus. The small cardiac vein parallels the course of right coronary artery and drains into the distal coronary sinus. There are 3 to 1 2 smaller anterior cardiac veins, some of which drain directly into the right atrium. Many small thebesian veins drain directly into cardiac chambers, most commonly the right atrium and the right ventricle. Venous anatomy is extremely variable, with multiple venous anasto­ moses. The coronary sinus is the largest cardiac vein, meas­ uring approximately 2 to 5 cm in length and 3 to 5 mm in diameter. It is located in the posterior AV groove and drains into the right atrium.

Cardiac Veins The venous system of the heart consists of the coronary sinus, cardiac veins, and the thebesian venous system (Fig. 3 . 14). The cardiac veins generally follow the anatomic course of the coronary arteries, and return blood to the right atrium via the coronary sinus. The great cardiac vein parallels the left anterior descending coronary in the anterior interventricular

Left main tru nk

Lefi circumflex branch

Diagonal branches

Left anterior descending

Posterolateral circumflex --t-'1" /31 No /32 /31 , {32 , and a (a > /32 at high dose)

/31 > {32 , a

Dopa (low doses, 1 0) Arterial, venous dilator Venous A arterial dilator Direct arterial vasodilator a antagonist Diuretic Phosphodiesterase inhibitor

35

Physiologic Effects

1' afterload, usually no change in HR 1' contractility, 1' HR, -1- afterload Marked 1' afterload, modest 1' CO

Low doses 1' CO , -1- SVR, Variable effects on MAP; higher doses 1' SVR, 1' CO 1' CO, -1- SVR, 1' HR with or without a small reduction in BP At moderate doses, 1' contractility, 1' HR 1' CO; high doses (> 1 0 µg/kg) cause vasoconstriction and 1' SVR -1- preload, -1- afterload -1- preload A -1- afterload -1- afterload, no effect on preload -1- afterload, 1' contractility, 1' HR -1- preload Similar to dobutamine, no change in HR, -1- pulmonary vascular resistance

HR, heart rate ; CO, cardiac output; SVR, systemic vascular resistance; MAP, mean arterial pressure.

36

SE CTI ON I • FUNDAMENTALS

cardiac cycles per minute. The principal determinants of myocardial performance are preload, afterload, and con­ tractility, reflected clinically in the SV, the volume of blood expelled from the heart during one cardiac cycle. However, global performance of the heart is generally measured by the ability of the heart to meet the metabolic demands of the body This is reflected in the cardiac output (CO) or cardiac index, defined by the following: CO (L I min) = SV X HR Cardiac index is simply the CO per unit of body surface area (BSA) (Umin/m2 ). Thus, for a given SV, a faster H R results in a higher cardiac index and a slower HR results in a lower cardiac index. Under certain circumstances, HR may also influence myocardial per­ formance during a single cardiac cycle. For example, tachy­ cardia in a patient with severe LV hypertrophy and abnormal relaxation might shorten diastolic filling time and prevent complete isovolumic relaxation, resulting in ineffective pre­ loading. Conversely, bradycardia in a patient with severe aor­ tic regurgitation may overload the left ventricle by allowing a greater volume of regurgitation during diastole. Both condi­ tions may result in reduced cardiac performance and pulmo­ nary congestion, though for different physiologic reasons.

Myocardial Performance Myocardial performance reflects the ability of the heart to meet the metabolic demands of the body There are many surrogate measures of myocardial performance, but CO is arguably the most complete. CO can be assessed both inva­ sively (thermodilution, Fick calculation) and noninvasively (echocardiography, cardiac magnetic resonance imaging [MRI] ) . As mentioned previously, the ejection fraction is a reasonable measure of SV On the cardiology boards, expect to see SV or CO used in reference to myocardial performance. G RA P H I C I L L U S T R AT I O N O F M YO C A R D I A L P E R F O R M A N C E The concepts o f preload, afterload, and contractility can be represented graphically by the use of Frank-Starling curves, force-tension curves, and pressure-volume (PV) loops, respectively These graphics allow the reader to visualize the interplay between the components of myocardial per­ formance and understand how individual disease states and interventions alter myocardial performance. Expect to see these on the cardiology boards.

Starling's Law and Fran k-Starling Curves Starling's law dictates that cardiac performance (defined by SV) increases as preload is increased. However, there is a nonlinear relationship between EDP (a measure of preload) and SV, as shown in Figure 4. 1 . When afterload and con­ tractility are held constant, reduced preload (shift along the line to the left) will reduce SV while increased preload (shift along the line to the right) will increase SV However, if

Preload

FIGURE 4.1 Fra n k-Sta rling cu rves. As preload increases, SV increases.

optimal preloading conditions are exceeded, the SV will no longer increase in response to higher filling pressure, which is depicted by the plateau of the curve. In addition to preload, ventricular contractility and afterload can influence Frank-Starling curves. When con­ tractility is increased and preload is held constant, the SV increases (entire line shifts upward and to the left) . When contractility decreases, the SV decreases in response (entire line shifts downward and to the right) . When afterload increases while preload and contractility are held constant, the SV decreases (entire line shifts downward and to the right) , while SV increases when afterload decreases (entire line shifts upward and to the left) .

Force-Tension Curves These curves describe the relationship between CO and after­ load (Fig. 4.2). When preload and contractility are held con­ stant, any decrease in afterload will produce an increase in SV (shift along the line to the left) , whereas an increase in after­ load will produce a comparable decrease in SV (shift along the line to the right) . As with the Frank-Starling relationship, preload and contractility also influence the force-tension relationship . As preload increases for a given afterload, SV also increases (entire line shifts upward and to the right) , whereas SV decreases if preload is decreased (entire line shifts downward and to the left) . When contractility increases for a given afterload, the SV increases (entire line shifts upward and to the right) . When contractility decreases, SV also decreases (entire line shifts downward and to the left) . Pressure-Volume Loops If the ventricular volumes during one cardiac cycle are plotted against simultaneous pressures within the ventricle, "

E

0 ::::

> " ""

e Zi

A fterl oad

FIGURE 4.2 Force-tension cu rves. As afterload is red uced, SV

increases.

CHAPTER 4 • CARD IOVASCULAR PHYSIOLOGY: FLOW-VOLUME LOOPS

37

Together these two pressure-volume relationships are the primary components of the PV loop.

Pressure

Closes

Isovolumic Contraction

Diastolic Pressure-

:=---._ ··•

Volume

�--9 °' .,,,F---,, -A-----;End Diastole _

Volume

Relationship

(pre load)

FIGURE 4.3 PV loops. Ti me A is the onset of systole,

i m m ed iately fol lowi ng the closure of the m itral va lve. This is fol l owed by the period of i sovol u m i c contraction (change i n pressu re with no c h a n g e i n vol u me). T h e aortic va lve opens at point B, and the point of maxi mal ventricular activation is reached at point C, after which the aortic va lve closes. This is fol l owed by a period of isovo l u m i c relaxation (change in pressu re with no change i n vol u me). The m itra l va lve opens at point D, fol l owed by the fi l l ing of the LV. Systole i ncludes the time period of isovo l u m i c contraction a n d ejection (point A to C). Diastole incl udes the period of isovo l u m i c relaxation and fi l l i n g (point C to A). The d otted l i nes represent the end-systolic a n d EDPVR, which represent the bou ndaries of the PV loops.

a PV loop is constructed as shown in Figure 4 . 3 . The PV loop is a rectangular illustration of the pressure and volume events that comprise a single cardiac cycle. The base of this rectangle is formed by the end-diastolic pressure-volume relationship (EDPVR) , a line that describes the properties of the ventricle at the point of maximal diastolic relaxation. The EDPVR is nonlinear and analogous to the Frank-Starling curve since it illustrates the relationship between diastolic filling and myocardial performance. The end-systolic pressure-volume relationship (ESPVR) describes the systolic filling changes in the left ventricle. This curve is linear, beginning at the intersection of the pressure and volume axes, and touching the PV loop at the point of maximal end-systolic activation, when the aortic valve closes prior to isovolumic relaxation.

I NT E R PRETI N G TH E P R E S S U R E -VO L U M E LO O P

Ventricular Vol u mes The maximum volume in the ventricle during the cardiac cycle is the EDV, the point on the x-axis where the mitral valve closes and isovolumic contraction begins. The minimum LV volume is the end-systolic volume (ESV) , the point on the x-axis at the end of isovolumic relaxation prior to the opening of the mitral valve. The SV is simply the difference between the two volumes: SV = EDV - ESV

l ntracardiac Pressures As shown in Figure 4. 3 , point B represents the point at which the ventricle begins to eject blood into the vasculature (open­ ing of the aortic valve) . At this time, the ventricular pressure just exceeds aortic pressure. During the ejection phase, aortic and ventricular pressures are nearly equal, so the point of greatest pressure on the loop represents the greatest pressure in the aorta and is equal to the systolic blood pressure (SBP) . End-systolic pressure (Pe,) is the pressure on the PV loop at the end of systole and is only slightly less than the maximal pressure (SBP) . Point D, following isovolumic relaxation, rep­ resents the pressure in the left atrium (LAP) at the time the mitral valve opens. EDP is represented at the end of diastole (point A) and is influenced by the compliance of the chamber. Compliance Compliance is the change in volume for a given change in pressure or, in mathematical terms, the reciprocal of the derivative of EDPVR. EDPVR is nonlinear and hence com­ pliance varies with volume. Change in volume for a given change in pressure is greater at low volumes (greater compli­ ance) than at higher volumes (lower compliance) . Changes in compliance are related to structural and pressure changes of the heart, pericardium, and thorax.

Valvular Heart Disease: PV Loops

Va lve Condition

Aortic stenosis Aortic regurgitation Mitral regurgitation Mitral stenosis

Afterload

I I J,

Pre load

Contractil ity (Early)

I H I

Contractil ity (Late)

38

SE CTI ON I • FUNDAMENTALS

f

3 VJ

(!)

VJ (!)

I



1-4

------� Normal

�.,

�-----� Aortic stenosls

_.

:

. .� • • • •

_: / ...

·····

·

Ao rt i c reg u rg itation M itra! reg u rgitation

... ... .. . M itra! stenosls ' � ---� --+-· ------� � ---700---1-l . : . ' . : .

:1

:

. - - =r : •

I

;:



! : : = I



. : • . L--""I.!"!' ._ -.... ...... . .... ..... ..... . '

50

1 50 Volume

250

(\)

3

. . . . ..

,._.,

VJ VJ

(\)

� 1-4

. . . ...�----­ . .

Hypertro p h l c

CM

Restrictive CM Normal

1 00--w-----.-.�-----;-i.---,,�::--� -

50

Dilated CM

1 50

Volume

250

FIGURE 4.5 PV loops in va rious cardiomyopathies.

FIGURE 4.4 PV loops in va rious va lvu lar disease states, with

cu rves as depicted here fo r aortic and m itra l reg u rg itation representi ng deco m pensated states.

Elastance and Contractility Defined as the linear relationship between the change in pressure for a given change in volume at end systole on the ESPVR, elastance is represented by the slope, Emax or Ees· Elastance is a surrogate for contractility because it is inde­ pendent of external conditions such as preload or afterload. Stroke Work Stroke work represents the work of the heart during each heart beat and is represented by the area of the PV loop.

Cardiomyopathy: PV Loops Condition

Dilated Hypertrophic Restrictive

ESV

i j, J, or H

EDV

i j, J, or H

ESV, end-systolic volume; EDV, end-diastolic volume.

Contractil ity

j, i J, or H

S U M M A RY OF T H E A C T I O N S OF T H E A U TO N O M I C N E RVO U S S YS T E M O N T H E CO M P O N E N TS O F M YO C A R D I A L PERFORMANCE al stimulation: i afterload, i preload

[31 stimulation: i contractility, i HR [32 stimulation: J, afterload

VA LV U LA R H E A RT D I S E A S E A N D P R E S S U R E -VO L U M E LO O P S The PV loops in various valvular diseases are depicted in Table 4.2 and Figure 4.4.

C A R D I O M YO PAT H Y A N D P R E S S U R E -VO L U M E LO O P S The PV loops in various cardiomyopathies (CM) are depicted in Table 4.3 and Figure 4 . 5 .

CHAPTER 4 • CARD IOVASCULAR PHYSIOLOGY: FLOW-VOLUME LOOPS

39

QU ESTIONS AND ANSWERS Questions

1 . Which of the following medications will take this patient from point A to point B on the Fra nk-Starling and force-tension curves shown?



B.

r

;,.,. :,) c

..... :Zi �

a. b. c. d. e.

Prcloa -

"..>

� \:::

z

l

3. A patient with congestive heart fai l u re is started on ora l ca ptopril. Which d i rection would this patient move on the Fran k-Sta rling plot shown?

B

�A

----

-

A licrload

2. Which of the following medications wi l l take a patient from point A to point B on the force-tension cu rve shown?

a. b. c. d. e. 4. Which of the fol lowi ng med ications wi l l ta ke the pa­ tient from point A to point B on the cu rve shown?

a. b. c. d. e.

Epinephrine Phenylephrine Furosemide lsoproterenol Digitalis (Con tin ued)

40

SE CTI ON I • FUNDAMENTALS

a. b. c. d. e.

Norepi nephrine Phenylephrine Fu rosemide lsoproterenol Digita l i s

5 . W h i c h o f the fol l owing medications will cause t h e ind icated c h a n g e (solid l o o p t o dotted l i ne) on the flow-volume loop shown?

a. b. c. d. e.

Phenylephrine Ca ptopril Hyd ra lazi ne + nitrates Digitalis + I ntravenous fl uids (IVF) Epinephrine

7. Com pa red to the normal PV loop shown (solid l i n es), the PV loop to the right (dotted l i n es) would best reflect which of the fol l owing conditions?

a. b. c. d. e.

Norepi nephrine N itroprusside M i l rinone Hyd ralazine Phenylephrine

6. Which of the following medications, administered acutely, will shift the pressure-volume (PV) loop as outlined below (from the solid line to the dotted line)?

a. b. c. d. e.

Hypertrophic ca rdiomyopathy Restrictive ca rdiomyopathy Di lated cardiomyopathy Pericardia! constriction M itra l stenosis

8. Which of the fol lowi ng va lvu lar disease states would change the PV loop in the manner shown (solid l ines to dotted l i nes)?

CHAPTER 4 • CARD IOVASCULAR PHYSIOLOGY: FLOW-VOLUME LOOPS

41

afterload. Digitalis increases contracti lity but does not infl uence afterload. 3 . Answer D: Captopril red uces both afterload and

preload. At a given contractility, it would be expected to i m p rove SV. Preload a lso decreases with c but so does SV (as i n d i u resis, n itrates). Preload is unchanged with b a nd e. The decreased SV with b could be seen with a n agent such as phenylephri ne. The i ncreased SV of e could be seen with a p u re afterload red ucer l i ke hyd ra la­ zine. Preload and SV increase with a, which might occur with IV hyd ration. its effects on al receptors while increasi ng contractil ity by its action on /31 receptors. Phenylephrine increases afterload but should not i m p rove SV. Furosemide does not increase afterload. lsoproterenol and d igitalis can i m p rove SV, but neither increases afterload.

4. Answer A: Norepinephrine increases afterload by

a. b. c. d. e.

M itra l reg u rg itation M itra l stenosis Aortic reg u rg itation (early) Aortic stenosis (early) Restrictive cardiomyopathy

9. What is not a determ inant of myocard ial perfo r­ ma nce? a. b. c. d. e.

Compliance Heart rate (HR) Preload Afterload Contracti l ity

5 . Answer C: M i l rinone increases contractil ity and decreases afterload, resulting in a g reater SV. Norepi­ nephrine may increase contractility but a lso increases afterload. N itroprusside red uces afterload and preload but does not affect contractil ity. Hyd ralazine red uces afterload and i m p roves SV but does not affect contrac­ til ity. Phenylephrine increases afterload and does not i m p rove SV. 6. Answer A: Phenylephrine increases afterload more than it increases preload and wou l d be expected to red uce SV as shown. Ca ptopril and hyd ralazine wou l d red uce afterload and l i kely i m p rove SV. Epinephrine would increase contractility and SV and can increase afterload at higher doses, but we wou l d also expect to see increased contractility. Digita l i s would a lso increase contractil ity, and IV fl u ids would increase preload as wel l .

1 . Answer C: Hyd ra lazi ne red uces afterload and increases stroke vol u me (SV) without changing preload. Propranolol acutely red uces contractility and would not be expected to improve SV. Norepi nephrine and phen­ ylephrine both increase afterload. Fu rosemide red uces pre load.

7. Answer C: I n dilated ca rdiomyopathy, t h e ventricle operates at hig her fi l ling pressu res with g reater af­ terload. SV and contractil ity a re red uced, as depicted. Hypertrophic ca rdiomyopathy wou l d be associated with a "spike and dome" pattern of early increased systolic pressu re followed by abrupt decrease before retu rn of systolic ejection i n mid-late systole. There should be no major change i n preload. Contractil ity is usua l ly increased, and SV wou l d be preserved or augmented. Restrictive ca rdiomyopathy and perica rdia! constriction can both result i n lower SVs, but afterload is genera l ly normal or red uced. There wou l d a lso be early eq ualiza­ tion of the diastolic pressure-vo l u m e cu rve. M itra l ste­ nosis wou l d be associated with red uced SV secondary to reduced pre load; afterload and contracti lity wou l d be u naffected.

2 . Answer D: lsoproterenol red uces afterload while

8. Answer D: Early aortic stenosis is cha racterized by

i ncreasing contractil ity. Epinephrine and phenylephrine both increase afterload. Fu rosemide does not affect

sign ificantly increased afterload and increased LV sys­ tolic pressu re with increased contracti l ity and preserved

1 0. Which pa rameter is used in Laplace law to ca lculate left ventricular (LV) wa l l stress? a. b. c. d. e.

Left atria l radius LV wa l l thickness Left atrial pressu re Right ventricu lar wa l l th ickness Right atrial pressu re

Answers

(Con tin ued)

42

SE CTI ON I • FUNDAMENTALS

SV, as shown. The increased contractility is the result of g reater LV mass produced by com pensatory hyper­ trophy. I n later stages, the LV beg i n s to d i late, increas­ ing afterload more with a d rop in contractility and SV. Preload eventual ly rises. M itra l reg u rg itation would be associated with increased d iastolic fi l l ing pressu re. Contractility is preserved or enha nced (at least i n itial ly) with decreasing LV systolic pressure that is proportional to the reg u rg itant vol u me. SV decreases as the reg urgi­ tant vol u me increases. M itra l stenosis would prod uce a n underfi l led ventricle with low SV and normal after­ load. Early aortic reg u rg itation wou l d g reatly increase

d iastolic fi l ling pressu re with increased contractil ity and normal to increased SV. Over time, contractil ity would decrease and afterload would increase as the LV dilates. 9. Answer A: Myoca rdial performance is determi ned

by contractility, H R, preload, and afterload. Although com plia nce does have a n effect on preload, it is not a d i rect determ inant of myocardial performa nce. 1 0. Answer B: LV wa l l th ickness.The compo­ nents that are used in Laplace law to calcu late wa l l stress include LV pressu re, LV rad i u s, and LV wa l l thickness.

Basic C ardiac Electrophy siolo gy Sergio G . Thal and Patrick ] . Tchou

T

he aim of this chapter is to cover the main aspects of basic cardiac electrophysiology, developed in a review fashion for the Cardiovascular Medicine Board Exam­ ination. The information is organized as follows: 1 . Basic action potential (AP) and ion channel implications

2. Electrical activity coupling mechanisms

• Drug binding (local anesthetics) • Susceptibility to many different neurotoxins

Levels of Na+ Channel Activity Regu lation 1.

3. Conduction system anatomy

4.

Local electrophysiology characteristics of various conduction system components

Transcriptional regulation of Na• channel proteins is a mecha­ nism to control Na• channel expression at a genomic level. This pattern of regulation can be influenced by feedback originating in the tissue electrical activity The exact mechanism of this gene regulation remains incompletely understood.

2 . Phosphorylation/dephosphorylation of the a subunit 3. Glycosylation. The regulation mechanism affects all channel

M E M B RA N E ACT I O N P OT E N T I A L The initiation o f the cell membrane AP is the first event in a process that ends with a cardiac contraction. Grossly, myo­ cardial cells can be divided into those dependent on sodium ions (Na+) or calcium ions (Ca2 +) to drive AP depolarization.

Sodium-Dependent Cel ls Each AP starts with net movement of ions across the cell membrane. In a steady state, the membrane is polarized near -90 mV The transmembrane ionic current is the result of the balance between many inward and outward ionic cur­ rents. The sodium channel is voltage sensitive. This means that the probability of the channel being open for transport of the sodium ion increases with increase of transmembrane voltage (toward zero) . When a cell receives depolarizing cur­ rent, sodium channels open and increase the inward current. When the inward current exceeds the total outward current, a rapid opening of sodium channels occurs that overwhelms any outward current, resulting in the rapid upstroke portion of the AP termed Phase 0 (Figs. 5 . 1 and 5 .2). Sodium channels are characterized by a protein that works as a voltage-gated system. The active portion of this channel is the a subunit, which consists of a 2,000-amino acid glycopro­ tein. Properties of these channels include the following: • Selective permeation • Gating (activation and inactivation)

subunits.

Abnormalities of the sodium channel can result in both the long QT syndrome and the Brugada syndrome. Phase 1 (see Fig. 5 .2) starts with the opening of a rapid outward potassium (K+) current called I,0• This determines a fast early repolarization with a prominent notch shape that approximates the membrane potential to 0 mV These chan­ nels are characterized by outward movement of K• ions, which constitutes the principal source of membrane repolarization early during the AP. The channels inactivate soon after acti­ vation, although not as rapidly as the sodium current does. A dynamic interaction of four a subunits and an apparatus composed of a cytoskeleton and signaling complexes mainly form the K• pore. During the AP, these K• channels activate in response to membrane depolarization and inactivate in a timed manner. The channels are regulated by: • Angiotensin II, which reduces Ito fast velocity • a-Adrenergic stimulation, which reduces Ito fast velocity



Hyperthyroidism, which increases Ito current density

• Aldosterone, which mediates a receptor-specific downregula­ tion of I'°

In human pathophysiology, these channels provide an early repolarization current that can drive the transmem­ brane voltage toward resting membrane potential when the sodium current is dysfunctional. Thus, in Brugada 43

44

SECTI O N I • FUNDAMENTALS

1 5 mV O mV 2

0

-90 mV FIGURE 5.1 Action potential (sod i u m channel tissue). AP model of a sod i u m tissue. N u m bers 0, 1, 2, 3, 4 deli neate the different phases of the AP.

syndrome, in which there is an abnormality in the sodium current that results in depressed sodium conductance, the I,0 current may cause full repolarization in a portion of the myocardium early during the AP, resulting in a large volt­ age gradient between the repolarized part and parts that have more normal AP Such gradients have been demon­ strated in isolated tissue preparations to be capable of initi­ ating reentrant wavefronts. These reentrant wavefronts can initiate polymorphic ventricular tachycardia or ventricular fibrillation. The next portion of the AP, termed Phase 2 or the plateau phase (see Fig. 5 .2), is the result of the balance of two differ­ ent ion currents. During Phase 0 , at the level of -40 mV, Ca2 + channels open, creating an inward Ca2 + current. This current, acting as an antagonist to the outward K+ current, exerts its action by stabilizing transmembrane potential during the pla­ teau phase. This phase concludes as the Ca2 + current declines by inactivation of L-type Ca 2 + channels. These channels are also the critical initiators of cardiac excitation-contraction coupling through the initial increase in intracellular Ca2 +, which triggers the release of Ca2 + from the sarcoplasmic reticulum, which in turn provides a contraction signal to the cellular contractile elements. At a level of -40 mV of mem­ brane potential, these channels rapidly activate, reaching a peak in approximately 2 to 7 milliseconds. Inactivation of the channel depends on time, membrane potential, and Ca2 + concentration.

Extra Cell

I ntra Cell

Phase o

Phase 1

Phase 2

Phase 3

Phase 3 (see Fig. 5 . 2) , the repolarization phase, is dom­ inated by the outward current of K+ through the so-called "delayed rectifier" K+ channels, which are responsible for the return of the cell membrane to its resting polarized state. Two types of delayed rectifiers are important in the repolarization of human ventricular myocardium, a rapidly activating IK, and a more slowly activating IKs that peaks late in the AP, during Phase 3 . Abnormalities in either of these two types of delayed rectifier K+ channels can cause the long-QT syndrome. Phase 4 (see Fig. 5 .2) constitutes a stable polarized membrane. This stabilization of membrane AP after the descending Phase 3 is achieved mainly by the action of the voltage-regulated inward rectifiers (IKJ) . These channels behave differently than the delayed rectifiers, which open in response to depolarization. The inward rectifier K+ chan­ nels are opened at near-resting membrane potential, stabiliz­ ing the resting membrane potential near the K+ equilibrium potential, but close in response to depolarization, facilitating the AP, hence the description of "inward rectifying. "

Myocardial Tissues That Have Calcium­ Dependent AP versus Tissues with Sodium Channels The main differences between these two types of myocardial tissue can be found in Phases 4 and 0 of the AP Calcium­ dependent tissues are the principal cellular component of the specialized conduction system and the sinus node. These cells have the ability to generate a spontaneous AP based on the differential characteristic of Phase 4. This difference is produced by ion currents that affect Na+ and K+ concentra­ tions, called Ir, which activate at membrane potentials below -40 mV, and the K rectifier currents. These currents confer an unstable electrical property, causing these cells to develop spontaneous diastolic depolarization and automatic onset of APs in a rhythmic fashion. Once spontaneous diastolic activity raises the membrane potential to a value of -40 mV, opening of the slow Ca 2 + channels results in an inward Ca 2 + current CL-type Ca) that generates the slow AP upstroke (Phase 0) . Na+ channels possess a small, if any, role in the AP generation in these particular cells (Fig. 5 .3). O mV

1 -2

Phase 4 ATPase

-40 mV

Na

K Cl Ito lto2

K

Ca

K

Na

FIGURE 5.2 Main ion channel activities in AP phases. Na,

sod i u m; K, potassium; Cl, chloride; Ca, ca lcium.

K

-90 mV FIGURE 5.3 Action potential (ca lcium channel tissue). AP model of a ca lcium tissue. N u m bers 0, 1 , 2, 3, 4 delineate the different phases of the AP.

CHAPTER 5 • BAS I C CARD IAC ELECTROPHYSIOLOGY E L E C T R I C A L CO U P L I N G C E L L S { G A P J U N CT I O N ) GAP junction channels are the functional units that produce direct ionic communication between cardiac cells and play a maj or role in the propagation of the AP from one myocardial cell to the next. The molecular unit of the GAP junction is a protein called connexin. The oligomerization of six con­ nexins forms a connexon, and two connexons form the final channel called the GAP junction. Among the 20 different subtypes of connexins identified in the human genome, con­ nexin 43 is the most abundant in myocardial cells. Besides the genomic regulation of channel expression, they are also affected by the activity of protein kinase activity, low intra­ cellular pH, and dephosphorylation. GAP junction channels are not uniformly distributed within cardiac tissues. They are almost absent in the sinus node, are found in low concentration in various areas of the atrioventricular node (AYN) , and demonstrate significant expression in the faster conducting atrial and ventricular muscle as well as His-Purkinje fibers. In these cells, the dis­ tribution of the connexons is not uniform. They are more concentrated along the ends of the myocytes than along the sides of the cell, thus giving a directional propensity for AP propagation. This gives rise to anisotropic propagation of depolarization, with faster conduction velocities along the muscle fiber orientation compared to the slower transverse fiber-orientated velocities. CO N D U CT I O N SYST E M A N ATO M Y A N D P H YS I O LO G Y The sinus node is located beneath the epicardial surface o f the crista terminalis, at its junction with the high right atrium. It possesses a spindle-shaped structure and measures an aver­ age 1 0 to 20 mm in the long axis and 2 to 3 mm in the trans­ verse axis. It is composed of a cumulus of small cells called P cells; the main component of the natural pacemaker. They are grouped in elongated clusters and are centrally located within the sinus node. Transitional cells called T cells sur­ round the P cells and transmit the impulse generated by the P cells to the surrounding atrium. The final synchronized activity of the sinus node is achieved via the presence of GAP junctional channels that electrically couple the depolarization of P cells. At the periphery of the node, strands of nodal cells interdigitate with atrial cells, forming lateral connections and transferring the pacemaker impulse to the atrial cells. This organization is believed to be important to impulse propaga­ tion from a small source (the nodal cells) to a large reservoir (the atrial myocardium) , preventing excessive dampening of the pacemaker current within the nodal cells by the large reservoir of atrial myocardium. Cells within the sinus node demonstrate spontaneous diastolic depolarization, initiating APs in a repetitive fash­ ion. These APs are calcium channel dependent and possess a similar morphology to those of the atrioventricular nodal

45

cell, characterized by a slow Phase 0 upstroke velocity Sinus node cells do not possess the GAP junction protein connexin 4 3 . Therefore, electrical coupling at the node center is poor, reflected as a low measured conduction velocity. The periph­ ery is associated with an increase in conduction velocity This characteristic is most likely important in isolating the sinus node from the potential suppressive hyperpolarizing influence of the atrial myocardium. Normal sinus node function is affected by age. In the young, the intrinsic heart rate is faster, but vagal tone pre­ dominates at rest, causing slowing of the heart rate. In the elderly, resting autonomic tone tends to shift away from vagal predominance to sympathetic outflow. Thus, the extrinsic sinus rate at rest, the rate as modified by autonomic tone, tends to be similar within the ages of adulthood. The impulse generated at the sinoatrial node is next transmitted to the AYN through atrial myocytes. There is anatomic evidence for the presence of three atrionodal path­ ways traversing the right atrium. A fourth pathway, called Bachmann bundle, derived from the anterior atrionodal pathway, directs impulse propagation to the left atrium via the interatrial septum. Anatomically, these so-called path­ ways do not demonstrate any specialized conduction tissue. Rapid conduction along these intra- and interarterial paths appears to be correlated with fiber size and orientation rather than the presence of specialized conduction tissue. The AYN is a fusiform structure located subendocardi­ ally along the annular regions of the interatrial septum, with its distal end, the compact node, at the superior corner of the triangle of Koch. The triangle of Koch is defined by the inser­ tion of the septal leaflet of the tricuspid valve, the tendon of Todaro, and the line that connect the os of the coronary sinus and the tricuspid annulus. The body and the proximal end (the tail) of the AYN are directed posteriorly along the tricuspid annulus. A second tail extends from the body of the AYN along the mitral annulus. The so-called slow path­ way of the AYN corresponds to the tail of the AYN , whereas the fast pathway involves atrial inputs into the distal com­ pact node. Similar to the sinus node, transitional cells sur­ round this structure. Circulation to the AYN is provided in nearly 90% of individuals by branches of the right coronary artery extending superiorly from the crux into the trigone area along the AV annulus. The His bundle is the anatomic structure that connects the compact AYN to the bundle branches. At the junction between the distal AYN and the proximal His bundle, the cells undergo a gradual change from possessing node-like APs to having His-Purkinje APs. That is the APs change from having slow upstrokes dependent on Ca2 • current to fast upstrokes dependent on Na• current. The branches from the anterior and posterior descending arteries provide circu­ lation to this portion of the conduction system and confer a better security margin for ischemic damage. The His bundle penetrates the AV ring at the central fibrous body and then arches anteriorly and inferiorly along the crest of the septal myocardium that forms the lower edge of the membranous

46

SECTI O N I • FUNDAMENTALS

ventricular septum. As it courses along the crest, left-sided fibers in the His bundle drop over the crest into the left ven­ tricle, forming the posterior, septal, and anterior fascicles of the left bundle branch. The His bundle then continues its course over the right ventricular septum as the right bundle branch. The right bundle brunch adopts a subendocardial trajectory over the right side of the interventricular sep­ tum and transmits the cardiac impulse to the Purkinje fib­ ers located at the apical portions of the right ventricle. The bundle branches spread into a smaller Purkinj e bundle and then into finer fibers that terminate at the myocardium. This branching structure of the His-Purkinje system facilitates a near-synchronous arrival of the sinus impulse at the myocar­ dial endocardial surface. Electrophysiologically, the AVN can be differentiated into three portions: atrionodal, compact node, and nodo­ His. The compact node area presents a response character­ ized by an AP with a slow rate of rise during its upstroke and a low amplitude. The other two zones have transitional characteristics between the compact node zone and the atrial and His bundle potentials, respectively Calcium-type APs characterize the main AVN cellular type. Differentiating the AVN from the sinus node, GAP junctions play an important role in AVN conduction. Con­ nexin 45 is present in this portion of the conduction system, though at a low level. It has also been demonstrated that the expression of connexin 45 constitutes the molecular basis of AVN dual pathways. S U G G E ST E D R E A D I N G S Beardslee MA, Tadros PN , Laing JG, et al. Dephosphory­ lation and intracellular redistribution of ventricular connexin43 during electrical uncoupling induced by ischemia. Circ Res . 2000;87(8) :656-662 .

Boyett MR, Kodama I. The sinoatrial node, a heterogeneous pacemaker structure. Cardiovasc Res. 2000 ;47(4) :658-687. Coppen S R . Diversity of connexin expression patterns in the atrioventricular node: vestigial consequence or functional special­ ization? ] Cardiovasc Electrophysiol. 2002 ; 1 3(6): 625-626 . d e Carvalho A, d e Almeida D . Spread of activity through the atrioventricular node. Circ Res. 1 960;8:80 1-809 . Difrancesco D, Mazzanti M, Tromba C. Properties of the hyperpolarizing-activated current (if) in cells isolated from the rab­ bit sino-atrial node . ] Physiol. 1986;377:6 1-88. Douglas P, Zipes M]] . Cardiac Electrophysiology. From Cell to Bedside. 4th ed. Philadelphia: WB Saunders; 2004. Gourdie RG, Green CR, Rothery S , et al. The spatial dis­ tribution and relative abundance of gap-junctional connexin40 and connexin4 3 correlate to functional properties of compo­ nents of the cardiac atrioventricular conduction system. ] Cell Sci. 1993 ; 1 05 (4) :985-9 9 1 Irisawa H. Cardiac electrophysiology: past, present and future. Part II. Membrane currents in cardiac pacemaker tissue. Ex­ perientia. 1 987;43: 1 1 3 1- 1 1 3 5 . Kwak BR, De Jonge HR, Lohmann S M , e t al. Differential regulation of distinct types of gap junction channels by similar phosphorylating conditions. Mol Biol Cell. 1995;6(12) : 1 707- 1 7 1 9 . Makowski L , Phillips WC , Goodenough DA. Gap junc­ tion structures. II. Analysis of the x-ray diffraction data. ] Cell Biol . 1977;74(2) : 629-645 Morley GE, Delmar M . Intramolecular interactions mediate pH regulation of connexin43 channels. Biophys]. 1996 ;70(3): 12941 302. Nikolski V P, Lancaster M K , Boyett M R , et a l . Cx4 3 and dual-pathway electrophysiology of the atrioventricular node and atrioventricular nodal reentry Circ Res . 2003;92( 4) :469-4 75. Trabka-Janik E, Lemanski L F, Delmar M, e t a l . Immunohis­ tochemical localization of gap junction protein channels in ham­ ster sinoatrial node in correlation with electrophysiologic mapping of the pacemaker region. ] Cardiovasc Electrophysiol. 1 994;5(2) : 1 25-1 3 7 .

QU ESTIONS AND ANSWERS Questions

1 . Which of the fol l owing is not a characteristic of sod i u m chan nels? a. Selective permeation b. Pump electrolytes exchange mechanism c. Gating d. Drug binding e. Susceptibil ity to many d ifferent neu rotoxins 2. Which of the fol l owing is not a characteristic of Phase 1 of the action potential (AP)? a. Phase 1 sta rts with the opening of a rapid outward K+ ion cu rrent ca l l ed /to' b. These K+ channels activate i n response to mem brane depolarization. c. These K+ channels inactivate i n a ti me-dependent m a n ner.

d. a-Ad renergic sti m u l ation increases /to maxi m u m cu rrent. e. Aldosterone mediates a receptor-specific down­ reg u lation of /to·

3. Which of the fol lowing is not a characteristic of calci u m chan nels? a . During Phase 0, at the level of -40 mV, Ca 2 + chan­ nels open, creating a n inward Ca 2+ cu rrent. b. Th is cu rrent acts as an agonist to the outwa rd K+ cu rrent. c. Phase 2 concl udes as Ca 2 + cu rrent declines by inactivation of L-type Ca 2 + chan nels. d. I nactivation of the chan nel depends on time, mem brane potential, and Ca concentration. e. Intracel lular Ca 2+ concentration acts as a critical initiator of cardiac excitation-contraction coupling.

CHAPTER 5 • BAS I C CARD IAC ELECTROPHYSIOLOGY

4. Which of the fol lowi ng is the main mechanism by which resting mem brane potentia l (Phase 4 of the AP) is maintai ned? a. Delayed rectifier K+ channels b. Voltage-reg ulated i nward rectifiers c. These channels open i n response to depolarization. d. These channels open after reaching the resting mem brane potential. e. These potassiu m chan nels stabil ize the resting mem brane potential near the sod i u m eq u i l i b­ ri u m potential. 5 . Which of the following statements a bout GAP j u nc­ tions is wrong? a. GAP j u nction chan nels are the fu nction a l u n its that a l low d i rect ionic com m u nication between cardiac cel ls. b. GAP j u nctions play a major role in the propaga­ tion of the AP. c. The molecu lar u n it of the GAP ju nction is a pro­ tei n ca lled connexi n. d. Connexi n 43 is the most abundant i n cardiac conduction system cel l s. e. Connexin 43 may be affected by the activity of protein kinase, low i ntrace l l u l a r pH, and dephos­ phorylation. Answers 1 . Answer B: Sod i u m channels a re cha racterized by

a protein that works as a voltage-gated sod i u m chan­ nel. The active portion of this channel is the a subun it, which consists of a 2,000-a mino acid g lycoprotein. The other choices a re a l l properties of these chan nels. 2 . Answer D: Phase 1 sta rts with the open ing of a

rapid outwa rd K+ ion cu rrent cal l ed iwThis determines a fast early repola rization. These K+ chan nels activate i n response t o mem brane depolarization and i nactivate i n a time-dependent m a n ner.These channels may b e reg u­ lated by the fol lowi ng means: • Ang iotensin I I red uces l ,0 maxi m u m velocity. • a-Adrenergic sti m u lation red uces l,0 fast velocity.

47

• Hyperthyroidism i ncrease l,0 cu rrent density.

• Aldosterone mediates a receptor-specific downregulation of l,0•

3. Answer B: During Phase 0, at the level of -40 mV, Ca 2 + channels open, creating a n inward Ca 2 + cu rrent. Th is cu rrent act as a n a ntagonist to the outwa rd K+ cu rrent. Phase 2 concl udes as Ca 2+ cu rrent decl ines by inactiva­ tion of L-type Ca 2+ channels, letting the plateau phase subside. These channels are also the critica l i n itiators of cardiac excitation-contraction coupling through the i n itial increase i n intrace l l u l a r Ca 2 + concentration that triggers the release of Ca 2 + from the sarcoplasmic reticu l u m, which i n turn provides a contraction signal to the contractile elements of the cel l. I nactivation of the channel depends on time, mem brane potential, and Ca concentration. 4 . Answer B: The stabil ization of resting mem brane

potentia l after the descending Phase 3 of the AP is achieved mainly by the action of the voltage-reg ulated inward rectifiers ( /K1). These chan nels behave d ifferently than the delayed rectifiers, which open in response to depolarization.The i nwa rd rectifier K+ channels are opened near resting mem brane potential, stabilizing the resting mem brane potential near the K+ eq u i l i b­ ri u m potential, but close i n response to depola rization, facil itating the AP, hence the description as "inwa rd rectifyi ng." S. Answer D: GAP junction channels are the functional

u n its that a l low d i rect ionic com m u nication between car­ diac cells and play a major role i n the propagation of the AP from one cell to the next.The molecular u n it of the GAP junction is a protein cal led Connexin.The oligomer­ ization of six connexi ns forms a connexon, and two con­ nexons form the final channel cal led the GAP junction. Among the 20 different su btypes of connexins identified in the h u man genome, con nexin 43 is the most abun­ dant i n myocardial cel ls. Besides the obvious genomic reg u lation of the expression of these chan nels, they may also be affected by the activity of protein kinase activity, low i ntracellular pH, and dephosphorylation.

C ardiac Biocheinistr y Mosi K. Bennett and Marc S . Penn

T

he biochemistry of cardiac tissue involves tightly regu­ lated interactions among ions, proteins, receptors, sec­ ond messenger systems, and various cellular structures as well as extracardiac influences. Several abnormalities involving neurohormonal pathways as well as derangements of the contractile apparatus of the cardiac myocyte have been demonstrated in cells isolated from failing hearts. This chap­ ter reviews some of the salient features of the biochemistry of the cardiac myocyte.

CA R D I AC CO N T RACT I L I T Y A N D C A LC I U M H O M E O S TA S I S The cardiac myocyte is an interconnected network of myofi­ brils surrounded by sarcoplasmic reticulum (SR) . Each myofi­ bril comprises sarcomeres made up of thick myosin filaments and thin actin filaments that form the basic contractile unit of the cardiac myocyte (Fig. 6. 1). The active sites of the actin filaments are covered in the resting state by two regulatory proteins, tropomyosin and troponin. Intracellular Ca2 • is the most important determinant of myocardial contractility and relaxation. 1 Once contraction ensues, calcium (Ca2•) entry through L-type Ca2• channels triggers an exponential release of Ca2• from the SR through ryanodine receptors. 2 Calcium then binds troponin, leading to a conformational alteration of tropomyosin, exposing the actin active site, facilitating a "slid­ ing" interaction between the actin filaments and the myosin heads as well as the hydrolysis of ATP (adenosine triphos­ phate) , thus providing energy for contraction. 3 Following a cycle of excitation-contraction coupling, diastolic relaxation is initiated by cytosolic Ca2 • sequestration in the SR by the SR-Ca2 • ATPase (SERCA2a) pump (-75%) and exportation extracellularly by the Na•/Ca2 • exchanger (-25%) located on the sarcolemmal membrane.4•5 Abnormal cardiac SR function and Ca2 • signaling represent a characteristic of both systolic and diastolic Congestive Heart Failure (CHF) .6-S There is good evidence that changes in either expression or function of spe­ cific calcium-handling proteins lead to increased intracellular 48

diastolic Ca2 • levels, decreased intracellular Ca2• transients, delayed Ca 2• efflux, and depressed contractility.9, 1 o,u .2 6 For example, ryanodine receptors are upregulated and progres­ sively activated by phosphorylation, contributing to the SR Ca2 • leak observed in CHF Phospholamban is a regulatory protein that exerts an inhibitory effect on SERCA2a, limit­ ing its ability to remove cytosolic Ca2 • following contraction. In failing hearts, phospholamban expression is altered and SERCA activity is decreased, resulting in diastolic and systolic dysfunction.5•6• 12 • 1 3 , 1 4-- i 7 An understanding of the molecular mechanisms of heart failure is necessary to understand the potential for therapeutic targets. This fact is underscored by the recent clinical trial investigating whether the function of SERCA can be restored via gene transfer. 1 8 /3- A D R E N E R G I C S I G N A L I N G Three ,13-adrenergic receptor (,13-AR) subtypes have been characterized, /31 , ,132 , and {33 . Catecholamines act to increase myocardial contractility primarily through {31 -adrenergic receptor stimulation leading to G protein-mediated adenyl cyclase activation and cyclic adenosine 3'5' monophosphate (cAMP) generation, which triggers protein kinase A (PKA)­ dependent phosphorylation of voltage-gated L-type Ca• channels, ryanodine receptors, and phospholamban, which derepresses SERCA2a, leading to excitation-contraction coupling and positive inotropy (Fig. 6.2). 1 9-21 ,132 -Adrenergic receptors as well as muscarinic cholinergic receptors, through an inhibitory G protein, provide negative control of adr­ energic stimulation by inactivating adenyl cyclase, thereby limiting the generation of cAMP 22 The role of {33 -adrenergic receptors is poorly defined, but there is some evidence that ,133 -adrenergic receptors maintain coronary vasomotor tone through the nitric oxide (NO) pathway. 2 3 /3-Arrestins also serve to restrict cAMP generation by increasing cAMP deg­ radation and desensitizing the /3-receptor. 24 Derangements in chronic ,13-adrenergic signaling that have been implicated in the pathogenesis of CHF include /3-AR downregulation, ,13-AR

CHAPTER 6 • CARD IAC BIO CHEMISTRY

\/

Tropon i n Tropomyosin

\

Z-line ?ef?

:r:: ��....__ J

Z-line

____

::::c�� 1

Myosin (thick) filament



Act

FIGURE 6.1 Sa rcomere a natomy.

membrane. 3 1•33 The Na+/K+ ATPase works constitutively, using energy from the hydrolysis of ATP to maintain a high intracellular K+ concentration and a high extracellular Na+ concentration. 34 Ca1+ is removed from the cytosol into the extracellular fluid by a sodium-calcium exchange (NCXl) pump driven by the preexisting Na+ gradient.35 Inhibiting the Na+/K+ ATPase promotes enhanced Na+/Ca1+ exchange, leading ultimately to increased intracellular Ca1+ being available to the contractile apparatus, potentially leading to increased myocardial contractility.

(th;o) filameot

uncoupling from second messenger systems, upregulation of /3-adrenoreceptor kinase (/3ARK1), and altered calcium trafficking.15-3o f3-Receptor blockade can restore calcium homeostasis and upregulate SERCA2a, ultimately improving cardiac performance with long-term treatment. 3 1 D I G I TA L I S A N D T H E N A + - K+ AT PA S E Digitalis, a cardiac glycoside derived from the foxglove plant, has been used for centuries to treat heart failure and atrial fibrillation. Digitalis functions by inhibiting the sodium pump (Na+fK+ ATPase) found in the cardiac cell

P H O S P H O D I ESTE RAS E I N H I B I T I O N Phosphodiesterase inhibitors (PDis) such as milrinone affect contractility by inhibiting phosphodiesterase 3 (PDE3), increasing intracellular cAMP and Ca1+ , which leads to increased inotropy. 3 6 PDis also have vasodilating properties that are important in unloading the failing ventricle.37 Unfor­ tunately, the gain in cardiac performance is tempered by increased arrhythmogenesis, myocardial oxygen consump­ tion, and cardiac death, mitigating its usefulness beyond being a bridge to cardiac transplantation in end-stage CHF 38 •3 9 R E N I N - A N G I OT E N S I N SYST E M The renin-angiotensin system (RAS) has a detrimental role in the pathogenesis of heart failure. Beyond its influences

131 receptor

Muscarinic Acetylcholine receptor /

13 arrestin

L-Type Ca2+ channel

--

I

0

iNOS

SE RCA2a

t

I

eNOS

C>



NO

cG M P

� Positive regulation ....... Negative regu lation

Legend • • •

49

Sarcomere

FIGURE 6.2 {3-Ad renergic and NO reg u lation of the cardiac myocyte.

'-.... L-Type Ca2+ channel

50

I

SECTI O N I • FUNDAMENTALS

tS

Angiotensinogen

I Angiotensin I

�/I lt \@>1 \:1' I

An g ioten s in

JI'

,

Increased contractility

Water retention (vasopressin) . Sodium

Cardiac retention h ype rt rop h Y (aldosterone) Catecholamine release

Vasoconstriction

FIGURE 6.3 Ren in-ang iotensin system .

on blood pressure and salt and water regulation, it has stimulatory effects on the sympathetic nervous system, direct effects on myocardial hypertrophy, and indirect effects on myocardial contractility Numerous large randomized clinical trials have demonstrated the symptom relief and survival ben­ efit in patients with CHF treated with angiotensin-converting enzyme (ACE) inhibitors.40-42 Angiotensinogen is cleaved to angiotensin I by the renally produced enzyme renin in response to renal hypoperfusion. Angiotensin I is then cleaved by ACE into the potent vasoconstrictor angiotensin II. Angiotensin II stimulates catecholamine release, increases cardiac hypertrophy, regulates blood pressure (angiotensin II receptors) , and increases blood volume by stimulating aldos­ terone and vasopressin release, enhancing sodium and water retention (Fig. 6.3).43 ACE inhibitors also increase the gener­ ation of bradykinin (thought to mediate the cough associated with ACE inhibitors) , which is a nitric oxide synthase (NOS) agonist and may attenuate /3-adrenergic contractility through NO signaling.44 Bradykinin degradation may also have unto­ ward effects on myocardial contractility that are offset by ACE inhibition.45 ACE-2 is an ACE isoform that is thought to be an impor­ tant regulator of cardiac contractility It catalyzes the cleav­ age of angiotensin I to angiotensin 1-9 and of angiotensin II to angiotensin 1-7. ACE-2 is not inhibited by ACE inhibi­ tors, nor is bradykinin a by-product of its activity46-48 ACE-2 is upregulated within the myocardium with angiotensin II receptor blockade.49 ACE-2 deficiency diminishes car­ diac contractility and upregulates hypoxia-induced genes, suggesting its role in RAS modulation following ischemia­ mediated cardiac injury 50 N I T R I C OX I D E NO plays an important role in the endothelium-dependent functions of coronary vasomotor tone and thrombogenesis, but it also has direct effects on myocardial relaxation. NO is

generated by the enzyme NOS, which has three isoforms: eNOS (endothelial) , iNOS (inducible) , and nNOS (neu­ ronal) . NO affects myocardial relaxation through effects on excitation-contraction coupling, regulation of adrenergic signaling, and mitochondrial metabolism. 5 1 Attenuation of /3-adrenergic stimulation by NO (see Fig. 6.2) is mediated by cyclic guanosine 3'5'-monophosphate (cGMP)-dependent phosphodiesterase Ell regulation of cAMP levels, protein kinase G-mediated downregulation of L-type Ca• chan­ nels,52 ·53 and the desensitization of troponin I to calcium. 54 NO may also influence myocardial relaxation by enhancing the activity of the delayed rectifier K• current55 as well as cGMP-mediated inhibition of phospholamban and its nega­ tive control over SERCA2a.56 REFERENCES 1 . Morgan JP Abnormal intracellular modulation of calcium as a maj or cause of cardiac contractile dysfunction. N Engl ] Med. 1 9 9 1 ;325 625. 2 . Lehnart SE, Wehrens XH, Kushnir A, et al. Cardiac ryanodine receptor function and regulation in heart disease. Ann N Y Acad Sci. 2004; 1 0 1 5 : 144- 1 5 9 . 3 . Guyton A C . Textbook of Medical Physiology. 8th e d . Philadel­ phia: WB Saunders; 1 99 1 : 68-72. 4. Schulze DH, Muqhal M, Lederer WJ , et al. Sodium/calcium exchanger (NCXl) macromolecular complex. ] Biol Chem. 2003;2 78(3 1 ) : 28849-28855 . 5 . Arai M , Matsui H , Periasamy M. Sarcoplasmic reticulum gene expression in cardiac hypertrophy and heart failure. Circ Res. 1 994;74:555-564. 6. Schmidt U, Hajj ar RJ , Helm PA, et al. Contribution of abnormal sarcoplasmic reticulum ATPase activity to systolic and dia­ stolic dysfunction in human heart failure. ] Mal Cell Cardiol. 1 998;30: 1 929- 1 93 7. 7. Lehnart SE, Maier LS, Hasenfuss G. Abnormalities of calcium metabolism and myocardial contractility depression in the fail­ ing heart. Heart Fail Rev. 2009; 14(4) : 2 1 3-224. 8. Gwathmey JK, Copelas L, Mackinnon R, et al. Abnormal intracellular calcium handling in myocardium from patients with end-stage heart failure. Circ Res. 1987;6 1 : 70-76. 9 . Whitmer JT, Kumar P, Solaro RJ . Calcium transport properties of cardiac sarcoplasmic reticulum from cardiomyopathic Syr­ ian hamsters (BIO 53.58 and 14.6): evidence for a quantitative defect in dilated myopathic hearts not evident in hypertrophic hearts. Circ Res . 1 988;62 :8 1-85 . 1 0 . Gwathmey JK, Slawsky MT, Hajjar RJ , et al. Role of intracel­ lular calcium handling in force-interval relationships of human ventricular myocardium. ] Clin Invest. 1990;85 : 1 599-1 6 1 3 . 1 1 . Hasenfuss G, Mulieri LA , Leavitt BJ , e t al. Alterations o f con­ tractile function and excitation contraction coupling in dilated cardiomyopathy. Circ Res. 1 992;70 : 1 225-1232. 1 2 . Schmidt AG, Zhai J , Carr AN , et al. Structural and functional implications of the phospholamban hinge domain: impaired SR Ca2 • uptake as a primary cause of heart failure. Cardiovasc Res. 2002;56(2):248-259. 1 3 . Mercadier JJ , Lompre AM, Due P, et al. Altered sarcoplasmic reticulum Ca2 <

Identify Cases and Controls

Exp°'"" pee""'

Cases

Controls

Exposure absent FIGURE 7.1 Case-control study d iagram.

3. The cohort consists of individuals with and without the expo­ 4. 5.

sure variable present at the time of inception. The cohort is followed over time for the occurrence of a clearly and objectively defined outcome. The occurrence of the outcome is compared in individuals with and without the exposure variable.

In effect, a randomized controlled trial is a kind of cohort study, except that the exposure variable is determined by the investigator, not by nature , using a randomization technique. Pros pective-versus-Retrospective Stud ies

In prospective-versus-retrospective studies, prospective data are obtained and coded at the time they are first available and, in the case of cohort studies, prior to the outcome. Ret­ rospective data are obtained at a later time, often after the outcome has occurred. Prospective studies are much less likely to be subject to observation bias or problems with missing data. Meta-Ana lysis

Ca se-Control Study

A case-control study (Fig. 7. 1) is a somewhat weaker study design than a cohort study In a case-control study: 1 . A "case" group of subjects with a given outcome is identified. 2 . A "control" group of subjects, without the outcome, is identified. 3 . The occurrence of an exposure variable is compared between

the case group and the control group.

Stu d ies of Diag n ostic Tests

In studies designed to assess the value of a diagnostic test, a group of patients suspected of having a certain disease (out­ come) undergo the diagnostic test, and then the results of the diagnostic test are compared against an accepted standard. Cost- Effectiveness Stu dies

Cost-effectiveness studies usually take the form of a cohort study in which (a) the cost of an intervention is measured, (b) the outcomes of performing or not performing an intervention are compared, or (c) the cost of preventing an outcome is meas­ ured; typically; this last is recorded as dollars per year of life saved or dollars per quality-adjusted year of life saved (QALY) . Ra ndom ized Contro l l ed Tri a l s

Randomized controlled trials (Fig. 7.2) are the gold standard for assessing a treatment or a prevention measure. In these studies:

Meta-analyses are systematic reviews of specific clinical questions with data pooled from multiple previously com­ pletecl!published studies. Steps in a meta-analysis include: 1.

Clinical hypothesis is defined.

2. Literature is searched. 3 . Studies are selected based on prespecified criteria.

4. 5.

Consistent summary measures are collected from each identified study. Pooled analysis is performed.

Most meta-analyses only pool randomized clinical trials, but meta-analyses can include cohort studies alone or mixed with clinical trials. Meta-analyses may suffer from publica­ tion bias as negative studies have traditionally been more difficult to publish. Genome-Wide Association Studies

Genome-wide association studies (GWAS) are a contem­ porary form of case-control studies that focus on genetic data. DNA from people with (cases) and without (controls) a disease is collected and placed on gene chips. These chips are read into computers that identify DNA variations between the two groups. DNA variations that are more fre­ quently detected are "associated" with the disease and hint at chromosomal regions that may be responsible for the disease.

1 . A cohort of patients at risk for an outcome is defined. 2 . Determination of which patients receive treatment or preven­

tion is made entirely at random. 3. An outcome is measured after a predetermined follow-up period.

Statistical Tests Statistics is the science by which observations made of a sample are assessed with respect to their likely validity in the entire universe.

Inception Cohort

Type I a n d Type I I Erro rs Treatment A

Treatment B

><

FIGURE 7.2 Random ized trial diagra m .

Outcome

No outcome

Commonly reported statistics include two types of statisti­ cal error analysis. In type I errors, an association between an exposure and an outcome is in fact a spurious one that has resulted from random chance. The "p value" refers to the likelihood that an observed association is due to chance alone. In type II errors, on the other hand, the lack of

CHAPTER 7 • CLIN I CAL EPIDEMIOLOGY AND BIOSTATISTICS a n observed association between a n exposure and a n out­ come is in fact due to chance because the sample size was not large enough to detect an association if one in fact exists. This is one of the most common errors reported in clinical literature. Hypothesis Testi ng

Statistical tests also aim to determine whether a "null hypothesis" should be rejected, where the null hypothesis is that no asso­ ciation exists between the exposure and the outcome. Today many clinical researchers are moving away from this sort of hypothesis testing and more toward estimation of effects along with confidence intervals, discussed below

57

occurrences are suspect. Studies with > 1 00 outcomes may be compelling. Absol ute Event Rates

Absolute event rates are generally considered the most honest way to present data. How many outcomes were associated with exposures? How many outcomes occurred among those not exposed? Be suspicious if raw data are not provided. A careful reading of the raw data will enables a reader to distinguish between "statistical significance" and "clinical significance. " It is the latter that we really care about. Relative Risk or Risk Ratio

Co m pa risons of Conti n u ous Va ria bles

Continuous variables are variables that can have an infinite

number of values, such as age, height, blood pressure, or cholesterol level. They are described using means, standard deviations, ranges, quartiles, quintiles, deciles, and so on. When continuous variables are normally distributed (i. e . , described by a Gaussian or bell-shaped curve), t tests are generally used to compare the means of two groups and AN OVA is used to compare means of three or more groups. When the continuous variables cannot be assumed to be normally distributed, nonparametric testing, such as the Wilcoxon rank-sum, which compares median values and distributions of two groups, or the Kruskal-Wallis test, which compares medians and distributions of three or more groups, is often used. To compare the strength of a presumed linear asso­ ciation between two continuous variables (e .g. , left ven­ tricle mass versus blood pressure) , researchers often use tests of correlation (r value) , such as Pearson or Spearman tests. In these tests , the square of the r value describes how much the variability of one variable can be attributed to the other.

Relative risk or risk ratio (RR) is the proportion of event rates according to exposure , or RR

OE /NE 00 /N0

where OE is the number of patients with exposure who had the outcome . NE is the number of patients with expo­ sure, 00 is the number of patients without exposure who had the outcome and N0 is the number of patients without exposure. A risk ratio of 1 .0 implies no association; a value > l implies an increased risk, and a value < l implies a protective effect. =

Relative Risk Red uction

Relative risk reduction (RRR) is defined as the proportional reduction in rates, or RRR

=

Absol ute Risk Red uction Co m pa risons of Catego rica l Va ria bles

Variables that can only have a finite set of values (e.g. , gen­ der, presence or absence hypertension, use of a certain med­ ication) are called categorical variables . For most samples, these kinds of variables are compared using the chi-square test. However, if the sample size is very small, researchers may instead use the Fischer exact test.

Data Presentation and Reporting of Outcomes The statistical tests discussed above tell only part of the story The strengths of associations can be described in a number of ways.

Absolute risk reduction (ARR) is the difference between absolute event rates, a more honest way of presenting data, or ARR

Oo OE No N E

=

N u m be r N eeded to Treat

Number needed to treat (NNT) is the number of patients who would need to be exposed in order to prevent one out­ come, or NNT

=

1 ARR

Confidence I nterva l N u m ber of O utco mes

Knowing the number of outcomes is essential to determining the strength of a study In general, studies with 1 . 5 (acyanotic) Atrial septal defect (ASD) Ventricular septal defect (VSD) Patent ductus arteriosus Partial anomalous pulmonary venous return

2 . Increased (overcirculation) • • • •



3. Decreased (undercirculation) •

• •





Right-to-left shunt Tetralogy of Fallot Ebstein anomaly with ASD Pulmonary oligemia Ebstein anomaly (severe ± ASD)

65

4. Admixture shunt (no pulmonic stenosis or atresia/cyanotic) •



• •



Transposition complexes Truncus arteriosis Univentricular heart (single ventricle) Total anomalous pulmonary venous return: types I and II Tricuspid atresia + VSD

5. High-output states •



Anemia Pregnancy

Decreased P u l m o n a ry B l ood Flow (Fig. 8. 1 )

B FIGURE 8.1 Decreased pul monary blood flow. A: Tetra logy of Fa l l ot. B: Ebstein anoma ly.

I n c reased P u l monary B l ood Flow (Fig. 8.2)

FIGURE 8.2 I ncreased pul monary blood flow. A: I n c reased: bala nced (overci rcu lation). B: ASD (overci rcu lation).

66

SE CTI ON II • CARD IOVASCULAR IMAGIN G AND STRESS TESTING

I n creased: Red istri b uted (Fig. 8.3)

FIGURE 8.3 I ncreased : red istributed. A: Pulmonary venous hypertension (PVH). B: Hypertrophic cardiomyopathy (HCM) PVH .

Pulmonary Arterial Hypertension (PAH)

Causes of PAH

• Precapillary: pulmonary hypertension (both primary and pul­ monary hypertension associated with hepatic disease, drugs/ toxins, HIV) , congenital cardiovascular disease , chronic

thromboembolism, chronic alveolar hypoxia (COPD , interstitial lung disease, hypoventilation) • Postcapillary: left sided cardiovascular disease, mitral stenosis, aor­ tic valve disease, cardiac tumors, extrinsic pulmonary venous com­ pression, fibrosing mediastinitis, pulmonary venoocclusive disease

I ncreased: Centra l (Fig. 8.4)

A FIGURE 8.4 A: Id iopathic (pre-ca p i l l a ry) PAH; B: Post-ca p i l l a ry PAH .

B

CHAPTER 8 • CHEST RADI O GRAPHY FOR THE CARDIOVASCULAR MEDICINE B OARD S

Pulmonary Venous Hypertension

Stages of PVH

Stage

1:

Mitra! valve disease Mitra! stenosis Mitra! regurgitation

Pulmonary vascular redistribution

PCWP: Acute, 13 to 18 mm Hg; chronic, 18 to 22 mm Hg

Stage 2 : Redistribution + interstitial pulmonary edema

PCWP: Acute, 18 to 25 mm Hg; chronic, 23 to 30 mm Hg

Stage 3 : Redistribution + interstitial and alveolar pulmonary edema PCWP: Acute, >25 mm Hg; chronic, >30 mm Hg

Ca uses of PVH

Pulmonary venoocclusive disease Pulmonary vein stenosis (postradiation therapy, post-atrial fibril­ lation ablation) Left atriaVleft ventricular obstruction

Left ventricular compromise Dilated cardiomyopathy Acute or chronic myocardial ischemic disease Restrictive cardiomyopathy Pericardia! disease Constrictive pericarditis

Pulmonary Edema Patterns Increased hydrostatic pressure gradient Increased capillary permeability Decreased osmotic pressure gradient Lymphatic incompetence

Myxoma or other tumors

TA B L E Radiographic Featu res of Different Types of Pulmonary Edema

Cardiac silhouette Pulmonary blood flow Pulmonary blood volume Peribronchial cuffing Air bronchograms Lung edema Pleural effusions

Ca rdiac I nj u ry

Fluid Overload

I nj u ry

Often enlarged Redistributed Normal or Increased

Normal or enlarged Balanced Increased

Not enlarged Normal Normal

Very common Not common Even Very common

Not common Not common Central Very common

Not common Very common Peripheral Not common

C A R D I O M E D I A ST I N A L S I L H O U E T T E PAT T E R N S Normal landmarks • Central Tracheobronchial tree-PA relationship Early branching to middle lobe on right Right PA descends anterior to bronchus Left PA passes over and descends posterior to bronchus • Right Right chambers Ascending aorta Superior vena cava/azygos vein

• Left Aortic "knob" (distal arch + isthmus) Descending aorta Main PA Descending aorta Left atrium Left ventricle

67

68

SE CTI ON II • CARD IOVASCULAR IMAGIN G AND STRESS TESTING

Normal Cardiac Silhouette (Fig. 8.5)







·,)J' ••

®



�1]



7, descending aorta; 2, aortic knob (dista l a rch a n d isth m u s); 3, ascending aorta a n d proximal a rch; 4 , r i g h t ventricle; 5 , l e ft ventricle; 6 , left atri um; 7, r i g h t atri um; 8 , s u perior vena cava and azygous vei n .

FIGURE 8.S

Normal Size The most commonly used parameter for assessment of car­ diac size is the cardiothoracic ratio, which corresponds to the maximum transverse diameter of the cardiac silhouette in relation to thoracic width. The accepted upper limit of normal is 50% in adults. It should be emphasized that evaluation of individual cardiac chambers with CR is not reliable, with the occasional exception of left atrial dilation, which can be associated with a double density projecting over the right heart border as well as splayed central bronchi. Interval changes in the size of the cardiac silhouette are of clinical interest, although this is always subject to vari­ ability in technique between CRs.

TA B L E

Enlargement of Cardiovascu lar Structure: Basic Causes Cause

Examples

Decreased integrity of wall

Post-myocardial infarction (MI) left ventricular true aneurysm Dilated left atrium in mitral regurgitation; dilated left ventricle in aortic insufficiency Atria dilate (dilated left atrium in mitral stenosis) ; ventricular hypertrophy (thick left ventricle in hypertension)

Volume overloading

Pressure overloading (differential response)

CHAPTER 8 • CHEST RADI O GRAPHY FOR THE CARDIOVASCULAR MEDICINE B OARD S

Pectus Excavatum (Fig. 8.6)

FIGURE 8.6

Pectus excavatu m. 2. Heart usually displaced to the left; right heart border not visible

Fi n d i n g s 1 . Pectus excavatum configuration of ribs

Straight or upsloping posterior ribs Sharply downsloping anterior ribs

Pericard ia I Cyst (Fig. 8.7)

FIGURE 8.7

69

Perica rd ia! cyst.

Fi n d i n g s 1 . The arrows mark the pericardia! cyst

70

SE CTI ON II • CARD IOVASCULAR IMAGIN G AND STRESS TESTING

Aortic Stenosis (Fig. 8.8)

FIGURE 8.8

Aortic stenosis. 2. Dilated ascending aorta beyond stenotic AV (short arrow)

Fi n d i n g s 1 . Calcified AV (long arrow)

Pseudocoarctation (Fig. 8.9)

FIGURE 8.9

3. Normal cardiac silhouette (because only pressure and not

volume overload present)

Pseudocoa rctation.

Fi n d i n g s 1 . "Double left aortic arch sign" (arrows mark the " 2 " arches)

2. Congenital elongation of the thoracic aorta associated with

"kinking" of the aorta at the relatively fixed ligamentum arterio­ sum. There is no physiologic obstruction, so there is an absence of collateral vessel development and subsequent rib notching.

CHAPTER 8 • CHEST RADI O GRAPHY FOR THE CARDIOVASCULAR MEDICINE B OARD S

Coarctation (Fig. 8.1 O)

FIGURE 8.1 0

Coa rctation. 2. Rib notching (long arrows) on the inferior portion of the poste­

Fi n d i n g s 1 . Number " 3 " sign (short arrows)

Valvar Pulmonic Stenosis (Fig. 8.1 1 )

FIGURE 8.1 1

71

rior ribs (third to ninth) from pressure erosion by dilated inter­ costal arteries that serve as collateral blood flow between the internal mammary arteries and the descending aorta

Va lva r pulmonic stenosis.

Fi n d i n g s 1 . Enlarged main PA, the degree o f this enlargement does not pre­

dict the severity of stenosis

2. Selective enlargement of the left PA (long arrow) with normal­

sized right PA (short arrow) 3. Normal cardiac silhouette and normal to decreased pulmonary

vascular markings, depending on the degree of stenosis

72

SE CTI ON II • CARD IOVASCULAR IMAGIN G A N D STRESS TESTING

M itral Stenosis (Fig. 8.1 2)

FIGURE 8.1 2

Mitra l stenosis. 2. Increased left atrial size (short arrows)

Fi n d i n g s 1.

Increased pulmonary vascularity (long arrow)

3. Normal-sized left ventricle

M itral Regurgitation (Fig. 8.1 3)

FIGURE 8.1 3

Mitra l reg u rg itation.

Fi n d i n g s 1.

Increased pulmonary vascularity

2. Increased left atrial size (short arrows)

3. Increased left ventricle size secondary to volume overload (long

arrow)

CHAPTER 8 • CHEST RADI O GRAPHY FOR THE CARDIOVASCULAR MEDICINE B OARD S C A LC I F I C AT I O N PATT E R N S • Cardiac

73

Aortic valve Coronary artery Myocardium (post-MI) • Paracardiac

Left atrium Mitra! annulus Mitra! valve

Calcific Constrictive Pericarditis (Fig. 8.1 4)

Pericardium Thoracic aorta

t FIGURE 8.1 4

Calcific constrictive pericarditis. 2. Calcifications along atrial surfaces suggest pericardium rather

Fi n d i n g s

Left Atrial Wal l Calcification (Fig. 8.1 5) 1 . Extensive pericardia! calcification (arrows)

FIGURE 8.1 5

than myocardium origin.

Left atrial wa l l calcification.

Fi n d i n g s

2. Bjork-Shiley valve in the mitral position (long arrow)

1 . Severe diffuse calcification of the left atrium (short arrow)

3. Single right ventricular (RV)-lead pacemaker (black arrows)

74

SE CTI ON II • CARD IOVASCULAR IMAGIN G AND STRESS TESTING

M itral Annular Ca lcification (Fig. 8.1 6)

FIGURE 8.1 6

Mitra l a n n u l a r calcification.

Fi n d i n g s 1.

Extensive mitral annulus calcification, seen best o n the lateral radiograph as a large backward "C'

Calcified Post-M l Left Ventricular True Aneu rsym (Fig. 8.1 7)

FIGURE 8.1 7

Calcified post-Ml left ventricu l a r true aneursym.

Fi n d i n g s 1.

Calcified left ventricular aneurysm (arrows)

CHAPTER 8 • CHEST RADI O GRAPHY FOR THE CARDIOVASCULAR MEDICINE B OARD S

Coronary Artery Calcification (Fig. 8.1 8)

FIGURE 8.1 8

75

Coronary a rtery calcification.

Fi n d i n g s 1 . Extensive coronary calcification o f the left anterior descending

artery (arrow) , best seen on the lateral radiograph.

AC K N OW L E D G M E N TS The authors acknowledge Drs. Ross Downey, Richard White, and Richard Krasuski for their contributions to the first edi­ tion of this chapter.

76

SE CTI ON II • CARD IOVASCULAR IMAGIN G AND STRESS TESTING

QU ESTIONS AND ANSWERS Questions

1 . Ms. G. is a 28-year-old woman who i m m i g rated to the U n ited States from South America approxi mate­ ly 4 years ago. She has been told in the past that she has a heart murmur and recently noted the on set of exertional dyspnea. Her electroca rdiogra m (ECG) is notable for atrial fibril lation, and exa m ination reveals a d iastolic m u rm u r best heard at the a pex. Her chest rad iograph is displayed below. Her atrial fibrillation is most l i kely secondary to:

a. b. c. d. e.

a. b. c. d. e.

Atrial myxoma Increased transmitral g radient Id iopathic "pri mary" pul monary hypertension "Lone" atrial fibril lation Adva nced left ventricular dysfu nction

2. Mr. S. is a 38-year-old man with no prior medica l h istory. Recently he has noticed a prog ressive red uction in his exercise tolerance. On exa m i nation he has a soft systolic m u r m u r at the u pper sternal border and a split second heart sound that does not appear to change with respi ration. His chest radiograph is displayed below. The most l i kely explanation for these findi ngs is:

Pa roxysmal atria l fibril lation U ndiagnosed pulmonary stenosis U ndiagnosed ASD U ndiagnosed pri m a ry pul monary hypertension Left ventricular dysfu nction with m itra l reg u rg itation

3. Ms. B. is a 74-year-old woman who i m m ig rated to the U n ited States from Southwest Asia 6 yea rs ago. For the last 2 yea rs she has noted prog ressive fatigue and lower-extremity edema. Her chest radiog raph is displayed below. A rig ht heart catheterization i n this patient would be expected to show a l l of the fol lowi ng characteristics except:

. . . . . . . a. b. c. d. e.

C:: l{J\P ! � R 8



C:: l{ EST RA]) l () GRAPHY FOR

Elevated pul monary capillary wedge pressu re Diasto l ic eq ualization of pressu res Elevated RV pressu re A significant step-u p in oxygen satu rations Normal to decreased card iac output

4. M r. N . is a 32-year-old man with recently diag nosed hypertension that has been refractory to medica l thera py. His exa m is notable for a loud systolic murmur and weak peripheral pulses. His chest radiog raph is shown below. The most appropriate surg ical i ntervention for this patient is:

THE CARDIOVASCULAR MEDICINE B OARD S

77

elevated atrial pressure resulting i n stretching of the atrial myoca rd i u m . 2. Answer C : U s i n g t h e organized approach t o radio­ g raph interpretation, the pulmonary vascu lar pattern shows ba lanced overcirculation, and the pul monary arteries a re q u ite prominent. The right ventricle appears mildly enlarged. No significant calcification is pres­ ent.The case vignette is notable for the physical exa m, which is consistent with a n ASD. Although this patient is certainly at increased risk for atrial fi bri l lation because of his congenital lesion, it is not the primary explanation for the findings. I n pul monic stenosis an oligemic pul monary blood flow pattern is normal ly seen, and in left ventricular dysfunction with m itral reg u rgitation one expects to see a large left ventricle with pulmonary vascular redistribution suggestive of pul monary venous hypertension. Primary pulmonary hypertension does not account for the physical findings i n this case; additional ly, a loud pul monic closure sound would usua l ly be present. 3 . Answer D: Using the organized approach to rad io­

a. b. c. d. e.

Resection and end-to-end a nastomosis Fonta n proced u re Glenn shunt Mitra I va lve repa ir AV replacement

Answers 1 . Answer B: Using the organ ized approach to rad io­ g raph i nterpretation, the pulmonary vascular pattern shows evidence of vascular red istribution suggestive of pul monary venous hypertension. The left atri u m is enlarged and the left ventricle is norma l i n size, sug­ gesting possible m itra l valve pathology. No cha m ber calcification is present to assist i n the diagnosis. The case vignette descri bes a classical presentation for m itra l stenosis. Though persistent inflam mation may contri bute to atrial fibril lation i n this d isorder, the primary mechanism i n m itral stenosis is sti l l felt to be

g raph i nterpretation, the pul monary vascular pattern shows redistri bution and pul monary venous engorge­ ment suggestive of elevated left heart fi l l i n g pressure. There is chamber enlargement of both ventricles. The most nota ble featu re of this radiograph, however, is the perica rdia I calcification, which appears circu mferential. The vig nette descri bes a case of constrictive perica r­ d itis, possibly from old tu berculous infection. I n this condition, one expects elevated fi l l ing pressure and a promi nent Kussmaul sign (fa i l u re of the j u g u l a r venous pressure to d rop with inspiration). A perica rdia! knock is often present as wel l . A step-u p i n oxygen satu ra­ tions (suggestive of an i ntraca rd iac shu nt) wou l d not be expected in this patient. 4. Answer A: Using the organized approach to rad io­

g raph i nterpretation, the pul monary vascular pattern does not appear to be particularly prominent. The cham bers of the heart a lso do not appear to be abnor­ mal i n size. There is, however, sign ifica nt rib notching. This is due to the promi nent col l atera ls that develop to bypass the circulation and a l low adeq uate blood flow to reach the peri phery. The presence of these collater­ als com bi ned with the case history suggests the aortic coa rctation. The classical surg ical correction of this a bnormality is resection and end-to-end anastomosis of the aorta. Recently, percutaneous techniq ues have become more popular, not only for post­ operative recu rrence of coa rctation, but a lso as primary thera py.

F undainentals of Doppler Echocardio graphy Andrew C .Y. To and L. Leonardo Rodriguez

B A S I C P R I N C I P L E S OF U LT R A S O U N D Sound waves are mechanical vibrations produced by a source that are transmitted through a medium such as air. As sound waves travel through a medium, the particles of the medium are packed (compression) , alternating with being spaced apart (rarefaction) . Sound waves can be represented graphi­ cally as sine waves (Fig. 9 . 1 ) . The wavelength (A,) is the dis­ tance between two similar areas along the wave path and is measured in millimeters. The frequency (f) is the number of wavelengths per unit time. Frequency is expressed in hertz (Hz) , which is equivalent to cycles per second. Hence, the velocity of sound in a medium (c) is the product of wave­ length and frequency; and wavelength and frequency are inversely related. c = Af

The amplitude of the sound wave (loudness) is measured in decibels (dB) , which is in logarithmic scale. The propagation velocity of sound is determined by the stiffness of the medium and is also related inversely to its density. In human tissue, sound wave propagation velocity is 1 ,540 mis (Table 9 . 1 ) . Humans hear sound waves with frequencies between 20 Hz and 20 kHz; hence, ultrasound is defined as sound with frequencies higher than 20 kHz. Diag­ nostic medical ultrasound uses transducers with frequencies between 1 and 20 MHz. I N T E RACT I O N O F U LT R A S O U N D WITH TISS U E Ultrasound beam travels in a straight line in a homogene­ ous medium; however, when the beam travels through a medium with two or more interfaces or in a heterogeneous medium, the path is altered. The interaction of ultrasound 78

with tissue can be in the form of reflection, scattering, refrac­ tion, or attenuation. When the ultrasound beam encounters a bound­ ary between two different media, part of the ultrasound is reflected back toward the transducer and another part con­ tinues into the second medium. The amount of reflection depends on the difference in acoustic impedance between the two media. The amount of ultrasound reflected back is constant, but the amount received back at the transducer varies with the angle of the ultrasound beam to the tissue interface. Because the angles of incidence and reflection are equal, optimal return of the reflected ray occurs when the beam is perpendicular (90 degrees) to the tissue interface. Scattering occurs when the ultrasound beam strikes smaller structures, less than one wavelength in the lateral dimension. This results in the ultrasound beam being radi­ ated in all directions, with a minimal amount returning to the transducer. Scattering of ultrasound produced from moving red blood cells is the principle behind Doppler echocardiography. When the speed of sound differs in the two media, the acoustic impedance is different, and the ultrasound waves in the second medium are deflected from their original orienta­ tion. This is known as refraction. Because blood and most tissues have similar sound velocities, this is not a prominent effect in echocardiography. When ultrasound travels through a biologic medium, part of the energy is absorbed and converted into heat. This process whereby ultrasound signal strength reduces is called attenuation. The degree of attenuation depends on the ultrasound frequency and on the differences in acoustic impedances between the two media. Lower ultra­ sound frequencies have a lower attenuation and penetrate deeper into tissues. Air has high acoustic impedance, which causes significant attenuation if there is any air between the

CHAPTER 9 • FUNDAMENTALS O F D O PPLER E C H O CARDIOGRAPHY

�I

1

Velocity = 1,540 mis

-----

Round-trip distance = 40 cm

Amplitude

11



Echo

Time fer echo to return

=

0 .26 ms

FIGURE 9.2 The time for the u ltrasound beam to return to the transducer from a particular structure is a measure of the struc­ tu re's dista nce from the transducer.

l

FIGURE 9.1

2o cm

Ultrasound Pulse

One Cycle

79

Wavelength

Fig u re of sound wave.

transducer and the body tissue. Applying water-soluble gel on the transducer minimizes contact with air and hence attenuation. TRA N S D U C E RS The ultrasound transducer is the small hand-held probe that transmits acoustic energy and receives the returning echoes. Piezoelectric crystal converts electrical energy into sound energy and vice versa. Piezoelectric elements lack a center of symmetry and are anisotropic. When an electric current is applied, the polarized particles within the crys­ tal are aligned, causing the crystal to expand and produce a mechanical effect. This is known as the direct piezoelectric effect. An alternating current causes the crystal to compress and expand alternately, which produces an ultrasound wave by compressions and rarefactions. Piezoelectric crystals gen-

Velocity of Sound through Different Media Material

Air Fat Water Human soft tissue Brain Liver Kidney Blood Muscle Lens of eye Bone

Velocity of Sound (mis)

330 1 ,450 1 ,480 1 ,540 1 ,540 1 ,550 1 ,560 1 ,5 70 1 ,580 1 ,620 4,080

erate an electric current when their shapes are altered while being struck by ultrasound waves. Therefore, the transducer functions both as a transmitter, transmitting a burst of ultra­ sound, and as a receiver, receiving the ultrasound signals reflected by internal tissue interfaces. A typical pulse lasts for only 1 to 6 µs. The transducer frequency is determined by the nature and thickness of the piezoelectric element. Image formation is based on the time interval between the ultrasound transmission and the arrival of its reflected signal. Deeper structures have longer flight times (Fig. 9 . 2) . The time delay between transmission and reception i s deter­ mined by the depth (d) of a certain structure and the speed of sound in blood: d = ct / 2

The factor 2 appears because t includes the time to and from the object. Knowing that the speed of sound in blood is 1 ,540 mis, d 77 t cm ( t in ms ) =

Resolution of the imaging system is defined as the smallest

distance between two points that can be distinguished by the system as separate entities. Axial resolution refers to the ability to differentiate between points lying along the path or axis of the ultrasound beam. Lateral resolution refers to the ability to differentiate between points that are lateral to the beam, relative to the beam. Axial resolu­ tion is related to the ultrasound's wavelength, frequency, and the duration of the transmitted pulse. Lateral resolu­ tion is dependent on the distance of the specular reflector to the transducer and is a function of the beam width, which is defined as the diameter of the beam at a par­ ticular point . In the near field, the beam is maintained as a cylinder with a diameter comparable to the transducer. However, at points farther away from the transducer, the beam diverges and widens into a cone. This area is the far field. Beam width is a function of transducer size, shape, frequency, and focusing. The larger the transducer, the longer the near field is . The lateral resolution is dependent on the gain of the system. Specular reflectors along the center of the beam produce stronger echoes than those that are at the beam margins. When the gain or sensitivity is set low, echoes from

80

SE CTI ON II • CARD IOVASCULAR IMAGIN G AND STRESS TESTING

beam margins with lower amplitude may not be recorded, which makes the beam appear narrower. With higher gain, the echoes at the margins are recorded, and the beam width appears greater.

I M AG I N G M O D A L I T I E S There are several imaging modalities in echocardiography A and B modes have only historical importance. M mode (motion) displays axial information along a single scan line, displaying depth on the vertical axis and time on the horizontal axis. This provides high temporal resolution and rapid sampling rates, with the ability to visualize wall or valve motion. M-mode measurements have been the stand­ ard in echocardiography in quantifying chamber size and endocardial thickening (Fig. 9 . 3) . Two-dimensional (2-D) echocardiographic imaging is generated by sweeping ultrasound beam through an arc across a particular area of the heart. Electronic sweeping is accomplished by the use of phased array transducers. Trans­ ducer arrays are groups of individual transducers or trans­ ducer elements. Linear arrays are a group of transducers or transducer elements lined up next to each other in a straight row. The transducers are then pulsed individually or in groups. This requires a large window, which limits their use in cardiac imaging. Phased arrays contain multiple element transducers that sweep the ultrasound beam electronically through an arc. Exciting the transducers in sequence gener­ ates an ultrasound wave that propagates at an angle to the transducer and sweeps the beam from side to side. A focused transducer is used to decrease diversion in the far field of the ultrasound beam. By placing a concave acoustic lens on the transducer surface or by altering the transducer curvature, the ultrasound beam is narrowed at a point away from the transducer. The focal zone is the area where the beam is narrowest and divergence is smallest.

FIGURE 9.3

Phased array transducers can also focus the beam electronically by altering the shape of the wavefront according to the tim­ ing of firing of the individual transducer elements. Two-dimensional echocardiography displays ultrasound data in a spatial orientation relative to time and localize depth by the reflected wave timing. This limits the amount of data that can be collected in a period of time and hence temporal resolution. The pulse duration (PD) is the time needed for the pulse to travel from the transducer to the tis­ sue and back. This is dependent on the depth of the tissue and the speed of sound in that tissue: PD = 2d ! c

The pulse repetition frequency (PRF) is the rate at which individual pulses are transmitted (per second) and is equal to c/2d. Since the speed of sound in human tissue is 1 ,540 mis, this translates to: PRF = 77 Id pulses I ms

The number of lines per sweep depends on the time taken to produce one scan line and the time set for each sweep. The frame rate is the number of images acquired per second. In cardiac applications, the frame rate is typically >30 frames per second. A higher frame rate is preferred to visualize myocar­ dial and valvular motion well. However, increasing the frame rate leads to fewer scan lines per frame, resulting in less data acquired per frame and therefore decreased image quality Echoes received by tissues produce vibrations within the piezoelectric crystal that translate into a small voltage. To form a final image, the electrical signal goes through com­ plex signal processing that initially is amplified by a radiof­ requency amplifier and compressed logarithmically in order to be displayed in varying shades of gray Serial processing occurs when one scan line is produced for each ultrasound pulse. This method limits the frame rate. With phased array transducers, it is possible to send out several scan lines simultaneously in different directions.

M-mode display.

CHAPTER 9 • FUNDAMENTALS O F D O PPLER E C H O CARDIOGRAPHY Through parallel processing, the data from each scan line are analyzed separately, which increases the frame rate. Dynamic range (expressed in decibels) refers to the amplitude ratio of largest signal displayed to the smallest signal detected above the system noise. Noise is a combina­ tion of all signals that reach the transducer from structures outside the ultrasound beam axis. These signal amplitudes are compressed into shades of gray, where the gray scale dis­ plays strong and weak echoes in various shades of gray The dynamic range consists of the number of levels of gray in an image and can be adjusted. Echo image data are obtained in a polar coordinate sys­ tem and are converted into a video image by means of a digital scan converter. Attenuation occurs when deeper structures produce weaker echoes than structures closer to the transducer. Elec­ trical energy produced by these echoes is therefore less. Time gain compensation applies greater amplification for echoes returning at longer intervals from the initial pulse, which cor­ responds to the depth of the structure. As attenuation varies in individuals, time gain compensation can be adjusted by the user. Near-field gain can be set lower while far-field gain can be gradually increased to achieve better image quality H A R M O N I C I M AG I N G When a sound pulse of frequency fo propagates through tis­ sues, nonlinear interactions occur, generating a pulse with frequencies at multiples of the fundamental frequency Jo: 2fo (second harmonic) , 3fo (Fig. 9.4A,B) . This is caused by minor distortions in the tissue, producing a very slight change in the shape of the wave as it propagates. The energy of these harmonic frequencies returning from the tissue is significantly less than the fundamental fre­ quency In order to benefit from harmonic imaging, the fun­ damental frequency needs to be filtered out, so that only the harmonic frequencies are passed to the demodulator. Harmonic generation increases with the distance of propagation and that there is a nonlinear relation between fundamental and harmonic frequency energies. These aspects of harmonic imaging help to understand why it is useful in reducing near-field artifacts (low harmonic energy close to the transducer) and side-lobe artifacts. Second harmonic tissue imaging is now used routinely in adult echocardiography The benefits can be striking par­ ticularly in patients with difficult images. In general, there is an improved signal-to-noise ratio with brighter tissue and superior endocardial definition. Of note, valvular structures appear thicker when imaged using second harmonics com­ pared to fundamental imaging. In some cases, it may be use­ ful to turn harmonic on and off when evaluating valve leaflets. Another use of harmonic imaging is contrast echocar­ diography New contrast agents have been manufactured with very small ( 1 - to 5-µm) bubbles capable of crossing the pulmonary capillary bed. The contrast effect is produced by microbubbles having different acoustic impedances than

81

Fu ndame nta l

f.o B

fo '2fo Original-pulse

Pul e after propagation

FIGURE 9.4 A: Tissue harmonic imaging uti l izes pulse frequen­ cies at m u ltiples of the fu ndamenta l frequency f0• B: Harmonic frequencies a re of lower energy than the fu ndamental frequency.

blood, causing the reflection and scattering of ultrasound. Ultrasound causes compressions and rarefactions of the microbubbles with a resonant frequency that is inversely related to the diameter of the microbubble. This ultrasound­ microbubble interaction also generates harmonic frequen­ cies. Compared to soft tissue, the microbubbles are strong reflectors, so tuning the ultrasound receiver to the second harmonic frequency displays the contrast agent preferen­ tially within the image. The approved indication for contrast imaging is left ven­ tricle opacification. In patients with limited views, contrast significantly improves endocardial definition. Its main appli­ cation is in suspected wall motion abnormalities at rest and after stress, or in suspected left ventricular (LV) thrombus. The use of contrast agents for myocardial perfusion, although promising, is still investigational. Myocardial perfusion and viability are potential uses of this technique, although they have not been approved for clinical use. TH R E E-D I M E N S I ONAL E C H O CA R D I O G RA P H Y Two-dimensional echocardiography is now the standard ultrasound imaging modality, although 3-D imaging is

82

SE CTI ON II • CARD IOVASCULAR IMAGIN G AND STRESS TESTING

increasingly adopted as it provides a different imaging approach and several distinct advantages over 2-D echo­ cardiography Initially, 3-D images are constructed and dis­ played using conventional 2-D imaging with a multiplanar transducer. Tomographic slices of the heart are obtained and constructed into a 3-D image. Image acquisition relies on matrix array transducer that replaces a single row of elements found in the conventional linear 1 -D transducer with a 2-D grid of elements. As in a lin­ ear array transducer, the timing of individual transducer ele­ ments transmitting and receiving ultrasound energy controls the direction of the ultrasound beam. A matrix array trans­ ducer offers steering in both within a slice and elevation of a beam, allowing for interrogating the entire pyramid-shaped volume. Parallel processing allows the volumetric device to receive multiple lines for any given transmit line to gener­ ate the 3-D volume . Real-time 3-D , "Live 3-D ," imaging can be performed at a lower temporal resolution (volume per second) . Alternatively, a segmented "full-volume" 3-D dataset is obtained at higher temporal resolution by com­ bining data from several cardiac cycles via ECG gating. The latter has the option of displaying images with color Doppler mapping. In the above 3-D image acquisition modes, datasets can be analyzed offline in either volume­ rendered or multiplanar reconstruction modes . Offline analysis tools are especially useful for accurately quan­ tifying volume and mass, as well as visualizing complex anatomical abnormalities. Three-dimensional echo is commercially available for both transthoracic and transesophageal echocardiography This technique has distinct advantage over 2-D echocardi­ ography including chamber quantification, valvular heart disease especially mitral valve diseases, congenital heart dis­ ease, and intraoperative applications. D O P P L E R E C H O CA R D I OG RA P H Y Doppler echocardiography utilizes the Doppler principle to determine the direction, velocity, character, and timing of blood flow within the cardiovascular system. The Doppler principle states that the frequency reflected on a moving object is a higher observed frequency than when it moves away from the observer. The Doppler shift (.1F) is the differ­ ence in frequency between the received frequency (Fr) and the transmitted frequency (Ft) : ilF = Fr - Ft

Signal backscatter from small moving objects such as red blood cells produces a change in frequency of the signal, creating a Doppler effect. The Doppler shift is related to the velocity of the moving source (V) : ilF = V I 'A

Knowing that A = elf, and that the speed of sound tends to remain constant in tissue, change in the transmitted frequency will alter the wavelength. Therefore, ilF · a · V

=>

A

= c I Ft = V I ilF

Rearranging this expression produces: ilF = V · Ft ! c

The ultrasound beam may be at an angle to the direction of blood flow. The true velocity is equal to the measured veloc­ ity divided by the cosine of the angle fJ. Therefore, ilF = Ft · V · cos O / c

where V is the true velocity of blood flow: As the sound path consists of the transmitted portion from the transducer to tissue and the reflected portion from the tissue back to the transducer, the equation is multiplied by 2, which produces the final Doppler equation: ilF = 2Ft · V · cos O / c

Since it is the velocity of the moving obj ect that is of interest, rearranging the equation produces:

V = ilF · c 1 ( 2Ft · cos O ) The angle the ultrasound beam makes with the direction of blood flow is important. When the beam is parallel to the direction of flow, the angle is 0 degrees and cos 0 degrees = 1 . When the beam is perpendicular to the direction of flow, the angle is 90 degrees and cos 90 degrees = 0, which means that there is no Doppler shift. Angles 1 2 beats. A reduction in vagal activity, which

92

SECTI O N II • CARD IOVASCULAR IMAGIN G AND STRESS TESTING TA B L E Class I Ind ications for Exercise Stress Testi ng A. Diagnosis of Obstructive CAD

Adult patients (including those with complete RBBB or 250 mm Hg or DBP > 1 1 5 mm Hg SBP, systolic blood pressure; VT, ventricular tachycardia; IVCD, intraven­ tricular conduction defect; DBP, diastolic blood pressure. (Adapted from Gibbons RJ , Balady GJ , Beasley JW, et al. ACC/AHA guide­ lines for exercise testing: executive summary: a report of the American College of Cardiology/American Heart Association Task Force on Practice Guidelines (Committee on Exercise Testing) . Circulation. 1997;96:345-354.)

are more likely to have ST depression during exercise than women not taking these supplements. However, despite the decreased accuracy of exercise ECG stress testing in women, the 2005 Guidelines for the Role of Noninvasive Testing in the Clinical Evaluation of Women with Suspected Coronary Artery Disease recommends exercise ECG stress testing as the first diagnostic test of choice in intermediate risk, symp­ tomatic women with normal baseline ECGs. In women who are asymptomatic, other parameters gained from the exer­ cise stress test such as poor exercise capacity, low HRR, and failure to reach the target heart rate are more predictive of outcome than ECG changes associated with exercise.

Metabolic Gas-Exchange Analyses Metabolic stress testing is indicated for the evaluation of exercise capacity and response to therapy in patients with heart failure being considered for heart transplantation as well as for differentiation of cardiac from pulmonary causes of exercise intolerance. It may also be used for the evalu­ ation of exercise capacity when subjective measurement is unreliable, for the assessment of responses to specific

94

SECTI O N II • CARD IOVASCULAR IMAGIN G AND STRESS TESTING TA B L E

lltEI

Age a n d Gender Estimated Fu nctiona l Capacity Estimated Functional Capacity (M ETs) Fa ir

Average

Age (y)

Poor

:::; 2 9 30-39 40-49 50-59 :2'.60

17 >16 >15 >14 >13

The prognostic information gained from stress testing is just as important, and includes exercise capacity, HRR, chronotropic competence, and the DTS. S U G G ESTE D READ I NGS Cole CR, Blackstone EH, Pashkow FJ , et al. Heart-rate recovery immediately after exercise as a predictor of mortality: N Engl ] Med. 1 999;34 1 1 3 5 1-1357. Diaz LA, Brunken RC , Blackstone E H , e t al. Independent contribution of myocardial perfusion defects to exercise capac­ ity and heart rate recovery for prediction of all-cause mortality in patients with known or suspected coronary heart disease. ] Am Coll Cardiol. 2001 ;37(6) : 1 558-1 564. Froelicher VF, Lehmann KG , Thomas R, et al. The elec­ trocardiographic exercise test in a population with reduced work­ up bias: diagnostic performance, computerized interpretation, and multivariable prediction. Veterans Affairs Cooperative Study in Health Services #0 1 6 (QUEXTA) Study Group . Quantitative Exercise Testing and Angiography. Ann Intern Med. 1998; 1 2 8 : 965-974. Frolkis JP, Pothier CE, Blackstone EH, Lauer MS. Frequent ventricular ectopy after exercise as a predictor of death. N Engl ] Med. 2003 ;348(9) : 78 1-790 Gibbons RJ , Balady GJ , Beasley JW, et al. ACC/AHA guidelines for exercise testing. A report of the American College of Cardiology/American Heart Association Task Force on Practice Guidelines (Committee on Exercise Testing) . ] Am Coll Cardiol. 1997;30( 1 ) : 2 60-3 1 1 . Gibbons R], Balady GJ , Bricker JT, et al. ACC/AHA 2002 guideline update for exercise testing: summary article. A report of the American College of Cardiology/American Heart Associa­ tion Task Force on Practice Guidelines (Committee to Update the 1997 Exercise Testing Guidelines) . ] Am Coll Cardiol. 2002 ;40(8) : 1 5 3 1-1 540.

CHAPTER 1 0 Gibbons RJ, Chatterjee K , Daley J, e t al. ACOAHNACP-ASIM guidelines for the management of patients with chronic stable angi­ na: a report of the American College of Cardiology/American Heart Association Task Force on Practice Guidelines (Committee on Man­ agement of Patients with Chronic Stable Angina) . ] Am Coll Cardiol. 1 999;33(7):2092-2 197. Jouven X, et al. Heart-rate profile during exercise as a predic­ tor of sudden death. NEJM. 2005;352 ( 1 9) : 195 1-1958. Kohli P, Gulati M . Exercise stress testing in women: going back to the basics. Circulation. 20 1 0 ; 1 2 2 :2570-2580. Lauer MS. Exercise electrocardiogram testing and prog­ nosis. Novel markers and predictive instruments. Cardiol Clin. 200 1 ; 1 9(3) :40 1-4 1 4 . Libby P, e t al. Braunwald's Heart Disease. 8th e d . Philadelphia: Saunders; 2008. Mark DB, Shaw L, Harrell FE Jr, et al. Prognostic value of a treadmill exercise score in outpatients with suspected coronary artery disease. N Engl ] Med. 1 99 1 ;32 5 : 849-853.



ELECTRO CARDIO GRAPHIC STRESS TESTI N G

95

Nishime EO, Cole CR, Blackstone EH, et al. Heart rate recovery and treadmill exercise score as predictors of mortality in patients referred for exercise ECG. JAMA. 2000;284( 1 l ) : 1 392-1 398. Patterson RE, Horowitz SE Importance of epidemiology and biostatistics in deciding clinical strategies for using diagnostic tests: a simplified approach using examples from coronary artery disease. ] Am Coll Cardiol. 1 989 ; 1 3 : 1 653-1665. Topol EJ , Califf RM . Textbook of Cardiovascular Medicine. Vol. 355. Philadelphia: Lippincott Williams &: Wilkins; 2007. Vivekananthan DP, et al. Heart rate recovery after exercise is a predictor of mortality, independent of the angiographic severity of coronary disease. ]ACC. 2003 ;42(5) :83 1-838 . Wei M, Kampert JB, Barlow CE, et al. Relationship between low cardiorespiratory fitness and mortality in normal-weight, over­ weight, and obese men. JAMA. 1 999;282 ( 1 6) : 154 7-1 5 5 3 .

QU ESTIONS AND ANSWERS Questions

1 . A 40-year-old asym ptomatic man with no risk factors undergoes stress testing as pa rt of an "Executive Physical" prog ra m . His resting elec­ trocardiog raphy (ECG) is normal and he is ta king no med ications. He has a n exercise capacity of 1 4 METs ( 1 3.S m i n utes on the Bruce protocol), no angina, a pea k heart rate of 1 80, and 1 mm of down-sloping ST-seg ment depression noted in lead VS. His Duke tread m i l l score (DTS) is: a. 9 b. 8.5 c. 7.5 d. 3.5 e. 2 2. Given these test resu lts, the next most appropriate step is: a. No fu rther ca rd iac testing b. A stress imaging study c. A repeat stress test i n 1 year d. Coronary angiogra phy 3. A 55-yea r-old wom a n presents with i nterm it­ tent substernal chest pa i n that rad iates to the left arm. The pain is not clea rly exertional and is not clea rly relieved with rest. There is no histo ry of gastrointestinal problems; her symptoms are not related to meals or body position. Her resting ECG is normal and she is ta king no medications. She is referred for a n exercise test and is fou n d to have ST-seg ment depression. Ass u m i n g that the true, u n biased sensitivity of exercise ST-seg ment changes is 45% and the specificity is 85%, the

l i ke l i hood that she has at least one 50% coronary a rtery stenosis is: a. 0.25 b. 0.50 c. 0.75 d. 0.80 e. 0.90 4. A 60-year-old man with chronic obstructive pul­ monary d isease (COPD) (FEV1 1 .25) and chronic ischemic card iomyopathy (EF 30%) is referred for metabolic stress testing, which shows the follow­ ing: pea k Vo2 1 5 m l/kg/min, pea k Vco2 1 8 m l/kg/ m i n, Vo2 at anaerobic threshold 1 0 m l/kg/m in, pea k VE 45 l/m i n . Which of the fol l owing is true? a. The test was su bmaxi m a l . b . T h e pri m a ry l i mitation t o exercise is card iac. c. The pri m a ry l i mitation to exercise is pulmonary. d. The patient should be referred for ca rdiac transpla ntation. e. I t is not possible to differentiate ca rd iac from pul monary l i mitations to exercise i n this patient. 5 . A 60-year-old man presents with exertional pressu re-l i ke chest d iscomfort that is relieved with rest and that often radiates to the left a rm and jaw. His resting ECG is norma l. He is ta king no med ica­ tions. Which of the following is true? a. The patient should be referred for coronary angiogra phy. b. The patient should have an exercise test to determine whether obstructive coronary a rtery d isease (CAD) is present.

(Continued)

96

SECTI O N II • CARD IOVASCULAR IMAGIN G AND STRESS TESTING c . The patient s h o u l d b e referred for a n exercise imaging study. d. The patient should have a n exercise test to determine his short- and long-term prog nosis. e. The patient need not have a ny test; he should be started on a beta-blocker, aspirin, and a l i pid­ lowering agent and then fol l owed. 6. In which of the fol l owing patients is an exer­ cise ECG stress test recom mended by class I indications? a. A 45-year-old man with a past medical history of hypertension who presents with postprandial abdominal discomfort for the past few weeks b. A 65-yea r-old female with a history of inferior myoca rd ial i nfa rction, status post percutaneous coronary i ntervention with stent placement i n the right coronary a rtery and hypertension, who presents with worsening exertional, sub­ sternal chest pain for the past month, relieved with rest and n itrog lycerin c. A 50-year-old man with a fam i ly history of early CAD who presents for a routi ne physica l exam and is noted to have left bundle branch block on resting ECG d. A 60-year-old female with a history of 40-pack­ year smoking, hyperlipidemia, and diabetes mellitus, who presents with new symptoms of chest discomfort brought on with exertion for the past 2 weeks. Symptoms are relieved with rest. Her resting ECG has a right bundle branch block. e. An 85-year-old man with exertional chest pain and shortness of breath. He has a harsh systolic murmur at the right u pper sternal border with radiation to the carotids and a soft P 2 on exam. 7. A 55-year-old woman with a history of hyperten­ sion and hyperl ipidemia presents to the ER with sym ptoms of su bsternal chest pa i n radiating to the left arm that started 2 days prior to presenta­ tion. ECG reveals no a bnorma l ities and she is chest pai n free after two sublingual n itrog lycerin ta blets. She is ruled out for acute myoca rd ial i nfa rction by three sets of bioma rkers 8 hours apart. What is the next recommended step? a. Refer to the catheterization lab for i mmediate PCI. b. Order a n exercise ECG stress test. c. Order a Persa ntine n uclear stress test because the exercise ECG test wi l l l i kely show false posi­ tive ST depressions i n female patients. d. Send the patient home with no fu rther testing. 8. The patient i n Question 7 undergoes a n ECG exer­ cise stress test with the fol l owing resu lts:

She completes 8 m i n utes of a mod ified Bruce protocol . She reaches a heart rate of 1 60 bpm, which decreases to 1 52 bpm after 1 m i n ute of recovery. She has m i ld, nontest-limiting angina d u ri n g the exa m. Her ECG has no ST changes but does show occasional premature ventricu lar complexes (PVC's). Which of the fol lowi ng is a true statement based on these resu lts? a. Based on her DTS score, she has a mortal ity risk of < 1 % per yea r. b. Her resu lts can not be i nterpreted beca use of the poor sensitivity and specificity of exercise ECG stress testi ng i n women. c. She has an increased risk of morta lity based on her heart rate recovery (H RR). d. PVCs have no prog nostic value i n stress testing. e. Her stress test should have been i m med iately term inated when she experienced sym ptoms of a n g i na. 9. All of the fol lowi ng patients have contraindica­ tions to ECG exercise stress testing except: a. A 70-year-old man who had chest pa i n 4 8 hours prior t o presenting t o t h e E R . He is cu rrently chest pain free. His ECG shows new q waves i n V 1 -V3 that were not present on a n ECG 1 month prior. b. A 45-yea r-old woman with postpa rtu m cardio­ myopathy who presents with deco m pensated heart fai l u re. c. A 65-yea r-old man who presents with chest pa i n for the past few days brought on by exertion and deep breaths and relieved with rest. He was d ischarged 1 week prior after un­ dergoing a right tota l knee replacement. d. A 55-year-old man with a history of hyperten­ sion who presents with complai nts of exertional chest pai n for the past month. His resting blood pressure is 1 60/1 00 and he has a heart rate of 65 with first degree AV block on resting ECG. e. An 85-yea r-old woman with recent history of syncope. On exa m, she has a systolic ejection m u rm u r with radiation to both carotid a rteries. Her ca rotid pu lses a re delayed. 1 0. Which of the fol l owing parameters is a ma rker of severe CAD? a. U p-sloping ST depression in recovery b. 1 -m m ST elevations in leads with q waves c. Exercise ca pacity of 55% to 60% of maximal myocardial activity on rest perfusion images can be used to identify myocardial viability. In a meta-analysis, radionuclide techniques and dobutamine

CHAPTER 13 • NU CLEAR STRESS TESTIN G echocardiography had similar positive and negative pre­ dictive values for identifying segments with improvement in wall motion following revascularization. 3 1 The nuclear imaging techniques appeared to be slightly more sensitive in identifying viability, as defined by an improvement in func­ tion following revascularization, whereas dobutamine echo­ cardiography appeared to be slightly more specific. S U M M A RY Nuclear cardiac imaging techniques have become an integral component of the practice of clinical cardiology. As with any diagnostic tool, the physician must have a working knowl­ edge of the strengths and limitations of the imaging technol­ ogy in order to utilize this technology for optimum clinical benefit. In appropriately selected patients, MPI is very useful for identifying obstructive CAD , for characterizing the func­ tional significance of equivocal coronary stenoses, and for risk stratification. In those with CAD and systolic ventricu­ lar dysfunction, nuclear imaging techniques can be used to quantitate the severity of the ventricular dysfunction and to monitor the myocardial response to treatment noninva­ sively. Nuclear imaging techniques may also identify those individuals with ischemic cardiomyopathy who are likely to benefit functionally and prognostically from coronary revascularization. AC K N OW L E D G M E N TS The authors would like to thank Dr. Wael Jaber and Dr. Manuel Cerqueira for their thoughtful review of this chapter, as well as Dr. Omosalewa Lalude for her contribu­ tions to the prior version of this chapter. REFERENCES 1 . Di Carli M, Czernin J, Hoh CK, et al. Relation among stenosis severity, myocardial blood flow, and flow reserve in patients with coronary artery disease. Circulation. 1995;9 1 : 1 944-1 95 1 . 2 . Medrano R , Lowry RW, Young JB, e t al. Assessment o f myo­ cardial viability with 99mTc sestamibi in patients undergoing cardiac transplantation. A scintigraphic/pathological study. Circulation. 1 996;94: 1 0 1 0- 1 0 1 7 . 3 . Canty J M , Fallavollita JA. Hibernating myocardium. ] Nucl Car­ diol. 2005 ; 1 2 : 1 04- 1 1 9 . 4. Travin M l , Bergmann SR. Assessment o f myocardial viability. Semin Nucl Med. 2005 ;35:2-1 6. 5 . Hendel RC, Jamil T, Glover DK. Pharmacologic stress test­ ing: new methods and new agents. ] Nucl Cardiol. 2003 ; 1 0 : 1 9 7-204. 6 . Cerqueira MD , Nguyen P, Staehr P, et al. Effects of age, gender, obesity, and diabetes on the efficacy and safety of the selective A2A agonist regadenoson versus adenosine in myocardial per­ fusion imaging integrated ADVANCE-MP! trial results. ]ACC Cardiovasc Img. 2008 ; 1 (3):307. 7. MahmarianJJ , Cerqueira MD , Iskandrian AE, et al. Regadenoson induces comparable left ventricular perfusion defects as ade­ nosine: a quantitative analysis from the ADVANCE MPI 2 trial. ]ACC Cardiovasc Img. 2009 ;2(8):959.

1 31

8. Lapeyre AC, Goraya TY, Johnston DL, et al. The impact of caf­ feine on vasodilator stress perfusion studies. ] Nucl Cardiol. 2004; 1 1 506-5 1 1 . 9 . Henzlova MJ , Cerqueira MD , Hanesen CL, et al. ASNC guide­ lines for nuclear cardiology procedures. Stress protocols and tracers. Available on line at www. asnc. org Guidelines and Stan­ dards. Stress protocols and tracers 2009. 10. Hendel RC, et al. ACCF/ASNC/ACR/AHA/ASE/SCCT/SCMR/ SNM 2009 appropriate use criteria for cardiac radionuclide imaging. ] Am Coll Cardiol. 2009 ;53: 220 1-2229. 1 1 . Wackers FJT, Brown KA , Heller GV, et al. American Society of Nuclear Cardiology position statement on radionuclide imaging in patients with acute ischemic syndromes in the emergency de­ partment or chest pain center. ] Nucl Cardiol. 2002 ;9:246-250. 12. Antman EM, Anbe DT, Armstrong PW, et al. ACC/AHA guide­ lines for the management of patients with ST-Elevation myo­ cardial infarction. ] Am Coll Cardiol. 2004;44 : 6 7 1-7 1 9 . 1 3 . Mahmarian JJ , Dwivedi G, Lahiri T. Role of nuclear cardiac imaging in myocardial infarction: postinfarction risk stratifica­ tion. ] Nucl Cardiol. 2004; 1 l : 1 86-209. 14. Brown KA , Heller GV, Landin RS, et al. Early dipyridamole 99mTc-sestamibi single photon emission computed tomographic imaging 2 to 4 days after acute myocardial infarction predicts in­ hospital and postdischarge cardiac events. Comparison with sub­ maximal exercise imaging. Circulation. 1999 ; 1 00:2060-2066. 1 5 . Anderson JL, Adams CD, Antman EM, et al. ACC/AHA 2007 guidelines for the management of patients with unstable an­ gina/non-ST-elevation myocardial infarction. ] Am Coll Cardiol. 2007 ;50: 652-726. 16. Machac ] . Cardiac positron emission tomography imaging. Se­ min Nucl Med. 2005;35 : 1 7-36. 1 7 . Hansen HL. The conundrum of left bundle branch block [edi­ torial] . ] Nucl Cardiol. 2004; 1 1 : 90-92. 1 8 Vaduganathan P, He ZX, Raghavan C, et al. Detection of left anterior descending coronary artery stenosis in patients with left bundle branch block: exercise, adenosine or dobutamine imaging? ] Am Coll Cardiol. 1 996;28(3) : 543 . 1 9 . Shaw LJ , Iskandrian AE. Prognostic value of gated myocardial perfusion SPECT. ] Nucl Cardiol. 2004; 1 1 : 1 7 1-185. 20. Zellweger MJ , Lewin HC, et al. When to stress patients after 1 coronary artery bypass surgery?' : risk stratification in patients early and late post-CABG using stress myocardial perfusion SPECT: implications of appropriate clinical strategies. ] Am Coll Cardiol . 200 1 ;37(1): 1 44- 1 5 2 . 2 1 . Brown KA . Advances i n nuclear cardiology. Preoperative risk stratification. ] Nucl Cardiol. 2004; 1 1 : 335-348. 22. Mieres JH, Shaw LJ , Arai A, et al. Role of noninvasive testing in the clinical evaluation of women with suspected coronary artery disease. Consensus statement from the Cardiac Imag­ ing Committee, Council on Clinical Cardiology, and the Car­ diovascular Imaging and Intervention Committee, Council on Cardiovascular Radiology and Intervention, American Heart Association. Circulation. 2005 ; 1 1 1 : 682-696. 23. Berman DS, Wong ND, Gransar H, et al. Relationship between stress-induced ischemia and atherosclerosis measured by coro­ nary calcium tomography ] Am Coll Cardiol. 2004;44:923-930. 24. Thompson RC, McGhie AI , Moser Kw, et al. Clinical utility of coronary calcium scoring after nonischemic myocardial perfu­ sion imaging. ] Nucl Cardiol . 2005 ; 1 2 : 392-400. 25. Wackers FJT, Young LH, et al. Detection of silent myocardial ischemia in asymptomatic diabetic subjects-the DIAD study Diabetes Care. 2004;2 7(8) : 1 954-1960.

1 32

SECTION I I • CARD IOVASCULAR IMAGIN G AND STRESS TESTING

26. Young LH, Wackers FJT, e t a l . Cardiac outcomes after screen­ ing for asymptomatic coronary artery disease in patients with type 2 diabetes-the DIAD study: a randomized controlled trial. JAMA. 2009;30 1 ( 1 5) : 1 54 7-1 5 5 5 . 2 7 . Greenland P, Alpert J S , Beller GA, e t a l . 20 1 0 ACCF/AHA guideline for assessment of cardiovascular risk in asymptom­ atic adults. ] Am Coll Cardiol. 20 1 0;56:2 1 82-2 1 9 9 . 28. Allman K C , Shaw LJ, Hachamovitch R, e t al. Myocardial viability testing and impact of revascularization on prognosis in patients with coronary artery disease and left ventricular dysfunction: a meta-analysis. ] Am Coll Cardiol. 2002;39: 1 1 5 1-1 1 58.

29. Beanlands RSB, Nichol G, Huszti E , e t al. F- 1 8-fluorodeox­ yglucose positron emission tomography imaging-assisted management of patients with severe left ventricular dys­ function and suspected coronary disease. ] Am Coll Cardiol. 2007;50(20) :2002-2 0 1 2 . 3 0 . Bonow RO, e t al. Myocardial viability and survival in ischemic left ventricular dysfunction. N Engl] Med. 20 1 1 ;364: 1 6 1 7-1625. 3 1 . Bax JJ, Poldermans D , Elhendy A, et al. Sensitivity, specific­ ity, and predictive accuracy of various noninvasive techniques for detecting hibernating myocardium. Curr Prob! Cardiol. 200 1 ;26: 1 4 1-188.

QU ESTIONS AND ANSWERS Questions

1 . A 65-year-old man with a 45-pack-year smoking history, hyperl ipidem ia, interm ittent claud ication, and hypertension has been experiencing short­ ness of breath with exertion for 3 months. He is referred for an adenosine card iac single photon emission com puted tomography (SPECT) study for sym ptom evaluation. His medications include metoprolol, lisinopril, aspirin, theophyl l ine, and sim­ vastatin. During the adenosine i nfusion, the patient does not report a ny chest pain, nor are there a ny ST-segment cha nges on the electrocard iogra m (ECG). Hig h-degree atrioventricu lar (AV) block develops 30 seconds following the sta rt of the ad­ enosine i nfusion, prompting the cardiology fellow attending the stress test to stop the i nfusion. The SPECT perfusion i mages are i nterpreted as normal, with no regional ischemia. Despite contin uation of medica l thera py the patient's sym ptoms persist, and 6 weeks later he is referred for ca rd iac cath­ eterization. At catheterization there is a proxi mal 75% right coronary artery stenosis, a 75% to 80% proxi mal left anterior descending artery stenosis, and a 70% to 75% stenosis of the proximal circum­ flex a rtery. Possible reasons for the absence of a reversible perfusion defect on the card iac SPECT study include a l l of the fol lowi ng except: a. I ngestion of a chocolate bar 3 hours before the test was performed b. Right bundle branch block (RBBB) on the resting ECG c. Provocation of"balanced ischemia" by the ad­ enosi ne stress d. Fai l u re to withhold the patient's med ications prior to the test e. Termi nation of the adenosine i nfusion at 30 seconds 2. Which of the fol lowing i n d ividuals is l i kely to ben­ efit most from n uclear stress imaging?

a. A 25-year-old man with mid line chest pain, which is tender to the touch and i ntermittently responsive to ibuprofen b. A 30-year-old woman who gets chest discomfort after eating highly seasoned food but who has no trouble when she plays tennis three times a week c. A 39-year-old male smoker with shortness of breath on exertion and a m i l d ly elevated low-density l i poprotein (LDL) cholesterol level. His father died suddenly at age 45, and his 42-year-old brother recently had two stents placed in one of his coronary a rteries. The rest­ ing ECG shows nonspecific ST-T-wave changes. d. A 76-year-old man, former smoker, with hy­ pertension and recent inferior wa l l myocard ial infarction (Ml) treated by placing two stents in the right coronary artery. He was awa kened by an episode of chest pain that lasted almost 20 min utes and that has not responded to sublin­ gual nitrog lycerin. e. A 55-year-old female with hypercholesterolemia, hypertension, frequent heartburn, and increasing shortness of breath on exertion. On echocardiog­ raphy, there is moderate left ventricular hypertro­ phy (LVH), and aortic valvu lar calcification with an estimated aortic valve area of 0.69 cm 2• 3. A 58-year-old male executive is seen for left-sided chest pain. He has a history of bilateral thumb pain for which he took a cox-2 inhibitor for 2 years before switching to naproxen. He works long hours and ad­ mits to fatigue and loss of libido. He has an elevated lipoprotein A (Lpa) level but an otherwise normal lipid profile.The hs-CRP level is normal and a cardiac SPECT study 3 years earlier was normal. He undergoes an exercise cardiac SPECT and exercises to 1 0 METs on the Bruce protocol, achieving 1 06% of his maximum predicted heart rate (MPHR). With exercise, he experi­ ences fatigue but no angina. No ST-segment changes are noted with stress.The myocardial perfusion im­ ages from the exercise study are shown (see figure).

CHAPTER 13 • NU CLEAR STRESS TESTI N G



..1 .\"• : 'f"' •- · M• 0.40.94 There was no significant difference between candesartan and placebo with respect to the primary endpoint of cardiovascular death or admission for HF A significant difference was seen in the secondary outcome of HF admission, however, with a relative risk reduction of 1 5 % . The I-PRESERVE study enrolled 4 , 1 28 patients with HF and LVEF >0.45 .95 The primary outcome was a composite of death or hospitalization for cardiovascular reasons. At a mean follow-up of 50 months, there was no significant difference between irbesartan and placebo in the primary outcome or in rates of HF admission.

M O N I TO R I N G A N D F O L LOW- U P

Disease Management Programs There is a growing interest in the development and imple­ mentation of disease management programs for HF These are generally multidisciplinary programs that involve for­ mal guidelines for inpatient management, structured dis­ charge planning, and close post-discharge follow-up . In a meta-analysis of small trials, disease management programs were associated with a reduction in mortality and readmis­ sion for HF96



MEDICAL TREATMENT O F HEART FAILURE

1 75

I m plantable Hemodynamic Monitors Direct measurements of right ventricular and pulmonary artery pressures with implanted devices have been trialed in very small studies. In one study, patients had decreased rates of HF readmission when data from the invasive monitor was available to physicians.97 Certain implantable defibrillators have internal sensors that measure thoracic impedance as an estimate of lung water caused by cardiogenic pulmonary edema. The utility of monitoring such parameters is still being actively studied.98 REFERENCES 1 . Hunt SA, Abraham WT, Chin MH, et al. 2009 Focused up­ date incorporated into the ACC/AHA 2005 guidelines for the diagnosis and management of heart failure in adults: a report of the American college of Cardiology Foundation/ American Heart Association task force on practice guidelines developed in collaboration with the International Society of Heart and Lung Transplantation. ] Am Coll Cardiol . 2009 ; 5 3 : e l-e9 0 . 2 . Lindenfield J, Albert N M , Boehmer JP, e t al. Executive Sum­ mary: Heart Failure Society of America comprehensive heart failure practice guideline. ] Card Fail. 2 0 1 0 ; 1 6:4 75-539. 3 . Coats AJ , Adamopoulos S, Radaelli A, et al. Controlled trial of physical training in chronic heart failure: exercise perfor­ mance, hemodynamics, ventilation and autonomic function. Circulation. 1 992 ;85(6) :2 1 1 9-2 1 3 1 . 4 . Hambrecht R , Niebauer J, Fiehn E, e t al. Physical training in patients with stable chronic heart failure: effect on cardiorespi­ ratory fitness and ultrastructural abnormalities of leg muscles. ] Am Coll Cardiol. 1 995 ;25(6) 1 239-1 249 5. Belardinelli R, Georgiou D, Cianci G, et al. Randomized, controlled trial of long-term moderate exercise training in chronic heart failure: effects on functional capacity, qual­ ity of life, and clinical outcome. Circulation. 1 9 9 9 ;99(9): 1 1 73-1 1 8 2 . 6. Haykowsky MJ, Liang Y, Fechter D , e t al. A Meta-analysis of the effect of exercise training on left ventricular remod­ eling in heart failure patients: the benefit depends on the type of training performed. ] Am Coll Cardiol. 2007;49(24) : 2329-2336. 7. O'Connor CM, Whellan DJ , Lee KL, et al. Efficacy and safe­ ty of exercise training in patients with chronic heart failure: HF-ACTION randomized clinical trial. JAMA 2009;30 1 ( 1 4) : 1439-1450. 8. Satwani S , Dec GW, Narula ] . 13-Adrergic blockers in heart failure: review of mechanisms of action and clinical outcomes. ] Cardiovasc Pharmacol Therapeut. 2004;9(4) : 243-255. 9 . Hall SA, Cigarroa CG, Marcoux L, et al. Time course of im­ provement in left ventricular function, mass and geometry in patients with congestive heart failure treated with 13 adrenergic blockade. ] Am Coll Cardiol. 1995;25 : 1 1 54-1 1 6 1 . 1 0 . Packer M , Bristow MR, Cohn JM, e t al. The effect o f carvedilol on morbidity and mortality in patients with chronic heart fail­ ure. N Engl ] Med. 1 996;334: 1 349- 1 3 5 5 . 1 1 . CIBIS II investigators. The Cardiac Insufficiency Biso­ prolol Study II (CIBIS II) : a randomized trial. Lancet. 1 999;353(9 146) 9-1 3 .

1 76

SECTION III



CONGESTIVE HEART FAILURE AND CARD I O MYOPATHY

1 2 . MERIT-HF study group . Effects of metoprolol CR/XL in chronic heart failure: Metoprolol CR/XL Randomized Inter­ vention Trial in Congestive Heart Failure (MERIT-HF) . Lancet. l 999 ;353(9 1 69) :200 1-2007. 13. Packer M, Coats AJ, Fowler M, et al. Effect of carvedilol on survival in severe chronic heart failure. N Engl ] Med. 200 1 ;344(22) : 165 1-1658. 14. Poole-Wilson PA, Swedberg K, Cleland JG, et al. Comparison of carvedilol and metoprolol on clinical outcomes in patients with chronic heart failure in the Carvedilol Or Metoprolol Eu­ ropean Trial (COMET) : a randomized controlled trial. Lancet. 2003;362 7-1 3 . 1 5 . Beta Blocker Evaluation o f Survival Trial (BEST) Investiga­ tors. A trial of the beta-blocker bucindolol in patients with advanced chronic heart failure. N Engl ] Med. 200 1 ;344: 1 659-1667. 1 6 . Yusuf S , Sleight P, Pogue J , et al. Effects of an angiotensin­ converting-enzyme inhibitor, ramipril, on cardiovascu­ lar outcomes in high-risk patients: the Heart Outcomes Prevention Evaluation Study Investigators. N Engl ] Med. 2000;342(3) 145- 1 5 3 1 7 . Konstam MA Rousseau M F, Kroneneberg MW, e t al. Effects of the angiotensin converting enzyme inhibitor enalapril on the long-term progression of left ventricular dysfunc­ tion in patients with heart failure. Circulation. 1992 ;86(2) : 43 1-438. 18. Brilla CG, Funck RC, Rupp H . Lisinopril-mediated regression of myocardial fibrosis in patients with hypertensive heart dis­ ease. Circulation. 2000 ; 1 02 ( 1 2) : 1 388-1393. 1 9 . The CONSENSUS Trial Study Group. Effects of enalapril on mortality in severe congestive heart failure: results of the Co­ operative North Scandinavian Enalapril Survival Study N Engl J Med. 1 987;3 1 6(23) : 1429-1435 20. The SOLVD Investigators. Effect of enalapril on survival in pa­ tients with reduced left ventricular ejection fractions and con­ gestive heart failure. N Engl ] Med. 1 9 9 1 ;325(5) :293-302. 2 1 . The SOLVD Investigators. Effect of enalapril on mortality and the development of heart failure in asymptomatic patients with reduced left ventricular ejection fractions. N Engl ] Med. l 992;327(10): 685-69 1 . 2 2 . Cohn JM, Johnson G , Ziesche S , et al. A comparison o f enala­ pril with hydralazine-isosorbide dinitrate in the treatment of chronic congestive heart failure. N Engl ] Med. 1 9 9 1 ;325(5): 303-3 1 0 . 2 3 . Packer M, Poole-Wilson PA, Armstrong P W, e t al. Comparative effects of low and high doses of the angiotensin-converting en­ zyme inhibitor lisinopril on morbidity and mortality in chronic heart failure. Circulation. 1999 ; 1 00:23 1 2-23 1 8 . 2 4 . Pfeffer MA Braunwald E, Moyle LA , e t al. Effect o f captopril on mortality and morbidity in patients with left ventricular dys­ function after myocardial infarction: results of the Survival And Ventricular Enlargement trial: the SAVE investigators. N Engl J Med. 1 992;327(10) 669-677 25. The AIRE Investigators. Effect of ramipril on mortality and morbidity of survivors of acute myocardial infarction with clini­ cal evidence of heart failure. Lancet. 1 993 ;342(8875) :82 1-828. 26. Kober L, Torp-Pederson C, Carlsen JE, et al. A clinical trial of the angiotensin-converting-enzyme inhibitor trandolapril in patients with left ventricular dysfunction after myocardial

27.

28.

29.

30.

31.

,

,

32.

33.

34.

35.

36.

37.

38.

39.

40.

41.

42 .

infarction: Trandolapril Cardiac Evaluation (TRACE) Study Group. N Engl ] Med. 1995 ;333(25) : 1 670-1676. Booz GW, Baker KM. The role of type 1 and type 2 angiotensin receptors in angiotensin II-induced cardiomyocyte hypertro­ phy Hypertension. 1996;28: 635-640 . Granger CB, McMurray JJ, Yusuf S, et al. Effects of cande­ sartan in patients with chronic heart failure and reduced left-ventricular systolic function intolerant to angiotensin­ converting-enzyme inhibitors: the CHARM Alternative trial. Lancet. 2003;362(9386) : 7 72-776. Pitt B, Segal R, Martinez FA, et al. Randomized trial of losar­ tan versus captopril in patients over 65 with heart failure (Evaluation of Losartan in the Elderly study, ELITE) . Lancet. 1997 ;349 747-752 . Pitt B, Poole-Wilson PA, Segal R, et al. Effect of losartan com­ pared with captopril on mortality in patients with symptom­ atic heart failure: randomized trial-the Losartan Heart Failure Survival Study ELITE II. Lancet. 2000;355 : 1 582-1587. Dickstein K, Kjekshus ] . Effects of losartan and captopril on mortality and morbidity in high-risk patients after myocar­ dial infarction: the OPTIMAAL randomized trial. Lancet. 2002 ;360(9335) 720-760 . Pfeffer MA, McMurray JJ , Velazquez EJ , et al. Valsartan, cap­ topril or both in myocardial infarction complicated by heart failure, left ventricular dysfunction or both. N Engl ] Med. 2003 ;349(20) 1893- 1 906. McMurray JJ , Ostergren J, Swedberg K, et al. Effects of can­ desartan in patients with chronic heart failure and reduced left-ventricular systolic function taking angiotensin-con­ verting-enzyme inhibitors: the CHARM-Added trial. Lancet. 2003;362(9386) : 767-77 1 . Cohn JN , Tognoni G . A randomized trial of the angiotensin-re­ ceptor blocker valsartan in chronic heart failure. N Engl ] Med. 200 1 ;345 1 667-1675. Pitt B , Zannad F, Remme WJ . The effect of spironolactone on morbidity and mortality in patients with severe heart failure. N Engl ] Med. 1999 ;34 1 ( 1 0) : 709-7 1 7 . Pitt B, Remme WJ, Zannad F, e t al. Eplerenone, a selective al­ dosterone blocker, in patients with left ventricular dysfunc­ tion after myocardial infarction. N Engl ] Med. 2003;348(14): 1 309-132 1 . Zannad F, McMurray JJ, Krum H , et al. Eplerenone in patients with systolic heart failure and mild symptoms. N Engl ] Med. 20 1 1 ;364( 1 ) : 1 1-2 1 . Cohn JN, Archibald DG, Ziesche S , et al. Effect o f vasodilator therapy on mortality in chronic congestive heart failure: results of a Veterans Administration Cooperative Study N Engl ] Med. 1986;3 14 1 547- 1 5 5 2 . Taylor AL, Ziesche S , Yancy C, e t a l . Combination of isosorbide dinatrate and hydralazine in blacks with heart failure. N Engl ] Med. 2004;35 1 (20) :2049-2057. The Multicenter Diltiazem Postinfarction Trial Research Group. The effect of diltiazem on mortality and reinfarction after myo­ cardial infarction. N Engl ] Med. 1 988;3 1 9(7) :385-392 . Packer M, O'Connor CM, Ghali JK, et al. Effect of amlodip­ ine on morbidity and mortality in severe chronic heart failure. N Engl ] Med. 1996 ;335 : 1 1 07-1 1 14 . Udelson J E , DeAbate CA, Berk M, e t a l . Effects of amlodip­ ine on exercise tolerance, quality of life, and left ventricular

CHAPTER 1 6

43.

44.

45 .

46 .

47 .

48.

49.

50.

51.

52.

53.

54.

55.

56.

57.

58.

function in patients with heart failure from left ventricular sys­ tolic dysfunction. Am Heart ]. 2000 ; 1 39(3) :503-5 1 0 . Elkayam U, Amin J, Mehra A, e t al. A prospective random­ ized, double-blind, crossover study to compare the efficacy and safety of chronic nifedipine therapy with that of isosorbide dinitrate and their combination in the treatment of chronic congestive heart failure. Circulation. 1990;82 : 1 954-1 96 1 . ALLHAT collaborative research group. Maj or outcomes in high-risk hypertensive patients randomized to angiotensin­ converting enzyme or calcium channel blocker vs diuretic: the Antihypertensive and Lipid-Lowering treatment to prevent Heart Attack Trial. ]AMA 2002 ;288(23) :298 1-2997. The Digitalis Investigation Group . The effect of digoxin on mortality and morbidity in patients with heart failure. N Engl J Med. 1997;336(8) :525-533. Uretsky BF, Young JB, Shahidi FE, et al. Randomized study as­ sessing the effect of digoxin withdrawal in patients with mild to moderate chronic congestive heart failure: results of the PROVED trial. ] Am Coll Cardiol . 1993;22 955-962 . Packer M, Gheorghiade M, Young JB, et al. Withdrawal of digoxin from patients with chronic heart failure treated with angiotensin-converting-enzyme inhibitors. N Engl ] Med. 1993;329: 1-7. Rathore SS, Curtis JP, Wang Y, et al. Association of serum di­ goxin concentration and outcomes in patients with heart fail­ ure. ]AMA 2003 ;289(7) :87 1-878. Guazzi M, Samaj a M, Arena R, et al. Long-term use of sildenafil in the therapeutic management of heart failure. ] Am Coll Car­ diol . 2007;50 2 1 36-2 144. Lewis GD , Shah R, Shahzad K, et al. Sildenafil improves exer­ cise capacity and quality of life in patients with systolic heart failure and secondary pulmonary hypertension. Circulation. 2007; 1 66: 1 555-1562. He K, Song Y, Daviglus ML, et al. Accumulated evidence on fish consumption and coronary heart disease mortality-a meta­ analysis of cohort studies. Circulation. 2004 ; 1 09:2705-2 7 1 1 . GISSI-HF Investigators. Effect of n-3 polyunsaturated fatty ac­ ids in patients with chronic heart failure (the GISSI-HF trial) : a randomized, double blind, placebo controlled trial. Lancet. 2008;372 : 1 223-1230. Swedberg K, Komaj da M, Bohm M, et al. Ivabradine and out­ comes in chronic heart failure (SHIFT) : a randomized placebo­ controlled study. Lancet. 2 0 1 0;376(9744) :875-885 . Ghali JK, Anand IS, Abraham WT, et al. Randomized dou­ ble-blind trial of darbepoetin alfa in patients with symp­ tomatic heart failure and anemia. Circulation. 2008; 1 1 7: 526-535. Anker SD, Colet JC, Filippatos G, et al. Ferric carboxymaltose in patients with heart failure and iron deficiency. N Engl ] Med. 2009 ;36 1 : 2436-2448. Talajic M, Khairy P, Levesque S, et al. Maintenance of sinus rhythm and survival in patients with heart failure and atrial fibrillation. ] Am Coll Cardiol. 2 0 1 0 ; 5 5( 1 7) 1 796-1802. Jafri SM, Ozawa T, Mammen E, et al. Platelet function, throm­ bin and fibrinolytic activity in patients with heart failure. Eur Heart ]. 1993 ; 14:205-2 1 2 . Yamamoto K , Ikeda U, Furuhashi K et al. The coagulation sys­ tem is activated in idiopathic cardiomyopathy. ] Am Coll Car­ diol. 1995;25 : 1 634- 1 640.



MEDICAL TREATMENT O F HEART FAILURE

1 77

59. Ahnert AM, Freudenberger RS . What do we know about anticoagulation in heart failure? Curr Opin Cardiol. 2008;23: 228-232. 60. Massie BM, Collins JF, Ammon SE, et al. Randomized trial of warfarin, aspirin, and clopidogrel in patients with chron­ ic heart failure: the Warfarin and Antiplatelet Therapy in Chronic Heart failure (WATCH) trial. Circulation. 2009 ; 1 1 9 : 1 6 1 6-1 624. 6 1 . Cokkinos DV, Haralabopoulos GC, Kostis JB, et al. Efficacy of antithrombotic therapy in chronic heart failure: the HELAS study. Eur] Heart Fail. 2006;8(4) :428-432 . 62 . Connors AF, Speroff T, Dawson NV, et al. The effectiveness of right heart catheterization in the initial care of critically ill pa­ tients. ]AMA 1996;2 76(1 1): 889-897. 63 . Shah MR, Hasselblad V, Stevenson LW, et al. Impact of the pulmonary artery catheter in critically ill patients: meta­ analysis of randomized clinical trials. JAMA 2005 ;294( 1 3) : 1 664-1670. 64. The ESCAPE Investigators. Evaluation Study of Congestive Heart Failure and Pulmonary Artery Catheterization Effective­ ness. JAMA 2005;294( 1 3) : 1 625- 1 633. 65. Hammarlund MM, Paalzow LK, Odlind B . Pharmacokinet­ ics of furosemide in man after intravenous and oral admin­ istration: application of moment analysis. Eur ] Clin Pharm. 1 984;26(2): 1 9 7-207. 66. Brater DC. Diuretic pharmacokinetics and pharmacodynam­ ics. In: van Boxtel CJ , Holford NH, Danhof M, eds. The In vivo Study of Drug Action: Principles and Applications of Kinet­ ic-Dynamic Modelling. Amsterdam: Elsevier Science; 1992: 253-275. 67. Felker GM, Lee KL, Bull DA, et al. Diuretic strategies in patients with acute decompensated heart failure. N Engl ] Med 20 1 1 ;364(9) : 797-805 68. Volz EM, Felker GM. How to use diuretics in heart failure. Curr Treat Opt Card Med. 2009 ; 1 1 :426-432 . 69. Ahmed A, Husain A, Love TE, e t al. Heart failure, chronic diuretic use, and increase in mortality and hospitalization: an observational study using propensity score methods. Eur Heart ]. 2006;27: 143 1-1439. 70. Hasselblad V, Stough WG, Shah MR, et al. Relation between dose of loop diuretics and outcomes in a heart failure popu­ lation: results of the ESCAPE trial. Eur ] Heart Fail. 2007;9: 1 064-1069. 7 1 . Konstam MA, Gheorghiade M, Burnett JC, et al. Effects of oral tolvaptan in patients hospitalized for worsening heart failure: the EVEREST Outcome Trial . JAMA 2007;297( 1 2) : 1 3 19-133 1 . 72. Massie BM, O'Connor CM, Metra M , et al. Rolofylline, an ad­ enosine A1 receptor antagonist, in acute heart failure. N Engl ] Med. 2 0 1 0;363 : 1 4 1 9-1428. 73. Costanzo MR, Guglin ME, Saltzberg MT, et al. Ultrafiltra­ tion versus intravenous diuretics for patients hospitalized for acute decompensated heart failure. ] Am Coll Cardiol. 2007 ;49(6) :675-683 74. Francis JS, Siegel RM, Goldsmith SR, et al. Acute vasoconstric­ tor response to intravenous furosemide in patients with chron­ ic congestive heart failure - activation of the neurohormonal axis. Ann Intern Med. 1985 ; 103: 1-6 . 75 . VMAC Investigators. Intravenous nesiritide versus nitro­ glycerin for treatment of decompensated congestive heart

1 78

76.

77.

78.

79.

80.

81.

82 .

83.

84.

85 .

86.

87.

SECTION III



CONGESTIVE HEART FAILURE AND CARD I O MYOPATHY

failure: a randomized controlled trial. ]AMA 2002 ;287( 1 2) : 1 5 3 1-1 540. O'Connor CM, Starling RC, Hernandez AF, et al. Effect of nesiritide in patients with acute decompensated heart failure. N Engl ] Med 20 1 1 ;365( 1 ) :32-43 . Peacock WF, Hollander JE, Diercks DB, et al. Morphine and outcomes in acute decompensated heart failure: an ADHERE analysis. Emerg Med ]. 2008;25(4) :205-209. Oliva F, l..atinin R, Politi A, et al. Intermittent 6-month low dose dobutamine infusion in severe heart failure: DICE Multi­ center trial. Am Heart ]. 1 999 ; 1 38:247-253. Dies F, Krell MJ , Whitlow P, et al. Intermittent dobutamine in ambulatory outpatients with chronic cardiac failure. Circula­ tion. 1 986;74(suppl II) :II-38 Giamouzis G, Butler J, Starling RC, et al. Impact of dopamine infusion on renal function in hospitalized heart failure patients: results of the Dopamine in Acute Decompensated Heart Fail­ ure (DAD-HF) trial. ] Cardiac Fail . 2 0 1 0 ; 1 6 : 922-930. DeBacker D , Biston P, Devriendt J , et al. Comparison of do­ pamine and norepinephrine in the treatment of shock. N Engl J Med. 2 0 1 0 ;362(9) 779-789 Cuffe MS, Califf RM, Adams KF, et al. Short-term intravenous milrinone for acute exacerbation of chronic heart failure: a ran­ domized controlled trial. JAMA 2002;287( 1 2) : 1 54 1-1 547. Packer M, Carver JR, Rodeheffer RJ, et al. Effect of oral milri­ none on mortality in severe chronic heart failure. N Engl ] Med. 199 1 ;325: 1468-1475. Mebazaa A, Nieminen MS, Packer M, et al. Levosimendan vs do­ butamine for patients with acute decompensated heart failure the SURVIVE randomized trial. JAMA 2007;297(1 7): 1 883-189 1 . Gislason GH, Rasmussen JN, Abildstrom SZ, e t al. Increased mortality and cardiovascular morbidity associated with use of nonsteroidal anti-inflammatory drugs in chronic heart failure. Arch Intern Med. 2009 ; 1 69 : 14 1-149. Flather MD, Yusuf S , Kober L, et al. Long-term ACE-inhibitor therapy in patients with heart failure or left-ventricular systolic dysfunction: a systematic overview of data from individual pa­ tients. Lancet. 2000;355(92 1 5) : 1 575-158 1 . Waldo AL, Camm AJ, deRuyter Hans, et al. Effect of d-sotalol on mortality in patients with left ventricular dysfunction af­ ter recent and remote myocardial infarction. Lancet. 1996; 348 : 7- 1 2 .

88. Kober L, Torp-Pederson C, McMurray JJ , et al. Increased mortality after dronedarone therapy for severe heart failure. N Engl ] Med. 2008;358(25) :2678-2687. 89. Nissen SE, Wolski K Effect of rosiglitazone on the risk of myo­ cardial infarction and death from cardiovascular causes. N Engl ] Med. 2007;356:245 7-247 1 . 90. Klapholz M , Maurer M , Lowe AM, e t al. Hospitalization for heart failure in the presence of a normal left ventricular ejection fraction. ] Am Coll Cardio!. 2004;43(8) : 1432-1438. 9 1 . Owan TE, Hodge DO, Herges RM, et al. Trends in prevalence of and outcome of heart failure with preserved ejection frac­ tion. N Engl ] Med. 2006;355 :25 1-259. 92. Yancy CW, Lopatin M, Stevenson LW, et al. Clinical presen­ tation, management, and in-hospital outcomes of patients admitted with acute decompensated heart failure with pre­ served systolic function - a report from the Acute Decom­ pensated Heart Failure National Registry (ADHERE) database. JAMA 2006;4 7(1): 76-84. 93. Fonarow G, Stough WG, Abraham WT, et al. Characteristics, treatments, and outcomes of patients with preserved systolic function hospitalized for heart failure: a report from the OPTI­ MIZE-HF registry. J Am Coll Cardio!. 2007;50(8): 768-777 94. Yusuf S , Pfeffer MA, Swedberg K, et al. Effects of candesartan in patients with chronic heart failure and preserved left-ven­ tricular ejection fraction: the CHARM-Preserved trial. Lancet. 2003;362 :777-78 1 . 95. Massie BM, Carson PE, McMurray JJ, e t al. Irbesartan in pa­ tients with heart failure and preserved ejection fraction. N Engl J Med. 2008;359(23) :2456-2467 96. McAlister FA, Stewart S , Ferrua S, et al. Multidisciplinary strat­ egies for the management of heart failure patients at high risk for admission. ] Am Coll Cardio!. 2004;44(4) : 8 1 0-8 1 9 . 97. Adamson P B , Magalski A, Braunschweig F, e t a l . Ongoing right ventricular hemodynamics in heart failure: clinical value of measurements derived from an implantable monitoring sys­ tem. ] Am Coll Cardio!. 2003 ;4 1 (4) : 5 65-567. 98. Whellan DJ, Al-Khatib SM, Kloosterman EM, et al. Changes in intrathoracic fluid index predict subsequent adverse events: results of the multi-site Program to Access and Review Trend­ ing INformation and Evaluate CoRelation to Symptoms in Patients with Heart Failure (PARTNERS HF) Trial. J Card Fail. 2008; 14(9) : 799.

QU ESTIONS AND ANSWERS Questions

1 . Which of the fol lowi ng l ifestyle mod ifications has been demonstrated to decrease mortal ity i n heart fa i l u re (H F) patients? a. Sod i u m restriction to 1 .05) functional metabolic stress test showing a peak V02 � 12 mllkg/ min (If intolerant to beta-blockers � 14 mllkg/min) Intractable or severe anginal symptoms with no revas­ cularization options Intractable life-threatening arrhythmias unresponsive to medical therapy, catheter-based intervention, or implantable cardioverter/defibrillator

1 83

Recom mended Eva l uation Prior to Transplantation

Complete history and physical examination Laboratory data: Complete blood count (CBC) with differential, complete metabolic panel Thyroid function studies Liver function panel, creatinine clearance Lipid profile, hemoglobin Ale, urinalysis Cardiovascular data: Electrocardiogram, chest x-ray, echocardiogram Exercise test with oxygen consumption Right and left heart catheterization, myocardial biopsyi Immunologic data: Blood type and antibody screen HLA typing Panel of reactive antibodies (PRA) screen Serology for infectious diseases: Hepatitis HBsAg, HBsAb , HBcAb, HepCAb Herpes group virus Human immunodeficiency virus CMV IgG antibody Toxoplasmosis Varicella and rubella titers EBY IgG and IgM antibodies Venereal Disease Research Laboratory (VDRL) or Rapid Plasma Reagin (RPR) Vascular assessment: Carotid Dopplers Peripheral vascular assessment Abdominal ultrasound Ophthalmology exam0 Cancer screening: Prostate-specific antigen° Papanicolaou smear, mammography0 Colonoscopy0 Psychosocial evaluation: Support system Substance abuse history Psychiatric history Baseline: Dental examination Bone density scan Pulmonary function tests "If appropriate.

The patient also needs immunologic testing including ABO blood typing, tissue typing for determination of human leu­ kocyte antigens (HLAs) , and screening for existing anti-HLA antibodies (Panel Reactive Antigen test) . It is imperative that a careful psychosocial evaluation be performed to iden­ tify patients with substance abuse, noncompliance, or any

1 84

SECTION III



CONGESTIVE HEART FAILURE AND CARD I O MYOPATHY

behavioral trait that would lead to adverse posttransplant outcomes. The list of contraindications for transplant listing is evolving constantly Mancini and Lietz5 published a general list in 2010 and is presented in Table 1 7. 3 . There is no con­ sensus for certain relative contraindications and hence the list varies based on individual institution experience and preference.

Exclusion Criteria for Cardiac Transpla ntation Absol ute Contraind ications:

Any systemic illness with a life expectancy < 2 y: Active or recent solid organ or blood malignancy within 5 y AIDS with frequent opportunistic infections Active systemic lupus erythematosis, sarcoid and amyloid with multisystem involvement Irreversible renal or hepatic failure being considered for only heart transplant Significant obstructive pulmonary disease (FEV 1 < 1) Fixed Pulmonary Hypertension:

PASP > 60 mm Hg Mean TPG > 15 mm Hg Pulmonary vascular resistance > 6 Wood units Relative Contraind ications:

Age > 65-70 y Active infection except device related infection in VAD patients Active peptic ulcer disease Severe diabetes with end organ damage (neuropathy, retinopathy) Severe peripheral arterial disease not amenable to intervention Morbid obesity (BMI > 35 kg/m2 ) or cachexia (BMI < 1 8 kglm 2) Creatinine > 2 . 5 mg/dl or creatinine clearance < 25 mUmin Bilirubin > 2 . 5 mg/dl, serum transaminases > 3x normal, INR > 1 . 5 off warfarin Severe pulmonary dysfunction with FEV1 < 40% of normal Recent pulmonary infarction within 6-8 wk Difficult to control hypertension Irreversible neurologic or neuromuscular disorder Active mental illness or psychosocial instability Drug, tobacco, or alcohol abuse within 6 mo Heparin-induced thrombocytopenia within 1 00 d

A patient who qualifies for a cardiac transplant gets a priority based on the severity of illness. Hence, the patients on the transplant list are divided into Status lA (highest priority), defined as patients limited to the intensive care units who are dependent on mechanical circulatory sup­ port devices (mechanical assist device, intra-aortic balloon pump , extracorporeal membrane oxygenator) or high-dose intravenous inotropes plus Swan-Ganz catheter. Patients who are mechanically ventilated or have ventricular-assist device-related complications such as a thromboembolism or a device infection and those with a mechanical assist device for a 30 day period are also listed as Status lA. Status lB includes patients on continuous intravenous inotropes or patients with ventricular-assist devices once their 30 day lA time has expired. A patient who does not meet criteria for Status lA or lB is listed as Status 2 . Status 7 patients are those who are considered temporarily unsuitable to receive a transplant (Table 1 7.4) . Inpatients in need o f a transplant are more clinically obvious than ambulatory patients. Determining the appro­ priate timing for consideration for transplant listing for

U NOS Status Defi n itions

Status lA: A patient listed Status lA has at least one of

the following devices or therapies in place. (A) Mechanical circulatory support that includes one of the following: LVAD and/or RVAD (max 30 d) Total artificial heart Intra-aortic balloon pump ECMO (B) Mechanical circulatory support with objective medical evidence of significant device-related complications such as thromboembolism, device infection, mechanical failure, and/or life-threatening ventricular arrhythmias ( C) Mechanical ventilation (D) High-dose single intravenous inotrope, or multiple intravenous inotropes, in addition to pulmonary catheter in place Status l B: A patient listed as Status lB has at least one of the following devices or therapies in place. (A) LVAD and/or RVAD after 30 day Status lA time has expired. (B) Continuous infusion of intravenous inotropes Status 2: A patient listed as Status 2 is one who does not meet criteria of Status lA or lB. Status 7 : A patient listed as Status 7 is considered temporarily unsuitable to receive a transplant.

CHAPTER 1 7 • HEART TRAN SPLANTATION ambulatory patients can be complicated. A patient should be considered for transplant when the expected survival without transplantation is lower than after transplantation. Variables like ejection fraction, NYHA class, and etiology of heart failure do not predict outcomes in heart failure consist­ ently In ambulatory patients, one of the best predictors of survival is peak oxygen uptake (V0 2 ) measured by a cardio­ pulmonary exercise testing, which is an objective measure of functional status. The patient should achieve maximal exer­ cise, represented by respiratory exchange ratio (RER) > 1 .05 in order for the V0 2 to accurately predict outcomes. In cases where a maximal threshold is not achieved, the carbon diox­ ide ventilator equivalent ratio (VENC0 2) >35 can be used as a cutoff to refer for transplant listing. The ISHLT guidelines recommend repeating the test every 6 to 1 2 months to objec­ tively reassess the need to remain on the transplant list. 3 The goal is to list a patient for transplantation after all medical and surgical options have been exhausted, but before the patient becomes debilitated with end-organ damage that may compromise posttransplant survival. Figure 1 7 . 1 pro­ vides an algorithm for patient selection for transplant listing.

CARDIOG E N I C SHOCK I n itial P resentation

Worse n i n g of prior h e a rt fai l u re .

1 85

P O S T T R A N S P L A N T M A N AG E M E N T ISSUES Management of heart transplant patients broadly comprises monitoring for and management of the following: (a) allograft rejection, (b) complications and drug interactions of immu­ nosuppressive agents, (c) infections, and (d) malignancies.

Rejection The transplanted heart is identified as foreign by the recipi­ ents' immune system and is subj ect to a constant attempt at immune destruction. Cellular identity of self and nonself for an organism is mediated primarily through a group of anti­ gens called maj or histocompatibility complex (MHC) and expressed on cell surfaces. In humans, this group of proteins is named human leukocyte antigens (as they were first iden­ tified on leukocytes) . Rejection can involve both cellular and humoral (anti­ body-mediated) immune injury to the allograft and is often classified into four maj or types: hyperacute, acute cellular,

CHRON IC H EART FAI LURE Worsening symptoms/ Recurring hospitalizations with optimal medical therapy and no reversible causes.

Optimization of Management: -Hemodynamic assessment (Right Heart catheter) -Address reversible causes -l notropes and optimization of medications -Role of resynchronization therapy?

-Cardiopulmonary exercise test. -Survival assessment with S H FS and H FSS scoring system

-lnotrope dependent? -Unresponsive to medical therapy?

Transplant listing if: -V02< 1 0- 1 2 -SHFM 1 y Survival 50 1 5-20 25-35

a

65 0

"Cyclosporine or FK 506, azathioprine or mycophenolate-mofetil, and prednisone. bAntilymphocyte antibody.

period. These monoclonal antibodies have been mostly used during induction to delay the introduction of calcineurin inhibitors. This approach creates a window to improve renal dysfunction exacerbated by ischemic-reperfusion injury: Basiliximab and Daclizumab are considered to be nondeplet­ ing induction agents because they do not affect resting lym­ phocytes. Basiliximab is a chimeric antibody (30% murine, 70% human protein) , whereas Dacliximab is a humanized antibody (10% murine, 90% human protein) , both designed to be less immunogenic than a fully murine monoclonal antibody: The IL-2 receptor antibodies appear to offer some advantage in heart transplant recipients, including the lack of the cytokine release syndrome and no reported increased risk of infections or malignancies. In 2009 , most patients undergoing induction therapy received IL2R antagonist. 2 A newer agent Alemtuzumab , a humanized murine monoclonal antibody against CD52 (cell surface protein expressed on lymphocytes, NK cells, monocytes, and thy­ mocytes) , is being studied in heart transplant recipients as an induction agent.

I S H LT G U I DELI N ES3: I N D U CTION AG ENTS Class I I a: 1.

l m m u nosuppressive i n d uction with polyclonal a nti body prepa rations may be beneficial i n patients at high risk of renal dysfu n ction when used with the i ntent to delay o r avoid the use of a CNI (ca lcineurin i n h i b itor) . (Level of Evidence: B)

Class II b: 1.

Routi ne use of i m m u nosuppressive ind uction in a l l patients h a s n ot been shown t o b e s u perior t o i m m u n o­ suppressive reg imens that do not employ such thera py. (Level of Evidence: B)

2.

Class II b: l m m u nosuppressive ind uction with a nti­ thymocyte g l o b u l i n (ATG) may be beneficial i n patients at high risk for acute rejection. (Level of Evidence: C)

C:: H J\ PTER Calcineurin I n h ibitors In the early 1 980s, cyclosporine (CsA) , a lipophilic endaca­ peptide calcineurin inhibitor derived from a plant fungus, was first introduced as an immunosuppressant. Its use resulted in a dramatic reduction in the incidence of acute rejection in heart transplant recipients. Today, the calcineu­ rin inhibitors are the cornerstone of therapy after heart trans­ plantation, in conjunction with an antiproliferative agent and corticosteroids (CSs) (the so-called triple-drug therapy) . Calcineurin, a calcium-dependent serine-threonine­ phosphatase, is a vital enzyme in the transcription of IL-2 and other cytokine genes. The interaction of IL-2 with its receptor on activated T cells induces T-cell proliferation, which triggers the emergence of effector cells responsible for tissue destruction, resulting in clinical acute rejection. The T-cell receptor (TCR) is activated in response to alloantigens, causing an increase in intracellular calcium, which in turn activates the cytosolic protein, calmodulin. Ca2 • -calmodulin interacts with calcineurin, activating its phosphatase moiety Calcineurin is then able to dephosphorylate the nuclear fac­ tor of activated T cells (NFAT) . NFAT translocates into the nucleus and causes transcription of T-cell-dependent lym­ phokines, such as IL-2 (and its receptor) , interferon-y, and tumor necrosis factor-a. The calcineurin inhibitors (cyclosporine and tacrolimus) exert their effects by binding to cytosolic proteins called immunophilins upon entry into the T cell. Cyclosporin binds to cyclophilin and tacrolimus binds to FK-binding protein- 12 (FKBP- 12). Binding of cyclosporine and tacrolimus to its respective immunophilin enhances the immunophilins affin­ ity to calcineurin. The immunophilin-drug complex inhibits the phosphatase activity of calcineurin, thereby preventing translocation of NFAT into the nucleus and therefore pre­ venting the transcription of IL-2 and other cytokine genes. The early preparation of cyclosporine (Sandimmune) was oil based and its bioavailability was unpredictable due to vari­ ations in absorption and metabolism. Neoral, the new micro­ emulsion formulation of cyclosporine, has demonstrated greater bioavailability and more predictable pharmacokinet­ ics than Sandimmune and hence translated into less rejection episodes, lower required dose, and less treatment failures. 1 8 Cyclosporine is absorbed in the upper GI tract and the maj or­ ity is eliminated through metabolism in the cytochrome P450 system. It also inhibits CYP3A4 enzymes altering metabolism of medications being processed through this enzyme. The side effects and toxicities of cyclosporine include nephrotoxicity, hypertension, gingival hyperplasia, hir­ sutism, neuropathy, hyperlipidemia, and hyperkalemia. Drug-level monitoring is helpful in lessening the risk of toxicity while maintaining antirejection efficacy Target CsA levels are measured trough levels. Because rejection is more prevalent early posttransplantation, higher levels of CsA are generally targeted (Table 1 7 . 7). Tacrolimus (Tac) , formerly called FK506, is a highly immunosuppressive calcineurin inhibitor. Tac is about

1 7 • HEART TRAN SPLANTATION

1 91

100 times more potent than CsA. Like CsA, it is metabolized via the cytochrome p450 3A-4 system, and its intravenous dose is one-fourth to one-fifth of its oral dose. Toxici­ ties include nephrotoxicity, neurotoxicity, hyperuricemia, hypomagnesemia, gastrointestinal (GI) symptoms, diabetes, hyperkalemia, hyperlipidemia, and alopecia. Drug monitor­ ing is very important to lessen the toxic effects while main­ taining efficacy (see Table 1 7 . 7) . Clinical trials have shown mixed results when the performance of Tac and conventional CsA was compared among heart transplant patients in conjunction to receiving azathioprine, mycophenolate mofetil (MMF) , and steroids. Early studies showed no significant differences in outcomes between the two calcineurin inhibitors, while later studies revealed significantly lower 6 month rates of any treated rejection and ISHLT cellular grade 3A rejection episodes in the tacrolimus group. 1 9 Monotherapy with tacrolimus was recently evaluated in the TICTAC (Tacrolimus in Combina­ tion, Tacrolimus alone compared) 20 trial showing compara­ ble rejection rates in comparison to using a combination of Tac/MMF. Several studies have also shown the effectiveness of replacing CsA with Tac in cases of refractory rejection, gingival hyperplasia, or hirsutism. Patients treated with CsA have higher cholesterol and triglyceride levels, more hyper­ tension, cholelithiasis, gingival hyperplasia, and hirusitism compared to patients on Tac, while the latter had more dia­ betes mellitus, tremor, and anemia. 1 9• 2 1 •22 The calcineurin inhibitor chosen is often dependent on the patient, the side­ effect profile, and the institutional experience. In view of the side effect profile of the CNis and sug­ gestive benefit of antiproliferative agents in regard to allo­ graft coronary vasculopathy, newer trials are evaluating CNI sparing regimens.

I S H LT G U I DELI N ES3: C N I USE 1 . Lower levels of C N l s in HT should be sought when CNls

Class I:

a re used i n conj u nction with MMF beca use with this combination lower levels a re safe a n d associated with lower rejection rates as wel l as i m p roved renal fu nction. (Level of Evidence: B)

1 . Calcineurin i n h i bitor-based thera py remains the sta n­

Class Ila:

d a rd i n i m m u nosuppressive protocols used after hea rt transplant. (Level of Evidence: B)

1 . T h e resu lts o f c l i n i c a l tria ls suggest that TAC-based regi­

Class llb:

mens may be associated with lower rejection rate but n ot s u perior su rviva l after HT than cyclosporine based reg imens. ( Level of Evidence: B)

2.

C N I monothera py with early CS withdrawal may be considered in h i g h ly selected individ uals. ( Level of Evidence: B)

1 92

SE C:: ! I ()� III

.



. . ... . ... . ..I. .O... MYOPATHY .. ....... . .. . . ... . .. . . .. . . ... . ... . ....... . .. . ... . ...... . . ... . . . . . .. . ...... . .... c=()� GESTIVE HEA �T FAILURE AND CARD . .

Drug Thera py i n Card iac Transplantation

Drug

Ta rget Level (ng/ml) [m]

Cyclosporine

275-375 200-350 150-300 150-250

Tacrolimus

1 0- 1 5 [0-2] 8-12 [3-6] 5-1 0 [>6]

Mycophenolate-mofetil

Sirolimus Everolimus

Routine levels not recommended. Trough < 1 . 5 mg-considered low 4-12 3-8

Azathioprine

Levels not monitored

[0-1 .5] [ 1 . 5-3] [3-6] [>6]

i cyclo levels via cyt P450: erythromycin, ketoconazole, diltiazem, cimetidine -!. cyclo levels: isoniazid, rifampin, phenytoin I nteractions

Same as cyclosporine

-!. absorption in the presence of antacids containing magne­ sium or aluminum hydroxide Same as cyclosporine

Allopurinol-i levels

Antiproliferative Agents Azathioprine (Imuran) is a purine analog that impairs DNA synthesis and acts as an antiproliferative agent. It suppresses both T- and B-cell synthesis. It is well absorbed in the upper GI tract and metabolized in the liver. Some of its metabolites are broken down via xanthine oxidase. Hence, a xanthine oxidase inhibitor such as allopurinol can increase the azathioprine levels up to four times. The usual dose of azathioprine is 1 to 3 mg/kg/d, with the aim of keeping the white blood cell count >3 ,000 and the platelet count > 1 00,000. Myelosuppression is the major toxicity, and it is generally dose dependent. With­ drawal of azathioprine usually reverses myelosuppression within 7 to 10 days. Other side effects include hepatotoxicity and pancreatitis. Malignancies, especially cutaneous malig­ nancies, may be more common when compared to other, newer agents. Due to its side effect profile and available alter­ natives, the use of azathioprine has declined. 2 Randomized trials have also shown that MMF, evorolimus, and sirolimus have lower rates of CAY when compared to azathioprine. 23-2 5 MMF (Cellcept) , also an antiproliferative agent, blocks the de nova pathway of purine synthesis in T and

Toxicities

N ephrotoxcity Hypertension Hypomagnesemia Hypertrichosis Gout Gingival hyperplasia Hyperlipidemia N ephrotoxicity N eurotoxicity HTN < cyclosporine i lipids < cyclosporine Glucose intolerance Alopecia Diarrhea Anemia Diarrhea Nausea Hyperlipidemia Thrombocytopenia Anemia Neutropenia Diarrhea Myelosuppression, leukopenia Keep WBC >3 ,000/mL

B lymphocytes that lack a robust salvage pathway. Mycophe­ nolic acid (MPA) , a product of a Penicillin fungus, is the active metabolite of MMF It is readily absorbed across the GI tract; however, the absorption of MPA is decreased in the presence of antacids containing magnesium and aluminum hydrox­ ides. Toxicities of MMF include GI symptoms (nausea, vom­ iting, and diarrhea) and myelosuppression. The incidence of these adverse events is higher in patients receiving >3 g/d. Most symptoms will resolve with reduction of dose. Though many centers have MPA serum levels available, the relation­ ship between MPA levels and rejection remains unclear. Hence, the current guidelines recommend against routine monitoring of drug level to adjust dosing3 (see Table 1 7 . 7) . In specific situations where there is a suspicion o f low-drug exposure leading to rejection, a trough level can be used to tailor dose adjustment. The serum levels of MPA are higher when this drug is administered with Tac compared to CsA, thus it is advisable to empirically reduce the dosage of MMF when switching from CsA to Tac. Comparing MMF to aza­ thioprine on background of CsA and prednisone, there appears to be a 3-year survival advantage and a reduction of

CHAPTER 1 7 • HEART TRAN SPLANTATION graft loss to rejection with MMF 2 3 However, one trial showed more opportunistic infections, diarrhea, and esophagitis in the MMF group compared to AZA-treated patients. MMF has become the dominant antiproliferative agent used in clinical practice.

TOR I n h ibitors Rapamycin (Sirolimus) is a macrolide antibiotic with a simi­ lar structure to tacrolimus. It is in the class of immunosup­ pressants called target of rapamycin (TOR) inhibitors. The TOR enzyme is a cytoplasmic protein responsible for con­ necting signals from the surface of the T cell to the nucleus for stimulation of growth and proliferation of the T lympho­ cytes. Rapamycin binds to TOR and inhibits cell prolifera­ tion stimulated by growth factors. It is known to inhibit platelet-derived growth factor and basic fibroblast growth factors in the arterial smooth muscle cells and endothelial cells, respectively Studies have shown a decrease in the inci­ dence of coronary allograft vasculopathy in heart transplant recipients receiving this immunosuppressant. 2 5 Common side effects include hyperlipidemia and thrombocytope­ nia. When rapamycin is used alone there appears to be no adverse effects on kidney function; however, when it is used in combination with calcineurin inhibitors, there is a poten­ tiation of the calcineurin inhibitor-induced nephrotoxicity Therefore, the dose and target levels of the calcineurin inhib­ itor must be reduced substantially Everolimus (RAD: Certican) is a derivative of sirolimus with an identical mechanism of action. RAD, like rapamycin, inhibits clonal expansion of T cells but does not inhibit T-cell activation. It exerts its affects by forming a complex with FKBP- 12 to inhibit the cyclin-dependent kinases termed the TOR. This leads to Gl S-phase cell cycle arrest. When com­ pared to sirolimus, everolimus has a shorter half-life (30 hours compared to rapamycin at 60 hours) as well as a relatively higher bioavailability Like the calcineurin inhibitors, RAD and rapamycin are biotransformed through the cytochrome P450, 3A-4 system. Side effect profile is similar to Sirolimus and has also been associated with a significant reduction in allograft vasculopathy measured by IVUS at 1 year (7) . Corticosteroids CSs are important for induction, maintenance, and treat­ ment of rejection in heart transplant recipients. CSs have immunosuppressive and anti-inflammatory effects and affect

I S H LT G U I DELI N ES3: CORTI COSTERO I D USE 1 . CS withdrawa l ca n be successfu l ly ach ieved 3 to 6

Class I:

months after HT in many low risk patients. ( Level of Evidence: B)

1 . Corticosteroid avoidance, early CS wea n i n g or very low

Class Ila:

dose mai ntenance CS thera py a re a l l accepta ble thera­ peutic a pproaches. (Level of Evidence: B)

2.

If used, CS wea n i n g should be attem pted if there a re significant CS side effects and no recent rejection episodes. (Level of Evidence: C)

the number, distribution, and function of T cells, B cells, macrophages, as well as endothelial cells. The usual treat­ ment for moderate rejection (grade 2R) without hemody­ namic compromise is pulse-dose steroids (250 to 1 ,000 mg solumedrol intravenously daily for 3 days) . Most rejection episodes respond to initial therapy Steroids are associated with many side effects includ­ ing cataracts, diabetes, myopathy, osteopenia, growth retar­ dation in children, aseptic necrosis, hirsutism, cushingoid appearance, and dermatologic problems. They also exacer­ bate hypertension and hyperlipidemia, and cause adrenal insufficiency Thus, it is important to give stress doses of hydrocortisone when indicated (illness, surgical procedures) to patients on chronic CSs. Current practice favors with­ drawal of steroids to minimal dosage or none at 6 months to 1 year posttransplant, provided there are no rejection epi­ sodes. Yet, data from the ISHLT registry between 2007 and 2009 shows that 89% of patients are still on steroids at the end of 1 year and 52% at 5 years. 2 Typically, serial EBMs are performed to monitor for rejection while CS are withdrawn.

Combination Reg imen Most transplant centers initiate triple therapy with a CNI, an antiproliferative agent, and a CS. Figure 17.2 reflects the idea of using medications acting on different pathways of T-cell activation in order to achieve effective immunosuppression. In general, the various clinical trials have not shown signifi­ cant mortality benefit of one combination over the other. Tac­ rolimus, mycophenolate, and prednisone are currently the dominant immunosuppressive choices. The mTOR inhibitors

I S H LT G U I DE LI N ES3: ANTI PROLI F ERATIVE AG ENTS

I S H LT G U I DELI N ES3: CHOICE OF I M M U NOSU PRESSION AG ENTS

Class Ila:

Class 1 1 8 :

1.

MMF, EVL or SRL as tolerated should be incl uded i n conte m pora ry i m m u nosuppressive reg imens beca use thera pies including these drugs have been shown to red uce onset and progression of CAV (assessed with IVUS). (Level of Evidence: B)

1 93

1 . T h e adverse events o f i m m u nosuppressive d rugs

observed in ra ndom ized cli nical tria ls underscore the need for individ ual ization of i m m u nosu ppres­ sant accord i n g to the characteristics and risks of the individ ual recipient. (Level of Evidence: C)

1 94

SECTION III



CONGESTIVE HEART FAILURE AND CARD I O MYOPATHY

are not used widely due to their side effect profile. Sirolimus and evorolimus are used in 1 1 % of patients at 1 year and 23% at 5 years posttransplant. 2 P O S T T R A N S P L A N T CO M P L I C AT I O N S

Infection Preventing allograft rejection with immunosuppressive agents increases the risk of infection posttransplantation. Infections continue to be one of the leading causes of death after cardiac transplantation. Knowing the timetable of com­ mon infections following solid-organ transplant will aid in formulating a differential diagnosis and determining the tim­ ing of the various preventative strategies. A pretransplant infectious disease evaluation is used to identify any condition that would disqualify a poten­ tial recipient for transplantation, update immunizations, identify and treat active infections, and define the risk of infection in order to determine the strategy for preventing posttransplant infections. During the first 30 days after transplantation, there are generally three types of infections that occur: (a) active infection transmitted with the allograft, (b) untreated pre­ transplant infection in the recipient, and (c) nosocomial infections, which are commonly related to surgical wounds or indwelling catheters. More than 95% of the nosocomial infections during this period are bacterial or fungal (Candida species) . In contrast, late infections that occur 1 to 6 months following transplantation are generally caused by opportun­ istic organisms such as Pneumocystis carinii (PCP) , Aspergi!­ !osis species, Nocardia asteroids, and Listeria monocytogenes and viral infections such as CMV or EBV, which are by far the most common. After 6 months posttransplantation, most patients require decreasing levels of immunosuppression and thus their infectious disease risks become similar to those of the general population. The maj ority of patients require antiviral, antibacterial, and antifungal prophylaxis for 6 to 12 months posttransplantation. Approximately 5% to 10% of transplant recipients experience recurrent rejection episodes and thus are still at risk of developing opportunistic infec­ tions secondary to increased immunosuppressive therapy There are no specific guidelines for endocarditis proph­ ylaxis in heart transplant patients. Although the incidence of endocarditis is low in this population, the mortality rate is approximately 80% . Individuals who develop valvular heart disease have a higher risk than those who do not. The use of antibiotic prophylaxis for dental procedures in heart trans­ plant patients is considered reasonable. Vira l I nfections CMV remains the most important infection affecting the morbidity and mortality of heart transplant recipients. The serologic (presence of antibody to CMV) status of the donor and the recipient is a predominant predictor of posttrans­ plant CMV disease events. Donor seropositive (D+ )/recipient

seronegative (R-) bears the greatest risk of developing CMV clinical disease, which can present as leukocytopenia, pneu­ monia, colitis, gastritis, esophagitis, hepatitis, or myocardi­ tis. With D+/R- status, there is an increased incidence of tissue-invasive CMV, recurrent CMV, ganciclovir-resistant CMV, and CAV Patients at highest risk receive prophylaxis with oral valgancyclovir with or without CMV hyperim­ mune globulin (CMV-IVIG, Cryptogam) . Active CMV dis­ ease must be treated with intravenous gancyclovir with or without CMV-IVIG, depending on whether or not invasive CMV is present.

Fungal I nfections Candida species and Aspergillosis are the most common fun­ gal infections after transplantation. Oral clotrimazole or nystatin is used during the first 3 to 6 months or during periods of enhanced immunosuppression, when there is an increased risk of opportunistic infection. Protozoa! Infections Trimethoprim-sulfamethoxazole (TMP-SMX) is highly effec­ tive against PCP and Nocardia infections. Prior to the insti­ tution of PCP prophylaxis, approximately 10% of cardiac transplant recipients developed PCP, with a mortality rate up to 40% . Nowadays, with TMP-SMX prophylaxis, PCP is exceedingly rare. Toxoplasmosis is also a concern in heart transplant recipients. A Toxoplasma-seronegative recipient of a Toxoplasma-seropositive donor is at highest risk of devel­ oping toxoplasmosis posttransplant. Prophylaxis with TMP­ SMX is also effective posttransplantation and during episodes of increased immunosuppression therapy (steroid-resistant rejection) . Active toxoplasmosis can present as myocarditis and is treated with pyrimethamine and sulfonamide. Malignancies Cutaneous malignant lesions are the most common tumors after cardiac transplantation and account for nearly 40% of de nova cancers. 2 6 Posttransplantation, the incidence of squa­ mous cell carcinoma and basal cell carcinoma is increased, with basal cell carcinoma being the most common type (unlike the general population, in which squamous cell car­ cinoma is the most common type of skin cancer) . PTLD is a unique type of lymphoma that occurs in approximately 3.4% of all heart transplant recipients. 2 7 Approximately 90% of all PTLDs are associated with EBV These tumors are B-cell in origin and range from a benign polyclonal process to a highly malignant monoclonal lymphoma. Typically the tumor arises 12 to 18 months following transplant and is most commonly located intra­ abdominally The patient may have a mononucleosis-like presentation. Risk factors for developing PTLD include EBV­ seropositive donor to EBV-seronegative recipient, type of organ transplanted (lung and heart have the highest inci­ dence) , preceding CMV infection, and the level and type of immunosuppression used posttransplantation. PTLDs have

CHAPTER 1 7 • HEART TRAN SPLANTATION variable prognoses, with treatment strategies geared toward drastically decreasing background immunosuppressant drug therapy. This tactic may lead to a regression of PTLD in 23% to 50% of the patients. 2 7 Malignant lymphomas (even if EBY initiated) usually require cytotoxic chemotherapy as well, and despite aggressive therapy, have poor response rates (2 . 5 mg/ dL, while 4% are on chronic dialysis. 2 Risk factors for late renal dysfunction include chronic administration of calcineurin inhibitors (cyclosporine and tacrolimus) , preexisting renal dysfunction, diabetes, hyper­ tension, and generalized atherosclerosis. The renal toxicity associated with calcineurin inhibitors includes early func­ tional nephrotoxicity and late structural nephrotoxicity. The early form of nephrotoxicity occurs when calcineurin inhibitors are administered for the first time. The calcineurin inhibitors cause vasoconstriction of the afferent arterioles, resulting in a decrease in renal blood flow and a decrease in glomerular filtration rate. The late form of renal dysfunction is thought to be caused by a combination of the acute reno­ vascular effects plus direct toxic effects on renal tubular epi­ thelial cells. Cyclosporine has been shown experimentally to cause apoptosis in tubular and interstitial cells, potentially inducing tubular atrophy and subsequent fibrosis. Both early and late renal effects are dose related. The management of chronic renal insufficiency is to minimize the dosage of calcineurin inhibitors, which may or may not halt the pro­ gression of the renal dysfunction. Another alternative is to switch to a sirolimus-based regimen, which may have renal­ sparing effects if initiated before renal dysfunction is pro­ gressive. Kidney transplantation should be considered for end-stage renal dysfunction. Hypertension The maj ority of adult cardiac transplant recipients are diag­ nosed with arterial hypertension. The use of cyclosporine is linked directly to the development of posttransplant hyper­ tension. Three proposed mechanisms are direct sympathetic activation, increased responsiveness to circulating neurohor­ mones, and direct vascular effects. A common endpoint of

1 95

these proposed mechanisms is vasoconstriction of the renal vasculature, leading to sodium retention and an elevated plasma volume. Hypertension has been found to be less com­ mon in patients receiving tacrolimus than in those receiving cyclosporine. Steroids also play a role in the development of hypertension posttransplant. The mineralocorticoid activity causes sodium retention and also contributes to the increase in plasma volume. The denervated transplanted heart may not respond well to the increased after load, and persistent hypertension may lead to left ventricular hypertrophy and subsequent left ventricular systolic and diastolic dysfunction. Initial nonpharmacologic therapy should include sodium restriction. First-line pharmacologic agents include calcium channel blockers and angiotensin-converting enzyme (ACE) inhibitors. Calcium channel blockers are the most com­ monly used class of medications followed by ACE inhibitor. Diltiazem has the advantage of increasing the cyclosporine level by competing with cytochrome P450, thereby decreas­ ing the required cyclosporine dose. Monotherapy is effective in 5 years. Mechanical torsion on the tricuspid annulus due to the biatrial anastamosis accounts for at least mild-moderate regurgitation. The more anatomi­ cally correct, bicaval anastamotic technique has decreased the incidence but not prevented it. Recipients with preexisting pulmonary artery hypertension often experience acute right ventricular dysfunction and subsequent chronic dilation, which contributes to the regurgitation. More importantly, repeated EMBs with damage to the valve, chordal apparatus, and papillary muscles account for the majority of severe cases of TR. Generally, even severe TR is well tolerated, but in a small minority of patients with progressive right heart failure due to TR, tricuspid replacement is needed ( 1 2 months posttransplant is 8 to 1 0 ng/m l. The patient should discontinue tacrolimus u ntil target trough levels are obtai ned. This clinical sce­ nario could represent a rejection episode; however, with the g iven history, tacrolimus toxicity is most l i kely the cul prit.Tacrolimus taken with MMF is a common immu­ nosuppression reg imen.There is no increased incidence of renal fai l u re or episodes of cel l u l a r rejection with this pa rticular regimen. When ta king Tac with MMF, the MMF dose should be decreased to lessen the l i kelihood of developing MMF toxicity (myelosu ppression). The patient is defi n itely compliant, secondary to elevated trough levels of i m m u nosuppressants. 2. Answer C: The patient's episode of rejection is most l i kely secondary to discontin u i n g diltiazem 3 weeks ea rlier. Di ltiazem i n h i bits the cytochrome P450 system, ca using increased levels of cyclosporine and therefore req u i ring lesser dosages to achieve trough levels. Once di ltiazem was discontinued, the dose of cyclosporine should have been increased. Almost a l l rejection episodes G rade 3 R (3A) and higher and all rejection episodes that show hemodyna mica l ly instabil ity (rega rd l ess of g rade) a re treated with augmented

CHAPTER 1 7 • HEART TRAN SPLANTATION

i m m u nosu ppression. It is i m portant for transplant reci pients to com m u n icate any changes i n medications to the transplant clin ic, so these compl ications can be avoided. 3. Answer B: Risk factors for posttransplant mortal ity include but a re not l i m ited to short-term ventricu lar assist device use ( 4 Wood u nits), mechanical venti lation at the time of transpla nt, active i nfection, active smoki ng, dia betes particularly with end-organ damage, and hepatic or renal insufficiency. 4. Answer B: Serial echocardiograms are i mportant for fol lowing g raft fu nction posttransplant; however, they are not very sensitive for detecting early CAV. Periodic right ventricu lar biopsy is the standard method of survei llance for cell u l a r rejection, but adds little to the diagnosis of CAV. Positron emission scans are used to de­ tect ischemia, and sca rred and hibernati ng myocard i u m;

1 99

however, they suffer from poor sensitivity i n detecting CAV. Studies have shown i ntracoronary u ltrasound (IVUS) to be the most sensitive tool in detecting and fol lowing the prog ression of CAV, com pa red to coronary angio­ g ra ms. S. Answer A: PTLD is a unique polyclonal B-cell

lym phoma that occurs i n approxi mately 3.4% of a l l heart transplant recipients. Ninety percent of PTLDs are associ­ ated with EBV; with EBV D+/R- being a hig h-risk group for developing PTLD. These tumors usually a rise 1 2 to 1 8 months following tra nsplant, after a monon ucleosis­ l i ke i l l ness (fever, sore th roat, myalgias, and lym phade­ nopathy) and commonly are located i ntra-abdominal ly. Treatment incl udes decreasing the level of i m m u nosu­ pression, surgical debu l ki ng, cytotoxic che­ mothera py, and radiation thera py if indicated; however, the response rate of adva nced disease to treatment is poor ( 1 . 8 in patients not anticoagulated Hemodynamically significant aortic insufficiency • Active infection • • Psychosocial issues that might impair compliance • Irreversible and significant end-organ dysfunction • Significant "fixed" pulmonary hypertension • Highly calcified aorta (particularly at proposed site of outflow cannula anastomosis)

evidence suggests that in patients implanted with CF pumps, less acutely advanced heart failure patients have higher short- and long-term survival following LVAD implanta­ tion and shorter length of stays compared to patients who are more acutely ill.9 It is generally accepted that MCS is used to rescue patients with INTERMACS profiles 1 and 2 who are potential BTT candidates. There is now a shift to implant LVADs in inotrope-dependent, advanced heart fail­ ure patients with lower INTERMACS profiles. Studies are underway to further investigate their outcomes compared to those on chronic inotropic therapy: The first set of formal recommendations regarding patient selection for mechanical support was described

l ntermacs Levels

Profile 1 Profile 2 Profile 3 Profile 4 Profile 5 Profile 6 Profile 7

Critical cardiogenic shock Progressive decline Stable but inotrope dependent Resting symptoms Exertion intolerant Exertion limited Advanced NYHA III

. . . . . . . . .

. . . . . ...... . . .

in the consensus statement from the Conference on the Current Applications and Future Trial Design of the Mechanical Cardiac Support in 2ooo w A set of guidelines still utilized was published by the International Society for Heart and Lung Transplantation (ISHLT) in 2006 u Due to the varied experiences in the use of MCS as well as the many types of available devices, a single set of guidelines applica­ ble to all patients is not possible. However, the discussion below highlights the most important general considerations in selecting appropriate candidates for MCS for end-stage heart failure.

Age Age alone should not be a contraindication to MCS. In gen­ eral, patients over the age of 60 have more comorbidities than younger patients with an inverse survival relationship , but acceptable outcomes have been reported in multiple studies in select patients ::;70 years of age. 12- 1 4 Although policies vary by institution, if a durable assist device is implanted in a patient > 70 years old, they are generally for patients with the intention of DT or for select patients as a bridge-to-transplant. Body Habitus Because of the size of the larger, first-generation pulsa­ tile intracorporeal LVADs, it was recommended that these devices should not be implanted in patients with a body surface area of < 1 . 5 m2 to ensure adequate thoracic and abdominal capacity to accommodate the size of the device. These devices are rarely used in the current era of MCS. The new generation of CF pumps allows a larger group of indi­ viduals to benefit from MCS. Individuals whose body sur­ face area is >2 . 5 m2 or with an ideal body weight of > 1 50% may pose technical challenges intraoperatively. Currently; a BMl of >35 would not permit a patient to be a transplant candidate, although a device may be placed as a bridge to candidacy pending appropriate weight loss. There is limited experience regarding concurrent weight loss surgery to allow patients to then become transplant eligible, but a topic that we will hear more about in the future. Renal Function There have been multiple studies showing improvement of renal function following implantation of both pulsatile and CF devices in patients without intrinsic kidney disease. 1 5- 1 8 However, patients with preoperative creatinine ;::: 3 . 0 mgldL have worse outcomes following implantation. 1 9 Patients with creatinine values in this range may still be considered for MCS if renal injury is acute and is likely to be reversible. Patients with end-stage renal disease on dialysis are not can­ didates for MCS. There have been cases of patients being supported with VAD therapy that have progressive renal disease and require renal replacement therapy. This poses a much higher risk of morbidity and mortality, primarily due to infection.

C:I-_If\PTER

18 • DEVI CES FOR HEART FAILURE

205

Pulmonary Function Patients with severe underlying lung disease (baseline FEV1 3x the nor­ mal limit are at higher risk for morbidity. If known RV dys­ function is present with associated liver dysfunction, then the need for biventricular support should be considered in patients otherwise being considered for LVAD implanta­ tion. Patients with intrinsic cirrhosis or portal hypertension should not be considered candidates for MCS. In certain cases, a liver biopsy and hepatology evaluation may be use­ ful in differentiating primary cirrhosis from secondary con­ gestive hepatopathy. Patients with viral hepatitis are not a contraindication to MCS. Coag ulation Abnormalities Intra-and perioperative bleeding is a significant risk factor for morbidity and mortality following MCS implantation, and the identification of preoperative coagulation abnormal­ ities is essential. Patients with a spontaneous international normalizing ratio (INR) >2. 5 off any anticoagulation are at higher risk and generally not candidates for MCS . In addi­ tion, patients with perioperative heparin-induced thrombo­ cytopenia (HIT) are generally not considered candidates, but if MCS is considered, alternative methods of anticoagulation such as direct thrombin inhibitors should be used. Infectious Concerns Sepsis is one of the most common causes of morbidity and mortality following MCS implantation. If an infectious etiol­ ogy is discovered, aggressive anti-infective therapy should be administered and implantation, if possible, be delayed until the infection is cleared. In fact, a white blood cell count of > 1 0 ,000/µL has been found to be a risk factor of post­ implant mortality. 12 Patients considered for MCS therapy are more prone to develop fungal infections that are particularly difficult to eradicate. 2 0. 21 If a device gets implanted while a patient is actively infected, the offending organism can colo­ nize the hardware and will be unable to be eradicated even ' with long-term intravenous anti-infective therapy. Arrhythmias Unless there is a primary underlying proarrhythmic etiol­ ogy; such as giant cell myocarditis, ventricular arrhythmias

206

SECTION III • CONGESTIVE HEART FAILURE AND CARD I O MYOPATHY

often resolve following LVAD implantation, since wall stress is reduced and the LV is unloaded. LVADs, in the presence of normal pulmonary vascular resistance (PVR) , essentially produce a Fontan-like circulation allowing the RV to fill the pump despite even the most malignant dysrhythmias and is tolerated well clinically in MCS-supported patients n At this time, pacemakers and implantable cardioverter defibrillators (ICDs) maintain their usual indications in MCS patients, although little has been studied in this particular field.

Pulmonary Hypertension Elevated pulmonary pressures are frequently seen in patients with advanced heart failure. This may be due to chronic congestion, severe mitral insufficiency, and elevated pul­ monary capillary wedge pressure leading to remodeling of the pulmonary vasculature. Careful consideration must be made to identify whether the patient has pulmonary venous hypertension, pulmonary arterial hypertension, or a combination of both with a transpulmonary gradient > 1 5 , PVR > 3 WU, and an elevated pulmonary capillary wedge pressure. In some instances, an LVAD will be placed as DT, since the elevated PVR precludes listing for transplant. With medical therapy and unloading of the ventricle with the LVAD, there may be improvement in PVR and allow listing for transplant. Right Ventricular Function The evaluation of RV function deserves special attention in patients being considered for LVAD , especially in patients implanted with a second-generation CF device. RV failure following LVAD implant is an important cause of morbid­ ity and mortality for patients that often require prolonged inotropic therapy or ultimately placement of a RVAD . There have been a number of risk scores devised in an attempt to better predict which patients may develop RV failure after LVAD implantation, to allow for better medical opti­ mization prior to the LVAD implant or for planned simul­ taneous biventricular support. Fitzpatrick et al. performed a retrospective analysis from 266 patients who underwent LVAD implantation at a single center from 1 995 to 2007 and found that the multivariate risk factors predicting the need for biventricular mechanical support were cardiac index :"::: 2 . 2 Umin/m2 , RV stroke work index :"::: 0 .25 mm Hg xUm 2 , severe preoperative RV dysfunction, preoperative creati­ nine :2: 1 . 9 mgldl, previous cardiac surgery, and systolic blood pressure :"::: 9 6 mm Hg. 23 Multiple types of LVADs, the maj ority of which were pulsatile devices, were reviewed in that study: A recent review of patients implanted with a CF LVAD enrolled in the HM II LVAD BTT clinical trial showed that multivariate independent predictors of RV failure following LVAD implant included a central venous pressure/pulmonary capillary wedge pressure ratio >0 .63, need for preoperative ventilator support, and blood urea nitrogen level >39 mgldl. 2 4 The difficulty in consistently predicting RV failure lies, in part, on the unmasking of native RV dysfunction in the presence of restored cardiac output

and increased venous return following LVAD implantation. It is estimated that approximately 20% to 25% of patients undergoing LVAD implant develop RV failure. 2 5 This contin­ ues to be an area of active investigation.

Anatomic Considerations Perhaps one of the most important anatomic considerations prior to LVAD implantation is the competency of the aortic valve. Following LVAD implantation, the LV is unloaded and pressure rises in the aortic root with output from the device. Pathologic specimens either at time of transplant or autopsy may show commissural fusion of aortic leaflets leading to a central jet of aortic insufficiency: Mild to moderate aortic regurgitation prior to implant may progress to more severe grades of aortic regurgitation and worsening heart failure. This situation is best avoided by preemptively repairing at least moderate grades of aortic insufficiency at the time of LVAD implantation. Other important considerations include repairing severe mitral and tricuspid insufficiency and con­ verting mechanical aortic and mitral valves to bioprosthetic valves at the time of VAD implant. There is increased risk of thrombosis of the mitral valve due to a lower INR goal and in the aortic valve that may remain in the closed position due to VAD speed and lack of native contractility: Intera­ trial septal abnormalities such as a patent foramen ovale and atrial septal defect are generally closed at the time of surgery: Complex congenital heart anatomy and hypertrophic car­ diomyopathy may present surgical challenges that preclude LVAD implantation. N utritional Status Cardiac cachexia (BMI 1 . 1 , use of vasodilator therapy, mean pulmonary artery pressure �25 mm Hg, aspartate aminotransferase > 45 U/mL, hematocrit �34%, blood urea nitrogen >5 1 U/dL, and lack of intravenous inotrope use were all multivariate risk factors for 90-day in-hospital mortality following LVAD

207

implantation. It should be noted that these studies evaluated patients implanted with the pulsatile LVADs, although their applicabilities to those patients considered for CF LVAD implantation are likely to be similar and studies are currently under way to investigate this question.

O U TCO M E S A F T E R C I R C U LATO RY S U P P O RT I M P LA N TAT I O N Infection and bleeding were two of the most common adverse events seen post VAD implant in the 2005 Registry from the International Society for Heart and Lung Transplantation Mechanical Circulatory Support Device (ISHLT-MCSD) . 1 2 Table 1 8 . 6 summarizes the postimplantation patient-related events. Infection was seen in approximately one-third of patients with almost 30% having significant bleeding epi­ sodes. Neurologic dysfunction was observed in 14% of the registry patients. In 655 patients analyzed in the ISHLT-MCSD Regis­ try (entered between January 2002 and December 2004) , 1 -month survival was 83% and 1 2-month survival was 50%, sensoring patients at the time of transplantation. Age played a significant factor in outcomes, with an individual 40 years of age having a predicted 6-month mortality of about 5 % versus a 70-year-old patient with a predicted mortality of almost 20% after a LVAD alone was implanted. With the combination of a LVAD and RVAD during the same opera­ tion, for a 50-year-old individual, the mortality approaches 20% , compared to a little more 5% for an LVAD alone. In individuals under the age of 30 years receiving MCS as a bridge to transplant, 5 1 % were transplanted at the 6-month mark, with 33% alive and still waiting transplantation and 10% dying before transplant. This should be compared to those individuals >50 years of age who , at the 6-month mark, say only 39% transplanted, with a 33% pretransplant mortality and 27% of patients still waiting for transplant. Interestingly, though recovery sometimes occurs and MCS devices can be explanted, this is a rare event in patients >50 years of age, with only a 0.4% recovery-explantation rate noted over a 1 2-month period of time, compared to a 6-month explanted-recovery rate observed in patients Glycogen

I nflammatory (myocarditis) Stress-p rovoked ("tako-tsu bo") Peri partum Tachycardia-ind uced

storage

I nfants of insuli n-dependent diabetic mothers

Conduction Defects

Mitochondrial myopathies

Ion Channel Disorders



LOTS

Brugada S OTS CVPT

Asian SUNDS

group P antigen, which serves as a cellular receptor fo r this virus. There fore, initial parvovirus infection o f endothelial cells o f small coronary arteries may cause end othelial dys­ function, vasospasm, and ischemia. This may b e a co fa ctor fo r myocardial damage progression and cou ld also mimic myocardial infarction presentation, what has b een calle d "parvomyocarditis. "22 Interestingly, other investigators have not found histopatho logic signs o f chronic ischemic disease like sub end ocardial fib rosis and vacuolization on end omyo­ cardial b iopsy o f HF patients with a positive tissue PCR fo r PVB 1 9 . These findings contra dict the theory o f end othelial an d ischemic damage relate d to PVB 1 9 u

Clinical Presentation Myocarditis can b e totally asymptomatic or can present with a chest pain syndrome ranging from the mild persistent chest pain o f acute myopericarditis (35%) to severe symptoms that resemble myocardial ischemia. 4 Ab out 60% o f patients may have history o f arthralgias, malaise, fevers, sweats, or chills consistent with viral infections (pharyngitis, tonsil­ litis, upper respiratory tract infection) usually ab out 1 to 2 weeks prior to onset. The hallmark symptoms o f acute or ful­ minant myocarditis are those o f acute-onset HF in a person without known cardiac dys function or with low cardiovas­ cular risks. The diagnosis is usually presumptive, b ase d on patient demographics and the clinical course (spontaneous recovery following supportive care or death) . Patients with an acute HF presentation usually will have tachycardia with a S3 gall op, jugular venous distension, and peripheral e dema.

FIGURE 1 9.1 Classification of primary cardiomyopathies predominantly involvi ng the hea rt. (From Maron BJ, Towbin JA, Thiene G, et al. Contempo­ ra ry defi n itions and classification of the cardiomyopathies: an America n Heart Association Scientific Statement from the Cou ncil on Clinica l Cardiol­ ogy, Heart Fa i l u re and Tra nspla ntation Com m ittee; Quality of Ca re and Outcomes Research and Functional Genom ics and Translational Biology I nterdisciplinary Working Grou ps; and Cou ncil on Epidemiology and Prevention. Circulation. 2006; 1 1 3: 1 807- 1 8 1 6, with permission.)

An au dible pericardial friction rub may accompany in cases o f myopericarditis. In some instances, patients may present with arrhythmia in the fo rm o f palpitations cause d by supraven­ tricular or ventricu lar tachyarrhythmia, syncope cause d by heart block ("Stokes-Adams attack" ) or su dden cardiac death . Additional findings may accompany specific fo rms o f myocarditis. In patients with acute rheumatic fever, associate d signs inclu de erythema marginatum, po lyarthralgia, chorea, and sub cutaneous no dules Q ones criteria fo r rheumatic fever) . In cases o f sarcoid myocarditis, lymphadenopathy and arrhythmias are common (up to 70% o f a ffecte d indi­ viduals) . Chagas acute infection may present with arrhyth­ mias and cardiac conduction abnormalities. Hypersensitive or eosinophilic myocarditis is o ft en associate d with a pruritic macul opapular rash (and history o f o ffending drug use) and eosinophilia in their bl oo d work . The typical presentation o f a patient with giant-cell myocarditis invo lves sustaine d ven­ tricu lar tachycardia and rapidly progressive HF lea ding to cardiogenic shock . These features have l ow speci ficities but are o ften use ful and may raise the suspicion o f underlying myocarditis.

Eval uation Infl ammation is the hallmark feature o f myocarditis. Clini­ cally, an early onset o f fever, tachycardia, hypotension, re duce d ventricu lar fu nction, el evate d acute phase reac­ tants (erythrocyte se dimentation rate or C-reactive protein) , leu kocytosis, an d increase d cardiac enzymes ( CK-MB/car­ diac troponins ) are pre dictive o f myocarditis. However, the

CHAPTER 1 9 • MYO CARDITIS AND DILATED CARDIOMYOPATHY 2 1 7 prevalence o f an increase d troponin T in b iopsy-proven myocarditis is only 35 % to 45 % . 23 A l ower level o f troponin I at a dmission has b een associate d with an increase d risk fo r death , heart transp l antation, or persistent ventricu lar dys function in patients with fu lminant myocarditis. 24 The presence o f eosinop hilia may suggest hypersensitive (eosin­ op hilic) myocarditis. Novel infl ammatory markers that are still un der investigation inclu de tumor necrosis fa ctor (TNF ) -a, interleu kin ( IL) - 1 0 , serum-so lubl e Fas, an d so l­ uble Fas- ligan d levels. 2 5 . 26 E levation o f these markers por­ ten ds a worse prognosis, al though they are rarely use d in the clinical setting. Serum viral antib o dy titers are usually increase d fo urfo ld or more in the acute p hase an d gra dually fall during convalescence. However, measurement o f vira l antib o dy titers is infrequently indicate d due to the usual l ow viral l evels at the time o f HF presentation an d the lack o f evi­ dence for antiviral therapy: Because o f their l ow specifi city, measurement o f anticardiac antib o dy titers is not in dicate d (only 62 % o f myocarditis cases have titers � 1 :40 ) . Screen­ ing antinuclear antib o dies an d rheumatoi d fa ctor are o ften in dicate d to ru l e out common rheumato l ogic problems. Disease-speci fi c testing is in dicate d i f particu l ar conditions such as systematic lupus erythematosus, po lymyositis, Weg­ ner granu l omatosus, or sclero derma are suspecte d . The electrocardiogram generally reveals sinus tachy­ d car ia, although sometimes ST-segment deviation can b e found , making it necessary to rule out ischemia especially in patients with cardiovascu lar risk factors. In some cases, fascicular block , atrioventricular conduction disturb ances, or ventricul ar tachyarrhythmias may b e hemo dynamically significant. A comp lete echocardiogram is a standard proce­ dure fo r patients with suspecte d myocarditis to (a) exclu de al ternative causes o f HF-like valvular disease, (b ) quantify the degree o f le ft ventricular dys function to monitor response to therapy, and (c) detect the presence o f intracardiac thrombi. Occasionally, fo cal wall motion abnormalities and presence o f pericardial fluid may prompt further workup or interven­ tion. Fulminant myocarditis is o ften characterize d by near normal diastolic dimensions and increase d septal wall thick­ ness, whereas acute myocarditis o ften has increase d diastolic dimensions but normal septal wall thickness. 27 Coronary angiography is o ften per fo rme d to rule out coronary disease as cause o f new-onset HF in patients with risk factors or with a clinical presentation that mimic myocardial ischemia ("pseu­ d oinfarct pattern" ) . This is especially relevant in the presence o f fo cal wall motion abnormalities on echocardiography and l ocalizing electrocardiograp hic changes. Several specialize d imaging proce dures are available to detect the presence o f myocarditis, although they are rarely use d clinically: Antimy­ osin scintigraphy using indium-III monoclonal antimyosin antib o dy provides identifi cation o f myocardial inflammation, with a high sensitivity (9 1 % to 1 00 %) and negative pre dic­ tive value (93 % to 1 00 %) but relatively low speci ficity (28 % to 33 %) o f detecting myocarditis. Gallium scanning has b een utilize d to identi fy severe myocardial cellular infiltration with high speci ficity (98 %) but l ow sensitivity (36 %) . 4

Ga d olinium-enhance d cardiac magnetic resonance imaging (MRI ) is b eing use d with increasing frequency fo r noninvasive evaluation o f patients with suspecte d myocardi­ tis. Cardiac MRI can assess different markers o f tissue injury, inclu ding intracellular and interstitial e dema, capillary l eak­ age with hyperemia, and cellular necrosis with fib rosis. The International Group on Cardiovascular Magnetic Resonance in Myocarditis recommended to perform a cardiac MRI when the patient was symptomatic, if the clinical suspicion o f myo­ carditis was high , and if the MRI result will likely impact clin­ ical management. The authors propose d three tissue markers (the "Lake Louise Criteria" ) to confirm the diagnosis o f myo­ carditis (Table 19.3 ) . I f all sequences can b e perfo rme d an d two or more o f the three tissue-b ase d criteria are positive, myocardial inflammation can b e pre dicte d with a diagnostic accuracy o f 78 % ; if only late gad olinium enhancement imag­ ing is perfo rme d , the diagnostic accuracy falls to 68 % . 28 Histology remains the go ld standard fo r the diagnosis o f myocarditis, although endomyocardial b iopsy is insensitive and not without risks. False negative rates are high29 (even with multip le biopsy samples) due to the small numb er o f lymphocytes reviewe d , difficulties in distinguishing cell types,

Cardiac M R I Diagnostic Criteria for Myocarditis •

In the setting o f clinically suspecte d myocarditis, cardiac MRI findings are consistent with myocardial inflammation, if at least two o f the foll owing criteria are present: 1 . Regional or gl ob al myocardial SI increase in Tl-weighte d images 2. Increased global myocardial early gadolinium enhancement ratio between myocardium and skeletal muscle in gadolinium-enhance d T l -weighted images 3 . There is at least one focal lesion with nonischemic regional distribution in inversion recovery-pre­ pare d ga do linium-enhance d Tl-weighte d images (" late ga d olinium enhancement" ) • A cardiac MRI stu dy is consistent with myocyte injury and/or scar caused by myocardial inflammation if late ga dolinium enhancement (criterion 3 ) is present • A repeat MRI stu dy b etween 1 an d 2 wk a fter th e initial assessment is recommende d i f: 1 . none o f the criteria are present, but the onset o f symptoms has b een very recent and there is strong clinical evid ence fo r myocardial inflammation. 2. one o f the criteria is present • The presence o f LV dys function or pericardial e ffusion provides additional supportive evidence for myocarditis Reprinted from Friedrich MG, Sechtem U, Schulz-Menger ] , et al. Cardiovascular magnetic resonance in myocarditis: A JACC White Paper. ] Am Coll Cardiol. 2009 ; 5 3 ( 1 7 ) : 14 75-1487, with permission from Elsevier.

218

SECTION III



CONGESTIVE HEART FAILURE AND CARDIOMYOPATHY

wide interobserver variability, and the patchy distribution o f myocardial inflammation in most patients. Gui de d biopsy using delay enhancement in contrast MRI was evaluate d by Mahrholdt et al . , where 2 1 patients in whom biopsy was ob taine d from the region o f contrast enhancement. In these patients, histopathol ogic analysis reveale d active myocarditis in 1 9 patients (PVB 1 9 , n 1 2 ; human herpes virus type 6, n 5). In contrast, the remaining 11 patients in whom biopsy cou ld not b e taken from the region o f contrast enhancement, active myocarditis was found only in one case. 30 In patients with suspecte d myocarditis, end omyocar­ dial b iopsy is generally reserve d fo r suspecte d etio l ogy in wh om a positive histo logic diagnosis will determine a spe­ cific treatment, such as giant-cell an d sarcoi d myocarditis. Giant-cell myocarditis, which can b e suspecte d in subjects presenting with rapi dly progressive HF symptoms despite conventional therapy and new-onset frequent ventricu lar tachyrrhythmia or conduction disturb ances, could b ene fit from immunosuppressive therapy In most cases, the histo­ logic criteria only provi de con firmation o f the diagnosis and perhaps some prognostic in fo rmation. It is also important to recognize that as the interval from illness onset to the time o f the b iopsy increases, the yield o f the proce dure gets lower. A negative end omyocardial b iopsy can not convinc­ ingly exclu de underlying myocarditis or sarcoi dosis due to its patchy process an d b iopsy samp ling error an d there fo re clinical correlation is necessary =

=

Treatment and Prognosis There are no hard -an d- fast ru les fo r managing myocarditis once the acute events have occurre d . In general , patients are treate d in the same manner as i f they have chronic HF Clinical foll ow-up shou ld b e cl ose, as persistent chronic infl ammation may lea d to dilate d cardiomyopathy (initially 1 to 3 month intervals fo r me dication an d physical activ­ ity titration) . Serial echocardiographic assessment o f ven­ tricu l ar structure and function is o ften per fo rme d , al though there is no agreement regarding the frequency o f echocar­ diographic assessment fo ll owing myocarditis. There is a theoretical increase d risk o f myocardial in flammation an d necrosis, cardiac remo deling, an d death with exercise in animal mo dels. 31 There fo re, patients su ffering from myocar­ ditis are usually a dvise d to ab stain from vigorous exercise fo r several months in order to limit myocardial deman ds. Depending on the clinical presentation, stand ard HF ther­ apy with diuretics, angiotensin-converting enzyme (ACE) inhibitors, b eta-bl ockers, an d ald osterone antagonists shou ld b e use d to del ay or reverse disease progression o f cardiac dys fu nction. Although not proven in human stu d­ ies, proarrhythmic properties o f digoxin have b een ob serve d in animal mo dels o f myocarditis, and there fo re shou ld b e avoi de d . Anticoagu lation is usually recommende d and indi­ cate d to prevent thromb oemb o lic events in patients with apical aneurysm and thrombus (such as in Chagas cardio­ myopathy) , atrial fib rill ation, and prior emb o lic episo des. Permanent pacemakers shou ld b e imp lante d fo r persistent

heart block or b ra dyarrhythmia. Imp lantabl e cardioverter d e fib rillators ( ICDs) are in dicate d only a fter acute phase reso lution, in patients with primary and second ary in dica­ tion o f su dd en cardiac d eath prevention. Inotropic therapy o ften is reserve d fo r those experiencing severe hemo dy­ namic compromise (particu larly in fulminant myocarditis) . Sometimes, intra-aortic b all oon counterpu lsation can b e use d for hemo dynamic support an d a fterl oa d re duction to prevent fu rther deterioration. Mechanical assist devices (l e ft ventricu lar assist device [ LVAD ] ) and even extracorporeal memb rane oxygenation (ECMO ) have b een use d in cases o f fulminant myocarditis with the h ope for recovery and/ or b ri d ge to transp lantation. Early consi deration fo r car­ diac transp lantation shou ld b e given especially in severe, progressive, b iopsy-proven giant-cell myocarditis an d peri­ partum cardiomyopathy Registry data have suggeste d that patients with myocarditis may have increase d rej ection and re duce d survival a fter heart transp l antation as compare d to th ose with out, and myocarditis may recur in the all ogra ft in less severe fo rms. Table 1 9 .4 summarizes the major clinical trials on immunosuppression therapy fo r myocarditis and inflamma­ tory cardiomyopathy Routine immunosuppression therapy (inclu ding steroi ds) , antiviral regimen, and nonsteroid al anti-inflammatory agents are not warrante d b ase d on current clinical evi dence. The fin dings from the Myocarditis Treat­ ment Trial (with oral pre dnisone and cyclosporine)32 and the IMAC stu dy (with intravenous immunogl obulin) 5 indi­ cate d that routine immunosuppression therapy may not b e e ffective. The more recently publishe d TIMIC-ran domize d controlle d trial showe d positive results with pre dnisone and azathioprine therapy in 85 patients with virus-negative chronic inflammatory cardiomyopathy, where 88% o f them improve d their l eft ventricular ejection fraction (LVEF) and ha d reverse cardiac remo deling a fter 6 months o f treat­ ment. 33 It seems that the duration o f symptoms appears to b e a major determinant in the response to immunosuppression. A recent meta-analysis o f immunosuppression in patients with IDCM suggeste d that acute HF (symptoms duration o f 50 mm Hg but no or only mild symptoms are generally treated medically until more severe symptoms manifest. Young patients with marked LVOT obstruction (gradient ?: 75 mm Hg) should be considered for septal myectomy despite the lack of signifi­ cant symptoms. In assessing risk and benefit of septal myec­ tomy, the young age of this subgroup decreases the operative risk. Septal myectomy is not indicated in midcavity obstruc­ tion. However, in one study, patients with apical hypertro­ phy complicated by progressive, drug-refractory diastolic heart failure with severely limiting symptoms experienced improved functional status following apical myectomy 40 Septal myectomy involves resecting part of the proxi­ mal septum through an aortotomy so that the outflow tract obstruction is lessened (Fig. 22 . 1 2) . Sometimes myectomy may be combined with other cardiac surgery such as coro­ nary artery bypass surgery, mitral valve repair, or mitral valve replacement. Operative mortality for isolated myectomy is low, at approximately 0% to 4%. 4 1-45 Increasing age and concomitant

CHAPTER 2 2 • HYPERTROPHIC CARDIO MYOPATHY

255

population. Although randomized comparisons are needed, nonrandomized data suggest that survival may actually be improved in HCM patients who undergo myectomy. 44 Preexisting conduction abnormalities influence the like­ lihood of needing permanent pacemakers postmyectomy. Left bundle branch block is common after surgical myec­ tomy, occurring in 93% of subjects.52 Thus, subj ects with preexisting right bundle branch block are at high risk for requiring a permanent pacemaker postmyectomy. In sub­ jects with normal conduction systems on ECG, there was a 2 % rate of permanent pacemaker implantation postmy­ ectomy, whereas for patients with preexisting conduction abnormalities, there was a 10% incidence of permanent pacemaker implantation.5 2

Percutaneous Alcohol Septal Ablation For patients with medically refractory HCM and resting or provocative gradients :2'.50 mm Hg who are poor surgical candidates or for those who choose not to undergo open heart surgery, alcohol septal ablation is another option (Fig. 22. 1 3) . In the late 1 990s, unbridled enthusiasm for alcohol ablation resulted in the fact that by 2000 , >3 ,000 alcohol ablations had been performed for HCM, more than

FIGURE 22.1 2

Septa I myectomy. Septa I myectomy i nvolves resecting a portion of the proximal septum.

cardiac procedures may increase the surgical risk. Septal myectomy is associated with high success rates in decreas­ ing LVOT gradients46-48 and in improving symptoms46•4B-50 and exercise capacity. 47 Symptom improvement occurs from decreasing the LVOT gradient as well as decreasing the sever­ ity of any associated mitral regurgitation. Results postmyec­ tomy are durable. Rarely is reoperation needed secondary to recurrence of LVOT obstruction. 4 Long-term survival in HCM patients undergoing iso­ lated myectomy is 93% to 96% at 5 years and 83% to 87% at 1 0 years. 44•51 Multivariate predictors of overall mortal­ ity include age :2'.50 years at time of surgery, concomitant coronary artery bypass graft surgery, female gender, history of preoperative atrial fibrillation, and left atrial diameter of :2'.46 mm. 45 For patients undergoing a myectomy combined with other cardiac surgery, primarily coronary artery bypass graft surgery or valve surgery, 5-year survival was 80% and 1 0-year survival was also 80% .5 1 Retrospective, nonrandomized data suggest that long-term survival for HCM subjects undergoing myec­ tomy does not differ significantly when compared to the age- and sex-matched general population. 44 Furthermore, myectomy patients had higher survival rates than obstruc­ tive HCM patients who did not undergo surgery. 44 Thus, myectomy patients appear to fare no worse than the general

FIGURE 22.1 3 Alcohol septa l a blation. In a lcohol septa l abla­ tion, a bal loon i s inflated in the proxi m a l septa I perforator a n d a lcohol is i njected i nto the septa l a rtery through the dista l port of the bal loon.The goal is to create a control led myoca rd ial i nfa rction of the proximal septum, resulting in shrinkage of the septu m a n d lessening of the LVOT obstruction.

256

S E C T I O N III • CONGESTIVE HEART FAILURE A N D CARD IO MYOPATHY

the number of myectomies performed since the introduction of myectomies approximately 40 years ago .53 This optimism has been tempered and presently, alcohol ablation is con­ sidered second-line therapy behind myectomy for medically refractory, obstructive HCM.54 Review of the patient's cardiac anatomy is critical in selecting subjects for alcohol ablation. In order for alcohol ablation to succeed, LVOT obstruction needs to be second­ ary to contact of the mitral valve with the proximal septum. If the LVOT obstruction actually occurs in the mid-distal LV cavity, then alcohol ablation will not be of benefit. Because of the lack of randomized controlled trials and a suitable control population, alcohol ablation has not been shown to improve survival. Alcohol ablation does result in decreased LVOT gradients and an improvement in symp­ toms, 21 ·55·56 with persistence of benefit at 2 to 3 years.57 Effec­ tiveness of alcohol ablation extends to include the elderly population.58 There is also a decrease in LV filling pressures59 and a decrease in septal thickness. 56·60 In 3-month follow-up data, we reported a decrease in LVOT gradient from 64 to 28 mm Hg and an improvement in New York Heart Associa­ tion (NYHA) class from 3 . 5 to 1 . 9 after alcohol ablation.56 Predictors of unsatisfactory outcomes after alcohol septal ablation include a residual LVOT gradient after ablation of >25 mm Hg in the cardiac catheterization lab as well as a peak creatinine kinase (CK) of < 1 ,300 U/L6 1 At the Cleveland Clinic, most alcohol ablations have been performed on elderly, suboptimal surgical candidates. We generally prefer that the septum be between 1 .8 and 2 . 5 cm, to provide a safety margin; if the septum is too thick, favorable ablation results may be difficult to attain, whereas if it is too thin, the patient is at higher risk for development of a ventricular septal defect. A septum < 1 . 8 cm thick in a patient with the clinical picture of HCM often indicates that

mitral valve abnormalities, such as long leaflets, abnormal insertion of the papillary muscles, or anterior displacement of the mitral valve apparatus may be the primary etiology for the LVOT obstruction. Such mitral valve abnormalities contraindicate alcohol septal ablation. Complications of alcohol ablation include right bundle branch block,5 2 ·62 ·63 complete heart block (requiring a per­ manent pacemaker) , a large anterior wall myocardial infarc­ tion, ventricular tachycardia or fibrillation, and pericarditis. The risks of alcohol ablation include a 2% to 4% procedural mortality rate and a 9% to 27% incidence of patients requir­ ing permanent pacemakers.5 2 ,57,59,60,63,64 Alcohol ablation, unlike septal myectomy, results in myocardial scar. Thus, a theoretical risk exists that alcohol ablation may increase the risk of SCD , especially in light of the fact that an arrhythmogenic substrate is already present with HCM. One study of 123 HCM patients who already had implantable cardioverter-defibrillators (ICDs) for primary prevention of SCD and were undergoing alcohol ablation found that alcohol ablation was not proarrhythmic. 65 This is in contrast to other published studies that have shown that ICD therapy was fourfold more common after alcohol ablation than following the more established surgical myec­ tomy.4 However, SCD has been reported several months after successful alcohol ablation. 66

Com parison of Septa I Myectomy and Alcohol Ablation Overall, comparisons between myectomy and alcohol ablation indicate that both are effective in reducing LVOT gradient and improving symptoms, but the procedural complication rate exceeds that of myectomy67 (Table 22.3). A comparison of the two modalities at the Cleveland Clinic

TA B L E Com pa rison of Septa l Myectomy and Percuta neous Alcohol Septa l Ablation Surgical Myectomy

Percuta neous Alcohol Septa l Ablation

Invasiveness Onset of reduction in LVOT gradient

Invasive Instantaneous

Procedural mortality4·77 Effect on LVOT gradient4·77 Conduction abnormality postprocedure54 Recovery time Need for permanent pacemaker-all patients54·60 Need for permanent pacemaker if no preexisting conduction abnormalities54 Length of follow-up Success rate

1 %-2% Decreases to < 1 0 mm Hg Left bundle branch block 1 wk 3%-10%

Less invasive Some effect instantly, but more often 6-1 2 mo for full effect 1 %-2% Decreases to 95 %

6-8 y >85 %

CHAPTER 2 2 • HYPERTROPHIC CARDIO MYOPATHY suggested slight superiority of myectomy o n the basis of larger and more consistent reductions in LVOT gradient.56 This nonrandomized study of 5 1 HCM patients who under­ went either myectomy or alcohol ablation found that of the 26 patients who underwent septal myectomy, LVOT gradient was significantly reduced, from 62 mm Hg premyectomy to 7 mm Hg postmyectomy In the 25 alcohol ablation subjects, LVOT gradient was significantly reduced from 64 mm Hg preablation to 28 mm Hg. New York Heart Association class improved significantly, from 3 . 3 to 1 . 5 in the myectomy group and from 3 . 5 to 1 .9 in the alcohol ablation group.56 In this study, five patients underwent myectomy secondary to persistent provocable gradients from alcohol ablation. A recent meta-analysis of septal myectomy versus alco­ hol ablation concluded that there is a similar mortality rate and functional status between the two procedures, but with increased conduction abnormalities and a higher postinter­ vention LVOT gradient.68 Another nonrandomized cohort study of 44 patients found similar improvements in LVOT gradients and NYHA classification after either myectomy or ablation.69 However, in this study, myectomy was noted to have superior results with respect to exercise parameters, including peak oxygen consumption and peak work rate achieved. A nonrandomized study compared 4 1 alcohol ablation patients from Baylor to an age- and gradient-matched cohort of myectomy patients performed at the Mayo Clinic.64 The functional and hemodynamic changes after 1 year were simi­ lar in the two groups, although the alcohol ablation group did have a significantly higher incidence of permanent pacing.64 Associated severe coronary artery disease or valvular abnormalities that warrant surgical intervention are factors that further tip the balance toward myectomy over alcohol ablation, for concomitant cardiac surgery can be performed at the time of myectomy It is crucial to completely assess the degree and etiology of any mitral regurgitation that exists. A subject with HCM and severe mitral regurgitation second­ ary to SAM could potentially be a candidate for either septal myectomy or alcohol ablation if the primary abnormality is the septal thickness, and it is believed that reducing the septal thickness will alleviate the SAM. In contrast, a sub­ ject with HCM and severe mitral regurgitation secondary to intrinsic valvular abnormalities would not be a good candi­ date for alcohol ablation because in this instance, decreasing the septal thickness would not positively impact the mitral regurgitation.

Permanent Pacemaker I m plantation Pacemaker implantation has been used historically to alle­ viate the symptoms of HCM, but this procedure has fallen out of favor. It was hypothesized that initiating ventricular contraction at the right ventricular apex and distal sep­ tum would alter the sequence of ventricular contraction such that the outflow gradient would be decreased and

257

symptoms improved. However, this was not borne out in double-blind, randomized crossover trials.70.7 1 Furthermore, in a nonrandomized, concurrent cohort study of 39 patients who underwent either surgical myectomy or received per­ manent pacemakers, surgical patients demonstrated larger decreases in LVOT gradient (76 to 9 mm Hg versus 77 to 55 mm Hg) and larger improvements in symptoms and exer­ cise duration.72 I N F ECTIVE E N DOCARDITIS P R O P H Y LAX I S While the latest American College of Cardiology (ACC)/ Amer­ ican Heart Association (AHA) guidelines reversed the long-standing recommendation of routine antimicrobial prophylaxis for infective endocarditis in patients with HCM, it is notable that this change was not done in response to any new data n Given that infective endocarditis is a well­ documented and profound complication in patients with HCM, routine antimicrobial prophylaxis for infective endo­ carditis should be strongly considered.74 S U D D E N C A R D I AC D E AT H The most serious complication o f HCM is SCD , with an inci­ dence of 0. 1 % to 0. 7% per year. 1 4• 1 5 The first presentation of HCM may be SCD , generally from ventricular arrhythmias. Among subjects with HCM, SCD is more common in adoles­ cents and young adults,75 but it can occur at any age. Holter monitors have been recommended as a means of risk stratification for primary prevention of SCD. Ventricular arrhythmias are very common, with 88% of HCM patients having PVCs and 3 1 % of HCM patients having nonsustained ventricular tachycardia on 24-hour Holter monitoring. 76 Nonsustained ventricular tachycardia had a 95% negative predictive value and 9% positive predictive value for SCD .76 Thus, the absence of nonsustained ventricular tachycardia on 24-hour Holter is reassuring, but is nonspecific if pre­ sent. Electrophysiologic testing has not been shown to be predictive of SCD in HCM, and presently has little role in risk stratification in HCM. 4 While survivors of SCD warrant an ICD , primary prevention of SCD in HCM patients is less well defined. An ICD firing rate of 1 1 % per year has been reported in ICDs implanted for secondary prevention and 5 % per year when implanted for primary prevention of SCD n ACC/ AHA/North American Society of Pacing and Electrophysi­ ology 2002 guidelines designate ICD implantation for sec­ ondary prevention to be a Class I indication, whereas ICD implantation for primary prevention in HCM is a Class Ilb indication.78 Antiarrhythmic therapy for primary prevention generally is not recommended in asymptomatic patients. Major risk factors for SCD in HCM are shown in Table 22.4 and include LV wall thickness >30 mm 1 0 , pro­ longed or repetitive episodes of nonsustained ventricular tachycardia on Holter monitor79 , family history of SCD , no

258

SECTION III • CONGESTIVE HEART FAILURE A N D CARD IO MYOPATHY

j TA B L E --�1 Risk Factors for SCD • LV wall thickness >30 mm • Prolonged or repetitive episodes of nonsustained ventricular tachycardia on Holter monitor • Family history of SCD • No change or a decrease in blood pressure with exercise • Syncope or near syncope

change or a decrease in blood pressure with exercise80, and syncope or near syncope. 1 3·8 1 An LVOT gradient of �30 mm Hg is considered a minor risk factor for SCD .82 In a multicenter registry study of ICDs implanted between 1 986 and 2003 in 506 unrelated patients with HCM, there was a 3 . 6% rate per year of appropriate ICD therapy for primary prevention of SCD . 83 Of the patients that had an ICD placed for primary prevention and experienced appropriate therapy for ventricular tachycardia, 35% had only one risk factor. This suggests that a single risk factor for SCD may be enough to warrant implantation of an ICD in select patients with HCM. H C M A N D AT H L E T I C S Patients with HCM should b e restricted from competitive athletics or strenuous athletic activity because of the risk for SCD.84·85 Low-level exercise and participation in informal recreational activities, such as bowling and golf, are gener­ ally acceptable but should be considered on an individual basis. AT R I A L F I B R I L L AT I O N A N D H C M Atrial fibrillation, which occurs in 28% of HCM subj ects, is the most prevalent sustained arrhythmia in HCM. 1 4 HCM subjects with atrial fibrillation have lower long-term survival rates compared to those in sinus rhythm. 1 4·86 One study attributed the lower survival to an excess of heart failure­ related deaths as opposed to SCD.86 Atrial fibrillation is a significant cause of morbidity in HCM. Strokes occur in 6% of subjects with HCM, nearly all of whom have atrial fibrillation.87 Medical treatment of persistent atrial fibrillation in HCM includes anticoagula­ tion with warfarin and rate control, preferably with beta­ blockers. HCM patients who develop atrial fibrillation may pre­ sent with acute clinical deterioration. The hypertrophied ventricle is stiff and may require atrial contraction for opti­ mal filling. Losing the atrial contribution to ventricular fill­ ing may result in decreased cardiac output and potentially pulmonary edema. The substantial morbidity and increased mortality associated with atrial fibrillation in the setting of

HCM justifies an aggressive approach to attempting to main­ tain sinus rhythm. Given that HCM patients often tolerate atrial fibrillation poorly, expeditious TEE followed by electrical cardioversion is generally preferred. Amiodarone or sotalol is the preferred therapy for pharmacologic conversion to sinus rhythm or maintenance of sinus rhythm in HCM patients. Digoxin should be avoided in HCM patients, particularly in those with resting or latent obstruction, because of its positive inotropic effect. Atrial fibrillation ablation or the maze pro­ cedure may be considered for those with refractory, highly symptomatic atrial fibrillation. In a small number of patients with severe HCM and atrial fibrillation, we have performed combined maze-myectomy procedures.88 H C M A N D P R E G N A N CY Although pregnant women with HCM are at slightly higher risk for maternal or fetal complications than the average pregnant woman, the absolute morbidity and mortality rate for asymptomatic pregnant women with HCM is low.89•90 However, patients with resting or provocable LVOT obstruc­ tion should be referred to a high-risk obstetrician for care in collaboration with a cardiologist. Generally, such women do not need to undergo cesarean section and can deliver vaginally: Adequate fluid intake during pregnancy should be emphasized in pregnant women with HCM to ensure that the LV does not become underfilled. Certain beta-blocking drugs, such as extended-release metoprolol, can be contin­ ued during pregnancy but require increased monitoring for fetal bradycardia. While there is a lack of evidence, there is a theoretical concern for sympathetic blockade and loss of venous return from the lower extremities with spinal anes­ thesia. However, there are case reports of successful use of both regional and general anesthesia in patients with HCM.9 1 N O N O B S T R U CT I V E H C M Nonobstructive HCM is diagnosed when there is ventricular thickness of > 1 5 mm in the absence of other etiologies, and when no significant LVOT obstruction exists (i.e. , LVOT gra­ dient 50 mm Hg) that is refractory to medical therapy, with alcohol septal ablation only in patients deemed poor operative candidates The available therapeutic options for HCM are associated with high success rates in improving symptoms and decreasing LVOT gradients in combination with low mortality rates. REFERENCES 1 . Maron BJ . Hypertrophic cardiomyopathy: a systematic review. JAMA . 2002;287: 1 308-1320. 2 . Maron BJ, Gardin JM, Flack JM, et al. Prevalence of hypertro­ phic cardiomyopathy in a general population of young adults. Echocardiographic analysis of 4 1 1 1 subjects in the CARDIA Study Coronary Artery Risk Development in (Young) Adults. Circulation. 1995;92: 785-789. 3 . Maron BJ. Sudden death in young athletes. N Engl ] Med. 2003;349 1 064-1075. 4. Maron BJ, McKenna WJ, Danielson GK, et al. American Col­ lege of Cardiology/European Society of Cardiology clinical ex­ pert consensus document on hypertrophic cardiomyopathy A report of the American College of Cardiology Foundation Task Force on Clinical Expert Consensus Documents and the European Society of Cardiology Committee for Practice Guide­ lines. ] Am Coll Cardiol. 2003;42 : 1 687-1 7 1 3 . 5 . Maron MS, Olivotto I, Zenovich AG, e t al. Hypertrophic car­ diomyopathy is predominantly a disease of left ventricular out­ flow tract obstruction. Circulation. 2006; 1 14:2232-2239. 6. Lever HM, Karam RF, Currie PJ, et al. Hypertrophic cardiomy­ opathy in the elderly Distinctions from the young based on cardiac shape. Circulation 1 989;79 :580-589. 7. Binder J, Ommen SR, Gersh BJ, et al. Echocardiography-guid­ ed genetic testing in hypertrophic cardiomyopathy: septal mor­ phological features predict the presence of myofilament muta­ tions. Mayo Clin Proc. 2006;8 1 :459-467. 8. Sherrid MV, Chu CK, Delia E, et al. An echocardiographic study of the fluid mechanics of obstruction in hypertrophic cardiomyopathy J Am Coll Cardiol. 1993;22 : 8 1 6-825 .

259

9 . Maron BJ , Wolfson JK, Epstein S E , e t al. Intramural ("small vessel") coronary artery disease in hypertrophic cardiomyopa­ thy ] Am Coll Cardiol. 1 986;8 545-5 5 7 . 1 0 . Spirito P, Bellone P, Harris KM, et a l . Magnitude of left ven­ tricular hypertrophy and risk of sudden death in hypertrophic cardiomyopathy N Engl ] Med. 2000;342 : 1 778- 1 785. 1 1 . Gilligan DM, Chan WL, Ang EL, et al. Effects of a meal on hemodynamic function at rest and during exercise in pa­ tients with hypertrophic cardiomyopathy ] Am Coll Cardiol. 1 99 1 ; 1 8:429-436. 12. Maron BJ , Casey SA, Hauser RG, et al. Clinical course of hy­ pertrophic cardiomyopathy with survival to advanced age. ] Am Coll Cardiol. 2003;42:882-888. 13. Elliott PM, Poloniecki J , Dickie S, et al. Sudden death in hyper­ trophic cardiomyopathy: identification of high risk patients. ] Am Coll Cardiol. 2000;36:22 1 2-22 1 8 . 14. Cecchi F, Olivotto I, Montereggi A , e t a l . Hypertrophic cardiomyopathy in Tuscany: clinical course and outcome in an unselected regional population. ] Am Coll Cardiol. 1995;26: 1 529-1536. 15. Maron BJ, Casey SA, Poliac LC, et al. Clinical course of hy­ pertrophic cardiomyopathy in a regional United States cohort. JAMA. 1999;28 1 : 650-655 . 1 6 . Maron BJ, Tholakanahalli VN , Zenovich AG, e t al. Usefulness of B-type natriuretic peptide assay in the assessment of symp­ tomatic state in hypertrophic cardiomyopathy Circulation. 2004; 1 09 984-989. 17. Drinko JK, Nash PJ , Lever HM, et al. Safety of stress testing in patients with hypertrophic cardiomyopathy Am ] Cardiol. 2004;93 1 443, 1444, A l 2 . 1 8 . Kwon D H , Setser RM, Thamilarasan M, e t al. Abnormal papillary muscle morphology is independently associated with increased left ventricular outflow tract obstruction in hypertro­ phic cardiomyopathy Heart. 2008;94: 1 295-130 1 . 1 9 . Kato TS, Noda A, Izawa H , et al. Discrimination of nonobstruc­ tive hypertrophic cardiomyopathy from hypertensive left ven­ tricular hypertrophy on the basis of strain rate imaging by tissue Doppler ultrasonography Circulation. 2004; 1 1 0:3808-3814. 20. Rajiv C, Vinereanu D , Fraser AG. Tissue Doppler imaging for the evaluation of patients with hypertrophic cardiomyopathy Curr Opin Cardiol. 2004; 1 9 : 430-436 . 2 1 . King G, Foley J B , Royse C F, e t al. Myocardial stiffness and the timing difference between tissue Doppler imaging Ea and peak mitral valve opening can distinguish physiological hypertrophy in athletes from hypertrophic cardiomyopathy Eur] Echocardiogr. 2006;7 :423-429 . 22. Popovic ZB, Kwon D H , Mishra M, e t al. Association be­ tween regional ventricular function and myocardial fibrosis in hypertrophic cardiomyopathy assessed by speckle tracking echocardiography and delayed hyperenhancement magnetic resonance imaging. ] Am Soc Echocardiogr. 2008;2 1 : 1 299-1305. 2 3 . Yang H, Sun JP, Lever HM, et al. Use of strain imaging in de­ tecting segmental dysfunction in patients with hypertrophic cardiomyopathy ] Am Soc Echocardiogr. 2003 ; 1 6 :233-239. 24. Moon JC, Fisher NG, McKenna WJ, et al. Detection of api­ cal hypertrophic cardiomyopathy by cardiovascular magnetic resonance in patients with non-diagnostic echocardiography Heart. 2004;90: 645-649 . 2 5 . Rickers C, Wilke NM, Jerosch-Herold M, et al. Utility of car­ diac magnetic resonance imaging in the diagnosis of hypertro­ phic cardiomyopathy Circulation. 2005 ; 1 1 2 :855-86 1 .

260

SECTION III • CONGESTIVE HEART FAILURE AND CARD IO MYOPATHY

26. Harrigan CJ, Appelbaum E, Maron BJ , et al. Significance of papillary muscle abnormalities identified by cardiovascu­ lar magnetic resonance in hypertrophic cardiomyopathy Am ] Cardiol. 2008; 1 0 1 : 668-673. 27. Lima JA, Desai MY. Cardiovascular magnetic resonance im­ aging: current and emerging applications. ] Am Coll Cardiol. 2004;44: 1 1 64-1 1 7 1 . 28. Desai MY, Dhillon A , To AC. Cardiac magnetic resonance in hypertrophic cardiomyopathy Curr Cardiol Rep. 2 0 1 1 ; 1 3 : 6776. 29. Kwon DH, Desai MY. Cardiac magnetic resonance in hyper­ trophic cardiomyopathy: current state of the art. Expert Rev Cardiovasc Ther. 2 0 1 0;8: 1 03-1 1 1 . 30. Mahrholdt H , Wagner A, Holly TA, et al. Reproducibility of chronic infarct size measurement by contrast-enhanced mag­ netic resonance imaging. Circulation. 2002 ; 106:2322-2327. 3 1 . Kim RJ , Fierro DS, Parrish TB, et al. Relationship of MRI de­ layed contrast enhancement to irreversible injury, infarct age, and contractile function. Circulation. 1 999; 1 00: 1 992-2002. 32. Kwon DH, Setser RM, Popovic ZB, et al. Association of myo­ cardial fibrosis, electrocardiography and ventricular tachyar­ rhythmia in hypertrophic cardiomyopathy: a delayed contrast enhanced MRI study Int ] Cardiovasc Imaging. 2008;24:6 1 7625 . 33. Adabag AS, Maron BJ, Appelbaum E, et al. Occurrence and frequency of arrhythmias in hypertrophic cardiomyopathy in relation to delayed enhancement on cardiovascular magnetic resonance. ] Am Coll Cardiol. 2008;5 1 : 1 369-1 374. 34. Cecchi F, Olivotto I, Gistri R, et al. Coronary microvascular dysfunction and prognosis in hypertrophic cardiomyopathy N Engl ] Med. 2003;349 1027-1035. 3 5 . Bos JM, Towbin JA, Ackerman MJ . Diagnostic, prognostic, and therapeutic implications of genetic testing for hypertrophic cardiomyopathy ] Am Coll Cardiol. 2009;54 : 2 0 1 -2 1 1 . 36. Maron BJ, Seidman JG, Seidman CE. Proposal for contempo­ rary screening strategies in families with hypertrophic cardio­ myopathy ] Am Coll Cardiol. 2004;44:2 1 25-2 1 3 2 . 37. Gilligan D M , Chan Wl, Joshi J , e t al. A double-blind, place­ bo-controlled crossover trial of nadolol and verapamil in mild and moderately symptomatic hypertrophic cardiomyopathy ] Am Coll Cardiol. 1 9 9 3 ; 2 1 1 672-1 679. 38. Wigle ED, Rakowski H , Kimball BP, et al. Hypertrophic car­ diomyopathy. Clinical spectrum and treatment. Circulation. 1995;92: 1 680-1 692 . 39. Matsubara H, Nakatani S, Nagata S, et al. Salutary effect of diso­ pyramide on left ventricular diastolic function in hypertrophic obstructive cardiomyopathy ] Am Coll Cardiol. 1995;26: 768775. 40. Schaff HV, Brown ML, Dearani JA, et al. Apical myectomy: a new surgical technique for management of severely symptom­ atic patients with apical hypertrophic cardiomyopathy ] Thorac Cardiovasc Surg. 2 0 1 0 ; 1 39 : 634-640 . 4 1 . Minami K, Woltersdorf H, Kleikamp G, e t al. Long-term results after myectomy in 64 patients with hypertrophic obstructive cardiomyopathy (HOCM) . Morphological and hemodynamic aspects. ] Cardiovasc Surg (Torino) . 2000;4 l :801-806. 42 . Smedira NG, Lytle BW, Lever HM, et al. Current effectiveness and risks of isolated septa! myectomy for hypertrophic ob­ structive cardiomyopathy Ann Thorac Surg. 2008 ;85: 12 7-1 3 3 . 43. DearaniJA, Ommen SR, Gersh BJ, e t al. Surgery insight: Septa! my­ ectomy for obstructive hypertrophic cardiomyopathy-the Mayo

44.

45.

46.

4 7.

48.

49.

50.

51.

52.

53.

54.

55.

56.

57.

58.

59.

Clinic experience. Nat Gin Pract Cardiovasc Med. 2007;4:503512. Ommen SR, Maron BJ , Olivotto I, e t a l . Long-term effects of surgical septa! myectomy on survival in patients with ob­ structive hypertrophic cardiomyopathy ] Am Coll Cardiol. 2005 ;46470-476. Woo A, Williams WG, Choi R, et al. Clinical and echocardio­ graphic determinants of long-term survival after surgical myec­ tomy in obstructive hypertrophic cardiomyopathy Circulation. 2005 ; 1 1 1 :2033-204 1 . ten Berg JM, Suttorp MJ , Knaepen PJ , e t al. Hypertro­ phic obstructive cardiomyopathy Initial results and long­ term follow-up after Morrow septa! myectomy. Circulation. 1 994;90 : 1 78 1 - 1 785. Redwood DR, Goldstein RE, Hirshfeld J, et al. Exercise per­ formance after septa! myotomy and myectomy in patients with obstructive hypertrophic cardiomyopathy. Am ] Cardiol. 1979;44:2 1 5-220. Mohr R, Schaff HV, Danielson GK, et al. The outcome of surgical treatment of hypertrophic obstructive cardiomy­ opathy. Experience over 15 years. ] Thorac Cardiovasc Surg. 1989;97: 666-674. McCully RB, Nishimura RA, Tajik AJ, et al. Extent of clinical improvement after surgical treatment of hypertrophic obstruc­ tive cardiomyopathy Circulation. 1996;94:467-47 1 . Schulte HD, Borisov K , Garns E , e t al. Management o f symp­ tomatic hypertrophic obstructive cardiomyopathy-long­ term results after surgical therapy Thorac Cardiovasc Surg 1999;47 : 2 1 3-2 1 8 . Minami K, Boethig D , Woltersdorf H, e t al. Long term fol­ low-up of surgical treatment of hypertrophic obstructive cardiomyopathy (HOCM) : the role of concomitant cardiac procedures. Eur] Cardiothorac Surg. 2002 ;22: 206-2 1 0. Qin JX, Shiota T, Lever HM, et al. Conduction system abnor­ malities in patients with obstructive hypertrophic cardiomy­ opathy following septa! reduction interventions. Am ] Cardiol. 2004;93 : 1 7 1- 1 7 5 . Maron BJ. Role of alcohol septa! ablation i n treatment of ob­ structive hypertrophic cardiomyopathy Lancet. 2000;355 :425426. Maron BJ, Dearani JA, Ommen SR, et al. The case for surgery in obstructive hypertrophic cardiomyopathy. ] Am Coll Cardiol. 2004;44:2044-2053. Knight C, Kurbaan AS, Seggewiss H, et al. Nonsurgical septa! re­ duction for hypertrophic obstructive cardiomyopathy: outcome in the first series of patients. Circulation. 1997;95:2075-208 1 . Qin JX, Shiota T, Lever HM, e t al. Outcome o f patients with hypertrophic obstructive cardiomyopathy after percutaneous transluminal septa! myocardial ablation and septa! myectomy surgery ] Am Coll Cardiol. 200 1 ;38: 1 994-2000. Faber L, Meissner A, Ziemssen P, et al. Percutaneous trans­ luminal septa! myocardial ablation for hypertrophic obstruc­ tive cardiomyopathy: long term follow up of the first series of 25 patients. Heart. 2000;83 :326-33 1 . Gietzen FH, Leuner CJ , Obergassel L, e t al. Transcoronary abla­ tion of septa! hypertrophy for hypertrophic obstructive cardio­ myopathy: feasibility, clinical benefit, and short term results in elderly patients. Heart. 2004;90:638-644. Seggewiss H, Faber L, Gleichmann U. Percutaneous translumi­ nal septa! ablation in hypertrophic obstructive cardiomyopa­ thy Thorac Cardiovasc Surg. 1 999 ;47: 94-100.

CHAPTER 2 2 • HYPERTROPHIC CARDIO MYOPATHY 6 0 . Gietzen FH, Leuner CJ, Raute-Kreinsen U , e t al. Acute and long­ term results after transcoronary ablation of septa! hypertrophy (TASH) . Catheter interventional treatment for hypertrophic ob­ structive cardiomyopathy. Eur Heart ]. 1 999;20 : 1 342-1 354. 6 1 . Chang SM, Lakkis NM, Franklin ] , et al. Predictors of outcome after alcohol septa! ablation therapy in patients with hypertro­ phic obstructive cardiomyopathy. Circulation. 2004 ; 1 09 :824827. 62 . Shamim W, Yousufuddin M, Wang D , et al. Nonsurgical reduc­ tion of the interventricular septum in patients with hypertro­ phic cardiomyopathy. N Engl ] Med. 2002;347 : 1 326-1333. 63 . Talreja DR, Nishimura RA, Edwards WD, et al. Alcohol septa! ablation versus surgical septa! myectomy: comparison of ef­ fects on atrioventricular conduction tissue. ] Am Coll Cardiol. 2004;44:2329-2332. 64. Nagueh SF, Ommen SR, Lakkis NM, et al. Comparison of ethanol septa! reduction therapy with surgical myectomy for the treatment of hypertrophic obstructive cardiomyopathy. ] Am Coll Cardiol . 200 1 ;38: 1 70 1 - 1 706.s 65. Cuoco FA, Spencer WH III, Fernandes VL, et al. Implantable cardioverter-defibrillator therapy for primary prevention of sudden death after alcohol septa! ablation of hypertrophic car­ diomyopathy. ] Am Coll Cardiol. 2008;52: 1 7 1 8- 1 72 3 . 66. Hirata K, Wake M, Asato H, et a l . Sudden death of a case o f hypertrophic obstructive cardiomyopathy 1 9 months after suc­ cessful percutaneous transluminal septa! myocardial ablation. Circ ]. 2003;67:559-5 6 1 . 6 7 . Sorajj a P, Valeti U, Nishimura RA, e t al. Outcome o f alcohol septa! ablation for obstructive hypertrophic cardiomyopathy. Circulation. 2008; 1 1 8: 1 3 1-139. 68. Agarwal S , Tuzcu EM, Desai MY, et al. Updated meta-analysis of septa! alcohol ablation versus myectomy for hypertrophic cardiomyopathy. j Am Coll Cardiol . 2 0 1 0 ; 5 5 : 823-834. 69. Firoozi S , Elliott PM, Sharma S , et al. Septa! myotomy-myecto­ my and transcoronary septa! alcohol ablation in hypertrophic obstructive cardiomyopathy. A comparison of clinical, haemo­ dynamic and exercise outcomes. Eur Heart ] 2002 ;23: 1 6 1 71 624. 70. Nishimura RA, Trusty JM, Hayes DL, et al. Dual-chamber pac­ ing for hypertrophic cardiomyopathy: a randomized, double­ blind, crossover trial. ] Am Coll Cardiol. 1 997;29 :435-44 1 . 7 1 . Maron BJ , Nishimura RA , McKenna WJ , e t al. Assessment of permanent dual-chamber pacing as a treatment for drug-re­ fractory symptomatic patients with obstructive hypertrophic cardiomyopathy. A randomized, double-blind, crossover study (M-PATHY) . Circulation. 1 999;99 2927-2933. 72 . Ommen SR, Nishimura RA, Squires RW, et al. Comparison of dual-chamber pacing versus septa! myectomy for the treat­ ment of patients with hypertrophic obstructive cardiomyopa­ thy: a comparison of objective hemodynamic and exercise end points. ] Am Coll Cardiol. 1 999 ;34: 1 9 1-196. 73. Bach DS, American College of Cardiology/American Heart As­ sociation. Perspectives on the American College of Cardiology/ American Heart Association guidelines for the prevention of in­ fective endocarditis. ] Am Coll Cardiol. 2009 ;53: 1 852-1 854. 74. Maron BJ , Lever H. In defense of antimicrobial prophylaxis for prevention of infective endocarditis in patients with hypertro­ phic cardiomyopathy. ] Am Coll Cardiol. 2009;54:2339 ,40; au­ thor reply 2340. 75. Maron BJ, Olivotto I, Spirito P, et al. Epidemiology of hyper­ trophic cardiomyopathy-related death: revisited in a large

76.

77.

78.

79.

80.

81.

82 .

83 .

84.

85 .

86.

87.

88.

89.

90.

91.

261

non-referral-based patient population. Circulation. 2000; 102:858864. Adabag AS, Casey SA, Kuskowski MA , et al. Spectrum and prognostic significance of arrhythmias on ambulatory Holter electrocardiogram in hypertrophic cardiomyopathy. ] Am Coll Cardiol . 2005 ;45 :697-704. Maron BJ, Shen WK, Link MS, et al. Efficacy of implantable cardioverter-defibrillators for the prevention of sudden death in patients with hypertrophic cardiomyopathy. N Engl ] Med. 2000 ;342 :365-373 . Gregoratos G, Abrams J, Epstein AE, et al. ACC/AHA/NASPE 2002 Guideline Update for Implantation of Cardiac Pacemak­ ers and Antiarrhythmia Devices-summary article: a report of the American College of Cardiology/American Heart Asso­ ciation Task Force on Practice Guidelines (ACC/AHA/NASPE Committee to Update the 1998 Pacemaker Guidelines) . ] Am Coll Cardiol . 2002 ;40 : 1 703-1 7 1 9 . Monserrat L , Elliott PM, Gimeno JR, e t al. Non-sustained ventricular tachycardia in hypertrophic cardiomyopathy: an independent marker of sudden death risk in young patients. ] Am Coll Cardiol. 2003 ;42 873-879. Lim PO, Morris-Thurgood JA, Frenneaux MP. Vascular mecha­ nisms of sudden death in hypertrophic cardiomyopathy, in­ cluding blood pressure responses to exercise. Cardiol Rev. 2002 ; 1 0 : 1 5-23 . McKenna WJ , Behr ER. Hypertrophic cardiomyopathy: man­ agement, risk stratification, and prevention of sudden death. Heart. 2002;87: 1 69-1 76 . Maron M S , Olivotto I, Betocchi S , e t al. Effect of left ven­ tricular outflow tract obstruction on clinical outcome in hypertrophic cardiomyopathy. N Engl ] Med. 2003;348: 295-303. Maron BJ, Spirito P, Shen WK, et al. Implantable cardioverter­ defibrillators and prevention of sudden cardiac death in hyper­ trophic cardiomyopathy. JAMA. 2007 ;298 :405-4 1 2 . Maron BJ, Shirani J, Poliac L C , e t al. Sudden death i n young competitive athletes. Clinical, demographic, and pathological profiles. JAMA. 1 996;276 199-204. Maron BJ, Isner JM, McKenna WJ . 26th Bethesda conference: recommendations for determining eligibility for competi­ tion in athletes with cardiovascular abnormalities. Task Force 3: hypertrophic cardiomyopathy, myocarditis and other myo­ pericardial diseases and mitral valve prolapse. ] Am Coll Car­ diol. 1 994;24:880-885 . Olivotto I, Cecchi F, Casey SA, et al. Impact of atrial fibrillation on the clinical course of hypertrophic cardiomyopathy. Circula­ tion. 200 1 ; 1 04:25 1 7-2524. Maron BJ, Olivotto I, Bellone P, et al. Clinical profile of stroke in 900 patients with hypertrophic cardiomyopathy. ] Am Coll Cardiol . 2002 ;39:30 1-307. Chen MS, McCarthy PM, Lever HM, et al. Effectiveness of atrial fibrillation surgery in patients with hypertrophic cardiomyopa­ thy. Am ] Cardiol. 2004;93 3 73-375. Thaman R, Varnava A, Hamid MS, et al. Pregnancy related complications in women with hypertrophic cardiomyopathy. Heart. 2003;89: 752-756. Autore C, Conte MR, Piccininno M, et al. Risk associated with pregnancy in hypertrophic cardiomyopathy. ] Am Coll Cardiol. 2002 ;40 1 864-1 869. Poliac LC, Barron ME, Maron BJ. Hypertrophic cardiomyopa­ thy. Anesthesiology . 2006 ; 1 04: 1 83-192.

262

SECTION III • CONGESTIVE HEART FAILURE AND CARD IO MYOPATHY

QU ESTIONS AND ANSWERS Questions

1 . All of the fol lowi ng increase the g radient i n hyper­ trophic cardiomyopathy (HCM) except: a. Va lsa lva maneuver b. Squatting c. Amyl n itrite d. lsoproterenol 2. Which of the fol lowi ng is a n appropriate screening protocol for fam i ly mem bers of patients with HCM? a. Serial ECG and echocardiogra m every 5 years for a 1 4-yea r-old child of a n HCM patient. b. Serial ECG and echocardiogra m on a fi rst-deg ree adult relative every 1 0 years of an HCM patient. c. Serial ECG and echocardiogra m on a gene­ negative fi rst-deg ree relative of a patient who is gene positive for a f3-myosin heavy-chain m uta­ tion every 5 years. d. Cessation of screening for a myosin-binding pro­ tei n C m utation positive fi rst-deg ree relative of a patient with a myosin-binding protei n C m uta­ tion positive patient. e. Serial ECG and echocardiogra m every 1 8 months for a 1 7-year-old child of a n HCM patient. 3. All of the fol lowi ng a re true of the Brocken brough response except: a. There is increased fi l l i n g of the left ventricle (LV) with the com pensatory pa use. b. The premature beat causes a decrease in con­ tractility in HCM but not i n normal individ uals. c. There is an increase i n ventricu lar pressu re i n both normal individuals and in patients with HCM. d. There is a decrease i n aortic pressure i n HCM. e. There is a n increase i n aortic pressu re i n normal individ uals. 4. Echoca rdiog ra phy is the pri m a ry cli nical moda l ity for diagnosing HCM. Which of the fol l owing find­ ings is (a re) com m o n ly seen i n HCM? a. A septum > 1 5 mm b. Preclosure of the aortic va lve c. Anterior displacement of the papi l l a ry m u scles d. Elongated m itra l leaflets e. All of the choices 5. All of the fol lowi ng d rugs a re usefu l in the treat­ ment of HCM except: a. Metoprolol b. Disopyramide c. Enalapril d. Di ltiazem e. Phenylephrine

6. Which of the fol lowi ng is a risk factor for sudden death i n HCM? a. Septa I thickness >30 m m b . Prolonged or repetitive episodes o f nonsustai ned ventricular tachyca rdia c. Fam i ly history of sudden death d. Syncope or near syncope e. No change or a decrease in blood pressu re with exercise f. All of the choices 7. All of the fol l owing a re appropriate med ications to treat patients with HCM a n d atrial fibril lation except: a. Sota lol b. Metoprolol c. Amiodarone d. Digoxin e. Vera pa m i l Answers 1 . Answer B: The Va lsa lva maneuver decreases venous return and thus decreases ventricular vol u me, thus ac­ centuati ng the systolic anterior motion (SAM) and thus increasing the g rad ient. Amyl n itrite causes peri pheral vasodi l ation and tachyca rd ia. Both of these factors cause the LV to decrease in size and thus increase the g rad ient. lsoproterenol increases the contractil ity and thus decreases ventricu lar vol u me, which increases the g radient. Standing decreases venous return and decreases ventricular vol u me. Squatting increases the vascular resista nce and venous retu rn, thus increasing ventricular vol u m e and red ucing the SAM, which red uces the g radient. 2. Answer E: First-deg ree relatives of a m utation­ positive HCM patient who a re m utation negative do not need routine screeni ng. If a m utation is identified i n the i ndex case and the fi rst-deg ree relative is also gene posi­ tive, then they should have routine screening ECG and echocard iog ra m . If no gene m utation is identified i n the index patient, then a l l fi rst-deg ree relatives should be screened. Screening should occur every 1 2 to 1 8 months while i n adolescence.This frequent cycle should be fol l owed i nto early adu lthood at which point screening can be sca led back to a m i n i m u m of every 5 years. Exceptions to the ru le include relatives who do not have the gene m utation that has been identified i n the i ndex HCM patient in which case screening can be stopped. 3. Answer B: The Brocken brough maneuver ca uses the contractil ity to increase i n both normal i n d ivid uals and i n patients with HCM. A l l of the other statements are true.

CHAPTER 2 2 • HYPERTROPHIC CARDIO MYOPATHY 4. Answer E: The defi n ition for the diagnosis of HCM is that the septu m must be 1 5 m m or g reater i n the absence of any d isease known to cause hypertrophy. Preclosure of the aortic va lve is com monly seen on M-mode echo of the aortic va lve i n the presence of left ventricu l a r outflow tract (LVOT) obstruction. Anterior displacement of the pa pillary m uscles is freq uently seen i n HCM and contri butes to the development of outflow tract obstruction. Elongated m itra l leaflets have been recog n ized for some time i n HCM but a re now more easily seen with better instrumentation. 5. Answer C: Enalapril is a n angiotensin-converting enzyme i n h i bitor, and it can worsen obstruction by decreasing both preload and afterload. Metoprolol is a beta-blocker and thus, by slowing the heart rate, may a l low for prolonging d iastolic fi l l i n g and lessen the provoca ble outflow tract g radient. It is also a negative inotrope. It is somewhat less helpfu l if there is resting obstruction. Disopyramide has a negative inotropic

263

effect on the LV and thus freq uently d i m i n ishes LVOT obstruction. Diltiazem is a ca lci u m channel blocker that has some negative inotropic effect and may lessen LVOT obstruction. I n addition, it improves diastolic fi l l i ng. Phenylephrine may be l ife saving i n the treatment of hypotension-associated severe LVOT obstruction. It is a pure vascular constrictor and does not increase the contractility of the hea rt. 6. Answer F: Althoug h a l l of these factors have been shown to have a high negative pred ictive accu racy, the positive predictive accu racy is low. 7. Answer D: Digoxin should be avoided as it increases contractility, which in turn can increase obstruction. All other medications a re reasonable as pa rt of a rhythm­ or a rate-contro l l i n g strategy.

C ongenital Heart Disease in the Adult Richard A. Krasuski and David S . Maj dalany

A

dults with congenital heart disease ( CHD) are a rapidly growing population of patients owing to advances in the diagnosis and treatment of children with CHD. Most children with CHD are now expected to survive to adulthood either with or without the aid of surgical correc­ tion or palliation. According to recent estimates, there are now nearly a million adults with CHD, and these numbers should continue to rise with further advancements in diag­ nosis and treatment. Although ideally served by cardiologists with advanced training in adult CHD, most of these patients receive the majority of their care from primary care physicians and general cardiologists, even though few cardiology train­ ing programs have a formalized adult CHD curriculum. Being aware of the often unique clinical presentations, and having a general understanding of the anatomy and the pathophysi­ ologic consequences of congenital disease is vital to facilitating the timing of percutaneous, electrophysiologic, and surgical interventions.

G E N E R A L CO N C E PTS An organized approach to diagnosis and management is especially important in patients with CHD , and the critical first step is gathering historical data. Reviewing the pediatric and operative records, if available, is essential in understand­ ing the complexities of the cardiac and vascular anatomy and to define the outcomes of previous diagnostic studies and surgeries. Surgical procedures have changed considerably over the last several decades, and anatomic presumptions based on current practice may not apply Certain signs and symptoms should prompt an exten­ sive evaluation of adults with CHD , particularly syncope and progressive exertional dyspnea. Arrhythmias are not uncommon in adults with CHD and often originate near the myocardial scars of previous surgeries. The most com­ mon of tachycardia seen is macroreentry within the atrial muscle. Supraventricular arrhythmias, such as atrial flutter or fibrillation, are often poorly tolerated due to a depend­ ence on atrial mechanical function. Ventricular arrhythmias, 264

typically microreentrant ventricular tachycardia, which can result in sudden death, may develop in adult patients with CHD as a late complication of prior ventriculotomy and patching of a ventricular septal defect (VSD) . The incidence of ventricular arrhythmias in adults with corrected tetralogy of Fallot (TOF) is estimated between 0. 5% and 6 %, with independent risk variables that include a widened QRS interval (> 1 80 milliseconds) on a surface electrocardiogram (ECG) , significant right ventricular dilation, and older age at time of surgical repair. Hemodynamic derangements can be quite subtle, such as pulmonic regurgitation following a patch outflow repair of TOF. Since pulmonary regurgitation has a low-pressure gradient, it can be missed during auscultation and routine echocardiography and can eventually result in right ventricle (RV) enlargement and increased risk of sudden death. Diagnostic imaging is a critical adjunct, and less inva­ sive modalities such as echocardiography are an important first step (see Chapter 24) . Limitations of echocardiography include difficult windows due to excessive scar tissue from previous surgeries, concomitant lung disease, and obesity Subsequent computerized tomography (CT) scanning or magnetic resonance imaging (MRI) may add substantially to the anatomic description, especially in patients with unclear great vessel or pulmonary vascular anatomy The use of MRI has expanded with more widely available scanners and simplified scanning protocols. It is important to remember, however, that CT scanning is complicated by the need for intravenous contrast and exposure to radiation, and MRI is generally not compatible with current implantable cardiac devices. Diagnostic cardiac catheterization, though generally performed later in the diagnostic workup of CHD patients than in the past, remains the gold standard for pressure measurement, cardiac output calculation, and vascular resistance determination. The relative size of shunts lesions can be assessed using oximetry, and the hemodynamic con­ sequences of additional blood flow can be assessed. Most importantly, cardiac catheterization affords the opportunity

CHAPTER 2 3 • C O N GEN ITAL HEART DISEASE IN THE ADULT to intervene and palliate or repair anatomic defects or to clarify the suitability of further surgical intervention. Anatomic shunting can be quantified in the catheteriza­ tion laboratory by examining the blood oxygen saturations in the respective chambers. The mixed venous (MV) satura­ tion is the saturation of blood returning to the right atrium (RA) with contributions from the inferior vena cava (IVC) , superior vena cava (SVC) , and coronary sinus (CS) . IVC sat­ uration is normally higher than the SVC due to high renal blood flow and less oxygen extraction by the kidney. The CS saturation is very low, but its volume of contribution is neg­ ligible and usually ignored. To normalize the MV saturation, three times the SVC saturation is added to the IVC saturation and the sum divided by 4. Because so much mixing of blood with differing satura­ tions occurs in the RA, an 1 1 % increase in oxygen step-up (saturation increase from a chamber to its successive cham­ ber) is required to diagnose a shunt lesion between the SVC and the RA. A 7% increase is necessary to detect a shunt between the RA and the RV and a 5% increase to detect a shunt between the RV and the pulmonary artery (PA) . A quick and simple measure of the overall size of a left-to-right shunt ratio can be obtained by using the formula: (aortic saturation-MY saturation)/(PV saturation-PA saturation) . The PV saturation can be assumed to be 97% if not directly measured. In general, a "significant shunt" is present when the shunt ratio is � 1 . 5 : 1 .0. This simplified definition may not apply to older adults, however. As pulmonary hypertension develops and RV compliance falls, a left-to-right shunt that was 3 : 1 for 30 years may become < 1 . 5 : 1 due to the gradual reversing of the shunt. In fact, the left-to-right shunt may totally reverse at some point and result in arterial desatura­ tion, the so-called Eisenmenger syndrome. The significance of a shunt in the adult must, therefore, be examined in the context of the other hemodynamics, chamber sizes, and the history of the defect over time. Pulmonary hypertension is a frequent complication of certain CHDs. It can be secondary to pulmonary venous hypertension from elevated left-sided filling pressures, or the result of systemic-to-PA shunting. For unclear reasons, shunts proximal to the tricuspid valve (atrial septal defects [ASDs] or partial anomalous pulmonary venous return) infrequently result in pulmonary hypertension (- 1 5 % of cases) despite high pulmonary blood flow. The development of pulmonary hypertension from shunts distal to the tricus­ pid valve, however, is very dependent on pulmonary blood flow. For example, a large unrestricted VSD may not result in pulmonary hypertension if the pulmonary circuit is pro­ tected by concomitant pulmonary valvular or subvalvular obstruction. To help differentiate the cause of pulmonary hyperten­ sion, the pulmonary vascular resistance should be deter­ mined: (mean PA pressure-mean pulmonary capillary wedge pressure [mm Hg] )/(pulmonary blood flow [liters per minute]). Higher resistances (>7 Wood units or a ratio

265

of the pulmonary-to-systemic vascular resistance of >0. 5) have been associated with considerably higher perioperative mortality. In addition, assessment of pulmonary vascular reactivity with endothelium-dependent vasodilators, such as inhaled nitric oxide or intravenous adenosine, may provide additional prognostic information in these patients by con­ firming whether any of the observed pulmonary hyperten­ sion has a vasoconstrictor component. In patients with shunt lesions and pulmonary arterial hypertension (mean PA pres­ sure �25 mm and mean pulmonary capillary wedge pressure :5: 1 5 mm Hg) , growing evidence supports the use of selective pulmonary vasodilator therapy (such as endothelin block­ ers and phosphodiesterase-5 inhibitors) to improve exer­ cise capacity and reduce symptoms. No patients with CHD should be started on these medications, however, without first undergoing thorough hemodynamic assessment in the catheterization laboratory. T Y P E S O F CO N G E N I TA L L E S I O N S Congenital heart lesions can be divided into three general categories (by descending incidence) : simple shunt lesions, obstructive lesions, and complex lesions-acyanotic and cyanotic. The most frequently encountered abnormalities in these categories are mentioned below.

Shunt Lesions Intracardiac shunts are the most common form of congenital heart lesion and are frequently diagnosed in otherwise healthy adults. They are associated with increased pulmonary blood flow, which can lead to right heart chamber enlarge­ ment and arrhythmias, as well as pulmonary hypertension. The surgical correction of many of these lesions has been determined to be safe and efficacious. Recently, percutane­ ous device closures have been increasingly utilized in order to avoid the morbidity and mortality of surgery. There are three main types of shunt lesions to be aware of: ASD , VSD, and patent ductus arteriosus (PDA) . All these lesions dem­ onstrate left-to-right shunting under normal physiologic conditions. Atria l Septa l Defect

The ASD is the most common congenital heart defect encountered in adults (excluding mitral valve prolapse and bicuspid aortic valve) , accounting for up to 1 5 % of all adult CHD . It results from the failure of proper embryologic devel­ opment of the atrial septum. Almost a third of patients with ASD will have associated additional malformations such as pulmonary stenosis, VSD , mitral valve prolapse, subaor­ tic stenosis, aortic coarctation, and anomalous pulmonary venous drainage. There are many different types of ASD (see Chapter 24), the most common of which (75% of the cases) is the secundum ASD, in which the defect lies in the middle of the atrial septum. The secundum ASD is often mistaken for other abnormalities

266

SECTION IV • C O N GEN ITAL HEART DISEASE

or overlooked because the symptoms associated with it, typi­ cally fatigue, palpitations, and breathlessness, can be subtle and nonspecific. Other less common variations of ASD include the sinus venosus ASD in which there is abnormal fusion of the vena cava (superior or inferior) to the left atrium. This defect is almost always associated with partial anomalous return of the pulmonary veins (right superior or both right pulmonary veins draining into the SVC or RV). Because of its location, this defect can be missed on transthoracic echocardiography and usually requires either transesophageal echo or advanced radiographic imaging to make the diagnosis. The primum ASD involves the lower portion of the atrial septum and typi­ cally affects the ventricular septum as well (the so-called AV canal defect) . Both AV valves are structurally abnormal and the mitral valve is typically cleft. This defect is commonly seen in patients with trisomy 2 1 (Down syndrome) . The least common ASD, the CS septal defect, involves unroofing of the CS, which results in shunting from the left to the RA. Com­ monly, a persistent left SVC or an abnormal pulmonary venous drainage accompanies CS ASD. Important differences in the clinical findings among the various types of ASD are listed in Table 23. 1 . ASD should b e suspected whenever right heart enlarge­ ment is present without an alternative explanation. Physi­ cal examination findings, such as a fixed split second heart sound (due to loss of differential effects on right- and left­ sided filling pressures from a drop in intrathoracic pressure that normally occurs during inspiration) and a pulmonic outflow murmur (the result of increased pulmonary blood volume from shunting) , can also be overlooked. The flow of blood across the defect (shunt) is determined by the size of the defect and the compliance of the atria. Occasionally, patients can present late in life with ASD-related symp­ toms when the left atrial pressure rises because of a stiff left ventricle and diastolic dysfunction (usually the result of long-standing hypertension or coronary artery disease) . On electrocardiography, an incomplete right bundle branch block, right-axis deviation, abnormal P-wave axis, and right atrial enlargement are commonly seen. Due to anatomic position of the conduction bundles, a superior left axis is

usually noted in primum ASD . On chest x-ray prominent pulmonary arteries, right atrial and ventricular enlargement and pulmonary plethora can be seen. The larger the left-to-right shunt in patients with ASD , the greater is the risk for long-term complications such as atrial fibrillation (typically occurring in the fifth decade) and pulmonary hypertension. The latter condition affects up to 5% to 10% of adults with ASD , and if left uncor­ rected can result in Eisenmenger syndrome. In Eisenmenger syndrome, the pulmonary vascular resistance increases to the point that shunting is reversed (becoming right to left) and systemic oxygenation decreases. Patients with this com­ plication will not improve their oxygen saturation when oxy­ gen is administered to them (the telltale sign of a right-to-left shunt) . Multiple complications eventually ensue, and until

recently, this condition was considered irreversible. Another condition associated with ASD is stroke, which presumably results from paradoxical embolization (blood clots forming in the extremities and reaching the cerebral circulation by passing through the ASD) . The guideline-based indication to repair an ASD is right heart enlargement from volume overload resulting from the ASD , regardless of whether the patient is symptomatic or asymptomatic. At this time, only the secundum ASD has been successfully occluded through percutaneous methods. All other types of ASD require surgical closure. Repair of an ASD may also be reasonable in the con­ text of paradoxical embolism or documented platypnea­ orthodeoxia and should be considered in the presence of a hemodynamically significant net left-to-right shunt and PA pressure or pulmonary vascular resistance 65%) in the absence of dehydration. If phlebotomy is attempted, it should be accompanied by at least equal fluid replacement. Repeated phlebotomy can result in iron defi­ ciency and actually increases the risk of hyperviscosity. Iron should be repleted in these patients if deficiency is present. Patients with Eisenmenger syndrome often develop proteinu­ ria and a decreased glomerular filtration rate ( GFR) . Because of the low GFR and the high turnover of red blood cells, elevated uric acid levels are frequently seen and can result in acute renal failure, particular after administration of contrast dye if the patient is not adequately hydrated. Patients with Eisenmenger physiology are at risk of both thrombosis and hemorrhage, hemoptysis that may be life threatening, cer­ ebral abscesses, stroke, scoliosis, and arthropathy as well as

pigment gallstones. They may present with cardiac ischemia in the setting of coronary artery compression by a dilated PA, right ventricular ischemia, or atherosclerosis. Patients with Eisenmenger physiology should avoid dehydration, moderate-to-severe strenuous exercise, expo­ sure to excessive heat, chronic high altitude, and pregnancy. If catheterization or noncardiac surgery is required, they should be hospitalized in centers with adult CHD expertise and experienced cardiac anesthesia. All intravenous lines should be filtered to exclude air bubbles. Improved quality of life has been noted with the use of pulmonary vasodila­ tors in patients with Eisenmenger physiology and survival may be positively impacted. Transplantation has also offered limited survival benefits albeit with significant quality of life improvement for this patient population. G E N E R A L M A N AG E M E N T S T R AT E G I E S Although adult patients with CHD can be intimidating at first presentation, sticking to basic concepts can be helpful in choosing appropriate management strategies. Patients with intracardiac shunts should be counseled to avoid high-risk activities such as scuba diving and have filtering devices placed on all intravenous lines whenever hospitalized to prevent the risk of paradoxical emboliza­ tion. Noncardiac surgery should be considered on a patient­ per-patient basis only after the risks and benefits have been carefully considered, particularly in the patient with Eisen­ menger syndrome. Most adults with CHD will require life­ long follow-up and should be referred to tertiary care centers with expertise in their care. Some patients with CHD are at increased risk of devel­ oping IE and should be educated on the recommendations for prophylaxis that were revised in 2007. According to the new guidelines, antibiotic prophylaxis before dental proce­ dures is recommended to the "high-risk" group of patients with CHD including (1) those with prior IE; (2) those with prosthetic heart valves; (3) those with palliated or unre­ paired cyanotic CHD , including surgically constructed pal­ liative conduits and shunts; (4) those with repaired CHD with prosthetic material or device, whether placed percuta­ neously or surgically, during the first 6 months postproce­ dure; and (5) those with repaired CHD with residual defects at the site or adjacent to the site of a prosthetic device or patch that prevents endothelialization. Approximately 18% of congenital heart defects are associated with a congenital syndrome, including coexisting cognitive and neurologic deficits or chromosomal abnor­ malities (Down syndrome with trisomy 2 1 and TOF with 22ql l . 2 deletion) . These patients should be appropriately screened for coexisting noncardiac conditions affecting them including sleep apnea, endocrinopathies, renal disease, and psychiatric issues with appropriate referrals provided. Patients should also be counseled on important topics such as pregnancy, genetic counseling, and contraception. For illustrative cases of CHD lesions, please refer to Chapter 24.

CHAPTER 2 3 • C O N GEN ITAL HEART DISEASE IN THE ADULT S U G G EST E D R E A D I N G S Bashore TM. Adult congenital heart disease: right ventricular outflow tract lesions. Circulation. 2007; 1 1 5 : 1 933-1947. Diller GP. Gatzoulis MA. Pulmonary vascular disease in adults with congenital heart disease. Circulation. 2007; 1 1 5 : 1 0391050. Rhodes JF, Hij azi ZM, Sommer RJ . Pathophysiology of con­ genital heart disease in the adult, part II. Simple obstructive lesions. Circulation. 2008; 1 1 7 : 1 228-1237. Sommer RJ, Hij azi Z M , Rhodes JF Jr. Pathophysiology of congenital heart disease in the adult: part I : Shunt lesions. Circula­ tion. 2008 ; 1 l 7 : 1 090- 1 099.

271

Sommer RJ , Hij azi ZM, Rhodes JF. Pathophysiology of con­ genital heart disease in the adult: part III: Complex congenital heart disease. Circulation. 2008; 1 1 7 : 1 340-1350. Spence MS, Balaratnam MS, Gatzoulis M A . Clinical update: cyanotic adult congenital heart disease. Lancet. 2007;370: 1 530-1532. Warnes CA. Adult congenital heart disease importance of the right ventricle. ] Am Coll Cardiol. 2009 ;54: 1 903- 1 9 1 0 . Warnes CA, Williams R G , Bashore TM, e t al. ACC/AHA 2008 Guidelines for the Management of Adults with Congenital Heart Disease: a report of the American College of Cardiology/Amer­ ican Heart Association Task Force on Practice Guidelines (writing committee to develop guidelines on the management of adults with congenital heart disease) . Circulation. 2008; 1 1 8:2395-245 1 .

QU ESTIONS AND ANSWERS Questions

1 . You are seeing a 27-year-old female posta l employee i n outpatient clinic. She has a history of asthma treated with i n halers, though her pul monary function tests and methacholine chal lenge were recently norma l. She has noted prog ressive fatigue over the past months and finds that getting u p large h i l ls on her mail route gets her out of breath. On examination, she has a fixed split second heart sound and soft systolic ejection m u r­ m u r over the left u pper sternal border. Her lungs a re clear and a l l her extrem ity pulses are equal and of normal i ntensity. All the fol lowi ng wou l d be expect­ ed to be present on her diagnostic studies except: a. U nexplained right heart enlargement on echo­ ca rdiogra phy b. An RSR' (inco m plete bundle branch block) pat­ tern on electrocard iog ra m (ECG) c. U nexplained mild pulmonary hypertension d. Right-to-left s h u nt by bubble study on echoca rdiography e. Decreased pulmonary vascula rity on chest x-ray 2. You a re eva l uating a 35-year-old female l ibrarian with a history of com plex congen ital heart disease (CH D). She tel l s you that she has a "hole i n the heart" but was told a bout 1 0 yea rs ago that it was "too late to operate." Her health has recently been sta ble and she denies worsening dyspnea, headaches, chest pain, or other symptoms. She is cu rrently able to wa l k from her home to her m a i l box (-300 ft) before she has to stop and rest. On exa m, she a ppears cya notic but is breathing comforta bly. She has a loud pul monic closure sound and a l lNI holo­ systolic m u r m u r at the right lower sternal border. There are no surg ica l scars over her back or chest and her pulses a re equal in a l l fou r extrem ities. Her bloodwork demonstrates:

White blood cel l count = 8,000 Hemoglobin = 22.4 Platelets = 295,000 U rea n itrogen = 1 Creatinine = 0.9 Reasonable thera peutic considerations i n this patient include all of the fol lowi ng except: a. Endocarditis prophylaxis with denta l proced u res b. Supplemental oxygen to wear at night or with exertion c. Phlebotomy of 2 u n its with equal vol u m e repletion d. I nvasive assessment of cardiac hemodynamics fol l owed by i nitiation of bosenta n e. Avoiding studies that a d m i n ister intravenous contrast dye 3. Which of the fol l owing statements a bout atrial septa I aneurysm (ASA) is true? a. The term ASA is used to describe a very floppy interatria l septu m. b. An ASA occurs when there is overabundant tis­ sue i n the septum prim u m . c. T h e most widely accepted defi nition o f a n ASA is > 1 5-mm excursion from left-to-right atri u m on echocardiog raphy. d. The presence of an ASA i n patients with patent fo­ ra men ova le (PFO) and presu med embolic stroke appears to i ncrease the risk of subseq uent stroke. e. All of the choices 4. Which of the fol l owing statements about PFO is the most accu rate? a. PFO is u nco m mon in the genera l popu lation, less common i n patients with cryptogenic stroke; and its optimal ma nagement is wel l defi ned and evidence based.

(Contin ued)

272

SECTION IV • C O N GEN ITAL HEART DISEASE

b. PFO is com mon i n the genera l popu lation, more common i n patients with cryptogenic stroke; and its opti mal ma nagement i n most cases remains to be defi ned. c. PFO is u ncom mon i n the genera l popu lation, more com mon i n patients with cryptogenic stroke; and its opti mal ma nagement i n most cases remains to be defi ned. d. PFO is com mon i n the genera l popu lation, less common i n patients with cryptogenic stroke; and its opti mal ma nagement i n most cases remains to be defi ned. e. PFO is com mon i n the genera l popu lation, more common i n patients with cryptogenic stroke; and its opti mal ma nagement is wel l defi ned and evidence based. 5. All of the fol l owing statements about anomalous pulmonary venous return a re correct except: a. Anomalous pul monary venous retu rn can lead to right heart enlargement and elevations in pul monary a rterial pressu res. b. Sinus venosus atrial septa I defects [ASDs] are of­ ten associated with anomalous pul monary veins. c. Ca rdiac computed tomography, magnetic resonance imaging (MRI), and transesophageal echocardiography are all reasonable modal ities to assess for anomalous pul monary veins. d. A bubble study during transthoracic echocardiogra­ phy can help detect an anomalous pulmonary vein. 6. You a re seeing a 35-year-old flight attendant with a history of coarctation of the aorta. At age 1 0, the patient u nderwent a coa rctation resection and end-to-end anastomosis. She has noted no l i m ita­ tions since that time but notes that in genera l she has had slightly "less stam ina" that her friends and col leagues over the past several years. On exam, she has eq ual blood pressu res i n a l l fou r extremities and a soft systolic ejection m u r m u r. All of the fol lowing statements are true concerning this patient except: a. It is l i kely that the patient has a bicuspid aortic va lve. b. There is a 1 0% cha nce that the patient has a berry aneurysm i n the bra i n . c . T h e type o f surgery that t h e patient underwent ma kes the possibil ity of aneurysmal dilatation at the surgical site very l i kely. d. The patient is at a hig her risk of developing hypertension than the genera l population. e. All of the choices a re true. 7. The lesions that constitute tetralogy of Fa llot (TOF) incl ude all of the fol lowi ng except: a. A ventricu lar septal defect (VSD) b. An overriding aorta

c. An ASD d. Right ventricular outflow obstruction e. Right ventricular hypertrophy 8.

A 2 1 -yea r-old col lege student presents for a routine medical checku p. He has never seen a n adult ca rdiologist and last saw a pediatric cardi­ ologist while i n high school. He has been told he has congenitally corrected tra nsposition with an i ntact ventricu lar septum and no known va lvular dysfu nction. All of the fol lowi ng concerns about this young man are valid except: a. His systemic right ventricle (RV) is at risk for dilatation and fai l u re. b. He has a 1 0% l ifetime risk of develop Eisen­ menger synd rome. c. His systemic tricuspid va lve is at risk for devel­ oping significa nt reg u rgitation. d. He has a 32% l ifetime probabil ity of developing complete heart block. e. All of the choices are correct.

9.

All of the fol lowi ng synd romes and cardiac anomalies a re associated except: a. Trisomy 21 and atrioventricu lar canal defects b. Noonan synd rome and pulmonic stenosis (PS) c. Holt-Ora m syndrome and ASDs d. Marfa n synd rome and m itral valve prola pse e. Wi l l iams syndrome and VSDs

1 0.

Which of the fol lowi ng statements regarding Ebstein anomaly is not correct? a. An ASD or a PFO is present in up to 80% of patients. b. The ca rdinal feature is an apica l ly displaced tricuspid va lve resulting in atrial ization of ven­ tricu lar tissue. c. Wolf-Pa rkinson-White synd rome is common in these patients and m u ltiple tracts can exist. d. A bicuspid aortic va lve is com monly present. e. A "sai l sound" is a common finding on physica l exa m ination.

11.

You a re seeing a 34-year-old gentleman i n clinic. He has a history ofTOF and underwent a pa l l ia­ tive Blalock-Taussig at 1 0 months fol l owed by a complete repai r at age 3. He has been reasonably active for severa l years but recently has been "slowing down" a l ittle bit. His physica l exa m i na­ tion demonstrates scars over his left sca pulae and midstern u m . He has a I l l out of VI systolic ejection m u r m u r and a I I out of IV d iastolic m u r m u r over the left u pper stern u m . He has clear l u ngs and equal pu lses i n a l l four extremities and no pe­ riphera l edema. An echocard iogram is somewhat l i m ited i n q u a lity due to the fact he is a rather large individual, but you are able to see evidence that the right heart appears enlarged and there

CHAPTER 2 3 • C O N GEN ITAL HEART DISEASE IN THE ADULT

time and states he now gets out of breath with anything more than moderate activity. On exa m i na­ tion, he has a systolic ejection murmur heard best over the left upper sternal border. There is no radia­ tion to the carotid arteries. An echocardiogra m is performed and demonstrates a doming pul monic valve with trace regu rgitation. The Doppler tracing shown is obtained across the pulmonic valve.

is some pul monic reg u rg itation present. The ECG shows some widening of the QRS complex with right bundle branch block morphology. The most reasonable next step in the diagnostic eva l uation of this patient would be: a. A repeat echoca rdiogra m with a sa line micro­ cavitation (bubble) study b. An electrophysiologic study to look for ven­ tricular arrhythm ias c. A ca rd iac catheterization to formally examine the hemodynam ics d. A ca rd iac MRI study e. The i nitiation of d i u retics and d ig italis 1 2.

1 3.

1 4.

You have been asked to see a 45-yea r-old woman with a VSD. She has been i n excel lent health for m a ny yea rs and voices no particular complai nts. She had been ta king a ntibiotic prophylaxis with denta l proced u res but d isconti nued this as a resu lt of the recent g uideline cha nges. On exa mi­ nation, she has a l l lNI pansystolic murmur and norma l i ntensity fi rst and second heart sou nds. Her l u ngs a re clear and she has no j u g u l a r venous distention. All of the fol lowi ng characteristic wou l d a rg u e for a ben ign c l i n ica l cou rse in this patient except: a. A loud m u r m u r b . N o r m a l i ntensity heart sounds c. A supracrista l (or su baortic) morphology d. The a bsence of right or left heart enlargement e. All of the choices are benign characteristics. A 28-year-old woman is referred to you for eva l ua­ tion of a heart m u r m u r. She states she is a long­ distance ru n ner and has not noted any significant sym ptoms. On exa m ination, you note very brisk pulses and her blood pressu re is 1 00/40 m m Hg. Her murmur extends from systole i nto diastole, and there is a near "mach inery"-type q u a l ity to it. The remainder of her physica l exa m i nation is essential ly u n remarkable, as is her bloodwork. The most l i kely ca rdiac anomaly i n this case is: a. An ASD b. Coarctation of the aorta c. A patent d uctus arteriosus (PDA) d. Congenita l ly corrected tra nsposition with a VSD and p u l monic va lve stenosis e. VSD A 40-year-old man is referred to you for eval uation of a heart murmur. He had been active u ntil a bout 3 years ago when he experience severe pain i n his rig ht knee that was eventually diagnosed as a liga­ mental tear. He underwent open operative repair and has been limited si nce that time. As a result, he has gained approxi mately 30 pounds si nce that

273

The most appropriate next step wo uld be: a. Contin ued observation with yea rly visits and echoca rdiography b. Referra l for pul monic va lvu loplasty c. Referra l for surgery after diagnostic angiography d. Stress echocard iography e. M RI 1 5.

A 1 9-yea r-o ld basketba l l player is brought to the emergency department after he col l apsed on the cou rt. He received bysta nder ca rdiopu l monary resuscitation and was apparently defibri l lated us­ ing a n automatic external defi bril lator (AED) . All of the fol lowi ng abnormalities should be part of the d ifferential diag nosis for this young man with the exception of: a. Anomalous orig i n of the coronary arteries (from opposite cusps). b. Hypertrophic cardiomyopathy c. Congenital ly prolonged QT synd rome d. ASD e. Arrhythmogenic right ventricular dysplasia

Answers 1 . Answer E: The l u n g x-ray i n this case would be expected to demonstrate increased l u ng vascula rity. The patient describe in this case has a n ASD.The tel ltale physica l exa m ination findings are the p u l monic outflow m u r m u r resu lting from increased p u l monary blood flow due to left-to-right s h u nting and the fixed split second heart sound. ASDs, if sufficiently large, lead to right heart enlargement and a n i ncom plete right bundle branch block pattern on electrocard iography. Pul mo­ nary hypertension ca n result from increased blood flow

(Contin ued)

274

SECTION IV • C O N GEN ITAL HEART DISEASE

and u p to 1 0% may develop Eisen menger physiology if uncorrected. As with a ny atrial flow comm u nication a bubble study on echoca rd iogra phy would be expected to be positive providing the right atrial pressu re ca n be made to exceed the left atrial pressure (such as follow­ ing a Valsa lva maneuver) . 2. Answer C: Phlebotomy should not be performed i n patients with Eisen menger physiology u n less they demonstrate evidence suggesting active sludging due to polycythemia. Suggestive sym ptoms incl ude head­ aches and visual cha nges. U n n ecessa ry phlebotomy can provoke i ron deficiency, which can fu rther increase the risk of sludging and its consequences. If phlebotomy is necessa ry, eq ual vol u m e replacement with sa line is essential. I ron levels should be checked in these patients and repleted as necessa ry. As with a l l cya notic heart disease, endocarditis prophylaxis for denta l proced u res is reco m mended. The use of oxygen has not been wel l studied i n this population but is reasonable if it affords the patient symptomatic i m p rovement. An ora l endothelin blockade, bosenta n, has been demon strated in a ra ndomized, placebo-control led study to i m p rove fu nctional capacity and red uce symptoms com pa red to placebo for patients with Class I l l symptoms. Since our patient seems fai rly l i mited, this may be a very reason­ able thera peutic option for her. Any proced u res req u i r­ ing the use of a nesthesia or contrast dye should be a pproached very carefu lly i n patients with Eisen menger synd rome due to the risk for adverse consequences. 3. Answer E: An ASA is a floppy i nteratrial septum

resulting from overabundant tissue i n the septum prim u m . A total septa I excursion of > 1 5 m m has been most widely accepted as the defi nition for this entity. If the septa l excursion is less, it is genera l ly referred to as a "red u ndant atrial septu m." I n patients with cryptogenic stroke and a PFO, a concu rrent ASA appears to signifi­ cantly i ncrease the risk of future stroke. 4. Answer B: The foramen ovale is the i nterface be­ tween the septum pri m u m and the septum secu n d u m and i n utero provides an i m portant route that blood can take to bypass the fetal lungs (which a re colla psed u ntil birth). I n a bout 25% of h u ma ns, the fla p of tissue mak­ ing up the foramen ova le does not fuse after birth and resu lts i n a PFO. Several stud ies have demonstrated a n i ncreased incidence o f P F O i n patients with cryptogenic (otherwise u nexplained) stroke. Though severa l man­ agement strategies exist for patients with cryptogenic stroke and PFO including a nticoagulation, a nti platelet thera py, and percuta neous and surgical closu re, no clear consensus rega rding thera py exists and stud ies a re ongoing to try to answer this very i m porta nt q uestion. 5. Answer D: A bubble study would not be expected to be abnormal in the presence of an anomalous

pul monary vei n u n less a concu rrent ASD was present. Normal ly, a l l fou r pul monary veins d ra i n back to the left atri u m . Rarely one or m u ltiple pulmonary veins can d rain back to the RA and result i n a left-to-rig ht s h u nt. Th is can result i n right heart enlargement and even pulmonary hypertension. Anomalous pul monary vei ns a re present i n most sinus venosus ASDs and i n u p to 1 0% of secu n d u m ASDs. Though tra nsthoracic echocar­ diogra phy is genera l ly unable to image a n anomalous pulmonary vein, CT, MRI, and transesophagea l echocar­ diogra phy a re a l l hel pfu l i n its detection. 6. Answer C: End-to-end resection of an aortic coarctation is most l i kely to be compl icated by eventual recoarctation, which can often be approached per­ cutaneously. Another procedu re that was previously popula rized for coarctation repair, the so-ca l led patch aortoplasty, can lead to aneurysmal dilatation, and these patients require very close monitoring. Coarctation of the aorta is believed to resu lt from the migration of ductus arteriosus tissue into the aorta proper. As a resu lt, con­ striction of the aorta occurs and leads to upper-extrem ity hypertension and lower-extrem ity hypoperfusion. In the adult, this lesion is most com monly diagnosed d u ring evaluation for secondary causes of hypertension . A bicus­ pid aortic valve is present in 50% to 85% of patients and ascending aortic enlargement can also be seen. There is a lso a 1 0% chance of having a concu rrent berry aneu­ rysm. Despite surgical or percutaneous repa ir, patients are at increased risk of developing hypertension, even i n t h e absence o f an appreciable residual g radient. 7. Answer C: The lesions of TOF include a VSD, a n over­ rid ing aorta, the presence of right ventricu lar outflow obstruction (va lvu lar or subva lvu lar), and right ventricu­ lar hypertrophy. The concu rrence of a n ASD has been referred to a "pentalogy," but is not part of the pri m a ry lesion com plex. 8. Answer B: In the absence of any sig nificant shunt lesions, there is no risk to this patient of developing Eisen­ menger syndrome. Congenitally corrected transposition of the g reat vessels implies that the patient has both atrio­ ventricu lar and ventriculoarteria l discordances. In other words, his venous draining enters the RA, which is con­ nected to a left ventricle and then is taken to the l u ngs via the pulmonary artery (PA). Pul monary venous retu rn then enters the left atriu m and enters the RV, which then pumps blood to the body through the aorta. Although this reproduces a near-normal circulation, the RV has not been adequately designed to withstand the workload of bei ng a systemic ventricle. As a result, it begins to fail. Also, the tricuspid valve (the systemic AV valve-valves always fol low their respective ventricles) begins to leak. Patients with congenitally corrected transposition have conduc­ tion issues and are prone to developing heart block.

CHAPTER 2 3 • C O N GEN ITAL HEART DISEASE IN THE ADULT

9. Answer E : The characteristic card iovascu lar lesion

of Wil liams synd rome is suprava lvu lar aortic stenos is, though coarctation of the aorta and peripheral PA stenosis has a l so been described.The characteristic lesion of trisomy 21 (Down synd rome) is an atrioven­ tricu lar canal defect (a lso known as a pri m u m ASDNSD), though ASD, VSD, and PDA are also com mon. Noonan syndrome has most classica l ly been associated with dysplastic or stenotic pul monic valves. Holt-Oram syn­ d rome is an a utosomal dominant disorder i n which ASDs and VSDs are most common. Marfan synd rome's most worrisome cardiovascular i nvolvement is of the aorta, which can lead to dissection and even death. Many of these patients have concurrent m itra l valve prolapse, though it is less l i kely a cause of morbid ity or mortal ity.

1 0. Answer D: A bicuspid aortic valve does not appear to be a common finding i n most patients with Ebstein anomaly. Ebstein a nomaly is cha racterized by apical displacement of the septa I and the posterior tricuspid va lve leaflets, leading to atria lization of the RV. An atrial flow com m u n ication (ASD or P FO) exists i n up to 80% of patients. The ECG often demonstrates very large or "Hima layan" P waves. Wolf-Parkinson-Wh ite synd rome is present i n u p to 30% of patients with half of these having m u ltiple accessory tracts. The loud snapping sound of the ba llooning leaflets has been compared to that of a sai l flapping i n the wind and can be a very characteristic exa m ination fi nding.

1 1 . Answer D: Primary repair ofTOF entails not only closing the VSD but also resecting the right ventricular outflow obstruction and often placing a patch over the resected tissue. Because the valve is often dysplastic, sig­ n ificant reg u rg itation of the pulmonic valve resu lts.This patient demonstrates the typical examination of patient with prior repai r compl icated by sig nifica nt pulmonic valve reg urgitation. These patients do rema rka bly wel l for many years b u t then develop prog ressive right heart dilatation, heart fai l ure, and arrhythmias. Widening of the QRS complex has been wel l described as a precu rsor to adverse clinical outcomes. The timing of reoperation to i m plant a pul monic valve is very challenging, and the status of the RV appears to be the most i mportant deter­ m i n i ng factor. A bubble study would only clarify whether an atrial level or a pulmonary shunt was present, which is u n l i kely to be a pathophysiologic contri butor to this case. Though a rrhythmias are a complication of right heart di latation, the role of electrophysiologic testing in this popu lation of patients is fa r from clear, and in this patient without a history of syncope would not be ind icated. It is also u n l i kely that invasive hemodynamics wou ld provide crucial diagnostic information. Finally, without evidence for significant vol u m e overload, the initiation of diuretics and digoxin would not be recommended at this time.

275

1 2. Answer C: Of the va rious types of VSDs, su prac­ rista l (su baortic) defects should be monitored closely because of their predi l ection for spontaneous closure by aortic leaflet tissue resulting i n significant aortic reg u rg itation.The patient i n this case has a sma l l, restrictive, and asym ptomatic VSD. Flow i n such cases is determined by the size of the s h u nt and the com p l i­ ance of the ventricles. Smaller lesions i n general will have increased turbu lence and thus louder m u r m u rs. Th us, a louder m u r m u r isolated to systole is reassuring i n this case as is the presence of normal i ntensity heart sounds. I n the presence of pulmonary hypertension, the p u l monic com ponent of the second heart sound is of­ ten accentuated. Ca rd iac cha m ber e n l a rgement resu lts from vol u m e overload and its a bsence in this case again suggests a more ben ign lesion. 1 3. Answer C: PDA is the persistence of a n i n utero com m u nication between the aorta and the left PA, which is again desig ned to bypass blood away from the colla pse l u ngs. It is the third most common congenital heart defect found i n adu lts and is genera l ly found in isolation i n the adu lt. Most adult patients with patent d uctus a re asym ptomatic, though this depends on the size of the left-to-right s h u nt and the size of the d uctus. Freq uently, this lesion is discovered by the u n usual qual­ ity of a continuous m u rm u r at the left upper sterna l bor­ der that can often sou nd l i ke a n i n n ocent venous h u m . Because a patent d uctus is a n aortopul monary runoff, however, the pu lse pressure frequently is widened, and the pu lses are brisk to bou nding. Because of the risk of endocard itis, some advocate repa i r, even if the s h u nt is not sig n ifica nt. Fortunately, most ductus lesions can be now be closed in the catheterization la boratory without the need for surgery. 1 4. Answer B: Th is patient has PS with pea k g radient of 60 mm Hg. Class I guideline ind ications for va lvu­ loplasty a re a peak echo g rad ient ;::: 6 0 mm Hg in the asymptomatic a d u lt and ;::: s o mm Hg if sym ptomatic. Stress echocardiography would be of l ittle uti lity as the rest g radient is a l ready sufficient to recom mend a n in­ tervention; i n patients with less g radient and symptoms, a stress test may demonstrate a provokable g rad ient that correlates with sym ptoms and would i m ply a ben­ efit from i ntervention. Magnetic resonance is usefu l to establish the location of na rrowi ng (valvu l a r, subvalvular, or supravalvu l a r) i n d ifficult to image cases, but i n this patient the echo images have esta bl ished that the nar­ rowi ng is valvu lar. Though surgery can be performed i n these patients i f m u ltiple lesions coexist, percutaneous ba l l oon va lvotomy is now the preferred therapy i n the majority of patients with isolated va lvu lar PS. 1 5. Answer D: Though a n ASD does increase the risk of developing atrial fibril lation later i n l ife and u n repaired

(Contin ued)

276

SECTION IV • C O N GEN ITAL HEART DISEASE

may shorten lifespan, there has been no link between ASD and sudden death. The most common lesions to exclude in a young person who suffers a sudden death include hypertrophic cardiomyopathy, a n autosomal dominant disorder that results i n abnormal myocar­ dial architectu re and increases arrhythmogenic risk. Abnormal coronary a rtery orig ins a ppea r to be a risk for sudden death as well, though the mechanism remains poorly understood and controversial. It may be due to

compression between the g reat vessels or an abnormal slit-like orifice at the ta ke-off of the vessel from the aortic cusp. Congenital QT-prolongation can resu lt i n sudden death and can occasionally be diag nosed from a n ECG. Arrhythmogenic RV dysplasia and Brugada syndrome a re other abnormalities that disturb the normal electrophysiologic m i l ieu and i ncrease the risk for sudden death.

Essential Echocardio graphic linages in Adult C on g enital Heart Disease Ellen Mayer Sabik

C

ongenital heart disease is by definition an abnormality in cardiac structure that is present at birth, even if it is not diagnosed until later in life. These defects are usually the result of altered embryonic development of a normal structure or failure of development. Four categories of etiologic agents may be responsible for this abnormal de­ velopment, and these are the same influences that may cause cancers. They include hereditary and chromosomal defects (Table 24. 1), viruses (rubella with patent ductus arteriosus [PDAJ ), chemicals (thalidomide with truncus arteriosus or tetralogy of Fallot) , and radiation (x-irradiation with ventric­ ular septal defects [VSDs] ). Although these agents cause cer­ tain known defects, most defects have no specific cause and the etiology may in fact be multifactorial. The incidence of congenital heart disease (excluding bicuspid aortic valve and myxomatous mitral valve (MV) disease with mitral valve pro­ lapse [MVP]) is approximately 0.5% to 0.8% of live births. Congenital cardiac malformations are much more common in stillbirths than in live births. Some congenital lesions have a high rate of survival without surgery and may be seen in the unoperated adult with different relative frequencies (Table 24.2). Other lesions, with worse prognosis, are usu­ ally not seen in adults. However, as both diagnosis and treat­ ment (both medical and surgical) improve, more of these patients are surviving into adulthood and are more likely to be seen in a cardiology office as adults. Thus, all cardiologists should be familiar with the lesions discussed in this chapter. AT R I A L S E PTA L D E F ECTS Atrial septal defect (ASD) accounts for 22% of adult congen­ ital defects. Excluding bicuspid aortic valves and MVP, ASDs

are the most common form of adult congenital heart disease. They make up 10% of all congenital heart defects and dem­ onstrate a female-to-male preponderance of 3 : 2 . Diagnosis of ASD is aided by the following features: • On auscultation, a wide fixed split S2 with a pulmonary flow murmur is heard. • On electrocardiogram (ECG) , ostium primum (OP) ASD shows marked left-axis or right bundle branch block (RBBB) with signs of right ventricular (RV) enlargement. There may be first-degree atrioventricular (AV) block. Ostium secundum (OS) ASD is marked by RSR or rSR in V1 , QRS 50mm Hg). b. EKG may show right axis deviation and RVH. c. Pul monary Artery may be dilated on chest X-ray (CXR). d. Pul monary vascular markings may be increased i n severe cases. e. The deg ree of RVH correlates with the severity of PS. Answers 1 . Answer B: A cleft MV is part of a n atrioventricu lar (AV) ca nal defect, which is due to fai l u re of the embry­ onic endocardial cushions to meet and pa rtition the heart norma lly. A complete endoca rdial cushion defect has four com ponents: prim um ASD, cleft MV, i n let VSD, and a widened anterosepta l tricuspid com missu re. A partial AV ca nal defect does not have the VSD. 2. Answer C: The most common form is fusion of the

RCC and LCC, and the mechanism of Al in those patients

CHAPTER 2 4 • ESSENTIAL E C H O CARD I O GRAPHIC IMAGES IN ADULT C O N GENITAL HEART DISEASE

is prola pse of the conjoi ned cusp.The conjoined cusp i n the case of RCC and LCC fusion is anterior, and thus the Al is di rected posteriorly. At least 50% of patients with coa rctation of the aorta have a bicuspid valve. A bicuspid aortic va lve with severe Al can often be surgica l ly repai red, depending on the expertise of the surgical center. 3. Answer C: A sinus venosus ASD is a defect located near the j u n ction of the i nferior vena cava (IVC) or su perior vena cava (SVC) and the right atrial (RA). It is typica lly d ifficult to see by su rface echocardiog ram, often req u i ring a transesophageal echocardiography (TEE) for d iagnosis. It is usual ly associated with d rain­ age of the right pulmonary vei ns to the RA. 4. Answer E: Ebstei n's anomaly of the TV is character­ ized by apical displacement of the TV i nto the RV. As a resu lt, a portion of the RV becomes atrial ized. TV tissue is dysplastic, with portions of the septa l and i nferior leaflets becoming adherent to the RV. Clin ica l manifes­ tations depend on associated conditions. An i m porta nt associated defect is PS or atresia. Other associations include pri m u m ASD and VSD, and congenita l ly cor­ rected tra nsposition of the g reat vessels. 5. Answer C: The turbulent, high-velocity jets pro­ d uced by the mem brane damage the aortic va lve over time, and patients often develop Al that req u i res su rgery.The su baortic mem brane is a fixed obstruction, which req uires the left ventricle to develop high i ntra­ cavita ry pressu res for ejection. As the left ventricu l a r (LV) p u m ps against t h e fixed obstruction, LVH develops (sim i l a r to what is seen with va lvu l a r AS). Su baortic mem bra nes are known to recur occasionally postresec­ tion, although the freq uency with which this occurs is un known.

293

6. Answer B: Patients with a bicuspid aortic valve

have a n aortopathy i nvolving cystic medial necrosis and decreased expression of fi b ri l l i n - 1 with a tendency towa rd aneurysm formation and a n increased risk for aortic dissection. The g uideli nes used for timing of aortic surgery for a dilated aorta i n a patient with a bicuspid aortic va lve are the same as those used in Ma rfa n's patients. 7. Answer A: Secundum ASDs are the most common form of ASD at 75% with pri m u m ASD representing 20%, sinus venosus ASDs in 5% and unroofed coronary sinus ASDs bei ng rare. Supracristal is a type ofVSD, not ASD. 8. Answer C: A prim um ASD is part of either a partial

or com plete AV ca nal defect. The other features of a complete AV canal defect include i n let VSD, cleft MV, and Widened a nterosepta l tricuspid com missu re. The featu res ofTetralogy of Fa l l ot include VSD, RVH, l nfu n­ dibular PS, and overriding aorta. 9. Answer D: Fifty percent of patients with coa rcta­ tion of the aorta have a bicuspid AV; however, the percentage of patients with bicuspid AVs who have a coa rct is m uch smaller. CW Doppler through the proxi­ mal descending aorta in a patient with a coarctation of the aorta does show a high peak g rad ient as wel l as a g radient that persists into d iastole. Pan d iastolic flow reversa l i n the descending aorta by PW Doppler is char­ acteristic of severe Al, not a coarctation of the aorta. 1 0. Answer D: All of the above are true i n a patient with PS except answer d. In fact pul monary vascu lar markings may be decreased i n patients with severe PS due to decreased flow to the lungs due to the severe obstruction to flow at the level of the pul monic va lve. There is not a n increase i n l u ng markings i n these patients.

Tw-elve-Lead Electrocardio gra p hy Gregory G . Bashian and Curtis M . Rimmerman

T

he electrocardiogram (ECG) is an essential diagnostic test. In many ways, it is an ideal diagnostic modality because it is noninvasive, is readily performed with­ out discomfort or potential patient injury, is inexpensive, and its results are immediately available. Most important, it provides a diagnostic window of cardiovascular surveillance for a multitude of cardiac pathophysiologic problems, in­ cluding valvular, myocardial, pericardial, and ischemic heart disease. The ECG's diagnostic utility is critically dependent on its accurate interpretation. This chapter addresses the diagnostic possibilities encountered in routine ECG inter­ pretation, including a broad collection of clinical examples. A clinical history, detailed interpretation, and diagnostic summary are included for each tracing. A detailed review of this chapter will provide comprehensive preparation for the Cardiovascular Medicine Subspecialty Board Examination.

B OA R D P R E PA R AT I O N To receive a passing score on the Cardiovascular Medicine Subspecialty Board Examination, the examinee must also receive a passing score on the ECG subsection. To best prepare for the ECG section, familiarization with the scoring sheet is essential. The scoring sheet is sent to each examinee before the test date, with diagnoses grouped systematically for easy refer­ ence. In preparation, understanding and being able to recog­ nize each diagnosis is a "foolproof' preemptive approach. A R E CO M M E N D E D A P P ROAC H TO E L E C T R O C A R D I O G RA M I N T E R P R E TAT I O N To ensure accurate and consistent ECG interpretation, a sys­ tematic approach is required. ECG interpretation is not an exercise in pattern recognition. To the contrary, employing a methodical strategy based on a thorough knowledge of the cardiac conduction sequence, cardiac anatomy, cardiac physiology, and cardiac pathophysiology can be applied to all ECGs, regardless of the findings. 294

One systematic approach to each ECG is to ascertain the following in this recommended order: 1.

Assess the standardization and identify the recorded leads accurately

2. Determine the atrial and ventricular rates and rhythms. 3. Determine the P-wave and QRS-complex axes.

4.

5.

6.

Measure all cardiac intervals. Determine if cardiac chamber enlargement or hypertrophy is present. Assess the P-wave, QRS-complex, and T-wave morphologies.

7. Draw conclusions and correlate clinically

Cardiac pathology is manifest differently on the surface ECG, depending on which lead is interrogated. Each lead provides an "electrical window of opportunity" and, by this virtue, offers a unique electrical perspective. The experienced electrocardi­ ographer amalgamates these different windows into a men­ tal three-dimensional electrical assessment, drawing accurate conclusions pertaining to conduction system and structural heart disease. For example, precordial lead V1 predominantly overlies the right ventricle, explaining why right ventricular cardiac electrical events are best observed in this lead. Like­ wise, precordial lead V6 overlies the left ventricle. This lead optimally represents left ventricular cardiac electrical events.

Assess the Standard ization and Identify the Recorded Leads Accurately Standard ECG graph paper consists of 1-mm x 1 -mm boxes divided by narrow lines, which are separated by bold lines into larger, 5-mm x 5-mm boxes. At standard speed (25 mm/s) , each small box in the horizontal axis repre­ sents 0.040 second (40 milliseconds) of time and each large box represents 0.200 second (200 milliseconds) . At standard calibration ( 1 mV/10 mm) , each vertical small box represents 0 . 1 mV and each vertical large box represents 0 .5 mV One must be very careful to inspect the standardization square wave (1 mV in amplitude) at the left of each ECG to

CHAPTER 2 5 • TWELVE-LEAD ELE CTRO CARD IO GRAPHY determine the calibration of the ECG. ECGs with particu­ larly high or low voltages are often recorded at half stand­ ard or twice standard, respectively In these cases, the 1 -mV square wave possesses an amplitude of either 5 or 20 mm. This distinction is important because it will affect the inter­ pretation of all other voltage criteria. All further references to amplitude in this chapter are under the assumption of the default or preset standardization ( 1 mV/10 mm) .

Determine the Atrial and Ventricular Rates and Rhythms The first step in determining the rate and rhythm is to iden­ tify atrial activity If P waves are present, it is important to measure the P wave to P wave interval (P-P interval) . This determines the rate of atrial depolarization. To estimate quickly either an atrial or ventricular rate on a standard 1 2-lead ECG, one can count the number of 5-mm boxes in an interval and divide 300 by that number. For example, if there are four boxes between P waves, the rate is 300 divided by 4, or 75 complexes per minute. Once the atrial activity and rate are identified, the P-wave frontal plane axis should be ascertained. A normal P-wave axis (i.e. , -0 to 75 degrees) typically reflects a sinus node P-wave origin. A simple way of determining a normal P-wave axis is to confirm a positive P-wave vector in leads I, II, III, and aVF An abnormal P-wave axis supports an ectopic or non-sinus node P-wave origin. Several possible atrial and junctional rhythms are listed below. They are grouped by cardiac rhythm origin and sub­ sequently subcategorized by atrial rate. Distinguishing fea­ tures are italicized for emphasis. Rhyt h m s of S i n u s Nodal and Atria l Orig i n

A normal sinus rhythm (NSR) is defined as a regular atrial depolarization rate between 60 and 1 00 per minute of sinus node origin, as demonstrated by a positive P-wave vector in leads I, II, III, and aVF

295

This may be caused by frank sinus arrest or may be simply a sinus pause secondary to: • Nonconducted premature atrial contraction (PAC) , in which case, a P wave can be seen deforming the preceding T wave • Sinoatrial block (SA block) , which, like atrioventricular (AV) nodal block, has several forms

First-degree SA block involves a fixed delay between the depolarizing SA node and the depolarization exiting the node and propagating as a P wave. Because the delay is fixed, this delay cannot be detected on the surface ECG. Second-degree SA block has two varieties. In Type I (Wenckebach) SA block, there is a progressive delay between SA nodal depolarization and exit of the impulse to the atrium. This is manifest as a progressive shortening of the P-P inter­ val until there is a pause, reflecting an SA node impulse that was blocked from exiting the node. In Type II SA block, there is a constant P-P interval with intermittent pauses. These pauses also represent an SA node impulse that was blocked from exiting the node. However, in this case, the duration of the pause is a multiple of the basic P-P interval. Third-degree SA block demonstrates no P-wave activity; as no impulses exit the sinus node. On the surface ECG, this is indistinguishable from sinus arrest, in which there is no sinus node activity Sinus Node Reentrant Rhythm Sinus node reentrant rhythm is characterized by a reentrant circuit involving the sinus node and perisinus nodal tissues. Given the sinus ori­ gin, the P-wave morphology and axis are normal and indistin­ guishable from a normal sinus P wave. The rate is regular at a rate of 60 to 1 00 per minute. (This is very similar to NSR, except characterized by abrupt onset and termination.)

Normal Sinus Rhythm

regular atrial depolarization rate 1 60 milliseconds.)

Sinus Arrest or Pause Sinus arrest or pause is characterized by a pause of >2.0 seconds without identifiable atrial activity

Sinus Node Reentrant Tachyca rd ia

Sinus node reentrant tachycardia is characterized by a reentrant circuit involving the sinus node and perisinus nodal tissues. Given the sinus origin, the P-wave morphology and axis are normal and indistinguish­ able from a normal sinus P wave. The rate is regular at a rate of ,?1 00 per minute. (This is very similar to sinus tachycardia, except characterized by abrupt onset and termination.) Ectopic Atrial Rhythm Ectopic atrial rhythm is character­ ized by a regular atrial depolarization at a rate of 60 to 1 00 per minute from a single nonsinus origin, as reflected by an abnor­ mal P-wave axis. The PR interval may be shortened, par­ ticularly in the presence of a low ectopic atrial origin, closer to the AV node with a reduced intra-atrial conduction time. In the presence of slowed atrial conduction, the PR interval may be normal or even prolonged. Ectopic Atrial Bradycardia Ectopic atrial bradycardia is characterized by a regular atrial depolarization at a rate of :::; 60 per minute from a single nonsinus origin, as reflected by an abnormal P-wave axis. (This is similar to an ectopic atrial rhythm, except slower.)

296

SECTION V • CARD IAC ELECTROPHYSIOLOGY

Atrial Tachyca rd ia

Atrial tachycardia is characterized by a regular, automatic tachycardia from a single, ectopic atrial focus typically with an atrial rate of 1 80 to 240 per minute. The ventricular rate may be regular or irregular, depending on the AV conduction ratio. The P-wave axis is abnormal, given the ectopic atrial focus. (This is similar to an ectopic atrial rhythm, except faster.) Wa ndering Atrial Pacemaker

A wandering atrial pace­ maker (WAP) has a rate of 60 to 1 00 per minute from multi­ ple ectopic atrial foci, as evidenced by at least three different P-wave morphologies on the 12-lead ECG, possessing vari­ able P-P, PR, and R-R intervals. Be careful not to confuse this dysrhythmia with atrial fibrillation (AF) . Unlike AF, discrete P waves are identifiable. M u ltifoca l Atrial Tachyca rd ia Multifocal atrial tachycar­ dia (MAT) is characterized by a rate of >1 00 per minute with a P wave preceding each QRS complex from multiple atrial ectopic foci, as evidenced by at least three different P-wave morphologies on the 1 2-lead ECG possessing variable P-P, PR, and R-R intervals. The ventricular response is irregu­ larly irregular, given the unpredictable timing of the atrial depolarization and variable AV conduction. Nonconducted atrial complexes during the ventricular absolute refractory period are also often present. Be careful not to confuse this dysrhythmia with AF Unlike AF, discrete P waves are iden­ tifiable. (This is similar to WAP, but the atrial rate is faster.) Atrial Fibril lation

AF is characterized by a rapid, irregular; and disorganized atrial depolarization rate of 400 to 600 per minute devoid of identifiable discrete P waves, instead character­ ized by fibrillatory waves. In the absence of fixed AV block, the ventricular response to AF is irregularly irregular. Be careful not to con­ fuse this dysrhythmia with WAP or MAT. The key is the lack of identifiable P waves. Atrial Fl utter

Atrial flutter (AFL) is characterized by a rapid, regular atrial depolarization rate of 250 to 350 per minute, rep­

resenting an intra-atrial reentrant circuit. The atrial waves are termed "flutter waves" and often demonstrate a "saw­ toothed" appearance, best seen in leads V 1 , II, III, and aVF Although the atrial rate is regular, the ventricular response rate may be either regular or irregular, depending on the pres­ ence of fixed versus variable AV conduction. Common AV conduction ratios are 2 : 1 and 4: 1 . Rhyt h m s of Atrioventric u l a r Nodal a n d J u n ctional Orig i n Atrioventricular Noda l Reentra nt Tachycardia Atrio­ ventricular nodal reentrant tachycardia (AVNRT) is a micro-reentrant dysrhythmia that depends on the pres­ ence of two separate AV nodal pathways . Slowed conduction is present in one pathway and unidirectional conduction block is present in the second pathway. This is a regular rhythm with a typical ventricular rate of 1 40 to 200 per

minute, with abrupt onset and termination. Its onset is often initiated by premature atrial complexes (PA Cs) . Atrial activity typically consists of inverted or retrograde P waves occurring before, during, or after the QRS complex, best iden­ tifi ed in lead V1 . The QRS complex may be conducted nor­

mally or aberrantly. Atrioventricu lar Reentra nt Tachyca rdia

Atrioventricular reentrant tachycardia (AVRT) is a macro-reentrant circuit that consists of an AV nodal pathway and an accessory pathway. This dysrhythmia may conduct antegrade down the AV nodal pathway with retrograde conduction through the accessory pathway (orthodromic AVRT) , or antegrade down the acces­ sory pathway with retrograde conduction up the AV nodal pathway (antidromic AVRT) . As opposed to AVNRT, the P wave is always present after the QRS complex. With antidro­ mic AVRT, the QRS complex, by definition, is aberrantly conducted (wide) . J u n ctional Prematu re Complexes

Junctional premature complexes are premature QRS complexes of AV nodal origin that may have resultant retrograde P waves (a negative P-wave vector in leads II, III, and aVF) occurring immediately before (with a short PR interval), during, or after the QRS complex. AV J u nctional Bradyca rd ia

AV junctional bradycardia is characterized by QRS complexes of AV nodal origin that occur at a regular rate of 100 milliseconds) at a rate

CHAPTER 2 5 • TWELVE-LEAD ELE CTRO CARD IO GRAPHY of 1 00 per minute. A second set of P waves is noted as a constant P-P interval at a slightly longer P-P interval compared to the conducted P waves. This represents the native atrium in this cardiac transplant patient, which is still depolarizing via the native sinus node. The donor P wave that immediately precedes each QRS complex demonstrates first-degree AV block. Dif­ fuse nonspecific ST-T changes are present. QRS-complex frontal-plane right-axis deviation is present in that the QRS­ complex vector is negative in lead I and positive in leads II, III, and aVF Low-voltage QRS complexes are seen in the limb

leads, in that each complex is 60 per minute. The PR interval is prolonged to 220 milliseconds, representing first-degree AV block. Left atrial abnormality is seen, as the P-wave morphology is terminally negative in lead V 1 and bifid in lead II. Diffuse nonspecific ST-T changes are pre­ sent. Most important, each lead demonstrates a prominent prolonged QT interval. Both the QT-interval prolongation and ST-segment scooping are secondary to the concomitant quinidine effect and digitalis effect.

Commentary QT-interval prolongation in the setting of quinidine admin­ istration may represent quinidine toxicity and near-future proarrhythmia. Comparison with prior ECGs is important to confirm whether this is a new or preexisting finding. Keyword Diagnoses NSR First-degree AV block Left atrial abnormality Prolonged QT interval Nonspecific ST-T changes Digitalis effect Quinidine effect

CHAPTER 2 5 • TWELVE-LEAD ELE CTRO CARD IO GRAPHY

1

f

r

E LECTROCA R D I OG RAM # 2 7

1

I r

r

i f

f

329

fl'

n�zrrr� ' . ,, +1 +1d 1 r 'I ' , n�� � ,

.

IT

I

1 · '

r

!- -

_,__.

'25mmfs; IOmmfmV !OOH�

__

,

·--·-�

OOli(}

1 2ST. 7!t

C l /): 1 2

Clinical History A 77-year-old woman status post an acute left middle cerebral artery occlusion and urokinase administration is now experiencing recurrent atrial arrhythmias. Medications at the time of this ECG included diltiazem, topical nitroglyc­ erin, and isosorbide mononitrate. An echocardiogram per­ formed during this hospitalization demonstrated moderate left atrial enlargement and normal left ventricular systolic function without evidence of a prior myocardial infarction. Electrocardiogra m I nterpretation This ECG demonstrates two P waves for each QRS com­ plex, best seen in lead aVF The second P wave occurs on the downslope of the S wave at the beginning of the ST seg­ ment. This represents ectopic atrial tachycardia with 2: 1 AV conduction. There are small narrow inferior Q waves that are not of diagnostic significance.

Commentary When interpreting ECGs that show arrhythmias, it is impor­ tant to survey each lead, which may yield a subtle and differ­ ent clue. For this tracing, regular atrial activity is seen best in the inferior leads. Other leads such as lead V1 demonstrate nearly isoelectric atrial activity and suggest a junctional tach­ ycardia. When a tachycardia is present, it is important to ascertain the shortest P-P interval and compare it to the R-R interval. Without this approach, 2 : 1 AV conduction may be overlooked. Keyword Diagnoses Ectopic atrial tachycardia 2 : 1 AV conduction

330

SECTION V • CARD IAC ELECTROPHYSIOLOGY

E L E C T R O C A R D I O G RA M # 2 8

Clinical History A 66-year-old man status post recent coronary artery bypass graft surgery, paroxysmal AF, and a cerebrovascular acci­ dent has returned for a follow-up evaluation after his bypass surgery Other comorbidities include hypertension, non­ insulin-requiring diabetes mellitus, and hyperlipidemia. Electrocardiogram I nterpretation NSR is present. The 9th P wave is premature, reflecting a premature atrial complex with a similar QRS-complex mor­ phology The QRS-complex duration is prolonged but 1 00 per minute. The frontal-plane QRS­ complex axis demonstrates right-axis deviation, as the QRS­ complex vector is negative in lead I and positive in leads II, III, and aVF. There are diffuse low-voltage QRS complexes. Nonspecific ST-T changes are also seen. Alternation of the QRS-complex voltage, best seen in rhythm strip lead V 1 , is apparent. This alternation occurs with every other QRS complex and is termed electrical alternans. Electrical alter­ nans is an ECG marker of a large pericardial effusion.

����

r�� ���

Commentary The ECG findings of diffuse low-voltage QRS complexes and electrical alternans suggest the presence of a significant pericardial effusion and cardiac tamponade. The electrical alternans is secondary to the beat-to-beat variability of car­ diac position. This is sometimes referred to as a "swinging heart." Keyword Diagnoses Sinus tachycardia Right-axis deviation Nonspecific ST-T changes Low-voltage QRS Electrical alternans Pericardial effusion Cardiac tamponade

CHAPTER 2 5 • TWELVE-LEAD ELE CTRO CARD IO GRAPHY

337

E LECTROCA R D I OG RAM # 3 5

Clinical History A 69-year-old woman with a history of severe subaortic ste­ nosis presented to the hospital with a several-day history of dyspnea consistent with congestive heart failure. A cardiac catheterization demonstrated a 1 00-mm Hg pressure gradi­ ent between the left ventricular outflow tract and the left ventricle. Her medications at the time of this ECG included diltiazem, furosemide, and doxazosin.

Com mentary Most often, myocardial ischemia is a bedside diagnosis and requires clinical correlation. In this case, the down-sloping ST-segment depression in the setting of a congestive heart failure exacerbation and subaortic stenosis most likely does represent myocardial ischemia. To confirm this suspicion, a follow-up tracing should be obtained after treatment, to dem­ onstrate interval improvement and ST-T-change resolution.

Electrocardiogra m I nterpretation A P wave of normal axis precedes each QRS complex at a reg­ ular rate of approximately 1 1 0 per minute, reflecting sinus tachycardia. An rSR' QRS complex is present in lead V 1 with a QRS-complex duration of 140 milliseconds, consistent with complete RBBB. The P wave in lead V 1 demonstrates a terminal negativity and is bifid in lead II, supporting left atrial abnormality. Down-sloping 3- to 4-mm ST-segment depression is present in leads V4 to V6 , I, and II, consistent with myocardial ischemia and possibly an NSTEMI.

Keyword Diagnoses Sinus tachycardia Complete RBBB Left atrial abnormality Myocardial ischemia

338

SECTION V • CARD IAC ELECTROPHYSIOLOGY

E L E C T R O C A R D I O G RA M # 3 6

t

25mm'•

!OmmlmV IOOIV,

I

005C

12SL 78

ClD:

6 •

Clinical History A 29-year-old woman who was 3 7 weeks pregnant was admitted to the hospital for close observation of pregnancy­ induced hypertension. She has known complete heart block without cardiovascular symptoms requiring no specific treat­ ment or evaluation other than periodic Holter monitoring. Electrocardiogram I nterpretation On this tracing, the cardiac rhythm is best discerned in rhythm strip lead V 1 . P waves occur at regular intervals at an atrial rate of approximately 85 per minute. The P-wave axis as ascertained in leads I, II, and aVF is upright and normal. This suggests NSR. The PR interval varies and sug­ gests a lack of association between the P waves and the QRS complexes. The QRS complexes are of normal duration and occur regularly at a rate of approximately 45 per minute.

These findings collectively support NSR, junctional brady­ cardia, and complete heart block.

Commentary The ECG criteria for complete heart block include two inde­ pendent cardiac rhythms, lack of AV association, and a non­ competing ventricular rhythm that is slower than the atrial rhythm. Keyword Diagnoses NSR Junctional bradycardia Complete heart block

CHAPTER 2 5 • TWELVE-LEAD ELE CTRO CARD IO GRAPHY

339

E LECTROCA R D I OG RAM # 3 7

Clinical History A 72-year-old man was admitted to the hospital for further evaluation of an erythematous and bullous eruptive rash. His past medical history includes hypertension and chronic obstructive pulmonary disease, for which he takes pred­ nisone and numerous inhalers. Electrocardiogra m I nterpretation The ventricular rate is rapid, irregular, and > 1 00 per minute, representing a tachycardia. Each QRS complex is preceded by a P wave of differing morphology and PR-interval dura­ tion. This represents MAT. Nonspecific ST-T changes are present in the lateral leads.

Commentary MAT is a common dysrhythmia in patients with advanced chronic obstructive pulmonary disease. This dysrhythmia commonly demonstrates resistance to pharmacologic therapy and is best addressed by treating the underlying condition, in this case the chronic obstructive pulmonary disease. Keyword Diagnoses MAT Nonspecific ST-T changes

340

SECTION V • CARD IAC ELECTROPHYSIOLOGY

E L E C T R O C A R D I O G RA M # 3 8

Clinical History A 53-year-old man with diffuse coronary artery disease status post inferior and anterior myocardial infarctions 1 5 years prior to this ECG returns for routine cardiology follow-up. Subsequent to the myocardial infarctions, the patient underwent ventricular aneurysmectomy He contin­ ued with symptoms of stable angina pectoris in the setting of mild mitral insufficiency and moderate left ventricular systolic dysfunction. His medications include digoxin, furo­ semide, and captopril. Electrocardiogram I nterpretation On this tracing, with many findings, a systematic approach is necessary. Sinus bradycardia is present. The PR interval is prolonged, indicating first-degree AV block. The QRS­ complex axis is deviated leftward secondary to diagnostic Q-wave formation in leads II, III, and aVF, supporting an age-indeterminate inferior myocardial infarction. Addi­ tional Q waves are noted in leads V2 to V4 , represent­ ing an age-indeterminate anterior myocardial infarction. Premature complexes differing from the native QRS­ complex morphology are seen without a preceding P wave . These are premature ventricular complexes (PVCs) . The PR interval immediately following each PVC is prolonged

and reflects retrograde concealed conduction of the PVC into the conduction system slowing antegrade conduction to the ventricle . Unlike most PVCs, there is no compensa­ tory pause and therefore these are classified as interpolated PVCs.

Commentary Frequently, ECGs demonstrate a myocardial infarction in two separate myocardial territories, as demonstrated on this tracing. The PVCs are of complete RBBB morphology and therefore are left ventricular in origin. They demonstrate prominent inferior and anterolateral Q waves, supporting the presence of both prior myocardial infarctions. Keyword Diagnoses Sinus bradycardia First-degree AV block Inferior myocardial infarction, age indeterminate Anterior myocardial infarction, age indeterminate Interpolated PVC Concealed conduction

CHAPTER 2 5 • TWELVE-LEAD ELE CTRO CARD IO GRAPHY

341

E LECTROCA R D I OG RAM #39

Ill

n� I

VI

Vf>

25mm/s

i

lOmm/mV lOOHz

'-'----'---'---

005C

l2 S6

78

_ _ _ �---

-�...,____...__........ .. __. ... __.._....___...__...__........ .. .... ... .... ... _.._ ._ ....� .. ..._...._........ ... ... .. _...._

Clinical History A 5 7-year-old woman with a history of adenocarcinoma of the rectum and a pulmonary embolism presented to the hos­ pital urgently, secondary to severe shortness of breath and respiratory failure. Pulmonary angiography demonstrated evidence of both acute and subacute pulmonary emboli and severe pulmonary hypertension. The patient expired shortly after this ECG. Electrocardiogra m I nterpretation NSR is present. Frontal-plane QRS-complex right-axis devi­ ation is noted, given the positive QRS-complex vector in leads II, III, and aVF and a negative QRS-complex vector in lead I. Incomplete RBBB is best seen in lead V 1 with an rsR' QRS-complex pattern. Also notable in lead V 1 is a terminally negative P-wave vector suggesting left atrial abnormality In lead II, the P wave is peaked and 3 mm in amplitude, sup­ porting right atrial abnormality Nonspecific ST-T changes are noted throughout the tracing. Given the incomplete RBBB, right atrial abnormality, and right-axis deviation, RVH with secondary ST-T changes merits consideration.

Page I of I

Commentary This ECG is consistent with an acute pulmonary embolism. It demonstrates a dominant S wave in lead I and a Q wave with T-wave inversion in lead III. This is the so-called S l , Q3 , T 3 QRS-complex pattern described in the setting o f an acute pulmonary embolism. Keyword Diagnoses NSR Right-axis deviation Incomplete RBBB Left atrial abnormality Right atrial abnormality Nonspecific ST-T changes Pulmonary embolism

342

SECTION V • CARD IAC ELECTROPHYSIOLOGY

E L E C T R O C A R D I O G RA M # 4 0

Clinical History A 4 1 -year-old man with a history of intravenous substance use, endocarditis, and prior mitral and tricuspid valve replacement re-presents with symptoms and signs of con­ gestive heart failure. He has also noted recent-onset palpi­ tations. Electrocardiogram I nterpretation This ECG demonstrates a regular narrow QRS-complex tachycardia. P waves are possibly seen within the nadir of the ST segment in lead III. Determination of the exact car­ diac rhythm is difficult and would require further testing in the form of an electrophysiology study Therefore, this is best categorized as a supraventricular tachycardia. The QRS-com­ plex frontal-plane axis demonstrates right-axis deviation, as the QRS-complex vector is negative in lead I, isoelectric in lead II, and positive in leads Ill and aVF A prominent rsR' QRS complex of normal duration is seen in lead V 1 . In the

presence of QRS-complex frontal-plane right-axis deviation, this represents RVH. Diffuse nonspecific ST-T changes are also present.

Commentary This patient was known to have advanced tricuspid valvu­ lar heart disease, prosthetic valve mitral stenosis, pulmonary hypertension, and RVH. The atrial arrhythmias may be sec­ ondary to the cardiac valvular abnormality Keyword Diagnoses Supraventricular tachycardia Right-axis deviation RVH Nonspecific ST-T changes

343

CHAPTER 2 5 • TWELVE-LEAD ELE CTRO CARD IO GRAPHY E LECTROCA R D I OG RAM #41

Page

I

-or I

Clinical History A 67-year-old woman with dialysis-requiring renal failure is recently postoperative after an exploratory laparotomy for an ischemic bowel. This patient became septic, hypotensive, and hyperkalemic. This ECG represents her terminal heart rhythm prior to expiring.

Com mentary In extreme forms of hyperkalemia, ventricular arrhythmias are common, as is profound widening and prolongation of all ECG intervals. The ST-segment elevation in leads V2 and V3 has been termed a dialyzable current of injury

Electrocardiogra m I nterpretation Sinus tachycardia is present. The PR interval is prolonged, representing first-degree AV block. The QRS complex is markedly prolonged, demonstrated by a nonspecific intra­ ventricular conduction delay

Sinus tachycardia

Keyword Diagnoses First-degree AV block Nonspecific intraventricular conduction delay Hyperkalemia

344

SECTION V • CARD IAC ELECTROPHYSIOLOGY

E L E C T R O C A R D I O G RA M # 4 2

lommfmV

IOO�z

0050

1 281.. 78

CID,

I

l

Clinical History A 94-year-old woman was admitted to the hospital with acute-onset diarrhea and dehydration. She was noted to have lower-extremity swelling, and venous Doppler studies dem­ onstrated an acute deep venous thrombosis. A subsequent ventilation perfusion scan was interpreted as high probability for an acute pulmonary embolism. Electrocardiogram I nterpretation In the lead V1 rhythm strip, a P wave is seen preceding each QRS complex. A P wave is also noted immediately following each T wave. This demonstrates a regular P-P interval at an atrial rate of approximately 70 per minute, denoting NSR. The QRS complex is broadened, with an RSR' QRS-complex pattern in lead V1 , suggesting complete RBBB. The T wave is upright in lead V l > supporting primary T-wave changes. The QRS-complex frontal-plane axis is deviated leftward, with a positive QRS-complex vector in lead I and negative QRS­ complex vectors in leads II, Ill, and aVF, consistent with left anterior hemiblock. Given the bifascicular block, the 2 : 1 AV block most likely represents second-degree Mobitz Type II

P�gc

I of

1f

AV block. Diffuse nonspecific ST-T wave changes are seen. Sinus arrhythmia is also documented. The P-P interval encompassing the QRS complexes is shorter than the P-P interval between the QRS complexes. This is more precisely termed ventriculophasic sinus arrhythmia. This has no known clinical significance.

Commentary Given the bifascicular block and 2: 1 AV block, this patient has advanced conduction system disease. It is not known if these findings were new in the setting of her suspected acute pulmonary embolism. Keyword Diagnoses NSR 2 : 1 AV block Complete RBBB Left anterior hemiblock Nonspecific ST-T changes Sinus arrhythmia

CHAPTER 2 5 • TWELVE-LEAD ELE CTRO CARD I O GRAPHY

345

E LECTROCA R D I OG RAM #43 r

r

Clinical History A 6 1-year-old man was seen in cardiology outpatient follow-up after an acute inferior myocardial infarction 3 years prior to this ECG. This was followed by urgent right coronary artery percu­ taneous transluminal coronary angioplasty He feels well, with infrequent episodes of angina pectoris. His medications include metoprolol, aspirin, nicotinic acid, simvastatin, and vitamins. Electrocardiogra m I nterpretation The atrial rhythm is most easily identified in the lead V1 rhythm strip and lead aVF In these leads, P waves are seen to precede each QRS complex at a rate of approximately 85 per minute, representing NSR. The 2nd, 3rd, 8th, 9th, 1 0th, and 1 1 th QRS complexes are wide, with a complete left bundle branch configuration. This represents an AIVR. The native QRS complex is abnormal, with a wide Q wave present in leads Ill and aVF indicating an age-indeterminate inferior myocardial infarction. The first QRS complex is intermediate between the native QRS complex and the AIVR complex and represents a ventricular fusion complex. P-wave activity is seen during the AIVR as a downward deflection within the

1 1_ proximal ST segment of the third QRS complex noted best in leads II, Ill, and V 1 . This supports simultaneous atrial activ­ ity and AV dissociation.

Commentary This patient also had a history of syncope following his myo­ cardial infarction. An electrophysiology study demonstrated readily inducible sustained monomorphic VT, and he under­ went subsequent defibrillator placement. Keyword Diagnoses NSR AIVR Fusion complex Inferior myocardial infarction, age indeterminate AV dissociation

346

SECTION V • CARD IAC ELECTROPHYSIOLOGY

E L E C T R O C A R D I O G RA M # 4 4

Clinical History A 72-year-old man with recently diagnosed myasthenia gravis was admitted for rehabilitation. His past medical his­ tory includes diabetes mellitus, chronic obstructive pulmo­ nary disease, recurrent AF, and coronary artery disease. Electrocardiogram I nterpretation On this ECG, the atrial rhythm is best discerned in lead aVL. This demonstrates a P wave preceding and immedi­ ately following a diminutive QRS complex. This represents AFL with 2 : 1 AV conduction. Another possibility is a rapid ectopic atrial tachycardia. Diffuse nonspecific ST-T changes are present, as are frequent PVCs. The frequent PVCs occur at a constant interectopic interval with a differing coupling interval to the immediately preceding QRS complex. There is evidence of ventricular fusion complexes between the native QRS complex and a PVC. These features together confirm the presence of ventricular parasystole. Commentary This is an unusual tracing, as the atrial rhythm is best dis­ cerned in lead aVL. This underscores the importance of a

systematic evaluation of each ECG lead, particularly in the setting of an atrial dysrhythmia. The P wave immediately fol­ lowing the QRS complex is best seen in lead aVL and allows for the accurate diagnosis of this atrial dysrhythmia. When frequent PVCs are present, it is also important to evaluate for the presence of ventricular parasystole. Ventricular para­ systole is an independent automatic dysrhythmia that dis­ charges at a constant rate from the same ventricular focus.

Keyword Diagnoses AFL 2: 1 AV conduction PVC Nonspecific ST-T changes Ventricular parasystole Fusion complex

CHAPTER 2 5 • TWELVE-LEAD ELE CTRO CARD I O GRAPHY

347

E LECTROCA R D I OG RAM #45

Clinical History A 49-year-old man with recurrent idiopathic left VT was referred for radiofrequency ablation. A recent echocardio­ gram demonstrated normal left ventricular systolic function without evidence of a prior myocardial infarction. His medi­ cations include verapamil, sotalol, simvastatin, and aspirin. Electrocardiogra m I nterpretation This ECG demonstrates a regular wide QRS-complex tachy­ cardia at a rate of approximately 1 75 per minute. This tachy­ cardia demonstrates a complete RBBB morphology with a qR QRS-complex pattern in lead V 1 and terminal S-wave slowing in leads V 1 and V6 . The QRS-complex frontal-plane axis is deviated far leftward, is prolonged, and has a qR QRS­ complex pattern in lead V 1 , suggestive of VT. In the center of the tracing, best seen in leads V1 and aVF, a more narrow QRS complex occurs. This is a sinus capture complex and lends greater support to the diagnosis of VT. In lead aVF, within the sinus capture QRS complex, a prominent Q wave is seen with ST-segment elevation, suggestive of an acute inferior myocardial infarction. Periodic P waves are seen through­ out the tracing, suggesting an independent atrial rhythm and AV dissociation. The precise atrial rhythm diagnosis is

not discernible on this tracing. Wandering baseline artifact is also seen.

Com mentary This ECG contains important features supporting the pres­ ence of VT. When assessing a wide complex tachycardia, each of these features should be specifically sought. They include AV dissociation in the presence of an independent atrial rhythm and sinus capture complexes. Not seen on this ECG but often present in the setting of VT are ventricular fusion complexes. The apparent Q wave occurring in lead aVF remains unexplained, given the patient's normal heart func­ tion and normal regional wall motion on echocardiography: Keyword Diagnoses VT Sinus capture complex AV dissociation Inferior myocardial infarction, acute Baseline artifact

348

SECTION V • CARD IAC ELECTROPHYSIOLOGY

E L E C T R O C A R D I O G RA M # 4 6

n



� Ill a VF

25mmli>

l OmmlmV J OOHz

005C

1 2SL 78

C I D: 1 09

Clinical History A 48-year-old woman presented with severe hypertrophic car­ diomyopathy and pronounced symptoms of exertional dysp­ nea and presyncope immediately status post-percutaneous alcohol ablation of her first septal perforator branch of the left anterior descending coronary artery The patient was resting comfortably in the intensive care unit. Electrocardiogram I nterpretation The cardiac rhythm is NSR, as the P-wave vector is upright in leads I, II, III, and aVF The atrial rate is regular and slightly >60 per minute. Approximately 2 mm of ST-segment eleva­ tion is seen in lead V 1 , and 1 mm of ST-segment elevation is present in lead V2 • Reciprocal ST-segment depression is seen inferiorly and laterally This represents an acute septal myocardial infarction and acute myocardial injury

Page

I

of

l

Commentary The ST-segment elevation in leads V 1 and V2 reflects the proximal septal iatrogenic myocardial infarction created by the alcohol injection. This is a pure proximal septal myo­ cardial injury pattern reflected electrocardiographically The purpose of this procedure is to infarct the proximal inter­ ventricular septum and therefore reduce the degree of left ventricular outflow tract obstruction, avoiding otherwise necessary cardiac surgery Keyword Diagnoses NSR Septal myocardial infarction, acute Acute myocardial injury

CHAPTER 2 5 • TWELVE-LEAD ELE CTRO CARD I O GRAPHY

349

E LECTROCA R D I OG RAM #47

r

...141 f- fill

rI

r

[

:t

I ===--n

I.

I

""-'""25 � m� mls..;.._'--'-'--'--'-������__1;..��....'"-'..: ....""'".:.;...; ...."""' ....: :::....-"-"�=-=-'"'"""'""""'=-"""--='"-'-1 ,� f '--''----"- !L

Clinical History A 48-year-old woman presented with severe hypertrophic cardiomyopathy and pronounced symptoms of exer­ tional dyspnea and presyncope immediately status post­ percutaneous alcohol ablation of her first septal perforator branch of the left anterior descending coronary artery The patient was resting comfortably in the intensive care unit. Electrocardiogra m I nterpretation The cardiac rhythm is sinus bradycardia with a normal P-wave axis slightly ...

£

J: <











VN ERP

200

Cll



300



1 50



1 00 50

0

250

FIGURE 26. 1 4

300

A 1 -A2

350

400

• •

grade conduction down the slow AYN pathway is followed by retrograde conduction to the atria via the slow pathway (Fig. 26. 1 5) . This typical echo beat occurs with atrial activa­ tion occurring within 70 milliseconds of the onset of ventric­ ular activation; on intracardiac EGMs, atrial and ventricular activation occurs near simultaneously: AV Nodal Refractory Periods. The AV Nodal Effective Refrac­ tory Period (AVN ERP) is the longest A 1A 2 interval that fails to conduct through the AYN (Fig. 26. 1 6) . Prolongation may occur with high vagal tone or concomitant medica­ tions. Other refractory periods that can be measured include the AV Nodal Relative Refractory Period (AVN RRP) , which represents the longest A 1A2 • which results in an H 1 H2 > A 1A2 during atrial extrastimulus testing. The AV Nodal Functional Refractory Period (AVN FRP) is the shortest H 1 H 2 interval (AYN output) observed during extrastimulus testing.



500

Coupling Interval (ms)

4 50

I ncremental Ventricular Pacing While not a component of EP testing that directly assesses anterograde AV conduction, incremental ventricular pacing (pacing in the ventricle at faster and faster cycle lengths) can help determine whether retrograde conduction occurs via the AYN (Fig. 26. 1 7) or an accessory pathway (Fig. 26. 1 8) . Atrial activation occurring with a midline activation pattern that decrements with more rapid pacing rates or a shorter premature extrastimulus coupling interval suggests conduc­ tion via the His-Purkinje-AYN system. In this pattern, con­ centric activation is seen in the CS leads with earliest atrial activation occurring at the AYN , septal region, and later activation occurring at more lateral atrial sites (Fig. 26. 1 7) .

Normal AVN conduction cu rve.

Dual AVN Pathway Physiology. AV nodal conduction curves

(Fig. 26. 14) can be plotted (A 1A2 vs. A2 H2 or A 1A2 vs. H 1 H2 ) . A discontinuous AV nodal conduction curve (AH interval jump of >50 milliseconds after a decrease in A1A2 coupling interval of 1 0 milliseconds) suggests the presence of two conduction pathways (typically a fast-conducting AV nodal pathway with a longer refractory period than a slow-con­ ducting AV nodal pathway which has a shorter refractory period) (Fig. 26. 1 5) . Dual AYN physiology is confirmed by the occurrence of an AV nodal echo beat, in which antero-

1 -�---'

·-,.--.

r--1 -·

'"-----1

.v, ...._____j\__'-----i I

\...--,-. __,

i\•l l •

1 40 m•

I

.

. . . ..... . .,.. .. -

1 ---r--A-.

·r · -

--.,

,·-·

VI I �

350

O WHr 1 ,, LJ � \'6 I

'.

1

l

• • • •

400

,.vr I

.§.

e;;

300



200

"'ii �

£

i\ • l f . •10 ""

:c "'

..,,

250

1 50 1 00 50

B

I





• • . .. •

.

• • ••

AVl< .Ur.. �""7 l'Rl" lllO • •

A� Yan r...i...., £Rl'-'?O ..

Baseline

AVN RT 300

A 1 -A2 Coupling Interval 350

FIGURE 26.1 5 AH j u m p and AVN echo beat. A: S i n g l e atrial premature extrasti m u l i a re del ivered after eig ht-beat paced d rive cycles.The AH interva l is 1 40 m i l l iseconds with a cou p l i n g interva l of 290 m i l l iseconds. After a cou pling interva l of 280 m i l l i seconds, the AH interva l "j u m ps" to 470 m i l l i seconds, ind icati ng the presence of d u a l AVN pathway physiology. The atrial EGMs evident i n the CS leads (arrows) ind icate the AVN echo beat with retrog rade conduction to the atria. B: AVN conduction cu rve demonstrati ng an AH j u m p at the fast pathway ERP of 420 m i l l iseconds and i n d uction of echo beats a n d AVN RT.

400

(ms)

450

500

CHAPTER 2 6 • ELECTROPHYSIOLOGIC TESTING

363

i\ V F VI

V6

l l B Ed

H BEp CS 9- IO cs 7-

FIGURE 26.1 6

AV node effective refractory period (AVN ERP).

cs 1 --

In contrast, retrograde conduction via a left lateral free wall accessory pathway (Fig. 26. 1 8) would cause an eccentric activation pattern with earliest ventricular activation occur­ ring near the accessory pathway in the lateral CS leads and earliest retrograde atrial activation in the distal CS as well. PCL of VA Bloch. The longest ventricular PCL associated with failure of retrograde VA conduction is determined Antcf•ts:r-.-k Cu�h.kiioo S 1 n u � Uc.ll

r 7-X

Rt"uo;r.aJc (.'oc)(lu.. 1 i... tt1 Vc:ntri.nab.r P�""t'J Uut

1-- -,, ------1 1----•lt- I

by decremental ventricular pacing. Pacing is performed at shorter and shorter cycle lengths. During retrograde AV nodal conduction, VA intervals gradually increase as pac­ ing cycle length shortens. PCL is shortened until VA block occurs (e.g. , retrograde Wenckebach or 2 : 1 VA block) . In the presence of a typical accessory pathway, a constant VA interval is usually observed, and VA block occurs when the accessory pathway refractory period is reached. This usually occurs with a 2 : 1 VA block pattern, rather than in a retro­ grade Wenckebach pattern. Ventricular Extrastimulus Testing. Analogous to atrial extrastim­ ulus testing, single premature ventricular extrastimuli (V2) are delivered after eight-beat trains of ventricular pacing (V1 ) at several ventricular PCLs (e.g., PCL 600, 500, 400 milli­ seconds). The coupling interval of the extrastimulus (V1V2) is shortened by 10 to 20 milliseconds with each succeeding drive train. In this manner, retrograde VA conduction can be assessed. Decremental retrograde conduction suggests con­ duction is occurring via the His-Purkinje - AVN system. Retro­ grade conduction via a typical accessory pathway is generally nondecremental, unless the accessory pathway is an atypical, decremental pathway In addition, retrograde atrial activation patterns are examined to determine if atrial activation occurs with a typical AV nodal midline activation pattern (Fig. 26. 1 7). TAC H YA R R H Y T H M I A EVA L U AT I O N BY EPS

FIGURE 26.1 7

Normal a nterog rade and retrograde activation.

Ventricular Tachycardia Most patients undergoing EP study for assessment of ven­ tricular arrhythmias have coronary artery disease or dilated cardiomyopathy, and reduced left ventricular function.

364

SECTION V • CARD IAC ELECTROPHYSIOLOGY Retrograde Conduct ion

Antcrogrndc Conduction

Ventricular Paced Ueat

Sinus Beat

VAl-4 lS -----+J� fl----1 RVAJ-4 lS 1 1

l

1 1 1 1 t 1 1 1 1

l

111 1 1 11 1i

l

u1t 1 1 1 1

d 1 uu 111 1 l 111 1 1 11 11 l 1 1 1 1 111ttl1

1 1 1 1 1 t t 1 1

l

----1W\f----

1 1 1 1 1 1 1 1 1

l

1 1 1 1 1 11 tt

In selected patients, EP study may be useful for assessment of risk and need for ICD implantation, drug testing, assess­ ment of device/antitachycardia pacing function, or mapping for ablation. EP testing has limited sensitivity and specificity in the prediction of arrhythmic events in nonischemic dis­ ease. EP studies have been more useful in risk stratification of patients with CAD after MI. Based on data from Multi­ center Unsustained Tachycardia Trial (MUSTT) 4 and Multi­ center Automatic Defibrillator Implantation Trial (MADIT) ,5 survival is improved with ICD implantation in patients with CAD , prior MI, nonsustained VT, and LVEF '.0:40%, and inducible sustained VT or reproducibly inducible VF with double ventricular extrastimuli that is not suppressible with an antiarrhythmic drug (MADIT) .5 These studies provide a rationale for performing EP studies for risk stratification in these patient groups. MADIT Il6 demonstrated the value of prophylactic ICD implantation without EP testing in patients with CAD and LVEF '.O: 30%. DEFINITE7 and SCDHeFT8 studied the value of prophylactic ICD implantation and included patients with nonischemic cardiomyopathy SCD­ HeFT demonstrated survival benefits for ICD implantation in ischemic or nonischemic cardiomyopathy patients with heart failure and LVEF '.0:35 % without the need for EP test­ ing. EP testing is generally not necessary in patients who have criteria that already meet approved ICD indications.

l

1 1 1 11 1111

l

111 1 1 1 111

l

n 1 1 1 1 1 11

I

1 1

FIGURE 26.1 8 Abnormal anterog rade a n d retrog rade activation: left-sided accessory pathway.

Patients with normal LV function and VT usually have special types of ventricular arrhythmias that may be studied by EP testing, particularly in conjunction with mapping and ablation. Ve ntri c u l a r Prog ra m med Sti m u lation Protocol s

Several stimulation protocols have been of utility in stratify­ ing risk for sustained ventricular arrhythmias. Some of the most common are summarized below: RVA, RVOT Pacing sites: PCL 600 and 400 milliseconds Drive cycle lengths: (S 1 ) ; eight-beat drive trains Number of Extrastimuli: 1 to 3 extrastimuli (S 2 to S 4) Decrementing extrastimuli by 1 0 milliseconds starting with last until S2 is refractory RVA, RVOT Pacing sites: Drive cycle lengths: PCL 350, 400, 600 milliseconds (S 1 ) ; eight-beat drive trains Number of Extrastimuli: Four extrastimuli (S 2 -S 5) beginning at 290, 280, 270, 260 milliseconds decrementing 1 0 milliseconds with each drive until S2 is refractory

CHAPTER 2 6 • ELECTROPHYSIOLOGIC TESTING ..,. 1

FIGURE 26.1 9 Determi nation of VERP by ventricular extrastimu­ l u s del ivery. Single ventricu l a r prematu re extrasti m u l i a re i ntro­ d u ced with shortening cou p l i n g interva l s u ntil t h e sti m u l u s fa i l s t o capture the ventricle. I n t h i s exa m p l e, at P C L 4 0 0 m i l l isec­ onds, a ventricular prematu re extrast i m u l u s captu res the ven­ tricle at a cou p l i n g i nterva l of 280 m i l l iseconds, but fa i l s to capture the ventricle at 260 m i l l i seconds.

l l llll l ·Z

,- .....,_.

...

..

· �\) -

-r

-"""r _,_

.._, � t,V4 1•• t

»n

365

....,_...,.,-r-r-- ,--,

-"Vl/l.r---1.11 1

1 101

The VERP is 260 m i l l iseconds at PCL 400 m i l l iseconds.

Short-long-short protocol: RVA, RVOT Pacing sites: PCL 400; six- or eight-beat drive Drive cycle lengths: trains (S 1 ) Number of Extrastimuli: S2 at 600 milliseconds; S3 shorter and decrementing until S3 refractory Ventricular overdrive burst pacing. Ventricular Effective Refractory Period (VERP) . Single ven­ tricular premature extrastimuli are delivered with shorten­ ing coupling intervals until the stimulus fails to capture the ventricle (Fig. 26. 19). The VERP is the longest ventricular extrastimulus V 1V2 that fails to capture the ventricle during ventricular extrastimulus testing. It is measured from pacing stimulus to pacing stimulus and is recorded at different sites (e.g. , right ventricular apical [RVA] , right ventricular outflow tract [RVOT] ) and PCLs (e.g. , 600, 400 until S 2 is refractory) . Ventricular Functional Refractory Period (VFRP) . The VFRP is the shortest ventricular coupling interval produced with premature ventricular stimulation. The VFRP is measured from EGM to EGM and recorded at different sites (e.g. , RVA, RVOT) and PCLs (e.g. , 600 , 400 until S2 is refractory) . Induced Arrhythmias. Definitions of ventricular arrhythmias that can be induced with programmed ventricular stimula­ tion include the following: • Repetitive ventricular responses-1 to 3 PVCs • NSVT-three or more ventricular complexes lasting 30 seconds or requiring earlier termination due to hemodynamic compromise • Sustained monomorphic VT-sustained VT of uniform morphology • Sustained polymorphic VT-sustained VT of multiform morphology

• Pleiomorphic VT-multiple morphologies of monomorphic VT • Ventricular flutter-rapid VT50 milliseconds over a decrement of 1 0 milliseconds in A15 2) ::::} dual AVN physiology (Fig. 26.22 APD2 and APD3) . Induction of AV nodal echo beats o r AVNRT b y occurrence o f block typically i n the fast pathway and conduction delay in the slow pathway allowing recovery for retrograde fast pathway conduction and activation of retrograde atrial depolarization (Figs. 26.22 APD3 and 26.23) Induction of orthodromic AVRT by causing antegrade block in the AP so it is excitable when the impulse returns to conduct retro­ grade to the atrium.

367

CHAPTER 2 6 • ELECTROPHYSIOLOGIC TESTING l n u • Ith) I h m

r'i' I I , ..., .,�,.

A

F

-··-...\, - .....-..

----

Left free wa l l accessory pathway. In s i n u s rhyt h m (left panel}, there is fusion of ventricular activation occurring via the atrial node a n d accessory pathway. D u r i n g atrial pacing and i ntrod uction of prematu re atria l extrasti m u l i (right panel). preexcitation becomes more man ifest as activation via the AVN decrements and becomes later, leavi ng a larger component of ventricu­ lar activation to occur via the accessory pathway.

RA

HBE RVA CSS CS4 CS3

--lv .

A l rbl Pacini:

_.._ _

_}. r--\r---1 --I/ r- Ablat ion itc �

_

-�--·" ��

��.--Vr ----

A

FIGURE 26.21

-�1'"�

_

� 1---

-4

---i-�·-i,-"---­

RVA ----"'f CSS --J---.,L---CS4

c 2

CS2 c 1

____;�(..!'----t----.'----

blocked, slow AVN pathway ERP = AYN ERP), measured from EGM to EGM during atrial extrastimulus testing. Accessory Pathway Anterograde ERP = longest atrial coupling interval that produces a block in accessory pathway conduc­ tion, measured from EGM to EGM during atrial extrastimu­ lus testing. Accessory Pathway Retrograde ERP = longest ventricular cou­ pling interval that produces a block in retrograde accessory pathway conduction, measured from EGM to EGM during ventricular extrastimulus testing. Minimum Preexcited R-R during Atria! Fibrillation. Short R-R intervals suggest a short AP ERP and potential increased risk. Activation Patterns. As discussed above, the pattern of atrial and ventricular activation is examined. The anterograde ventricular activation sequence is the sequence of ventricular

Refractory Periods. As in the ventricle, refractory periods of

the components of the anterograde conduction system can be determined and are defined as follows: Atria! Effective Refractory Period = longest atrial coupling interval (A1 A2) that fails to capture the atrium, measured from pacing stimulus to pacing stimulus. Atria! Functional Refractory Period (AFRP) = shortest atrial coupling interval during premature atrial stimulation (A 1A2) , measured from EGM t o EGM. Fast AVN Pathway ERP = longest atrial coupling interval that produces an AH jump to conduction via the slow pathway, measured from EGM to EGM during atrial extrastimulus testing. Slow AVN Pathway ERP = longest atrial coupling interval that produces a block in slow pathway conduction (if only two pathways are present and the fast pathway has already

Single A P D3 FIGURE 26.22 Single atrial extrast i m u l i . Dual AVN physiology a n d ind uction of single typical AVN echo beat. Single APD, a n d single APD 2 : dec­ rementa l AVN cond uction with longer AH i nterva l after shorter A 1 -A2 cou p l i n g interva l . Decremental AVN conduction is demonstrated with AH 1 40 a n d 1 60 m i l l iseconds with shorten ing of APD cou p l i n g i nterva l (APD, to APDJ Single APD3: AH j u m p (>SO- m i l l i second i ncrease i n AH interva l for a 1 0- m i l l isecond decrease in A,-A2 cou p l i n g i nter­ va l) with single typical AV nodal echo (note atrial activation seen i n the CS with a short VA interva l). APD, atrial prematu re depola rization.

av---- �---'

V1



----�-'

.____,

A-H = 140 ms

·: �rr�CS

C

.

.

R;A34!.3



j

,

,___......., ,

-

1· ____....,_ ., __..,..__l--

......_.__.:-_.i

__

.....

,.,

,

__ _

_..

A-H "' 470 ms

.�.��*ii:.1 I�', J

CSB·7 2

_.CS6-5 2.

RVA3-4 2.

S-!i

l

1,

l

368

..

... .

SECTION V • CARD IAC ELECTROPHYSIOLOGY

.___j\____JL-



,.... ,. ..

"' u

.,,

----.,. . ._

I I-

I r---v

l�t-'V-· ·�r

'j

-

I

,,_ FIGURE 26.23 I n itiation of AVN RT single APDs CS 400/250 AH j u m p, i n itiation of AVN RT.

.... .

activation during sinus rhythm, atrial pacing, atrial extrastim­ uli, or SVT. Eccentric activation of the CS suggests a left-sided accessory pathway (Fig. 26 . 18 left panel) . The atrial activation sequence is the sequence of atrial activation during ventricular pacing, ventricular extrastimuli, or SVT. Eccentric retrograde activation of the CS suggests a left-sided accessory pathway (Figs. 26. 1 8 right panel and 26.24) . Inducible Supraventricular Tachyarrhythmias. Types of SVTs that may be induced include the following: AV node reentrant tachycardia-AYNRT is usually associated with dual AV nodal pathway physiology (discontinuous AYN conduction curves; an AH "jump") (Figs. 26. 1 5 and 26.22). In typical AVNRT, antegrade conduction occurs via the slow

AYN pathway (long AH) and retrograde conduction via the fast AYN pathway with near simultaneous atrial and ventric­ ular activation (Figs. 26.22 and 26.23). In atypical AVNRT, antegrade conduction occurs via the fast AYN pathway (with a short PR) and retrograde conduction via the slow AYN

pathway (long R-P interval) . Atrioventricular reentrant tachycardia-AVRT refers to acces­

sory pathway-mediated reentrant tachycardia. In AVRT, there is 1 : 1 AV association, as the atria and the ventricles are integral components of the reentrant circuit. In orthodro­ mic AVRT, antegrade conduction occurs via the AYN (with a narrow QRS in the absence of bundle branch block/aber­ ration) and retrograde conduction occurs via the accessory

r-------A-�-

FIGURE 26.24 Left-sided acces­ sory pathway. Retrograde atrial activation d u ring ventricular pacing-earl iest retrog rade atria l activation at CS 3 (arrows).

CHAPTER 26 • ELECTROPHYSIOLOGIC TESTING

ffr? )� �: ��A Ji .v AH

1Y FIGURE 26.25 Left-sided acces­ sory pathway mediati ng ortho­ d romic AVRT-earl iest retrograde atrial activation occu rs via a n accessory pathway a t CS 2-3 (VA interva l 95 m i l l iseconds).



�11,

2 6 1 Ill) to 111S

.

I "\J

4-----4""-----'11 ,.__ � .,...,t ,,_ v,__ _1 11 _0_1_n»

__, l r.-"'---I

_ _

.,. .--1 -i..______,,.,....,_._____�---1

__ _

V A 9 S Ill

369

'lr',�·���-.1v-1,,.._���--./\..-1r--���-.J��--l

�..,

�r· ----.fi->�....-----J(..J,�---�J't...11r-----J"..-i-------./fl..-'l.---1

r �'

6

VA

!IS m

H-� :: ::: : �,___.l _�,_____....-- l"k--i

pathway (Figs. 26.25 and 26.26) . In antidromic AVRT, ante­ grade conduction occurs via the accessory pathway (with wide QRS) and retrograde conduction via the AVN or another accessory pathway Atrial flutter-In type I (typical) atrial flutter, right atrial acti­ vation proceeds in a counterclockwise activation pattern through the posterior isthmus between the inferior vena cava and tricuspid annulus. There may also be clockwise activation utilizing the isthmus. Type II (atypical) atrial flutter refers to atrial flutter using non-isthmus-dependent flutter circuits. Atrial tachycardia-Atrial tachycardias may be macroreen­ trant in mechanism, including most incisional or scar-related

atrial tachycardias, or due to ectopic (to the sinus node) foci and/or automatic mechanisms. Atrial fi brillation-This most common sustained clinical arrhythmia typically initiates from pulmonary vein ostial or other focal triggering sites or microreentrant circuits. It may sustain with multiple wandering reentrant circuits. Sinus node reentrant tachycardia-This tachycardia is charac­ terized by a similar P wave morphology to sinus rhythm and may be induced and terminated with premature extrastimuli. Inappropriate sinus tachycardia-Inappropriate sinus tachy­ cardia (IST) is characterized by an inappropriately high resting sinus rate and enhanced sensitivity to adrenergic stimulation.

•••• . . . , , , .. , . . . . . . . . . , . . . . . . . . . , . . . . . . . . . 1 · · · · · · · · · 1 · · · · · · · · · 1 · · · · · · · · · · · · · · · · · · · 1 · · ' , , , , · · · · · · • • • 1 1 1' '' 1 '' ' ''' ' ' '1 ' ' ' ' ' ' ''' ' ' '

.. \'I

1

a\'f v

t

VI

FIGURE 26.26 Left free wa l l accessory pathway mediati ng orthod romic AVRT-ea rl iest atrial activation occu rs in the dista l CS at CS 1 -2 (arrows).

I

1.·s 1 o1 u

--------vv'v--------�-----------vv'v, ' 'IJ' v ' V'

1t v 11 l · • �.�



1

, , , , , • • • 1 • • • • • • • • • 1 • • • • • • • , , 1 • • • • • , • • • • • • • • • • • • 1 • • • • • • • • • 1 • • • • • • • • • 1 • • • • • • • • • t • • • • • • • • • 1 • • • • • • • • • 1 • • • • • • • • • 1 • • • • • • • • • 1 • • • • • • • • • 1 • • •_

370

SECTION V • CARD IAC ELECTROPHYSIOLOGY

FIGURE 26.27 Orthod romic AVRT with single VPD introduced d u ring His refractoriness. The single ventricu l a r extrasti mul u s del ivered d u ring H i s bund l e refractori ness advances retrograde atrial activation, suggesting the presence of a ret­ rog radely conducting accessory pathway, in this case located in the right posterosepta l reg ion.

Evaluation during tachycardia. Once a tachycardia is induced, various observations and maneuvers can be performed to help determine the SVT mechanism. These include: • Morphology: Narrow complex, RBBB or LBBB aberrant conduction, or preexcited • Atrial activation sequence • Ventricular activation sequence • HA or VA interval-short HA interval (< 1 0 0 milliseconds) suggests AVNRT, longer HA intervals (> 1 0 0 milliseconds) suggests orthodromic AVRT mediated by an accessory pathway. • Single ventricular premature extrastimuli during SVT (Fig. 2 6 . 2 7)-If single ventricular premature extrastimuli delivered during His refractoriness advances retrograde atrial activation, then a retrogradely conducting or concealed accessory path­ way is present. However, this only demonstrates the presence of an accessory pathway. It does not prove that the pathway is an integral part of the circuit, as it could be a bystander pathway. • Bundle branch block aberration in SVT (Figs. 26.28 to 26.30)­ VA interval prolongation during aberration in SVT indicates a retrogradely conducting accessory pathway ipsilateral to the bundle branch block. On a surface ECG recording, this may be manifest by a longer cycle length (slower rate) during the wide complex tachycardia/aberration than during narrow complex conduction (Fig. 26.28). The prolongation of the cycle length occurs due to a prolongation of VA conduction times. Bun­ dle branch block aberration ipsilateral to the accessory path­ way results in longer retrograde (VA) activation times due to additional time required for transseptal myocardial conduction (Figs. 26.28 to 26. 30). Demonstration of such a change in VA time with aberration demonstrates the presence of the accessory pathway ipsilateral to the bundle branch blocked and also indi­ cates that the accessory pathway is a component of the reentrant circuit.

Mapping During Ablation A diagnostic EP study is critical to confirmation and definition of arrhythmia substrate prior to ablation of most SVTs and VTs. Currently, various mapping techniques based on deter­ mination of earliest activation sites include the utilization of electrophysiologic recordings and various electroanatomic, contact catheter and noncontact mapping systems that can graphically tag and record activation times in three-dimen­ sional space with computer generation of a display of activa­ tion or voltage maps. Although ablation of some arrhythmias is based on anatomic locations (e.g. , slow pathway region for AVNRT or pulmonary vein antral isolation for atrial fibrilla­ tion ablation) , successful ablation of other tachycardias often requires determination of the earliest site of activation, which helps to determine the location of the targeted arrhythmia substrate. An example is shown in Figure 26.3 1 , which dem­ onstrates the fusion of atrial and ventricular EGMs on the ablation catheter at the site of an accessory pathway Abla­ tion here using radiofrequency energy resulted in prompt ablation of the pathway; loss of ventricular preexcitation, and restoration of normal AV conduction.

S U M M A RY This chapter aims to summarize the components of a com­ prehensive diagnostic EP study For users of this book aim­ ing for cardiovascular board exam review, I would suggest focusing upon: •



Recognition of the His bundle EGM and determination of the sites of AV block (AV nodal vs. infra-Hisian block) ; Recognition of VA dissociation during wide complex tachy­ cardia using intracardiac EGMs, indicating the rhythm is most likely VT;

371

CHAPTER 2 6 • ELECTROPHYSIOLOGIC TESTING

'----

B

--�

FIGURE 26.28 A: Conversion of wide com plex to na rrow complex tachycardia with longer RR i nterva l d u ring wide complex tachyca rd ia. Th is is diag nostic for AVRT with an accessory pathway i psi latera l to the bundle branch block. B: Orthodromic AVRT with ipsilatera l BBB. B B B a berration ipsilatera l to the accessory pathway resu lts in longer retrog rade (VA) activatio n times as a res ult of add ition a l time req u i red for transseptal myoca rd ial conduction.

lltl

.,,

, ..

Hfltl

IUltl

••

,.........,lf l l l U .,

....

111•

1Vl ._J, ---- -- �

VA 150

p

.......

, • , ...... .

I

'' . . . .. .

t1H 11

11

1

u1111111

p

11 u1 u1

1

111 1 11111

1

11111 1tU

1111111

A I Al 3GO

l �u u

111

1

1 1

1 1 11 1 1

11 1 1 1 0

11

1

1 11

1 1 1u

l

uu111

1

1

1111

Hll t lffl

l

lltll1tll

p

--�------ / \.__��--�-

_

VA

80

FIGURE 26.29

I n itiation of orthod romic AVRT with i n itial LBBB a berration. Retrograde VA activation times a re longer d u ring LBBB a berration, ind icat­ ing participation of a left-sided accessory pathway. Loca l VA time measured nea rest the accessory pathway (CS distal 55 m i l l i­ seconds) is s i m i l a r, but earl iest ventricular to atria l activation is longer with LBBB a berration.

J

1111

1

1 111 11 1 1

l

1u11

111

l

1111uut

l

11,u1

,1

l1

1111n11

l

u11tt111

l

uu1

375

"11t u u u 1 1 1lu1u1111

1t 11 1 1 1 1

1

11 1 Ht 1 H

_/'"'...... _

(

\_

11 1u 1111

l111111p1

372

SECTION V • CARD IAC ELECTROPHYSIOLOGY

FIGURE 26.30 RBBB a berration d u ring AVRT utilizing a right posterosepta l accessory pathway. Retrog rade VA activation times a re longer d u ring RBBB a berra­ tion, confi rming the presence of a rig ht-sided accessory pathway.

• Recognition of the initiation of AVNRT with demonstration of an "AH jump" and induction of an SVT with near simultaneous atrial and ventricular activation;

and later activation in the lateral CS/left atrial or ventricular free wall) ; • Recognition that bundle branch block that manifests during SVT with a longer cycle length or longer VA time indicates the presence of an accessory pathway ipsilateral to the bundle branch block.

• Recognition of a left free wall accessory pathway with abnor­ mal, eccentric early activation via a more distal CS location (e.g. , rather than the normal earliest activation at the septum

_______.,

� � H.�A HBE

RVA

CS I

A

n

A

B

m

PRE·A BLATION H11n1l1 ,.J e.iw!.•m1!.1t•Jo11111t

RF

:..;---

1� --------./�,.-�-----����--../ A B L A T I O N S IT E

��-----...�, -t�

CS3 css

\_.

�-

f...--.-

+

•' --

L�

---�

au11l1...J

1 1!.•••

llP ON

.. ... •• •• J... ....i..

LOSS

OP

PRl!eXCITATIO:f

FIGURE 26.3 1 Left-sided accessory pathway. A: Successfu l a blation site. B: Radiofreq uency ablation.

CHAPTER 2 6 • ELECTROPHYSIOLOGIC TESTING REFERENCES

1 . Epstein AE, DiMarco JP, Ellenborgen KA, et al. ACC/AHA/HRS 2008 Guidelines for Device-Based Therapy of Cardiac Rhythm Abnormalities: A Report of the American College of Cardiol­ ogy/American Heart Association Task Force on Practice Guide­ lines (Writing Committee to Revise the ACC/AHNNASPE 2002 Guideline Update for Implantation of Cardiac Pacemak­ ers and Antiarrhythmia Devices) Developed in Collaboration With the American Association for Thoracic Surgery and Soci­ ety of Thoracic Surgeons. ] Am Coll Cardiol. 2008;5 l ;e l-e62; originally published online May 15, 2008. 2 . Blomstrom-Lundqvist C, Scheinman MM, Aliot EM, et al. ACC!AHA/ESC guidelines for the management of patients with supraventricular arrhythmias-executive summary a report of the American college of cardiology/American heart association task force on practice guidelines and the European society of cardiology committee for practice guidelines (writing com­ mittee to develop guidelines for the management of patients with supraventricular arrhythmias) developed in collabora­ tion with NASPE-Heart Rhythm Society ] Am Coll Cardiol. 2003 ;42 1493-1 53 1 . 3 . Tracy CM, Akhtar M , DiMarco JP, et al. American College of Cardiology/American Heart Association clinical competence statement on invasive electrophysiology studies, catheter abla­ tion, and cardioversion. A report of the American College of Cardiology/American Heart Association/American College of Physicians-American Society of Internal Medicine Task Force on clinical competence. ] Am Coll Cardiol. 2006; 1 14: 1 654-1 668. 4. Buxton AE, Lee KL, Fisher JD, et al. A randomized study of the prevention of sudden death in patients with coronary artery disease. Multicenter Unsustained Tachycardia Trial Investiga­ tors. N Engl ] Med. 1999;3411 882-1890.

373

5 . Moss AJ , Hall WJ, Cannom DS, et al. Improved survival with an implanted defibrillator in patients with coronary disease at high risk for ventricular arrhythmia. Multicenter Automatic Defibrillator Implantation Trial Investigators. N Engl ] Med. 1996;335: 1 933-1940. 6. Moss AJ, Zareba W, Hall WJ , et al. Prophylactic implantation of a defibrillator in patients with myocardial infarction and re­ duced ejection fraction. N Engl ] Med. 2002;346:877-883. 7. Kadish A, Dyer A, Daubert JP, et al. Prophylactic defibrillator implantation in patients with nonischemic dilated cardiomy­ opathy N Engl ] Med. 2004;350 : 2 1 5 1-2 1 58 . 8 . Bardy G H , Lee KL, Mark DB, et al. Amiodarone o r a n implant­ able cardioverter-defibrillator for congestive heart failure. N Engl ] Med. 2005;352 225-237.

S U G G ESTE D A D D ITIONAL READ I N GS Knight BP, Zivin A, Souza J , et al. A technique for the rapid diagnosis of atrial tachycardia in the electrophysiology laboratory. ] Am Coll Cardiol. 1999;33:775-78 1 . Knight BP, Ebinger M , Oral H , et al. Diagnostic value of tachycardia features and pacing maneuvers during paroxysmal su­ praventricular tachycardia. ] Am Coll Cardiol. 2000 ;36:574-582. Zipes D P, Camm A], Borggrefe M, e t al. ACOAHA/ESC 2006 guidelines for management of patients with ventricular arrhythmias and the prevention of sudden cardiac death. A report of the Ameri­ can College of Cardiology/American Heart Association Task Force and the European Society of Cardiology Committee for Practice Guide­ lines (Writing Committee to Develop Guidelines for Management of Patients With Ventricular Arrhythmias and the Prevention of Sud­ den Cardiac Death) Developed in collaboration with the European Heart Rhythm Association and the Heart Rhythm Society Europace 2006;8: 746-837.

QU ESTIONS AND ANSWERS Questions

2. Where is the site of block?

1 . Where is the site of block?

l�

a. b. c. d.

AV node (AVN) I nfra-His I ntra-His AVN and I nfra-His

a. b. c. d.

AVN I nfra-His I ntra-His AVN and I nfra-His

(Con tin ued)

374

SECTION V • CARD IAC ELECTROPHYSIOLOGY

fol l owed by a His defl ection, but no fo l l owing ventricu­ lar EGM or QRS. Th us, the block occurs below the bundle of His (infra-Hisian block) .

3. Where is the site of block?

' ' """'___,__...,....__,..._ . __,,_____""'\/]

a. b. c. d.

AVN I nfra-H is I ntra-His AVN and I nfra-His

4. What is the d iagnosis?

a. b. c. d.

Oj hi I

,.

I

Orthod romic AVRT Left-sided accessory pathway Atrial tachyca rdia AVN reentra nt tachyca rd ia

5 . What is the diagnosis? 0 .;

a. b. c. d.

•. I

11

·• •



I

j i IHI

1

;..; 1,111111 Ii ii I I j"

·• i

"'

""'•'"""

1• • ·

I

'

Left-sided accessory pathway Right-sided accessory pathway AVN reentra nt tachycard ia Sinus tachycardia

Answers 1 . Answer B: The tracing shows atrial pacing with

right bundle branch block (RBBB) and second-deg ree AV block without prolongation of the PR or AH intervals prior to the blocked beat (thi rd paced beat). On this third paced beat, the His electrode shows a n atrial EGM

2. Answer A : T h e tracing shows atrial paci ng ( S , d rive) with 2 : 1 AV block. Inspection of the His bundle EGM tracings demonstrate S, atrial pacing sti m u l i fol l owed by atrial EGMs. After the fi rst paced beat, there is a His bundle EGM fol l owed by a ventricular EGM and QRS on the su rface electroca rd iogram (ECG). After the second paced beat, no His bundle EGM follows the atria l EGM. The next paced beats repeat this pattern. The block is at the level of the AVN, because conduction is blocked prior to a rriva l to the His bund le.

3. Answer D: This tracing shows second-degree AV block d u ring atrial pacing.The His bundle EGM demon­ strates the atrial pacing stimuli followed by atrial EGMs. After the fi rst atrial paced beat, there is a long AH interval followed by a His EGM, but no ventricular EGM or QRS. This beat blocks below the His bundle. After the second paced beat there is a slightly longer AH interval followed by a ventricular EGM on the RVA tracing and a corre­ sponding surface QRS. After the third paced beat, the AH is longer still, but there is no conduction after the His EGM to the ventricles. This beat again shows infra-H isian block. After the fourth paced beat, there is no His electrogram. This beat blocks in the AVN and the series shows AVN Wenckebach occu rring (gradually prolonging AH i nterval followed by block i n the AVN). The fifth paced beat shows conduction after the block with a shorter AH interval followed by conduction to the ventricles.The sixth paced beat shows a small His deflection with slightly longer AH, but i nfra-Hisian block (no ventricular activation). The seventh paced beat shows a slightly longer AH i nterval with conduction to the ventricles. Th us, the tracing demonstrates two levels of block-in the AVN (Mobitz I Wenckebach pattern) and infra-Hisian block.

4. Answer D: The tracing shows a narrow QRS complex tachycard ia with a cycle length of 350 m i l l i seconds. The coronary sinus (CS) atrial EGMs show a concentric atrial activation pattern (earliest at CS 7 to 8 at the septum and later at more dista l CS electrodes) with near simu ltaneous activation of the atri u m and ventricle. The earl iest atrial activation is l i kely the small deflection at the onset of the QRS on the HBE tracing, which actu a l ly slig htly precedes the ventricular activation. Th is pattern is consistent with AVN reentra nt tachycardia. 5. Answer A: This tracing shows a narrow complex tachy­ cardia with cycle length of 370 milliseconds.The antero­ g rade activation occurs via the AVN and H PS (AH seen in HBE 1 to 3 with narrow QRS). The earliest atrial activation occurs in the distal CS at CS 1 to 2.This eccentric activa­ tion pattern indicates retrog rade activation via a left lateral accessory pathway.The tachycardia is consistent with orthodromic AVRT using a retro­ g radely conducting left-sided accessory pathway.

Sudden C ardiac Death and Ventricular Tachy cardia Daniel J. Cantillon and Oussama Wazni

DEFIN ITION OF SU DDEN C A R D I AC D E AT H Sudden cardiac death (SCD) is defined by 2006 American College of Cardiology/American Heart Association/Heart Rhythm Society (ACC/AHA/HRS) guidelines as the abrupt cessation of cardiac activity so that the victim becomes unre­ sponsive, without normal breathing or circulation, which progresses to death in the absence of any corrective meas­ ures. The definition excludes noncardiac conditions such as pulmonary embolus, intracranial hemorrhage, or airway obstruction. However, it does not exclude nonarrhythmic deaths, as the terminal rhythm is often unknown. E P I D E M I O LO G Y O F S U D D E N C A R D I AC D E AT H The incidence o f SCD is estimated to b e 300,000 to 400,000 per year in the United States . The mortality rate is high, with only 2% to 1 5 % of patients reaching the hos­ pital alive . Fifty percent of these hospitalized patients die before discharge. There is a high recurrence rate of 3 5 % t o 5 0 % . More than 75% t o 85% of SCDs are associated with ventricular arrhythmias. The most common arrhyth­ mias are ventricular tachycardia (VT) (62 % ), torsades de pointes (TdP) ( 1 3%), primary ventricular fibrillation (VF) (8 %), followed by bradycardia (7%) . SCD is the first presentation of cardiac disease in 2 5 % of patients. The incidence increases with age at an absolute incidence of 0 . 1 % to 0. 2 % per year. Men are more commonly affected (3 : 1 ) . SCD in patients 35 years old, SCD is most commonly associated

with coronary artery disease. Determinants of survival are rapid external defibrillation and bystander cardiopulmo­ nary resuscitation. Hospital-based cooling protocols are being increasingly adopted to improve the neurologic prognosis for sudden death survivors. R I S K FACTO R S A N D PAT H O P H YS I O LO G Y Risk factors include the following: • Prior cardiac arrest: high recurrence rate, up to 35% to 50% at 2 years • Syncope in the presence of coexisting cardiac diseases • Reduced left ventricular (LV) function and congestive heart failure (CHF) • Ventricular premature contractions and nonsustained ventricu­ lar tachycardia (NSVT) post-myocardial infarction • Myocardial ischemia and/or documented scar • Conduction system disease

The pathophysiology is determined by trigger factors and an underlying substrate conducive to arrhythmia. The mecha­ nism of SCD can be related to any of the following patho­ physiologic mechanisms: • Anatomical reentry around scarred myocardium • Functional reentry using a diseased His-Purkinje system, or areas of nonhomogeneous (anisotropic) conduction • Ischemia, electrolyte imbalance, ion channel abnormalities, surges in neurosympathetic tone, antiarrhythmic drugs • Rapid and irregular ventricular activation (i. e . , atrial fibrillation [AF] with rapid ventricular response in Wolff-Parkinson-White [WPWJ ) . • Bradycardic SCD i s overall uncommon; however, i t remains impor­ tant in specialized situations like cardiac transplant recipients.

375

376

SECTION V • CARD IAC ELECTROPHYSIOLOGY

S U D D E N C A R D I AC D E AT H A N D CO R O N A RY A RT E RY D I S E A S E Coronary artery disease is present in 80% of those with SCD. Approximately 75% have a history of prior myocardial infarc­ tion (MI) . Sudden death can be the first clinical manifestation in up to 25% of patients with coronary artery disease. Approxi­ mately 65% have three-vessel obstructive coronary disease. Risk factors include depressed LV systolic function and frequent ven­ tricular premature depolarizations (VPDs). However, the Car­ diac Arrhythmia Suppression Trial (CAST) study demonstrated that suppression of VPDs with class IC medications resulted in higher mortality Data from large clinical trials have identified groups of patients with coronary artery disease, depressed LV systolic function, and nonsustained ventricular arrhythmias at increased risk for SCD, as discussed later in this chapter. N O N I S C H E M I C CA R D I O M YO PAT H Y Patients with impaired LV systolic function in the absence of coronary artery disease are at increased risk for ventricular tachyarrhythmias and SCD , particularly those with symp­ tomatic heart failure. In the heart failure population, total mortality is approximately 25% at 2 . 5 years, with SCD accounting for 25% to 50% of these cases. Mortality due to SCD is much higher in New York Heart Association (NYHA) classes II and III than in class IV patients, who have excess mortality due to pump failure.

Dilated Cardiomyopathy Fifty percent of deaths in this patient subgroup are arrhythmic. Left ventricular ejection fraction (LVEF) is predictive of sudden death, due to either circulatory failure or fatal arrhythmia. Ven­ tricular ectopy is very common and does not appear to be as predictive of SCD as it is in patients with coronary artery dis­ ease. Up to 80% of patients may have NSVT on Holter moni­ toring. Inducibility for ventricular tachyarrhythmias at the time of electrophysiologic (EP) study is also less predictive when compared to patients with coronary disease and has almost no role in risk stratification. While some modalities such as abnormal microvolt T-wave altemans and heart rate variability have been associated with increased risk in this population, only LVEF :::;3 5% and the presence of symptomatic heart fail­ ure are recommended by practice guidelines for the purposes of risk stratification, particularly regarding selecting candidates for implantable cardioverter-defibrillator (ICD) implantation. Hypertrophic Cardiomyopathy HCM is an autosomal dominant disease with incomplete penetrance associated with ever-increasingly discovered genetic mutations. The overall incidence of SCD is 2 % to 4% in adults and up to 6% in children. It is the most common cause of SCD in young athletes. Maron et al. have identified factors associated with increased risk such as the presence of NSVT, hypotension with exercise, unexplained syncope, septal thickness >3 cm, and a family history of sudden death

��

� 4+ � fLtv-tvFIGURE 27.1

V3

ECG reading i n ARVD.

in a first-degree relative younger than 50 years old. Among patients with HCM and primary prevention ICDs, appro­ priate therapy was uncommon among patients with none of these risk factors and occurred in 14% of patients with one risk factor, 1 1 % in patients with two risk factors, and 1 7% in patients with three or more risk factors. In addition, cardiac magnetic resonance imaging (MRI) has been increas­ ingly utilized for risk stratification in HCM patients as the presence of basal septal scar is associated with VT as well as histopathologic changes. The role of genetic testing for risk stratification has not been established despite the identifica­ tion of certain high-risk mutations, such as the LAMP2 gene.

Arrhythmogenic Right Ventricular Cardiomyopathy Progressive fibrofatty right ventricular tissue replacement is the pathologic hallmark of this disease. There is a strongly familial pattern; its prevalence may be up to 20% worldwide for SCD in young patients (U.S. 3%). MRI is the most useful imaging modality to make the diagnosis. Characteristic epsi­ lon waves may be present on the electrocardiogram (ECG) , as shown in Figure 2 7 . 1 . The frequency and the severity of ventricular arrhythmias in this disease are progressive, thus making these patients candidates for ICD implantation. I N H E R I T T E D A N D ACQ U I R E D C H A N N E LO PAT H I E S

Long-QT Syndrome Long-QT syndrome consists of the inherited abnormali­ ties that prolong cardiac repolarization as measured by the corrected QT (QTc) interval on surface ECG, which confer an

CHAPTER 2 7 • SUDDEN CARDIAC DEATH AND VENTRI CULAR TACHYCARD IA

· · · · · · · · · · · · · · · · · · · · · · · · · · · · · · · · · · · · · · · · · · · · · · · · · · · · · · · · · · · · · · · · · · · · · · · · · · · · · · · · · · · · · · · · · · · · · · · · · · · · · · · · · · · · · · · · · · · · · · · · · · · · · · · · · · · · · · ·

Fa milial Long-QT Synd romes Synd rome

Chromosome

Channel

LQT l LQT2 LQT3 LQT4 LQT5 LQT6 LQT7

l lp l 5 . 5 7q35-36 3p2 1-24 4q25-27 2 1 q22. l-22 .2 2 1 q22. l-22.2 1 7q23

IKs alpha (KVLQT l) IKr (HERG) INa (SCN5A) unknown IKs beta (KCNE l) IKr (KCNE2) K1,2 _ 1 (KCNJ2)

increased risk of SCD by polymorphic ventricular tachycardia (PMVT) or T dP. There are numerous identified mutations involving mostly sodium and potassium ion channels (Table 27. 1). Abnormal QTc cutoff values are commonly selected as >440 milliseconds in men and >460 millisec­ onds in women, although actual cardiac events occur on a skewed curve and are highest among patients with QTc > 500 milliseconds. The most common symptoms associ­ ated with long QT syndrome are palpitations and syncope, although associations also exist with seizure disorders. Characteristic clinical triggers for arrhythmia events have been described by genotype including exercise or swimming (LQT l ) , auditory stimuli, or during the postpartum period (LQT2) and during sleep (LQT3). Features associated with higher risk for SCD include the jervel and Lange-Nielsen syndrome (congenital deafness) , syncope or ventricular arrhythmias while on beta-blocker therapy, QTc > 500 mil­ liseconds with an LQT l or LQT2 genotype, female gender, and family history of SCD. Standard treatment includes beta-blocker therapy, but this remains somewhat controver­ sial in the case of LQT3 , where the clinical response rate is lowest. According to 2008 ACC/AHNHRS device therapy guidelines, patients with syncope or ventricular arrhythmias while on beta-blocker therapy can be considered for primary prevention ICD implantation. The 5-year sudden-death risk in patients on beta-blocker therapy (Long QT Registry) is < l % in asymptomatic patients, 3% in the syncope group, and 1 3 % in the SCD group. Mexiletine, a sodium chan­ nel blocker, may be helpful in reducing the burden of ven­ tricular arrhythmias among patients with LQT3 , due to the attributable gain-of-function mutation in SCN5A resulting in voltage-gated sodium channels remaining open longer than normal. A prolonged QT interval can also be acquired and sec­ ondary to other causes: • Electrolyte derangements • Acute hypokalemia • Chronic hypocalcemia • Chronic hypokalemia • Chronic hypomagnesemia

377

• Medical conditions • Bradyarrhythmias (complete heart block, sick sinus syn­ drome, bradycardia) • Cardiac (myocarditis, tumors) • Endocrine: hyperparathyroidism, hypothyroidism, pheo­ chromocytoma • Neurologic (cerebrovascular accident, encephalitis, head trauma, subarachnoid hemorrhage) • Nutritional (alcoholism, anorexia nervosa, liquid-protein diet, starvation) •

Drugs Antiarrhythmics: class IA (disopyramide, procainamide), class III (sotalol, dofetilide) • Tricyclic antidepressants (amitriptyline, desipramine) • Antifungals (itraconazole, ketoconazole) • Antihistamines (astemizole, terfenadine) • Antimicrobials (Bactrim, E-mycin, pentamidine) • Neuroleptics (phenothiazines, thioridazine) • Organophosphate insecticides • Promotility agents (cisapride) • Oral hypoglycemics (Glibenclamide) •

Brugada Synd rome The hallmark of this condition is ST-segment elevation in the right precordial leads (Fig. 27.2) attributable to sodium channel defects, including the SCN5A mutation. However, specific genetic mutations are identified in less than half of tested patients. Brugada syndrome is most common in Southeast Asia, affecting mostly men (4: 1). The usual mode of inheritance is autosomal dominant. The ST segment may normalize and may be unmasked by drugs (procainamide, flecainide, ajmaline) , which may uncover the characteristic ST elevation. Patients with Brugada syndrome are at increased

��

�� � � V3



�� FIGURE 27.2

ECG reading i n Brugada synd rome.

378

SECTION V • CARD IAC ELECTROPHYSIOLOGY

risk of SCD, particularly those with prior torsades/PMVT and unexplained syncope. The role of EP testing in risk stratifica­ tion remains controversial, although there are some data to support its use in identifying patients more likely to benefit from an ICD . Quinidine, due to blockade of I,0, may have a role in decreasing the burden of ventricular arrhythmias.

Short-QT Syndrome The short-QT syndrome is characterized by QTc intervals 0. 1 mV above the baseline in at least two leads, or notch­ ing of the terminal QRS in the inferior limb leads, lateral limb leads, or lateral precordial leads. Subsequent experimental data have linked defective modulation of cardiac repolari­ zation to increased risk for ventricular arrhythmias in such patients. These data challenge the long-held notion that early repolarization, commonly present among African American males and athletes, is always a benign finding. OT H E R C A R D I AC CO N D I T I O N S A S S O C I AT E D W I T H S U D D E N D E AT H

Wolff-Parkinson-White Syndrome WPW syndrome is caused by accessory atrioventricular con­ nections. There is a 0 . 1 % incidence of SCD per year, with risk related to the conduction properties of the bypass tract. SCD is related to AF conducting rapidly antegrade over the bypass tract into the ventricles, which can then degenerate into VF. The risk is elevated when the shortest R-R interval in AF is 95% cases. Asymptomatic patients with WPW in low-risk occupations or with loss of pre-excitation during exercise testing do not require ablation. Valvu lar Heart Disease Any primary valvular pathology associated with depressed LV systolic function confers increased risk for ventricular arrhythmias. Specific valvulopathies such as aortic stenosis,

when severe or critical, confer increased risk even when LV systolic function is preserved. Mitral valve prolapse, in rare cases, has been associated with ventricular arrhythmias and SCD also in the setting of preserved LV systolic function. However, literature in this field is limited to case reports and small series data. It remains unclear to what extent val­ vular correction modifies this risk, and the 2006 ACC/AHN HRS sudden death guidelines defer treatment recommenda­ tions according to the established criteria for mitral valve correction. Other SCD causes without primary arrhythmia etiology: •





• •

• • • •

Acute aortic dissection, particularly with retrograde extension causing hemopericardium Mechanical complications following MI such as rupture, tam­ ponade Congenital heart disease, including coronary anomalies (between aorta and pulmonary artery) Commotio cordis (VF associated with chest trauma)

Cyanotic heart disease, right-to-left intracardiac shunts Acute myocarditis

Infiltrative cardiomyopathies such as cardiac amyloid or sarcoid Chagas disease: multifocal myocarditis, CHF Muscular dystrophies: myocardial scarring, conduction system disease

EVA LU AT I O N A N D M A N AG E M E N T

Sudden Card iac Death Survivors A complete history and physical examination focusing on risk factors, medications, illicit substances, and family his­ tory should be obtained. Laboratory evaluation should iden­ tify any related electrolyte abnormalities, particularly among patients with renal dysfunction. A complete cardiac evalua­ tion includes a 12-lead ECG, ambulatory Holter or inpatient telemetry monitoring, a surface echocardiogram, an ischemia workup (stress testing or coronary angiography) , and possi­ bly MRI in selected scenarios such as to evaluate the possi­ bility of arrhythmogenic right ventricular dysplasia (ARVD) or infiltrative cardiomyopathies. Other imaging modalities, such as cardiac positron emission tomography (PET) scans, are selectively utilized to identify proinflammatory condi­ tions such as cardiac sarcoid. In general, ACC/AHA/HRS practice guidelines recommend identification and treatment of reversible causes among sudden death survivors. Among SCD survivors, diagnostic EP studies have limited prognos­ tic value and are not routinely performed except in selected cases such as investigating the etiology of an unknown wide­ complex tachycardia. In the maj ority of SCD survivors, an ICD is indicated in the absence of a transient arrhythmia due to identifiable, reversible causes (i. e . , VF within 24 to 48 hours of acute ST-segment elevation MI) . In the AVID trial, SCD survivors who received an ICD demonstrated an improved 3-year survival rate of 75.4% when compared to 64. 1 % with antiarrhythmic drug therapy

CHAPTER 2 7 • SUDDEN CARDIAC DEATH AND VENTRI CULAR TACHYCARD IA

patients with VT that is refractory to medications and requir­ ing multiple ICD shocks.

V E N T R I C U LA R TAC H YC A R D I A

Coronary Artery Disease Sustained monomorphic VT is most commonly related to scar created by prior MI that can be initiated by spontane­ ous ventricular ectopy. An acute ischemic event, in contrast, is more commonly associated with PMVT or VF such as in acute ST-segment elevation MI. This distinction becomes blurred when transient ischemia causes an increase in spon­ taneous ventricular ectopy capable of initiating monomor­ phic VT in a patient with underlying scar, or PMVTNF when critically timed VPDs occur during cardiac repolarization. For secondary prevention, ICD therapy is recommended for hemodynamically intolerant sustained ventricular tach­ yarrhythmias >30 seconds in duration or requiring abor­ tive therapy (i.e . , shocks) . This includes patients with acute MI with events occurring beyond 48 hours and not related to immediately reversible causes (i.e. , overinjection of con­ trast dye during angiography of the right coronary artery) . For primary prevention, ICD therapy is recommended in patients beyond 40 days post-MI with LVEF :S: 35% on opti­ mal medical therapy and with life expectancy > 1 year. Key trials in formulating these primary prevention indications include the MADIT-2 trial (ICD benefit for patients post­ MI with LVEF < 30% and NYHA class I symptoms) and SCD-HeFT (ICD benefit for LVEF :S: 35% and NYHA class II symptoms) . In addition, patients may be considered for a primary prevention ICD with prior MI, LVEF 3 cm b. Syncope or hypotension associated with exercise c. Dynamic left ventricular outflow-tract (LVOT) g ra­ d ient > 1 00 mm Hg by Doppler echoca rd iogra phy d. Nonsustai ned ventricu lar tachyca rdia (NSVT) 2. A pri m a ry prevention implantable ca rdioverter­ defibrillator (ICD) is most strongly indicated i n which of fol lowi ng patients? a. Male patient with syncope, QTc 440 m i l liseconds, and LQTl genotype not previously treated with beta-blockers

b. Young patient with syncope, NSVT, and diagnostic criteria for a rrhythmogenic right ventricular card iomyopathy (ARVC), including ca rdiac mag netic resonance imaging (MRI) c. Asymptomatic patient with newly diagnosed non ischemic dilated cardiomyopathy (DCM), left ventricular ejection fraction (LVEF) 35% d. Young patient without structural heart disease and monomorphic ventricular tachycardia (VT) (left bundle branch bundle [LBBB] morphology, right inferior axis, precordial R-wave transition in V3 ) 3. A d iag nostic EP study is least usefu l i n which of the fol lowing cli nical scena rios? a. Risk stratification in a patient with coronary artery disease, LVEF 38% with NSVT b. Risk stratification i n a patient with DCM, LVEF 20%, and heart fai l u re sym ptoms

CHAPTER 2 7 • SUDDEN CARDIAC DEATH AND VENTRI CULAR TACHYCARD IA

c . Risk stratification a n d eva l uation for a rrhythmia mechanism i n a patient with a Brugada pattern electroca rd iogram (ECG) and unexplained syncope d. Eva luation of arrhythmia mechanism and possible ablative therapy for a patient with symptomatic wide-com plex tachycardia of u nknown etiology 4. Bundle branch reentry VT is most com m o n ly associated with: a. Enhanced automaticity i n the right bundle b. Enhanced a utomaticity i n the left bundle c. Su pra normal cond uction i n the His bundle d. Abnormally slow conduction i n the His-Pu rkinje system

��

5 . The ECG shown is consistent with:

�� � � a. b. c. d.

V3

Acute a nterosepta l M l Abnormal SCN5A channel Abnormal KCQN 1 channel Old a nterosepta l M l with a n aneurysm

Answers 1 . Answer C: Clin ical featu res associated with SCD among patients with HCM include NSVT, hypotension associated with exercise, unexplained syncope, septa I th ickness >3 cm, and a fam i ly history of sudden death i n a fi rst-deg ree relative you nger than 50 yea rs old. Dynamic LVOT g radients by Doppler echocard iography, even when elevated, a re not a com m o n ly applied as a guidelines-based risk stratification tool for SCD.

381

2 . Answer B: ARVC is a prog ressive disease i nvolving fi brofatty infi ltration, and patients with VT a re h i g h ly l i kely to have recu rrences despite medical thera py. Among patients with sym ptomatic long QT synd rome, response to beta-blocker thera py is h i g h est among patients with the LQT1 genotype. The SCD risk is very low among male patients treated with beta-blockers, LQT1 genotype, and a QT interva l in this ra nge. Among patients with DCM, the 2008 g u ideli nes do not specify a treatment d u ration req u i rement, u n l i ke CMS re­ i m b u rsement criteria. However, ICD i m p l a ntation is reco m m ended for patients with LVEF ::; 35% a n d sym p­ tomatic heart fai l u re (class I recom mendation for NYHA fu nctional class I I or g reater, and class I I reco m m enda­ tion for NYHA fu nctional class I). I n choice d, all of these featu res a re associated with VT originating from the right ventricular outfl ow tract (RVOT), which is c u rable by catheter a blation a n d not associated with SCD. ICD i m plantation is a class Ill recommendation among such patients.

3. Answer B: A diag nostic EP study is least pred ictive for futu re ventricular tachyarrhythmia events among patients with nonischemic cardiomyopathy, and wou l d n o t a lter ma nagement i n the patient descri bed i n choice b, w h o is recom mended for a primary prevention ICD. The patient i n choice a, however, meets g uideli nes­ based criteria for fu rther risk stratification using EP study to evaluate cand idacy for a pri m a ry prevention ICD. EP testing for risk stratification among patients with the Brugada syndrome remains controversial. However, EP testing is not com pletely u n reasonable to eva l u ate a possible arrhyth mia mecha nism for a ny patient with u nexplai ned syncope. It is certai n ly not the wea kest ind ication among the choices listed. In keeping with this concept, choice d actu a l ly h i g h l i g hts the strength of a n EP study, which is to define the precise a rrhythmia mecha nism in a patient with a poorly tolerated, wide complex tachycardia that can n ot be defi n itively diag­ nosed by other c l i n ica l criteria (i.e., distinguishing VT from supraventricular tachycard ia (SVT) with a berra nt conduction or pre-excited tachyca rd ia). This is particu­ larly true where catheter a blation can be cu rative. 4. Answer D: Abnorm a l ly slow cond uction in the His-Pu rkinje system sets u p the conditions of reentry req u i red to susta i n this kind of tachyca rd ia. 5 . Answer B: The ECG is consistent with

abnormal SCN5A chan nel, which causes the Brugada synd rome.

Atrial Fibrillation and Flutter john Rickard and Mohamed Kanj

A

trial fibrillation (AF) is the most common sustained arrhythmia seen in clinical practice. There are esti­ mated to be more than 2 million patients with AF in the United States. The prevalence and incidence of AF in­ crease with advancing age. The mainstay of therapy includes pharmacologic rate control and antiarrhythmic therapy, cardioversion, and antithromboembolic management. Non­ pharmacologic therapies, including ablation, device, and surgical approaches, are also becoming increasingly utilized.

• • • • • • • • • •

E P I D E M I O LO G Y

Preva lence • • • • •

0.4% general population 0.2% in population 25 to 34 years old 2% to 5% in population >60 years old 1 8 % in population >85 years old 8% to 14% in hospitalized patients

Incidence • The incidence of AF increases from 1 60,000 new US cases year) in those under 40 years of age to 1 . 5% per year in females and 2% per year in males over the age of 80 (Kannel et al. 1 983). • 20% to 40% after cardiac surgery

FACTO R S P R E D I S P O S I N G TO AT R I A L F I B R I L L AT I O N The most common cardiovascular (CV) diseases associated with AF are hypertension and ischemic heart disease. Other predisposing conditions include: • • • • • •

Advancing age Rheumatic heart disease (especially mitral valve disease) Nonrheumatic valvular disease Cardiomyopathies Congestive heart failure (CHF) Congenital heart disease

382

Sick sinus syndrome/degenerative conduction system disease Wolff-Parkinson-White syndrome Pericarditis Pulmonary embolism Thyrotoxicosis Chronic lung disease Neoplastic disease Postoperative states Diabetes Normal hearts affected by high adrenergic states, alcohol, stress, drugs (especially sympathomimetics) , excessive caffeine, hypoxia, hypokalemia, hypoglycemia, or systemic infection

M O R B I D I T Y A N D M O RTA L I T Y

Survival The presence of AF leads to a 1 . 5- to 2-fold increase in total and CV mortality (Emelia et al. , 1 998) . Factors that may increase mortality include: • • • • • •

Age Mitral stenosis Aortic valve disease Coronary artery disease (CAD) Hypertension CHF

Patients with myocardial infarction (MI) or CHF have higher mortality if AF is present.

Stroke/Thromboem bolism AF predisposes to stroke and thromboembolism. • Five- to sixfold increased risk of stroke ( 1 7-fold with rheumatic heart disease [RHD] ) • 3 % t o 5 % per year rate o f stroke in nonvalvular AF • Single major cause (50%) of cardiogenic stroke • 75 ,000 strokes per year • Silent cerebral infarction risk • Risk increases with age, concomitant CV disease, and stroke risk factors

CHAPTER 2 8 • ATRIAL FIBRILLATION AND FLUTTER 3 8 3

Tachycardia-Ind uced Cardiomyopathy Persistent rapid ventricular rates can lead to tachycardia­ mediated cardiomyopathy and left ventricular (LV) systolic dysfunction. These are, however, reversible with ventricular rate control and regularization. Control can be achieved with medical rate control, atrioventricular (AV) node ablation, or achievement of sinus rhythm (SR) . An atrial cardiomyopa­ thy may develop leading to structural remodeling with an increase in atrial size.

• The circuit can propagate around the isthmus i n a clockwise or counterclockwise direction. • Counterclockwise atrial flutter is characterized by dominant negative flutter waves in the inferior leads and positive flutter deflection in lead V1 . • Clockwise atrial flutter is characterized by positive flutter waves in inferior leads and negative flutter waves in lead V 1 . • In contrast to coarse AF, the flutter waves on an ECG will usu­ ally have the same morphology, amplitude, and cycle length. • Ablation of the CTI is curative.

Sym ptoms and Hemodynamics

Non cavotricuspid I sth m u s-Dependent Atria l Fl utter

• • • •

Rapid ventricular rates Irregularity of ventricular rhythm Loss of AV synchrony Symptoms: limitation in functional capacity, palpitations, fatigue, dyspnea, angina, CHF

PAT H O G E N E S I S While the pathophysiology of AF remains incompletely understood, it has been shown that AF requires a trigger and a substrate to sustain reentry The triggering mechanism in most patients comes from ectopic firing within the pulmonary veins into which sleeves of atrial myocardium extend. Once AF has been sustained for a period of time, electrical and structural changes take place within the atria that can convert transient AF to persistent AF Electrical changes, such as shortening of the atrial refractory period, occur shortly after AF onset and are reversible with conversion back to SR. Structural changes may take longer to develop, however, and are less amenable to reversal. In patients with CHF, the pathophysiology of AF is somewhat different. In this patient population, areas of inter­ stitial fibrosis are found within the atria that lead to heteroge­ neous electrical conduction. These areas of slowed electrical conduction predispose to the development of AF • Electrical activation: rapid, multiple waves of depolarization with continuously changing, wandering pathways • Intracardiac electrograms: irregular, rapid depolarizations, often >300 to 400 beats/min (bpm) • Mechanical effects: • Loss of coordinated atrial contraction • Irregular electrical inputs to the AV node and His-Purkinj e system leading t o irregular ventricular contraction • Surface electrocardiogram: • No discrete P waves • Irregular fibrillatory waves • Irregularly, irregular ventricular response

Atrial Fl utter Reentrant Mechanism Cavotricuspid Isth m u s-Dependent Atria l Fl utter • Cavotricuspid isthmus (CTI)-dependent flutters refers to cir­ cuits, which involve the isthmus of tissue in the right atrium between the tricuspid annulus and inferior vena cava (IVC) (Fig. 28 1 ) .

• Noncavotricuspid isthmus (NCTI)-dependent flutters do not use the CTI . NCTI flutters are often related to atrial scar which creates a conduction block and a central obstacle that allows for reentry • NCTI can be found in patients with prior cardiac surgery involving the atrium, such as repair of congenital heart disease, mitral valve surgery, or maze procedure as well as in patients post pulmonary vein isolation procedures. • NCTI-dependent flutters are less common than CTI flutters.

Treatment • Atrial flutter may be difficult to treat medically (it is notoriously difficult to rate control) and may develop with organization of AF reentrant flutter circuits during treatment with antiarrhyth­ mic therapy. • Successful ablation is dependent on identifying a critical portion of the reentry circuit where it can be interrupted with catheter ablation.

AT R I A L F I B R I L LAT I O N D E F I N I T I O N S • Lone: Patients under the age of 60 years with absence o f cardiopulmonary or other conditions predisposing to AF • New Onset: First episode of AF • Recurrent: Has two or more paroxysmal or persistent episodes • Paroxysmal: Self-terminating within 7 days, generally lasting 24 hours • Persistent: Is not self-terminating within 7 days or is terminated with treatment • Permanent: Persistent despite cardioversion

EVA L UAT I O N

History • Precipitating factors and conditions • Alcohol, caffeine, sympathomimetics, herbal supplements, or other drug use • Duration and frequency of episodes • Degree of associated symptoms • Manner of AF initiation • Prior therapies for AF (past antiarrhythmic drugs that may have failed or past ablation attempts)

384

SECTION V • CARD IAC ELECTROPHYSIOLOGY

;����, �i�� � j�� ..� vr-.,-..,-. \f 1

aVR

l

I

I V.l

_

...

.

I

6 : 00

7 : 00 8:00 9 : 00

®:®®

1 2 : 00

1 : 00

®:®

@:®®

RF L i n e

FIGURE 28.1

cs

\....

l1 't

Type I cou nterclockwise r i g h t atrial fl utter.

Docu mentation of Atrial Fibril lation and I nitiation • ECGs, rhythm strips • Transtelephonic (remote) event monitoring • Evaluation for precipitating bradycardia, paroxysmal supraventricu­ lar tachycardia (PSVT), atrial flutter, atrial ectopy, atrial tachycardia

Diagnostic Testing • Lab studies-thyroid function, renal, and hepatic tests • Echocardiogram-evaluate LV function, valves, atrial size • Functional stress testing or cardiac catheterization-evaluate for CAD in patients with risk factors and evaluate candidacy for l C agents

M A N AG E M E N T O F AT R I A L F I B R I L L AT I O N

Treatment Strategies • Ventricular rate control • AV nodal-blocking drugs • Atrioventricular node (AVN) modification/ablation and pacing

• Achievement and maintenance of SR • Antiarrhythmic drugs • Cardioversions • Nonpharmacologic therapies - Ablation - Surgery-Maze procedure • Anticoagulation

Atria l Fibril lation Follow-U p I nvestigation of Rhythm Management The Atrial Fibrillation Follow-Up Investigation of Rhythm Management (AFFIRM) study (Wyse et al. , 2002) was a multicenter trial of rate versus rhythm control strategies (Table 28. 1). It tested the hypothesis that in patients with AF, total mortality with primary therapy intended to main­ tain SR is equal to that with primary therapy intended to control heart rate. The study randomized 4,060 patients (>65 years old or with risk factors for stroke) , with a pri­ mary endpoint of total mortality No significant difference in total mortality was found among strategies, although there was a strong trend toward better survival in the rate-control arm. The study also showed that continued anticoagulation is important even in the rhythm-control arm, so this may be

CHAPTER 2 8 • ATRIAL FIBRILLATION AND FLUTTER 3 85 TA B L E Rate Control versus Rhythm Control

Maintenance of SR

Control of heart rate alone

Potential Benefits

Potentia l Risks

Better control of symptoms Reduced risk from anticoagulation Avoidance of electrical and structural remodeling Lower risk of adverse effects (including death) Possibly lower cost

Increased risk of adverse effects (including death) Higher cost

the best strategy in relatively asymptomatic older patients with good rate control.

Control of Ventricular Rate Rapid ventricular rates can cause symptoms and/or ven­ tricular dysfunction. The goal of treatment, a heart rate of 70 to 1 00 bpm at rest, can be achieved pharmacologi­ cally with agents that slow AV nodal conduction, such as digoxin, beta-adrenergic blockers, and calcium channel blockers (Table 2 8 . 2) . These agents , however, should not be used in patients with ventricular preexcitiation due to the risk of very rapid antidromic conduction during AF over the pathway. In patients who are hemodynami­ cally stable with evidence of pre-excited AF, amiodarone , ibutilide , procainamide , or disopyramide are acceptable choices. The RACE II trial compared strict rate control (resting heart rate 60: 500 µg b.i.d. 40-60: 250 µg b.i.d. 20-40: 125 µg b.i.d.

36% 58%-7 1 %

Procainamide

Side Effects/Com ments

i QT, proarrhythmia/TdP, potential i AV node conduction, diarrhea, nausea, i digoxin levels, thrombocytopenia -!- BP, CHF, drug-induced lupus, agranulocytosis; active metabolite NAPA with class III activity accumulates in renal failure Anticholinergic effects (e.g. , urinary retention, dry eyes/mouth) , CHF Proarrhythmia visual disturbance, dizziness, CHF, avoid in CAD/LY dysfunction CHF, avoid in CAD/LY dysfunction

CHF, bronchospasm, bradycardia, i QT proarrhythmia/T dP Pulmonary toxicity; bradycardia hyper- or hypothyroidism, hepatic toxicity; GI (nausea, constipation) , neurologic, dermatologic, and ophthalmologic side effects, drug interactions Diarrhea, nausea, and vomiting Exclude CrCl 75 years old) with risk factors for stroke are at higher risk (8% per year).

j TA B L E •�:•"'1 Stroke/Th romboembolism Reduction in AF: Aspirin versus Control

AFASAKI SPAFI EAFT AFI ESPS2

Aspirin

Control

5.2 3.6 15.5 6.3 13.8

6.2 6. 3 19.0 8.1 20.7

RRR%

16 42 17 21 33

p Value

NS 0.02 0.12 0.05 0.16

RRR% , relative risk ratio (percent). AFASAKI, atrial fibrillation, aspirin, anticoagulation study; SPAFI , stroke prevention in atrial fibrillation; EAFT, European atrial fibrillation trial; AF! , atrial fibrillation investigators; ESPS2, European stroke prevention study 2 .

Most strokes associated with AF appear to result from cardiac emboli, presumably from thrombi formed most commonly in the left atrial appendage, a small, finger-like outpouching of the left atrium adjacent to the mitral valve. Patients with paroxysmal AF have a stroke rate of 3.7% per year with events clustered at the onset of the arrhyth­ mia. The incidence of embolism is 6.8% in the first month and decreases to 2% per year over the subsequent 5 years. Patients with paroxysmal AF appear to be at similar risk as patients with chronic, persistent AF and generally are treated similarly with regard to anticoagulation. Patients with a single AF event, with no other risk factors or structural heart disease and 75 years LV dysfunction Increased left atrial size Rheumatic mitral valve disease Prosthetic valves Mitra! annular calcification Increased wall thickness Thyrotoxicosis Peripheral vascular disease

C H A O S I I S CO R E The CHADS II score is a well-known, commonly used index used to gauge stroke risk in patients with nonrheumatic AF CHADS stands for: Congestive heart failure (active within the last 1 00 days or evidence of LV dysfunction) , Hypertension (blood pressure consistently above 140/90 mm Hg or treated

392

SECTION V • CARD IAC ELECTROPHYSIOLOGY

·�:Wi1 TA B L E

Stroke/Th rom boembolism Red uction in AF Wa rfarin versus Aspirin Warfarin

AFASAKI SPAF2 'S,. 75 >75 EAFT AFASAK2

Aspirin

RRR%

p Value

2.7

6.2

48

300 beats/

min [bpm] ) may be conducted to the ventricle in a 1 : 1 fashion (Fig. 29.9). AV conduction at that rate may degenerate rap­ idly into ventricular fibrillation and subsequent cardiac arrest. In these patients, procainamide, flecainide, propafenone, or amiodarone should be considered. In case of atrial fibrillation/ flutter with rapid response and hemodynamic instability, elec­ trical direct-current cardioversion is the treatment of choice. As shown in Table 29.3, atrial fibrillation may be a coexisting or presenting arrhythmia in about 20% to 40% of patients with WPW syndrome. Atrial fibrillation in the pres­ ence of an AP with rapid antegrade conduction can result in degeneration to ventricular fibrillation and subsequently result in sudden cardiac death. The risk of sudden death for patients with WPW syndrome is not clear but is definitely not very high. Population-based studies suggest an inci­ dence of 0 . 1 5 % per year, and sudden death occurs almost exclusively in previously symptomatic patients.7 This occasional occurrence of ventricular fibrillation as the initial manifestation of WPW syndrome has stimu­ lated interest in the possibility of identifying asymptomatic patients who may be at risk for this complication. These patients possess antegrade conducting APs (pre-excitation during normal sinus rhythm) and no symptoms of SVT Screening these patients involves evaluating the conduc­ tion properties of the AP An easy and noninvasive method is to observe for intermittent ventricular pre-excitation by electrocardiogram, which may involve supervised tread­ mill stress testing or simply ambulatory Holter monitor­ ing during the patient's daily activities. This intermittent

CHAPTER 2 9 • SUPRAVENTRICULAR TACHYCARDIAS TA B L E Tachya rrhyt h m ias i n 16 1 WPW Patients

RT l RT + AF l AF RT + AF + VF AF + VF VF RT + VF Total WPWs

89 32 15 13 5 4 3 161

RT, reciprocating tachycardia; AF, atrial fibrillation; VF, ventricular fibrillation.

pre-excitation refers to the abrupt loss of pre-excitation or the delta wave from one beat to the next. This phenomenon suggests an AP that is incapable of extremely rapid ante­ grade conduction and therefore carries a low risk for sudden cardiac death. 1 7 If normalization of the QRS is not observed during Holter monitoring or during exercise, electrophysiologic evaluation must be considered to assess the conduction property of the AP. The most direct method for such risk stratification is the induction of atrial fibrillation in an elec­ trophysiologic laboratory setting and the determination of the shortest R-R interval. This provides information as to the antegrade conduction capabilities of the AP. Studies have shown that patients with WPW syndrome who experienced and survived an episode of sudden cardiac death had the shortest R-R intervals. 1 8 A shortest R-R interval of >250 milliseconds indicates an AP that is incapable of danger­ ously rapid antegrade conduction. However, caution should be exercised about drawing conclusions if the shortest R-R interval is 1 00 beats/min (bpm) and a QRS duration > 120 milliseconds on a 12-lead electrocardiogram (ECG) . Utilizing the ECG, the correct mechanistic diagnosis of a WCI rhythm is often difficult. Besides being an intellectual exercise, it is very im­ portant to establish the correct diagnosis in order to deliver appropriate acute therapy and to plan subsequent long-term patient management. Several criteria and algorithms have been developed to help distinguish among different causes of WCI. When used individually, none of these criteria reaches 1 00% specificity; however, when properly applied together and in conjunction with the clinical history and presenta­ tion, the algorithms serve as a guide to the correct diagnosis in the maj ority of the cases. WCI can result from either a ventricular or a supraven­ tricular mechanism. Ventricular tachycardia (VT) originates below the level of the His bundle. Supraventricular tach­ ycardia (SVT) originates in or involves structures above the His bundle. SVT may involve atrial tachycardia, atrial fibrillation, atrial flutter, atrioventricular (AV) node reen­ trant tachycardia (Fig. 30. 1 ) , or AV reentrant tachycardia. AV reentrant tachycardia may be either orthodromic reen­ trant tachycardia or antidromic reentrant tachycardia (Fig. 30.2). Orthodromic reentrant tachycardia occurs when antegrade ventricular conduction occurs via the AV node and retrograde conduction to the atrium is via the accessory pathway. Antidromic reentrant tachycardia

occurs when ventricular antegrade conduction occurs over the accessory pathway and retrograde conduction occurs via the AV node. D I F F E R E N T I A L D I AG N O S I S WCI can occur by three different mechanisms: 1.

VT is the most common cause of WCT in the general popula­ tion, accounting for >80% of all cases. It is even more common in patients with structural heart disease, and it may occur in 98% of patients with a prior history of a myocardial infarction. VT may be either monomorphic or polymorphic. Monomorphic VT occurs when the QRS morphology is stable and uniform, whereas polymorphic VT occurs when the QRS complexes vary in morphology

2. The second mechanism of WCT occurs when the tachycar­

dia originates above the ventricle and has abnormal ventricu­ lar activation, also known as SVT with aberrancy It accounts for 1 5 % to 20% of all cases of WCT and includes a variety of disorders. a. The first example is SVT with bundle branch block aber­ ration, which may be either a right bundle branch block (RBBB) or a left bundle branch block (LBBB) morphology (Fig. 30.3). Activation of the ventricle through the His­ Purkinje system (His bundle and both bundle branches) results in a narrow QRS complex. Activation of the ven­ tricle unilaterally via one bundle branch results in a wide QRS complex, because activation of the remainder of the ventricular myocardium is dependent on slow myocardial 41 3

41 4

SECTION V • CARD IAC ELECTROPHYSIOLOGY Atrial Tachycardia •

RA

LA

RV

LV

RA

Fl utter c i rcuit--�

LA

Atrial Fibrillation



RA

LA

RV

LV

LA

RA

AVNRT Circuit--+-----'+ �� · � '� /



·

I· " ·,- . -. '. - · -+--!-!I -t--+--t - �-+-- !--+-.+---1--1-+-+-+---1-+-+- � -·-i--

-·- -�-+--r--+-+-i- o

·-

. --r--+-+-+-1-+--1·-+--t--t-+-i--11-+-+-+--,.--+-+-i--l-+-+-1+1 J� . 1 1-riJ ·�� J t-r �M;d- V Pace

••

· '\r·�·.

Clin ical Presentation In order to diagnose the etiology of the WCT, it is important to evaluate the clinical presentation. As mentioned before, obtaining an accurate patient history is crucial in formulat­ ing an accurate rhythm diagnosis. A prior history of heart disease, myocardial infarction, or congestive heart failure makes the diagnosis of VT highly suggestive as the cause of the WCT. Akhtar et al. have reported that the positive pre­ dictive value of a WCT representing VT in a patient with a prior history of myocardial infarction is 98% . Tchou reported that of patients who had a prior myocardial infarction and a first episode of tachycardia occurring after the infarction, 28 of 29 patients presented with VT and were diagnosed cor­ rectly. The older the patient is, the more likely that the tachy­ cardia is ventricular; however there is a significant overlap

Atrial Tracki n g RA



_

LA



SVT

· · · · · ·

Pace make r - M e d i ated Tachycard i a

... . '.. �L .. ..

A Sense => V Pace : .

... .

. . . . . .

Pacemaker lead

RV

FIGURE 30.7

D I AG N O S I S

LV

...

.

.

RA •

· · · ·

LA



·

. J� RV

LV

Ventricular paced tachycardia. Left: Atria l tracking. Right: Pacemaker-med iated tachyca rdia.

41 7

CHAPTER 30 • WIDE-COMPLEX TACHYCARD IA

t

t I

i

I -� - -t FIGURE 30.8

t

t

Artifact m i m icking WCT.

with SVT patients. It is also helpful to know if there is any presence of congenital heart disease, or if the patient has a pacemaker or defibrillator. Knowing that the patient has an implantable cardioverter-defibrillator (ICD) raises a concern for pacemaker-associated tachycardia, but more important, the presence of the device suggests that the patient has risk factors for VT. A history of a prior similar episode may also be useful. The first occurrence of the arrhythmia after a myo­ cardial infarction is highly suggestive of VT, whereas SVT may be more likely if there is recurrence of the arrhythmia over several years. The presence of other medical conditions can point to a diagnosis of WCT. For example, in a patient with renal failure, the WCT may be attributable to hyper­ kalemia. In a patient with known peripheral vascular disease, the WCT may be indicative of VT, because such patients are likely to have underlying coronary artery disease. Knowing what medications the patient is taking, espe­ cially cardiac medications, is vital when evaluating WCT. It is important to identify medications that prolong the QT inter­ val, such as dofetilide, sotalol, quinidine, and erythromycin, which can all cause torsade de pointes, a form of polymorphic VT. Electrolyte abnormalities caused by certain medications such as diuretics (hypokalemia and hypomagnesemia) or angiotensin-converting enzyme (ACE) inhibitors (hyper­ kalemia) may predispose to VT. Patients who are on digoxin are more susceptible to an arrhythmia when hypokalemia is present. The most common arrhythmias are monomorphic VT, bidirectional tachycardia, and junctional tachycardia, and typically occur when the plasma digoxin concentration is >2 .0 nglmL. As stated earlier, Class IC agents can cause rate-related aberrant conduction during SVT. Symptoms such as palpitations, lightheadedness, or chest pain are gen­ erally not useful in evaluating the etiology of the WCT. One of the priorities in evaluating a patient with WCT is determining if the patient is hemodynamically stable or unstable. This requires knowing the patient's blood pres­ sure and heart rate. In a patient who is unstable, emergency cardioversion is required and the mechanism of the arrhythmia may not necessarily be known. VT can be present when the patient is hemodynamically stable and should not

t

t

t

t

t

I

'

t

·t

t

I

t

t

���1 �{V\,_,·�� � : I

t VJY , I

, I

be mistaken for SVT, lest the patient be given inappropriate medical therapy (such as adenosine or verapamil) that can lead to hemodynamic compromise with VT. When the patient is hemodynamically stable, a more detailed physical exam can be performed. Inspection of the chest can point to underlying cardiovascular disease when there is a sternal incision, a pacemaker, or defibrillator. AV dissociation occurs in 60% to 75% of patients with VT and is a result of the atria and ventricles depolarizing independently: It almost never occurs in SVT. This finding is usually identifiable on the surface ECG. However, it is also possible to make this diagnosis on physical exam by assess­ ing the jugular venous pulsation. Cannon A waves are irreg­ ular pulsations that are of greater amplitude than the normal jugular venous waves, and occur intermittently when the atrium and ventricle contract simultaneously: When the tachycardia rate is slower, there can be variable intensity of the first heart sound. However, evaluating this may not be practical in an acute situation. Laboratory tests should be performed for patients with WCT to determine potassium and magnesium levels. If the patient is on digoxin, it is also important to obtain the serum digoxin level. If a chest x-ray is available, one can readily identify the presence of a pacemaker, defibrillator, or sternal wires that might point to underlying structural heart disease.

Provocative Maneuvers Certain bedside maneuvers can be performed to distinguish VT from SVT. The Valsalva maneuver or carotid sinus mas­ sage enhances vagal tone, which depresses sinus nodal and AV nodal activity: These maneuvers will slow the heart rate during sinus tachycardia, but once they are completed, the heart rate will increase again. If the patient is in SVT, these maneuvers may terminate the rhythm. If the patient is in an atrial tachycardia or flutter, the rhythm will persist though the ventricular rate may be slower, thus uncovering the back­ ground atrial activity: These maneuvers can also elicit VA con­ duction block, which can induce AV dissociation during VT. Certain medications can be used to diagnose the tach­ yarrhythmias. For example, adenosine, given in 6- to 12-mg

41 8

SECTION V • CARD IAC ELECTROPHYSIOLOGY

boluses intravenously during WCT, can result in one of the following scenarios: 1 . The tachycardia terminates, making it more likely to be

supraventricular in etiology, invoking AV node participation. Some atrial tachycardias may also terminate with adenosine. 2. AV block occurs, uncovering the background atrial activity such

as atrial tachycardia, flutter, or fibrillation, thus allowing the diagnosis of an atrial tachyarrhythmia . 3 . If 1 : 1 AV association i s present and evident during WCT, aden­

osine-induced AV block results in AV dissociation, thus making the diagnosis VT

Adenosine has a short half-life of about 1 0 seconds. However, it has to be used with caution, because it may cause hemody­ namic compromise in a patient with VT. Some paroxysmal VT in structurally normal hearts may terminate with adenosine. Termination of the rhythm with lidocaine suggests VT as the mechanism. Amiodarone and procainamide, however, will not diagnose the rhythm if the WCT is terminated. Beta­ blockers may be given as well. They can terminate SVT or uncover AV dissociation during VT in a manner similar to adenosine. It is important that verapamil not be given if the diagnosis is in question, because it can lead to significant hemodynamic compromise in VT and induce ventricular fibrillation and cardiac arrest.

ECG Criteria The most reliable way to differentiate VT from SVT is by eval­ uating the ECG. A 1 2-lead ECG is more helpful than a rhythm strip. A rhythm strip may be additive as a result of analyzing the beginning and termination of the tachycardia. A previous ECG during a normal rhythm will help to identify the base­ line QRS morphology and the presence of Q waves that might

FIGURE 30.9

suggest a prior myocardial infarction. Ventricular pre-excita­ tion may be suggested if there is the presence of delta waves. There are several ECG criteria and different algorithms that may be used to differentiate VT from SVT in WCT: 1.

The tachycardia rate has no diagnostic value in determining the mechanism of the WCT.

2. Regularity of the RR intervals is also not a useful criterion, because

VT can be irregular in patients on antiarrhythmic medications. 3. QRS-complex duration can be useful in differentiating VT from SVT.

4.

5.

The WCT is more suggestive of VT when the QRS duration is > 140 milliseconds with an RBBB morphology and > 1 60 milliseconds with an LBBB morphology A study by Wellens showed that all of 70 patients with WCT due to SVT had QRS-complex durations 140 milliseconds. Another study, by Akhtar, showed that 1 5 % of patients with VT had QRS-complex duration 140 milli­ seconds with RBBB pattern or > 1 60 milliseconds with LBBB pat­ tern correlates with VT. Wide QRS-complex duration can still be seen with pre-excitation, ventricular pacing, use of antiarrhythmic drugs, and marked baseline intraventricular conduction delays. VT in structurally normal hearts may have a relatively narrow QRS complex in a case with idiopathic left ventricular VT. The QRS-complex axis may also be helpful in diagnosing WCT. A right superior QRS-complex axis in the frontal plane is more suggestive of VT. Presence of LBBB and right-axis deviation is also almost always due to VT. Presence of Q waves that are also pre­ sent in normal sinus rhythm suggests prior myocardial infarction, which makes the diagnosis of VT more likely Pseudo-Q waves can be seen in SVT, which represents retrograde atrial activation. QRS-complex concordance in the precordial leads is highly pre­ dictive of VT, with a specificity as high as 90% or greater. The sensitivity is low because it is only present in S) qRs

rS (Broad r >30 ms) Notching/delay in S QS � 70 ms RT taller than RS QR, QS QrS, qR Rr'

rS, QS (rapid downstroke)

RS (R < S) QS, Qrs QR Monophasic R

rR' Monophasic R No Q waves

CHAPTER 3 0 • WIDE-COMPLEX TACHYCARD IA

Normal Cond uction

T

-f�'-f

l

42 1

41 JoJ ! Jl�_J_ -!:_jL-v i "!m+ Yif"1LBBB

Rapid Downstroke

SVT with Abe rration

RBBB

R

Broad

r

; rs r;

_A

LBBB

Mono R

No Q wave

RBBB

Triphasic: qRs RS>1

rS (R0.3 m/s/y increase in velocity)8 or a peak jet velocity >4 mis suggest that patients have 10 y or at least until age 40, whichever is longer. Consider life-long prophylaxis if high risk (e.g. , health care worker, teacher) 1 0 y or "well into 2 . Rheumatic fever with adulthood," whichever carditis but without valve is longer. disease 3 . Rheumatic fever without 5 y or until age 2 1 , whichever is longer. carditis or valve disease From Bonow RO , Carabello BA, Chatterjee K, et al. 2008 Focused Update Incorporated into the ACC/AHA 2006 Guidelines for the Management of Patients with Valvular Heart Disease: A Report of the American College of Cardiology/American Heart Association Task Force on Practice Guidelines (Writing Committee to Revise the I 998 Guidelines for the Management of Patients with Valvular Heart Disease) Endorsed by the Society of Cardiovascular Anesthesiologists, Society for Cardiovascular Angiography and Interventions, and Society of Thoracic Surgeons. ] Am Coll Cardiol. 2008 ; 5 2 : e l-e l 4 2 , with permission from Elsevier.

Primary prevention of rheumatic fever includes prompt diagnosis of group A streptococcal infections and treatment with appropriate antibiotics, usually a penicillin derivative or macrolide. Patients who develop acute rheumatic fever require long-term secondary prophylaxis, usually with monthly intra­ muscular injections of benzathine penicillin (Table 33. 1). Sym ptomatic Patients

For patients with severe AS who develop symptoms, experi­ ence a decline in LV systolic function 5 mis if operative mortality < l % 6. Patients with mild AS undergoing coronary artery bypass graft (CABG) with risk of rapid progression (i.e. moderate to severe valve calcification) 7. Asymptomatic patients with severe AS and none of the above

AHA/ACC Class

Ila Ilb Ilb Ilb

Ilb

III

From Bonow RO , Carabello BA, Chatterjee K, et al. 2008 Focused Update Incorporated Into the ACC/AHA 2006 Guidelines for the Management of Patients With Valvular Heart Disease : A Report of the American College of Cardiology/American Heart Association Task Force on Practice Guidelines (Writing Committee to Revise the 1 998 Guidelines for the Management of Patients With Valvular Heart Disease) Endorsed by the Society of Cardiovascular Anesthesiologists, Society for Cardiovascular Angiography and Interventions, and Society of Thoracic Surgeons. ] Am Coll Cardiol. 2008 ; 5 2 : e l-e l 42, with permission from Elsevier.

and "extremely severe" AS (AVA 60 mm Hg, or jet velocity >5 mis) if the operative mortality is < l % (Table 33.2).5 Va lve Replacement O ptions Mechanical Va lves Mechanical prostheses have excellent longevity but require systemic anticoagulation. For patients with normal ejection fractions, contemporary bileaflet or single-leaflet valves require an INR of 2 to 3. Older­ generation valves should be anticoagulated to an INR of 2 . 5 to 3 . 5 . Additionally, any patient with atrial fibrilla­ tion, depressed LV function, prior thromboembolism, or a hypercoagulable state should have a target INR of 2 . 5 to 3 . 5 . Mechanical valves should especially be considered in younger patients ( age 65 years, and in patients for whom anticoagulation is problematic. All patients with tissue valves should be treated with aspirin (8 1 mg daily) . Homografts Cadaveric homografts are the valve of choice for patients with active infective endocarditis because they may resist infection more than tissue or mechanical valves. Additionally, they do not require long-term anticoagulation. However, the durability of homografts is no better than that of tissue valves and reoperation is much more challenging with homografts than with mechanical or other tissue valves.

Recommendations for Ba l l oon Valvuloplasty i n Young Patients with Noncalcific AS Ind ication

1 . Symptomatic patients with peak gradient >50 mm Hg by catheter­ ization 2. Peak gradient >60 mm Hg by catheterization, regardless of symptoms 3 . Asymptomatic patients with a peak gradient >50 mm Hg by catheterization and ECG abnor­ malities suggestive of ischemia 4. Asymptomatic patients with a peak catheterization gradient >50 mm Hg who wish to play competitive sports or desire pregnancy 5 . BAY is recommended over valve surgery when possible 6. Asymptomatic peak gradient 400 milliseconds) suggests milder disease. PHT is dependent on multiple variables, including systemic vascular resistance and LV and aortic compliance, and thus changes in these variables reduce the utility of PHT. d. Diastolic flow reversal in the descending aorta: If the reversed flow is pan-diastolic and exceeds 25 cm/s, severe AI is likely (Fig. 33 .4) .

FIGURE 33.4 Conti nuous-wave Doppler flow profi le in the descending aorta, showi ng flow reversa l at approximately 30 emfs. Th is profi le suggests severe Al.

470

SECTION VI • VALVULAR HEART DISEASE likely follow within 3 years. Once symptoms develop , the rate of mortality increases to 10% per year. Medica l Treatment

Medical therapy for patients with severe AI includes afterload reduction with vasodilators in certain situations. Vasodilators carry an ACC/AHA Class I recommendation for patients with severe AI and symptoms of LV dysfunc­ tion who are unable to undergo surgery, and a Class Ila recommendation for short-term therapy prior to AYR.5 They also should be used in patients with AI who have hypertension. Vasodilators may be considered (Class Ilb) in asymptomatic patients with severe AI and normal LV function but LV cavity dilation. They are not indicated (Class III) for patients with mild or moderate AI and normal LV function (Table 33.5). Dihydropyridine calcium channel blockers are first-line agents, although angiotensin receptor inhibitors (ACE) inhibitors are frequently used as well. Endocarditis prophylaxis is no longer indicated for patients with AI, though is reasonable (Class Ila) for patients with previous endocardi­ tis or prosthetic material.7 For patients with mild-moderate TA B L E I n dications for Vasodilator Thera py i n Patients with Severe Al

FIGURE 33.5

M-mode image through the mitral va lve in a patient with severe acute Al. Classic fi ndings shown include fluttering of the anterior leaflet (fl) and early closure of the mitra l va lve (c'). e.

M-mode echocardiography: On classic M-mode imaging, fluttering of the mitral valve is seen with moderate to severe AI. Fluttering may be seen in both acute and chronic AI. In severe acute AI, premature closure of the mitral valve is also seen. Diastolic MR may be noted on color M-mode or color Doppler (Fig. 33.5)

Treatment of Aortic Insufficiency Acute Aortic I n s ufficiency

Because acute severe AI is poorly tolerated, emergency or urgent surgery is advised. If a delay is necessary before surgery, IV vasodilators become the treatment of choice. Increasing the heart rate will decrease the diastolic period, and may temporize the hemodynamic effects of acute severe AI . Intra-aortic balloon pumps are absolutely contraindi­ cated in severe AI . C h ronic Aortic I n s ufficiency Asymptomatic Patients

Chronic AI is usually well toler­ ated for years before symptoms develop . In asymptomatic patients with normal LV function and severe compensated AI, the progression rate to symptoms is 4% per year, and the progression to LV dysfunction is 1 . 3% per year. The risk of sudden death is very low in asymptomatic patients (4. 5 cm) , beta-blockers can be used carefully to decrease aortic wall stress. Relative bradycardia, however, may worsen the AI. Indications for Surgery

Patients with severe AI who have symptoms, LV dilatation or dysfunction, or (in the case of secondary AI) who have enlarging aortas should undergo valve surgery. ACC/AHA indications for aortic valve surgery for AI are listed in Table 3 3 . 6 .5 Class I indications for AYR include symptomatic patients with severe AI (irrespective of LV

I n d ications for Aortic Va lve S u rgery i n Patients with Al Ind ication

1 . Patients with severe symptom­ atic AI 2 . Patients with severe asymptom­ atic AI and LVEF 7.5 cm or LVIDs >5 . 5 cm) 5 . Patients with moderate AI under­ going open heart or aortic surgery 6. Asymptomatic patients with normal EF and moderate LV cavity dilatation (LVEDd 7075 mm, ESD 50-55 mm) 7. Patients with asymptomatic severe AI and normal LV function without LV cavity dilation (LVIDd 7. 5 cm or left ventricular inter­ nal dimension in systole [LVIDs] >5 . 5 cm) . Patients with moderate AI undergoing CABG or aortic surgery may be considered for AYR (Class Ilb) , as well as asymptomatic patients with severe AR and LV dilation (LVIDd >7.0 cm or LVIDs >5.0 cm) if there is evidence of progressive dilation, decreasing exercise tolerance , or abnormal hemodynamic response to exercise . For patients with aortic root dilatation and significant AI, progression of the aortic diameter >45 mm is gener­ ally accepted as an indication (Class Ila) for aortic root and aortic valve surgery.5 Lower thresholds (>40 mm) may be considered (Class Ilb) for patients with Marfan syndrome or bicuspid aortic valves, particularly if the rate of aortic dilata­ tion is accelerating (>0 . 5 cm/year) n Surgical options for AI include valve repair or replace­ ment. Valve repair may be considered for noncalcified bicuspid valves with substantial AI. Repair results for regur­ gitant trileaflet valves have been disappointing. For valve replacement, the decision to use mechanical versus biopros­ thetic valves is based on a number of considerations (see discussion above, for AS) . By guidelines, patients younger than age 65 years, and patients with end-stage renal disease or other disorders that affect calcium metabolism, should receive mechanical valves. P U L M O N I C VA LV E D I S E A S E

Pulmonic Stenosis Pulmonic stenosis (PS) is nearly always a congenital defect, although very rare cases of acquired disease have been reported with rheumatic heart disease, carcinoid heart dis­ ease, and rubella. PS may be a component of more com­ plex congenital diseases, where it is often associated with a VSD ; most frequently, however, PS is an isolated congeni­ tal defect. 12 Noonan syndrome is classically associated with isolated PS. PS may be due to valve doming in the setting of com­ missural thickening, valve dysplasia in the setting of valve thickening and annular hypoplasia, or unicuspid/bicuspid valve pathology (often seen with tetralogy of Fallot) . H i sto ry a n d P hysica l Exa m

Symptoms with PS are rare unless the transvalvular gra­ dient exceeds 50 mm Hg, so mild-moderate stenosis is often subclinical. When symptoms are present, they relate to decreased cardiac output and usually include fatigue, dyspnea on exertion, and decreased functional capacity. With more severe disease, presyncope and syncope may develop .

472

SECTION VI • VALVULAR HEART DISEASE

The hallmark of PS on jugular venous examination is a prominent A wave, which reflects increased right ventricular (RV) end-diastolic pressure. The classic auscultatory findings include a widely split S2 , and a crescendo-decrescendo systolic murmur at the pulmonic position. When murmurs are associated with peripheral pulmonary stenoses, they may be heard over the lateral chest wall, the axillae, or in the back. An ej ection click may also be appreciated, which moves earlier in systole as the severity of stenosis increases. Signs of severe stenosis include a late-peaking systolic murmur, decreasing intensity of P2 , and the complete disappearance of the ejection click. Unlike other right-sided valvular lesions, respiration tends to decrease the intensity of the murmur. Clinical signs of right ventricular hypertrophy (RVH) or RV failure do not present until late in the disease. Diag n ostic Stu d ies

The CXR classically shows asymmetric PA enlargement, with a prominent left PA. The heart size is usually normal. The key finding on echocardiography is the transpulmonic gradient, which is calculated from the peak jet velocity across the pulmonic valve. PS is likely to be severe when the pulmonic jet velocity exceeds 3 mis (estimated peak gradient 36 mm Hg) , at which point cardiac catheteriza­ tion is recommended (Class I) for further evaluation and consideration of balloon valvuloplasty Follow-up echo­ cardiography is recommended for surveillance every 5 to 10 years. Cardiac catheterization is recommended for patients whose echocardiogram is suggestive of severe PS (Class I) . Guidelines for treatment (i.e. , balloon valvuloplasty) are based upon peak-to-peak gradient across the PV obtained at catheterization. Treatment of P u l mo ni c Stenosis

Endocarditis prophylaxis is no longer indicated for patients with PS, but is reasonable (Class Ila) for patients with cyanotic congenital heart disease or prosthetic material.7 For patients with elevated gradients or with symptoms, balloon valvotomy is the treatment of choice. Indications for balloon valvotomy are listed in Table 3 3 . 7 . Class I indications for intervention include symptoms with a peak-to-peak gra­ dient at catheterization of >30 mm Hg, or a peak-to-peak gradient >40 mm Hg even without symptoms. Balloon val­ votomy for PS with a peak-to-peak gradient 30 to 39 mm Hg is reasonable (Class Ilb). 5

Pulmonary Insufficiency The main etiologies of significant pulmonary insufficiency (PI) are annular dilation due to pulmonary hypertension, dilation of the PA (which may be idiopathic or secondary to Marfan syndrome) , a late complication of tetralogy of Fal­ lot repair, or a primary valve disorder, caused by carcinoid, rheumatic disease, or endocarditis.

Recom mendations for Va lve I ntervention in Patients with PS Ind ication

AHA/ACC Class

1 . Symptomatic PS with peak-to­ peak gradient >30 mm Hg 2. Asymptomatic PS with peak-to­ peak gradient: >40 mm Hg 30-39 mm Hg 7.5 cm, LVI Ds >5.5 cm) is a reasonable indication (Class I la) for AVR even in the presence of normal LV function. Patients with asymptomatic severe AS may be consid­ ered (Class l l b) for isolated AVR if the AS is thought to be "extremely severe" on the basis of AVA < 0.6 cm 2, mean gradient >60 m m Hg, or peak AV jet velocity >5 m/s; the patient in (c) does not meet these criteria and is therefore a Class I l l (not recommended) indication for AVR. 8. An swer E: Bal loon aortic vavu loplasty may be

considered (Class l l b) as a pa l l iative treatment for sym ptomatic patients with severe AS u n a b l e to

u ndergo trad itio n a l s u rgical AVR, or as a brid g i n g strat­ egy for patients who a re too h i g h risk for s u rgical AVR at a g iven point i n time. It is not i n d i cated (Class I l l), however, for patients as a n a lternative to s u rgical AVR (a) or for patients req uiring u rgent nonca rdiac s u rgery (c). Utmost caution is req u i red on the part of the a n esthesia tea m to avoid hypotension i n patients with severe AS u n d ergoing nonca rdiac s u rgery, as preload dependence is i m pe rative in mainta i n i n g hemodyna mic sta b i l ity. 9. Answer A: The ACC guidelines provide a Class I indica­ tion for serial echocardiog raphy every 3 to 5 years for pa­ tients with mild AS and every 1 to 2 years for patients with moderate AS. For patients with severe AS, surveillance echocardiography is recommended on a n annual basis, or even more frequently if clinically indicated. Patients with symptomatic AS should not have stress testing (Class I l l). 1 0. Answer B: A common misinterpretation d u ring echoca rdiogra phy is the mistaken sampling by CW Doppler of the MR envelope as the aortic outflow. MR velocity is usua l ly in the ra nge of 4 to 5 mis and characteristica l ly begins at the time of ventricu lar systole. I n contrast, aortic outflow beg ins after the period of isovu l metric contraction, which on average is approxi mately 80 m i l l iseconds. Gallava rdi n phenom­ enon is present when the musical component of the AS m u r m u r is heard at the a pex; it does not typical ly rad iate to the axil la, and this patient did not have evidence of AS. The Austin-Fli nt murmur is a diastolic m u r m u r of MS heard i n patients with severe Al and is thought to be due to prematu re closure of the MV d u ring d iastole due to the eccentric jet of Al h itti ng the a nterior MV leaflet.

Mitral and Tricuspid Valve Disease William ] . Stewart

M

itral valve disease is a common valvular abnormal­ ity, resulting from various etiologies and having well-understood, varied, and interesting clinical manifestations. Tricuspid valve disease is less common, oc­ curring most often as a functional result of left-sided heart disease and/or pulmonary hypertension. M I T R A L VA LV E A N ATO M Y The mitral valve apparatus consists o f anterior and posterior leaflets, chordae tendineae, anterolateral and posteromedial papillary muscles, and mitral annulus. To be inclusive, it also includes the atrial and ventricular myocardium. Mitral valve dysfunction may result from aberrations of any portion of the mitral valve apparatus, as a result of mechanical, traumatic, infectious, degenerative, congenital, or metabolic causes.

M I T R A L VA LV E P R O LA P S E Mitral valve prolapse (MVP) is found in approximately 2 % of the population and is equally common in men and women. It is the most common cause of mitral regurgitation (MR) in the United States. Most such patients have a minor amount of MR and therefore a benign prognosis, with no significant cardiovascular symptoms or manifestations such as conges­ tive heart failure. The diagnosis of MVP is made usually by bedside physical examination, finding a mid-to-late systolic click or multiple clicks, sometimes associated with a late sys­ tolic or pansystolic murmur. The murmur becomes earlier and louder with standing and the Valsalva maneuver, result­ ing from reduction in preload, which brings the mitral leaf­ lets closer together before left ventricular (LV) contraction. The murmur of mitral prolapse becomes softer and later with squatting due to an increase in preload. The diagnosis of MVP is best confirmed echocardio­ graphically The best two-dimensional (2-D) echocardio­ gram criterion is leaflet displacement beyond the line of the mitral annulus in the long-axis view. Because of the saddle­ shaped configuration of the mitral valve, caution must be

taken when MVP is diagnosed only from parasternal long­ axis, apical four-chamber, and apical two-chamber views. M-mode criteria require 2 or 3 mm of displacement, either as late systolic or holosystolic hammocking (Fig. 34. 1 ) . The presence of an eccentric jet direction of MR makes the diag­ nosis of MVP more likely In general, prolapse with leaflet thickness >5 mm is considered "classic" MVP, whereas pro­ lapse with thinner valve leaflets is considered "nonclassic prolapse. " Accepted indications for performing echocardiographic study in mitral prolapse include establishing the diagnosis, determining the severity of MR, evaluating leaflet morphol­ ogy, and defining LV size and function. 1 The list implies that echocardiography should be used when it can add infor­ mation to findings available from history and the physical examination. Indications for echocardiography may also include exclusion of MVP in patients diagnosed with MVP when there is no clinical evidence to support the diagnosis (Table 34. 1). Subsequent or serial echocardiograms are not usually necessary if the patient is asymptomatic, unless there are clinical indications of severe or worsening MR. Most patients with mitral prolapse, even if there is a murmur, or if the echocardiogram shows significant MR, do not need antibiotic prophylaxis for endocarditis. Although that has been common practice prior to new guidelines that were published several years ago, the newest guidelines do not advise prophylaxis for native valve disease. Prophylaxis during procedures likely to cause bacteremia (including teeth cleaning) is recommended if the patient has had previ­ ous endocarditis or if a prosthetic valve has been implanted (Table 34.2). The natural history of MVP is frequently benign. Follow-up studies in large population samples show that most patients with MVP do quite well, and most do not develop any significant congestive heart failure, atrial fibrillation (AF) , stroke, or syncope. After a prolonged asymptomatic interval, a small percentage of patients with MVP develop more severe MR, ruptured mitral valve chordae (flail) , left atrial and ventricular enlargement, or AP In addition, with gradual 477

478

SECTION VI • VALVULAR HEART DISEASE

FIGURE 34.1

M-mode echoca rd iog ra phy showi ng late-systolic prola pse (arrow) of the mitral va lve.

progression of MR, LV dilatation and dysfunction may occur, leading to congestive heart failure. A substantial negative effect on survival has been seen in patients who develop LV dysfunction, AF, left atrial enlargement, age >50 years, and flail mitral leaflet. Recently, quantitatively severe regurgita­ tion has also been associated with adverse prognosis. 3•4

The development of infective endocarditis in patients with MVP is another mechanism of increase in MR. Predictors of infective endocarditis in patients with MVP include male gender, age >45 years, the presence of MR, and leaflet thicken­ ing and redundancy Patients with MVP with significant MR also have a small but significantly increased risk of sudden death, most likely secondary to ventricular tachyarrhythmias, which may be predicted by concomitant systolic dysfunction. MVP has been associated with a pattern of multiple nonspecific symptoms such as palpitations, atypical chest pain, syncope, and anxiety, and this constellation has been frequently termed the "mitral valve prolapse syndrome. " No such associations have been found in multiple studies, but a small group of patients may have a complex set of symptoms associated with MVP. For example, a few studies have shown a pattern of autonomic dysfunction, with increased catecho­ lamines and decreased vagal tone, in patients with MVP. The mainstay of medical management of patients with MVP is reassurance. Beta-blockers are the treatment of choice for patients with increased adrenergic symptoms such as pal­ pitations, chest pain, or anxiety, though some of these effects may be the placebo effect. In patients with MVP and transient ischemic attacks (TIAs) or stroke, the treatment is usually just aspirin (8 1 to 325 mgld) . Warfarin may be indicated

TA B L E Eva l uation and Management of the Asym ptomatic Patient in MVP Class I

1 . Echocardiography is indicated for the diagnosis of MVP and assessment of MR, leaflet morphology, and ventricular compensation in asymptomatic patients with physical signs of MVP. (Level of Evidence: B) Class I l a

1. Echocardiography can effectively exclude MVP in asymptomatic patients who have been diagnosed without clinical evidence to support the diagnosis. (Level of Evidence: C) 2 . Echocardiography can be effective for risk stratification in asymptomatic patients with physical signs of MVP or known MVP. (Level of Evidence: C) Class I l l

1. Echocardiography is not indicated to exclude MVP in asymptomatic patients with ill-defined symptoms in the absence of a constellation of clinical symptoms or physical findings suggestive of MVP or a positive family history (Level of Evidence: B) 2 . Routine repetition of echocardiography is not indicated for the asymptomatic patient who has MVP and no MR or MVP and mild MR with no changes in clinical signs or symptoms. (Level of Evidence: C) From Bonow RO , Carabello BA, Chatterjee K, et al. ACC/AHA 2006 guidelines for the management of patients with valvular heart disease: a report of the American College of Cardiology/American Heart Association Task Force on Prac­ tice Guidelines (Writing Committee to Revise the 1998 guidelines for the management of patients with valvular heart disease) developed in collaboration with the Society of Cardiovascular Anesthesiologists endorsed by the Society for Cardiovascular Angiography and Interventions and the Society of Thoracic Surgeons . ] Am Coll Cardiol. 2006;48 : e l-e l 48, with permission from Elsevier.

CHAPTER 3 4 • MITRAL AND TRI CUSPID VALVE DISEASE

479

TA B L E Recom mendations for I nfective Endoca rditis Prophylaxis i n Va lvular Heart Disease

Prophylaxis against infective endocarditis is reasonable for the following patients at highest risk for adverse outcomes from infective endocarditis who undergo dental procedures that involve manipulation of either gingival tissue or the periapical region of teeth or perforation of the oral mucosa: •

Patients with prosthetic cardiac valve or prosthetic material used for cardiac valve repair.



Patients with previous infective endocarditis. (Level of Evidence: B) Patients with CHD . (Level of Evidence: B) Unrepaired cyanotic CHD, including palliative shunts and conduits. (Level of Evidence: B) Completely repaired congenital heart defect repaired with prosthetic material or device, whether placed by surgery or by catheter intervention, during the first 6 months after the procedure. (Level of Evidence: B) Repaired CHD with residual defects at the site or adjacent to the site of a prosthetic patch or prosthetic device (both of which inhibit endothelialization) . (Level of Evidence: B) Cardiac transplant recipients with valve regurgitation due to a structurally abnormal valve. (Level of Evidence: C







• •

(Level of Evidence: B)

Adapted from Bonow RO , Carabello BA, Chatterjee K, et al. 2008 focused update incorporated into the ACC/AHA 2006 guidelines for the management of patients with valvular heart disease : a report of the American College of Cardiology/American Heart Association Task Force on Practice Guidelines (Writing Committee to Develop Guide­ lines for the Management of Patients With Valvular Heart Disease) . ] Am Coll Cardiol. 2008 ; 5 2 : e l-e l 42; Wilson W, Taubert KA, Gewitz M, et al. Prevention of infective endocarditis: guidelines from the American Heart Association: a guideline from the American Heart Association Rheumatic Fever, Endocarditis, and Kawasaki Disease Committee, Council on Cardiovascular Disease in the Young, and the Council on Clinical Cardiology, Council on Cardiovascular Surgery and Anesthesia, and the Quality of Care and Outcomes Research Interdisciplinary Working Group. Circulation . 2007; 1 1 6 : 1 736- 1 754. Ref. (5,6).

in some patients with MVP and recurrent TIA or stroke. In addition, in patients with AF, there should be a low thresh­ old for instituting anticoagulation, individualized for the patients risks of stroke versus bleeding. Surgery for MVP is only a consideration in patients with more severe MR, similar to other forms of nonischemic MR, which is discussed later. AC U T E M I T R A L R E G U R G I TAT I O N Acute M R is an uncommon medical condition o f grave importance, requiring urgent medical and often surgical intervention. Acute MR also occurs due to disruption of mitral valve leaflets, chordae tendineae, or papillary muscles that may result from infective endocarditis, acute myocardial infarction, trauma, or rheumatic fever. The most common cause is probably acute myocardial ischemia leading to acute LV enlargement, severe MR, and acute pulmonary edema. High left atrial pressure and reduced left atrial compli­ ance secondary to severe MR are the mechanisms of pulmo­ nary edema. A less common complication of severe acute MR is reduced forward flow with cardiogenic shock. Acute MR usually presents as sudden and marked increase in congestive heart failure symptoms, with weakness, fatigue, dyspnea, and sometimes respiratory failure and shock. Peripheral

vasoconstriction, pallor, and diaphoresis are usually associ­ ated presenting signs. In some patients, a loud systolic mur­ mur and a diastolic rumble or third heart sound are heard. In others, a very soft murmur or no murmur is heard, because the severity of MR and the lack of atrial compliance lead to midsystolic equalization of pressures between the left atrium and ventricle midway through systole. In addition, the acute nature of the condition obscures the mitral murmur by other aspects of the patients distress, including orthopnea, pre­ cluding a good exam in the left lateral decubitus position. Echocardiography is the diagnostic procedure of choice. In acute coronary syndromes, emergency catheterization and cardiac surgery are life saving. There is little need for contrast LV angiography, except in cases where there is dis­ crepancy in clinical and noninvasive findings. In some cases, hemodynamic measurements and monitoring may also be helpful in management. Acute MR after myocardial infarction is discussed in detail in another chapter of this book. It is the cause of about 7% of cases of cardiogenic shock after myocardial infarction. The onset of the MR is most commonly between days 2 and 7 after myocardial infarction. The MR in most patients, like our example at the beginning of this chapter, involve a functional mechanism, from apical tethering of normal leaflets as a

480

SECTION VI • VALVULAR HEART DISEASE

consequence of acute LV dysfunction and enlargement. Focal infarction most commonly involves the posteromedial papil­ lary muscle, because it derives its blood supply solely from one artery, the right coronary artery. In contrast, the anterolateral papillary muscle has a dual blood supply; often derived partly from the circumflex and partly from left anterior descending artery Despite the devastating effects of acute severe MR, the infarct size is not always large, with some smaller infarctions ( 0.4 cm2 is indicative of severe regurgitation, whereas 4.5 cm systolic diameter), AF, or significant pulmonary hypertension. In addition, many centers recommend mitral valve surgery for selected patients with severe MR when they also have a flail mitral leaflet, 12 or when there is exercise echo evidence of "latent LV dysfunction," defined by a declining EF or increasing end systolic volume (ESV) with exercise. 1 3 . 1 4 Guidelines do not advocate mitral valve surgery for patients with ejection fraction 60 ml >0.4 cm 2 >200 m l/s >400 m m 2 >0.6 cm 2

4. What is the most common etiology of tricuspid reg u rg itation (TR)? a. b. c. d. e.

Rheu matic disease Prolapse or fla i l Tra uma Ca rcinoid Left-sided heart fai l u re

S. A SS-year-o ld man has posterior m itral va lve prola pse (MVP) that was detected on a recent physical exa m. His MR is a nteriorly di rected and his ROA measures 0.S cm 2 on proxi mal conver­ gence method. He has an LVE F of SO%, normal

. . . . . . . . . . . . . . . . . . . . . . .

. . . . . . . . . . . . . . . . . . . . . . . . . . .

CHAPTER 3 4 • MITRAL AND TRI CUSPID VALVE DISEASE

· · · · · · · · · · · · · · · · · · · · · · · · · · · · · · · · · · · · · · · · · · · · · · · · · · · · · · · · · · · · · · · · · · · · · · · · · · · · · · · · · · · · · · · · · · · · · · · · · · · · · · · · · · · · · · · · · · · · · · · · · · · · · · · ·

pul monary artery pressu res, a n d can r u n 3 miles every other day without difficu lty. He has never experienced AF to his knowledge. His left ventricu­ lar (LV) end systolic d imension is 4.2 cm and his LV end d iastolic di mension is 6 cm. His left atria l diameter is 4.5 cm. What do you recommend now? a. Watchfu l wa iting with repeat echocard iog ra m i n a yea r b . M itra l va lve replacement (MVR) with a bioprosthesis g iven his age c. MVR with a mechanical va lve g iven his age d. M itra l va lve repa i r with i n the next few months e. Afterload reduction with a n angiotensin­ converti ng enzyme (ACE) i n h i bitor and ca refu l monitori ng of his LV fu nction 6. A 70-year-old woman with long-standing m itra l stenosis (MS) comes to see you. She claims that she is perfectly fine and can do all her activities of daily l iving. Her fam i ly who acco m pa ny her suggest that she has g rad u a l ly red uced her activi­ ties and is now house bound.The va lve is heavi ly calcified and her mean g radient across it is 6 mm Hg by echoca rdiog raphy. There is 2+ M R. The pul­ monary a rtery pressu re is 50 m m Hg systolic. Her other va lves a re thickened but not stenotic. She has 3+ TR. She has long-standing AF that is rate controlled. She is ta king coumadin, digoxin, and a beta-blocker. The next step i n her eva l uation is: a. Start fu rosemide to red uce her fi l l i ng pressu res. b. Do a stress echoca rd iog ra m to assess her func­ tional capacity and change i n valve g radients, reg u rg itation, and pul monary pressu res with stress. c. Recommend cardiac catheterization to mea­ sure her i ntracard iac pressu res i nvasively. d. Perform tra nsesophageal echoca rdiography (TEE) to better assess her MS. e. Recommend bal loon va lvu loplasty now to i m p rove her va lve g radients. 7. A 45-year-old woman presents with increasing weight gain and abdominal fu llness for a number of months. She sustained a motor vehicle accident about 6 months ago and had a major i njury to her chest. She had made a reasonable recovery but is now l i mited. On examination, she is i n normal sinus rhythm. Her venous pressu re wave is prominent and elevated. She has a Grade 3/6 systolic murmur at the right sternal border. She has abdominal disten­ tion with hepatomegaly. She has 3+ pitting edema bilateral ly. The most likely cause of her problems is: a. Carcinoid synd rome with resu ltant severe TR b. Rig ht heart fai l u re from respi ratory insufficiency fol l owing her trau matic i nj u ry to the chest

503

c. Constrictive pericard itis conseq uent on her chest i nj u ry d. lschemic i nj u ry to the right coronary a rtery and ru ptu red pa pillary m uscle to the tricuspid valve e. Tra umatic chordal rupture of tricuspid va lve leaflet with severe TR 8. The fol lowing is true about tricuspid stenosis: a. It often occu rs as an isolated lesion indepen­ dent of other va lve i nvolvement. b. Bal loon va lvu loplasty is the treatment of choice in most situations. c. If m itra l va lve d isease is present, tricuspid stenosis is usua l ly evident before m itra l va lve disease is manifest. d. Is often accompan ied by severe TR. e. Often an audible open ing snap (OS) is present over the tricuspid va lve. 9. A 55-yea r-old woman has severe symptomatic MS and has a va lve a rea of 0.8 cm 2 · She has 2+ M R, moderate aortic reg u rg itation (AR), and a spl itta­ bility score of 1 1 . She had an open com m issu roto­ my in the past. Which of the fo l l owing statements is most l i kely to be correct? a. She s h o u l d n ot u n dergo m itra l ba l loon va lvu­ loplasty as the deg ree of M R and spl itta b i l ity score a re a bsol ute contra i n d ications to the proced u re. b. You tel l her that ba l l oon m itra l va lvu loplasty is u n l i kely to be successfu l beca use of her prior com m issu rotomy. c. You tel l her that ba l l oon m itra l va lvu loplasty may be attem pted but she is u n l i kely to have an opti mal result based on her spl ittabil ity score and her prior com missu rotomy but may provide sym ptomatic rel ief. d. You recommend watchfu l waiting as she wi l l l i kely i m prove with a n exercise prog ra m . e . You tel l her that her only option is MVR and that this wi l l be a low-risk proced u re with estimated mortal ity of 0.5 cm 2 • He

now has LV dysfu nction as his LVEF is 50%.This should be at least 60% i n someone with severe M R and normal LV fu nction and is a Class I ind ication for surg ical i nter­ vention. Mitra l va lve repa i r is favored over MVR in the treatment of M R when feasible as long-term outcomes are genera l ly better and LV fu nction is more l i kely to be preserved postoperatively with repai r. Waiti ng fu rther i n this man may only cause his contractile fu nction to deteriorate fu rther. There is no evidence that afterload red uction or any other medical therapy will a lter the natura l history of M R. Either a mecha n ica l or biopros­ thetic va lve m i g ht be reasonable choices in this i n d i­ vid ual if repai r were not feasi ble.

7. Answer E: Trau matic i nj u ry to the tricuspid va lve may occur fo l l owing a motor vehicle accident and can present i nsidiously with severe TR and right heart fai l­ u re. Surgical repa i r is usually feasible and cu rative. 8. Answer D: Tricuspid stenosis is relatively uncom­

mon and is al most always seen i n the setting of sig n ifi­ cant rheumatic m itra l va lve disease. It usua l ly man ifests late often with accompa nying sig n ificant TR. Ba l loon va lvu loplasty is feasible in some insta nces but is rarely considered because of concom itant severe TR. 9. Answer C: Ba l l oon va lvu loplasty is not contra i ndi­

cated based on either the spl ittabil ity score or a history of prior com missu rotomy or 2+ MR. The resu lts are less l i kely to be optimal in this situation but may afford symptomatic rel ief. It is u n l i kely that her sym ptoms will i m p rove with exercise as she has critica l MS. MVR wi l l have a > 1 % risk g iven h e r prior surgical proced u re. 1 0. Answer C: MVP is eq ually prevalent in men and women but men a re more l i kely to have com p lications such as endocarditis and significant MR. In most patients, m itral prolapse is a benign condition and does not req u i re su rgery. M itra l va lve thickening of > 5 mm­ classic m itra l prola pse- is more l i kely to have a com­ plicated cou rse. M itra! prola pse is best detected and diagnosed on the long-axis views and not the a pical fou r-chamber echocard iog raphic view beca use the saddle shape of the m itra l a n n u l u s may lead to a pparent but factitious prolapse on the apical four-ch a m ber view.

Infective Endocarditis Peter Zimbwa and Steven M. Gordon

Over the last three decades, the overall incidence of infective endocarditis (IE) and the associated mortality have remained constant, between 1 . 7 and 6.2 per 1 00,000 people per year, and between 10% and 30% , respectively 1-4 The clinical spectrum of IE has, however, undergone dramatic changes. These changes are noted in: • The at-risk population • The underlying susceptible cardiac lesions • The etiologic pathogens • The clinical presentation • The diagnostic evaluation • The antimicrobial agents • The recommendations for prophylaxis

In industrialized countries, the population at risk for IE has become older, parallel to an ageing population. The atten­ dant degenerative valve lesions such as calcific aortic stenosis and myxomatous mitral regurgitation (MR) have superseded rheumatic heart disease as the primary risk factors for IE. In developing countries where antibiotic use is not as wide­ spread, however, rheumatic heart disease remains the key risk factor for IE. Invasive medical interventions, particu­ larly with intravascular catheters, hemodialysis, and cardiac implantable electronic devices, have led to the emergence and prominence of health care-associated (nosocomial) IE. There has also been growth in those at risk in indus­ trialized countries among injection drug users (IDU) , and patients with prosthetic valves. The etiologic pathogens have remained largely unchanged, with viridans streptococci and staphylococci species accounting for the majority of cases. However, their relative contributions and antibiotic sensitiv­ ities have changed, mirroring the risk factors. For instance, the maj ority of cases in the 1960s and 1970s were caused by viridans streptococci, and 1 5 % of cases were due to staphy­ lococci. More recently, staphylococci have surpassed strep­ tococci as the most common cause of IE accounting for 3 1 % of the cases, whereas viridans streptococci were identified in

1 7% . 2 In most urban areas, IDU may account for the maj or­ ity of IE that is most commonly caused by Staphylococcus aureus.3· 1 1 Such patients tend to be younger and present more acutely with fever and sepsis syndromes rather than as the classic Oslerian subacute and chronic presentation of fever of unknown origin, with regurgitant valvulitis, splinter hemorrhages (Fig. 3 5 . 1), Osler nodes (Fig. 3 5. 2) , Janeway lesions (Fig. 35 .3), or Roth spots. The International Collaboration on Endocarditis­ Prospective Cohort Study (ICE-PCS) , Murdoch and coinves­ tigators 2 prospectively collected data on 2 , 78 1 patients with definite IE by the modified Duke criteria at 58 hospitals in 25 countries fromjune 1 , 2000, through September 1 , 2005 . The median age of the cohort was 57.9 years, and 72. 1 % had native valve endocarditis (NYE) . Most patients (77%) presented early (within 30 days) with few of the classic fea­ tures of IE. Recent health care exposure was found in 25% of patients. S. aureus was the most common pathogen (3 1 . 2%) . Left-sided IE was more common (mitral valve 4 1 . 1 % , aor­ tic valve 37.6%). Common complications included stroke ( 1 6 .9%), embolization other than stroke (22 .6%), heart failure (32 .3%), and intracardiac abscess (14.4%). Surgical therapy was common (48.2%), and in-hospital mortality was high ( 1 7 . 7%). Increased risk of in-hospital death was associated with prosthetic valves (PV) , older age, pulmo­ nary edema, S. aureus, coagulase-negative staphylococcal infection, mitral valve vegetation, and paravalvular compli­ cations. Viridans streptococcal infection and surgery were associated with a decreased risk. DEFINITIONS IE is defined as a microbial infection o f the endocardium. Acute and subacute endocarditis are further subdivisions based on the tempo and severity of the infection 1 with sub­ acute presentation usually defined as >2 months of symp­ toms before diagnosis. Prosthetic valve endocarditis within 1 2 months after valve surgery is defined as early and is often 505

506

SECTION VI • VALVULAR HEART DISEASE

FIGURE 35.1 Spli nter hemorrhages.

FIGURE 35.2 Osler nodes.

caused by drug-resistant, surgery-related pathogens, such as methicillin-resistant Staphylococcus aureus (MRSA) . 1 Infec­ tions that are acquired after this period, presumably after endothelialization of the valve prosthesis, are defined as late. Late PV is more likely caused by the same set of pathogens that cause NYE, such as oral streptococci, and the HACEK group (Haemophilus species, Aggregatibacter (formerly Act­

the same period. 1-4 While three decades ago , IE was mainly a complication of rheumatic heart disease in children and young adults, in the industrialized world, IE is now mainly seen in much older adults rendered vulnerable by new risk factors including degenerative heart valve disease, valve prostheses, hemodialysis, intravascular catheters, cardiac implantable electronic devices, and IDU. 1 -4 Left-sided IE is more frequent than right-sided IE, accounting for over two-thirds of NVE. 2 •6 Prosthetic valve endocarditis of the left side has the worst prognosis, asso­ ciated with mortality that can exceed 40% .5-7 The cumula­ tive risk for PV increases with duration of the prosthesis, reported to be approximately 1 % at 1 year and 2% to 3% at 5 years.8•9 Right-sided IE commonly occurs in the setting of IDU, indwelling central venous catheter, cardiac implant­ able electronic devices, congenital heart disease (CHD) , and human immunodeficiency virus (HIV) infection. 1 0· 11 The inci­ dence of IE associated with IDU is 1 50 to 2 ,000 per 1 00,000 person-years with in-hospital mortality

E P I D E M I O LO G Y I E i s uncommon. The incidence o f community-acquired NYE has remained between 1 . 7 and 6.2 per 1 00,000 person­ years over the last three decades. The associated mortality has also remained unchanged (between 10% and 30%) over

FIGURE 35.3 Janeway lesions.

CHAPTER 3 5 • INFECTIVE E N D O CARDITIS patients with acquired immunodeficiency syndrome (AIDS) , mortality of right heart endocarditis can reach 50% . 12 . 1 3 Hospital-acquired IE constitutes 9% to 29% of all cases and has increased in frequency in recent years owing to greater use of invasive procedures. 1 4• 1 5 MRSA, coagulase­ negative staphylococci, and gram-negative bacilli tend to predominate as causative agents, with mortality rates as high as 40% to 60% . Fowler et al. 1 6 recruited 324 patients with S. aureus bacteremia caused by an infected intravascular device to define patient and bacterial characteristics asso­ ciated with the development of hematogenous complica­ tions (including endocarditis) . On multivariable analysis, symptom duration, hemodialysis dependence, presence of a long-term intravascular catheter, or a noncatheter device, and infection with MRSA placed the patients at a higher risk of developing hematogenous complications. R I S K FACTO R S Important risk factors for I E include: • Degenerative valvular heart disease • Prosthetic heart valves • Increased exposure to nosocomial bacteremia • Poor dental hygiene • Long-term hemodialysis • Inj ection drug use • HIV infection

PAT H O G E N E S I S Four main mechanisms are responsible for the initiation and localization of infection of the endocardium: 1. A previously damaged cardiac valve or a situation in which a jet

effect is produced by blood flowing from a region of high pres­ sure to one of low pressure 2. A sterile platelet fibrin thrombus 3. A pathogen in the bloodstream

4.

High titer of agglutinating antibody for the infecting organism

Damaged endocardium results in exposure of the underly­ ing extracellular matrix proteins, which engenders throm­ bosis with production of tissue factor, and the deposition of fibrin and platelets. 1 7 Pathogens then adhere to the damaged endothelium and set up infection via adhesins which include proteins and polysaccharides collectively known as Micro­ bial Surface Component Reacting with Adhesive Matrix Molecules (MSCRAMMs) . 1 8 In experimental IE using animal models, both the magnitude of bacterial inoculum and the adhesive properties of bacteria were important in determin­ ing the likelihood of subsequent infection. 1 9 Inoculating animals with bacterial bolus injections between 1 05 and 1 07 colony-forming units (CFU)/mL induced IE. 2 0 Veloso et al. 21 tested whether IE could also be induced by injecting the same absolute number of bacteria but at a very low level. They found that such low-grade continuous infusion

507

(over >10 hours) was as infective as high-grade bolus infu­ sion, confirming that perhaps the most critical factor for IE induction is total bacterial burden, rather than peak bacte­ rial concentration. Transient, recurrent, low-grade, and short duration ( 1 to 1 00 CFU/mL for < 1 0 minutes) bacteremia occurs during chewing and brushing teeth. 22 Such "normal" cumulative exposure exceeds a single tooth extraction by the order of 1 05 D. 24 This likely explains why most cases of IE occur without antecedent dental procedures and why health care-associated IE arises from recurrent health care-related bacteremia, 2 5. 2 6 a point for consideration in IE prophylaxis. M I C R O B I O LO G Y In recent times, staphylococci, particularly S . aureus, have overtaken viridans streptococci as the most common cause of IE. 1 •2 In the future, we foresee a further shift in IE being caused by S. aureus. Coagulase-negative staphylococci are the most common pathogens in early PV 1 Staphylococcus lugdunensis, a coagulase-negative organism, tends to cause a particularly virulent form of IE with high rates of perival­ vular extension and metastatic seeding. 1 The most common streptococci isolated from patients with IE are Streptococcus sanguis, Streptococcus bovis, Streptococcus mutans, and Strepto­ coccus mitis. 1 Endocarditis by group D streptococci, mainly Streptococcus gallolyticus, previously known as S. bovis, is prevalent among the elderly and is associated with preexist­ ing colonic lesions. 1 Polymicrobial IE is encountered most often in the setting of IDU and is uncommon. 1 The HACEK group are gram-negative aerobic organ­ isms associated with the so-called "culture-negative" IE and are now readily identified by blood culture systems. Cul­ ture-negative IE accounts for around 1 0 % of most reported series of IE. 2 The most common cause of culture-negative endocarditis is prior administration of antibiotics. To over­ come this problem, special blood culture media contain­ ing charcoal have been devised to inhibit the effects of antibiotics. 2 7 When blood cultures from patients with IE remain negative at 48 to 72 hours, the laboratory should be alerted for prolonged incubation or for plating of sub­ cultures on enriched media. A list of organisms that cause culture-negative endocarditis is provided in Table 35 . 1 . A comparison of microorganisms identified in published series of blood culture-negative IE shows that the most common etiologies are Bartonella species, Coxiella burnetti, and Tropheryma whipplei. 2 8 Bartonella species are fastidious gram-negative coccoba­ cilli that are mainly transmitted by arthropod vectors. Bar­ tonella henselae infections occur following exposure to cats or cat fleas and are associated with various diseases including cat-scratch disease, meningoencephalitis, bacillary angioma­ tosis, peliosis hepatitis as well as a subacute IE. Among the 24 known Bartonella species, most cases of IE are caused by B. henselae or Bartonella quintana and account for approximately 1 % to 3% of IE cases and 9% to 10% of culture-negative IE. C. burnetii is the cause of Q fever and is a common cause of IE in parts of the world where sheep , cattle, and

508

SECTION VI • VALVULAR HEART DISEASE

Organisms Causing Cultu re-Negative Endocard itis Organism

Approach

Abiotrophia species (previously classified as nutritionally variant streptococci)

Grow in thioglycolate medium of blood culture and as satellite colonies around S. aureus on blood agar or on medium supplemented with pyri­ doxal hydrochloride or L-cysteine Serologic tests Lysis-centrifugation system for blood cultures PCR of valve or embolized vegetations; special culture techniques avail­ able, but organisms are slow growing and may require a month or more for isolation Serologic tests PCR, Giemsa stain, or immunohistologic techniques on operative specimens Blood cultures positive by day 7; occasionally require prolonged incubation and subculturing Culture from blood has been described Serologic tests Direct staining of tissue with use of fluorescent monoclonal antibody Histologic examination (silver and PAS stains) of excised heart valve; PCR or culture of vegetation Subculture from blood cultures, lysis-centrifugation pellet from blood cultures or operative specimens on BCYE agar; direct detection on heart valves with fluorescent antibody Serologic tests Serologic tests Prolonged incubation of standard or lysis-centrifugation blood cultures Regular blood cultures often positive for Candida species; lysis-centrifugation system with specific fungal medium can increase yield; testing urine for H. capsulatum antigen or serum for Cryptococcus neoformans polysaccharide capsular antigen can be helpful Accessible lesions (such as emboli) should be cultured and examined histologically for fungi

Bartonella species (usually Bartonella hensla or B. quintane)

Coxiella burnetii (Qjever)

HACEK organisms Chlamydia species (usually Chlamydia psittaci) Tropheryma whippleii

Legionella species

Brucella species (usually Brucella melitensis or B. abortus)

Fungi

PCR, polymerase chain reaction; HACEK organisms Haemophilus species (H. parainfluenzae, H. aphrophilus , and H. paraphrophilus), Actinobacillus actinomycetemcomitans, Cardiobacterium hominis, Eikenella corrodens, and Kingella kingae; PAS , periodic acid-Schiff; BCYE , buffered charcoal yeast extract. Reproduced from Mylonakis E, Calderwood SB. Infective endocarditis in adults. N Engl ] Med. 2 0 0 1 ;345 ( 1 8) : 1 3 1 8- 1 3 3 0 , with permission from the Massachusetts Medical Society.

goats are birthed. 2 9 Coxiella tends to infect prosthetic valves or previously damaged aortic and mitral valves and causes small subendothelial vegetations that are often missed by echocardiography The organism resides in the acidic phago­ lysosome, where antibiotic activity may be inhibited. Tropheryma whipplei is a rod-shaped bacterium that is identified as PAS-positive microorganism inside the macrophages.30 It is the causative pathogen for Whipple dis­ ease characterized by chronic enteritis with malabsorption, arthritis, lymphadenopathy, uveitis, encephalitis, and demen­ tia. It can also cause culture-negative endocarditis. Diagnosis is best achieved by polymerase chain reaction (PCR) .

Brucellae are zoonotic infections. 3 1 Humans are infected through the ingestion of contaminated meat or unpasteur­ ized milk, the inhalation of infectious aerosols, or direct con­ tact with infected tissues. This is mainly a disease of farmers, abattoir workers, veterinarians, and shepherds. Because vegetations are large and valve destruction commonly occurs, most patients require a combination of antimicro­ bial therapy and valve replacement. Legionella IE often pre­ sents as a febrile illness present over many months. 32 Most patients have prosthetic valves. Embolic events are unusual with this organism. Pseudomonas aeruginosa is a rare cause of IE and occurs in the setting of IDU. 2

CHAPTER 3 5 • INFECTIVE E N D O CARDITIS Fungal IE is usually caused by Candida species but may also include other species such as Aspergillus and Histoplas­ mosis capsulatum. 33 It usually manifests as bulky vegetations, perivalvular extension of infection, metastatic seeding, and embolization to large blood vessels. Predisposing factors for fungal IE are IDU, prolonged antibiotic therapy, immu­ nosuppression, intravenous catheters, cardiac implantable electronic devices, prior or concomitant bacterial endocar­ ditis, and disseminated fungal infection. The rate of recovery of filamentous fungi such as Aspergillus is 38°C ( 1 00.4°F) Petechiae and splinter hemorrhages are excluded. None of the peripheral lesions are pathognomonic for IE. Presence of rheumatoid factor, glomerulonephritis, Osler nodes, or Roth spots Positive blood cultures that do not meet the maj or criteria Serologic evidence of active infection; single isolates of coagulase-negative staphylococci and organisms that very rarely cause IE are excluded from this category.

Cases are defined clinically as definite if they fulfill two maj or criteria, one maj or criterion plus three minor criteria, or five minor criteria; they are defined as possible if they fulfill one maj or and one minor criterion, or three minor criteria. HACEK, Haemophilus species (H. parainjluenzae, H. aphriphilus , and H. paraphrophilus), A. actinomycetemcomitans, Cardiobacterium hominis, Eikenella corrodens, and Kingella kingae. Reproduced from Mylonakis E, Calderwood SB. Infective endocarditis in adults . N Engl ] Med. 2 0 0 1 ;345 ( 1 8) : 1 3 1 8- 1 3 3 0 , with permission from the Massachusetts Medical Society

512

SECTION VI • VALVULAR HEART DISEASE TA B L E Echocardiogra p h ic Featu res that Suggest Potential Need for S u rgical I ntervention

Vegetation Persistent vegetation after systemic embolization Anterior mitral leaflet vegetation, particularly with size > 1 0 mm" 1 embolic event during first 2 wks of antimicrobial therapy" Increase in vegetation size despite appropriate antimicrobial therapy"·b Valvular dysfunction Acute aortic or mitral insufficiency with signs of ventricular failureb Heart failure unresponsive to medical therapyb Valve perforation or ruptureb Perivalvular extension Valvular dehiscence, rupture, or fistulab New heart blockb,c Large abscess or extension of abscess despite appropriate antimicrobial therapyb See text for more complete discussion of indications for surgery based on vegetation characterizations. 'Surgery may be required because of risk of embolization. bSurgery may be required because of heart failure or failure of medical therapy. 'Echocardiography should not be the primary modality used to detect or monitor heart block. From Baddour LM, Wilson WR, Bayer AS, et al. Infective endocarditis: diagnosis, antimicrobial therapy, and management of complications. Circulation. 2005 ; 1 1 1 :e394--e434, with permission.

to rule out IE.57 For patients with intermediate probability of IE endocarditic, initial use of TEE is more cost-effective and diagnostically efficient than initial use of TTE, which, if negative, is followed by TEE.57 Intermediate probability patients include those with unexplained bacteremia and gram-positive cocci, those with catheter-associated S. aureus bacteremia, and those admitted with fever or bacteremia in the setting of recent IDU. The category of low prior prob­ ability includes patients with gram-negative bacteremia with a clear noncardiac source and patients with a firm alternate diagnosis or those in whom the "endocarditis" syndrome resolved within 4 days. There has been controversy whether vegetation size as measured by echocardiography is a prognostic indicator for embolism or even an indication for surgery, Di Salvo et aL 58 in a study of 1 78 patients with IE reported that those with vege­ tations >l cm or those with "highly mobile" vegetations may need to be considered for early surgery irrespective of their response to antimicrobials, the presence of valve destruction, or heart failure. On the other hand, De Castro et al.59 reported no relationship between vegetation size and the risk of embolization in a study involving 57 patients. Echocar­ diographic features that suggest potential need for surgical intervention are shown in Table 3 5 . 3 . CO M P L I C AT I O N S Certain conditions place patients at increased risk for com­ plications from IE. These are summarized in Table 35.4. Complications of IE may be classified as



• •



Cardiac including CHF, paravalvular extension Neurologic including stroke Systemic emboli including splenic abscess MAs, intracranial and extracranial

CHF may develop acutely as a result of valve perforation, rupture of mitral chordae, mechanical blockage of valve ori­ fice by bulky vegetation, fistulization of cardiac chambers, coronary vegetation embolization, or arrhythmia. 1 It may occur in a more gradual fashion as result of worsening val­ vular insufficiency, Heart failure as a result of IE is associated

Clinical Situations Constituti ng High Risk for Com p l ications for IE

Prosthetic cardiac valves Left-sided IE Staphylococcus aureus IE Fungal IE Previous IE Prolonged clinical symptoms (::'.") Cyanotic CHD Patients with systemic to pulmonary shunts Poor clinical response to antimicrobial therapy Reproduced from Bayer AS, Bolger AF, Taubert KA, et al. Diagnosis and management of infective endocarditis and its complications. Circulation, 1 998;98(25):2936-2948, with permission.

CHAPTER 3 5 • INFECTIVE E N D O CARDITIS with a grave prognosis and delaying surgery to the point of total ventricular decompensation increases operative mor­ tality In addition, poor surgical outcomes are portended by renal insufficiency and advanced age. 59 Aortic valve infection is more commonly associated with CHF than mitral valve infection. The left ventricle alone bears the brunt of the vol­ ume overload in the case of acute aortic insufficiency (AI) as opposed to acute MR where the left atrium and the pul­ monary vascular bed accommodate the regurgitant volume. Hence, new-onset, moderate to severe AI due to IE usually requires surgery The indications for surgery in right-sided IE are less clear-cut. Tricuspid and pulmonary regurgitation are well tolerated as long as there is no preexisting pulmonary vascular resistance. Extension of infection beyond the valve annulus is associated with higher mortality; more frequent CHF, and the need for surgery. This complication occurs in 10% to 40% of all NVE,59 and in 56% to 100% of all PV60 In native aortic valve IE, the extension tends to happen at the weakest portion of the annulus, which resides near the membranous septum and AV node. This is why abscesses and heart block are more frequent in this location.48 Clinical parameters or the size of the vegetation do not predict the possibility of periannular extension. Development of new AV block has a 77% positive predictive value for abscess for­ mation, but the sensitivity is only 42% .60 Urgent surgery is usually indicated for this condition and involves drainage of abscess cavities, excision of necrotic tissue, and closure of fistulous tracts in addition to valve replacement. 6 1 Neurologic complications develop in up to 40% of all patients with IE, 1 and over half of those requiring admis­ sion to intensive care units. Sonneville et al.62 reported that among 198 patients with definite left-sided IE, 1 08 (55%) experienced at least 1 neurologic complication. These may include embolic stroke with or without hemorrhage, rup­ tured intracranial MA, transient ischemic attack, meningitis, and encephalopathy 1 •62 Factors independently associated with neurologic complications were 5. aureus IE, mitral valve IE, and nonneurologic emboli.62 The maj ority of emboli lodge in the middle cerebral artery (MCA) distribution.48 The management of a patient with neurologic complications in the acute phase of IE is controversial. Any patient with IE and neurologic symptoms should have preoperative imag­ ing (CT or MRI of the head) . Four vessel cerebral angio­ grams may also be indicated if MA is suspected (usually after rupture causing CNS hemorrhage) . A ruptured MA should be clipped, resected, or embolized prior to cardiac surgery for IE.63 In patients with a hemorrhagic infarct, the current recommendation is to wait for 2 to 3 weeks between the neurologic event and cardiac surgery because of the risk of intracranial bleeding during anticoagulation with cardiopul­ monary bypass, or post cardiac surgery indications. 63·64 Systemic embolization most commonly involves the spleen, kidney; liver, and the iliac or mesenteric arteries. 1 Splenic abscess is a rare complication of IE and develops as a result of bacteremic seeding of a bland infarct caused by a splenic artery embolus, or by direct seeding of the

51 3

splenic tissue by an infected vegetation.48 It occurs in 5 % of all splenic infarctions, with viridans streptococci and S. aureus being the maj or causes. The diagnosis must be sus­ pected in any patient with IE and flank, back or abdominal tenderness in the left upper quadrant or left shoulder (from diaphragmatic irritation) .48 Abdominal CT and MRI are the most sensitive modalities to diagnose this complication, and definitive treatment is splenectomy with antibiotics before valve surgery; unless valve replacement is more urgent.48 MAs may be intra- or extracranial. They are uncommon complications of IE and result from septic embolization of vegetations first to the vasa vasorum, then into the intima, and finally through the outer layer of the vessel wall. The commonest sites are the branching points of arteries and occur, in decreasing order, in intracranial arteries, visceral arteries, and arteries of the lower and upper extremities. Intracranial MA occurs in 1 .2% to 5% of patients with IE and carry a high mortality rate of 60% . The bifurcations of the distal MCA are the most commonly involved arteries. Symptoms may include severe headache, altered mental status, hemianopia, or cranial neuropathies. Sudden hem­ orrhage may occur in the absence of other premonitory symptoms. 1 •48 Routine screening for intracranial MA is not recommended in the absence of neurologic symptoms or signs. Contrast-enhanced CT, MRI, and MRA are all useful techniques to diagnose intracranial MA, but the current gold standard is 4-vessel cerebral angiography Decisions con­ cerning the medical versus surgical treatment of intracranial MA need to be tailored according to the individual patient. A single intracranial MA distal to the first bifurcation of a maj or intracranial artery should be monitored closely with serial angiograms and must be excised if it enlarges or bleeds. In the case of multiple aneurysms, close monitoring is required with angiograms or CTs and if more than one aneurysm enlarges, prompt surgical excision is required.48 A less invasive alternative to surgery, especially in distally or peripherally located aneurysms is coil embolization.65 Extrac­ ranial MA are often asymptomatic. However, the appearance of a new, painful, pulsatile mass with IE should prompt the diagnosis of extracranial MAs. Hematuria and hypertension should suggest the rupture of a renal artery MA Massive bloody diarrhea should suggest the rupture of an intrab­ dominal MA into the bowel. Hematemesis, hematobilia, and jaundice should suggest rupture of a hepatic artery MA. Mor­ tality in patients with IE is high and revascularization should be established through extra-anatomic routes via uninfected tissue planes. Long-term, suppressive antibiotic therapy will probably be required as patients are at high risk of recur­ rence of infection especially in the interposed vascular grafts in previously infected areas. T R E AT M E N T Certain principles are important when considering treatment of IE.65 The regimen must be bactericidal. Prolonged therapy is often necessary Vancomycin is less rapidly bactericidal than

514

SECTION VI • VALVULAR HEART DISEASE

semisynthetic penicillins and first-generation cephalosporins. IE is one of the situations where skin testing should be per­ formed on patients with a questionable history of immedi­ ate hypersensitivity reactions to penicillin. The American Heart Association46 and European Society of Cardiology56 have published guidelines for the treatment of adults with IE due to streptococci, enterococci, staphylococci, and HACEK microorganisms. These are summarized in Table 3 5 . 5 .

Caveats for the Common Causes of I E Minimum inhibitory concentrations (MIC) should b e deter­ mined for streptococci against penicillin as treatment is dependent on the values obtained. 1 A 2-week regimen65 may be appropriate in certain situations such as those cases of uncomplicated IE caused by highly penicillin-susceptible viridans streptococci, S. bovis in those patients at low risk for complications from gentamicin therapy For patients aller­ gic to beta-lactams, vancomycin is an effective alternative. Enterococcus faecium and E. fecalis46• 5 6 are the two maj or ente­ rococcal causes of IE. These organisms are relatively resistant to penicillin, expanded spectrum penicillins, and vancomy­ cin. They are also uniformly resistant to cephalosporins and relatively resistant to aminoglycosides. The combination of penicillin, vancomycin, or ampicillin with aminoglycosides is necessary to exhibit a synergistic bactericidal effect on these isolates and standard therapy should continue for 4 weeks. All enterococcal isolates causing IE must be screened for anti­ microbial susceptibility against penicillin, ampicillin, vanco­ mycin, gentamicin, and streptomycin. Optimal therapy has not been determined for isolates with high level resistance to both gentamicin and streptomycin. For organisms with intrinsic high-level resistance to penicillin (MIC > 1 6 µglmL) , vancomycin is the preferred agent for combination. Vanco­ mycin may enhance the nephrotoxic potential of amonigly­ cosides. Serum levels of aminoglycosides should be carefully monitored during therapy of enterococcal endocarditis. Peak levels of gentamicin should not exceed 3 and trough levels should not exceed 1 . Note that these levels are not as high as those when aminoglycosides are used in the synergistic treatment of gram-negative systemic infections. Daptomycin has proven in vitro activity against MRSA, methicillin resistant Staphylococcal epidermidis (MRSE), glycopeptide-intermediateS. aureus, and Vancomycin-resistant enterococcus (VRE) faecium in an in vitro pharmacodynamic model with simulated endocardial vegetations. 66 For methi­ cillin-susceptible staphylococci NVE,67 nafcillin or oxacillin must be used with a brief 3 to 5 day course of gentamicin. Though the aminoglycoside offered no significant mortal­ ity or morbidity benefit, it shortened the duration of posi­ tive blood cultures in a multicenter collaborative study while harmful side effects were reduced. For methicillin-suscepti­ ble staphylococcal IE in prosthetic valves, nafcillin or oxacil­ lin with rifampin for 6 weeks plus gentamicin for the first 2 weeks is recommended. For methicillin-resistant staphylo­ coccal native valve IE, vancomycin, usually with gentamicin added for the first 3 to 5 days of therapy is the standard. For

methicillin-resistant staphylococcal IE on prosthetic valves, combination therapy is advocated with vancomycin and rifampin for 6 weeks plus gentamicin for the first 2 weeks. 68 The benefit of rifampin69 in MRSA endocarditis has been derived from the ability of this drug to sterilize "foreign body infection" in experimental animal model. Coagulase-negative staphylococci are now the commonest cause of PV, particu­ larly in the first 12 months following surgery The organisms are usually methicillin resistant, and treatment should be the combination described. If the organism is resistant to gen­ tamicin, an aminoglycoside to which susceptibility is dem­ onstrated should be chosen. If the isolate is resistant to all aminoglycosides, this component should be omitted from the regimen. In the situation of right-sided native valve IE7°·7 1 caused by methicillin-susceptible S. aureus in IDUs, limited data suggest that a 2-week course of nafcillin or oxacillin with gentamicin may be sufficient. This regimen may not be suit­ able in IDUs with left-sided IE, metastatic IE such as lung abscess, underlying HIV, or vegetations > 1 to 2 cm. HACEK organisms7 2 should be considered ampicil­ lin resistant and monotherapy with this drug is no longer recommended. Limited data suggest that a third gen­ eration cephalosporin such as ceftriaxone or cefotaxime sodium should be used for 4 weeks in native valve and for 6 weeks in PV Aztreonam, trimethoprirn/sulfameth­ oxazole , or the fluoroquinolones are the recommended alternative agents for patients with HACEK IE unable to tolerate cephalosporins .

Caveats for the Uncom mon Causes of I E C. burnetii73 I E is usually treated with a combination of doxycycline and rifampin, trimethoprim-sulfamethoxazole, or fluoroquinolones. The optimal duration of therapy is unknown. Valve replacement is only indicated for CHF, PV, or uncontrolled infection. Many experts recommend long-term and possibly indefinite therapy in this set­ ting. Yet others have suggested a minimum of 3 years of therapy once phase I IgG antibody titers drop to < 1 :4,000 and phase I IgA antibody is undetectable. Few patients with Brucella 3 1 IE have been cured with antimicrobial therapy alone. Most require valve replacement in addition to the following: doxycycline plus either streptomycin or gentamicin OR doxycycline plus either trimethoprirn/ sulfamethoxazole or rifampin. Again, the optimal duration of therapy is unknown, but authorities recommend this reg­ imen for 8 weeks to 1 0 months following valve replacement. In Legionella3 2 IE, cure has been obtained by prolonged par­ enteral therapy with either doxycycline or erythromycin fol­ lowed by an oral course for a prolonged period. The total duration of therapy is usually 6 to 1 7 months. Pseudomonas aeruginosa74 IE of the right side usually requires the com­ bination of high doses of an antipseudomonal penicillin (piperacillin) and an aminoglycoside (tobramycin) . Left­ sided Pseudomonas IE rarely responds to medical therapy alone and surgery is considered mandatory In fungal IE59 caused by Aspergillus or Candida species, Amphotericin B

CHAPTER 3 5 • INFECTIVE E N D O CARDITIS

51 5

Therapy of NVE Caused by Highly Penicillin-Susceptible Viridans Group Streptococci and S. bovis Duration (wk)

Strength of Recommendation

1 2-18 million U/24 h IV, either continuously or in 4 or 6 equally divided doses

4

IA

2 g/24 h IV/IM in 1 dose Pediatric dose1' : penicillin 200,000 U/kg per 24 h IV in 4-6 equally divided doses; ceftriaxone 100 mg/ kg per 24 h IV/IM in 1 dose 1 2-18 million U/24 h IV either continuously or in 6 equally divided doses

4

IA

2

IB

2 g/24 h IV/IM in 1 dose

2

IB

Gentamicin sulfate'

3 mg/kg per 24 h IV/IM in 1 dose Pediatric dose: penicillin 200 ,000 U/kg per 24 h IV in 4-6 equally divided doses; ceftriaxone 100 mg/ kg per 24 h IV/IM in 1 dose; gentamicin 3 mg/kg per 24 h IV/IM in 1 dose or 3 equally divided doses'

2

Vancomycin hydrochloridef

30 mg/kg per 24 h IV in 2 equally divided doses not to exceed 2 g/24 h unless concentrations in serum are inappropriately low Pediatric dose: 40 mg/kg per 24 h IV in 2-3 equally divided doses

4

Reg imen

Dosage0 and Route

Aqueous crystalline penicillin G sodium

Comments

Preferred in most patients >65 y or patients with impairment of eighth cranial nerve function or renal function

or

Ceftriaxone sodium

Aqueous crystalline penicillin G sodium or

Ceftriaxone sodium plus

IB

2-wk regimen not intended for patients with known cardiac or extracardiac abscess or for those with creatinine clearance of 29 ,000 patients in 52 countries showed that >90% of the risk of an index MI is attributable to the following factors regardless of gender, eth­ nicity, or geography: tobacco , hyperlipidemia, hypertension, diabetes, obesity, sedentary lifestyle, alcoholism, low intake of fruits and vegetables, and psychosocial index. 3 Tobacco

According to the AHA, in 2009 >46 million Americans 18 years of age and older are active cigarette smokers, rep­ resenting nearly 2 1 % of the total population. This consti­ tutes 23% of all men and 18% of all women in the United States. While the total number has decreased by 3 . 5 % since 1 998, the number of new smokers every year has seemingly plateaued at approximately 2 . 4 million. Even more dis­ couraging, most new smokers (58.8%) in 2008 were < 1 8 years o f age. From 1 980 t o 2002 the number o f high school seniors who had smoked within the past month declined 1 2 . 5 % , and since 1965, smoking among persons 18 years of age and older has decreased 4 7% overall. The prevalence of smoking is highest among white men (25%), black men (23%), white women (2 1 %), and black women ( 1 9%). Of all ethnic groups, Asians and Hispanics had the lowest overall prevalence. Smokers have a two- to fourfold higher risk of CAD than nonsmokers, and two- to threefold higher risk of CAD-related mortality, and twofold greater risk for stroke. Even more striking, smokers have a > 1 0-fold risk of devel­ oping peripheral artery disease than nonsmokers. 1 3 The INTERHEART study estimated that tobacco accounts for 36% of the population attributable risk of a first MI.3 Impor­ tantly, it is estimated that the total estimated cost of smoking to society, including direct medical costs and lost productiv­ ity, was $ 1 93 billion per year between 2000 and 2004. Smoking cessation has proven benefits for CAD . According to the WHO , the risk of CAD decreases by 50% 1 year after abstention, and within 1 5 years the relative risk of death from CAD for an ex-smoker becomes equivalent to that of a lifetime nonsmoker. Lipids

The prevalence of lipid disorders in patients with CAD and in the general population is extraordinarily common. Of peo­ ple with premature CAD , 75% to 85% have dyslipidemia. In the United States in 2008, approximately one-third of all adults >20 years of age had an LDL cholesterol � 1 30 mgldl. For further details on dyslipidemia and its effects on coro­ nary disease, refer to Chapter 1 5 . Hypertension

According to AHA figures, 76 million Americans carried the diagnosis of hypertension in 2008. An additional 59 million people are classified as having prehypertension (systolic blood

pressure 120 to 139 mm Hg, or diastolic blood pressure 80 to 89 mm Hg) and are at risk for overt hypertension. Systolic blood pressure, diastolic blood pressure, and pulse pressure have all been separately identified as risk factors. The preva­ lence of hypertension in blacks in the United States is among the highest in the world and continues to increase.6 From 1988 to 2002 , the prevalence increased from 35.8% to 4 l .4% u According to the INTERHEART study, hypertension accounts for 18% of the population attributable risk of first MI. 3 Fur­ ther details on hypertension and its cardiovascular effects are included in Chapter 52. Dia betes Mel l itus

The prevalence of physician-diagnosed and undiagnosed diabetes mellitus among adults in the United States was 25.4 million in 2008. An additional 8 1 . 5 million (3 7%) have pre­ diabetes, with fasting blood glucose of 1 00 to < 1 26 mg/dl. From 1 990 to 2002 , the prevalence of diabetes increased 6 1 % in the United States. Mexican Americans and blacks currently have the highest rates of any ethnic group u The prevalence of diabetes in individuals 40 to 74 years of age is 1 1 . 2% for whites, 18.2% for blacks, and 20.3% for Hispan­ ics. Despite the higher prevalence in Hispanics, blacks were more likely to die from diabetes.7 Worldwide, the prevalence among all ages is 2 . 8 % , and it is expected to nearly double to 4.4% by 2030. Diabetes increases the risk of MI or cer­ ebrovascular accident by two- to threefold and doubles the risk of SCD. According to the INTERHEART study, diabetes accounts for 10% of the population attributable risk of first MI. 3 In 2002 the National Cholesterol Education Program (NCEP) was compelled by these and additional statistics to elevate diabetes to the category of CAD equivalent. O besity a n d the Metabolic Synd rome

According to the AHA, over 149 million Americans (67%) were overweight or obese in 2008, with 75 million (34%) being overtly obese. Further, the prevalence of overweight and obese children has quadrupled since the 1 960s, reach­ ing 32% of all American children in 2008. Obesity has reached true epidemic proportions in the United States and shows no signs of slowing down in the near future. From 1 999 to 2000 and from 2007 to 2008, the prevalence of obesity increased from 28% to 32% in adult men and 33% to 36% adult women, respectively 1 3 Mexican Americans con­ stitute the ethnic group with the greatest prevalence. Obesity is now recognized as an independent risk factor for CAD , although it also mediates its effects through other factors that are highly associated with it, such as hypertension, insulin resistance, and hypertriglyceridemia. Obesity accounts for roughly 20% of the population attributable risk of a first MI. The metabolic syndrome is defined by having �3 of the following: a fasting plasma glucose � 1 00 mg/dl or under­ going drug treatment for elevated glucose, HDL cholesterol 1 0-fold. b. The majority of new smokers start the ha bit d u r­ ing their early 20s, often while i n col l ege. c. Ciga rette smoking has a twofold g reater risk of suffering a CVA. d. The overal l prevalence of smokers has decreased in the United States compared to the mid 20th century. 7. Which ethnic g roup has the hig hest preva lence of diabetes? a. Whites b. H ispan ics c. Asians d. Blacks 8. Which of the following is not considered a criterion for the meta bolic synd rome? a. Havi ng a fasting blood g l u cose � 1 00 mg/d l b. U ndergoing treatment for hypertension c. Havi ng a body mass i ndex (BMI) � 30 d. Trig lycerides � 1 SO mg/d l 9. Treatment with statins has been shown to red uce the i ncidence of major cardiovascu l a r events i n nonhyperl i pidemic patients with w h i c h abnormal novel risk ma rker? a. High-sensitivity (-reactive protein (hs-CRP) b. Alanine a m i n otransferase c. NT- B N P d. Cystatin C 1 0. All of the fol lowi ng a re true rega rding sudden ca rdiac death (SCD) except: a. The incidence is higher in men than women. b. The median su rviva l rate fol lowi ng ventricu lar fibril lation is approximately 2 1 %.

CHAPTER 3 8 • CORO NARY ARTERY DISEAS E : DEMO GRAPHICS AND INCIDENCE

c . T h e r i s k o f developing S C D is fou r t o s i x times higher i n patients fol lowi ng a n i ndex M l . d. The most com mon presenting a rrhyth mia for SCD is polymorphic VT. Answers 1 . Answer C: Black America ns have a morta lity that is

1 .6 times that of wh ite Americans. Add itiona l ly, blacks are more l i kely than whites to suffer from hypertension, dia betes, and obesity. 2. Answer D: It is estimated that rou g h ly 2 1 % off a l l new acute Mis were sym ptomatica lly silent i n 2007. It i s thought that silent M i s a re associated with worse over­ a l l outcomes, largely due delayed diagnosis and risk factor prog ression. The i ncidence of STEM I has decl ined over the past severa l decades. Between 1 999 and 2004, the incidence of NSTEM I actu a l ly increased, largely due to the widespread availability of sensitive troponin as­ says. Since 2004, they have g radually decli ned. 3. Answer E: All of the mentioned factors a re con­ tributory for CAD becoming the predominant cause of death in the U n ited States for the past centu ry. Many of the developing world cou ntries a re cu rrently undergo­ ing similar transitions with i m p rovements i n overa l l public health systems, cleaner water, more abu ndant food, and modern ization and industrialization of industries. Wh ile the burden of i nfectious diseases and m a l n utrition decrease, the diseases associated with longer l ife such as cancer, hypertension, dia betes, and heart disease predominate. 4. Answer B: The incidence of a ny man ifestation of

CAD in women lags behind that in men by 1 0 years, whereas the incidence of Ml or sudden death lags by 20 years. H owever, age is one of the strongest pred ic­ tors of mortality in CAD, i ndependent of race. S. Answer D: Lost prod uctivity from premature death

accou nts for an estimated $95 billion overa l l cost to so­ ciety per year.This is in excess of the direct costs associ­ ated with hospita lizations, medications, outpatient visits, and emergency room visits combined. CVD expenditures exceed the cost of treating a l l forms of cancer. The most

549

expensive direct expenditure for CVD comes from inpa­ tient hospitalizations. The cost of CVD has increased de­ spite a reduction in tota l yearly patient hospitalizations. 6. Answer B: The highest i ncidence of new smokers

is observed in persons < 1 8 yea rs of age. Accord ing to a report from the Surgeon General, >80% of a l l smok­ ers sta rted smoking when < 1 8 yea rs, with the age of i n itiation most com m o n ly bei ng 1 4- and 1 5 yea rs old. Cigarette smoking has decli ned i n the U n ited States when compared to the mid 20th centu ry, however, si nce 2005 the preva lence seems to have plateaued at arou nd 20%. Smoking is known to g reatly increase the risk of PAD, stroke, and CAD. 7. Answer B: Hispa n ics have the highest prevalence of dia betes i n the U n ited States. However, morta l ity rates related to dia betes a re hig hest among blacks. 8. Answer C: BMI is not part of the contempora ry

defi n ition for the meta bolic synd rome. However, a waist ci rcu mference � 1 02 cm i n men or �88 cm i n women is included. All o f t h e other factors a re included. Of note, for each of the pa rameters i nvolving g l ucose, hypertension, or l ipids, the cutoff va lues a re releva nt only for people not a l ready being actively treated. Once treatment has been i n itiated, that particu lar risk factor should be incl uded in the req u i red �3 for the defi n ition. 9. An swer A: hs-CRP was shown i n the l a rge J u piter

study to be a potential mod ifiable risk ma rker when treated with rosuvastatin, even in patients without hyperlipidemia. Alanine a m inotransferase, NT- B N P, and Cystatin C a l l have been loosely correlated with pred icting adverse ca rdiovascular risk but not tested rigorously with rega rd to the effects statins have on them. 1 0. Answer D: The most common arrhythmia associat­ ed with SCD is VF, accounting for >50% of all CAD-related mortality. While survival following treated VF is as high as 21 %, the median rate of surviva l to hospital discharge is < 1 0%. Patients with documented CAD have far higher risk of developing SCD than the general population, as do patients with cardiomyopa­ thies and other genetic predispositions.

S table An gina : Dia gnosis , Risk S tratification , Medical Therapy, and Revascularization S trate gies Kellan E . Ashley and C onrad C . Simpfendorfer

T

he syndrome of angina pectoris, first described in 1 768 by William Heberden, is most commonly the symptomatic result of fixed coronary artery obstruc­ tion and impaired endothelial vasomotor activity in patients with advanced coronary atherosclerosis. In stable angina, symptoms occur in a predictable and reproducible fashion during periods of physical or emotional stress, when increas­ es in heart rate, cardiac contractility, and afterload increase myocardial oxygen requirements. Relief is usually brought on with rest or nitroglycerin. Symptom severity can be wide­ ly variable from patient to patient and is most commonly graded according to the Canadian Cardiovascular Society (CCS) scale (Table 39. 1 ) . 1 The epidemiology of coronary artery disease (CAD) and stable angina has changed in the United States. As a result of improved medical and revascularization strategies, the myocardial infarction (MI) survival rate has improved, thus allowing patients to live longer with more chronic manifesta­ tions of CAD . From 1997 to 2007, the annual death rate due to CAD decreased by 26.3%, with more than 80% of deaths in patients 65 years of age or older. 2 With the aging of the population, there has been an increase in the prevalence of CAD , with current estimates showing over 1 6 million Amer­ icans living with stable CAD . This represents approximately 7% of the U.S. population over 20 years of age. 2 Despite these substantial improvements, CAD remains the number one killer of both men and women in the United States, causing about one in six deaths in 2007. 2

550

Stable angina pectoris remains among the most common initial clinical manifestations of CAD . Of the 1 6 million Americans with known CAD , an estimated 9 million live with chronic stable angina, a number that is expected to increase as the population continues to age. 2 The presence of symptomatic CAD affects quality of life negatively It imparts significant morbidity and mortality, with estimates of just over 1 0 % annual incidence of either a nonfatal MI or coronary death in patients presenting with stable angina. 3 While most of the early data were in men, more recent data suggest an equally poor outcome in women presenting with angina.4 D E F I N I T I O N O F S TA B L E A N G I N A Angina is defined as the sensation of chest discomfort that occurs in the setting of myocardial ischemia in the absence of myonecrosis.5 It is traditionally described in terms of its clinical setting, characteristics (quality, location, radiation, etc.), precipitating or alleviating factors, and time course to cessation. The classic description is of substernal pres­ sure or heaviness that can radiate to the left arm, jaw, neck, or shoulders. Chronic stable angina usually occurs due to fixed coronary obstructions of �50% of the diameter of the left main trunk or �70% of the diameter of one of the other epicardial coronary arteries and is a demand phenomenon.5 The pain is predictable, occurring with physical activity at a known threshold or related to emotional stress. Most often,

CHAPTER 39 • STABLE ANGINA TA B L E

CCS Classification of Angina

Class I Class II Class III Class IV

No angina with ordinary physical activity. Strenuous activity may cause symptoms. Angina causes slight limitation on ordinary physical activity. Angina causes marked limitation on ordinary physical activity. Angina occurs with any physical activity and may be present at rest.

From Campeau L Letter: grading of angina pectoris. Circulation . 1 9 7 6 ; 54(3) : 522-523

it is relieved quickly with rest and does not last longer than 1 0 minutes. 5 Typical stable angina meets all three of the following criteria: substernal location with the characteris­ tic quality and duration, provoked by exertion or emotional stress, and relieved by rest or nitroglycerin. 6 Unfortunately, classic symptoms or triggers are not pre­ sent in all patients, most notably diabetics, women, and the elderly. This is referred to as atypical angina and can some­ times lead to later diagnoses and poorer outcomes.5 Atypical angina meets two of the three characteristics listed above. Patients who do not experience any chest discomfort can still be diagnosed as having stable angina. Diabetics and the elderly are more likely to experience anginal equivalents, such as dyspnea, diaphoresis, fatigue, nausea, light-headedness, altered sensorium, or syncope.5 It has been theorized that the lack of chest discomfort is related to altered pain percep­ tion or autonomic neuropathy. In its most ominous form, the ischemia can be completely silent. 5 Finally, noncardiac chest pain is defined as pain that meets one or zero of the characteristics of typical angina. 6 PAT H O P H YS I O LO G Y O F S TA B L E ANGINA Simply put, angina occurs as the result of an imbalance between myocardial oxygen/substrate supply and demand.5 Conditions that alter oxygen or other substrate (glucose and free fatty acids) supply can produce angina even in spite of a normal demand state. Conversely, with exercise or emotional stress, myocardial demand is increased due to increased heart rate, blood pressure, and left ventricular contractility in relation to a sympathetic nervous response. With signifi­ cant CAD , the oxygen/substrate supply is fixed due to the inability of coronary autoregulation to increase coronary blood flow due to the fixed stenosis. Oxygen extraction in the coronary arteries is already at a maximum at baseline, so increased extraction cannot satisfy the imbalance between supply and demand. Thus, the patient experiences angina in these states of increased demand.5

551

D I AG N O S T I C T E ST I N G / R I S K S T R AT I F I C AT I O N Evaluation o f the patient with symptoms suspicious for CAD begins with assessing for the presence of CAD historical predictors, such as pain character and setting, age, gender, diabetes mellitus, smoking, hypertension, and hyperlipi­ demia.7 This allows one to establish the pretest likelihood of disease and helps determine which, if any, diagnostic test­ ing is needed. Each patient presenting with suspected stable angina should have a thorough history and physical exami­ nation as well as an electrocardiogram (ECG) (American College of Cardiology/American Heart Association [ACC/ AHA] Class I recommendation) .6 There are certain features in this initial workup that can indicate a high, intermediate, or low likelihood of CAD and help to guide further evalu­ ation (Table 39.2). In those with a significant suspicion for CAD , the spectrum of risk is broad and warrants different evaluation and treatment strategies for different levels of risk. Conversely, patients with low-risk features may war­ rant a more conservative and less invasive evaluation and treatment course. However, stratifying patients in this man­ ner is not fail-safe; it is still possible to have obstructive CAD but with low-risk symptoms or features. The only way to completely rule out the presence of CAD is with a coronary angiogram.5

Electrocardiography Although the 12-lead ECG is normal in more than half of patients with chronic stable angina, an ECG is a read­ ily available first-line test that provides both diagnostic and prognostic information (ACC/AHA Class I recommenda­ tion) .6 A normal ECG at the time of diagnosis is associated with a favorable long-term prognosis, whereas abnormali­ ties such as left ventricular hypertrophy, Q waves suggestive of prior MI, and persistent ST-segment depression identify patients at higher risk of future adverse events.6·8 In the half of patients with a normal baseline ECG, one obtained dur­ ing an episode of pain will be abnormal in another 50% of patients. Evidence of ST-segment deviation signals a high likelihood of CAD and has a more unfavorable prognosis. 6 Patients with ECG abnormalities at baseline can have "pseu­ donormalization" of these during a pain episode; this also indicates a high likelihood of CAD .6 Depending on the ECG and the underlying likelihood of CAD , a decision is made to either pursue further noninvasive testing or proceed straight to a coronary angiogram. Exercise ECG Testing One of the oldest and most widely used noninvasive tests for CAD is the treadmill exercise ECG, because of its widespread availability, low cost, and ease of performance.5 This is gen­ erally the first test selected to evaluate patients with an inter­ mediate likelihood of CAD and a normal baseline ECG who are able to exercise (ACC/AHA Class I recommendation) .6

552

SECTION V I I • CORONARY ARTERY DI SEASE A N D CARD IAC CATHETERIZATION

C l i n ica l Featu res and Likelihood o f C A D i n Patients Presenting with Angina

High Likeli hood of CAD

I ntermediate Likeli hood of CAD

Low Likeli hood of CAD

At least one of the following:

No high risk features and one of the following: Rest pain that lasts longer than 20 min but is relieved with rest or nitroglycerin Nocturnal angina Age > 65 years

No high or intermediate risk features and one of the following: Increased frequency, severity, or duration of pain Pain provoked at lower threshold New onset (within 2 weeks to 2 months) angina Normal ECG or minor ECG changes (i. e . , T-wave flattening, T-wave inversions in leads with dominant R waves, etc.)

Rest pain that lasts longer than 20 min Associated pulmonary edema Associated hypotension Associated S 3 gallop or new/ worsening rales Rest angina with new ST-segment deviation �1.0 mm Associated with new or worsening mitral regurgitation murmur

Associated with dynamic T-wave changes New onset (:Q weeks) angina with moderate or high likelihood of CAD Resting ST-segment depression ::;; 1 mm in multiple leads Q waves on ECG

From Gibbons RJ , Abrams J , Chatterjee K, et al. ACCJAHA 2002 guideline update for the management of patients with chronic stable angina-summary article: a report of the American College of Cardiology/American Heart Association Task Force on Practice Guidelines (Committee on the Management of Patients With Chronic Stable Angina). Circulation. 2003 ; 1 0 7 ( 1 ) : 149- 1 5 8 , with permission.

The absolute contraindications to exercise ECG testing include hemodynamically significant arrhythmias, within the first 48 hours of MI, symptomatic heart failure, sympto­ matic or severe aortic stenosis, myocarditis, acute aortic dis­ section, and acute pulmonary embolus. There are also ECG abnormalities that are not suited for exercise ECG testing, as they make interpretation of the exercise ECG impossible. These include Wolff-Parkinson-White syndrome, paced ventricular rhythms, > 1 mm resting ST-segment depression, and complete left bundle branch block (ACC/AHA Class III recommendation) . 6 In a large meta-analysis of exercise stress ECG, the mean sensitivity and specificity for detecting angiographically sig­ nificant CAD were 68% and 77% , respectively.6 Although exercise ECG is less sensitive than stress tests performed with imaging modalities, particularly in women, it remains the primary noninvasive tool for both the diagnosis and risk stratification of patients with suspected CAD and inter­ pretable ECGs. The ACC/AHA guidelines recommend that, unless cardiac catheterization is more urgently indicated, symptomatic patients with suspected or known CAD should be considered for exercise ECG testing to assess the risk of future cardiac events and the possible need for angiography.6 As a diagnostic tool, exercise ECG testing is most useful in patients with stable chest pain syndromes and an interme­ diate risk of CAD (ACC/AHA Class I recommendation) . In those with a low or high pretest probability of CAD , exercise

ECG has a Class Ilb recommendation. 6 As a prognostic tool, it can help to identify patients with extensive atherosclerosis who would benefit from coronary angiography and possible revascularization. In addition to the ECG portion, there are other vari­ ables that contribute to the interpretation of the test. The usual definition for a positive exercise ECG test is 1 mm or more of ST-segment elevation or horizontal or downsloping ST-segment depression at a point 60 to 80 milliseconds after the QRS complex during exercise or recovery.6 Additionally, symptoms, exercise capacity, and hemodynamic and rhythm response to exercise should be considered.6 The most impor­ tant prognostic variables measured during exercise test­ ing are exercise capacity, typically expressed in metabolic equivalents of task (METs) , and exercise-induced ischemic ST-segment changes. The Duke Treadmill Score (DTS) inte­ grates these two objective variables with the subjective pres­ ence or absence of anginal symptoms to generate a risk score that separates patients into high- , moderate-, and low-risk subsets (5 % , 1 .2 5 % , and 0.25% annual mortality rates, respectively) (Table 39.3).9 Patients with high-risk DTSs frequently have left main or three-vessel CAD that would benefit from revascularization, and these patients should be referred for coronary angiography. Low-risk patients, on the other hand, have an excellent prognosis that is unlikely to improve with further evaluation or revascularization and thus can be treated safely with medical therapy.

CHAPTER 39 • STABLE ANGINA TA B L E

D u ke Tread m i l l Score

Calculation:

DTS exercise time in minutes on the Bruce protocol (5 x ST-segment deviation in mm) - (4 x angina index) (angina index: 0 none, 1 nonlimiting, and 2 exercise limiting) =

=

=

=

Risk stratification: Low risk, DTS �5 0.25% annual mortality 1 .25% annual mortality Intermediate risk, DTS - 1 0 to +4 High risk, DTS :S: - 1 1 5.25% annual mortality From Mark DB, Shaw L, Harrell FE Jr, et al. Prognostic value of a treadmill exercise score in outpatients with suspected coronary artery disease. N Engl ] Med. 1 9 9 1 ;325( 1 2) : 849-85 3 , with permission from the Massachusetts Medical Society

Echocardiography Most patients undergoing a diagnostic evaluation for stable angina do not require echocardiography. More specifically, in patients with a normal ECG, no history of prior MI, and no clinical signs or symptoms of heart failure, valvular disease, or hypertrophic cardiomyopathy, it is currently contraindi­ cated to obtain an echocardiogram (ACC/AHA Class III rec­ ommendation) . 6 An exception is when there is a murmur suspicious for aortic stenosis or hypertrophic cardiomyopa­ thy on physical exam or when the echocardiogram can be obtained during or within 30 minutes of presentation with chest pain to evaluate for regional wall motion abnormali­ ties (ACC/AHA Class I recommendation) . 6 In this setting, regional wall motion abnormalities have a positive predictive value for ischemia of approximately 50%, whereas normal studies identify patients at low risk for an acute infarction.6· 1 0 Stress Testing with N uclear or Echocardiographic I maging Although stress imaging modalities have greater diagnostic accuracy than exercise electrocardiography, the increased cost of these tests precludes their routine use in all patients with suspected CAD . Most commonly, nuclear (single posi­ tron emission computed tomography [SPECT] or positron emission tomography [PET] ) or echocardiographic stress imaging is reserved as first-line testing in patients with abnor­ mal baseline ECGs (i.e. , pre-excitation, resting ST-segment depression � l mm) or with symptoms and history of prior revascularization (percutaneous coronary intervention [PCI] or coronary artery bypass grafting [CABG] ) (ACC/AHA Class I recommendation) .6 In patients with either paced ventricu­ lar rhythms or left bundle branch block, pharmacologic stress myocardial perfusion imaging is preferred over stress

553

echocardiography due t o the difficulty with interpreting echocardiographic wall motion in these conditions (ACC/ AHA Class I recommendation) .6 For patients who are una­ ble to exercise, pharmacologic stress myocardial perfusion imaging or dobutamine stress echocardiography is equally preferred (ACC/AHA Class I recommendation) .6 For a sum­ mary of the ACC/AHA recommendations, see Table 39.4. As mentioned, stress myocardial perfusion imaging has a higher sensitivity for the diagnosis of CAD than exer­ cise ECG in patients with intermediate risk. The data vary depending on the population studied, but generally the sen­ sitivity is accepted to be around 90% for myocardial perfu­ sion imaging. 5 Similar numbers are seen with exercise stress echocardiography (sensitivity around 85%) and dobutamine echocardiography (sensitivity around 82%). 6 The choice of test depends on both patient characteristics and local exper­ tise in performing the different imaging modalities. There are some special populations or situations in which exercise stress imaging should be considered over exercise ECG. Women have an overall lower prevalence of CAD than men and thus have a lower pretest probability of disease, making false-positive exercise ECGs more common. The higher sensitivity of stress imaging, therefore, could theoretically improve on the positive predictive value of the exercise ECG.6 Additionally, elderly patients are oftentimes less able to exercise due to comorbid medical conditions or deconditioning. Therefore, pharmacologic stress imag­ ing may be the test of choice in this population.6 As with exercise electrocardiography, stress imaging results can also provide prognostic information, separating patients who are appropriate for medical therapy (low risk, :S: l % annual mor­ tality) from those who may benefit from further angiographic evaluation and possible revascularization (intermediate risk, 1 % to 3 % ; high risk, �3% annual mortality) (Table 39.5). 11

Coronary Angiography Coronary angiography remains the gold standard diag­ nostic test for CAD . Additionally, coronary angiography is able to provide anatomic definition of disease extent and severity as well as prognostic information, identifying those patients who would achieve survival benefits related to sur­ gical revascularization. Specifically, from the early studies of CABG, patients with severe left main trunk stenosis, three­ vessel disease, and two-vessel disease involving the proximal left anterior descending (LAD) coronary artery are known to derive a survival benefit with bypass surgery over medical therapy. 12 In general, coronary angiography is performed in patients with stable chest pain syndromes when noninvasive tests are inconclusive or cannot be performed, when clinical evaluation or noninvasive testing suggests high-risk features (see Table 39.5), and when symptoms persist despite appro­ priate medical therapy. Less commonly, diagnostic coronary angiography is recommended for patients in whom coronary artery spasm is suspected, those with occupations that neces­ sitate a definitive diagnosis (e.g. , pilots, police, professional

554

S ECTION V I I • CORONARY ARTERY DI SEASE AND CARD IAC CATHETERIZATION

ACC/AHA Recom mendations for Exercise ECG Testing and Stress I maging Studies in Sta ble Angina Pectoris Exercise ECG Testing without Imaging

Class I Class Ila Class Ilb

Class III

For diagnosis of obstructive CAD in patients with an intermediate pretest probability of CAD For risk assessment and prognosis in patients undergoing initial evaluation Suspected vasospastic angina High pretest probability of CAD Low pretest probability of CAD Digmcin therapy with < l mm ST-segment depression on baseline ECG ECG criteria for LV hypertrophy and l mm at rest) that preclude exercise ECG testing (dipyridamole or adenosine MPI preferred in patients with LBBB or electronically paced ventricular rhythm) Pharmacologic stress imaging in patients with an intermediate pretest probability of CAD who are unable to exercise Exercise myocardial perfusion imaging or exercise echocardiography in patients with prior revascular­ ization (either PCI or CABG) (pharmacologic stress can be used for those unable to exercise.) Patients with low or high pretest probability of CAD with abnormal baseline ECGs (pre-excitation or >l mm ST-segment depression) that preclude exercise ECG testing (dipyridamole or adenosine MPI preferred in patients with LBBB or electronically paced ventricular rhythms) Exercise MPI or exercise echocardiography in patients with intermediate pretest probability of CAD who are taking digoxin and have < l mm ST-segment depression on baseline ECG or have LV hypertrophy and < l mm ST-segment depression on baseline ECG Exercise or dobutamine echocardiography in patients with LBBB Exercise myocardial perfusion imaging in patients with LBBB Risk stratification in patients with severe comorbid conditions likely to limit life expectancy or prevent revascularization

CABG, coronary artery bypass grafting; CAD , coronary artery disease; ECG, electrocardiogram; LBBB, left bundle branch block; LV, left ventricular; MP!, myocardial perfusion imaging; PC!, percutaneous coronary intervention. From Gibbons RJ , Abrams J, Chatterjee K, et al. ACCJAHA 2002 guideline update for the management of patients with chronic stable angina-summary article: a report of the American College of Cardiology/American Heart Association Task Force on Practice Guidelines (Committee on the Management of Patients With Chronic Stable Angina) . Circulation . 2003 ; 1 0 7 ( 1 ) : 149- 1 5 8 , with permission.

athletes) , and for survivors of sudden cardiac death u For the list of ACC/AHA recommendations related to coronary angiography, see Table 39.6. For patients determined to be at low risk of adverse events, medical therapy alone is usually sufficient and may be superior to an invasive approach. For the moderate­ or high-risk subsets, there is older evidence from rand­ omized trials that medical therapy coupled with surgical revascularization improves long-term survival over medical therapy alone. 12 Therefore, the judicious use of noninvasive

and invasive studies can help establish the diagnosis of CAD while simultaneously performing the critical risk stratifica­ tion that is essential in determining the appropriate risk­ reducing treatment strategies, from medical therapy alone to medical therapy plus revascularization.

Conclusion Chest pain is the most common initial presenting symp­ tom for patients diagnosed with CAD . Although coronary angiography provides powerful prognostic information and

CHAPTER 3 9 • STABLE ANGINA

555

Risk Stratification Based on Findings of Noninvasive Testing High R i s k (�3% Annual Mortal ity)

Severe resting LV dysfunction (EF < 35%) Severe exercise LV dysfunction (EF < 35%) Stress-induced, multiple moderate perfusion defects Stress-induced moderate perfusion defect with LV dilation or increased lung uptake (Thallium-20 1)

High-risk DTS (2 segments) developing at low dose of dobutamine (::; 1 0 µg/kg/min) or low HR (< 120 bpm)

Stress echocardiographic evidence of ischemia I ntermed iate Risk (1 %-3% Annual Mortal ity)

Mild/moderate resting LV dysfunction (EF 35 %-49%) Stress-induced moderate perfusion defect without LV dilation or increased lung uptake (Thallium-20 1)

Intermediate DTS (- 1 1 < DTS < 5) Limited stress echocardiographic ischemia with a wall motion abnormality only at higher doses of dobutamine involving ::;2 segments

Low Risk (::;1 % Annual Mortality)

Low-risk DTS (�5)

Normal or small myocardial perfusion defect at rest or with stress (unless high-risk DTS or severe resting LV dysfunction)

Normal stress echocardiographic wall motion or no change of limited resting wall motion abnormalities during stress (unless high-risk DTS or severe resting LV dysfunction) DIS, Duke Treadmill score ; EF, ejection fraction; HR, heart rate ; LV, left ventricular. From Gibbons RJ , Abrams ] , Chatterjee K, et al. ACC/AHA 2002 guideline update for the management of patients with chronic stable angina-summary article: a report of the American College of Cardiology/American Heart Association Task Force on Practice Guidelines (Committee on the Management of Patients With Chronic Stable Angina) . Circulation. 2003 ; 1 07 ( 1 ) : 149- 1 5 8 , with permission.

remains the gold standard for diagnosis, noninvasive tests are often more appropriate initial tools for patients with low or intermediate clinical predictors of CAD. Even for patients with a high pretest probability of CAD , noninvasive testing can be a useful prognostic tool that allows for selection of patients who warrant further invasive evaluation. M E D I C A L T R E AT M E N T F O R STA B L E ANGINA There are two primary goals of medical treatment for chronic stable angina: prevent MI and cardiovascular death as well as improve quality of life by decreasing anginal symptoms and occurrence of ischemia.6 However, the process of atherosclerosis cannot be reversed by medications or by

revascularization procedures. Lifestyle changes can and do influence the disease course and are most often underuti­ lized in the treatment of patients with stable CAD . 1 4 Life­ style changes are inexpensive, readily available, and very effective but require a motivated patient as well as support and emphasis from the physician. Lifestyle changes should be implemented first-line and should be complementary to medical therapy. PCis became increasingly more common in the late 1 990s and early 2000s due to an overall favorable effect on reducing anginal symptoms in numerous studies of PCI versus medical therapy in CAD. 1 5-2 3 The early meta-analyses of these studies did not show a reduction in mortality or MI incidence compared to medical therapy alone in these patients, 2 4• 2 5 even though a more recent meta-analysis did

556

SECTION V I I • CORONARY ARTERY DI SEASE A N D CARD IAC CATHETERIZATION TA B L E

ACC/AHA Recommendations for Coronary Angiogra phy in Sta ble Angina Pectoris

Class I

Class Ila

Class Ilb

Class III

CCS Class III or IV angina despite OMT High-risk criteria on clinical assessment or noninvasive testing regardless of anginal severity Stable angina patients with sudden cardiac death or serious ventricular arrhythmia Stable angina patients with congestive heart failure Equivocal findings/uncertain diagnosis after noninvasive testing Unable to undergo noninvasive testing due to disability, illness, or morbid obesity Occupational requirement for a definitive diagnosis (e.g. , pilots, bus drivers, police) CCS Class III or IV angina that improves to Class I or II with medical therapy Worsening abnormalities on serial noninvasive tests Suspected nonatherosclerotic cause for chest pain or myocardial ischemia (coronary anomaly, Kawasaki disease, primary coronary artery dissection, radiation-induced vasculopathy) Suspected coronary spasm where provocative testing is felt necessary High pretest probability of left main trunk or three-vessel CAD Patients with recurrent hospitalizations for chest pain in whom a definitive diagnosis is deemed necessary Patients with an overriding desire for a definitive diagnosis and more than low probability of CAD CCS Class I or II angina with demonstrable ischemia but no high-risk features on noninvasive testing Risk of coronary angiography outweighs the benefits. Patients with CCS Class I or II angina who respond to medical therapy and have no evidence of ischemia on noninvasive testing Patients who prefer to avoid revascularization Angina in patients who are not candidates for revascularization or in whom revascularization is not likely to improve quality or duration of life Patients with an overriding desire for a diagnosis but who are low probability of CAD

CAD , coronary artery disease ; CCS, Canadian Cardiovascular Society; OMT, optimal medical therapy From Scanlon PJ , Faxon DP, Audet AM, et al. ACC/AHA guidelines for coronary angiography: executive summary and recommendations. A report of the American College of Cardiology/American Heart Association Task Force on Practice Guidelines (Committee on Coronary Angiography) developed in collaboration with the Society for Cardiac Angiography and Interventions. Circulation. 1 999;99(17):2345-2357, with permission.

show a possible improvement in overall mortality with PCI. 2 6 As mentioned previously, CABG does have clear-cut long­ term survival benefits versus medical therapy alone, but only in a minority of patients with high-risk angiographic fea­ tures. 12 An initial trial of medical therapy; therefore, remains the mainstay of treatment for the maj ority of patients with chronic, stable CAD . This is achieved through a combina­ tion of therapies that target both ischemic symptoms and modifiable risk factors known to aggravate angina and car­ diovascular disease. Medications known to reduce the risk of Ml and death receive the highest priority Medications aimed

at improving quality of life by reducing the frequency and severity of anginal episodes serve as important supplemen­ tary therapies.

Medical Therapy to Improve Su rvival As discussed in the next section, the symptoms of angina may be effectively reduced with the use of standard antianginal medications (e.g. , beta-blockers, nitrates, and calcium channel blockers [CCBs] ), but these therapies have not been shown to improve survival or reduce MI incidence in patients with otherwise uncomplicated stable angina. Therefore,

CHAPTER 39 • STABLE ANGINA the management of patients with stable CAD has evolved to include a set of standard therapies directed specifically at reducing adverse clinical outcomes such as death and Ml . Anti platel et Th era py

The benefit of aspirin in a broad spectrum of patients with both stable and unstable atherosclerotic syndromes has been well established for decades. 2 7 Aspirin exerts its antiplatelet effects by inhibiting cyclooxygenase, thus preventing the release of the prothrombotic platelet-aggregant thromboxane A2 . Although it does not improve symptoms, clinical trials of aspirin in patients with chronic stable angina have dem­ onstrated risk reductions for adverse cardiac events that are of a magnitude similar to that seen in patients with unstable coronary syndromes. 2 7 In the Swedish Angina Pectoris Aspi­ rin Trial, 2 8 the largest randomized trial of aspirin therapy for chronic stable angina, the addition of 75 mg of aspirin to sotalol resulted in a 34% reduction in the primary composite endpoint of MI and sudden death and a 22 % to 32 % reduc­ tion in the measured secondary vascular endpoints (vascular death, all-cause mortality, and stroke) . A similar 33% reduc­ tion in adverse cardiovascular events (vascular death, stroke, and MI) was demonstrated among 2 ,920 patients with sta­ ble angina included in a meta-analysis performed by the Antithrombotic Trialists' Collaboration. 2 7 Therefore, aspirin, administered at 75 to 1 62 mg daily, is first-line therapy in all chronic CAD patients (ACC/AHA Class I recommendation) . 29 Thienopyridines are a second class of beneficial anti­ platelet agents that exert their effects by irreversibly and selectively inhibiting the binding of adenosine diphosphate (ADP) to receptors on the platelet surface, thus preventing platelet activation. Without platelet activation, the glycopro­ tein Ilb/Illa receptor is unable to undergo a conformational change, which then makes it unable to bind fibrinogen or von Willebrand factor. In this manner, platelet aggregation is inhibited. There are currently three drugs in this class on the market: ticlopidine, clopidogrel, and prasugrel. The oldest, ticlopidine, initially showed benefit in several atheroscle­ rotic processes including post-PCI , 30-34 unstable angina, 35 and peripheral arterial disease. 3 6-38 However, its widespread use was limited by its side effect profile, which included a risk of neutropenia as well as thrombotic thrombocytopenic purpura. Due to its more favorable side effect profile and reduc­ tion in cardiovascular events, clopidogrel has become the thienopyridine of choice in combination with aspirin for acute coronary syndromes,39 ST-segment elevation MI,40·4 1 and post-PCI.42 ·43 However, no study has specifically addressed its effect in patients with stable angina.6 In the Clopidogrel versus Aspirin in Patients at Risk for Ischemic Events (CAPRIE) trial,44 clopidogrel appeared to be more effective than aspirin, with an overall 8. 7% reduction in the combined primary endpoint (MI, vascular death, or ischemic stroke) , in high-risk CAD patients (i.e. , those with recent MI or stroke or with symptomatic peripheral arterial disease) . To date, the only setting in which clopidogrel has not been

557

shown t o improve cardiovascular outcomes is long-term pri­ mary or secondary prevention in patients with established atherosclerosis or multiple risk factors.45 Given the limited data for clopidogrel in stable coronary syndromes, it remains an ACC/AHA Class Ila recommenda­ tion as a replacement for aspirin in patients with a contrain­ dication. 6 Prasugrel, the newest member o f this class, has been shown to be a more potent antiplatelet agent than clopidog­ rel46 and has been shown to be more effective in reducing cardiovascular endpoints (cardiovascular death, nonfatal MI, or nonfatal stroke) in patients with acute coronary syn­ dromes. 47 However, prasugrel has a higher risk of maj or bleeding and to date has not been studied in stable CAD . Lipid-Lowering Th e ra py

Lipid management has been guided for decades by the National Cholesterol Education Program (NCEP) Adult Treatment Panel (ATP) . All NCEP ATP reports have targeted low-density lipoprotein cholesterol (LDL-C) as the driving risk factor for CAD .48 However, prior to the third report of NCEP, most of the evidence for LDL-C lowering was based on trials of lipid-lowering agents other than HMG-CoA reductase inhibitors (statins), such as bile acid sequestrants, fibric acid derivatives, and niacin.6·48 In 200 1 , the third report of the NCEP ATP was published,49 which reviewed the data from several randomized trials of statin therapy in CAD . With these guidelines, a treatment algorithm for LDL-C was established that focused on statins as first-line agents for all patients with stable CAD . In 2004, some revi­ sions to those recommendations were required based on the results of several newer randomized trials not included in the original guidelines.48 In general, statins lower LDL-C, total cholesterol, and triglycerides (TG) , while raising high-density lipoprotein cholesterol (HDL-C) , all of which are favorable in cardiovas­ cular disease. In aggregate, the randomized trials and meta­ analyses of statins in primary and secondary prevention show reductions in MI, stroke, and cardiovascular death by about one-third each, as well as a reduction in total mortality by about one-fifth.50 The ATP III algorithm categorized patients into three risk categories: (a) established coronary heart dis­ ease (CHD) or CHD risk equivalents, (b) two or more CHD risk factors, or (c) 0 to 1 CHD risk factor.48 High risk was defined as those with CHD or CHD risk equivalents (known noncoronary atherosclerotic vascular disease, diabetes, or two or more CHD risk factors with 10-year risk for CHD > 20%) .49 According to the original NCEP ATP III report49 and the subsequent revisions based on newer trial data,48 statin therapy and therapeutic lifestyle changes are indicated for all stable CAD patients with an LDL-C ;::: 100 mg/dL, with a goal of LDL-C < 1 00 mg/dL (ACC/AHA Class I recommenda­ tion) . 2 9 For patients deemed high risk, there is the optional goal of treating to a more aggressive LDL-C < 70 mgldL or with a high-dose statin (ACC/AHA Class Ila recommenda­ tion) . 2 9 If on-treatment LDL-C is ;::: 1 00 mg/dL, lipid-lowering

558

SECTION V I I • CORONARY ARTERY DI SEASE A N D CARD IAC CATHETERIZATION

therapy should be intensified (ACC/AHA Class I recommen­ dation) . 2 9 If the LDL-C is 70 to 100 mgldl at baseline, it is reasonable to treat to an LDL-C 60 min 1-2 min

Oral Oral

30-60 min 30-60 min

Oral

30-60 min

Usual Dose

0.3-0.6 mg/dose, up to 1 . 5 mg total dose as needed 0.4 mg, 1-2 sprays as needed, up to 3 doses 5 min apart 7. 5-40 mg, 6 x 6 inches or 1 5 x 1 5 cm 0.2-0 .8 mglh!d, removed at night for 1 2-h nitrate-free period 5-200 µg/min, titrated to symptom relief 5-80 mg 2-3 times daily 20 mg twice daily, 7-8 h apart 30-240 mg daily

SR, sustained release.

Commonly used nitrate preparations in stable angina are listed in Table 39.9. Sublingual nitroglycerin tablets and spray are effective preparations and are indicated for imme­ diate relief of angina (ACC/AHA Class I recommendation).6 Long-acting nitrate preparations, such as isosorbide mononi­ trate, isosorbide dinitrate, transdermal nitroglycerin patches, and nitroglycerin ointment, are indicated for initial therapy in stable CAD when beta-blockers are contraindicated or not tolerated or in combination with beta-blockers when beta­ blockers alone are ineffective (ACC/AHA Class I recommenda­ tion) .6 These long-acting agents should be administered so as to incorporate a nitrate-free interval of at least 8 hours to pre­ vent tolerance. Additionally, nitrates should not be used in the same 24-hour period as a type 5-cyclic guanosine monophos­ phate-dependent phosphodiesterase inhibitor (e.g. , sildenafil, tadalafil) , as this can lead to severe hypotension. 1 4 Co m b i nation Th era py

For many patients receiving treatment for stable angina pec­ toris, the symptoms persist despite monotherapy, illustrating the frequent need for combination pharmacotherapy Although not all published trials of combination therapy have demonstrated greater efficacy over monotherapy, meta­ analysis data suggest that the combination of a beta-blocker and CCB allows for greater exercise tolerance when com­ pared to either medication used alone.8 1 The combination of long-acting, second-generation vasoselective dihydropyri­ dine CCBs with beta-blockers appears to be a particularly effective antianginal regimen, as measured by indices of

angina, exercise tolerance, and nitroglycerin consump­ tion. 8 2 In the International Multicenter AnGina Exercise (IMAGE) study, the combination of metoprolol and nifedi­ pine improved ischemia and exercise tolerance over either drug alone. 68 Nitrates also improve symptoms when used in com­ bination with beta-blockers or CCBs, as mentioned previ­ ously 76-79 Both CCBs and long-acting nitrates are indicated in combination with beta-blockers for the initial treatment of stable angina when beta-blockers alone are ineffective (ACC/ AHA Class I recommendation) . 6 Extensive data with the combination of all three classes of antianginals are lacking. However, some analyses estimate that 5% to 1 5 % of patients are refractory to even triple antianginal therapy83 Ra n o lazi ne

Ranolazine is the first antianginal drug approved by the United States Food and Drug Administration (FDA) in more than 20 years and is used primarily in those patients refractory to traditional agents. 1 4 Ranolazine, a piperazine derivative, inhibits late sodium channels by lowering total inward sodium influx and thus the subsequent intracellu­ lar calcium overload that is associated with ischemia. For­ tunately, at therapeutic levels, ranolazine does not alter fast inward sodium channels; the late inward sodium channels are inhibited in ischemic tissue only 1 4 By preventing the intracellular calcium overload, there is myocardial diastolic relaxation and a rebalancing of oxygen demand and supply in the coronary vasculature.

562

SECTION VII • CORONARY ARTERY DI SEASE AND CARD IAC CATHETERIZATION

Because ranolazine is cleared by hepatic enzymes and is also a substrate of P-glycoprotein, there are a number of important drug interactions to be aware of. Any cytochrome P 3A4 inhibitor (e.g. , ketoconazole, clarithromycin, pheny­ toin) will raise the levels of ranolazine and can lead to side effects such as dizziness, headache, and nausea. 1 4 Ranola­ zine itself is a mild inhibitor of some cytochromes (CYP 3A4 and CYP 2D6) . This can lead to increases in levels of some statins and should be a concern in patients with stable CAD. Also, because of P-glycoprotein competition, ranolazine can lead to increases in digoxin levels in patients on this medi­ cation, so dose reduction or further monitoring is usually warranted. 1 4 Finally, ranolazine may prolong the QT inter­ val. Thus, patients with congenital long QT syndrome or who are on medications that prolong the QT interval should likely not receive this medication. In preliminary trials, ranolazine showed a significant improvement in exercise duration and ischemia in patients with stable angina. 84·85 In the first of the maj or clinical studies, three doses of ranolazine were assessed in stable angina patients previously responsive to traditional thera­ pies.86 Ranolazine resulted in improvement in symptoms of angina and exercise duration as monotherapy, with the optimal dose established at 1 ,000 mg twice daily86 The subsequent trial evaluated ranolazine in combination with a beta-blocker or CCB and again showed improvement in anginal symptoms and ischemia in the patients receiving the combination therapy87 In the Metabolic Efficiency with Ranolazine for Less Ischemia in Non-ST Segment Elevation Acute Coronary Syndromes (MERLIN)-TIMI 36 trial, there was no improvement in the primary endpoint of cardiovas­ cular death, MI , or recurrent ischemia between ranolazine and placebo in patients with non-ST-segment elevation acute coronary syndromes.88 Further analysis showed no impact on cardiovascular death and MI , but ranolazine did show an improvement in angina and duration of exercise.89 Currently, ranolazine use is not reflected in the ACC/AHA guidelines, as it was not FDA approved at the time of the last update.

Conclusion Although no single class of medical therapy directed at symptom relief has proven to be prognostically superior in the treatment of uncomplicated stable angina pectoris, beta­ blockers have been shown to reduce mortality in high-risk subsets of cardiovascular disease (prior MI, heart failure, hypertension) and therefore serve as first-line agents for symptomatic treatment. CCBs and long-acting nitrates are reserved for combination therapy in patients with persistent symptoms or as second-line agents in patients who are una­ ble to tolerate beta-blockers (see Table 39. 1 0) . I n addition t o symptomatic treatment, i t i s essential that individual patient risk factors be identified and treated. The greatest emphasis should be placed on the treatment of modifiable factors that have the greatest potential for pre­ venting disease progression and reducing the risk of future

ischemic events. This includes smoking cessation, physical activity in sedentary patients, weight management, anti­ platelet therapy with aspirin, and aggressive treatment of concomitant hyperlipidemia, hypertension, and diabetes mellitus. 2 9 Newer pharmacologic agents targeting metabolic pathways (e.g. , nicorandil, trimetazidine) and sinus rate­ lowering drugs (e.g. , ivabradine) have unique mechanisms of action that may provide additive benefits when combined with traditional therapies, but further investigation is still required before these medications receive FDA approval. R E VA S C U L A R I Z AT I O N I N S TA B L E A N G I N A P E CTO R I S In 2007, there were an estimated 408 ,000 coronary artery bypass surgeries, over 1 million diagnostic cardiac cath­ eterizations, and just under 1 .2 million inpatient PCis in the United States. 2 Even though the overall percentage has decreased in recent years, the maj ority of these were per­ formed electively in patients with stable ischemic syndromes. Despite the more widespread availability of mechanical revascularization in the management of stable angina, evi­ dence-based medicine suggests that patients with low-risk features are best managed medically, with revascularization reserved for those with refractory symptoms or high-risk clinical and angiographic features. As both medical and revascularization therapies continue to improve, identifying the patients most likely to derive sufficient symptomatic or survival benefit to warrant the immediate risk of an inva­ sive procedure and selecting the most appropriate mode for revascularization remain an important challenge.

Coronary Artery Bypass Grafting versus Medical Therapy The initial studies comparing medical therapy and CABG in stable coronary disease were performed prior to the advent of percutaneous therapies and before the routine use of antiplatelet and lipid-lowering pharmacotherapies. The three largest trials were the Veterans Administration Coop­ erative Study (VA Study) ,90·9 1 the Coronary Artery Surgery Study (CASS) ,92 and the European Coronary Surgery Study (ECSS) .93·94 In these trials, patients with significant CAD were variably defined angiographically Left main stenosis was generally considered significant if �50% in all studies. By contrast, in CASS, a stenosis of �70% in a maj or epicar­ dial coronary artery segment was considered significant,92 while in the others, it was �50% .9 1 ·93 Either way, patients were randomized to medical therapy alone or in combi­ nation with surgical revascularization. In all three trials, patients who underwent CABG had a marked improvement in anginal symptoms, exercise tolerance, and quality of life compared to medically treated patients. 12 Generally follow­ ing CABG, angina control is more prominent early in the postoperative period and decreases over time, with more than 90% of patients free of symptoms 1 year after surgery, 78% at 5 years, and 52% at 1 0 years.95 Accelerating vein

CHAPTER 3 9 • STABLE ANGINA

563

ACC/AHA Recommendations for Pharmacothera py i n Sta ble Angina Pectoris Pharmacotherapy to prevent sym ptoms Beta-blockers

Class I: indefinitely in all patients with prior MI, acute coronary syndrome, or LV dysfunction with or without heart failure symptoms, unless contraindicated Calcium channel blockers

Class I: in combination with beta-blockers when beta-blockers alone are ineffective Class I: initial therapy in patients where beta-blockers are contraindicated or in those unable to tolerate beta-blockers Class Ila: long-acting nondihydropyridine calcium antagonists instead of beta-blockers as initial therapy Nitrates

Class I: long-acting nitrates in combination with beta-blockers when beta-blockers alone are ineffective Class I: long-acting nitrates as initial therapy in patients where beta-blockers are contraindicated or in those unable to tolerate beta-blockers Class I: Sublingual nitroglycerin or nitroglycerin spray for immediate relief of angina Pharmacothera py to prevent death or M l Aspirin

Class I: indefinitely in all patients without contraindications, at a dose of 75-162 mg/d Clopidogrel

Class Ila: in patients for whom aspirin is absolutely contraindicated ACE inhibitors

Class I: indefinitely in patients with LV systolic dysfunction (EF � 40%), HTN, DM, or CKD, unless contraindicated Class Ila: in patients with mildly reduced or normal LV function (if risk factors are well controlled and the patient is revascularized) Angiotensin receptor blockers

Class I: in patients who have HTN , have indications for but are intolerant of ACE inhibitors, have heart failure, or have had MI with LV EF � 40% Class Ilb : in combination with ACE inhibitors in patients with heart failure due to LV systolic dysfunction Aldosterone receptor blockers

Class I: in post-MI patients without significant renal dysfunction (creatinine 80% proximal LAD coronary stenosis, and preserved left ventricular function to medical therapy, CABG using an IMA graft, or PTCA. 2 0 There was no difference in survival or MI between the treatment arms at 3 years. Angina was improved compared to medical therapy following either form of revascularization, although initial CABG provided greater relief and fewer repeat procedures than initial PTCA.

565

The trial out of Lausanne, Switzerland, which did not include a medical treatment arm, reported similar sur­ vival and symptomatic benefit at 5-year follow-up among 134 patients with a proximal LAD stenosis randomized to PTCA or bypass surgery with an IMA graft. 1 04 There were also more repeat revascularization procedures in the PTCA group . Lastly, a more recent trial comparing the more con­ temporary approach of minimally invasive CABG with IMA versus stenting for a proximal LAD stenosis also failed to detect a difference in death or MI. 1 05 There were more often recurrent symptoms and repeat interventions in the PCI group , while the CABG group more often had adverse events (i.e. , reoperation for graft occlusion, perioperative MI, stroke, chest wall hernia requiring surgical repair, etc.).

PCI versus CABG in M ultivessel CAD Multiple early randomized trials comparing initial PCI, con­ sisting mostly of PTCA, versus CABG in multivessel CAD have shown that, except for the subset of patients with diabe­ tes, the long-term risk of death or MI is equivalent with both procedures. Percutaneous revascularization, however, is con­ sistently associated with less anginal relief and the need for repeat revascularizations. The largest single study comparing these revascularization strategies for multivessel CAD was the Bypass Angioplasty Revascularization Investigation (BARI) . 1 06 In this study, 1 ,829 patients with multivessel CAD were ran­ domized to PTCA versus CABG and followed for an average of 5.4 years. In the periprocedural time period, there was no difference in in-hospital mortality or stroke rates between the two arms; however, CABG patients had a higher incidence of perioperative MI and PTCA patients had a higher likeli­ hood of early reintervention. Long-term follow-up evened out the rates of MI between the two treatment groups and there was no difference at 5 years. The trend for revascu­ larization held for the follow-up period, with PTCA patients requiring more repeat revascularization procedures (54% vs. 8% in the CABG arm) , most of which occurred within the first year of follow-up. Importantly, on subgroup analysis, as initially noted in this trial and confirmed in subsequent publications, diabetic patients with multivessel disease have a significant survival benefit with surgery over PTCA (5-year survival 80.6% with CABG vs. 65.5% with PTCA, p 0.003). The survival outcomes for nondiabetics were identical. A meta-analysis published in 1 995 combined the results of the available eight randomized trials at that time, which enrolled 3 , 3 7 1 total patients with a mean follow-up of 2. 7 years. 1 07 There was no detected difference in mortality or MI, but the analysis did confirm that patients treated with PTCA experienced less complete relief of anginal symptoms and required more repeat revascularizations (3 .3% CABG vs. 33. 7% PTCA) . In contrast, a later meta-analysis of 1 3 trials, including 7,964 patients and 4 trials in which stents were used as the initial PCI, demonstrated a significant sur­ vival advantage favoring CABG over PCI at 5 years but, with longer follow-up out to 8 years, was no longer significant. 1 08 Subgroup analyses suggested that the mortality reduction =

566

SECTION VII • CORONARY ARTERY DI SEASE AND CARD IAC CATHETERIZATION

was limited to diabetics, while nondiabetics again had equiv­ alent outcomes. Anginal symptoms and repeat revasculariza­ tions were again significantly reduced following CABG, but the difference was markedly attenuated in patients receiving coronary stents (repeat revascularization rates of PCI group cut in half by the use of stents) rn8 The early trials of PTCA versus CABG had important limitations. For the most part, the patient populations were younger and at lower risk. Although stenting was initially introduced for the management of complications related to PTCA and was not used in the early trials, it is now the dom­ inant PCI modality because it significantly reduces resteno­ sis compared with PTCA. There were four subsequent trials that evaluated PCI incorporating BMS versus CABG in mul­ tivessel CAD . 1 8. i o9-rn In the meta-analysis of the aggregate data from these 4 trials of 3 ,05 1 patients with multivessel CAD, there was no difference in the combined endpoint of death, Ml, or stroke at 1 year between the groups. 1 12 Once again, CABG was superior in need for repeat revasculariza­ tion procedures (4.4% vs. 18% in PCI patients) ; however, for the first time, PCI was better in relieving anginal symptoms, with 82 % freedom from angina versus 77% in the CABG group (p 0.002) . Even though the need for repeat revascu­ larization was still higher for percutaneous procedures, this meta-analysis showed that the gap had narrowed consider­ ably with the use of BMS. 112 In 2009 , the results of the Bypass Angioplasty Revas­ cularization Investigation 2 Diabetes (BARI-2D) trial were published and contradicted the results of earlier trials in regard to the treatment of diabetic patients n 3 BARI-2D randomized 2 ,368 patients with diabetes and stable CAD to medical therapy or prompt revascularization, either by CABG or PCI. The decision of CABG or PCI, however, was at the discretion of the treating physician. At 5 years, there was no difference in mortality between medical therapy and revascularization (survival 88 .3% in revascularization group vs. 87.8% in medical therapy group) . At baseline, the patients stratified to CABG over PCI had more extensive CAD , with more three-vessel CAD , proximal LAD disease, and chronic total occlusions. In spite of this, there was no difference in mortality between either the CABG or PCI arms and the medical therapy arm. While there was no difference in maj or cardiovascular events (death, MI, or stroke) in PCI versus medical therapy, there was significant improvement in maj or cardiovascular events in the CABG group over medical therapy alone. When the interaction between study group assignments was evaluated, there was a statistically significant benefit in prompt revascularization, over medical therapy, in patients selected for CABG over those stratified to PCI. There are some limitations to conclusions drawn from these data. First, this study was not designed to evaluate PCI versus CABG in diabetic patients; as mentioned previously, the patients stratified for CABG had more extensive CAD . Additionally, the use of drug-eluting stents (DES) was low (around 35%) in the PCI arm, as was the use of antiplatelet agents (around 20%). The best revascularization strategy in diabetic patients is, therefore, still an area of some debate. =

The ongoing Future REvascularization Evaluation in patients with Diabetes mellitus: Optimal management of Multivessel disease (FREEDOM) trial will help answer this question. 11 4 This study is an open-label, prospective randomized trial of PCI with DES versus CABG in diabetic patients in whom revascularization is indicated. Also published in 2009 were the results of the Syn­ ergy between Percutaneous Coronary Intervention with Taxus and Cardiac Surgery (SYNTAX) trial. 11 5 In this study, 1 ,800 patients with three-vessel or left main CAD were ran­ domized to CABG or PCI with Taxus DES. At 1 year, the rates of death and MI were similar between the two groups, while stroke was more likely to occur with CABG (2 .2% vs. 0.6% with PCI) and an increased rate of repeat revascu­ larization was more likely with PCI ( 1 3 . 5 % vs. 5 . 9 % with CABG) . Further, the investigators used an angiographic scoring tool (the SYNTAX score) to obj ectify the severity of CAD . Stratified by SYNTAX score, patients with low or intermediate scores had similar rates of maj or adverse car­ diac or cerebrovascular events whether undergoing PCI or CABG. However, in those with high SYNTAX scores, the CABG group had much lower rates of maj or adverse cardiac or cerebrovascular events (all-cause mortality, stroke, MI, or repeat revascularization) . 11 5

Conclusion Data from randomized trials and observational registries indicate that the benefits of revascularization in stable coro­ nary syndromes are proportional to the patient's estimated long-term risk while on medical therapy. 1 07• 1 08 For low-risk patients with single-vessel CAD , medical therapy remains the initial treatment of choice, with revascularization reserved for symptom relief when medical treatment has failed. Patients with multivessel CAD are more complicated. Surgi­ cal revascularization likely provides the best long-term sur­ vival benefit for diabetics with multivessel disease and for all patients with high-risk angiographic features, such as severe left main stenosis, three-vessel disease, or two-vessel disease involving the proximal LAD . For the remaining patients with moderate-risk multivessel disease, revascularization and medical therapy appear to provide similar outcomes. For a complete listing of the ACC/AHA recommendations for revascularization in stable angina, see Table 39 . 1 1 . R E F RACTO RY A N G I N A Considerable progress has been made over the last 25 years in expanding the therapeutic options available in ischemic heart disease, including pharmacologic and revasculariza­ tion therapies that improve both symptoms and progno­ sis. However, despite the efficacy of these treatments, there remains a subset of patients with severe symptoms who are refractory to conventional medical therapy and are deemed to be unsuitable for coronary revascularization. As many as 1 . 7 million patients in the United States are suffering from refractory angina pectoris, with the prevalence increasing as the population ages and patients live longer with their

CHAPTER 3 9 • STABLE ANGINA

567

ACC/AHA Recommendations for Revascula rization i n Sta ble Angina Pectoris Class I

CABG for >50% left main trunk stenosis CABG for three-vessel disease, especially if abnormal LV function or diabetes CABG for two-vessel disease with significant proximal LAD CAD and abnormal LV function (EF < 50%) or ischemia on noninvasive testing PCI for two- or three-vessel CAD with significant proximal LAD CAD who have anatomy suitable for PCI and have normal LV function and are not diabetic PCI or CABG for one- or two-vessel CAD without significant proximal LAD CAD but with a large area of viable myocar­ dium and high-risk criteria on noninvasive testing CABG for one- or two-vessel CAD without significant proximal LAD CAD in patients who have survived sudden cardiac death or sustained VT CABG or PCI for restenosis in patients with prior PCI that is associated with a large area of viable myocardium and/or high-risk criteria on noninvasive testing PCI or CABG for persistent symptoms despite OMT Class I l a

Repeat CABG for multiple SVG stenoses, especially if/when significant stenosis involves a graft to LAD . PCI may be appropriate in focal stenoses or in poor candidates for reoperation. PCI or CABG for one- or two-vessel CAD without significant proximal LAD disease but with a moderate area of viable myocardium and ischemia on noninvasive testing PCI or CABG for one-vessel disease with significant proximal LAD disease Class l l b

PCI (over CABG) for two- or three-vessel CAD with significant proximal LAD CAD in patients with anatomy suitable for PCI and who are diabetic or have abnormal LV function PCI for left main disease in patients who are not operative candidates PCI for one- or two-vessel CAD without significant proximal LAD CAD in patients who have survived sudden cardiac death or sustained VT Class I l l

PCI or CABG for one- or two-vessel CAD without significant proximal LAD CAD in patients who have mild symp­ toms that are unlikely due to myocardial ischemia, or in patients who have not received an adequate trial of medical therapy and have only a small area of viable myocardium or have no ischemia on noninvasive testing PCI or CABG for borderline coronary stenoses (50%-60% in locations other than the left main trunk) and no ischemia on noninvasive testing PCI or CABG for insignificant coronary stenosis (65 years old, and nearly 50% occur in women. When women present with chest pain, the etiology is less likely to be secondary to obstructive coronary disease, and when coronary disease is present, it tends to be less severe than in men. In-hospital mortality of UA and NSTEMI patients is less than that of STEMI patients, although because the former are at risk for recurrent events, their long-term risk is equivalent or worse compared to STEMI patients. C L I N I C A L P R E S E N TAT I O N UA and NSTEMI patients typically present with substernal chest discomfort, described as a pressure or a heavy sensa­ tion. This more accurately describes angina than terminol­ ogy such as "pain." Symptoms typically last 1 0 mg/L is considered an acute-phase response and should be repeated in 3 weeks. An elevated CRP predicts future cardiac events better than elevated cholesterol or pres­ ence of the metabolic syndrome. An elevated troponin I or T also carries independent prognostic information. A meta-analysis showed that

577

troponin-positive ACS patients have a fourfold increased risk for death compared to troponin-negative patients . Similarly, an analysis of the thrombolysis in myocar­ dial infarction (TIMI) Illb trial documented an eightfold increased risk of death at 42 days for patients with an elevated troponin I (>9 nglml) compared to troponin­ negative patients . The 42-day mortality in patients with an elevated troponin was 7 . 5 % , compared to 1 % in those with a negative troponin. An elevated brain natriuretic peptide (BNP) predicts increased risk for adverse cardiac events across the spec­ trum of ACS , although the predictive effect is greatest for UA and NSTEMI. BNP is a cardiac neurohormone synthesized in the ventricles and released as a larger peptide which is then cleaved into smaller portions including BNP and inac­ tive N-terminal proBNP peptide (NT-proBNP) . The release of BNP reflects the decompensated state of the ventricles, and it causes vasodilatation, natriuresis, and diuresis, lead­ ing to some improvement of the loading conditions of the failing heart. Even though BNP is the active hormone, both forms can be measured and serve as markers of congestive heart failure (CHF) . The combination of multiple biomarkers has incre­ mental value. An hs-CRP combined with cardiac biomark­ ers (i.e. , troponin I or T) and markers of pressure/volume overload (i. e . , BNP) predict an increased risk for maj or cardiac events. In the OPUS-TIMI 1 6 trial there was a six­ fold increased risk for 30-day cardiac events when all three markers were elevated. Similarly, in the TACTICS-TIMI 1 8 trial there was a 1 3-fold increased risk in 30-day cardiac events. So hs-CRP, troponin I or T, and BNP provide prog­ nostic information in ACS patients. Other inflammatory markers such as CD-40 ligand are experimental, although they may have a role in the future in predicting the overall risk for cardiac events in ACS.

Risk Scores The TIMI risk score incorporates data derived from the TIMI l lB trial and has been validated by three additional trials. The TIMI risk score is an easily used model that has important prognostic and therapeutic implications. It incor­ porates seven variables that are readily available from the history, ECG, and cardiac biomarkers (Fig. 40. 1). The pres­ ence of six or seven risk factors predicts a 40% incidence of death, MI, or ischemia requiring repeat revascularization by 30 days. This is in contrast to zero or one risk factor, where the 30-day cardiac event rate is 50%, ST deviation (transient ST elevations, ST depres­ sions, or T-wave inversions), :2:2 anginal events in the past 24 hours, aspirin use in the last 7 days, and elevated cardiac biomarkers (i.e . , elevated CK-MB or troponin) .

578

S E C T I O N V I I • CORONARY ARTERY DI SEASE A N D CARD IAC CATHETERIZATION 45

Hi t o ry and Presen t a t i o n Age 2: 65 2: 3 CAD risk factor oronary stenosis 2: 5 0%

Poi n t s I

Aspirin use in t he past w e ek

2 angina! events i n the last day Elevated cardiac biomarkers

2:

....

cu () rn ell >

30

*

rusk factor are d efi n e d

as

premature CAD, HT , D M , hyper l i pi de m i a, & active moking

J schemic ECG

Q) ex: :::::

� -.



.r::.

+-'

0

15

0

0/1

2

Total T I M I R is k Score

5

6/7

FIGURE 40.1 TIMI risk model for prediction of short-term adverse cardiac events in UA/NSTEM I patients. (Ada pted from Antman EM, Cohen M, Bern ink PJ, et al. The TI M I risk score for unstable angi na/non-ST elevation M l : a method for prognostication and thera peutic decision making.JAMA. 2000;284:835-842.)

Another risk score which has been utilized exten­ sively is the global registry of acute coronary events (GRACE) score . This score is the composite of nine variables , which, when added together, can be plotted on a nomogram to determine mortality risk from dis­ charge to 6 months . The variables included in the score include age , history of CHF, history of MI , heart rate and blood pressure on presentation, presence of ST-segment depression on initial ECG, serum creatinine and elevated cardiac biomarkers during hospitalization, and no per­ cutaneous coronary intervention (PCI) performed during hospitalization. With either scoring system in patients presenting with UA/NSTEMI, there is progressively greater benefit of more aggressive therapies as the risk score rises. M A N AG E M E N T

I n itial Approach The initial assessment of UA/NSTEMI coronary syndromes includes establishing intravenous access and starting sup­ plemental oxygen in patients who are hypoxic or who show signs of respiratory distress. Simultaneously, an ECG must be interpreted, a targeted history and physical exam taken, and cardiac biomarkers measured. Preferred cardiac bio­ markers include troponin I or T and CK-MB . Total CK (with­ out MB) should not be used to evaluate an ACS (class III recommendation) .

The primary management focus during an ACS , while antithrombotic and anti-ischemic medicines are adminis­ tered (Table 40 .2), is to determine a patient's suitability for early invasive therapy versus conservative therapy. While fibrinolytic therapy plays an invaluable role in STEMI patients, it should not be used for the management of UA/NSTEMI unstable coronary syndromes (class III recom­ mendation) .

I nvasive Therapy Early trials failed to show a benefit from an invasive approach in UA/NSTEMI patients. A meta-analysis performed in the current PCI era analyzed all available studies that rand­ omized patients to early invasive therapy versus conservative management. In those studies, patients who were treated conservatively could have an angiogram performed if they had recurrent chest pain, ischemic ECG changes, a large reversible defect with noninvasive stress testing, or elevated cardiac biomarkers. Only contemporary trials that used gly­ coprotein (GP) Ilb/llla inhibitors and intracoronary stents were included. Five studies, involving nearly 7,000 UN NSTEMI patients, were analyzed. This analysis revealed a 6- to 12-month survival advantage from early invasive ther­ apy compared to conservative management (RR = 0.80, 95% CI 0 . 63 to 1 .03) . In contrast, studies that enrolled patients before the routine use of stents and GP Ilb/llla inhibitors revealed a harmful association from early invasive therapy (RR 1 . 3 1 , 95% CI 0.98 to 1 . 75).

CHAPTER 40 • UNSTABLE CO RONARY SYNDROMES

579

TA B L E

Class I Anti-ischemic Recom mendations

Bed rest with continuous ECG monitoring Supplemental oxygen to keep Sa0 2 > 90% NTG (SL X 3 , IV for ongoing ischemia, heart failure, hypertension) Oral beta-blockers within 24 h for patients without HF, low-output state, risk of cardiogenic shock, markedly prolonged PR interval, second- or third-degree heart block, or severe asthmaa,b IABP for refractory ischemia or hemodynamic instability ACE-I in first 24 h for heart failure or LV dysfunction (EF < 40%) after Ml' ARBs for those with ACE-I intolerance Discontinuation of nonsteroidal anti-inflammatory drugs (NSAIDs) Aspirin (continued indefinitely)d Clopidogrel (loading followed by maintenance dose)' The listed therapies are indicated for ongoing ischemia. 'Intravenous beta-blockers class Ila. bNondihydropyridine calcium channel blockers may be used when beta-blockers are not successful, or there is a contraindication to their use . 'ACE-I are continued when ischemia is controlled, especially for LV dysfunction or diabetes. dClopidogrel can be substituted in patients who cannot take aspirin due to hypersensitivity or maj or gastrointestinal intolerance. 'Prasugrel can be substituted if planning for PC! with risk of bleeding is low and CABG considered unlikely--class Ilb recommendation. From Anderson jl, Adams CD , Antman EM, et al. ACC/AHA 2007 guidelines for the management of patients with unstable angina/non-ST-elevation myocardial infarction: a report of the American College of Cardiology/American Heart Association Task Force on Practice Guidelines (Writing Committee to Revise the 2002 Guidelines for the Management of Patients With Unstable Angina/Non-ST-Elevation Myocardial Infarction) developed in collaboration with the American College of Emergency Physicians, the Society for Cardiovascular Angiography and Interventions, and the Society of Thoracic Surgeons endorsed by the American Association of Cardiovascular and Pulmonary Rehabilitation and the Society for Academic Emergency Medicine . ] Am Coll Cardiol. 2007;50(7) : e l-e l 5 7 , with permission from Elsevier.

The most contemporary large-scale data on this topic derive from the Invasive versus Conservative Treatment in Unstable coronary Syndromes (ICTUS) trial, which ran­ domized 1 ,200 UNNSTEMI patients to routine invasive or selective invasive management. Patients in the selective invasive arm were treated medically and only in cases of refractory angina or a positive exercise stress test under­ went coronary angiography with or without revasculariza­ tion. Results at the end of 1 year and after 3 years follow-up showed there was no significant difference in the compos­ ite ischemic end point. The investigators postulate that due to the high rate of revascularization in the selective inva­ sive therapy arm (4 7%), use of aggressive medical therapy in both arms (including routine use of clopidogrel in the conservative arm) and low event rate, there was little incre­ mental benefit to be observed with an early invasive strategy Given the results of ICTUS, the ACC/AHA guidelines recog­ nize that an initially conservative (selective invasive) strategy may be considered as a treatment option in stabilized UN NSTEMI patients. Earlier trials comparing early invasive versus conserva­ tive management include third randomized intervention treatment of angina (RITA-3) and FRagmin and fast revas­ cularization during InStability in Coronary artery disease (FRISC II) . In the RITA-3 trial, 1 ,8 1 0 UNNSTEMI patients

were randomized to interventional versus conservative treat­ ment. Like ICTUS, at 1 year, death and MI rates were similar, but at 5 years, a significant reduction in death or MI emerged in the early invasive treatment arm. Benefits were seen mainly in high-risk patients. Similarly, an invasive strategy was favored at 5 years in the FRISC II trial for the primary end point of death or nonfatal MI (HR 0 .8 1 , p 0.009). Here, the benefit was confined to males, nonsmokers, and patients with two or more cardiac risk factors. A meta-analysis of seven randomized trials of manage­ ment strategies in UNNSTEMI, including ICTUS, supports the long-term benefit of an early invasive strategy Among 8,375 patients, the incidence of all-cause mortality at 2 years was 4.9% in the early invasive group compared with 6 .5 % in the conservative groups (RR 0. 75, 95% CI 0 . 63 to 0 .9 0 , p 0 . 00 1), while also showing a significant reduction in nonfatal MI and hospitalization Data from the ABOARD , TIMACS, and ISAR-COOL studies helped to determine the optimal timing of the inva­ sive strategy These three trials, taken together with earlier studies, do provide support for a strategy of early angiog­ raphy and intervention to reduce ischemic complications in patients who have been selected for an initial invasive strategy, particularly among those at high risk (defined by GRACE score > 1 40) , whereas a more delayed approach is =

=

580

SECTION VII • CORONARY ARTERY DI SEASE AND CARD IAC CATHETERIZATION

reasonable in low- to intermediate-risk patients. The "early" time period in this context is considered to be within the first 24 hours after hospital presentation, although there is no evidence that incremental benefit is derived by angiography and intervention performed within the first few hours of hospital admission. The advantage of early intervention was achieved in the context of intensive background antithrom­ botic therapy: Therefore, current ACC/AHA guidelines recommend (class I) an early invasive approach to patients with angina in the presence of heart failure symptoms (pulmonary edema, an S3 gallop , or new mitral regurgitation) , known left ventric­ ular dysfunction, hemodynamic instability, positive noninva­ sive stress test (large area of ischemia) , sustained ventricular tachycardia, or prior revascularization (prior coronary artery bypass grafting [ CABG] , or PCI within the last 6 months) . The updated guidelines additionally recommend that individuals with rest angina despite intensive anti-ischemic therapy or with new ST depressions or elevated cardiac biomarkers be directed to early invasive therapy: Routine invasive therapy is discouraged in low-risk patients and those with extensive comorbidities (class III recommendation) . Intermediate-risk patients can initially be treated by either an early invasive or a conservative approach with careful monitoring for the development of high-risk features. High­ risk features include refractory pain, angina with dynamic ECG changes, or elevated cardiac biomarkers. Such a change in clinical status should advance therapy to a more invasive approach along with adjunctive GP Ilb/Illa inhibitor use. Low-risk patients can often be treated as outpatients or screened for MI with serial cardiac enzymes in a chest pain unit with a goal of early discharge. Invasive therapy is discour­ aged in these patients. Risk-factor modification is emphasized to all patients regardless of their risk at presentation. Once the decision is made to perform coronary angi­ ography, the patient's suitability for coronary revasculariza­ tion is determined. Two options for revascularization are PCI (i.e. , percutaneous transluminal coronary angioplasty [PTCA] and intracoronary stents) or CABG. The choice of which revascularization to perform is beyond the scope of this chapter, although several general guidelines exist. Severe left main trunk disease is usually an indication for CABG, although left main PCI can be performed in select cases (i.e. , the patient is not a candidate for open heart surgery) . Severe three-vessel disease or severe two-vessel disease involving the left anterior descending artery: along with left ventricular dysfunction or diabetes, also favor CABG (Fig. 40.2).

Anti platelet Agents As pirin

Aspirin is the cornerstone of treatment for all unstable coro­ nary syndromes unless there is a serious contraindication to its use (class I recommendation) . Aspirin blocks the conversion of arachidonic acid to thromboxane A2 by irreversibly acety­ lating cyclooxygenase (Fig. 40.3). Full-dose aspirin exerts

maximal antiplatelet effects within 30 minutes of absorption; therefore an initial 325 mg of aspirin orally (or by rectal sup­ pository if necessary) is given during an ACS (see Table 40.2). Low-dose aspirin (75 to 150 mg daily) is effective in primary prevention by reducing the incidence of MI. Optimal dosing of aspirin has been debated for some time, and the recently published CURRENT-OASIS 7 trial demonstrated no dif­ ference in outcome between patients treated with low-dose and high-dose aspirin, in all patients and in the PCI sub­ group . This indicates that either dose can be used provided a 300-mg loading dose is used before initiation of a mainte­ nance dose. There was an increase in minor bleeding events with high-dose aspirin, and therefore lower doses of aspirin should be used unless there is concern for recurrent ischemia. In secondary prevention, aspirin improves survival. Bleed­ ing complications increase with increasing dosage; therefore ongoing aspirin therapy is typically an 8 1 -mg tablet daily unless there is a compelling reason for using a higher dosage, such as mitigating the cutaneous side effects of niacin. Th ienopyri d i nes a n d ADP I n h i bitors

Despite aspirin's proven benefit in reducing Ml and death, it does not fully block platelet aggregation, especially when aggregation is induced by adenosine diphosphate (ADP) . Other agents such as thienopyridines play a valuable role in the management of UNNSTEMI patients because they complement the actions of aspirin. Thienopyridines are rep­ resented by ticlopidine, clopidogrel, and prasugrel. These agents act by inhibiting ADP receptor-mediated platelet activation. Ticagrelor has also recently been approved for use in the United States. In contrast to the other thienopyridines, ticagrelor has a binding site different from ADP, making it an allosteric antagonist, and the blockage is reversible. Newer agents, cangrelor and elinogrel, which are direct inhibitors of the ADP receptor P2Y l 2 , are also on the horizon. Thieno­ pyridines may be used alone if a patient has a hypersensi­ tivity to aspirin, but they are ideally used adjunctively with aspirin. Clopidogrel is preferred over ticlopidine, given its lower incidence of neutropenia and thrombocytopenia and its rapid onset of action. A loading dose of 300 to 600 mg of clopidogrel produces maximal antiplatelet effects in 4 to 6 hours. The important studies that document the benefit of clopidogrel in unstable coronary syndromes are the CAPRIE and CURE trials. The CAPRIE trial randomized over 1 9 ,000 vascular disease patients to receive aspirin or clopidogrel. Vascular disease was manifested as a history of ischemic stroke, Ml, or symptomatic peripheral arterial disease. After nearly 2 years of follow-up , clopidogrel reduced a composite end­ point of ischemic stroke, MI , or death from vascular causes (5 . 3 % vs. 5 . 8 % , p 0 . 04) . The CURE trial tested protection from ischemic events beyond that of aspirin by the addition of clopidogrel to aspirin and standard medical therapy in patients with ACS . This trial randomized over 1 2 ,000 non-ST-elevation ACS patients within 24 hours of their onset of chest pain =

CHAPTER 40 • UNSTABLE CO RONARY SYNDROMES

581

10 9

9.0

8

c 111

Q) �

(0 a..

0 Cl>

-

s

O>

� c:

GI a..

7.3

7

5.7

6

5

5.8 4. 1

4

3

2.2

2

0 ..._

_,____

_

OASI S 5 Death. M l , ()( ref ctory schemL& al 9 days

OASIS

5

Absol ute Risk Red uction

95% C I

Hazard Ratio

p

(424)

OASIS 6

-0. 1

1 .01

0.90 to 1 . 1 3 o.oor

or bleedlng at 9 days

1 .9

0.52

0.44 to 0.61 less than 0.001 t

OASIS 6 Compo outoom aod major days

pnm ry

bleedlog al 9 le

1 .7

0.81 0.73 to 0.89 less than 0.00 1 t

FIGURE 40.2 Revascu l a rization strategy i n UA/N STEMI. 'There i s confl icti ng information about these patients. Most consider CABG to be preferable to PCI. CABG, corona ry a rtery bypass g raft; LAD, left a nterior descending coronary a rtery; PCI, percuta neous coronary i ntervention; UA/NSTEMI, u n sta ble a n g i na/non-ST-elevation myoca rd ial infa rction. (From Anderson J L, Ada m s CD, Antman EM, et a l . ACC/AHA 2007 g u ideli nes for the ma nagement o f patients with u n stable a n g i na/non-ST-Elevation myoca rd ial i nfa rction: a report of the America n College of Cardiology/American Heart Association Task Force on Practice Guidelines (Writing Comm ittee to Revise the 2002 Guidelines for the Management of Patients With U n sta ble Angina/Non-ST-Elevation Myoca rd ial I nfa rction) developed in colla boration with the America n Col lege of Emergency Physicians, the Society for Cardiovascu lar Angiog ra phy and Interventions, and the Society of Thoracic Surgeons endorsed by the America n Association of Ca rd io­ vascu lar a n d Pulmonary Reha b i l itation and the Society for Academic Emergency Medicine.J Am Coll Cardiol. 2007;50(7) :e 1 -e 1 57, with permission from El sevier.)

to clopidogrel plus aspirin versus aspirin alone. Patients were eligible if they had ischemic ECG changes or elevated cardiac biomarkers. Individuals who were randomized to clopidogrel received a loading dose of 300 mg, followed by 75 mg/d for 3 to 12 months. Aspirin could be given at a dose of 75 to 325 mg/d. This was largely a conservatively treated population, as 1 00 bpm

Killip C lass I I -IV

20

Poi n ts

.l -

Weight < 67 kg

Anterior ST-elevation or Lllllll

2

Time to Rx > 4 hrs

10

Total TIMI Ri s k Score

M A N AG E M E N T

I n itial Approach The initial assessment of a patient suspected of having an AMI is to establish intravenous access and start supplemen­ tal oxygen for individuals who are hypoxic or who show signs or respiratory distress. Simultaneously, a targeted his­ tory and physical exam should be obtained. The history and physical exam provide prognostic information, but also can suggest an alternative diagnosis and help identify mechani­ cal complications of STEMI. It is important to rule out other causes of chest pain such as aortic dissection. Pain arising from a gall stone, renal stone, pancreatitis, esophageal dys­ motility, pneumothorax, pleuritis as well as impending her­ pes zoster may frequently mimic this presentation. If reperfusion with fibrinolysis is considered, the history and physical exam should screen for contraindications to its use. Because the most feared complication with the use of fibrinolytics is intracranial hemorrhage (ICH) , patients with an increased risk for this complication must be identi­ fied. Risk factors for ICH are advanced age, female gender, uncontrolled hypertension, and low body weight. Patients with coagulopathies (e.g. , patients on Coumadin therapy) are also at increased risk for bleeding. Absolute and relative contraindications to fibrinolysis are listed in Table 4 1 . 4. Reperfusion Therapy Time is of paramount importance in reinstituting coronary flow. The greatest improvement on mortality comes from reperfusion within the first hour, the so-called golden hour. Reperfusion therapy can be considered up to 1 2 hours from the onset of chest pain and even longer in select cases. In order to facilitate rapid coronary reperfusion, a pharmaco­ logic or mechanical approach should be decided on quickly The current goal for door-to-lytic time is 30 minutes,

TA B L E

Contraind ications to Fibrinolysis Absol ute Contraindications

Any prior intracranial hemorrhage Known intracranial neoplasm Active bleeding Suspected aortic dissection Known structural cerebral vascular lesion Ischemic stroke within the past 3 months Severe closed head or facial trauma within the past 3 months Relative Contraind ications

Severe hypertension on presentation (blood pressure > 1 80/1 1 0) History of chronic, severe, poorly controlled hypertension History of ischemic stroke greater than 3 months before presentation, dementia, or other intracranial pathol­ ogy not listed as an absolute contraindication Traumatic or prolonged CPR (greater than 1 0 minutes) Use of anticoagulation Recent internal bleeding (within 2-4 weeks) Pregnancy Active peptic ulcer disease Vascular punctures at noncompressible sites Adapted from Antman EM, Anbe DT, Armstrong PW, et al. ACC/AHA guidelines for the management of patients with ST-elevation myocar­ dial infarction-executive summary. A report of the American College of Cardiology/American Heart Association Task Force on Practice Guidelines (Writing Committee to revise the 1 9 9 9 guidelines for the management of patients with acute myocardial infarction) ] Am Coll Cardiol. 2004;44 : 6 7 1-7 1 9 .

CHAPTER 4 1 • ACUTE MYO CARD IAL INFARCTION whereas the goal for door-to-balloon time is 90 minutes if the patient presents to a PCI capable facility and 120 minutes if the patient requires transfer for PCI. In general, if primary PCI can be performed immedi­ ately (i. e . , the patient presents to a center capable of perform­ ing PCI) , this is the preferred choice for reperfusion (class I recommendation) . This information comes from a meta­ analysis of 23 trials that randomized nearly 8,000 STEMI patients to fibrinolytic therapy versus primary PCI. The hospitals included in the analysis were largely experienced providers of coronary intervention and were able to deliver mechanical reperfusion in a timely fashion, although some studies enrolled patients who were transferred for primary PCI versus given immediate fibrinolysis. This study was a contemporary analysis , as stents were used in 1 2 of the trials and glycoprotein (GP) Ilb/Illa inhibitors were used in eight. General inclusion criteria required that patients have ischemic symptoms within the previous 6 to 1 2 hours and at least 1 -mm ST elevations in contiguous leads or a new/presumable new complete LBBB. Patients also needed to be candidates for fibrinolysis to be eligible for enroll­ ment. A notable exception was the SHOCK trial, as this study enrolled patients with cardiogenic shock and chest pain within the preceding 36 hours. Since the SHOCK trial was the outlier to the overall analysis, the analysis was per­ formed with and without this study: Most patients (76%) received fibrin-specific (i.e . , t-PA) lytic agents, whereas the remainder received streptokinase. This analysis revealed a short-term survival advantage as well as a reduction in recurrent MI and hemorrhagic stroke in those who received primary PCI. Short-term mortality was 7% in the primary PCI group, compared to 9% in the fibrinolytic group (p 0. 0002) . Long-term mortality was also significantly reduced (p 0 . 00 1 9) . Thus, among patients who present within 12 hours of the onset of chest pain to a tertiary care center that is capable of performing primary PCI expedi­ tiously, data supports the use of mechanical reperfusion. When individuals present to a community hospital without primary PCI capabilities, the question becomes whether to transfer the patient to a primary PCI center or to administer immediate fibrinolysis. Fibrinolysis is limited by postlysis TIMI 3 flow of 70 years, heart rate > 1 00 beats/min (bpm) , systolic blood pressure < 1 00 mm Hg, Killip class II/III , com­ plete LBBB, or anterior MI. Although this was the smallest study included in the meta-analysis, there was no noticeable harm in transferring high risk patients for PCI. The CAPTIM trial was unique in that patients were randomized to a reper­ fusion strategy before arrival to the hospital, which enabled fibrinolytics to be given in an even more timely fashion. This was the only trial that showed a nonsignificant trend in mor­ tality favoring fibrinolysis. The PRAGUE-2 trial examined the optimal reperfusion strategy based on time from the onset of chest pain. The study was stopped prematurely, as mor­ tality was increased 2 .5-fold among patients who presented more than 3 hours from the onset of chest pain who received fibrinolysis ( 1 5 % with fibrinolysis and 6% with primary PCI, p < 0.02) . In Patients who presented within 3 hours from the onset of chest pain, mortality was similar between the two reperfusion strategies (7.4% with fibrinolysis and 7.3% with primary PCI) . So, while transferring a STEMI patient for primary PCI versus immediately administering fibrinolysis is con­ troversial, some patient characteristics and logistical considerations favor one approach over another. Accord­ ing to the 20 1 1 ACCF/AHA/SCAI guidelines for PCI, if a patient presents with STEMI and can undergo PCI within 1 20 minutes of first medical contact, this is the preferred approach. Conversely, if the patient cannot receive PCI within 1 20 minutes of first medical contact, and there are no contraindications to fibrinolysis , fibrinolytics should be administered within 30 minutes of hospital presentation (class I recommendation) . It is important to note that the effectiveness of fibrinolytics are highly time dependent with a marked efficacy when administered in the first hour fol­ lowing STEMI onset. Accordingly, for patients who are at high risk for bleeding or who present more than 3 hours after the onset of chest pain, transfer for primary PCI is favored. Additionally, patients who are in cardiogenic shock benefit from mechanical revascularization, but have not been shown to have a mortality reduction with fibrinolysis. Lastly, in the elderly who have an increased risk of ICH, patients with contraindications to fibrinolytics or when the diagnosis of STEMI is in doubt, PCI should be considered. When interhospital transfer for primary PCI is planned, the

596

SECTION VII • CORONARY ARTERY DI SEASE AND CARD IAC CATHETERIZATION

DIDO (door in to door out) time at the originating hospital is an important performance measure and should ideally be 40,000 patients

with AMI and revealed the superiority of accelerated tPA over streptokinase. Accelerated tPA with intravenous heparin resulted in a 14% reduction in mortality and higher rates of TIMI 3 flow at 90 minutes (54% vs. 3 1 %) compared to streptokinase-based regimens. The accelerated tPA dose is a 1 5-mg bolus, then 0 . 75 mg/kg (up to 50 mg) over 30 min­ utes, followed by 0. 5 mg/kg over 60 minutes (up to 35 mg) . Reteplase is less fibrin-specific than alteplase. This agent is equivalent to alteplase in terms of efficacy, although it is easier to administer (two 1 0-mg boluses administered 30 minutes apart). Tenecteplase is the easiest lytic to administer, because it is given as a single bolus (dose ranges from 30 to 50 mg, adjusted for body weight) . See Table 4 1 . 5 for dosing. This agent is more fibrin specific and has a slower plasma clearance than the other fibrin-specific agents. The ASSENT 2 trials showed the noninferiority of tenecteplase compared to alteplase. In this trial, there was also less major bleeding with tenecteplase, and a trend toward less ICH in elderly women. Equivalent efficacy, enhanced safety, and ease of administra­ tion make tenecteplase an attractive fibrinolytic agent.

Percutaneous Coronary I ntervention When PCI is selected for reperfusion, eligibility criteria are the same as those used for fibrinolytics: 1 mm or more of ST elevations in contiguous leads or a new/presumably new complete LBBB within 1 2 hours of the onset of chest pain. A posterior MI should be treated as a STEMI equivalent. The goal of PCI is to achieve optimal revascularization of the infarct-related artery by establishing TIMI 3 flow. Multives­ sel revascularization at the time of primary PCI is usually not indicated (class Ill recommendation) , except in patients with cardiogenic shock. Several approaches to PCI exist in the setting of STEMI. Most data support the use of primary PCI. In primary PCI,

Weig ht-based Dosi ng of Tenecteplase Weight (kg)

2 or EF ::; 35% for inferior MI 2. Anterior MI with � 2 mm or more ST elevation in 2 or more leads Defi ned in TRANSFER-AM I

1 . � 2 mm ST elevation in 2 anterior leads or ST elevation � 1 mm in inferior leads with at least: SBP < 1 00 mm Hg HR > 100 bpm Killip class II-III � 2 mm ST-segment depression in anterior leads � 1 mm of ST elevation in right-sided lead V4 indica­ tive of RV infarct Adapted from Kushner FG, Hand M, King SB, et al. 2009 Focused Updates: ACC/AHA Guidelines for the Management of Patients With ST-Elevation Myocardial Infarction (Updating the 2004 Guideline and 2007 Focused Update) and ACC/AHNSCAI Guidelines on Percutaneous Coronary Intervention (Updating the 2005 Guideline and 2007 Focused Update). ] Am Coll Cardiol. 2009;54:2205-224 1 .

597

of negative results in the ASSENT 4 and FINESSE trials. In the ASSENT 4 trial, STEMI patients were randomized to either full-dose tenecteplase plus PCI or primary PCI with unfractionated heparin (UFH) . Patients who received fibrinolysis had an increased incidence of the composite endpoint of death, cardiogenic shock, or congestive heart failure (CHF) . In the FINESSE trial, STEMI patients were randomized to abciximab followed by PCI, abciximab and reteplase followed by PCI, or primary PCI with abciximab given in the cath lab . The primary endpoint was a compos­ ite of death from all causes, VF occurring >48 hours follow­ ing randomization, or cardiogenic shock and CHF within 90 days. The primary outcome did not differ between the groups, however, there was a significant increase in maj or and minor bleeding in the patients who received fibrinoly­ sis. Given this , facilitated PCI is no longer recommended as a management strategy for STEMI patients. The role of late or delayed PCI in asymptomatic indi­ viduals 12 to 24 hours after the initial event is unclear as there is a paucity of evidence. In the BRAVE 2 trial, per­ formance of PCI in this time window was associated with a decreased infarct size on SPECT. Performance of PCI in hemodynamically stable patients with an occluded infarct artery > 72 hours after the initial event is unwarranted. The OAT trial was a randomized study that enrolled patients with total occlusion of the infarct-related artery 3 to 28 days after Ml. Patients were randomized to PCI with stent­ ing and optimal medical management versus medical man­ agement alone. The primary endpoint was a composite of death, MI, and New York Heart Association (NYHA) class IV heart failure. PCI with stenting did not reduce the occur­ rence of death, reinfarction, or heart failure, and there was a trend toward excess reinfarction during 4 years of follow-up. Consequently, AHA recommends against PCI of an occluded infarct-related artery >24 hours after STEMI if the patient is hemodynamically stable and does not have signs of severe ischemia (class III recommendation) . Historically, STEMI patients who were selected for mechanical reperfusion underwent percutaneous translumi­ nal coronary angioplasty (PICA) . With the advent of intrac­ oronary stents, randomized trials were designed to determine if PCI using intracoronary stents would improve outcomes. A meta-analysis that involved nearly 3,000 patients with STEMI who were randomized to PICA versus PCI with intracoronary stents revealed an advantage to the use of stents. This analysis documented a reduction in the compos­ ite endpoint of death, MI, and target vessel revascularization at 6 months by the use of stents (14% vs. 26%, p < 0.000 1), a difference that was driven by a reduction in the need for target vessel revascularization. The largest trial in this analy­ sis was the CADILLAC trial. This study showed no reduction in death or MI from the use of stents, although there was less clinical and angiographic restenosis at follow-up . Drug-eluting stents (DES) dramatically reduce resteno­ sis compared to bare metal stents. However, because they were not initially evaluated in patients with AMI, the use

598

SECTION VII • CORONARY ARTERY DI SEASE AND CARD IAC CATHETERIZATION

of a drug-eluting stents in this setting is considered "off­ label." Since 2005 , several trials have demonstrated the ben­ efit of drug-eluting stents in the setting of AMI. One such trial, HORIZONS-AMI, randomly assigned STEMI patients to either paclitaxel-eluting stents or bare metal stents. At 1 2 months, the rate of ischemia-driven target vessel revas­ cularization was significantly lower in patients treated with paclitaxel-eluting stents. In addition, paclitaxel-eluting stents were not inferior to bare metal stents in the rates of the combined safety endpoint of death, stroke, stent thrombosis, or reinfarction. In 2009 , a meta-analysis of data from 1 8 reg­ istries and 1 3 randomized trials further supported the ben­ efit of drug-eluting stents compared to bare metal stents in patients with STEMI. In the registries and randomized trials, drug-eluting stents significantly reduced the rate of target vessel revascularization without increasing the rate of death or MI. Accordingly, the 20 1 1 ACCF/AHNSCAI PCI guide­ line states that DES can be used as an alternative to BMS in cases where the risk of restenosis is high, as long as the patient can tolerate and comply with prolonged dual anti­ platelet therapy (DAPT) (class I recommendation) . Safety of DES stents, especially in the year following implantation, is closely linked to compliance with dual antiplatelet therapy. Consequently, patients who are likely to be noncompliant with dual antiplatelet therapy and those requiring an urgent noncardiac surgical intervention may benefit from place­ ment of bare metal stents or balloon angioplasty alone. With the success of intracoronary stents and adjunctive antiplatelet as well as antithrombotic therapy, PCI is usu­ ally successful in achieving TIMI 3 flow in the infarct-related artery in >90% of subjects. Thrombus aspiration catheters have been shown to improve ST-segment resolution and myocardial blush, and are associated with improved clini­ cal outcomes. In the TAPAS study, patients who received aspiration thrombectomy were significantly more likely to have complete resolution of ST-segment elevation when compared with patients who underwent conventional bal­ loon angioplasty and PCI (56.6% vs. 44. 2 % , p < 0.001). At 1 year, there was also a significant reduction in the rates of cardiac death (3 .6% vs. 6.7% , p 0.02) and cardiac death or nonfatal reinfarction (5 .6% vs. 9 . 9 % , p 0 .009) . Based on current guidelines, thrombus aspiration is considered rea­ sonable during PCI in patients with STEMI who have a high clot burden and short ischemic times. Coronary artery bypass grafting (CABG) is still indi­ cated for left main disease, failed PCI, or mechanical compli­ cations of infarction (e.g. , myocardial rupture) . Additionally, patients with three-vessel disease (or two-vessel disease that includes the proximal left anterior descending artery) in the setting of left ventricular dysfunction or diabetes may have a better clinical outcome with surgery. =

=

Anti platelet Agents just as aspirin is the cornerstone of treatment for all UN NSTEMI patients, it is also a class I recommendation for STEMI patients (see Fig. 4 1 .4) . Aspirin is associated with

a mortality benefit similar to that achieved by streptoki­ nase. Unless there is a serious contraindication to its use, a loading dose of 1 62 to 325 mg of nonenteric coated aspirin is currently recommended for all STEMI patients (class I recommendation) . If there is any question as to whether the patient received aspirin prior to arrival in the emergency department, another dose should be given. If the patient is vomiting, aspirin can be given by rectal sup­ pository if necessary (at the same dose) . All post-PCI STEMI patients should receive aspirin indefinitely. Whereas previ­ ous guidelines recommended high dose aspirin for at least one month, the 20 1 1 PCI guideline states that 81 mg of aspi­ rin is reasonable (class Ila recommendation) . When signifi­ cant hypersensitivity to aspirin exists, clopidogrel should be given in its place (class I recommendation) . Thienopyridines should be used routinely in all patients with STEMI regardless of whether or not reperfusion therapy is received and should be continued for at least 1 year (class I recommendation) . An important caveat with thienopyri­ dines is the increased risk for major bleeding during surgery. It is currently recommended that clopidogrel and prasug­ rel be held for 5 and 7 days, respectively, prior to CABG, unless the need for urgent revascularization outweighs the risk of potential excessive bleeding (class I recommenda­ tion) . In patients in whom PCI is planned, a loading dose of clopidogrel or prasugrel should be given prior to or at the time of PCI. Currently, the recommended loading dose of clopidogrel is 300 to 600 mg (class I recommendation) . Although results of the CURRENT-OASIS 7 trial suggest that patients may benefit from a 600 mg clopidogrel loading dose compared to 300 mg, there is currently insufficient data to establish superior safety and efficacy of this higher loading dose. The recommended loading dose of prasugrel is 60 mg (class I recommendation) . Prasugrel is considered to be superior to clopidogrel in onset of action and potency of platelet inhibition. The TRITON-TIMI 38 investigators evaluated the efficacy and safety of prasugrel compared to clopidogrel in 1 3 ,608 patients with moderate to high-risk ACS undergoing PCI. Patients who received prasugrel had significantly fewer ischemic events, including stent thrombosis. The risk of major bleeding, including fatal hemorrhage, was higher with the use of prasugrel though overall mortality did not differ between the two groups. Currently, the AHNACC guidelines do not consider one agent superior to another; however, clopidogrel is pre­ ferred in certain situations. Prasugrel is not recommended in patients with a history of transient ischemic attack (TlA) or stroke due to the risk of ICH (class III recommenda­ tion) . Also , in patients who have received fibrinolysis, clopidogrel is the thienopyridine of choice. This recom­ mendation is based on results of the CLARITY-TIMI 28 trial. This trial revealed that in STEMI patients who undergo fibrinolysis, there is a reduction in the composite endpoint of occluded infarct-related artery, death, or recurrent MI before angiography by the addition of clopidogrel to aspirin,

CHAPTER 4 1 • ACUTE MYO CARD IAL INFARCTION heparin, and standard medical therapy. The recommended loading dose of clopidogrel following fibrinolysis is 300 mg if given within 24 hours of fibrinolysis and 600 mg thereafter (class I recommendation) . In patients who receive PCI, thienopyridine therapy should be continued for at least 1 year (class I recommen­ dation) . Fifteen months of therapy is preferred in patients undergoing drug-eluting stent placement (class Ilb recom­ mendation) . If the risk of bleeding outweighs the potential benefit of thienopyridine therapy, earlier discontinuation should be considered. The maintenance dose of clopidogrel and prasugrel is 75 mg and 1 0 mg daily, respectively. Ticagrelor is a reversible and direct-acting oral antago­ nist of the platelet adenosine diphosphate receptor P2Y 12 • It provides faster and greater platelet inhibition than clopi­ dogrel, without an increase in bleeding complications. Based on results of the PLATO trial, it is now considered an acceptable alternative to clopidogrel or prasugrel (class I recommendation) . PLATO was a multicenter, randomized trial that evaluated the benefit of ticagrelor compared to clopidogrel in patients with ACS . At 1 year, the primary end point, a composite of death from vascular causes, MI, or stroke, was significantly less in patients who received ticagrelor without an increase in maj or bleeding. Patients who receive ticagrelor should not be treated with high-dose aspirin, as high-dose aspirin has been associated with worse outcomes in these patients . G P l l b/l l l a I n h i bitors

The benefits of GP Ilb/Illa inhibition during primary PCI in the pre DAPT era are well documented. An analysis that included ADMIRAL, CADILLAC, ISAR-2, and the RAPPORT trials revealed a reduction in rates of the composite endpoint of death, recurrent MI, or target revascularization by 6 months with the adjunctive use of abciximab during PCI compared to placebo (OR = 80, 95% CI 0.67 to 0 .97) . The efficacy of these agents in the setting of dual oral antiplatelet therapy is less certain. The dose of abciximab is a 0.25- mg/ kg intra­ venous bolus, followed by an infusion of 0. 125 mg/kg for 1 2 hours. Based on the 20 1 1 ACCF/AHNSCAI PCI guide­ line treatment with abciximab , eptifibatide, or tirofiban is reasonable at the time of PCI in selected patients with STEMI (class Ila recommendation) . The routine use of these agents prior to arrival in the cardiac catheterization lab is not rec­ ommended (class III recommendation) .

Antithrombotic Agents All STEMI patients should receive anticoagulant therapy, which has traditionally been unfractionated heparin (UFH) . The dose of UFH varies depending on the reperfusion strat­ egy selected. The dose is 60 U/kg as a bolus (maximum 4,000 U) , followed by 1 2-U/kg!h infusion (maximum 1 ,000 U/h) to achieve a partial thromboplastin time (PTT) of 45 to 65 seconds in patients undergoing fibrinolysis or patients undergoing PCI with an adjunctive GP Ilb/Illa inhibitor. The goal of intraprocedural activated clotting time (ACT) in

599

this case is 200 to 250 seconds. For patients undergoing PCI without adjunctive GP Ilb/Illa inhibitor, the dose of UFH is 80 U/kg as a bolus, followed by 1 8-U/kg!h infusion to achieve a PTT of 50 to 75 seconds and an ACT of 300 to 350 seconds during the PCI. In general, heparin should not be continued after PCI, because there is increased risk for maj or bleeding and no incremental benefit. Exceptions to this rule include patients at high risk for systemic emboli, such as with large anterior infarction/left ventricular throm­ bus and atrial fibrillation. Deep venous thrombosis should be prevented during periods of immobilization by subcuta­ neous UFH, 5 ,000 to 7,000 U, twice to three times per day when therapeutic doses of heparin are not being used. Bivalirudin is considered an acceptable alternative to UFH for primary PCI (class I recommendation) . This rec­ ommendation is based on results of the HORIZONS AMI trial. This trial randomized 3 ,600 patients to either biva­ lirudin and provisional GP Ilb/Illa inhibitor or UFH and planned GP Ilb/Illa inhibitor prior to primary PCI. Only 7.5% of patients in the bivalirudin group received a GP Ilb/ Illa inhibitor. The primary end points were maj or bleed­ ing and the 30-day rate of combined adverse clinical events (major bleeding, death, reinfarction, target-vessel revascu­ larization, and stroke) . Patients in the bivalirudin group had a significant reduction in the primary end point, most of which was due to a reduction in the rate of maj or bleed­ ing. The benefit of bivalirudin was maintained at 1 year. The use of bivalirudin in the setting of fibrinolysis was evaluated in the HER0-2 trial. In this trial, 1 7,073 STEMI patients were randomized to streptokinase and bivalirudin or streptokinase and UFH. The primary endpoint of mor­ tality was not reduced by bivalirudin, although reinfarction was reduced by 30% within 96 hours. There was a small increase in mild to moderate bleeding with bivalirudin. If this agent is selected, the dose is 0. 75 mg/kg bolus, followed by an infusion at 1 . 75 mglkg!h with a PTT not to exceed 75 seconds. Thus, according to ACC/AHA guidelines, it is reasonable to consider bivalirudin as an alternative to UFH in patients who have been treated with streptokinase and have a known heparin allergy (class Ila indication) . Fondaparinux is a synthetic heparin pentasaccharide that acts through antithrombin to selectively inhibit factor Xa. The dose is 2 . 5 mgld given subcutaneously. Its efficacy in STEMI was evaluated in the OASIS-6 trial. In this trial patients were classified as stratum 1 , meaning UFH was not indicated, or stratum 2 , meaning UFH was indicated. Patients in stratum 1 were randomly assigned to fondaparinux or placebo. Patients in stratum 2 were randomly assigned to fondaparinux or UFH. Primary PCI was performed in 0.2% of stratum 1 patients and 53% of stratum 2 patients. Throm­ bolysis was performed in 78% of stratum 1 patients and 16% of stratum 2 patients. The most common thrombolytic used was streptokinase and a quarter of patients did not receive any form of reperfusion therapy. For the entire population (strata 1 and 2), there was a significant reduction in the pri­ mary endpoint of death or reinfarction at 30 days (9 . 7 vs.

600

SECTION VII • CORONARY ARTERY DI SEASE AND CARD IAC CATHETERIZATION

1 1 .2 % , HR 0.86) . When the strata were evaluated individu­ ally, there was a significant reduction in the primary end­ point in strata 1 ( 1 1 .2% vs. 14.0%, HR 0 . 79) but not strata 2 (8.3% vs. 8.7%, HR 0.96) . The lack of benefit in strata 2 was due to worse outcomes in patients undergoing pri­ mary PCI. The use of fondaparinux in primary PCI was asso­ ciated with an increase in guiding catheter thrombosis and in coronary dissection, no reflow, and abrupt closure. Based on these results, the use of fondaparinux as the sole antico­ agulant during primary PCI is a class III recommendation. Currently, if a patient receives fondaparinux then undergoes PCI, the ACC/AHA recommends additional IV treatment with an anticoagulant that possesses anti-Ila activity such as heparin, enoxaparin, or bivalirudin. Based on results of the FUTURA/OASIS 8 trial, standard-dose heparin is preferable to low-dose heparin as there is no increase in maj or bleed­ ing and a statistically significant reduction in CV death, MI, TVR, and stent or catheter thrombosis. Low-molecular-weight heparin (LMWH) may be con­ sidered as an alternative to UFH in patients undergoing fibrinolysis (class Ilb recommendation) . The ASSENT-3 trial tested various antithrombotic regimens with weight­ based tenecteplase. LMWH was represented by enoxa­ parin initiated by 30-mg intravenous bolus, followed by 1 .0 mg/kg subcutaneously every 1 2 hours up to discharge or revascularization, for a maximum of 7 days. Tenecteplase plus enoxaparin reduced a composite endpoint of death, in-hospital reinfarction, or in-hospital refractory ischemia compared to UFH. The ExTRACT-TIMI 25 trial randomized 20,506 STEMI patients undergoing fibrinolysis to either enoxaparin throughout the index hospitalization or UFH for 48 hours. The primary endpoint was death or nonfatal MI at 30 days. The enoxaparin group had a significant reduction in the primary endpoint, primarily due to a significant reduc­ tion in reinfarction (3 .0% vs. 4.5%). The enoxaparin group also had a significant reduction in urgent revascularization (2 . 1 % vs. 2.8%). Unfortunately, the interpretation of this study is limited by the difference in duration of therapy between the two groups. The mean duration of enoxaparin therapy was 7 days, whereas the mean duration of therapy with UFH was 48 hours. Currently, based on ACC/AHA guidelines, patients who undergo reperfusion with fibrino­ lytics should receive anticoagulant therapy with UFH, enoxaparin, or fondaparinux for a minimum of 48 hours and preferably the duration of the index hospitalization, up to 8 days (class I recommendation) . If more than 48 hours of therapy is required, enoxaparin or fondaparinux are preferable to UFH because of the risk of heparin-induced thrombocytopenia (class I recommendation) . For patients undergoing PCI after having received enoxaparin, additional dosing in the cardiac catheterization lab should be based on the time at which the last dose was received. If the last dose was within 8 hours, no additional enoxaparin should be given. If the last dose was given 8 to 12 hours earlier, or if the patient has received less than 2 subcutaneous doses, an IV dose of 0.3 mg/kg should be given. If the last dose

was given > 1 2 hours earlier, another 1 mg/kg subcutaneous dose should be administered (class I recommendation) . For patients who are > 75 years old or who have renal insuffi­ ciency (creatinine >2 . 5 mg/dL for men and >2 .0 mg/dL for women) the use of a LMWH is not recommended (class III recommendation) .

Anti-lschemic Agents Nitroglycerin and beta-blockers are first-line anti­ ischemic agents (class I recommendation) . Nitroglycerin is initiated by a 0. 4-mg sublingual tablet (repeated several times every 5 minutes if symptoms persist and hypoten­ sion does not develop) , followed by intravenous infusion of 10 to 20 µg/min (titrated up until resolution of symp­ toms or until hypotension develops) . An intravenous dose of 200 µg/min is considered a ceiling, although the dose is occasionally increased to 400 µg/min if needed. Nota­ bly, large-scale randomized trials have failed to observe any reduction in mortality with nitroglycerin, and indi­ cations for this agent in the setting of STEMI are thus to relieve ischemia , hypertension, or pulmonary congestion. Nitrates should not be utilized in the setting of a sus­ pected right ventricular (RV) infarction as venous pooling can result in significant hypotension. Sildenafil use within 24 hours of presentation is a class III recommendation against the use of nitroglycerin. Similar caution is applica­ ble to other PDE5 inhibitors . Beta-blockers are administered along with nitroglyc­ erin and help to blunt the reflex tachycardia that may occur from their use. A large body of evidence supports the use of beta-blockers (class I recommendation) . A pooled analysis from the prefibrinolytic era in >24,000 patients (dominated by the ISIS- 1 trial) documented a 14% reduction in 7 -day mortality (23 % long-term reduction) among patients who received beta-blockade. Interestingly, in the reperfusion era, only the CAPRICORN trial with carvedilol has shown a mor­ tality reduction with a beta-blocker. Other trials in the reper­ fusion era have only shown reduced reinfarction or recurrent ischemia. Oral beta-blockers should be administered in the first 24 hours to all STEMI patients without a contraindication (class I recommendation) . Medical contraindications to beta-blockers include significant conduction abnormali­ ties (marked first-degree AV block, or second/third-degree block) , asthma, or decompensated heart failure. The use of beta blockade with metoprolol in the COMMIT trial was associated with an increased risk of precipitating cardiogenic shock. Blunting the heart rate in patients with compensatory tachycardia likely resulted in this finding. Current guidelines highlight caution in patients at risk for cardiac shock. Risk factors include age > 70, systolic blood pressure < 120 mm Hg, heart rate > 1 1 0 or < 60 bpm, and delay in reperfusion. Patients with a contraindication to beta-blocker therapy within the first 24 hours of STEMI should be reevaluated for candidacy throughout the hospitalization (class I recommen­ dation) . If there are contraindications to beta-blocker use, a

CHAPTER 4 1 • ACUTE MYO CARD IAL INFARCTION nondihydropyridine calcium channel blocker (i.e . , diltiazem or verapamil) may be considered to control anginal symp­ toms. Morphine (1 to 5 mg intravenously) is also considered a class I anti-ischemic medication and is particularly helpful for anxious patients and to control the pain of infarction.

Secondary Prevention I n h i bition of the Reni n-Angiotensi n-Aldosteron e System

Angiotensin-converting enzyme inhibitors (ACE-I) are indi­ cated in all STEMI patients with a left ventricular ejection fraction (LVEF) 2 . 5 or a serum potassium > 5 were excluded. Study participants were randomly assigned to eplerenone or placebo. Patients in the eplerenone group had a significant reduction in mortality ( 14.4% vs. 1 6 . 7%), cardiovascular mortality (12.3% vs. 14.6%), and com­ bined cardiovascular mortality or hospitalization for car­ diac events (26 . 7% vs. 30.0%). Currently, the ACC/AHA recommends the use of aldosterone blockade in post-Ml patients who: have an EF < 40% , are already receiving ther­ apeutic doses of ACE-I and beta-blockade, and have either heart failure or diabetes, assuming the patient does not have significant renal dysfunction or hyperkalemia (class I recommendation) .

601

Ventricular fibrillation (VF) or pulseless ventricular tach­ ycardia (VT) should be treated with immediate unsynchro­ nized electric shock. If the arrhythmia is refractory to electric shock, 300 mg of IV amiodarone should be administered followed by repeat electric shock (class Ila recommenda­ tion) . When present, electrolyte and acid-base disturbances should be corrected. For shock refractory VT or VF, treat­ ment with boluses of IV procainamide may be considered (class llb recommendation) . Prophylactic antiarrhythmic therapy in the setting of STEMI is not recommended (class Ill recommendation) . A patient with sustained polymorphic VT or symptomatic sustained monomorphic VT should receive unsynchronized electric shock (class I recommendation) . Treatment of stable sustained monomorphic VT includes Amiodarone 1 50 mg IV over 1 0 minutes or synchronized electrical cardioversion (class I recommendation) . For refractory polymorphic VT, attempts should be made to reduce adrenergic stimulation and myocardial ischemia with beta-blockers, lABP use, and reperfusion therapy (class Ila recommendation) . Procaina­ mide therapy may be considered if the VT is not associated with angina, pulmonary edema, or hypotension (class llb recommendation) . Lipid Management

A fasting lipid profile should be checked within 24 hours of presentation in all patients with STEMI . Target LDL-C is < 1 00 mgldL (class I recommendation) and further reduc­ tion in LDL-C to 48 hours after MI, and who had received CABG or three-vessel PCI as manage­ ment for their MI were excluded. While the patients who received an ICD had less death due to arrhythmia compared to controls, at a mean follow-up of 30 months, there was no significant difference in all-cause mortality (7 . 5 % vs. 6.9%). The IRIS trial also evaluated the benefit of prophylactic ICD insertion in patients with a recent MI (5 to 3 1 days) . Results again showed no difference in all-cause mortality between patients who did and did not receive an ICD . MADIT II evaluated the benefit of delaying ICD inser­ tion until at least 1 month after MI. The trial enrolled 1 ,232 patients with a history of MI more than 30 days prior to enrollment (more than 90 days if bypass surgery was per­ formed) and an LVEF :S: 30% . Patients were randomized to prophylactic ICD implantation or standard medical therapy:

At an average follow-up of 20 months, ICD implantation sig­ nificantly reduced all-cause mortality (14.2% vs. 19.8% for standard therapy) . This survival benefit was entirely due to a reduction in SCD (3 .8% vs. 10.0% for standard therapy) . The SCD-HeFT trial randomized 2,52 1 patients with ischemic or nonischemic cardiomyopathy, a LVEF :S: 3 5 % , and NYHA class I I o r III heart failure t o ICD implantation, amiodarone, or placebo. At 5 years, all-cause mortality was significantly reduced in patients who received an ICD (29% vs. 36% with placebo) . This benefit did not differ based on the etiology of heart failure, but was nullified in patients with NYHA class III symptoms. Amiodarone therapy was not beneficial. Based on multiple studies, including the above trials, the current ACC/AHA guidelines for ICD insertion are as follows: An ICD should be inserted in any patient with VF or sustained hemodynamically significant VT that occurs 48 hours after acute MI (class I recommendation) . This is provided that the arrhythmia is not secondary to recurrent ischemia or MI. Patients whose MI occurred at least 40 days prior, who have a LVEF of :S:35 % and NYHA class II or III heart failure, should also receive an ICD (class I recommen­ dation) . In addition, patients with NYHA class I heart failure are candidates for ICD insertion if their LVEF is :S:30% at least 40 days after MI (class I recommendation) . If a patient receives CABG, the LVEF and NYHA functional class should be reassessed 90 days after the procedure to determine ICD candidacy: An ICD should not be inserted in patients with­ out ventricular arrhythmia 48 hours after STEMI and who have an LVEF > 35% 40 days after the MI or 3 months after bypass grafting (class III recommendation) . Wearable cardioverter-defibrillators have been used in patients who are considered at risk for SCD but do not meet the above criteria, such as patients waiting for reassessment of LVEF after coronary artery revascularization. The efficacy of wearable cardioverter-defibrillators was initially evaluated by a clinical trial that consisted of two components. The first component (the WEARIT study) enrolled 1 77 patients with a LVEF < 30% and NYHA class III or IV heart failure. The sec­ ond component (the BIROAD study) enrolled 1 1 2 patients with a recent MI or recent CABG who were considered high risk for SCD but did not meet criteria for an ICD or refused implantation. At the end of the 901 patient month obser­ vational period, there were six successful and two unsuc­ cessful defibrillation attempts. Both unsuccessful attempts were because the device was being worn incorrectly: There were six instances of SCD during the study; in five cases the device was not being worn and in one case it was being worn incorrectly: While wearable cardioverter-defibrillators are currently not recognized in the ACC/AHA guidelines, there appears to be benefit to their use in select patients.

Pericarditis Acute pericarditis develops in 10% to 1 5 % of AMI patients within 2 to 4 days. Pain that occurs within the first 24 hours

CHAPTER 4 1 • ACUTE MYO CARD IAL INFARCTION of a STEMI is unlikely to be secondary to pericarditis. Peri­ cardial effusion is common, although frank tamponade is infrequent. Unlike ischemic pain, pericarditic pain is more often sharp, worse with deep inspiration and recumbency. A pericardial friction rub is helpful in making the diagnosis, although it is not always present. The ECG may show diffuse ST elevation with PR depression. The treatment consists of aspirin (650 mg, three to four times per day) . Alternatively, 600 to 800 mg of ibuprofen four times per day may be used. Indomethacin is effective, although it should be avoided given its reduction in coronary blood flow and gastrointesti­ nal toxicity. Colchicine, 0.6 mg twice a day, may be added to aspirin or ibuprofen for refractory cases. Steroids should be avoided if possible, because of the concern for increased risk of myocardial rupture. Dressler syndrome is the finding of pleuropericarditis 1 to 2 weeks after the infarct. This inflammatory reaction occurs in 1 % to 2 % of AMI patients. The clinical course is usually benign, although constrictive pericarditis may result. The treatment is generally the same as for acute pericarditis. PREDISCHARG E RISK S T R AT I F I C AT I O N Stress testing is a widely used mechanism for risk strati­ fication after AMI. In STEMI patients who do not receive a left heart catheterization, exercise testing to assess for myocardial ischemia should be performed while in the hospital or early after discharge (class I recommenda­ tion) . If the patient has baseline abnormalities that prevent ECG interpretation, echocardiography or nuclear imag­ ing should be added to standard exercise testing (class I recommendation) . It should also be considered prior to the hospital discharge of STEMI patients in order to guide cardiac rehabilitation or determine the significance of a lesion seen on coronary angiography (class Ilb recommen­ dation) . Exercise testing should not be performed within 2 to 3 days of STEMI or in patients with UA, decompen­ sated heart failure, or life-threatening arrhythmias (class III recommendation) . It should also not be used to risk stratify patients who have already received a cardiac catheterization (class III recommendation) . Every patient after an AMI should have an assessment of left ventricular function. Patients with moderate to severe left ventricular dysfunction are at higher risk for adverse events. For these individuals, the use of beta-blockers and ACE inhibitors is especially important. Additionally, the implantation of an ICD may be indicated, after a period of convalescence from an AMI.

603

window of opportunity (generally 70 years, female gender, no history of prior MI or angina, transmural myocardial involvement, poor coronary collateral blood flow, and hypertension. 3 2-35 S i g n s a n d Sym ptoms

The acute presentation of free wall rupture is typically one of cardiovascular collapse and electromechanical dissociation. This is commonly associated with transmural, through-and­ through tears that cause abrupt tamponade. The subacute presentation is less severe, and patients slowly begin to manifest signs of CGS (see above) from tamponade. This is due to the slow egress of blood into the pericardial space from gradual or incomplete rupture of the infarcted myo­ cardium. It can also occur when thrombus or pericardium incompletely seals off the rupture site, also known as a pseu­ doaneurysm, or contained rupture. Patients may experi­ ence persistent or recurrent chest pain with ST- and T-wave abnormalities. Additionally, they may experience episodes of transient hypotension, nausea, a feeling of doom, and/or have a fleeting pericardial friction rub prior to decompensa­ tion. Signs of tamponade, such as hypotension, tachycardia, and neck vein distention, may be present. Diagnosis

In addition to the above symptoms and signs, the ECG often demonstrates persistent ST elevation and evidence of infarct extension or expansion.35 Hemodynamics by RHC will demonstrate elevation of intracardiac pressures, along with equalization of diastolic filling pressures and a reduced cardiac output. Pulsus paradoxus can be appreciated on the intra-arterial waveform, along with blunting of the Y decent on the right atrial and pulmonary arterial pressure waveforms (Fig. 42 . 6). TTE will demonstrate a large pericar­ dial effusion and signs of tamponade, including right atrial

CHAPTER 42 • C OMPLICATIONS OF MYO CARDIAL INFARCTION

61 5

Lead I I

Inspiration Respirometer

FIGURE 42.6 Hemodynamic fi n d i n g s in ta m pon­ ade. N ote that the aortic pressu re tracing demon­ strates hypotension a n d pulsus paradoxus (drop in systolic blood pressu re by > 1 0 m m H g u pon inspiration). I n add ition, the right atrial pressu re is elevated a n d the y-descent is extremely blu nted (arrow) . a, atrial contraction; x, atrial relaxation; v, atrial fi l l i ng (ventricular systole); y, atria l em pty­ ing (ventricular d iastole). (Modified from Wu LA, N i s h i m u ra RA. Pulsus paradoxus. N Engl J Med. 2003;349(7) :666.)

1� n o

(;f 1' rrn t:J11 '\) \1\ .

__

fl

n



r\fl p � {/\f' n

ft

\} \( \J \_(\) \! \ \J__

fl

\

70

Aortic pressu re

50

Right atrial p ressure ---- 0 mmHg a Magn ification of 20 right atrial pressu re waveform

collapse during ventricular systole (Fig. 4 2. 7) , RV collapse during ventricular diastole (Fig. 42 .8) , respiratory varia­ tion of the tricuspid and mitral valve inflow velocities, and a plethoric IVC (see Fig. 42. 7) that fails to collapse by 50% of its diameter with inspiration. It is important to note that pericardial effusion is a common finding following uncom­ plicated MI. Its presence should heighten one's suspicion for the possibility of subacute rupture. Serial echocardiog­ raphy and close clinical observation can then be performed in order to exclude further accumulation of pericardial fluid. Treatment

Patients with acute or subacute rupture should be sup­ ported with intravenous fluids and, if hypotensive, with vasopressors while emergency cardiothoracic surgical con­ sultation is obtained. Pericardiocentesis should be per­ formed only in the operating room, as decompression of the pericardial space will result in further bleeding. If a patient is hemodynamically unstable despite treatment with fluids and vasopressors, pericardiocentesis can be performed as a last resort because decompression may be the only chance for survival.

Pseudoaneurysm Although it is an infrequent complication of AMI, it is impor­ tant to recognize a pseudoaneurysm because it is prone to rupture. It occurs when pericardial adhesions and throm­ bus seal off an area of myocardial rupture. Although this can happen at any location, a recent review found that the posterior wall was the most common area of involvement. This was followed by the lateral, apical, and finally infe­ rior regions of the myocardium.36 In comparison to a true aneurysm, there is no myocardium between the LV cavity

y

and the pericardial space (Fig. 42 .9). Risk factors for the development of postinfarction pseudoaneurysm are similar to those for myocardial free wall rupture. Sym ptoms a n d Signs

Pseudoaneurysms are often silent and are discovered on follow-up imaging or postmortem. Gradual enlargement of the aneurysmal cavity can lead to progressive heart failure symptoms, although this is rare. Some patients present with ventricular arrhythmias. Others develop arterial emboliza­ tion after expulsion of thrombus from the aneurysmal cavity The physical examination can be normal or consistent with CHF Some patients will have a new murmur on ausculta­ tion, although 30% will have no murmur. 2 8 Rarely, a patient will present in CGS. Diagnosis

The ECG may show persistent ST elevation or regional peri­ carditis, although it most often demonstrates nonspecific ST changes.36 CXR can demonstrate an abnormal bulge around the site of involved myocardium but more frequently shows cardiomegaly There are several imaging modalities avail­ able for diagnosis, including contrast ventriculography, TTE, TEE, magnetic resonance imaging (MRI) , and com­ puted tomography (CT) . None of these tests has been 1 00% accurate, and no adequate comparisons between modalities have been made. Contrast ventriculography is the "gold standard" and has been associated with a high degree of diagnostic accuracy One will see a narrow orifice leading to a saccular cavity If concomitant coronary arteriography is performed, there will be a lack of vessels at the site of the pseudoaneurysm. Because this is an invasive modality, TTE with color Doppler is a reasonable test to perform first,

61 6

SECTION VII • CORONARY ARTERY DI SEASE AND CARD IAC CATHETERIZATION

A

A

B

B

FIGURE 42.7 A: Su bcosta l long-axis view demonstrati ng a large pericardia! effusion adjacent to the RA. B: Same view demon­ strating right atrial wa l l i nversion (arrow) d u ring systole. Note that IVC plethora is present in both i mages. HV, hepatic vein; PE, pericardia! effusion; RA, right atri u m .

the right ventricle, while s m a l l and underli l led, rema i n s open d u ring systole (arrow). B: Same view demonstrati ng RV diastolic col l a pse (smaller arrow) . LV, left ventricle; P E, pericard i a ! effusion.

although its diagnostic accuracy was found to be 26% for this condition36 (Fig. 42 . 1 0) . Although TEE and MRI have shown a higher degree of diagnostic accuracy, only small numbers have been studied and a definitive conclusion regarding superiority cannot be made. If MRI is used, cine

runs will increase diagnostic sensitivity with its ability to highlight abnormal blood flow patterns and turbulence in and around the cavity of a pseudoaneurysm. In addition, it will often demonstrate loss of epicardial fat at the site of rupture.

;,,,, i

FIGURE 42.8 A: Pa rasternal short-axis view demonstrati ng that

Segment lnfarcted

Thrombus Mural

Thinned Out Scar

Myocardial

Aneurysm

Wide base 2 . Walls composed of Myocardium 3. Low-risk of free rupture 1.

Pseudo-Aneu rysm (Contained Rupture)

2 . Walls composed of thrombus & per i card i um 3 . Hig h-risk of free rupture 1 . Na rrow base

FIGURE 42.9 Pseudoa neurysm versus aneu­ rysm. (Modified from Cercek B, Shah P K. Compli­ cated acute myoca rd ial infa rction: hea rt fa i l u re, shock, mechanical com p l i cations. Cardio/o Clin. 1 99 1 ;9(4):569-593.)

CHAPTER 42 • C OMPLICATIONS OF MYO CARDIAL INFARCTION

61 7

calcified with time. True aneurysms have a wide base and are frequently associated with mural thrombus. 2 6 Signs a n d Sym pto ms

A

Aneurysms place the entire ventricle, including the nonin­ farcted portion, at a mechanical disadvantage. Contractile energy is expended during passive outward expansion of the aneurysmal wall, and cardiac output decreases. This functional decline is more significant with acute aneurysms because the aneurysmal wall is more compliant and there­ fore expands to a greater degree during systole. Addition­ ally, the distorted geometry can lead to misalignment of the mitral valve apparatus and result in MR Patients can present early or several weeks following AMI. They can be asymptomatic, or develop CHF, CGS, or recurrent ventricular arrhythmias. The index event is less often systemic embolization. The physical examination may demonstrate signs of CHF and/or CGS. In addition, patients may have a diffuse, dyskinetic apical impulse that is shifted leftward. Auscultation may reveal a murmur suggestive of MR or a third heart sound. Diagnosis

B FIGURE 42.1 0 A: Apical long-axis view demonstrati ng a pseu­

doaneurysm of the posterior LV wa l l . B: Same view with Doppler demonstrating the rupture site (arrow) with turbu lence of blood flow i n and su rrou nding the cavity. LA, left atri um; AML, a nterior m itra l leaflet; PML, posterior m itra l leaflet; Pan, pseudoa neurysm; Ao, aorta.

Treatment

Once a pseudoaneurysm is diagnosed, urgent surgery is indicated because of a 30% to 45% risk of rupture. 36 If the pseudoaneurysm is incidentally diagnosed or the patient is asymptomatic, the patient should be monitored until surgi­ cal evaluation has occurred. Successful percutaneous closure of pseudoaneurysms has been recently reported.

Left Ventricular Aneurysm A true ventricular aneurysm differs anatomically from a pseudoaneurysm in that myocardium is present in its wall and there is no communication between the ventricular cavity and pericardial space (see Fig. 42 .9). Its incidence fol­ lowing AMI has been reported as high as 38% . With the advent of reperfusion therapy, its frequency has decreased to between 8% and 1 5 % . 2 6•37 Ventricular aneurysms most commonly complicate transmural anterior wall Mis and are thought to be the result of infarct expansion. In con­ trast to post-MI pseudoaneurysms, true ventricular aneu­ rysms rarely rupture because the walls become fibrotic and

In addition to the above physical findings, as with pseudoa­ neurysms, the CXR may demonstrate cardiomegaly and a bulge representing the aneurysmal area. The ECG will often show evidence of a transmural anterior MI and persistent ST­ segment elevation. TTE is the diagnostic test of choice and will show thinning of the myocardium and dyskinetic wall motion at the site of infarction. Thrombus should always be excluded, as it is found in more than half of the surgical and autopsy cases that have been studied. If there is inability to exclude thrombus with a standard surface echocardiogram, contrast can be given simultaneously to improve distinction between the ventricular cavity and endocardial lining. Other imaging modalities such as cardiac MRI or CT scanning can be useful in this regard. Treatment

Diagnosis of a ventricular aneurysm in itself does not change the treatment algorithm for a post-MI patient with compa­ rable degrees of heart failure and/or CGS (see above) . It is important to note that administration of an ACE-I within 24 hours of infarction is especially crucial in this situation, because of the drug's inhibitory effect on infarct expansion and beneficial effect on ventricular remodeling. If a patient is stable off mechanical and vasopressor support, an ACE-I should be started. Surgery, which should include an LV aneurysmectomy and concomitant CABG, may be indicated when there are symptoms and signs related to the aneurysm.5 However, careful evaluation and patient selection are necessary as there was no improvement in long-term outcome when aneurysmectomy was added to CABG alone in a recent RCT.38 Patients with small or moderate-sized, asymptomatic

61 8

SECTION VII • CO RONARY ARTERY DI SEASE AND CARD IAC CATHETERIZATION

aneurysms should not undergo surgery. They do require medical management for heart failure when it is present. Management of large, asymptomatic aneurysms remains controversial, and decisions to proceed with surgery are often individualized as above. Anticoagulation with warfarin for at least 3 months is indicated for all post-STEMI patients who develop a mural thrombus in the acute setting. This applies to diagnoses made within 1 month of the event. Anticoagulation is indi­ cated because systemic embolization can occur in as many as 10% with documented mural thrombi, and the risk of late thromboembolism appears to be decreased with oral anticoagulant therapy 39·40 Although the risk of embolization decreases dramatically in the subsequent months following the infarction, therapy should be continued indefinitely for those patients who are not at an increased risk of bleeding.5 Anticoagulation in these patients consists of the early admin­ istration of intravenous, unfractionated heparin or subcuta­ neous low-molecular-weight heparin, along with Coumadin therapy until the international normalized ratio is between two and three. Once this has been achieved, heparin may be discontinued. Patients who develop an LV aneurysm but no identifiable thrombus in the acute setting can similarly be anticoagulated because the incidence of thrombus in these patients, postmortem and intraoperatively, is at least 50%. 39 There is limited evidence to support long-term anticoagula­ tion in these patients, and practice patterns often differ. A D D I T I O N A L CO M P L I C AT I O N S

Right Ventricular Infarction with Hemodynamic Compromise RV infarction rarely happens in isolation and more com­ monly occurs during an inferior or inferoposterior LV MI. Patients present with various degrees of RV dysfunction, but only 10% to 1 5 % develop hemodynamically significant RV impairment. This typically occurs when there is an ostial or proximal RCA occlusion prior to takeoff of the RV marginal branches. Sym pto m s and S i g n s

If one understands the hemodynamic relationship between the LV, RV, and pericardium, the symptoms and signs of RV infarction become clear. It is important to realize that many of the hemodynamic changes overlap with tamponade, con­ strictive pericarditis, and restrictive cardiomyopathy This makes clinical context and echocardiographic examination very important. When RV infarction occurs, the RV filling pressure becomes elevated due to systolic and diastolic dysfunction, which in turn causes elevation of right atrial filling pres­ sures. Simultaneously, a decrease in RV output leads to a reduced LV end diastolic volume and the PCWP will be low. This is not always the case when there is concomitant LV dysfunction from a previous infarction or the current event.

LV preload becomes further reduced when intrapericardial pressure is increased by abrupt dilation of the RV Similar to tamponade, the LV and RV become interdependent.4 1 This combination of events leads to the triad of hypoten­ sion, elevated neck veins, and clear lung fields.42 Neck vein distention may not be seen if the patient is hypovolemic but may become apparent following aggressive fluid resuscita­ tion, one of the key aspects of treatment. Diagnosis

This diagnosis should be considered in any patient who pre­ sents with inferior ST-segment elevation on ECG. In fact, it is an ACC/AHA Class I indication to obtain a tracing of lead V4R and a TIE to look for RV infarction in patients with infe­ rior STEMI and hemodynamic compromise.5 RV infarction should also be considered in patients with ST depression in leads V 1 and V2 , as this may represent acute infarction of the posterior myocardium as opposed to septal, subendocardial ischemia. Again, TTE can confirm the diagnosis by dem­ onstrating hypokinesis and dilatation of the RV Right heart catheterization can help confirm the diagnosis, but findings are nonspecific and may overlap with those of tamponade, constriction, and restriction. One will see elevated RV filling pressures that are equal to or greater than LV filling pressures, normal or low pulmonary arterial and PCWP, and a reduced cardiac index. Another clue to significant RV involvement in patients with inferior or posterior MI is hypotension follow­ ing the administration of preload reducing agents such as diuretics and nitrates. Treatment

Similar to patients with AMI and CGS secondary to LV dysfunction, patients with AMI complicated by severe RV dysfunction should undergo emergency diagnostic angiog­ raphy and revascularization. If CABG is indicated, it is rea­ sonable (Class Ila ACC/AHA Recommendation) to delay it in patients with clinically significant RV dysfunction as the RV function frequently improves following several weeks of medical therapy5 Patients should be monitored in the ICU with both intra-arterial blood pressure monitoring and a RHC. If shock is present, the first line of therapy is aggressive fluid resus­ citation. This is done with isotonic saline until the PCWP is between 1 5 and 18 mm Hg. If shock remains after this is achieved, an inotropic agent should be added. Dobutamine is the preferred drug in this situation because it causes less hypotension. If vasopressors are required, a pure a-agonist should be avoided, as it will lead to pulmonary arterial vaso­ constriction and further decrease forward flow into the left ventricle. If severe LV dysfunction and an elevated PCWP exist, unlike the situation of isolated LV systolic dysfunction complicating AMI, sodium nitroprusside should be avoided as a reduction in preload might cause further deterioration of hemodynamics. These patients should be considered for IABP counterpulsation. It is important to avoid factors that

CHAPTER 42 • C OMPLICATIONS OF MYO CARDIAL INFARCTION increase RV afterload, such as hypoxemia, a-agonists, and elevations in PCWP, which include positive end-expiratory pressure (PEEP) . In addition, one should avoid agents that decrease RV preload. This includes medications such as nitrates, morphine, and diuretics; but also dysrhythmias that lead to disruption of atrioventricular (AV) synchrony, such as atrial fibrillation and high-degree AV block. Atrial fibrillation must be dealt with emergently in the hemodynamically unstable patient following RV infarction, with immediate direct-current cardioversion (DCCV) . If the patient is not hemodynamically compromised, a trial of antiarrhythmic therapy can be attempted; however, if sinus rhythm is not restored promptly, DCCV should be per­ formed. Bradyarrhythmias, a frequent complication of inferior myocardial infarction (IMI) with RV involvement, can be quite dangerous even when the atrium and ventricle contract synchronously. This is because the dilated right ventricle has a relatively fixed stroke volume and depends largely on heart rate to increase its output. Management of bradycardia in AMI is discussed in a subsequent section. It is important to know that if a patient with RV infarction requires temporary pacing; both atrial and ventricular leads should be placed, in order to maintain AV synchrony.

Dynamic Left Ventricular Outflow Tract Obstruction Although development of dynamic LVOT obstruction is a rare complication of MI, it is important to recognize because many of the traditional therapies used in the treatment of AMI complicated by CGS should be avoided. These include nitrates, afterload reduction, diuretics, IABPs, and inotropic agents. Dynamic LVOT obstruction most often occurs in the setting of an anteroapical MI with compensatory basal hyperkinesis. This combination of segmental wall motion abnormalities causes a decrease in the cross-sectional area of the LVOT and acceleration of blood flow across this region. The acceleration of blood flow decreases pressure above the mitral valve, causing systolic anterior motion (SAM) of the anterior mitral leaflet against the interventricular septum, which worsens the LVOT obstruction.43 These patients often have a single, significant stenosis in the LAD coronary artery, in addition to mild concentric LV hypertrophy or asymmet­ ric septal hypertrophy. Sym ptoms a n d Signs

Patients with dynamic LVOT obstruction usually have chest pain and evidence of an anterior or anteroapical STEMI. This complication has also been seen in non-ST-elevation myocardial infarction (NSTEMI) , but much less frequently. Symptoms and signs of CHF and CGS are often present (see above) . Patients can have a holosystolic murmur at the left lateral sternal border that radiates to the apex and represents MR in addition to a harsh crescendo-decrescendo systolic

61 9

murmur in the left second intercostal space, representing LVOT obstruction. Diagnosis

The possibility of LVOT obstruction should be considered in patients who have progressive hemodynamic deterioration in the setting of standard medical and mechanical therapies used to treat patients with AMI and CGS. The diagnosis is made by TTE. LVEF may be normal or depressed. Apical hypo or akinesis along with hyperkinesis of the basal seg­ ments of the heart will be seen, in addition to SAM and regurgitation of the mitral valve. The LVOT, best interrogated with continuous-wave Doppler in the apical five- and three­ chamber views, will demonstrate a gradient >30 mm Hg. Treatment

Standard revascularization and anticoagulant therapy for AMI must be instituted in these patients. What is different are the supportive measures used during the periinfarction period. This consists of beta-blockers and fluids. If shock is present, an a-agonist should be used. All of these therapies decrease the degree of LVOT obstruction. Phenylephrine, the most commonly used a-agonist, is started at 20 to 40 µg/min and titrated upward, until there is clinical improvement or the maximum dose has been reached.

Pericarditis There are two forms of pericarditis that occur in the setting of MI. The first, typically occurring within 24 to 96 hours of transmural MI, is a form of localized inflammation in the pericardial region above the necrotic myocardium, which tends to run a benign course. The second, a form of post­ cardiac injury syndrome also referred to as Dressler syn­ drome, can manifest 1 to 8 weeks following MI. Although the exact mechanism is unclear, it is felt to be the result of an autoimmune reaction involving myocardial antigen and antibody complexes. This form of pericarditis tends to be a more systemic inflammatory process, is often refractory to first-line therapies, and frequently recurs. Sym ptoms a n d Signs

Patients with pericarditis often develop positional chest pain. This tends to be sharp , pleuritic, exacerbated by recumbency, and commonly radiates to the trapezius ridge. If the patient has Dressler syndrome, he or she may also complain of arthralgias and myalgias. Dressler syndrome can also be associated with pleuritis and pleural effusions. Although these effusions are typically small, they may enlarge and cause dyspnea. Patients may be febrile in both forms of pericarditis, and those with Dressler syndrome can run fevers as high as 40°C. All patients with pericarditis may have leukocytosis and elevation of inflammatory markers such as the erythrocyte sedimentation rate and C-reactive protein. Physical examination may demonstrate a pericar­ dial friction rub .

620

SECTION VII • CORONARY ARTERY DI SEASE AND CARD IAC CATHETERIZATION

Diagnosis

Symptoms and the presence of a pericardial friction rub are quite specific for pericarditis. ECG can be helpful but is less sensitive, especially in the acute situation, as the evolution­ ary changes seen following MI can mask the typical ECG features of pericarditis (Table 42 .2). Although TTE is not diagnostic in situations of post-MI pericarditis, it must be obtained to rule out a significant pericardial effusion, seen more commonly in patients with Dressler syndrome. It is important to realize that the presence of an effusion is not diagnostic, as it is commonly seen following uncomplicated AMis. Likewise, absence of a pericardial effusion does not exclude the diagnosis. Treatment

There are two issues to consider and balance when treating patients with post-MI pericarditis. One is the need for anti­ inflammatory agents and the need to avoid anticoagulation. In terms of anti-inflammatory agents, aspirin is the first line of therapy If patients are refractory to standard doses, as much as 650 mg every 4 to 6 hours may be used. When high doses are needed, it is advisable to place the patient on an acid-suppressive regimen. Some patients will be refractory to or unable to take high-dose aspirin therapy In these patients, 0 . 6 mg of colchicine every 12 hours and/or 650 mg of acetaminophen every 4 to 6 hours can be tried. Nonsteroidal anti-inflammatory drugs (NSAIDs) other than aspirin and corticosteroids should be avoided unless used as a last resort. Corticosteroids and NSAIDs adversely

affect myocardial scar formation, which can lead to thin­ ning of the scar and, in some circumstances, infarct expan­ sion. There are reports suggesting that both drug classes put the patient at increased risk for myocardial rupture fol­ lowing AMI . Per ACC/AHA, the use of NSAIDs (except for aspirin) at the time of infarction are contraindicated, and these agents should be discontinued when patients present with MI (Class I ACC/AHA recommendation) . 1 7 There are no clear recommendations for patients who present with pericarditis after recovery from their AMI; however, given overall association of NSAIDs with progression of CAD , it is advisable not to use these agents in patients with estab­ lished disease. Clinical judgment is necessary if anticoagulation is required for a patient with post-MI pericarditis. It is an ACC/ AHA Class I indication to discontinue anticoagulant ther­ apy if an effusion develops or enlarges. This decision must be individualized and based on the risk-to-benefit ratio. If a decision is made to continue anticoagulation, the patient must be observed diligently for effusion enlargement and impending tamponade. A R R H Y T H M I C CO M P L I C AT I O N S

Bradyarrhythmias In the setting of AMI, management of bradyarrhythmias is complex because decisions regarding temporary and perma­ nent pacing must be made and require multiple considera­ tions. In the acute setting, if a patient is hemodynamically

ECG Cha nges in Perica rd itis versus STEMI

Stage

ECG Cha nges Typically Associated with Pericarditis0

Ila Ilb III IV

Diffuse, concave-up ST elevation; ST depression in aVR and V 1 ; PR depression in the limb leads and left chest leads; PR elevation in aVR ST and PR segments return to baseline, while T waves remain upright T waves begin to flatten and invert, largely in the leads that had ST elevation Diffuse T-wave inversion Prepericarditis ECG, however, T-wave inversions may persist

Stage

Evol utionary ECG Changes Typica l ly Associated with STEMI

II III

Convex-upward ST-segment elevation and upright T waves overlying area of infarct; ST-segment depres­ sion in opposite leads; Q waves begin to develop Gradual T-wave inversion followed by deep, symmetrically inverted T waves; Q waves continue to evolve Resolution of ST elevation,b T waves begin to normalize

"In pericarditis, the evolution of repolarization abnormalities does not always occur simultaneously as they typically do in MI . In addition, the distribution of repolarization abnormalities in myocardial infarction remains constant, whereas in pericarditis, multiple areas on the ECG can demonstrate different repolarization patterns. •If ST-segment elevation does not resolve by 6 weeks, consider the possibility of ventricular aneurysm or a large area of dyskinetic myocardium.

CHAPTER 42 • C OMPLICATIONS OF MYO CARDIAL INFARCTION stable despite a bradyarrhythmia, a decision must b e made regarding the need for prophylactic, backup pacing. This requires one to predict which patients are likely to progress to a life-threatening rhythm abnormality such as third­ degree AV block. The route by which pacing is performed must involve considerations regarding patient stability, the need for AV synchrony, and the bleeding risks associ­ ated with the use of thrombolytic and postinterventional therapies. Sinus bradycardia occurs in approximately 30% to 40% of AMis, most commonly with inferior MI and rep­ erfusion of the RCA.5 Although multiple mechanisms can be responsible, the most common is hyperactivity of para­ sympathetics due to stimulation of vagal afferents. This is termed the Bezold-jarisch reflex and causes both bradycar­ dia and hypotension. When patients become symptomatic from sinus bradycardia or from sinus pauses >3 seconds in duration, intravenous atropine is the first line of therapy5 This should be administered in doses of 0.5 to 1 mg every 3 minutes until the patient is no longer symptomatic or a total dose of 0.4 mg/kg has been reached. If symptomatic bradycardia persists, transcutaneous or transvenous pacing must be initiated. The development of atrioventricular conduction block (AVB) , intraventricular conduction delay (IVCD) , and/or bundle branch block (BBB) in the setting of AMI is asso­ ciated with an increased risk of in-hospital mortality Deci­ sions regarding prophylactic or therapeutic temporary pacing depend on the infarction location, the type of block and its presumed relationship to the AV node, the extent of preexisting conduction system disease, and the presence or absence of symptoms. When dealing with any form of heart block, its rela­ tionship to the AV node is an important factor to consider. This is significant because blocks proximal to or within the AV node, often referred to as intranodal block, are gen­ erally benign, with prophylactic and eventual permanent pacing typically not required. This is in contradiction to infranodal blocks, which tend to be more dangerous, often require prophylactic and therapeutic temporary pacing, and frequently result in permanent pacemaker insertion prior to hospital discharge.5·44 Typical intranodal blocks are first-degree and second-degree, Mobitz type I AVB . These are usually seen in inferior or inferoposterior AMis, and the RCA is usually the culprit artery, although the LCx can be involved. If third-degree AVB develops in the intranodal region, the QRS width is typically 90% of the total prog­ nostic information. Additional important prognostic factors include female gender, history of diabetes, hypertension, smoking, and vascular disease (Table 4 3 . 1). Advanced age has been recognized as an important pre­ dictor of mortality in several studies. In the NRMI (National Registry of Myocardial Infarction) registry, a community­ based database with information on >350 ,000 patients with acute MI at U.S. hospitals, in-hospital mortality ranged from 3% for patients younger than 55 years of age to 28% for individuals more than 84 years of age. Older patients are more likely to possess a history of a prior MI, have more severe coronary disease, and consequently are more likely to develop CHF and cardiogenic shock after MI. Addition­ ally; several reports have shown that older patients are also less likely to receive life-saving therapies such as immediate reperfusion therapy, beta-blockers, and aspirin, which may contribute to the worsened prognosis.

627

628

SECTION VII • CORONARY ARTERY DI SEASE AND CARD IAC CATHETERIZATION TA B L E

Pred ictors o f Morta l ity i n STEMI

Va riable

G ISSl-2 TIMI-II GU STO-I (n = 3,339) (n = 1 0, 2 1 9) (n = 4 1 ,02 1 ) ++

Age Prior MI Diabetes Smoking Hypertension Female gender Vascular disease +,

univariate predictor;

Ki l l i p Classification and Morta l ity from G U STO-I Trial

++

++ ++

+

++

++,

++ ++

+ +

+

Kil l i p Class

++ ++ ++

II

++

III

++

IV

++ +

multivariate predictor.

In several studies, women have been shown to have higher mortality after STEMI. In the GUST0- 1 trial, women had higher 30-day mortality ( 1 1 .3% vs. 5 . 5%), occurrence of shock (9% vs. 5%), and reinfarction (5 . 1 % vs. 3 . 6%) com­ pared to men. Part of this increased risk can be explained by the advanced age and increased prevalence of preexisting diabetes and hypertension. Additionally, women are more likely to present late during an infarction. Paradoxically, smokers possess a lower risk for early mortality, most likely because of their younger age. Dia­ betes mellitus has been associated with a 1 . 5 to 3 . 0 times higher mortality after STEMI . Whether this is due to a higher atherosclerotic burden or some other characteristic induced by the diabetic state, such as silent ischemia or a larger infarct size, remains unclear. Further, the nonfatal complications are also higher in diabetic patients, including a greater incidence of postinfarction angina, reinfarction, and heart failure.

Patients on 30-d Mortality (%) Entry (%)

Featu res

No CHF Early CHF (crackles, S3 ) Pulmonary edema ( 1/2 lung fields) Cardiogenic shock

70 20

2 20

5

33

5

66

CHF, congestive heart failure.

greater in patients experiencing anterior wall MI compared to after inferior MI, even when corrected for infarct size. Patients with RV infarction complicating inferior infarction have a higher mortality rate than patients sustaining an infe­ rior infarction without RV involvement. Patients with mul­ tiple leads showing ST-segment elevation and those with a high degree of ST-segment elevation have increased mortality, especially if their infarct is anterior. Patients with persistent or advanced heart block (e.g. , Mobitz type II, second-degree, or third-degree AV block) or new intraventricular conduction abnormalities (bifascicular or trifascicular) in the course of an acute MI have a worse prognosis than do patients without these abnormalities. The influence of high-grade conduction block is particularly important in patients with RV infarction, for such patients have a markedly increased mortality Other Mortality % 10

P hysica l Exa m i nation

The clues to right ventricular (RV) and LV dysfunction on physical examination provide the most important prognostic information. Accordingly, variables predictive of a worsened outcome include hypotension, tachycardia, jugular venous distension, an S3 gallop, pulmonary edema, and evidence of peripheral hypoperfusion, many of which are captured by the Killip classification (Table 4 3 . 2) . The physical exami­ nation can also help to identify mechanical complications of MI, such as acute mitral regurgitation, ventricular septal defect, and free wall rupture, all of which have been associ­ ated with significant mortality E l ectroca rd iog ra m

The ECG provides useful information about the location and size of infarction, likelihood of tissue reperfusion after treat­ ment, presence of ongoing ischemia, and conduction sys­ tem dysfunction. The finding of ST elevation or depression has similar prognostic implications (Fig. 4 3 . 1). Mortality is

9

8 7 6 5 4 . . . .

3 - · · · · ·

2 --

0

20

40

Days from Randomization 60

80

1 00

1 20

1 40

1 60

1 80

FIGURE 43.1 Mortal ity rate accord i n g to electroca rdiog ra phic findings on presentation with acute M l i n the G U STO l l b trial. (Ada pted from Savon itto S, Ardissino D, Granger CB, et a l . Prog nostic Va lue o f the Ad m ission Electroca rd iogra m i n Acute Coronary Synd romes. JAMA 1 999;281 :707-7 1 3 .)

CHAPTER 43 • RISK STRATIFI CATION AND PO ST-MYO CARD IAL INFARCTION THERAPY ECG findings suggesting a worse outcome are persistent hor­ izontal or downsloping ST-segment depression, Q waves in multiple leads, evidence of RV infarction accompanying an inferior infarction, ST-segment depressions in anterior leads in patients with an inferior infarction, and atrial arrhythmias (especially atrial fibrillation) . Other than these well-established predictors on ECG, ST­ segment resolution has generated renewed interest in deter­ mining effectiveness of reperfusion therapy. Resolution of ST elevation predicts successful perfusion at the myocardial level, which is the most important predictor of LV function and survival. Continuous ST-segment monitoring has been shown to yield important prognostic information after 60 minutes of observation. In the ASSENT 2 (Assessment of Safety and Efficacy of a New Thrombolytic) and ASSENT­ PLUS studies, the optimal cutoff for ST-segment resolution analyses was found to be 50%, measured at 60 minutes. Patients with ST resolution (40%) by this criterion had a 30-day mortality of only 1 .4%. Bioma rke r Assessment

Two separate groups of biomarkers have been used to pre­ dict outcome after MI. One group includes the myocar­ dial enzymes that predict infarct size and another group assesses the degree of systemic vascular inflammation. The prognostic value of CRP, endothelin, BNP, CD40 , and CD40 ligand has recently been investigated extensively These markers of inflammation seem to predict an active atherosclerotic disease process. Aggressive risk-factor mod­ ification may be more important when the levels of these markers are high. More conventional markers of myocardial damage include troponin-I or -T, creatine kinase (CK) , CK-MB 1 and 2 isoforms, CK-MB isoenzyme mass, and occasionally myoglobin. The presence and degree of troponin, CK, and CK-MB isoenzyme elevation on admission and thereafter have been associated with poorer outcome in the setting of both STEMI and NSTEMI. However, there is less informa­ tion on troponin levels in STEMI. In the GUSTO Ila study, 30-day mortality was substantially higher among patients who were troponin-T positive. Given their more rapid return to baseline, CK and CK-MB isoenzymes are also helpful for identifying high-risk individuals by facilitating the diagno­ sis of reinfarction shortly after an STEMI or NSTEMI. Cur­ rently, infarct size is determined by CK-MB mass; the role of troponin-I or -T in this matter has been less well established. I ma g i n g

Imaging at the time of acute infarction is used to determine the amount of jeopardized myocardium. Contrast echocardi­ ography and technetium-based imaging can be used to quan­ tify perfusion noninvasively Nuclear scanning is superior for quantifying perfusion, whereas echocardiography is better for assessing function. Acute imaging has been used princi­ pally in clinical trials to determine the degree of myocardial

629

salvage, which is the percent of ischemic myocardium at presentation that has adequate perfusion on follow-up. During Hospitalization Recurrent angina is an important predictor of a worsened outcome and the need for revascularization. Recurrent chest pain frequently signifies ischemic myocardium, either in the peri-infarct territory supplied by the infarct-related artery or ischemia at a distance secondary to a non-infarct-related artery Early revascularization is required in many patients who have postinfarct angina. Other important predictors include LV or RV dysfunction and mechanical complica­ tions of MI. Cardiogenic shock possesses a very high mor­ tality in which medical management is not effective. Early revascularization in patients who develop cardiogenic shock within 36 hours of an MI is recommended, based on the findings of the SHOCK (Should We Emergently Revascular­ ize Occluded Coronary Arteries for Cardiogenic Shock) trial, which showed reduced mortality with early revasculariza­ tion compared to medical stabilization (33 .3% vs. 5 1 .6%). Arrhythmias, including high-grade AV block, atrial fibrilla­ tion, or ventricular tachycardia, also predict poor outcome. Pred ischarge Assess ment

Although significant emphasis is placed on predischarge risk stratification, many high-risk patients will declare them­ selves clinically during their hospital stay The challenge for the clinician during the predischarge phase is to distin­ guish the few patients who remain at higher risk from the many relatively lower-risk patients. Although multiple test­ ing technologies have been developed to aid in this process, the low event rate in these patients C l -year mortality rates of 2% to 5%) mandates that these tests must be highly sensi­ tive and specific if they are to have clinical value. What tests should be routinely performed for predischarge risk stratifi­ cation is highly debated. Risk stratification at discharge can be accomplished by determining three factors: (a) resting LV function, (b) residual potentially ischemic myocardium, and (c) susceptibility to serious ventricular arrhythmias. More sophisticated testing may provide additional data but may not be as useful in changing patient outcomes. LV Function Assess ment

Assessment of LV function is typically performed by echo­ cardiography or by ventriculography at the time of cardiac catheterization. However, imaging of the left ventricle at rest may not distinguish between infarcted, irreversibly dam­ aged myocardium and hibernating myocardium. Therefore, many different techniques have been used to determine via­ ble myocardium, including dobutamine echocardiography, rest-redistribution thallium, positron emission tomography (PET) scanning, and magnetic resonance imaging (MRI) . Dobutamine echocardiography can provide functional assess­ ment along with information on viability and ischemia. How­ ever, the results are directly dependent on the expertise and

630

SECTION VII • CORONARY ARTERY DI SEASE AND CARD IAC CATHETERIZATION

experience of the interpreter. Radionucleotide imaging pro­ vides higher sensitivity to detect ischemia, but specificity can be compromised by the size of the patient, diaphragmatic or breast attenuation. Further, regional wall motion assessment is not as precise as with echocardiography Regardless of the imaging modality chosen, the prognosis is worse if there is significant LV dysfunction, or if there is a large amount of ischemic myocardium. Stress Testi ng

Patients who do not have high-risk features after successful thrombolysis should be considered for exercise stress test­ ing. Although the predictive accuracy of exercise stress test­ ing has diminished in the reperfusion era as a result of the lower incidence of adverse outcomes, it is still given a Class I indication under current American College of Cardiology/ American Heart Association (ACC/AHA) guidelines. In addi­ tion, although it is not known whether exercise testing can effectively risk-stratify patients who have not received acute reperfusion therapy, it is also assumed to be effective in this setting. Low-level exercise appears to be safe in patients who have been free of angina or heart failure and who possess a stable baseline ECG during the previous 2 to 3 days. Patients who are unable to exercise or who have baseline ECG abnor­ malities that would preclude interpretation should undergo an exercise test with imaging. Patients who cannot achieve a 3 or 4 MET workload, those who develop ischemia at a low level of exercise, or those in whom blood pressure (BP) drops during exercise should undergo coronary angiography No further testing should be necessary in patients without these high-risk findings. Assess ment for Risk of Sudden Ca rd iac Death

Determination of risk for SCD after MI is important because it is highest in the first 1 to 2 years after the index event. The most important predictor for SCD is LV dysfunction. Provocative electrophysiology studies are not necessary for risk stratification. Signal-averaged ECG, heart-rate vari­ ability, QT dispersion, and baroreflex sensitivity have been investigated to select specific patients with LV dysfunction who might benefit from an ICD . The presence of a filtered QRS complex duration > 120 milliseconds and abnormal late potentials recorded on a signal-averaged ECG after acute MI signifies somewhat higher risk for SCD . However, the signal-averaged ECG suffers from a high false-positive rate, which makes the test clinically less useful. Depressed heart­ rate variability is an independent predictor of mortality and arrhythmic complications after acute MI. A depressed baroreflex sensitivity value (3 .0 millisecond/mm Hg) is asso­ ciated with about a threefold increase in the risk of mortality These tests may provide useful prognostic information, but at present, only assessment of ejection fraction (EF) is neces­ sary to determine eligibility for a device, where significant LV dysfunction qualifies a patient for ICD placement. Recent data indicate, however, that ICD therapy is not beneficial in

the early post-MI period, and should be delayed for at least 1 month after an infarction. ICD implantation is generally deferred for 3 months after revascularization, either surgi­ cally or percutaneously, at which time reevaluation of LV function can be performed. Pred ischarge Ma nagement In contemporary practice, most patients with MI will undergo cardiac catheterization, even after receiving throm­ bolytics for STEMI, based on the CARESS in AMI and TRANSFER AMI trials and the most recent guidelines. In the minority of patients who do not undergo catheteri­ zation initially, a judgment is made as to the presence of clinical variables indicative of high risk for future cardiac events. Patients with spontaneous episodes of ischemia or depressed LV function who are considered suitable candi­ dates for revascularization based on their overall medical condition should be referred for cardiac catheterization. These patients are at increased risk of recurrent infarction (and subsequent increased mortality) , and may benefit from revascularization if severe coronary artery disease (CAD) is identified at catheterization. N O N - S T- E L E VAT I O N ACS Many patients with ACS present without ST elevation on ECG. It is important to note that although the risk of mortality during the index hospitalization is less than in those with ST-elevation ACS, the prognosis at 1 year is similar (see Fig. 4 3 . 1 ) . Typically, the underlying pathophysiology is a high-grade stenosis with plaque rupture, but unlike STEMI, the vessel is not totally occluded. Indeed, fibrinolysis has been shown to be of no benefit and may actually be harm­ ful in this patient cohort. Multiple trials have investigated the role of early angiography and PCI versus conservative management in these patients, and it appears that early invasive strategy in the high-risk population provides the best outcome and may even be more cost effective than a conservative strategy Non-ST-Elevation Ml Risk Stratification I n itial Presentation

A number of historical features predictive of a worse prog­ nosis following non-ST-elevation ACS have been derived from existing trial data. These features are summarized in Table 43 . 3 . The most important are older age, greater num­ ber of cardiac risk factors, known CAD , peripheral vascu­ lar or cerebrovascular disease, prior MI, previous PCI or coronary artery bypass graft surgery, history of CHF, a more severe anginal pattern, and the use of aspirin within a week of presentation. E l ectroca rd iogra m

Among patients with non-ST-elevation ACS, the pres­ ence of Q waves, ST changes associated with angina or at

CHAPTER 43 • RISK STRATIFI CATION AND PO ST-MYO CARD IAL INFARCTION THERAPY

1 8 (Treat Angina with Aggrastat and Determine Cost of Therapy with an Invasive or Conservative Strategy) analy­ ses (Fig. 43 .2). However, at the present time, there is no clear consensus on how to incorporate these markers in patient management.

TA B L E

Pred ictors of a Worse Prog nosis i n NSTEMI Va riable

P U RSUIT

TIMl-118

ESSENCE

(n = 9,461 ) (n = 1 ,957) (n = 3, 1 7 1 ) Age CAD Diabetes Vascular disease CHF history Anginal severity Aspirin within 7 d

++ ++

++

++

An essential element of risk stratification following an ACS is the quantification of short-term and long-term risk. Although there are many historical, physical exam, ECG, and biomarker variables that are significantly and independently associated with worse short-term outcome, the integration of these into an accurate estimation of risk is complex and has traditionally required the use of sophisticated multivariable modeling (Figs. 4 3 . 3 and 4 3.4 ). Nevertheless, simplified nomograms and risk scores incor­ porating the most important variables have been derived from a number of these analyses and allow for a reasonably accurate categorization of patients into low-risk, intermedi­ ate-risk, and high-risk groups. In the analysis by Boersma et al. patient age, heart rate, SBP, ST-segment deviation, signs of heart failure, and elevation of cardiac markers were the most important predictors of death or MI at 30 days. In the analysis by Antman et al. (TIMI risk score, see Fig. 43 .3), age >65 years, >3 coronary risk factors, prior CAD , ST deviation, >2 angina episodes in last 24 hours, use of aspirin within 7 days, and elevated cardiac markers were important in determining death, reinfarction, or recurrent severe ischemia requiring revascularization (termed TIMI risk score) .

++ ++

++ ++

Risk Scores

++

++

++

++

++

Other study predictors include female gender, number of risk factors, previous MI, prior PCI/CABG.

presentation (in particular, ST-segment depression) , T-wave inversions of significant amplitude (i. e . , >0.2 mV) , or the absence of ECG changes during angina are important pre­ dictors of future events (see Fig. 43 . 1) . When clinical vari­ ables are also considered, heart rate and the presence of ST depression on admission ECG are the most important mul­ tivariable predictors. Bioma rke rs

The presence and degree of troponin elevation on admission and thereafter can identify patients who are at increased risk of experiencing adverse outcomes. Cardiac troponin-I and troponin-T are particularly useful in identifying high­ risk patients with non-ST-elevation ACS . Other markers of inflammation, such as CRP, CD-40 , CD-40 ligand, fibrino­ gen levels , or brain natriuretic peptide (BNP) can add to the prognostic information in ACS . Adding multiple markers to assess a patient may add important prognostic informa­ tion, as illustrated in the OPUS-TIMI 1 6 (Oral Glycoprotein Ilb/Illa Inhibition with Orbofiban in Patients with Unsta­ ble Coronary Syndromes) trial and the TACTICS-TIMI

During-Hospita lization and Pred ischa rge Risk Stratification According to current ACC/AHA guidelines, patients who are deemed to be high risk, including those with recurrent ischemia or reinfarction, CHF, hemodynamic compromise, or life-threatening arrhythmias, are candidates for early angi­ ography Additionally, the guidelines recommend angiogra­ phy in those who have had prior PCI in the past 6 months, elevated cardiac biomarkers, new ST changes on EKG, high­ risk score, or prior coronary artery bypass grafting (CABG) .

A

FIGURE 43.2 Relative 30-day morta l ity risks in OPUS-TI M I 1 6 (A) and TACTICS-TI M I 1 8 (B) in patients stratified by the n u m ber of elevated cardiac bioma rkers (Tn l, CRP, and B N P). (From Sa bati ne MS, Morrow DA, de Lemos JA, et al. Mul­ tima rker Approach to Risk Stratification i n Non-ST Elevation Acute Coronary Syndromes. Circulation 2002; 1 05:1 760, with permission from Wolters Kluwer Health.)

B

30· Day Mor1ality RelaUve Risk

6

6 5

4

3

30 · Day Mortality Relative Risk

14

13

12

p < 0.001

10 8 6

p = 0.0 1 4

4

2

0

63 1

0

,

2

3

Nu mber of Elevated Cardiac Biomarkers

2 0 ........ .. -. . _.____ 0

1

2

3

Nu m b e r of Elevated Cardiac Biomarkers

632

S ECTION V I I • CORONARY ARTERY DI SEASE AND CARD IAC CATHETERIZATION

I ) gc > 65 )

2) � 3

1 4-Day Deat h/MVUrgent Revasc u l a rizat ion

D risk f.ic1ors

3) oronary o;1cn0>b

4)

5)

6) ,

dt'\iauon

' e re

50 45 40 35 30 25 20 15

I'>

>50%

:mgin. I S)mp1om'

>\ �

in I t 7 da).

7) El�"\'atfd cardl.

m. rker.

1 3.2 0-1

2

3

1 9.9

4

80 80



Male 3-4

9 � 1 2:

2

6

1

2

12 1 4 1

2

1 00

1 ( 2)

80

2

0

3

1

1 20

Yea Yea

0

3

0

0

2

26.2

5

N u mber of R i sk Factors

The FRISC II (Fast Revascularization during Instability in Coronary Artery Disease) and TACTICS-TIMI 18 studies reported significant decreases in the rate of death or MI at 6 months among patients randomized to early angiography with revascularization as needed (i.e. , early invasive approach.) The ISAR-COOL trial also demonstrated a decrease in death or large MI at 30-day follow-up in patients assigned to undergo angiography within 6 hours of presen­ tation compared with 3 to 5 days. The TIMACS trial reflects contemporary medical practice by including patients treated with aspirin, clopidogrel, heparin, fondaparinux, and gly­ coprotein Ilb/llla inhibitors. In this trial, the patients in the early invasive arm experienced less refractory ischemia, and patients with a higher GRACE risk score had fewer events of death, Ml, and stroke. In the absence of high-risk clinical features or post-ACS complications, patients who have not undergone coronary angiography should be considered at low or intermediate risk pending the results of further risk stratification. Non­ invasive testing provides useful supplementary information beyond that available from clinically based assessments of risk in this cohort. The purpose of noninvasive testing is to identify ischemia and estimate prognosis. Accordingly, noninvasive evaluation should include an assessment of LV function and/or ischemia in order to identify patients

0.th DeetMl l



>Cl 0

0 C')

6-7

50%

20%

40%

1 0%

20%

30%

1 5%

5%

0% 0

FIGURE 43.3 TIMI risk score (Ada pted from Antman EM, Cohen M, Bern i n k PJ LM, et a l . The TIMI Risk Score for U nsta ble Angina/Non-ST Elevation M l . JAMA 2000;284:835-842.)

who are at increased risk for adverse outcomes who are likely to benefit from coronary angiography and revascu­ larization. High-risk findings on noninvasive testing should direct patients to coronary angiography if they are eligible for revascularization (Table 43 .4) . It is not clear whether LV function assessment or myocardial perfusion imaging (with rest and during exercise or pharmacologic stress) is superior in assessing prognosis. The ability of most noninvasive tests to dichotomize patients into low-risk and high-risk groups appears similar (Table 4 3 . 5 ). Selection of the appropriate test should be based on patient characteristics, availability of the test, and institutional expertise in performance and interpretation. The ACC/AHA Guidelines recommend exercise ECG as the primary mode of noninvasive stress testing. Patients with baseline ECG abnormalities that preclude accurate interpretation (Table 4 3. 6) should undergo an exercise test with imaging. Those who are unable to exercise (the cohort at highest risk of future adverse outcomes) should undergo pharmacologic stress testing with imaging. According to the ACC/AHA Guidelines, stress testing is safe in low-risk patients who have been free of ischemia or CHF for 1 2 to 24 hours in intermediate-risk patients (Table 4 3 . 7) . Patients who do not have any high-risk findings on noninvasive eval­ uation require no further testing.

25%

c!

40.9

4

8

12 16

Risk Score

20

1 0%

0%

FIGURE 43.4 PURSUIT risk score 30-day outcome after non-ST-elevation ACS. (From Boersma E, Pieper KS, Steyerberg EW, et al. Predictors of Outcome in Patients With Acute Coronary Syndromes Without Persisent ST-Segment Elevation: Resu lts From an I nternational Trial of 946 1 Patients. Circulation. 2000; 1 0 1 :2557-2567, with permission from Wolters Kl uwer Health.)

CHAPTER 43 • RISK STRATIFI CATION AND PO ST-MYO CARD IAL INFARCTION THERAPY TA B L E

633

P O S T - M YO C A R D I A L I N FA R C T I O N TH E RAPY After M I , secondary prevention o f cardiovascular events depends on prompt institution of appropriate pharmacother­ apy, lifestyle changes, and comorbid disease management. In this area, recent advances have revealed the significant benefits of antiplatelet therapy, neurohormonal blockade, and lipid-lowering therapy. The importance of diet and exercise, as well as smoking cessation, cannot be overem­ phasized. In addition, optimal management of diabetes and hypertension are paramount to preventing further events. Residual LV function after MI is a strong determinant of the proper approach to post-MI pharmacotherapy. This section outlines the role of these therapies as well as lifestyle changes in the post-MI patient.

High-Risk Findings on Noninvasive Testing Leading to Coronary Angiography EKG-Abnormalities that Preclude Accu rate I nterpretation of an Exercise Stress Test

1. 2. 3. 4.

Severe resting LV dysfunction (LVEF < 0.35) High-risk treadmill score (score ::; -1 1) Severe exercise LV dysfunction (exercise LVEF < 0.35) Stress-induced large perfusion defect (particularly if anterior) 5 . Stress-induced multiple perfusion defects of moderate size 6. Large, fixed perfusion defect with LV dilation or increased lung uptake (thallium-2 1 0) 7. Stress-induced moderate perfusion defect with LV dilation or increased lung uptake (thallium-20 1) 8. Echocardiographic wall motion abnormality (involving >2 segments) developing at a low dose of dobutamine (::; 10 mg/kg/min) or at a low heart rate (< 120 bpm) 9. Stress echocardiographic evidence of extensive ischemia

Antiplatelet Therapy A large number of randomized, controlled trials, summa­ rized in meta-analysis by the Antiplatelet Trialists' Collabo­ ration, have documented the benefit of daily aspirin therapy after MI. Aspirin should be taken on a daily basis indefi­ nitely by post-MI patients who can tolerate it, based on a reduction in recurrent infarction, stroke, or vascular death.

TA B L E

Stress Test Pred ictors of Cardiac Death and M l Sensitivity CD

Exercise ECG ST depression Impaired SBP Time of exercise Exercise perfusion Reversible defect Multiple defects Exercise ventricular function Exercise RVG Peak EF 1 1 0 or 240 milli­ seconds, second- or third-degree heart block, active asthma or reactive airways disease) . Intravenous betablocker use is reasonable, but no longer has a class I recommendation level in the guidelines after the COMMIT trial demonstrated an increased risk of cardiogenic shock with IV betablocker use in STEMI patients, The BHAT trial (Beta-Blocker Heart

Attack Trial) , a prethrombolytic study, compared 180 to 240 mg of propranolol daily to placebo in post-MI patients, finding a 26% relative risk reduction for all-cause mor­ tality, and a 28% reduction in sudden death. Although metoprolol and atenolol are frequently prescribed to post­ MI patients, these agents have not been demonstrated to reduce mortality during long-term therapy. The MERIT­ HF (Metoprolol CR/XL Randomised Intervention Trial in Congestive Heart Failure) trial evaluated the use of meto­ prolol in chronic CHF with EF < 40% and found a 33% reduction in mortality. This trial included 66% of patients with ischemic cardiomyopathy. The COMET (Carvedilol or Metoprolol European Trial) compared carvedilol, a nonselective beta-blocker with alpha-blocking capability, with metoprolol in patients with chronic CHF, and found a 1 7% reduction in the risk of death from carvedilol, rela­ tive to metoprolol. More recently, the CAPRICORN trial (Carvedilol Post-Infarct Survival Control in Left Ventricular Dysfunction) tested carvedilol in post-MI patients with sig­ nificant LV dysfunction (EF � 0.40) . This trial found similar reductions in all-cause mortality and sudden death to the BHAT trial. Current ACC/AHA Guidelines support the use of beta blockers indefinitely in all post-MI patients without contraindication, and recommend an approach that incor­ porates gradual titration in those patients with moderate or severe LV dysfunction. Lipid Management Pharmacologic lipid management after MI is crucial for sec­ ondary prevention of cardiac events. Patients should have a lipid profile checked prior to hospital discharge after MI and should have statin therapy initiated before leaving the hos­ pital. The National Cholesterol Education Program (NCEP) published guidelines in 200 1 (ATP III) for lipid-lowering therapy that encourage the use of "therapeutic lifestyle changes" including weight reduction, increased physical activity, increased fiber intake, and reduced intake of satu­ rated fats and cholesterol. The recommended drug therapy for lipid lowering includes statins, bile acid sequestrants, nicotinic acid (niacin) , or fibric acids, depending on the patient's lipid profile and potential side effects. The ATP III guidelines also established a LDL goal of 0

'6 � � u ::-

-�

� c

::!:

� 0 .r:;

1i

CHAPTER 43 • RISK STRATIFI CATION AND PO ST-MYO CARD IAL INFARCTION THERAPY



UJ

0

·a·o-�g o( atorvastat in .

20 !S

10

'/�·

0

./

. ...-

. ..- ··

.../ ""'·"'

.... • • - · · · · · · · .r

_.... . .. . . . .. . .. . . .. .. .. .. ...

.

P aO.OOS

0

6

Pravast a t i n

2063

1 688

1 536

1423

8 10

13&

Atorv•s tatln

2099

1 73 6

1 59 1

1485

842

133

No. at Risk

12

ls

18

Months of Follow-up

21

24

27

30

FIGURE 43.6 Ka pla n-Meier estimates of the incidence of a l l­ cause morta l ity in the PROVE-IT TI M I 22 study. (From Ca nnon C, Braunwald E, McCa be CH, et a l . I ntensive versus Moderate Lipid Lowering with Statins after Acute Coronary Syndromes. N Engl J Med. 2004;350( 1 5):1 495-1 504, with perm ission.)

atorvastatin resulted in an LDL of 62 mg/dL, representing a 35% difference. The composite cardiovascular endpoint at 2 years was reduced by 16% with atorvastatin (Fig. 43 .6). This trial, as well as other recent trials such as MIRACL (Myocardial Ischemia Reduction with Acute Cholesterol Lowering) , and Phase Z of the A to Z trial, demonstrates that more intensive LDL lowering does result in additional benefit in high-risk patients. It may be that the additional benefit to aggressive lipid lowering relates to reduction in inflammation, as evidenced by recent data from the REVERSAL (Reversal of Atherosclerosis with Aggressive Lipid Lowering) trial, showing a relationship between pro­ gression of atherosclerosis and CRP levels. Risk-Factor Management Diabetic patients represent a high-risk subset because of macrovascular and microvascular complications including severe CAD, hypertension, peripheral vascular disease, and renal dysfunction. Prior conventional management of diabe­ tes in the setting of ACS dictated strict management of glu­ cose levels, but recent evidence has suggested potential harm with this strategy The NICE-SUGAR trial enrolled patients in an ICU setting and demonstrated increased incidence of hypoglycemia and death with an intense glucose control that attempted to keep the glucose level between 8 1 and 1 08 mg/dL. Standard therapy allowed for glucose levels up to 1 80 mgldL. Similarly, the ACCORD trial demonstrated increased mortality in type 2 diabetics who were assigned to the intensive therapy arm that targeted an A le level below 6% versus those in the standard therapy group that aimed for Al e levels between 7% and 7.9% . The American Diabe­ tes Association still recommends treating diabetic patients to achieve a target goal A le of 7% or less, but care should be taken to avoid hypoglycemia.

637

Besides medical management of hyperglycemia, other clinical trials have shown that combined neurohormonal blockade with ACE inhibitors, aldosterone antagonists, and beta-blockers are essential in treatment of diabetic patients with prior MI, and these medications should be continued with this patient population. There is some concern about beta-blockers masking the symptoms of hypoglycemia in diabetic patients, but they have been shown to be beneficial and should be used with appropriate caution in this high­ risk subset. Hypertension management post-MI is important in risk reduction for subsequent Ml. In addition to important lifestyle changes such as weight control, exercise, and sodium restric­ tion, current guidelines state that treatment of BP with drug therapy post-MI should be initiated to reach a target BP of 140/80 for all patients, and 130/80 for diabetics and patients with renal insufficiency. However, it is reasonable to treat all patients post-MI to a target BP of 120/80, considering the high-risk population represented by post-MI patients. These recommendations are based on the Seventh Report of the ] oint National Committee on Prevention, Detection, Evaluation, and Treatment of High Blood Pressure QN C-7). The same commit­ tee recommends the initiation of two agents if the BP is more than 20/10 mm Hg above goal. From a practical perspective, the post-MI patient should already be receiving a beta-blocker and ACE inhibitor for reasons detailed above, especially among patients with LV dysfunction (EF < 40%). Although achieving maximal doses of these medications is essential, the optimal method of reaching target dose is a matter of conjecture. In addition to beta-blockers and ACE inhibitors, thiazide diuretics and long-acting calcium channel antagonists are excellent antihypertensive therapy choices with excellent supporting data from large, multicenter randomized trials, in particular the ALLHAT results (Antihypertensive and Lipid­ Lowering Treatment to Prevent Heart Attack Trial) . Obesity is a major risk factor for coronary disease and should be carefully addressed in the post-MI patient as part of a comprehensive secondary risk-reduction strategy In particular, body mass index and waist circumference have been shown to be important in risk assessment. The desir­ able body mass index range is 1 8 . 5 to 24.9 kg/m2 , and the desired waist circumference is 65 yea rs old, at least th ree risk factors for CAD, sign ificant coronary stenosis (prior stenosis �50%), aspirin use with i n the last 7 days, severe angina (�2 episodes per day d u ring the last 24 hours), ST deviation on EKG, and positive cardiac bioma rkers. Th is patient's score is 5, assu m i n g his prior PCI was performed for a significant coronary stenosis.

Ml enrol led in GUSTO-I, a trial of lytic therapy, found that age was the most sign ificant pred ictor of 30-day mortal­ ity in a m ultivariable analysis. I n addition, anterior infarct location, higher Ki l lip class, elevated heart rate, and lower SBP were predictors, although they were not as significant as age. Together, these five characteristics included 90% of the prog nostic information in the baseline clinical data (Lee et al., Circulation. 1 995;9 1 : 1 659- 1 668).

7.

Answer E: Based on the TI M I risk score, the expected rate of a l l-cause mortality, Ml, and severe recu rrent ischemia at 14 days is 26%. Patients with high-risk TIM I scores (�3) should b e referred for coronary angiography over noni nvasive stress testing u n less there is a contra­ ind ication.

2. Answer E: The cu rrent ACC/AHA Guidelines for ST-segment-elevation myoca rdial i nfarction (STEMI) recom mend assessment of LV fu nction as part of a risk-stratification a lgorith m. It is accepta ble to proceed to ca rdiac catheterization, particu larly in patients with EF < 0.40 or with hig h-risk features. In patients who do not u ndergo ca rdiac catheterization, it is recom­ mended that those with an i nterpreta ble ECG, and who ca n exercise, undergo exercise stress testing, either as a su bmaxi mal stress test on days 4 to 6 or a symptom­ l i mited test on days 1 O to 1 4. EP testing is not part of the recom mended a lgorith m for risk stratification.

NEJM. 2007;35 7:200 1 -201 5) subgroups that were at high risk for bleeding included patients older than or eq ual to 75 years, weight but still off by approximately 75 % . =

Scenario 2 A morbidly obese patient (-50 cm thick in lateral dimen­ sion) undergoes a cardiac catheterization. Standard size

FIGURE 44.4 The x-ray focal spot is denoted on the fluoroscope

by a s m a l l red dot on the x-ray tube housing (black circle) . The sou rce to image dista nce is the distance between the x-ray foca l spot and the image receptor (A) . The source to skin entra n ce distance is the distance between the x-ray focal spot a n d the ta ble (B). The IRP is located at 1 5 cm below the isocenter of the C-a rm, (C).

collimator covers are removed and replaced with short colli­ mator covers allowing for a source to skin entrance distance of 30 cm. The procedure requires only one unique lateral view (Fig. 44. 6) . The K..r and K.,i are different by a factor o f (60/30) 2 4 . In other words, the K.,i is four times higher than the Ka,r displayed; the K..r is a gross underestimate in this scenario. Again, accounting for backscatter and tissue versus air differ­ ences, the actual Dskin,max is greater than the displayed value by approximately 4.5 times. =

I O N I Z I N G R A D I AT I O N A N D RA D I AT I O N E F F E C T S Radiation-induced detriment can b e broken down into two discrete categories, deterministic (nonstochastic) and sto­ chastic.

FIGURE 44.3 The a i r kerma at the reference plane, commonly

referred to c u m u l ative dose, is 1 ,260 mGy a n d the a i r kerma a rea 2 product, com monly referred to as DAP, is 6,090.4 µGym •

Determin istic Risk (Nonstochastic) Deterministic effects are those that have a threshold, below which a given effect is not expressed and above which the severity of the expression increases with increased radiation dose beyond the threshold. Primary examples of determinis­ tic effects are skin related erythema, epilation, and necrosis (Table 44.2).

646

SECTION VII • CORONARY ARTERY DI SEASE AND CARD IAC CATHETERIZATION Scenario

1

.._--+---

Procedure Table

Source to Skin Entrance

=

1 00 cm

------------------------------- ------- - l nte rventional Reference Plane = 60 cm

0

FIGURE 44.S I n Scenario l , a relatively s m a l l patient 20 cm thick i n the AP d i mension u ndergoes catheterization . The sou rce to image distance is 1 20 cm a n d the source to ski n entra nce d ista n ce is 1 00 cm. The I R P is approxi­ mately 60 cm a bove the x-ray tu be. Beca use the radiation i ntensity d rops

as the i nverse square of the dista nce from the x-ray sou rce, the a i r kerma at the reference plane in this exa m p l e g rossly overesti mates the a i r kerma a t skin entra n ce.

X-ray Focal Spot

Most deterministic effects are described for acute single exposures. However, patients will often undergo repeated interventions. How the skin reacts to multiple exposures is rather complex and greatly depends on the length of time between irradiations. Although beyond the scope of this book, there are formulized models that originated with radiation therapy patients for approximating single-dose estimations from multiple irradiations within a limited span of time. The primary radiation quantity associated with deter­ ministic effects is the Dskin,max· However, Dskin,max is not a quantity that is currently displayed on any fluoroscopic equipment; the closest quantity available is K..n but it should

be well understood that these are not the same quantity, and they can differ substantially even though they have the same unit of measure (Gy) . Dskin,max may be obtained directly by employing radiochromic films that can be calibrated to pro­ vide a dose estimate or by using smaller dosimeters such as thermoluminescent dosimeters (TLDs), placed on the patient's skin at the location where Dskin,max is most likely to occur. Based on K.,r though, patient education material should be made available to patients about the potential skin effects of radiation. Generally speaking, there are no limits on the amount of radiation a patient can receive for diagnostic or interven­ tional fluoroscopically guided procedures. It is assumed that

Scenario 2

F I G U RE 44.6 I n Scenario 2, a relatively large O

Source to i mage

=

90 cm

__

X-ray Focal Spot

Source to Skin Entrance l nterventional Reference 3 0 cm Plane = 90 cm

=

patient 50 cm thick i n the latera l d i mension undergoes catheterization. The sou rce to image d ista nce is 90 cm and the sou rce to skin entra n ce d ista nce is 30 c m . The IRP is approxi­ mately 60 cm a bove the x-ray tu be. Because the radiation inten sity d rops as the i nverse square of the dista nce from the x-ray sou rce, the a i r kerma at the reference p l a n e i n t h i s exa mple g rossly underestimates the a i r kerma at skin entrance.

CHAPTER 44 • RAD IATION SAFETY IN THE CARDIAC CATHETERIZATION LAB ORATORY TA B L E

Skin Effects from Va rious Radiation Doses Effect

Brief erythema, hair loss Persistent erythema, hair loss Skin fibrosis, atrophy Wet skin loss Skin necrosis Ulceration

Dose (Gy)

Onset

2-3

Hours to weeks

6-7

l-3 wk

1 0-1 1 15 18 20

1-3 mo 1 mo > 2 . 5 mo > 1 .5 mo

any use of radiation is offset by the potential benefit to the patient otherwise the procedure would not be performed. The one exception is a quasi-threshold established by the Joint Commission QC) that defines a sentinel event for > 1 5 Gy of cumulative skin entrance dose to a single field. Our catheterization laboratory has a specific protocol to deal with escalating doses of patient radiation dose (Fig. 44. 7). There is a great deal of ambiguity surrounding this sentinel event, not the least of which is what constitutes "cumula­ tive." The JC itself has suggested using 6 to 1 2 months as a period of time to include in the cumulative dose; however, they have not declared an official time period. If a patient has had multiple prolonged fluoroscopically guided proce­ dures within a time period of 6 months or less, it is best to consult with a medical physicist regarding the likelihood of skin injury with a subsequent fluoroscopic procedure and determination of a combined dose for determining the clas­ sification of a sentinel event. Stochastic Risk Stochastic effects are characterized by the absence of a threshold for expression. They are primarily governed by a statistical risk with the probability of onset increasing with increased radiation dose. There is an all or nothing expression of the effect. Although the likelihood increases with increased dose, the severity of the effect is unrelated to dose. The primary radiation quantity associated with stochas­ tic effects is the effective dose (ED) , commonly expressed in millisieverts (mSv) . Table 44. 3 outlines typical EDs asso­ ciated with various procedures that utilize ionizing radia­ tion. There is no fluoroscopic equipment currently available that displays an ED . The closest related quantity is the PKA ­ Again, PKA is not the ED and other factors must be taken into account to obtain the ED. For some specific procedure types, there are conversion factors that have been derived to convert PKA to an estimated ED.

647

The primary example of a stochastic effect is carcinogen­ esis. Although there is much debate surrounding exposure to low levels of ionizing radiation, the currently accepted model is the linear no-threshold (LNT) model of carcinogen­ esis. According to the most recent reports available, there is approximately a 10% increased risk of cancer incidence and approximately a 5% increase in cancer death per 1 ,000 mSv of ED above natural occurrence for adults. Pediatric lifetime risk is elevated by as high as a factor of 3 or 4 compared with adults depending on various factors. Pediatric risk is higher because on average children will live longer providing a longer poten­ tial period to clinically express any radiation-induced effects. R A D I AT I O N M A N AG E M E N T The ALARA principle states that radiation exposure to the patient and operator should be kept as low as reasonably achievable to minimize both the deterministic and stochastic effects, essentially limiting the amount of radiation used to that which is absolutely necessary under reasonable condi­ tions. In terms of operator and staff, the major contributor of exposure is scattered radiation from the patient and other objects in the path of the x-ray beam. Table 44.4 outlines some of the key components to a procedure that minimize patient radiation exposure. Patient Radiation Management Of the many specific parameters that affect patient dose, seven deserve special attention: 1.

Patient distance to the radiation source and distance to image receptor

2. Path length and approach of the x-ray beam 3. Dose and frame rate settings 4. Fluoroscopy time and the number of acquisitions

5.

6.

Use of magnification Collimation

7. Grid use

Patient Distance to the Radiation Sou rce and Distance to the I mage Receptor One of the easiest ways to reduce patient dose is to keep the image receptor as close to the patient as possible. Because of the inverse square law even a 1 0 cm gap can mean a 40% or greater difference in the dose to the patient. Ideally; the patient should be as far away from the x-ray source as possi­ ble; however, once we reach the surface of the image receptor, it does not serve much purpose to increase both the patient and image receptor distances from the source. Path Length and Approach of the X-Ray Beam Lateral, oblique, or other projections that extend the x-ray path length through the patient increase the dose in two

648

SECTION VII • CORONARY ARTERY DI SEASE AND CARD IAC CATHETERIZATION

[] C l eveland C l i n i c

Heart and Vascu lar I n stitute M i l ler Pavi l ion Card iovascu lar Labs Patient Fluoroscopic Doses C u m u lative Rad iation Dose ( G y) 2

5

10

Dose (m G y)

Notify Angiographer

2,000

Other Action --

5,000

1V1

1 0,000

vv

F/U Instructions* Consider m i n imizing digital acqu isitions, use fl uoro save

Instructions* Consider Stopping F/U

STOP I

*

**

Fol low- u p i n structi o n s : Patient wi l l receive post-p roced u re fo l low- u p i n struction sheet. Docu m e ntat i o n of Dose and counse l i n g with i n patient med ical reco rd Pati ent w i l l be advi sed to fo l l ow- u p with the a n g i og rapher in 3-6 months post-p roced u re . Senti n e l eve nt p roced u re : C l i n ical R i s k Management w i l l be notified

FIGURE 44.7 Protocol for i ncreasing doses of c u m u lative radiation in the cardiac catheterization la bo­

ratory. (Cou rtesy Cleveland C l i n ic.)

ways (as compared to a PA projection) . First, the more tissue in the path length, the more attenuation of the x-ray beam, which results in increased radiation output by the fluoro­ scopic system. As a general rule, approximately 25% of the x-ray beam is attenuated for each 1 cm of soft tissue tra­ versed. This does not mean that after 4 cm of soft tissue, there are zero photons left because entering the second centimeter of tissue was only 75% of the original beam. Therefore, after 4 cm of soft tissue, there is approximately (0. 75)4 0.32 of the original beam. Likewise, if we increase the path length through the patient by 4 cm, the system needs to increase its output by approximately 68% to compensate. Second, =

the longer the path length, the closer the skin entrance is to the x-ray source, which increases the radiation dose by the inverse square law. Dose and Frame Rate Settings The machine settings make a significant difference in the dose/dose rate delivered to the patient. Pulsed fluoroscopy can provide dose rates 25% to 50% lower than continuous fluoroscopy. Generally, 7. 5 to 10 pulses per second fluoros­ copy is recommended for coronary angiographic procedures. If the pulse rate is slower, temporal lag produces a stuttering effect in the image, whereas a higher pulse rate results in

CHAPTER 44 • RAD IATION SAFETY IN THE CARDIAC CATHETERIZATION LAB ORATORY TA B L E

Typical ED Esti mates for Va rious Procedures that Util ize Ionizing Radiation Proced u re

Chest x-ray Chest CT (standard) Chest CT (cardiac) Diagnostic coronary angiogram Percutaneous coronary intervention Radiofrequency ablation TIPS ERCP Tc-99 m cardiac perfusion Thallium cardiac perfusion

ED Estimate (mSv)

0.04 7.0 16 7.0 15.0 15.0 70.0 4.0 1 1 .4 16.9

unnecessarily high dose rates. The same concept applies to acquisition frame rates, the lower the frame rate, the lower the dose rate however the greater the temporal lag. Almost all fluoroscopic systems have the ability to choose between two or three different dose delivery modes. The dose rate mode can change the dose rate by 30% to 60% . The opera­ tor should choose the lowest dose mode available to start any procedure and then increase to a higher dose mode if there is too much noise in the image. Remember, the goal is to be able to perform the task at hand with the worst image

TA B L E

M i n i m izing Patient Radiation Exposu re in the Cardiac Catheterization Laboratory Ways to M i n i m ize Patient Radiation Exposure

• • • • • • • • • •

Limit foot pedal on-time Lower frame rate to 10 frames/s Limit number and duration of cine runs Substitute standard cine runs with low-dose acquisition Limit fluoroscopy time Work in shallow angulations when possible to avoid higher radiation doses Avoid lowering the table from isocenter Lower image detector as close to the patient as possible Use magnified views only when necessary Collimation

649

quality that will allow you to successfully complete the task. Better image quality almost always results from higher radia­ tion dose. Fluoroscopy Time and N u m ber of Acq uisitions Although it should be obvious, reducing the fluoroscopy time and the length and quantity of acquisitions will limit the amount of radiation delivered to the patient. Many systems now have the capability of recording fluoroscopic imaging sequences, which may allow for a reduction in acquisitions, thereby reducing patient dose. All modern fluoroscopic systems provide a last image hold feature allowing for the operator to stop x-ray production but still see the last image acquired. This feature can drastically reduce radiation doses if well utilized. Use of Magnification Magnification modes significantly increase the radiation dose; the greater the magnification, the greater the dose. The difference in dose rate between a 7 inches (-22 cm diagonal) field of view and a 5 inches (- 1 6 cm diagonal) field of view can be as high as a factor of 2 . The operator should always use the largest field of view possible while still being able to visualize the necessary anatomy and instrumentation. Collimation Although collimation does not generally affect the region of skin remaining in the field of view, it does reduce the overall amount of radiation absorbed by reducing the total quantity of tissue irradiated. Collimation reduces the PKA, which is the product of the K..r and the area of the field. The PKA is the closest related machine generated param­ eter to the ED . Therefore, collimation does not significantly affect deterministic effects; however, it does affect stochas­ tic effects. G rid Use All fluoroscopic systems are provided with a grid on the face of the image receptor. The purpose of a grid is to remove scatter radiation, increase the contrast, and improve image quality However, the down side of a grid is that it may absorb both scatter radiation and some primary radiation, which requires an increased output of the x-ray tube, thus increasing the patient dose. For petite patients and especially pediatric patients, it may be feasible to remove the grid, which will decrease the radiation dose to the patient; how­ ever, image contrast will suffer. Operator Radiation Ma nagement It is vitally important to first understand that the radiation dose to the operator is directly proportional to the radia­ tion dose of the patient. In fact, the patient is the primary

650

SECTION VII • CORONARY ARTERY DI SEASE AND CARD IAC CATHETERIZATION

M i n i m izing Operator Radiation Exposu re i n the Cardiac Catheterization La boratory Ways to M i n i mize Operator Radiation Exposure

Time • Reduce unnecessary fluoroscopy time Distance • Step back from the x-ray source and patient Angulation • Minimize LAO views • Minimize steep angulations Shielding • Leaded glasses • Leaded apron with leaded thyroid collar • Table skirt and moveable shields Acquisition mode • Substitute fluoroscopy recordings or low dose cine for standard cine

source of radiation exposure to the personnel in the lab. As a rule of thumb, the air kerma rate at 1 m from the patient is equal to 111 ,000th of the air kerma rate at the patient's skin entrance. Therefore, all the methods used to decrease the radiation exposure to the patient will in effect also reduce the exposure to the operator. In addition, there are ways an operator can further reduce his or her exposure. The cardinal principles of radia­ tion protection are time, distance, acquisition mode, and shielding (Table 44. 5). Scatter radiation to the operator is maximized in the LAO views and minimized in the RAO views when the operator is standing on the right side of a supine patient. By the inverse square law, the intensity of radiation decreases by the square of the distance from the source. Therefore, if we increase our distance from 1 m from the patient to 2 m from the patient, the air kerma rate will be approximately 1/4,000th that of the patient's entrance exposure (it will decrease by a factor of 4) . An operator can move farther away from the x-ray tube and patient by taking advantage of the catheter length, particularly during extrem­ ity cases and power injections. In general, shielding is a must in the catheterization laboratory. This includes a moveable acrylic shield, table skirt, leaded apron, thyroid collar, and leaded glasses. Lead aprons and thyroid collars are very effective at reducing radi­ ation exposure. Generally speaking, the operator is protected from 95% or more of the incident scattered radiation by the lead protective apparel. In addition, floor-standing shields are particularly useful in cases that utilize biplane angiog­ raphy to cover radiation scatter from the lateral x-ray tube.

The operator must carefully avoid inserting his hands in the direct beam of the x-ray tube. Leaded gloves may increase the dose to the patient and operator if they are introduced into the direct x-ray beam. It is also of utmost importance that all staff present during fluoroscopically guided proce­ dures wear dosimeters. Dosimeters are the only way that radiation management has to monitor the radiation doses to operators and staff. R E G U LATO RY R E C O M M E N DAT I O N S As stated earlier, ED (mSv) reflects stochastic risk in whole­ body equivalents by summing the weighted doses to each organ or tissue irradiated. The whole-body dose limits for radiation workers recommended by the National Council on Radiation Protection and Measurements (NCRP) are 50 mSv annually and the cumulative ED limit is 10 mSv times the operator's age in years (Table 44. 6) . These recommendations have been adopted by the Nuclear Regulatory Commission and all US states as annual occupational exposure limits. The NCRP in part bases its recommendations on scientific data from United Nations Commission on the Effects of Ionizing Radiation (UNCEIR) and the National Academy of Sciences in the Biological Effects of Ionizing Radiation (BIER) . Many of the US regulations are derived from NCRP reports and the recommended dose limits have been adopted into law. These recommendations are meant to ensure the cancer risk in radiation workers is that of workers in "safe" industries. Personal dosimeters are used to monitor the ED with either one or two badges. Recommendations on how to wear radiation badges have been published by the Society for

�-

TA B L E



Dose Limits Recommended by the NCRP

Recommended Dose Limits

Occupational exposures • ED • Equivalent annual dose for tissues and organs a. Eye b. Thyroid, skin, hands, and feet Embryo-fetus exposures • Total equivalent doses • Equivalent dose in 1 mo Public exposure Annual background radiation (average)

Annual: 50 mSv Cumulative: 1 0 mSv x age 1 5 0 mSv 500 mSv 5 mSv 0 . 5 mSv 1 mSv 3 . 6 mSv

CHAPTER 44 • RAD IATION SAFETY IN THE CARDIAC CATHETERIZATION LAB ORATORY Cardiovascular Angiography and Interventions. The badges most commonly used are either TLDs or optical stimulated luminescence ( OSL) dosimeters. Lithium fluoride crystals in the TLDs when heated emit light in direct proportion to the amount of radiation absorbed. Aluminum oxide crystals in the OSL dosimeters emit light after being stimulated by laser in direct proportion to the radiation absorbed. The natural background radiation from cosmic rays, rocks and soil, radon gas, and ingested food is on average 3 . 6 mSv per year in the United States. Given that the average chest x-ray results in 0.04 mSv, background radiation pro­ duces an ED that is equivalent to 90 chest x-rays per year. The average coronary interventional procedure can produce anywhere from 7 to 20 mSv. Skin injury risk is predictable and patient radiation exposure is monitored. If patient radiation doses approach concerning levels, the responsible physician must weigh the risks and benefits and continue only in cases when the potential benefit outweighs the potential risk. CO N C L U S I O N Basic rules o f radiation safety require the operator to min­ imize radiation exposure to the patient, him or herself, and to other personnel in the x-ray suite while obtaining adequate images for patient care. It is obviously impor­ tant for the operator to understand the main principles of radiation production, protection, and radiation-induced injury S U G G EST E D R E A D I N G S Edward L. Nickoloff, Keith] . Strauss, Bruce T. Austin, et al.

Cardiac Catheterization Equipment Performance. College Park: Ameri­ can Association of Physicists in Medicine (AAPM) ; 200 1 . Report 70. Stephen Balter, Marvin Rosenstein, Cindy L. O'Brien, et al.

Radiation Dose Management For Fluoroscopically-Guided Interven­ tional Medical Procedures. Bethesda: National Council on Radiation Protection & Measurements (NCRP) ; 2 0 1 0 . Report 1 68. Balter S , Hopewell J W, Miller D L , et al. Fluoroscopically guided interventional procedures: a review of radiation effects on patients' skin and hair. Radiology. 254(2) :326-34 1 .

65 1

Balter S , Moses ] . Managing patient dose in interventional cardiology Catheter Cardiovasc Interv. 2007;70(2) : 244-249 . Budoff M, Gupta M. Radiation exposure from cardiac imag­ ing procedures: do the risks outweigh the benefits? ] Am Coll Car­ diol. 2 0 1 0 ;56(9) : 7 1 2-7 14. Bushong S C . Radiologic Science for Technologists: Physics, Biol­ ogy, and Protection. 8th ed. St. Louis: Mosby; 2004. Chambers E, Fetterly K, Holzer R, et al. Radiation safety program for the cardiac catheterization laboratory. Catheter Cardio­ vasc Interv. 20 1 1 ;77:546-556. Chen ] , Einstein A, Fazel R, e t al. Cumulative exposure to ionizing radiation from diagnostic and therapeutic cardiac imag­ ing procedures: a population-based analysis. ] Am Coll Cardiol. 2 0 1 0 ; 5 6(9) : 702-7 1 1 . Fazel R, Krumholz HM, Wang Y, et al. Exposure to low-dose ionizing radiation from medical imaging procedures. N Engl ] Med. 2009;36 1 (9) 849-857 Gerber TC , Carr JJ , Arai AE, et al. Ionizing radiation in cardiac imaging: a science advisory from the American Heart Association Committee on Cardiac Imaging of the Council on Clinical Cardiology and Committee on Cardiovascular Imaging and Intervention of the Council on Cardiovascular Radiology and Intervention. Circulation. 2009 ; 1 1 9 (7) : 1 056- 1 065 . Hirshfeld JW, Balter S, Brinker JA, et al. American Col­ lege of Cardiology Foundation, American Heart Association, HRS, SCAI, American College of Physicians Task Force on Clinical Com­ petence and Training. ACCF/AHA/HRS/SCAI clinical competence statement on physician knowledge to optimize patient safety and image quality in fluoroscopically guided invasive cardiovascular procedures: a report of the American College of Cardiology Foun­ dation/American Heart Association/American College of Physi­ cians Task Force on Clinical Competence and Training. Circulation. 2005 ; 1 1 1 (4) 5 1 1-532 Jaco JW, Miller DL. Measuring and monitoring radiation dose during fluoroscopically guided procedures. Tech Vase Interv Radial. 2 0 1 0 ; 1 3(3) : 188-193. Mettler F, Huda W, Yoshizumi T, e t al. Effective doses in radiology and diagnostic nuclear medicine: a catalog. Radiology. 2008;248( 1 ) : 2 54. Mettler FA, Thomadsen B R , Bhargavan M, et al. Medical radiation exposure in the U . S . in 2006: preliminary results. Health Phys. 2008;95(5): 502-507. Miller DL, Balter S , Schueler B A , et a l . Clinical radiation management for fluoroscopically guided interventional procedures. Radiology. 20 1 0 ;257(2) :32 1-332.

QU ESTIONS AND ANSWERS Questions

1 . Which of these statements is TRUE rega rding radia­ tion dose term inology? a. Dose-a rea prod uct (DAP) reflects the total radia­ tion del ivered to the patient. b. Skin dose is measu red in u nits of Sieverts (Sv). c. Kerma reflects the biologic i m pact of different radiation types. d. Fluoroscopy time is a good measu re of radiation dose.

2. What is the g reatest sou rce of rad iation exposure to the operator? a. Flat-pa nel detector close to the operator b. Tu be leakage c. Scattered radiation from the patient d. RAO camera ang les 3. Which one of the fol l owing red uces air kerma rad ia­ tion dose at the reference plane? a. Col l i mation b. I ncreasing the source to i mage dista nce (Contin ued)

652

SECTION VII • CORONARY ARTERY DI SEASE AND CARD IAC CATHETERIZATION

c. C h a n g i n g the fi e l d o f view from 22 t o 1 6 c m d. Lowering the flat-pa nel detector t o the patient 4. Which of the fol l owing red uces OAP? a. b. c. d.

c. Anode d. Col l imator 1 0. Which statement below is TRUE?

Col l imation Lowering the table height Changing x-ray orientation from PA to AP I ncreasing the table height

a. b.

is eq ual to the patient's skin dose. is displayed on the mon itors in the cath­ eterization laboratory. c. Of the displayed radiation quantities, K.,, is closest related to determin istic effects. d. Of the displayed rad iation q u antities, PKA is closest related to stochastic effects. e. Both C and D

5 . Which of the fol lowi ng statements regarding sto­ chastic and determ inistic risks is TRUE? a. Determ i nistic risk is the major rad iation hazard posed to operators. b. For the same amount of rad iation expos ure, the stochastic risk is hig her in older operators than younger operators. c. A stochastic effect such as cancer cannot be pre­ dicted from a threshold level of radiation exposu re. d. A determin istic effect such as skin i nj u ry can not be pred icted from a threshold level of rad iation exposu re. 6. What is the maximal recom mended ED per year for an operator in the cardiac catheterization laboratory? a. b. c. d.

5 mSv 1 0 mSv 50 mSv 1 00 mSv

7. Which one of the fol lowi ng is TRU E regarding SCAI recom mendations rega rding personal dosimeter badges i n the cardiac catheterization laboratory? a. Wea r one badge i nside the thyroid col l a r and one badge at the waist outside the apron. b. Wea r one badge outside the thyroid col l a r and one badge at the waist outside the apron. c. Wea r another operator's badge if you forget you r own. d. Wea r one badge outside the thyroid col l a r. 8. Which of the fol lowi ng statements is FALSE rega rd­ ing rad iation-i nd uced skin i nju ry? a. Pea k skin dose is difficult to estimate because of varyi ng distance between the x-ray tube and the patient's back d u ring a procedu re. b. Radiation skin i nj u ry can manifest weeks to months after a proced u re. c. By the i nverse sq uare law, lowering the table height will decrease the skin dose (Gyt). d. There are m u ltiple ways to estimate skin dose including the use of rad iochromic fi l ms. 9. X-rays are generated at the: a. Generator b. Cathode

K.,,

o skin,max

Answers

1.

Answer A: Skin dose is estimated in u n its of Gray (Gy). Radiation from different sources can cause different deg rees of biologic damage for the same amount of en­ ergy absorbed and this is accou nted for by the equivalent dose. The ED is the sum of equivalent doses to each organ exposed to radiation and reflects stochastic risk. Fluoros­ copy time is not a good measure of radiation dose. 2. Answer C: The g reatest sou rce of rad iation exposu re to the operator is scattered radiation from the patient. The lead housing around the x-ray tube is meant to pre­ vent rad iation leakage. With respect to ca mera ang les, LAO views with the x-ray tube close to the operator on the right side of the patient are associated with higher operator exposu re than RAO views due to the i nverse sq uare law, which states that rad iation dose decreases the i nverse sq uare of the distance from the sou rce. 3. Answer D: Col limation red uces scatter radiation and

improves image q u a l ity. It does not red uce a i r kerma rad iation dose. I ncreasing the sou rce to i mage receptor distance and magnification of the field of view both in­ crease a i r kerma dose. Loweri ng the flat-pa nel detector does decrease the a i r kerma dose. 4. Answer A: Collimation reduces the area of tissue irradi­

ated without a change in the radiation beam intensity and therefore red uces OAP. OAP remains the same irrespective of table height or x-ray orientation and camera angle. 5. Answer C: Determin istic risk is the major hazard posed to patients from radiation-ind uced skin i nj u ry

and can be d i rectly correlated with pea k skin dose measurements. You nger operators theoretical ly have a higher stochastic risk for the same amount of radiation due to the longer potential to express that risk. There is no threshold level of radiation exposure with which a stochastic effect is guara nteed to occu r, althoug h the probability increases with hig her levels of exposu re. 6. Answer C: The whole-body dose l i m it for rad iation workers is 50 mSv a n n u a l ly. 7. Answer D: The recom mendations are if two badges are worn, one should be outside the thyroid col l a r and

CHAPTER 44 • RAD IATION SAFETY IN THE CARDIAC CATHETERIZATION LAB ORATORY

t h e second a t waist level under t h e apron. If o n e badge is worn, it should be outside the thyroid col l a r. You should never wea r another operator's badge. 8. Answer C: Lowering the table heig ht wi l l increase

the skin dose as the radiation source is closer to the skin.

9. Answer C: The cathode is made up of tungsten fila ments that become white hot from the current. The

653

electrons travel from the fila ments and strike the rotat­ ing a node, releasing the x-rays used for imaging.

1 O. Answer E: There is no fl uoroscope available that displays pea k skin dose measu rements and a i r kerma is j ust a n estimate.

Heinodynainic Measureinents James E. Harvey and Frederick A. Heu pler, Jr.

P H YS I C S O F P R E S S U R E M EAS U R E M E N T The most common method of measuring pressures in the cardiac catheterization laboratory is to use fluid-filled cath­ eter systems that convey the pressure wave from the site of interest through a catheter, manifold, and a pressure trans­ ducer that converts the pressure waveform to an electrical signal. A catheter with a pressure transducer at the tip pro­ vides a more accurate pressure recording, but these catheters are too expensive for routine clinical use. Fluid-filled catheters commonly produce several types of artifacts in recorded waveforms: 1 . Low-frequency response 2. Overshoot 3 . Zero level

Low-frequency response and overshoot are common to all types of fluid-filled pressure-transmitting devices. The natu­ ral resonant frequency of a catheter-manometer system is the frequency at which the system oscillates when stimu­ lated. The desirable frequency response for measuring intracardiac pressures in an adult with a fluid filled cath­ eter system is about 20 Hz or more. When the natural reso­ nant frequency response of a catheter system is below about 12 Hz, low-frequency catheter oscillation waves will obscure high-frequency cardiac waveforms. The operator should try to minimize the following factors that lower the frequency response of a catheter-manometer system: 1 . Air bubbles in the catheter system 2. High-viscosity fluid in the catheter (e.g. , contrast material

instead of saline) 3. Long fluid-filled tubing between the catheter and the pressure

transducer 4. A long catheter

5.

6.

A narrow-bore catheter A catheter made of soft, compliant material

654

Overshoot is produced by reflected waves within the catheter-manometer system. The magnitude of overshoot can be reduced by mechanical or electrical damp­ ing. Overdamping eliminates overshoot, but it reduces frequency response. Optimal damping reduces overshoot without producing a maj or drop in frequency response (Fig. 45 . 1 ) . The pressure transducer in a fluid-filled catheter system must be placed in a position equal to the mid-height atrial level to achieve the "zero level." This is approximately one­ half the distance between the front and the back of the chest in a supine patient. If the transducer is placed at the level of the anterior chest surface of a supine patient, the recorded pressures will be falsely low. Respiration produces cyclical changes in the absolute pressure of all intrathoracic cardiovascular structures. Pres­ sure measurement should be measured during end expira­ tion. The ultimate goal of setting up a fluid-filled catheter pressure measurement system is to achieve the highest fre­ quency response possible, optimally damp the system to eliminate overshoot, and locate the pressure transducer at the zero level. B A S I C I N T R A C A R D I AC WAV E F O R M S The basic configuration o f normal waveforms is similar for the right and left atria. The V-wave amplitude is generally greater than the A wave in the left atrium, whereas the A wave predominates in the right atrium (RA) . Electromechanical delay is about 40 to 80 milliseconds. The basic intra-atrial waveforms and the events to which they correspond are as follows: A: atrial contraction C: ventricular contraction V: rising atrial pressures during ventricular systole; occurs during the T wave C-V, or systolic: rapidly rising atrial pressure due to severe atrioven­ tricular valve regurgitation

CHAPTER 45 • HEMODYNAMIC MEASUREMENTS

655

Normal Val ues for l ntracardiac Pressu re

C: Overd a m ped.

FIGURE 45.1 A : U n derdam ped. B : Opti m a l ly d a m ped.

X descent: atrial relaxation; occurs after the A-wave peak, before the C wave X' descent: atrial relaxation; occurs after the C wave and before the V wave Y descent: opening of the atrioventricular valve; occurs after the peak of the V wave (Fig. 45.2).

Arrhythmias may produce a variety of changes in intracar­ diac pressures: 1 . Atrial fibrillation will eliminate A waves. 2. Junctional rhythm will displace A waves closer to the C wave. 3. Premature ventricular contractions (PVCs) and ventricular pace­

Chamber

Normal Pressure (mm Hg)

RA RV Pulmonary artery Left atrium

5 (±2) 25 (±5)/5 (±2) 25 (±5)/10 (±2) 10

the wedged catheter in the pulmonary arterial circuit and by the volume and compliance of the pulmonary venous circuit and left atrium. A stiff 7 French end-hole catheter (e.g. , Cournand) will wedge more distally in the pulmonary artery, with a mechanical time delay of about 70 to 80 milliseconds. A balloon-tipped catheter will wedge more proximally in the pulmonary artery branches, with a mechanical time delay up to 150 to 1 60 milliseconds. Proper wedging of the right heart catheter can be demonstrated by: 1.

A mean PCW pressure about 10 mm Hg lower than mean PA pressure

maker rhythm may produce cannon A waves in the atrium as a result of atrial contraction against a closed atrioventricular valve.

2. Blood withdrawn from the wedged catheter has an oxygen satu­

Normal values for intracardiac pressures are listed in Table 45 . 1 .

For the PCW to accurately reflect left atrial pressure, the pul­ monary artery catheter must be positioned in an area of the lung where the mean pulmonary capillary pressure (PJ exceeds the mean alveolar pressure (PA) . The lungs can be divided into three physiologic zones of blood flow. These zones are based upon the relative differences in alveolar pressure, mean pul­ monary artery pressure, and pulmonary capillary pressure:

Pul monary Ca pillary Wedge Pressu re Waveforms Pulmonary capillary wedge (PCW) pressures indirectly reflect left atrial pressures. PCW waveforms demonstrate a mechanical time delay, decreased amplitude, and decreased frequency response compared to simultaneously recorded left atrial waveforms (Fig. 45 . 3). The reason for these changes is the retrograde transmission of pressure waves from the left atrium through the pulmonary veins, capillaries, and arterioles to the wedged catheter in the pulmonary artery The mechanical time delay is determined by the location of

ration at least equal to arterial saturation.

S I M U LTAN EOUS LA + PCW

50

I FIGURE 45.2 Ti m i n g of the interatrial waveform with the electroca rd iogra m .

I

m m/s

25

T

mm/s

I

I

FIGURE 45.3 Electromechanical delay between left atrial wave­ form a n d pulmonary ca p i l l a ry wedge pressu re waveform.

656

SECTION VII • CORONARY ARTERY DI SEASE AND CARD IAC CATHETERIZATION

• Zone 1 (highest i n elevation) : This i s the part of the lung that is above the level where the alveolar pressure (PA) is equal to the pulmonary artery pressure (P,) , such that PA > P, > P, throughout this zone. There is minimal to no blood flow in this zone because the pressure exerted on the pulmonary artery by the alveoli prevents blood flow to the capillaries. • Zone 2 (middle elevation) : This region of the lung lies between where the pulmonary artery pressure equals the alveolar pres­ sure and where the alveolar pressure equals the pulmonary cap­ illary pressure; such that P, > PA > P,. •

Zone 3 (lowest in elevation) : This is the region of the lung that lies below the level where the alveolar pressure equals the pul­ monary capillary pressure, such that P, > P, > PA

If the pulmonary catheter is in an area where the alveolar pres­ sure is greater than the pulmonary capillary pressure, then the PCW waveform will be falsely elevated. Thus, the PCW pressure accurately reflects left atrial pressure only when the catheter is in zone 3 of the lung. In the catheterization labo­ ratory; a newly placed pulmonary artery catheter selectively advances to zone 3 , thereby assuring valid PCW measure­ ment. However, when in the intensive care unit, the patient can be repositioned or significant fluid changes can occur thereby changing the intrapulmonary hemodynamics. Thus it is often necessary to verify that the location of the cath­ eter tip still exhibits zone 3 physiology Marked respiratory variation in the PAWP tracing and a loss of the normal atrial pressure waveform suggest that the catheter is in a zone other than zone 3 . When using a balloon-tipped pulmonary artery catheter, withdrawing the catheter back to the right ventricle (RV) and then readvancing the catheter with the balloon tip inflated will usually reposition the catheter into zone 3 . Accurate PCW pressures may b e difficult to obtain in patients with near systemic levels of pulmonary hypertension. Characteristically, the PCW pressures may appear falsely ele­ vated, which may lead to a false impression of postcapillary pulmonary hypertension. In these cases, direct left ventricu­ lar and/or left atrial pressure measurement may be required to determine if the left heart pressures are truly elevated. Pressu re Wave Artifacts In addition to the artifacts that may be produced by low­ frequency response and overshoot, catheter structure or placement may introduce artifacts in pressure recordings. End-hole artifacts may occur when the tip of an end­ hole catheter becomes occluded during contraction of an atrial or ventricular wall. If this occurs during atrial systole with a catheter in the atrium, the A wave will appear greatly magnified (Fig. 45 .4). If end-hole occlusion occurs with a catheter in a pulmonary artery; the PCWP will appear falsely low and flat. Simultaneous recording of ventricular and aortic pres­ sures may occur when the tip of a pigtail catheter is located in the ventricle and the side port in the aorta. This may pro­ duce a bizarre-looking pressure wave with an apparently elevated diastolic left ventricular pressure (Fig. 45 . 5).

40

0 FIGURE 45.4 Fa lsely e levated A wave recorded from a n endhole catheter i n t h e r i g h t atri u m .

Catheter-whip artifact is a high-frequency oscillation that results from rapid movement of the catheter by blood flow. This is particularly likely to occur in the pulmonary artery and above a stenotic aortic valve. Card iac Output Measurement The fundamental function of the heart is to deliver enough blood to the systemic circulation to meet the oxygen demands of tissues. The normal cardiac output (CO) increases with body size and exercise and decreases with age. Numerous other factors may affect resting CO. In order to account for body size, the CO is normalized to body surface area (BSA) in square meters (m2) , and the result is the cardiac index. BSA may be obtained from a nomogram or calculated by the following formula: BSA (m ) 2

=

Ht (cm) x Wt (kg) 3, 600

The normal resting cardiac index falls from about 4. 5 Umin/ m2 at age 7 years to 3 Umin/m2 in middle age, and to 2 . 5 U min/m2 at age 70. The two maj or methods for measurement of CO in the cardiac catheterization lab are the Fick oxygen technique and the indicator dilution technique. The Fick Technique The Fick principle states that the total uptake or release of any substance (such as oxygen) by an organ (such as the lungs) is the product of blood flow to the organ and the arteriovenous

CHAPTER 45 • HEMODYNAMIC MEASUREMENTS 1 . Rest:Pressyre

Ca_loolatlons 1 1 : 1 6:31

I'

�-------... --...,._. --...c-� -�

�--+-+--+-----'+--+-+--+-+-+-

'tv 200

A

mm

Hg B

200

I� Summary: 92HR: LV 1 1 8/46/47 dP/dC828 dP/d1/P: 1 1 1 . Rest:Pressyre C81c\Jla!""1s 1 1 ; t 7:4 7

II

V 200 mm Hg B

1 80

'\ j\

i-----r----r--t--+--;-._+

8

I

I \\

i 1 ) \ l . m 1 l l \ I ILI \ ' 1

\--'-1 - --+-t--\---t :l� I __.._ , _

• . 1.���-$ymmary: 91 HA: LV t 1 4/1 4126 dP/dt: t 660 dP/d1/P :26

1\

-r--1 .._ I

Ji,

I

I

}\ A I

_,___U-+-_ ' w �

1---j.-j-

FIGURE 45.5 A: Pressure record ing from pigta i l catheter with end hole i n the left ventricle and side­

holes in the aorta revea l i n g a fa lsely elevated LVEDP. B: Pressu re record ing in same patient with pigta i l catheter completely i n left ventricle demonstrati ng t h e accu rate LVEDP.

160

I

140 1 20 100

2

657

658

SECTION VII • CORONARY ARTERY DI SEASE AND CARD IAC CATHETERIZATION

(A-V) concentration difference of the substance. If pulmo­ nary blood flow equals systemic blood flow, then 0 2 consumption CO = A V 0 2 difference -

Oxygen consumption can be estimated by measuring the oxygen uptake from room air by use of a Douglas bag or metabolic hood. In order to conserve time and expense, many laboratories use an assumed oxygen consumption based on the formula 125 mVmin/m2 for younger patients ( 1 1 0 mVmin/m2 for older patients) or 3 mVmin/kg. How­ ever, assumed oxygen consumption values may produce dis­ crepancies of ± 1 0% to 25% in about half of patients. A-V oxygen difference is obtained by subtracting the oxygen content of pulmonary venous (or systemic arterial) from pulmonary arterial (or "mixed venous") blood. Oxygen content may be calculated using the formula 0 2 content = 0 2 saturation x 1 . 36 (ml 0 2 /g Hb) x (g Hb/l 00 ml blood)

where Hb is hemoglobin. The final formula for calculation of CO then becomes

o2 consumption (mUmin) CO = ------��------------(systemic arterial o2 - mixed venous o2 saturation) x 1 . 36 Hb x 1 0

Estimation of pulmonary arterial oxygen content by using "mixed venous" blood from the venae cavae is less accurate. Mixed venous blood oxygen saturation is an estimation of what the pulmonary artery blood oxygen saturation would be if no shunt were present. This can be approximated by the following formula: 3 SVC + IVC . (while at rest) Mixed venous blood = 4

and Mixed venous blood =

SVC + 2IVC (while exercising) 3

where superior vena cava (SVC) and inferior vena cava (IVC) are the respective 02 saturations. Use of arterial blood to estimate pulmonary venous blood oxygen content is acceptable, because, in the absence of shunts, only a small amount of venous blood enters the arterial circuit within the heart via the Thebesian veins. Nar­ row A-V oxygen differences (as seen with high CO) are more likely to introduce error than wide differences (as seen with low CO) . Thus, the Fick method is most accurate in patients with low CO. Assume that a patient has the following measured values: Oxygen consumption = 250 mUmin

The mixed venous blood saturation will be 3 (O 7) + (0. 78) = 0. 72 4

-----

In this case, the CO can be calculated as 250 mUmin (0.97 - 0 72) x 1 . 36 x 14 x 1 0 2 = 5.25 Umin/m

CO =

-------­

Indicator Dil ution Methods The most commonly used indicator dilution method today is the thermodilution technique. This method utilizes a bolus injection of saline, followed by continuous measurement of the temperature of the blood by a thermistor in the pulmo­ nary artery The resulting curve is analyzed by computer to derive the CO using the basic indicator dilution equation. With this method, the temperature of the injectate (measured in the injectate fluid container before injection) is assumed to increase by a predictable amount during injection. Accurate measurement of both blood and injectate temperatures immediately before injection is important for measuring thermodilution COs. According to the formula for calculating thermodilution CO, the temperature dif­ ference between blood and injectate (typically 1 6°C when room-temperature injectate is used) is directly proportional to CO . Small errors in either of these measurements can pro­ duce errors in calculated CO. Thermodilution CO will be overestimated if the injec­ tate temperature is inappropriately increased by permit­ ting the injectate to remain in the syringe or by holding the syringe in the hand before and during injection. Use of cooled inj ectate, as opposed to room-temperature injectate, may produce an even greater mean error, probably because warming of the cooled injectate in the tubing and syringe produces an even greater increase in temperature than use of room-temperature injectate. Even though there is a theoreti­ cal advantage to iced injectate because of its greater signal­ to-noise ratio , most studies have shown no advantage to iced over room-temperature injectate. A dual-thermistor catheter appears to minimize these problems with injectate tempera­ ture, resulting in more consistent and accurate CO measure­ ments, but at increased expense. The thermodilution technique will overestimate CO in low-flow states because of warming of blood by the cardiac chambers. The thermodilution method is most accurate in high-flow states. It is unreliable in the presence of significant tricuspid regurgitation because the injectate is warmed dur­ ing its prolonged stay within the RA and RV Overall, the thermodilution method should have an error of no more than 5 % to 10% when performed correctly.

Femoral artery oxygen saturation = 97% SVC oxygen saturation = 70% IVC oxygen saturation = 78% Hb concentration = 14.0 g%

Shunt Ca lcu lation An intracardiac shunt is an abnormal communication between the left and right heart chambers. A left-to-right intracardiac

CHAPTER 45 • HEMODYNAMIC MEASUREMENTS shunt increases pulmonary blood flow in relation to systemic flow, and a right-to-left shunt does the opposite. Oximetry is the most common method for calculating intracardiac shunts in the catheterization laboratory; although dye dilution curves and angiography may also be used. Oxi­ metry is not as sensitive as dye dilution curves for detecting small shunts, but it should be capable of detecting any shunt that is large enough to merit surgical correction. Detailed oximetric analysis requires sampling in the RA (three sites) , SVC (high and low) , IVC (at renal artery level and below the diaphragm) , RV (three sites) , pulmonary artery; and aorta. When a left-to-right shunt exists at the atrial level, it is necessary to use the SVC and IVC oxygen saturations to calculate the mixed venous blood saturation, as described above. The same principle applies when a left-to-right shunt exists at the right ventricular level, especially when tricus­ pid regurgitation is present. A significant increase in oxygen saturation in the right side of the heart is considered to exist when there is > 7% increase from the SVC/IVC to the RA, >5% from the RA to the RV, and >5% from the RV to the PA. Left-to-right shunts are commonly expressed as Q/Qs. QP , or pulmonary flow, and � or systemic flow, are calculated ' using the formula given above for calculating CO. The A-V 02 difference for QP requires pulmonary arterial and pulmonary venous samples (or assumption of a pulmonary venous satu­ ration of 95%). The A-V 02 difference for Q.5 requires arte­ rial and mixed venous samples. A Q/Qs < 1 . 5 signifies a small left-to-right shunt, 1 . 5 to 2.0 an intermediate size, and >2 .0 a large shunt. A Q/Qs !+I

arterial pulse pressure of the sinus beat that follows a PVC is lower than the sinus beat that precedes the PVC (Fig. 45.9); with valvular AS, it is higher. In addition, both the Valsalva maneuver and nitroglycerin increase the gradient in HOCM, but decrease the gradient in valvular AS. Constrictive Physiology Pericardial tamponade and chronic constrictive pericardi­ tis are the two classic syndromes of constrictive physiol­ ogy. A third syndrome , effusive-constrictive pericarditis, has intermediate hemodynamic features. All three are characterized by diastolic dysfunction, with impaired atrial and ventricular filling patterns . Clinically, the dif­ ferentiation of pericardial tamponade and constrictive pericarditis is simple. However, the differentiation of con­ strictive pericarditis and restrictive cardiomyopathy may be much more difficult, even with the use of modern diag­ nostic tools. Perica rd ia I Ta m ponade

The classic features of pericardial tamponade include: • Elevation and equalization of right and left ventricular diastolic pressures and right and left atrial pressures (Fig. 45. 1 0) • Pulsus paradoxus, that is, exaggerated (> 1 0 mm Hg) inspiratory fall in arterial pressures • Prominent X descent with blunted Y descent • Arterial hypotension, as a late event

a PVC in patients with HOCM.

Pulsus paradoxus is a characteristic finding in pericardial tamponade, but it may be found in chronic obstructive pulmonary disease and rarely in pulmonary embolus and in constrictive pericarditis. Pulsus paradoxus in tampon­ ade is associated with narrowing of the pulse pressure dur­ ing inspiration, but the pulse pressure is normal in chronic pulmonary disease (Fig. 45. 1 1) . Pulsus paradoxus may be impossible to detect in a patient with an irregular rhythm such as atrial fibrillation.

20

Pul monary artery

Rlght1 ventricle

Right atrium

F I G U RE 45.1 0 Pressure tracing p u l l back from the pulmonary a rtery to the right atri u m demonstrati ng equalization of dias­ tolic pressu re seen in pericardia! ta m ponade.

msp

@

[)

·----

-----

s ---

[) ---s ----

©

[) -----

FIGURE 45.1 1

CHAPTER 45 • HEMODYNAMIC MEASUREMENTS

exp

Echocardiography is fairly sensitive for the detection of phases 2 and 3 of cardiac tamponade, characterized by right heart chamber collapse during diastole in the presence of pericardial effusion. However, the echocardiogram may fail to detect pericardial tamponade even in patients with phase 3 tamponade, in which the clinical findings are obvious on physical examination.

- - - - - - - - --

---

-

Con strictive Perica rd itis a n d Restrictive Card iomyopathy

The classic features of constrictive pericarditis include: • Elevation and equalization of diastolic pressures in all four car­ diac chambers • Deep, rapid Y descent (corresponding clinically to Friedreich sign-the abrupt collapse of the jugular vein during diastole)

- - -- -

• Attenuation of the X descent, which, in conjunction with the deep Y descent, produces an M or W configuration in the atrial tracing

----

• Elevation of the right atrial mean pressure during inspiration (corresponding clinically to Kussmaul sign-the elevation of jugular venous pressure with inspiration) • "Dip and plateau" pattern in right and left ventricular pressures - - -- - - - - - - - - - - -- - - - -

____

j ����-____

-----

• RVEDP > one-third the right ventricular systolic pressure (RVSP) • PA systolic pressure 5 m m 2 by IVUS has been shown to decrease the l i keli hood of ang io­ g raphic restenosis. c. Stent undersizing is a well-known predictor of stent thrombosis, and IVUS can help with choosing the optimal stent size based on vessel and l u men size. d. IVUS has played a very i m porta nt role in u nder­ standing the natura l history of atherosclerosis and the i mpact of cholesterol-lowering d rugs in red ucing the prog ression of atherosclerosis and even its reg ression. e. A minimal luminal area of 50% reduction in ST elevation to conservative therapy, repeat thrombolysis, or rescue PCI. At 6 months, patients treated with rescue PCI were significantly more likely to be free from death, reinfarction, stroke, or severe heart failure (adjusted hazard ratios of 0.43 ; p 0.001 for rescue PCI vs. repeated thrombolysis) . 2 5 In addition to the ECG require­ ment, ACC/AHA guidelines also support immediate PCI fol­ lowing the administration of fibrinolytic therapy for patients with recurrent MI (Class I indication) , moderate or severe ischemia (Class I indication) , cardiogenic shock/hemody­ namic instability (Class I indication) , LVEF < 40% (Class Ila indication) , and serious ventricular arrhythmias (Class Ila indication) (Table 4 7.3). Patients should be transferred to a hospital with PCI capabilities after the administration of fibrinolytics due to the numerous potential indications for PCI. In patients with successful fibrinolysis who are asymptomatic, earlier trials that predated the routine use of clopidogrel and GP Ilb/Illa inhibitors showed no evidence that immediate PCI of =

686

SECTION VII • CORONARY ARTERY DI SEASE AND CARD IAC CATHETERIZATION TA B L E

Recom mendations for the Use of PCI i n Patients with STEMI Class I Recommendations

1 . If immediately available, primary PCI should be performed in patients with STEMI (including true posterior MI) or MI with new or presumably new left bundle branch block who can undergo PCI of the infarct artery within 12 h of symptom onset, if per­ formed in a timely fashion (balloon inflation goal within 90 min of presentation) . (Level of Evidence: A) 2 . Primary PCI should be performed as quickly as possible, with a goal of a medical contact­ to-balloon or door-to-balloon time within 90 min. (Level of Evidence: B) 3 . Primary PCI should be performed in fibrinolytic-ineligible patients who present with STEMI within 12 h of symptom onset. (Level of Evidence: C) 4. Aspiration thrombectomy is reasonable for patients undergoing primary PCI. (Level of Evidence: B) Class Ila Recommendations

1 . It is reasonable to perform primary PCI for patients (even if fibrinolytic ineligible) with onset of symptoms within the prior 1 2-24 h and one or more of the following: a. Severe congestive heart failure (Level of Evidence: C) b. Hemodynamic or electrical instability (Level of Evidence: C) c. Evidence of persistent ischemia (Level of Evidence: C) Class I l l Recom mendations

1 . Elective PCI should not be performed in a non-infarct-related artery at the time of pri­ mary PCI of the infarct-related artery in patients without hemodynamic compromise. (Level of Evidence: C) 2 . Primary PCI should not be performed in asymptomatic patients more than 12 h after onset of STEM! who are hemodynamically and electrically stable. (Level of Evidence: C) Modified from the ACC/AHA guidelines: King SB III, Smith SC Jr, Hirshfeld JW Jr, et al. 2007 focused update of the ACC/AHA/SCAI 2005 guideline update for percutaneous coronary intervention: a report of the American College of Cardiology/American Heart Association Task Force on Practice guidelines. ] Am Coll Cardiol . 2008;5 1 : 1 72-209; Smith SC Jr, Feldman TE, Hirshfeld JW Jr, et al. ACC/AHA/SCAI 2005 Guideline Update for Percutaneous Coronary Intervention-summary article: a report of the American College of Cardiology/American Heart Association Task Force on Practice Guidelines (ACC/AHA/SCAI Writing Committee to Update the 200 1 Guidelines for Percutaneous Coronary Intervention) Circulation. 2006; 1 1 3 : 1 56-1 75 ; Kushner FG, Hand M, Smith SC Jr, et al. 2009 focused updates: ACC/ AHA guidelines for the management of patients with ST-elevation myocardial infarction (updating the 2004 guideline and 2007 focused update) and ACC/AHA/SCAI guidelines on percutaneous coronary intervention (updating the 2005 guideline and 2007 focused update) a report of the American College of Cardiology Foundation/American Heart As­ sociation Task Force on Practice Guidelines. ] Am Coll Cardiol . 2009 ;54:2205-224 1 .

the infarct-related artery provided any further reduction in death, reinfarction, or myocardial salvage (SWIFT and TIMI II) . Other trials such as SIAM III, GARCIA- 1 , and CAPITAL­ AMI showed significant reduction in mortality and ischemic events in those who underwent immediate PCI after suc­ cessful fibrinolysis. A larger, definitive trial has also provided evidence of benefit. The TRANSFER-AMI study randomized patients to either standard treatment (including rescue PCI) or immediate transfer to another hospital for PCI within 6 hours after fibrinolysis. 2 6 At 30 days, there was a signifi­ cant decrease in the number of patients with death, rein­ farction, recurrent ischemia, new or worsening congestive heart failure, or cardiogenic shock (relative risk with early PCI, 0.64; p 0.004) .This has been further clarified with =

results of a meta-analysis by D'Souza et al. , which showed that an early invasive PCI strategy following administration of fibrinolytic therapy results in a 53% reduction (odds ratio 0.47 [95% confidence interval (CI) , 0. 32-0.68, p < 0.000 1 ] ) in a combined endpoint o f 30-day mortality, reinfarction, and ischemia. The maj ority of the difference was in a significant reduction in both reinfarction and recurrent ischemia that 30-day mortality and maj or bleeding rates between strate­ gies were not significantly different. 2 7 While not yet reflected in the ACC/AHA guidelines, many providers now advocate routine angiography following the administration of fibrino­ lytic therapy; however, the optimal timing of PCI following the administration of fibrinolytic therapy remains unknown (Table 47.4) .

CHAPTER 4 7 • PERCUTANEOUS CO RONARY INTERVENTION

687

TA B L E Recommendations for the Use of PCI in Patients with Ca rdiogenic Shock Class I Recommendations

1 . Primary PCI should be performed for patients 20% from the preprocedural level. Chest pain following PCI is common and occurs in up to 40% of all patients. The maj ority of these patients do not develop evidence of periprocedural MI. Observational studies of post-PCI patients have found that the develop­ ment of Q waves or CK-MB >3 x ULN increases the risk of adverse outcomes and serious PCI-related complications (distal embolization, dissection, side branch occlusion, stent thrombosis) . As a result, some have suggested that periprocedural Mis are simply a reflection of patient comor­ bidities such as atherosclerotic burden and the complexity of the PCI . This has been illustrated by intravascular ultra­ sound studies that correlated the degree of atherosclerosis with the development of periprocedural MI. As such, efforts including the use of embolic protection devices when per­ forming PCI in saphenous vein grafts (that frequently have significant atherosclerotic/thrombotic burden) have shown the ability to reduce the incidence of periprocedural MI. In practice, the debate over the prognostic implications of periprocedural MI and its impact on therapy have kept many providers from routinely checking biomarkers follow­ ing PCI. Patients who have chest pain following PCI should have cardiac biomarkers checked as significant increases in combination with persistent pain and/or ECG changes may require repeat angiography or treatment with GP Ilb/Illa inhibitors. T E C H N I CA L A S P ECTS O F A N G I O G RA P H Y A N D P C I Covering all the technical aspects o f angiography and PCI in detail is beyond the scope of this book and the boards. The following sections attempt to cover the topics that may be seen on the general cardiology board exam. Access Catheterization originally was performed through a method known as the Sones technique that involved a cut-down of the brachial artery. As the indications for catheterization grew, accessing the vasculature through the femoral artery was found to be less invasive and associated with lower risks

to the patient. Femoral access replaced the Sones technique as the predominant method of catheterization over the past 20 years.78 Advances in catheter design and improvements in technique have made the routine use of catheterization via the radial artery feasible in routine clinical practice. Data from the NCDR CathPCI database have shown that radial artery catheterization is associated with lower odds of bleed­ ing complications at the expense of increased radiation exposure. While there is a learning curve associated with the adoption of catheterization from the radial artery; increased patient satisfaction and lower rates of vascular complications such as bleeding have increased its acceptance into clinical practice. Catheters Catheters commonly used to enter the left coronary cir­ culation are Judkins left QL) catheters and Amplatz left (AL) catheters. The JL catheter has a double curve with the length of the segment (in centimeters) between the first and second curves that determines the size of the catheter QL 3 . 5 , 4, 5, or 6 cm) . The AL catheter has a preshaped half-circle with a tip extending out of the curve and per­ pendicular to it, giving the catheter three curves. AL cath­ eters come in three sizes (0 . 5 , 0 . 7 5 , and 1 cm) based on the diameter of the half-circle/secondary curve. The increas­ ing use of radial artery catheterization has resulted in the development of new catheters, such as the Jacky and Tiger catheters, specifically designed for accessing the coronary arteries through the arm. These catheters have a double bend similar to the AL catheters but differ in the design of the tip. The Jacky (tip which points slightly outward) and Tiger catheter (straight tip) are able to engage both the left and right coronary artery by advancing the catheter into the aortic root and then manipulating the catheter into the dif­ ferent coronary ostia. During catheterization from the femoral artery, the JL4 is typically used as a starting catheter as it will be success­ ful in engaging the left coronary artery in the maj ority of patients. Patients with a large aortic root, tortuous aorta, or large body habitus may require a larger catheter QL5 , JL6) . Coronary artery anomalies may require the use of dif­ ferent catheters. For example, when the left main artery is located superiorly, a smaller JL catheter QL3 . 5) or AL cath­ eter is frequently used. Inferior origins of the left main may require a multipurpose catheter that has a downward tip . Short left main arteries or separate ostia for the left ante­ rior descending (LAD) and left circumflex are more easily accessed using AL catheters. The LAD artery can be cannu­ lated in these circumstances by clocking and advancing the catheter while the left circumflex artery can be accessed by counterclocking the catheter. The most common catheters used to enter the right coronary artery are the Judkins right QR) catheters, 3DRC, and Amplatz right (AR) . These cath­ eters can also be used to access saphenous vein grafts and the subclavian artery:

CHAPTER 4 7 • PERCUTANEOUS CO RONARY INTERVENTION Contrast Material Coronary angiography utilizes contrast agents that allow for the visualization of the coronary vasculature. Currently available contrast agents contain iodine that more readily absorbs x-rays as compared to the surrounding tissue. This differential absorption of radiation results in the ability of the contrast agents to provide contrast and visualization of the coronary arteries. Contrast agents are divided based on osmolality (high, low, isosmolar) (Table 4 7. 14) . The maj ority of contrast agents used in current practice are low osmolar or isosmolar because high-osmolar agents have been found to have more hypotension, myocardial depression, heart failure, and electrical abnormalities (bradycardia, QRS and QT-interval prolongation, ventricular fibrillation) . Prior ACC/AHA guidelines recommended the use of isosmolar contrast agents (i.e . , iodixanol) in patients with chronic kidney disease. This was based upon the RECOVER trial that randomized 300 patients with a creatinine clear­ ance 0. 5 mg/dl. Patients treated with iodixanol had significantly lower rates of CIN (7.9%) than patients treated with ioxaglate ( 1 7.0% ; p 0.02 1) .79 These recom­ mendations have since been modified after the publication of the CARE study This study randomized 482 patients with a creatinine clearance between 20 and 59 mVmin to iopamidol and iodixanol. Rates of CIN (defined as increase in serum creatinine �0.5) were similar in the two groups (4.4% after iopamidol and 6. 7% after iodixanol) suggesting no difference between the agents.so The lack of benefit with low osmolar agents was further supported with a subsequent =

701

meta-analysis.s 1 This analysis included 1 6 randomized controlled trials and 2 , 763 patients and did not find isosmo­ lar agents to be protective against CIN when compared to all low osmolar agents except ioxaglate and iohexol. As such, current ACC/AHA guidelines have been modified to state that patients with chronic kidney disease can be treated with either an isosmolar contrast agent or a low osmolar contrast agent other than ioxaglate or iohexol. Stents Initial attempts at percutaneous revascularization with angi­ oplasty were hindered by a high-rate acute vessel closure requiring cardiac surgery (often due to dissection) . In 1 993, the FDA approved the Gianturco-Roubin stent, making it the first coronary stents approved for routine clinical use.6 Since that time, coronary artery stenting has undergone many advances and now serves as the predominant form of coro­ nary revascularization. Initial stents were hindered by the proliferation of smooth muscle at the site of injury (known as restenosis). In response to this restenosis, stents coated with antiproliferative agents such as sirolimus, paclitaxel, everoli­ mus, and zotarolimus were developed. As compared to bare metal stents, drug-eluting stents (DESs) greatly reduce the risk of in-stent restenosis and target lesion revascularization. Some observational studies have suggested that drug­ eluting stents have a mortality benefit.S2 A mortality benefit has not been demonstrated in randomized data and is likely due to unmeasured differences in the two groups.s3 Given the con­ cern over stent thrombosis and the requirement that patients who receive DES be treated with DAPT for up to a year, it is important to understand the patient populations who ben­ efit most from drug-eluting stent implantation. Patients with

TA B L E Summary of the Various Types of Contrast Agents Used in Coronary Angiography

Class

Iodine (mg/ml)

Osmolal ity (m0sm/kg)0

Sod i u m (m Eq/L)

Sodium diatrizoate Sodium diatrizoate Sodium diatrizoate Ioxaglate

370 370 370 320

2 , 140 2 ,076 2 ,076 600

1 90 150 1 60 157

Iohexol Ioversol Ioxilan Iopamidol Iodixanol

350 320 350 200 320

844 702 695 413 290

Trace Trace Trace Trace Trace

Trade Name

Generic Name

MD 76 R Angiovist 370 Hypaque Hexabrix Omnipaque Optiray 320 Oxilan 350 Isovue 200 Visipaque 320

Ionic

High osmolar Low osmolar Nonionic

Low osmolar

Isosmolar

"Blood osmolality i s 275 mOsmol/L. Adapted from Askari AT, et al. , eds. Introductory guide t o cardiac catheterization. Philadelphia, PA: Lippincott Williams &: Wilkins.

702

SECTION VII • CORONARY ARTERY DI SEASE AND CARD IAC CATHETERIZATION

diabetes, small vessel diameter (:S:3.0 mm) , and long lesion length (:2:30 mm) are those patients who have consistently been shown to have the highest rate of restenosis and derive the most benefit from treatment with drug-eluting stents.84 AC K N OW L E D G M E N TS The authors would like to thank Soundos Moualla and Brendan Duffy, authors of this chapter in previous editions of the book, for their contributions and prior work. REFERENCES 1 . Roger VL, Go AS, Lloyd-Jones DM, et al. Heart disease and stroke statistics-20 1 1 update: a report from the American Heart Association. Circulation. 2 0 1 1 ; 1 2 3 : e l 8-e209. 2 . Cavender MA, Nicholls SJ , Lincoff AM. Strategies for the devel­ opment of new PPAR agonists in diabetes. Eur] Cardiovasc Prev Rehabil. 2 0 1 0 ; 1 7(suppl l ) : S32-S37 . 3 . King S B III, Smith SC Jr, Hirshfeld J W Jr, e t al. 2 0 0 7 focused update of the ACC/AHNSCAI 2005 guideline update for per­ cutaneous coronary intervention: a report of the American College of Cardiology/American Heart Association Task Force on Practice guidelines. ] Am Coll Cardiol . . 2008;5 l : 1 72-209. 4. Smith SC Jr, Feldman TE , Hirshfeld JW Jr, et al. ACC/AHN SCAI 2005 Guideline Update for Percutaneous Coronary In­ tervention-summary article: a report of the American College of Cardiology/American Heart Association Task Force on Prac­ tice Guidelines (ACC/AHNSCAI Writing Committee to Update the 200 1 Guidelines for Percutaneous Coronary Intervention) . Circulation. 2006; 1 1 3 : 1 56-1 7 5 . 5 . Kushner F G , Hand M, Smith S C Jr, e t al. 2009 focused updates: ACC/AHA guidelines for the management of patients with ST-el­ evation myocardial infarction (updating the 2004 guideline and 2007 focused update) and ACC/AHNSCAI guidelines on per­ cutaneous coronary intervention (updating the 2005 guideline and 2007 focused update) a report of the American College of Cardiology Foundation/American Heart Association Task Force on Practice Guidelines. ] Am Coll Cardiol. 2009;54:2205-224 1 . 6 . Roubin GS. Coronary stenting: the first 1 0 years. Eurointervention. 2006;2:40-44. 7. Epstein AJ , Polsky D , Yang F, et al. Coronary revascular­ ization trends in the United States, 200 1-2008. JAMA. 2 0 1 1 ;305 : 1 769-1 776. 8. An international randomized trial comparing four thrombo­ lytic strategies for acute myocardial infarction. The GUSTO investigators. N Engl ] Med. 1993;329: 673-682 . 9. Effectiveness of intravenous thrombolytic treatment in acute myocardial infarction. Gruppo Italiano per lo Studio della Streptochinasi nell'Infarto Miocardico ( GISSI) . Lancet. 1986 ; 1 :397-402 . 1 0 . Randomised trial of intravenous streptokinase, oral aspirin, both, or neither among 1 7 , 187 cases of suspected acute myo­ cardial infarction: ISIS-2 . ISIS-2 (Second International Study of Infarct Survival) Collaborative Group. Lancet. 1988;2: 349-360. 1 1 . Indications for fibrinolytic therapy in suspected acute myocar­ dial infarction: collaborative overview of early mortality and major morbidity results from all randomised trials of more than 1000 patients. Fibrinolytic Therapy Trialists'(FTT) Collab­ orative Group. Lancet. 1 994;343 : 3 1 1-322.

12. Gore JM, Granger CB, Simoons ML, et al. Stroke after throm­ bolysis. Mortality and functional outcomes in the GUSTO­ ! trial. Global Use of Strategies to Open Occluded Coronary Arteries. Circulation. 1995;92 : 2 8 1 1-2 8 1 8 . 1 3 . A clinical trial comparing primary coronary angioplasty with tissue plasminogen activator for acute myocardial infarction. The Global Use of Strategies to Open Occluded Coronary Ar­ teries in Acute Coronary Syndromes (GUSTO Ilb) Angioplasty Substudy Investigators. N Engl ] Med. 1 997 ;336: 162 1 - 1 628. 14. Keeley EC, Boura JA, Grines CL Primary angioplasty versus intravenous thrombolytic therapy for acute myocardial infarc­ tion: a quantitative review of 23 randomised trials. Lancet. 2003;36 1 : 1 3-20 . 1 5 . Sardella G, Mancone M , Bucciarelli-Ducci C, et al. Thrombus aspiration during primary percutaneous coronary intervention improves myocardial reperfusion and reduces infarct size: the EXPIRA (thrombectomy with export catheter in infarct-related artery during primary percutaneous coronary intervention) prospective, randomized trial. ] Am Coll Cardiol. 2009 ;53: 309-3 1 5 . 1 6 . Rodriguez A , Bernardi V, Fernandez M, e t al. In-hospital and late results of coronary stents versus conventional balloon angioplasty in acute myocardial infarction (GRAM! trial) . Gianturco-Roubin in Acute Myocardial Infarction. Am ] Car­ diol . 1998;8 1 : 1 286-1 29 1 . 1 7 . Antoniucci D , Santoro GM, Bolognese L , e t al. A clinical trial comparing primary stenting of the infarct-related artery with optimal primary angioplasty for acute myocardial infarction: results from the Florence Randomized Elective Stenting in Acute Coronary Occlusions (FRESCO) trial. ] Am Coll Cardiol . 1 998;3 1 : 1 234-1239. 1 8 . Stone GW, Brodie BR, Griffin JJ , et al. Prospective, multi­ center study of the safety and feasibility of primary stenting in acute myocardial infarction: in-hospital and 30-day results of the PAMI stent pilot trial. Primary Angioplasty in Myocar­ dial Infarction Stent Pilot Trial Investigators. ] Am Coll Cardiol. 1998;3 1 :23-30. 1 9 . Reynolds HR, Hochman JS. Cardiogenic shock: current con­ cepts and improving outcomes. Circulation. 2008; 1 1 7:686-697. 20. Goldberg RJ , Spencer FA, Gore JM, et al. Thirty-year trends ( 1 975 to 2005) in the magnitude of, management of, and hospital death rates associated with cardiogenic shock in patients with acute myocardial infarction: a population-based perspective. Circulation. 2009; 1 1 9 : 1 2 1 1-1 2 1 9 . 2 1 . Hochman J S , Sleeper LA , Webb J G , e t al. Early revasculariza­ tion in acute myocardial infarction complicated by cardiogenic shock. SHOCK Investigators. Should We Emergently Revas­ cularize Occluded Coronaries for Cardiogenic Shock. N Engl ] Med 1999;34 1 : 625-634. 22. Alexander KP, Newby LK, Armstrong PW, et al. Acute coronary care in the elderly, part II: ST-segment-elevation myocardial in­ farction: a scientific statement for healthcare professionals from the American Heart Association Council on Clinical Cardiol­ ogy: in collaboration with the Society of Geriatric Cardiology. Circulation. 2007; 1 1 5 :2570-2589 . 23. Ellis SG, Tendera M, de Belder MA, et al. Facilitated PCI in patients with ST-elevation myocardial infarction. N Engl ] Med. 2008 ;358:2205-22 1 7 . 2 4 . Califf RM, O'Neil W, Stack RS , e t al. Failure of simple clini­ cal measurements to predict perfusion status after intravenous thrombolysis. Ann Intern Med. 1988 ; 1 08: 658-662 .

CHAPTER 4 7 • PERCUTANEOUS CO RONARY INTERVENTION 2 5 . Gershlick AH, Stephens-Lloyd A, Hughes S, e t al. Rescue an­ gioplasty after failed thrombolytic therapy for acute myocardial infarction. N Engl ] Med. 2005 ;353:2758-2768. 26. Cantor WJ, Fitchett D , Borgundvaag B, et al. Routine early angioplasty after fibrinolysis for acute myocardial infarction. N Engl ] Med. 2009;360 2 705-2 7 1 8 . 27. D'Souza S P, Mamas MA, Fraser D G , Fath-Ordoubadi F Routine early coronary angioplasty versus ischaemia-guided angioplas­ ty after thrombolysis in acute ST-elevation myocardial infarc­ tion: a meta-analysis. Eur Heart ]. 20 1 1 ;32 :972-982 . 28. Roe MT, Messenger JC, Weintraub WS, et al. Treatments, trends, and outcomes of acute myocardial infarction and percutaneous coronary intervention. ] Am Coll Cardiol. 2 0 1 0;56: 254-263. 29. Bavry AA, Kumbhani DJ , Rassi AN , et al. Benefit of early in­ vasive therapy in acute coronary syndromes: a meta-analysis of contemporary randomized clinical trials. ] Am Coll Cardiol. 2006;48: 1 3 1 9- 1 3 2 5 . 30. Mehta S R , Granger C B , Boden WE , et a l . Early versus delayed invasive intervention in acute coronary syndromes. N Engl ] Med. 2009;360 : 2 1 65-2 1 75 . 3 1 . Antman EM, Cohen M, Bernink PJ, e t al. The TIMI risk score for unstable angina/non-ST elevation MI: A method for prognostication and therapeutic decision making. JAMA. 2000;284: 835-842 . 32. Campeau L Letter: Grading of angina pectoris. Circulation. 1 9 76;54: 522-523. 33. Coronary angioplasty versus medical therapy for angina: the second Randomised Intervention Treatment of Angina (RITA2) trial. RITA-2 trial participants. Lancet. 1997;350:46 1-468. 34. Henderson RA, Pocock SJ, Clayton TC, et al. Seven-year out­ come in the RITA-2 trial: coronary angioplasty versus medical therapy ] Am Coll Cardiol . 2003;42 : 1 1 6 1-1 1 70 . 3 5 . Davies RF, Goldberg AD , Forman S, e t al. Asymptomatic Cardiac Ischemia Pilot (ACIP) study two-year follow-up : outcomes of patients randomized to initial strategies of medical therapy ver­ sus revascularization. Circulation. 1 997;95:2037-2043 . 36. Boden WE, O'Rourke RA, Teo KK, et al. Optimal medical ther­ apy with or without PCI for stable coronary disease. N Engl ] Med. 2007;356: 1 503- 1 5 1 6 . 37. Shaw l], Berman D S , Maron DJ, e t al. Optimal medical therapy with or without percutaneous coronary intervention to reduce ischemic burden: results from the Clinical Outcomes Utilizing Revascularization and Aggressive Drug Evaluation (COURAGE) trial nuclear substudy Circulation. 2008; 1 1 7: 1283- 1 29 1 . 38. Patel MR, Dehmer GJ, Hirshfeld JW, e t al. ACCF/SCAI/STS/ AATS/AHA/ASNC 2009 Appropriateness Criteria for Coronary Revascularization: a report by the American College of Cardiol­ ogy Foundation Appropriateness Criteria Task Force, Society for Cardiovascular Angiography and Interventions, Society of Thoracic Surgeons, American Association for Thoracic Sur­ gery, American Heart Association, and the American Society of Nuclear Cardiology Endorsed by the American Society of Echocardiography, the Heart Failure Society of America, and the Society of Cardiovascular Computed Tomography. ] Am Coll Cardiol. 2009 ;53: 530-553. 39. Poldermans D , Hoeks SE , Feringa HH. Pre-operative risk as­ sessment and risk reduction before surgery ] Am Coll Cardiol. 2008;5 1 : 1 9 1 3-1 924. 40. The Bypass Angioplasty Revascularization Investigation (BARI) Investigators. Comparison of coronary bypass surgery with an­ gioplasty in patients with multivessel disease. N Engl ] Med. 1996;335 2 1 7-225 .

703

4 1 . Serruys PW, Morice MC, Kappetein AP, e t al. Percutaneous coro­ nary intervention versus coronary-artery bypass grafting for se­ vere coronary artery disease. N Engl ] Med. 2009;360:96 1-972 . 42 . Leon MB, Baim DS, Popma JJ , et al. A clinical trial compar­ ing three antithrombotic-drug regimens after coronary-artery stenting. Stent Anticoagulation Restenosis Study Investigators. N Engl ] Med. 1 998;339 : 1 665-1 6 7 1 . 43 . Savi P, Herbert JM. Clopidogrel and ticlopidine: P2Y l 2 adenos­ ine diphosphate-receptor antagonists for the prevention of ath­ erothrombosis. Semin Thromb Hemost. 2005 ;3 l : 1 74-183. 44. Steinhubl SR, Berger PB, Mann JT, 3rd, et al. Early and sus­ tained dual oral antiplatelet therapy following percutaneous coronary intervention: a randomized controlled trial. JAMA. 2002;288 :24 1 1-2420 . 45. Cavender MA , Rao S V Bleeding associated with current thera­ pies for acute coronary syndrome: what are the mechanisms? ] Thromb Thrombolysis. 2 0 1 0;30:332-339 . 46. Wiviott SD, Braunwald E, McCabe CH, et al. Prasugrel versus clopidogrel in patients with acute coronary syndromes. N Engl ] Med. 2007;357:2001-2 0 1 5 . 4 7 . James S, Akerblom A, Cannon C P, e t al. Comparison o f ticagrelor, the first reversible oral P2Y(l2) receptor antagonist, with clopi­ dogrel in patients with acute coronary syndromes: Rationale, de­ sign, and baseline characteristics of the PLATelet inhibition and patient Outcomes (PLATO) trial. Am Heart]. 2009; 157: 599-605 . 48. Wallentin L, Becker RC, Budaj A, et al. Ticagrelor versus clopi­ dogrel in patients with acute coronary syndromes. N Engl J Med. 2009;36 1 : 1 045- 1 0 5 7 . 49 . Gaglia MA, Jr. , Waksman R. Overview of the 2 0 1 0 Food and Drug Administration Cardiovascular and Renal Drugs Ad­ visory Committee meeting regarding ticagrelor. Circulation. 20 1 1 ; 1 23 45 1-456 . 50. Mehta SR, Bassand J P, Chrolavicius S , e t al. Dose comparisons of clopidogrel and aspirin in acute coronary syndromes. N Engl ] Med. 2 0 1 0;363:930-942 . 5 1 . Campbell CL, Smyth S, Montalescot G, et al. Aspirin dose for the prevention of cardiovascular disease: a systematic review. JAMA. 2007;297 2 0 1 8-2024. 52. Harrington RA, Califf RM. Late ischemic events after clopido­ grel cessation following drug-eluting stenting should we be worried? ] Am Coll Cardiol. 2006;48:2592-2595. 5 3 . Marni L, Kereiakes DJ, Normand SL, et al. Rationale and design of the dual antiplatelet therapy study, a prospective, multicenter, ran­ domized, double-blind trial to assess the effectiveness and safety of 12 versus 30 months of dual antiplatelet therapy in subjects undergoing percutaneous coronary intervention with either drug­ eluting stent or bare metal stent placement for the treatment of coronary artery lesions. Am Heart]. 2010;160: 1035-104 1 , 4 l e l . 54. Bhatt D L , Topol EJ. Current role of platelet glycoprotein Ilb/Illa inhibitors in acute coronary syndromes. ]AMA. 2000;284 : 1 549-1558. 5 5 . Giugliano RP, White JA, Bode C, et al. Early versus delayed, provisional eptifibatide in acute coronary syndromes. N Engl ] Med. 2009;360 : 2 1 76-2 1 9 0 . 56. Winchester DE, Wen X, Brearley WD, et a l . Efficacy and safety of glycoprotein Ilb/Illa inhibitors during elective coro­ nary revascularization: a meta-analysis of randomized trials performed in the era of stents and thienopyridines. ] Am Coll Cardiol. 2 0 1 1 ;5 7 : 1 1 90-1 1 9 9 . 5 7 . Stone GW, Witzenbichler B , Guagliumi G, e t a l . Bivalirudin during primary PCI in acute myocardial infarction. N Engl ] Med. 2008;358 22 1 8-2230.

704

SECTION VII • CORONARY ARTERY DI SEASE AND CARD IAC CATHETERIZATION

58. Van't H o f AW, Ten Berg J , Heestermans T , e t al. Prehospital initiation of tirofiban in patients with ST-elevation myocardial infarction undergoing primary angioplasty (On-TIME 2): a multicentre, double-blind, randomised controlled trial. Lancet. 2008 ;372 537-546. 59. Stone GW, Mclaurin BT, Cox DA, et al. Bivalirudin for patients with acute coronary syndromes. N Engl] Med. 2006 ;355:22032216. 6 0 . Ferguson JJ , Califf RM, Antman E M , e t al. Enoxaparin v s un­ fractionated heparin in high-risk patients with non-ST-segment elevation acute coronary syndromes managed with an intend­ ed early invasive strategy: primary results of the SYNERGY ran­ domized trial. JAMA. 2004;292:45-54. 6 1 . Peterson ED, Dai D, Delong ER, et al. Contemporary mortality risk prediction for percutaneous coronary intervention: results from 588,398 procedures in the National Cardiovascular Data Registry ] Am Coll Cardiol. 2010;55 : 1 923-1 932 . 62. Singh M, Rihal CS, Selzer F, et al. Validation of Mayo Clinic risk adjustment model for in-hospital complications after per­ cutaneous coronary interventions, using the National Heart, Lung, and Blood Institute dynamic registry. ] Am Coll Cardiol. 2003;42: 1 722-1 728. 63. Singh M, Rihal CS, Lennon RJ , et al. Bedside estimation of risk from percutaneous coronary intervention: the new Mayo Clinic risk scores. Mayo Clin Proc. 2007;82 : 70 1-708 . 64. Rihal CS , Textor SC, Grill DE, et al. Incidence and prognostic importance of acute renal failure after percutaneous coronary intervention. Circulation. 2002 ; 1 0 5 :2259-2264. 65. Pannu N , Wiebe N, Tonelli M. Prophylaxis strategies for con­ trast-induced nephropathy JAMA. 2006;29 5 : 2765-2779. 66. ACT Investigators. Acetylcysteine for Prevention of Renal Outcomes in Patients Undergoing Coronary and Peripheral Vascular Angiography: Main Results From the Randomized Acetylcysteine for Contrast-Induced Nephropathy Trial (ACT) . Circulation. 2 0 1 1 ; 124: 1 250-1259. 67. Cavender MA, Rao SV, Ohman EM. Maj or bleeding: manage­ ment and risk reduction in acute coronary syndromes. Exp Opin Pharmacother. 2008;9: 1 869- 1 883. 68. Rao SV, O'Grady K, Pieper KS, et al. Impact of bleeding sever­ ity on clinical outcomes among patients with acute coronary syndromes. Am ] Cardiol. 2005;96: 1 200-1206. 69. Eikelboom JW, Mehta SR, Anand SS, et al. Adverse impact of bleeding on prognosis in patients with acute coronary syn­ dromes. Circulation. 2006; 1 14: 774-782 . 70. Rao SV, Jollis JG, Harrington RA, et al. Relationship of blood transfusion and clinical outcomes in patients with acute coro­ nary syndromes. JAMA. 2004;292 : 1 555-1562. 71. Wang TY, Xiao L, Alexander KP, et al. Antiplatelet therapy use after discharge among acute myocardial infarction patients with in-hospital bleeding. Circulation. 2008 ; 1 1 8 : 2 1 39-2 145. 72. Spencer FA, Moscucci M, Granger CB, et al. Does comorbidity ac­ count for the excess mortality in patients with major bleeding in acute myocardial infarction? Circulation. 2007; 1 1 6:2793-280 1 . 7 3 . Mehran R , Rao SV, Bhatt DL, e t al. Standardized bleeding defi­ nitions for cardiovascular clinical trials: a consensus report from the bleeding academic research consortium. Circulation. 2 0 1 1 ; 123 2736-274 7 . 74. Steg P G , Huber K, Andreotti F, e t al. Bleeding i n acute coronary syndromes and percutaneous coronary interventions: position paper by the Working Group on Thrombosis of the European Society of Cardiology. Eur Heart ]. 2 0 1 1 ;32: 1 854-1864.

75. Cannon CP, Battler A, Brindis RG, et al. American College of Car­ diology key data elements and definitions for measuring the clin­ ical management and outcomes of patients with acute coronary syndromes. A report of the American College of Cardiology Task Force on Clinical Data Standards (Acute Coronary Syndromes Writing Committee) . ] Am Coll Cardiol. 200 1 ;38:2 1 14-2 130. 76. Thygesen K, Alpert JS, White HD. Universal definition of myo­ cardial infarction. Eur Heart ]. 2007;28 :2525-2538. 77. Locca D , Bucciarelli-Ducci C, Ferrante G, et al. New universal definition of myocardial infarction applicable after complex percutaneous coronary interventions? ]ACC Cardiovasc Interv. 2 0 1 0 ; 3 : 9 50-958. 78. Rao SV, Ou FS, Wang TY , et al. Trends in the prevalence and outcomes of radial and femoral approaches to percutaneous coronary intervention: a report from the National Cardiovas­ cular Data Registry. ]ACC Cardiovasc Interv. 2008 ; 1 :379-386. 79. Jo SH, Youn TJ , Koo BK, et al. Renal toxicity evaluation and comparison between visipaque (iodixanol) and hexabrix (ioxa­ glate) in patients with renal insufficiency undergoing coronary angiography: the RECOVER study: a randomized controlled trial. ] Am Coll Cardiol. 2006;48: 924-930. 80. Solomon RJ , Natarajan MK, Doucet S , et al. Cardiac Angiography in Renally Impaired Patients (CARE) study: a randomized double-blind trial of contrast-induced nephrop­ athy in patients with chronic kidney disease. Circulation. 2007 ; 1 1 5 : 3 1 89-3 1 9 6 . 8 1 . Reed M, Meier P, Tamhane UU, et al. The relative renal safety o f iodixanol compared with low-osmolar contrast media: a meta­ analysis of randomized controlled trials. ]ACC Cardiovascular Interv. 2009 ;2: 645-654. 82 . Douglas PS, Brennan JM, Anstrom KJ, et al. Clinical effec­ tiveness of coronary stents in elderly persons: results from 262,700 Medicare patients in the American College of Cardi­ ology-National Cardiovascular Data Registry ] Am Coll Cardiol. 2009 ;53 1 629- 1 64 1 . 83 . Kirtane AJ, Gupta A , Iyengar S , e t al. Safety and efficacy of drug-eluting and bare metal stents: comprehensive meta-anal­ ysis of randomized trials and observational studies. Circulation. 2009 ; 1 1 9 : 3 1 98-3206. 84. Stone GW, Parise H, Witzenbichler B, et al. Selection criteria for drug-eluting versus bare-metal stents and the impact of routine angiographic follow-up : 2-year insights from the HO­ RIZONS-AMI (Harmonizing Outcomes With Revasculariza­ tion and Stents in Acute Myocardial Infarction) trial. ] Am Coll Cardiol . 2 0 1 0;56 : 1 597-1604. 85. Yang X, Alexander KP, Chen AY, et al. The implications of blood transfusions for patients with non-ST-segment eleva­ tion acute coronary syndromes: results from the CRUSADE National Quality Improvement Initiative. ] Am Coll Cardiol. 2005;46 : 1 490-1495. 86. Nikolsky E , Mehran R, Dangas G, et al. Development and vali­ dation of a prognostic risk score for major bleeding in patients undergoing percutaneous coronary intervention via the femo­ ral approach. Eur Heart]. 2007;28 : 1 936-1945 . 87. Manoukian SV, Feit F, Mehran R, et al. Impact of major bleed­ ing on 30-day mortality and clinical outcomes in patients with acute coronary syndromes: an analysis from the ACUITY Trial. ] Am Coll Cardiol. 2007;49 1 362-1 368. 88. Inhibition of the platelet glycoprotein IIb/IIIa receptor with tirofiban in unstable angina and non-Q-wave myocardial infarction. Platelet Receptor Inhibition in Ischemic Syndrome

CHAPTER 4 7 • PERCUTANEOUS CO RONARY INTERVENTION Management i n Patients Limited b y Unstable Signs and Symptoms (PRISM-PLUS) Study Investigators. N Engl ] Med. 1998;338 : 1 488-1497. 89. Moscucci M, Fox KA, Cannon CP, et al. Predictors of maj or bleed­ ing in acute coronary syndromes: the Global Registry of Acute Coronary Events (GRACE). Eur Heart]. 2003;24 : 1 8 1 5-1823.

705

90. Kinnaird TD , Stabile E, Mintz GS , et al. Incidence, predictors, and prognostic implications of bleeding and blood transfusion following percutaneous coronary interventions. Am ] Cardiol. 2003 ;92: 930-935.

QU ESTIONS AND ANSWERS Questions

1 . A 73-year-old man underwent percuta neous coronary i ntervention (PCI) after a non-ST-elevation myocard ial i nfa rction (NSTEM I). He was pretreated with 325 mg of aspirin and 600 mg of clopidogrel, plus biva l i rudin d u ring the intervention. A DES is deployed i nto the mid-left a nterior descending (LAD). Fol lowi ng the proced ure, which of the follow­ ing anti platelet reg imens is most appropriate? a. Continue 325 mg aspirin and 75 mg clopidog rel for 1 2 months. b. Continue 8 1 mg aspirin for l ife and clopidogrel 75 mg for 6 months. c. Continue 8 1 mg aspirin for l ife and clopidogrel for at least 1 2 months. d. Continue 8 1 mg aspirin for 1 2 months and clopi­ dogrel 75 mg for 6 months. 2. A 62-year-old woman with stable angina has pre­ sented for elective PCI to a known foca l stenosis in a moderate-sized fi rst d iagonal branch. She is not ta k­ ing medications and has a severe a l lergy to shel lfish (h ives), and she also reports a n a l lergy to aspirin a lthoug h she can not specify the reaction. Which of the fo l l owing statements rega rding patient man­ agement is correct? a. Ad m i n i ster 1 00 mg hyd rocortisone IV, 50 mg diphenhyd ramine (Benadryl) IV, and 600 mg clopidogrel ora l ly before the proced u re, and then proceed with the sched u led intervention. b. Ad m i n i ster 40 mg methyl pred nisolone IV, 50 mg diphenhyd ramine (Benadryl) IV, 600 mg clopi­ dogrel orally, and proceed with the sched uled intervention. c. Ad m i n i ster 1 00 mg hyd rocortisone IV, 50 mg diphenhyd ramine (Benadryl) IV, 325 mg aspirin, and 300 mg clopidog rel ora l ly before the proced u re, and then proceed with the sched u led intervention. d. Consult a n allergy specialist and postpone the intervention.

3. A SO-yea r-old man presents to the emergency room (ER) a bout 1 00 m i n utes after the onset of chest pa i n . The pain is su bsternal, rad iates to the left a rm, and is associated with vom iting and diaphoresis. On exa m i nation, the patient is tachycardic with a heart rate of 1 04 and hypertensive with blood pressu re of 1 50/88. An electroca rd iog ra m showed 3 m m ST elevation i n t h e a nterior a n d latera l leads. Sym p­ toms were not relieved with sublingual n itrog lyc­ erin or beta-blocker thera py. An IV n itroglycerin d ri p was titrated to control the patient's sym ptoms. The closest i nterventional ca rdiology center is 1 20 m i n utes away. The patient has asked about the thera peutic options at this poi nt. What can you tel l h i m a bout t h e va rious thera peutic modalities? a. Primary PCI is recom mended over thrombolyt­ ics beca use PCI has short-term morta lity benefit, red uced reinfa rction risk, and red uced risk of stroke, regard l ess of d u ration of sym ptoms and time req u i red to PTCA. b. Throm bolysis followed by tra nsfer to center with PCI capabil ities c. Half-dose throm bolysis should be recom mended; then the patient should be transferred for PCI. d. Thrombolysis. Tra nsfer to center with PCI capa­ bil ities if chest pa i n and ECG changes do not resolve with i n 3 hours. 4. A 62-year-old woman, with ongoing tobacco abuse, diabetes, hypertension, and hyperlipidem ia, pres­ ents to the ER with a 2-day history of i ntermittent chest pa i n . On exa m i n ation, she is tachycard ic with a heart rate of 1 0 1 beats/mi n (bpm) and hyperten­ sive with a blood pressure of 1 60/1 00 m m Hg. The patient does not have a ny signs of heart fai l u re. An electroca rd iogram showed 2-mm depression in V2 -V6 . The patient is sta rted on heparin and g iven IV metoprolol and n itroglycerin. In addition, she was sta rted on a n eptifibatide i nfusion. The patient remai ned asym ptomatic u ntil her coronary ang io­ g ra m . The left heart catheterization showed 90% stenosis in the proximal third of the LAD and 80% (Continued)

706

SECTION VII • CORONARY ARTERY DI SEASE AND CARD IAC CATHETERIZATION

stenosis i n t h e proximal RCA, with 70% disease i n t h e middle ci rcu mflex a rtery.The best p l a n o f treat­ ment for this patient is: a. Complete revascu la rization with m u ltivessel PCI b. Referra l for th ree-vessel CABG c. PCI to the LAD with staged i ntervention to the RCA and LCx d. Medical ma nagement with no PCI or CABG 5. A previously hea lthy 78-yea r-old male presents to the emergency depa rtment (ED) complaining of chest pain and shortness of breath. His i n itial blood pressu re is 79/40 and his heart rate is 1 1 7 bpm. His i n itial ECG shows 3 m m ST elevations i n V 1 -V4 and his chest x-ray is nota ble for pul monary edema. What is the next best treatment option? a . Medica l ly sta bilize with IV d i u retics and a nti­ throm botic agents. Consider revascu larization once medica l ly sta bil ized. b. Fibrinolytic thera py c. U rgent angiography with goal to provide revascu larization therapy d. I ntu bate and place Swan Ganz catheter to help determi n e etiology of i l l ness. Place intra-aortic ba l l oon pump if the patient a ppears to have cardiogenic shock with low card iac i ndex and elevated pul monary ca pillary wedge pressu re. 6. A 56-year-old female with hypertension, obesity, ob­ structive sleep apnea, and fibromyalgia is undergoing angiography for chest pain. She describes her chest pai n as stabbing in nature. It occurs mostly when per­ forming housework or in periods of increased stress. It is not associated with shortness of breath. Her primary care physician evaluated her with a nuclear stress test that showed no evidence of ischemia; however, she continued to have this discomfort. The patient was scheduled for angiography to provide a definitive workup for her persistent chest pain. Angiography was notable for a 60% stenosis in the left circumflex artery and mild diffuse coronary artery disease in the LAD artery and right coronary artery. The most appropriate management is: a . Medical therapy with aspirin and stati n as there is no ind ication for revascu la rization. b. Beg i n secondary prevention medications and sched ule repeat stress test i n 1 year. c. PCI to the left circu mflex d. Perform FFR of the left circu mflex. 7. An 85-year-old female with a creatinine of 1 .6 is u ndergoing ang iography after a positive stress test. Which of the fol lowi ng i nterventions red uces her risk of contrast-ind uced nephropathy (CI N)?

a. Admission the night before catheterization for N-acetylcysteine and sodium bicarbonate infusion b. Biplane angiography and iodixanol contrast c. Hyd ration with normal sa line prior to the catheterization d. Disconti n uation of lisinopril 1 week prior to angiography 8. A 67-yea r-old male with dia betes and hyperten­ sion presents with three episodes of chest pain at rest over the past 24 hours. He is cu rrently chest pa in-free. His home medications include aspirin, l isinopril, and simvastatin. He contin ues to a buse tobacco. His fi rst set of card iac bioma rkers is no­ table for a tropon i n of 0.94 and a CK-MB of 54. The next step i n the ma nagement of this patient is: a. Give 325 mg of aspirin and 600 mg of clopi­ dogrel and beg i n i nfusion of eptifibatide and hepa ri n. Sched u l e angiography as soon as pos­ sible (urgent, not emergent). b. Give 325 mg of aspirin and 600 mg of clopidogrel and begin infusion of heparin. Schedule stress test once cardiac enzymes begin to decline. c. Give 325 mg of aspirin and 600 mg of clopidogrel and begin infusion of heparin. Schedule angiog­ raphy as soon as possible (u rgent, not emergent). d. Give 325 mg of aspirin and 600 mg of clopi­ dogrel and beg i n i nfusion of eptifi batide and hepa ri n. Sched ule stress test once card iac enzymes beg i n to decli ne. 9. Which of fo l l owing is not a relative contraind ica­ tion for the use of prasugrel in patients with acute coronary syndrome undergoing PCI? a. Weight 0. 7 cm, and central displacement of intimal calcium (Fig. 48.5). At times, intra­ mural echolucencies representing noncommunicating pock­ ets of fresh blood can be seen. Distinguishing intramural hematoma from severe atheroma, a thrombosed false lumen, or aneurysm with mural thrombus can be difficult. Angiog­ raphy is of limited diagnostic accuracy in the evaluation of hematomas, as it fails to image the aortic wall. If the clini­ cal history is concerning, a negative TEE should not repre­ sent the final diagnostic evaluation. CT and MRA represent highly accurate imaging modalities that are frequently used as an initial or complementary study in the evaluation of hematomas. Intramural hematomas can communicate with the adven­ titial space, lead to rupture, or progress to overt dissection

714

SECTION VIII • AO RTA/PERIPHERAL VAS CULAR D ISEASE

FIGURE 48.5

TEE i n both long- a n d short-axis views showing a n i ntra m u ra l hematoma, characterized by no d issection fla p, a crescent-shaped thicken ing of the aortic wa ll, centra l displacement of intimal calcium, a n d echolucent intra m u ra l pockets representi ng i ntra m u ra l blood.

with an intimal tear. However, they may also have a more benign course and gradually resolve with medical therapy and blood pressure control. Penetrating Aortic U lcer Penetrating aortic ulcer exists when an atheromatous plaque erodes inward into the aortic media. The advanced athero­ sclerotic disease burden prevents the erosion from extending longitudinally along the vessel as in classic dissection. The

ulcer is apparent on imaging modalities as an ulcer crater or contrast-filled outpouching. Depending on how far into the aortic wall the plaque erosion occurs, there may be formation of an intramural hematoma, saccular aneurysm, pseudoa­ neurysm, or even complete aortic rupture (Fig. 48.6). Clinical Presentation Patients with these acute aortic syndromes often present with the same chest and/or back pain as do patients with

AORTIC ATH E ROSCLEROTIC ULCERS lntima -----:oo. Media ---'== Adventitia Aortic atheroma

I ntimal plaque u lceration

Medial hematoma

Plaque u lceration

Adventitial false aneurysm

Transmural rupture

FIGURE 48.6 Schematic of a penetrating aortic u lcer a n d the prog ression to the va rious aortic wa l l complications.

CHAPTER 48 • DISEASES OF THE AO RTA

71 5

classic dissection. They may be associated with a higher inci­ dence of rupture than seen for classic dissections. Compared to intramural hematomas, patients with penetrating ulcers are usually older and tend to have more atherosclerotic burden. Isolated intramural hematomas can occur in both the ascending and descending aorta, whereas intramural hematomas associated with penetrating aortic ulcers are more commonly located in the descending aorta, where the atherosclerotic process is more common. Management As in aortic dissection, anti-impulse medical therapy should be initiated as soon as the diagnosis of a dissection variant is considered. Intravenous beta-blockade initially and, if needed for blood pressure control, sodium nitroprusside are the treatment agents of choice. For the dissection variants involving the ascending aorta, prompt surgical intervention is considered the treat­ ment of choice. However, some data suggest that select patients with intramural hematomas in the ascending aorta, particularly if small (< 1 1 mm) and with nondilated aortas, can be managed medically Recent data suggest that pene­ trating ulcer-like findings in an area of intramural hematoma can identify high-risk individuals. Symptoms of sustained or recurrent pain or findings of an increasing pleural effu­ sion are suggestive of disease progression and favor surgical intervention. Guidelines and management strategies for this patient population are still evolving. For the dissection variants that involve the descending aorta, especially intramural hematoma without penetrating ulcers, medical therapy is the preferred initial treatment. How­ ever, some have argued that there should be a lower thresh­ old for surgical intervention than for classic distal dissection, particularly when clinical signs of instability are present. The presence of a severely bulging hematoma or a deeply pen­ etrating ulcer may warrant surgical repair. The development of a saccular aneurysm or pseudoaneurysm should merit consideration for surgical repair. For those treated medically, serial imaging studies are warranted to assess for progression or increase in aortic diameter, in which case surgical repair or stent-graft placement may be considered. AO RT I C A N E U RYS M An aortic aneurysm is present when there is dilatation of the aorta, typically at least 1 .5 times its normal reference dimension for an adjacent segment. This dilatation may involve the entire circumference of the aortic wall (fusiform) or a localized pro­ trusion of one of the walls (saccular). Ectasia is characterized by dilatation < 1 . 5 times the normal reference dimension. Thoracic Aortic Aneu rysm The incidence of thoracic aortic aneurysm (TAA) is estimated at 5 . 9 cases per 100,000 patient years. Leading etiologies include congenital bicuspid aortic valve, Marfan syndrome

FIGURE 48.7 TEE in a long-axis view i l l u strati ng an ascend i n g TAA, with predominant d i l atation at the level o f the sinuses, i n a patient with Ma rfa n synd rome.

(Fig. 48. 7) , idiopathic annuloaortic ectasia, familial TAA syndrome, inflammatory aortitis, acquired due to increased age and hypertension, syphilis, and trauma. Descending TAA may extend distally and involve the abdominal aorta creating a thoracoabdominal aneu­ rysm. Patients are often asymptomatic at the time of pres­ entation, and the TAA may be diagnosed by an imaging modality ordered for other clinical indications. Physical findings may likewise be absent. When signs and symp­ toms do manifest, they are often the result of mass effect. The enlarging aorta may compress nearby structures such as the superior vena cava, the trachea, esophagus, and recurrent laryngeal nerve . This may result in superior vena cava syndrome , stridor, dysphagia, and hoarseness , respectively. Progressive dilatation of the aortic root can lead to aortic insufficiency, which can produce symptoms of congestive heart failure. Enlargement of the aortic sinuses can lead to narrowing of the coronary artery ostia, which can lead to myocardial ischemia and even infarction. Blood flow can be static in large aneurysms, predis­ posing to thrombus formation and distal embolization, a process which can be seen in descending thoracic and thora­ coabdominal aneurysms. N o n i nva sive I ma g i n g • TAA are often noted incidentally o n chest x-ray a s mediastinal widening or a prominent aortic knob . • Transthoracic echocardiography is the most common modality to initially diagnose and monitor dilatation of the aortic root. • CT scanning and MRA are the preferred techniques to accu­ rately define the entire thoracic aorta and its branch vessels and precisely measure TAA.

Because the thoracic aorta may be tortuous, care must be given to not measure off-axis axial cuts, as these can

716

SECTION VIII • AO RTA/PERIPHERAL VAS CULAR D ISEASE

overestimate the true cross section as compared to the actual orthogonal diameter. As suggested in the guidelines, measurements of aortic diameter should be made using the internal diameter when taken by echocardiography and the external diameter when taken by CT imaging or MRA. Medica l Treatment

There are data that beta-adrenergic blockade can slow the rate of thoracic aneurysm expansion in patients with Marfan syndrome, resulting in improved survival. Although the data are extrapolated to those without Marfan syndrome, it seems reasonable to recommend such therapy while TAA patients are being followed medically More recent studies have sug­ gested that angiotensin II receptor blockers may slow the rate of aortic dilation in Marfan patients; prospective investi­ gation using these agents is underway Recognizing that patients treated with beta-adrenergic blockade can still manifest aortic dilatation is important, as serial evaluation and imaging is required. Ma rfa n Synd rome, Thoracic Aortic Aneu rys ms, a n d Preg na ncy • Women with Marfan syndrome have an increased risk of aortic dissection during pregnancy, particularly during the third trimester. • The risk of dissection greatly increases if the aortic root diameter is >4. 0 cm or if there is evidence of rapid aortic root dilatation during pregnancy • If elective surgical repair is not performed prepartum, beta­ adrenergic blockade should be used during pregnancy, particu­ larly during the third trimester and peripartum. • Close echocardiographic follow-up and cesarean delivery should be considered if the aortic root size exceeds 4.0 cm or rapid aortic dilatation is evident.

I n d ications for S u rgical Treatment

Dissection and rupture are the feared complications of TAA, and prevention of these conditions is the purpose for elec­ tive surgical aortic repair. Size is a clear risk factor and prin­ cipal harbinger for dissection and rupture. In one series, the annual rate of dissection or rupture was 2 % for TAAs < 5 cm, 3 % for TAAs between 5 . 0 and 5 . 9 cm, and 7% for TAAs > 6 cm. Therefore, prophylactic surgical intervention should be considered before a TAA reaches a size that predis­ poses to aortic instability Although the optimal timing of prophylactic surgery remains uncertain, recommendations for surgical repair are >5 .0 to 5 . 5 cm for an ascending TAA and >6.0 to 6.5 cm for a descending TAA. Patients with Marfan syndrome, Loeys­ Dietz syndrome, bicuspid aortic valve, or family history of premature aortic instability should be considered for ear­ lier repair (perhaps at 4.5 to 5 . 0 cm and 5 . 5 to 6.0 cm for ascending and descending TAAs, respectively) . For patients undergoing aortic valve surgery, concomitant ascending aor­ tic repair should be considered for a TAA > 4.5 cm.

Rapid enlargement of the aorta (>0 . 5 to 0 . 75 cm/ year) or symptom development has also been advocated as indications for surgery The decision for operative repair must of course take into account the patient's medi­ cal comorbidities, and a risk/benefit ratio must be indi­ vidualized for each patient. Patients who are otherwise low medical risk may be considered for intervention at smaller aortic sizes. Abdom inal Aortic Aneu rysm • The incidence of abdominal aortic aneurysm (AAA) is estimated at 36.5 per 1 00 ,000 person years. • AAA represents the most common form of arterial aneurysm. • The maj ority of AAAs are infrarenal in location (75%). • Atherosclerosis is the dominant risk factor in the development of an AAA. Additional risk factors associated with AAAs are male gender (AAA is four to five times more common in men) , increasing age, smoking, and hypertension. • There is a clear familial predisposition to AAA, with relatives of affected patients having up to 25% increased risk for the devel­ opment of an AAA.

Asymptomatic AAA is often diagnosed on physical examination by abdominal palpation. The most com­ mon symptom is pain, and is usually steady The pain may be localized abdominal pain, or may radiate to the back, flank, or groin. Sudden onset of severe abdominal and back pain suggests rupture, representing a surgical emergency Up to only a third of patients with rupture will present with the classic triad of pain, pulsatile abdomi­ nal mass, and hypotension. Atheroemboli may be the first manifestation of an AAA .

N o n i nvasive I ma g i n g

Ultrasonography; CT scanning, aortography, and MRA have all been used in the initial diagnosis, sizing, and monitor­ ing of AAA. Ultrasonography represents the most practical method of screening and serial monitoring, while CT scan­ ning and MRA remain superior in accurately detailing the morphology and extent of the AAA. At initial diagnosis, the rate of dilatation cannot be determined and thus the next serial study should be per­ formed in 6 months. In general, for AAAs 5.0 cm, imaging every 3 to 6 months. Baseline AAA size is the best predictor of rate of dilatation. Larger aneurysms expand at higher rates than smaller ones. Medica l Treatment

Beta-adrenergic blockade with careful control of hyperten­ sion appears to have impact on delaying the rate of AAA expansion. Smoking should be discontinued, as rupture risk is greater among active smokers.

CHAPTER 48 • DISEASES OF THE AO RTA

71 7

I n d ications for S u rgical Treatment

Mortality from an AAA is primarily related to rupture. As with thoracic aneurysms, increasing size is the harbinger of rupture risk. Aneurysms 5 cm in size have a 22% risk of rupture over 2 years, with those >6 cm showing the sharpest rise in risk. As such, an aortic diameter of 5 . 0 to 5 . 5 cm is recommended as an indication for pro­ phylactic surgery in asymptomatic AAA patients. Although AAAs are less common in women, when they are present they are at greater risk of rupture and at smaller aortic diam­ eters than in men. Thus, it is recommended that women undergo prophylactic AAA repair at 4.5 to 5 . 0 cm. Aneurysms that expand rapidly (>0. 5 to 1 . 0 cm/year) are also associated with an increased risk of rupture, and are thus considered for elective surgical repair. Inflammatory AAA is present in up to 10% of cases. There appears to be a familial tendency for these, and they often occur in the context of smoking. Patients will present with constitutional symptoms and have an elevated sedi­ mentation rate in addition to the classic symptoms of pain. CT scanning or MRA can identify the inflammatory compo­ nent. Treatment is aortic surgery A relatively recent therapeutic option for AAA repair is the percutaneous placement of an endovascular stent graft. The endovascular stent graft is placed within the aneurysmal segment of the aorta , bridging the normal segments and excluding the aneurysm. However, just over half of all AAA possess anatomy favorable for stent­ graft placement. Data are still forthcoming on the long-term success of endovascular stent grafting. In randomized trials thus far, it appears that endovascular repair incurs a lower operative mortality compared to open AAA surgery, but no benefit in total mortality in the long term has been demonstrated. Nonetheless, the procedure remains an attractive alterna­ tive to conventional surgical repair, but is usually limited to patients with significant comorbid medical conditions who are at high surgical risk. Endovasc u l a r Stent-G raft Repa i r

AT H E R O M ATO U S AO RT I C D I S EA S E • Atherosclerotic plaques in the aorta can give rise to cerebral and peripheral embolic events (Fig. 48 . 8) . • TEE, in particular, has been a valuable imaging modality in assessing the presence, composition, and extent of these plaques. • Plaques >4 mm in thickness, or those with mobile or ulcerated components, appear to be strongly associated with subsequent embolic events.

Treatment strategies for patients with such plaques have not been evaluated in sufficient numbers in a prospective randomized fashion. However, there is evidence that lipid­ lowering therapy with a statin is a reasonable treatment

FIGURE 48.8 TEE in a short-axis view identifyi ng a protruding thick (>4-mm) atheroma i n the descend i n g thoracic aorta.

option, and anticoagulation with warfarin or antiplatelet therapy may benefit some patients. Earlier reports of a potential association between war­ farin and the cholesterol embolization syndrome have pro­ duced some reluctance to use such anticoagulant therapy in these patients, and further study is thus needed. The potential role of aortic replacement or removal of atheroma remains to be defined. It has become increasingly common for cardiac sur­ geons to assess the aorta before the institution of cardiopul­ monary bypass. The presence of significant plaque may alter the cross-clamp site or may even lead to endarterectomy or aortic replacement at the time of surgery Cholesterol Embol ization Synd rome The cholesterol embolization syndrome can be seen in patients undergoing diagnostic angiography, but can also occur spontaneously There is a reported association between warfarin anticoagulation and these events. The syndrome represents a showering of emboli, typi­ cally from the descending aorta. Patients most often present with the skin findings of livedo reticularis and blue toes, in the presence of palpable pulses. Renal insufficiency may occur, and may not be reversible. Transient eosinophilia is often present, and treatment is supportive. If the atheroma arose from an AAA, then surgical inter­ vention can help prevent future events. I N F LA M M ATO RY AO RT I T I S Giant Cel l Arteritis Giant cell arteritis is an inflammatory disease that affects the temporal arteries, producing local tenderness and head­ aches. Patients affected are typically over the age of 55 years, and women are affected twice as frequently as men.

71 8

SECTION VIII • AO RTA/PERIPHERAL VAS CULAR D ISEASE

The most devastating consequence is blindness. Although temporal arteritis is the hallmark of this disorder, there may be involvement of the thoracic aorta and the great vessels. This can lead to branch vessel occlusion, aneurysm formation, or even dissection. Corticosteroid treatment is the mainstay of therapy. With the development of advanced aortic involvement, surgical treatment may be required. Ta kayasu Arteritis Takayasu arteritis is an inflammatory disorder of the aorta that typically affects women under age 40 years. Its prev­ alence is greater in Asian and African populations than in those of European or North American descent. A subacute inflammatory illness phase is manifested by constitutional symptoms. Later, there is occlusive inflammation of the aorta and branch vessels, with segmental narrowing apparent. Symptoms of arterial insufficiency will be present, depending on the vessels involved. Acquired coarctation can occur, leading to hypertension, as can aneurysm formation. Treatment is corticosteroids. For occlusive lesions that do not respond to steroids, surgical bypass may be warranted. Syphilitic Aortitis Syphilitic aortitis represents a manifestation of tertiary syph­ ilis, which may occur 1 0 to 30 years after the initial infec­ tion. This inflammation results in a weakening of the vessel wall and can lead to aneurysm formation, usually saccular. Syphilitic aortitis most commonly affects the ascending aorta, and hence can result in aortic insufficiency. The arch may also be affected. Involvement of the descending aorta occurs less often. Other I nflammatory Aortitis Aortitis can also be seen in other systemic inflammatory diseases such as reactive arthritis, ankylosing spondylitis, rheumatoid arthritis, Wegener granulomatosis, and entero­ pathic arthropathies. A common genetic underpinning of these conditions is the HLA-B27 genotype, which should be considered in cases of lone aortic regurgitation, ascending aortic dilatation, and conduction system disease. Treatment involves addressing the underlying disorder, with surgery as needed for aneurysmal or aortic valvular complications. Mycotic Aneu rysms Bacteremia (from endocarditis, trauma, intravenous drug abuse) can result in infection within the weakened aneurysmal arterial wall. Persistent fevers after treatment of the inciting event should raise concern for an infected aneurysm. Mycotic aneurysms more commonly involve the abdom­ inal aorta. Atheromatous plaques can also become infected (bacterial aortitis) , serving as a nidus for infection requiring prolonged antibiotic therapy.

E S S E N T I A L FACTS Aortic Dissection • •



• •



• •

The hallmark of aortic dissection is an intimal flap. Increasing aortic size and aneurysm formation is a harbinger of aortic dissection. Proximal (ascending) aortic dissections are treated with surgery. In cases of cardiac tamponade, evacuation of hematoma should be performed in the operating room under cardiopulmonary bypass support. Distal (descending) aortic dissections are treated medically, with surgery guided by a complication-specific approach. Congenital bicuspid aortic valve, Marfan syndrome, Loeys­ Dietz syndrome, prior aortic surgery, and the peripartum period represent risk factors for aortic dissection in the young. A negative surface echocardiogram, absence of pulse deficits, or a normal mediastinum on chest x-ray does not exclude the presence of aortic dissection. Anti-impulse medical therapy with intravenous beta-blockade followed by sodium nitroprusside is the mainstay of medical treatment.

I ntra m u ral Hematoma and Penetrating Aortic Ulcer •

• •



Penetrating aortic ulcers arise more commonly in areas of ather­ omatous disease such as the thoracoabdominal aorta. Penetrating aortic ulcers that involve the ascending aorta are treated surgically: Intramural hematomas that involve the ascending aorta are generally treated surgically, although recent publications have raised some controversy and suggest that medical management may be an option in some populations. Neither of the aortic dissection variants involves an intimal dissection flap.

Aortic Aneu rysm •



Indications for surgery: Symptoms • Inflammatory or infectious • Rapidly expanding 0 . 5 cm/year, even if asymptomatic • >5 . 0 to 5 . 5 cm diameter for ascending thoracic • >6 .0 to 6.5 cm diameter for descending thoracic • >5 . 0 to 5 . 5 cm diameter for abdominal •

Earlier surgical intervention (>4 . 5 to 5 . 0 cm) is recommended in Marfan syndrome, Loeys-Dietz syndrome, and bicuspid aor­ tic valve patients.

• Beta-adrenergic blockade may slow the progression of aortic dilatation.

Atheromatous Aortic Disease •

Mobile, ulcerated, or thick atheromatous plaques (>4 mm) identified by TEE are associated with embolic events.

C:: J:I APTER 48 S U G G EST E D R E A D I N G S Coady MA, Rizzo JA, Elefteriades JA. Developing surgical . mtervention criteria for thoracic aortic aneurysms. Cardiol Clin. 1999 ; 1 7:827-839 . Coady MA, Rizzo JA, Elefteriades JA. Pathologic variants o f thoracic aortic dissections. Penetrating atherosclerotic ulcers and intramural hematomas. Cardiol Clin. 1 999 ; 1 7 : 63 7-657. Coady M A , Rizzo J A , Goldstein LJ , et a l . Natural history, pathogenesis, and etiology of thoracic aortic aneurysms and dissec­ tions. Cardiol Clin. 1999; 1 7 : 6 1 5-635 . Daily PO , Trueblood HW, Stinson E B , e t al. Management o f acute aortic dissections. Ann Thorac Surg. 1970; 1 0 :237-247 . Hagan P G , Nienaber C A , Isselbacher E M , e t al. The Inter­ national Registry of Acute Aortic Dissection (IRAD): new insights into an old disease. JAMA. 2000;283 :897-903 . Hiratzka LF, Bakris GL, Beckman JA, et al. ; American Col­ lege of Cardiology Foundation/American Heart Association Task Force on Practice Guidelines; American Association for Tho­ racic Surgery; American College of Radiology; American Stroke Association; Society of Cardiovascular Anesthesiologists; So­ ciety for Cardiovascular Angiography and Interventions; Soci­ ety of Interventional Radiology; Society of Thoracic Surgeons; Society for Vascular Medicine. 2 0 1 0 ACCF/AHNAATS/ACR/

ASNSCNSCAI/SIR/STS/SVM Guidelines for the diagnosis and management of patients with thoracic aortic disease. A Report of the American College of Cardiology Foundation/American Heart

• DISEASES OF THE AO RTA

71 9

Association Task Force on Practice Guidelines, American Associa­ tion for Thoracic Surgery, American College of Radiology,American Stroke Association, Society of Cardiovascular Anesthesiologists, So­ ciety for Cardiovascular Angiography and Interventions, Society of Interventional Radiology, Society of Thoracic Surgeons,and Society for Vascular Medicine. ] Am Coll Cardiol. 2 0 1 0 ;55 :e27-e l 2 9 . Isselbacher E M . Thoracic and abdominal aortic aneurysms. Circulation. 2005 ; 1 1 1 : 8 1 6-828. Isselbacher E M , Eagle KA, DeSanctis RW Diseases of the aorta. In: Braunwald E, ed. Heart Disease: A Textbook of Cardiovascu­ lar Medicine. 5th ed. Philadelphia: WB Saunders; 1997: 1 546- 1 58 1 . James KB, Healy B P. Heart disease arising during or second­ ary to pregnancy. Cardiovasc Clin. 1989 ; 1 9 : 8 1-96. Johnston Kw, Rutherford R B , Tilson MD, et a l . Sug­ gested standards for reporting on arterial aneurysms. ] Vase Surg. 1 99 1 ; 1 3 :452-458. Milewicz D M , Dietz H C , Miller DC. Treatment of aortic disease in patients with Marfan syndrome. Circulation. 2005 ; 1 l l : 1 50-1 5 7 . Pyertiz RE, McKusick VA. The Marfan syndrome: diagnosis and management. N Engl ] Med. 1 979;300: 772-777. Schoen FJ , Cotran RS . Blood vessels. In: Kumar V, Collins T, Robbins S, Cotran RS , eds. Robbins Pathologic Basis of Disease. 6th ed. Philadelphia: WB Saunders; 1 999:493-554. Spittell PC. Diseases of the aorta. In: Topol EJ, ed. Compre­ hensive Cardiovascular Medicine. Philadelphia: Lippincott-Raven; 1 998:303 1-305 1 .

QU ESTIONS AND ANSWERS Questions

1 . A 70-year-old man presents with the sudden onset of tearing chest pa i n . On presentation, he has a heart rate of 1 30 beats/m i n (bpm) with a systolic blood pressu re of 80 m m Hg. A bedside tra nsesoph­ ageal echocardiog raphy (TEE) demonstrates the presence of a proximal aortic dissection. A perica r­ dia I effusion with partia l diastolic collapse of the right ventricle is a lso present. Significant respi ratory va riation is noted across m itral and tricuspid Dop­ pler inflows. Appropriate treatment is: a. I m mediate percuta neous perica rd iocentesis to relieve the tam ponade, fol l owed by surgery to replace the ascending aorta b. To proceed i m mediately to the operating room c. Emergency angiography to define coronary a natomy, followed by surgery d. I ntra-aortic bal loon p u m p to stabil ize the hemodynamics, fol l owed by surgery 2. A 60-year-old hypertensive man presents with tea ring back pa i n . MRI confi rms the presence of a descending thoracic dissection originating beyond

the left su bclavian artery. Appropriate i n itial treatment includes: a. I mmediate surgery to replace the descending aorta b. I ntravenous n itroprusside followed by i m m ediate surgery c. I ntravenous n itroprusside alone; surgery for persistent pa in, or for i nvolvement of renal or mesenteric a rteries d. I ntravenous beta-blockade and n itroprusside; surgery for persistent pain, or for i nvolvement of renal or mesenteric arteries 3. A 56-yea r-old man presents for screening physical exa m i nation. He is asym ptomatic. Vita l signs reveal a heart rate of 80 bpm with a blood pressu re of 1 60/90 m m Hg. His exa m is remarkable only for a pulsati le mass i n the a bdomen. U ltrasound reveals the presence of a 3.9-cm abdominal aortic aneu­ rysm (AAA). Appropriate management incl udes a. I m m ediate referra l for surgery: b. Start a beta-blocker and repeat u ltrasound i n 6 months c. Refer for stenti ng of the AAA (Contin ued)

720

SECTION VIII • AO RTA/PERIPHERAL VAS CULAR D ISEASE

· · · · · · · · · · · · · · · · · · · · · · · · · · · · · · · · · · · · · · · · · · · · · · · · · · · · · · · · · · · · · · · · · · · · · · · · · · · · · · · · · · · · · · · · · · · · · · · · · · · · · · · · · · · · · · · · · · · · · · · · · · · · · ·

4. A 76-yea r-old woman with hypertension presents with severe chest pa i n . Her blood pressu re is 200/1 1 O mm Hg. Electrocard iog ra m revea ls nonspe­ cific ST-T changes. Chest x-ray is u n remarkable. CT sca n demonstrates the presence of a penetrating u lcer i n the ascending aorta. No d issection flap is seen. Appropriate ma nagement incl udes: a. Sta rt i ntravenous beta-blocker and n itroprusside while plans a re being made for surgery b. I ntravenous beta-blocker and n itroprusside, with surgery only if complications develop c. I ntravenous n itroprusside alone, with surgery only if com p l ications develop 5. A 23-yea r-old patient with Ma rfa n synd rome presents for routi ne eva l uation. He is asym ptom­ atic. Worku p incl udes a CT scan that reveals the presence of a 4.2-cm ascending aorta. Appropriate ma nagement includes: a. Refer for surgery b. Sta rt on beta-blocker and rei mage i n 6 to 1 2 months c. Reimage i n 6 to 1 2 months 6. The same patient returns for follow-u p i n 1 2 months. The aorta now measures 5 .0 cm i n size. He remains asym ptomatic. Appropriate management includes: a. Refer for surgery b. Continue beta-blocker, reassess i n 6 months c. Reassess i n 3 months 7. Which of the following disorders is associated with i nvolvement of the aorta? a. Ma rfa n synd rome b. Giant cel l arteritis c. Ankylosing spondylitis d. Syph ilis e. All of these disorders can have aortic i nvolvement. 8. Which of the fol l owing statements rega rding trans­ esophageal fi ndings of aortic atheroma is not true? a. Plaq ues >2 mm i n the ascending aorta a re associated with increased risk of stroke. b. Plaq ues >4 mm in the ascending aorta a re associated with increased risk of stroke. c. Mobile com ponents a re associated with a n increased r i s k o f stroke. d. Lim ited data suggest that these patients may benefit from a nticoag u lation thera py with wa rfarin.

. . . . . . . . . . . . . . . . . . . . . . . . . .

Answers 1 . Answer B: Th is patient should be ta ken to the operating room i m med iately. Percuta neous d rainage has been associated with increased morta lity i n this setting. Given the hemodynamic status, there is no time to proceed with angiography fi rst. Bal loon pumps are contraindicated with aortic dissection. 2. Answer D: I n itial thera py for descending aortic dissection is medical, with surgery reserved for special circumstances.The goal of treatment is red uction i n blood pressu re, as wel l as red uction i n dp!dt. Both beta-blockade, sta rted i m mediately, and n itroprusside should be used. 3 . Answer B : Asym pto matic a n e u rysms of 3.9 cm

have a very small risk of rupture. The patient should be fol lowed by seria l exa m i nation to assess size a n d rate o f expansion. Control o f his hypertension with beta-blockers may delay the g rowth of the a n e u rysm . There a re n o data as o f yet that endovasc u l a r stent g rafts wi l l lowe r the t h reshold for i ntervention for these a n e u rysms. 4. Answer A: Penetrating aortic ulcers i nvolving the

ascending aorta are genera l ly treated l i ke dissections, with pro m pt referra l for surgery. s. Answer B: The patient's aorta has not yet reached a size that would be considered for surgery i n the absence of sym ptoms. There a re data that beta-blockers can slow the rate of expa nsion of these aneurysms and improve su rviva l . 6 . Answer A: There h a s been ra pid g rowth i n t h e size

of the aneurysm (0.8 cm in 1 year). The patient should be referred for su rgery. 7. Answer E: All of the disorders listed ca n include i nvolvement of the aorta. 8. Answer A: P l a q ues >4 mm have been

associated with cerebral e m bo l i c events. The role of a nticoag u lation needs to be m o re clea rly defi n ed, but there a re some data to s u p po rt its use.

Venous Throinb oeinbolisITI Firas Al Solaiman and john R. Bartholomew

V

enous thromboembolism (VTE) is a common disease that includes both pulmonary embolism (PE) and deep vein thrombosis (DVT) . It is the third most frequently occurring cardiovascular condition after ischemic heart disease and cerebrovascular accidents in the United States. Approximately 1 million people de­ velop DVT and 600 ,000 develop PE each year, and death from VTE has been estimated to occur in as many as 60,000 to 200,000 Americans annually. ' E S S E N T I A L FACTS A B O U T V T E I t is important to recognize the natural history of VTE to more fully appreciate its short-term mortality and long-term morbidity. • VTE is a recurrent disease with a risk of recurrence up to 30% at 10 years for unprovoked events. 2 • PE is the third most common cause of hospital-related death and is the most common preventable cause of hospital death. 3 ·4 • The mortality rate for PE without treatment is approximately 30%. 5 • Patients with an acute PE who have right ventricular (RV) dys­ function documented by a transthoracic echocardiogram (TTE) have a higher in-hospital mortality ( 1 4%) and short-term mor­ tality (20%) rate at 3 months. 6 • Chronic thromboembolic pulmonary hypertension (CTPH) develops in as many as 3.8% of all PE patients by 2 years after their initial event. 7 • Approximately 40% to 50% of all patients with an acute symp­ tomatic DVT (proximal to the popliteal vein) , who are asympto­ matic for PE, will have a high-probability ventilation/perfusion scan. • The most common long-term complication of DVT is the post­ thrombotic syndrome (PTS) , characterized by chronic leg swell­ ing, pain, and nonhealing venous stasis ulcers, which occurs in as many as 30% of all patients within 10 years after a docu­ mented DVT.8·9

R I S K FACTO R S F O R V E N O U S THROMBOEMBOLISM Venous thrombosis results from the combination o f acquired and hereditary causes. The most common acquired and hereditary risk factors (referred to as hypercoagulable states or thrombophilia) are shown in Tables 49 . 1 and 49. 2 . As many a s 2 0 % of white patients presenting with an idiopathic or unprovoked DVT are heterozygous for the fac­ tor V Leiden mutation, while 6% are heterozygous for the prothrombin G202 1 0A mutation 1 0 Both of these disorders are rare in the African and Asian populations. 1 1• 12 Other, less common hereditary hypercoagulable states include protein C and S and antithrombin deficiencies, hyperhomocystine­ mia, elevated levels of factor VIII, and dysfibrinogenemia. D E E P VE I N T H RO M B O S I S : C L I N I C A L P R E S E N TAT I O N A N D D I AG N O S I S Although VTE is considered to be one disease entity, the clini­ cal presentation and diagnosis for DVT and PE are different. The characteristic symptoms for an acute DVT include leg or arm pain, swelling, increased skin temperature, and discolora­ tion (erythrocyanosis), although these findings may be absent. Unfortunately, the clinical examination is often unreliable and the diagnosis is only confirmed in 20% to 40% of patients pre­ senting with typical signs and symptoms u This is due in part to the varied differential diagnosis for DVT, which includes: • • • • • • • •

Cellulitis Arthritis, synovitis, myositis Lymphedema Arterial insufficiency Muscle ache or tear Baker cyst Chronic venous insufficiency Systemic causes of edema (congestive heart failure [CHF] , nephrotic syndrome, liver dysfunction, hypoalbuminemia)

72 1

722

SECTION VIII • AO RTA/PERIPHERAL VAS CULAR D ISEASE

TA B L E

TA B L E Acq u i red Risk Factors For VTE

Older age Ml, CHF, stroke, pneumonia Prolonged immobilization Long-distance travel (airplane or automobile trips) Surgery or trauma Smoking Obesity Malignancy Pregnancy, oral contraceptives, or hormone replacement therapy Previous VTE Pacemaker wires, CVP catheters Varicose veins Antiphospholipid antibody syndrome HIT Nephrotic syndrome, inflammatory bowel disease, myeloproliferative disorders

Clinical models have been developed to help diagnose an acute DVT. Wells et al. stratified outpatients presenting with a suspected DVT into low, intermediate, or high pretest prob­ ability categories based on a number of clinical "points."14 According to their model, 3% of low, 1 7% of moderate, and 75% of high pretest probability patients were diagnosed with a DVT. Although it is not widely used, this model may be a helpful objective assessment tool for clinicians (Table 49 .3). O BJ E C T I V E T E S T I N G F O R D E E P V E I N T H RO M B O S I S DVT can be confirmed using invasive and noninvasive stud­ ies as well as laboratory tests. These tests include: •

o-

Dimer assay

• Duplex ultrasonography • Venography

TA B L E Hereditary Risk Factors for VTE

Activated protein C resistance due to factor V Leiden mutation Prothrombin gene mutation (G202 1 0A) Antithrombin deficiency Protein C and S deficiencies Elevated factor VIII levels Increased levels of homocysteine

Clinical Featu re Score Accord ing to Wel l s et a l . Criteria Points

Active cancer Paralysis, paresis, or recent cast Recent immobilization for >3 d or major surgery 3 cm when compared with the asymptomatic leg Pitting edema Collateral veins Alternative diagnosis likely

1 1 1 1 1 1 1 1 -2

Risk score : low, 3 points . Adapted from Wells PS, Anderson DR, Bormanis ] , et al. Value of assess­ ment of pretest probability of deep-vein thrombosis in clinical manage­ ment. Lancet. 1 9 9 7 ; 3 5 0 : 1 795-1 878.

• Computed tomography venography (CTV) • Magnetic resonance venography (MRV) • Impedance plethysmography (IPG)

The more commonly used diagnostic tests include a n-dimer assay and duplex ultrasonography o-Dimer D- Dimer is a specific fragment of a fibrin clot whose pres­ ence indicates degradation of fibrin and serves as an indirect indicator for thrombotic activity D- Dimer assay has been utilized to a great extent in the outpatient setting and emer­ gency departments to rule out VTE, because of its high sen­ sitivity and negative predictive value. An elevated n-dimer level, however, is not specific for VTE and can be seen in a variety of conditions, including pregnancy, infection, dis­ seminated intravascular coagulation (DIC) , hemorrhage, malignancy, liver disease, surgery, trauma, cardiac or renal failure, acute coronary syndrome (ACS) , and acute nonlacu­ nar stroke. It is important to remember that not all n-dimer assays are alike and that it can be measured using a num­ ber of different methods. A recent analysis found that the Enzyme-linked immunosorbent assay (ELISA) and quanti­ tative rapid ELISA tests were superior to other methods. 1 5 Most physicians feel that anticoagulation can be with­ held from patients suspected of acute VTE in the outpatient setting if the n-dimer assay is negative. It is extremely impor­ tant, however, for clinicians to know the sensitivity and specificity of their hospital's n-dimer assay before making such a decision. If the clinician's suspicion of VTE remains high despite a negative n-dimer assay, further imaging stud­ ies are recommended.

CHAPTER 49 • VEN OUS THROMBOEMBOLISM

723

Duplex U ltrasonography Duplex ultrasonography is a readily available, noninvasive modality that can be performed routinely in the hospital or outpatient setting or at bedside for a critically ill patient. It has replaced venography as the diagnostic method of choice for acute DVT and is the most accurate noninvasive test cur­ rently available. Duplex ultrasonography allows for direct visualization of the venous system. An inability to compress the vein with the ultrasound transducer is considered diag­ nostic for DVT. The other ultrasound findings that may help in the diagnosis of acute DVT are listed in Table 49.4. Physicians must recognize that duplex ultrasonography is very operator dependent. Its sensitivity is approximately 95% and its specificity 96% in symptomatic patients with a proximal DVT, but it is less reliable in asymptomatic patients, those with thrombus above the inguinal ligament, or those with calf vein thrombosis. The sensitivity and specificity for isolated calf vein thrombosis approaches 60% to 70% . 1 6 In a study involving 375 patients, the validity of with­ holding anticoagulation in patients with a negative ultra­ sound and a low clinical suspicion for DVT was examined. 1 7 Only three patients who had anticoagulation withheld developed a new VIE event. Two patients developed an iso­ lated calf vein DVT and one patient a proximal DVT (total of 0.8%). No patient developed a PE at the 3-month follow-up. Duplex ultrasonography may also be useful in patients suspected of an acute PE. If the arms or legs are positive for an acute DVT, further confirmatory studies may be unneces­ sary in most patients, assuming that would not change the management of the patient.

Other Diag nostic Testing Options CTV of the legs can be performed in conjunction with a computed tomographic pulmonary angiogram (CTPA) of the chest used to rule out PE. Although more radiation is required, no additional contrast is needed and imaging of the more proximal leg veins (iliacs), pelvic veins, and the inferior vena cava (IVC) is possible. However, recent studies showed that routine CTV of the pelvis during CTPA does not signifi­ cantly improve the detection of VIE and therefore should not be performed routinely in all patients being evaluated for PE. 1 s, 1 9 MRV imaging can also be utilized to diagnose DVT. Its sensitivity and specificity has been reported to be >95 % when compared t o standard venography for the diagnosis of a proximal DVT, although outcome data are lacking. It has several advantages, including (a) detecting pelvic, iliac, and IVC thrombosis and (b) no need for ionizing radiation. Potential drawbacks include lack of availability, high cost, reader expertise, difficulty with morbidly obese patients, and the presence of metallic objects (stents or other hardware) in the area of interest. The MRV modality may be beneficial in pregnancy when there is a high clinical suspicion for an IVC, pelvic, or iliac vein DVT that is not detectable with duplex ultrasonography, or for patients with an allergy to contrast dye. However, this imaging technique should not be used in patients with acute or chronic renal insufficiency to prevent the adverse effect of nephrogenic systemic fibrosis (NSF) . 2 0 IPG has largely been replaced by duplex ultrasonogra­ phy at hospitals in the United States.

Venog raphy The venogram is an invasive procedure now replaced for the diagnosis of DVT by duplex ultrasonography An intraluminal-filling defect must be seen in at least two dif­ ferent projections for confirmation. A venogram should be considered in the appropriate clinical setting or when other tests are nondiagnostic. Despite its clinical utility, complica­ tions such as contrast allergy and postprocedural acute DVT should not be overlooked. The latter complication has been reported to occur in approximately 1 % to 2% of all patients.

P U L M O N A RY E M B O L I S M : C L I N I C A L P R E S E N TAT I O N A N D D I AG N O S I S Autopsy studies continue to demonstrate that most fatal cases of PE are unrecognized or not diagnosed. 21 Patients presenting with PE often have nonspecific signs and symptoms, making the diagnosis more difficult and frequently overlooked. In a review of the most common signs and symptoms of patients presenting with an acute PE without underlying cardiopul­ monary disease, dyspnea was most common, followed by pleuritic chest pain. These manifestations are valuable clues to the diagnosis in this patient population. However, in the individual with heart or lung disease, they may be mistaken for symptoms of the underlying disease process. Other signs and symptoms of an acute PE include cough, leg swell­ ing, thrombophlebitis, hemoptysis, palpitations, wheezing, angina-like pain, apprehension, and fever.n Patients may present with a massive or submassive PE , or they may be entirely asymptomatic. Patients who have a massive PE (systolic arterial pressure 6 points (high risk) was 78.4% , while that for 2 to 6 points (moderate risk) was 27.8% and for 1 00 bpm Immobilization or surgery within past 4 wk Previous DVT or PE Hemoptysis Malignancy

3 3 1 .5 1.5 1 .5 1 .5 1

Risk score: low probability < 2, moderate probability 2--0, high probability > 6. Adapted from Wells PS, Anderson DR, Rodger M, et al. Derivation of a simple clinical model to categorize patients probability of pulmonary em­ bolism: increasing the models utility with the SimpliRED o-dimer. Thromb Haemost. 2000;83 : 4 1 6-420 .

O BJ E C T I V E T E S T I N G F O R P U L M O N A RY EMBOLISM Traditional tests used to assist a physician faced with the presumptive diagnosis of an acute PE include a chest x-ray, electrocardiogram (ECG) , and an arterial blood gas. The chest x-ray may be more helpful to rule out pneu­ monia, a pneumothorax, or a malignancy. The most com­ mon x-ray features of acute PE are consolidation, a pleural effusion, atelectasis, Hampton hump (wedge-shaped opac­ ity along the pleural surface) , Westermark sign (oligemia) , and Palla sign (an enlarged right descending pulmonary artery) . 2 7 These latter three classic radiographic findings are rarely seen, however. An ECG may exclude cardiac causes that mimic PE, such as an Ml or a pericarditis. ECG findings suggestive of a PE include sinus tachycardia, new-onset atrial fibrillation or flutter, right bundle branch block, right-axis deviation, and nonspecific ST-T-wave changes. The classic finding of S 1 Q3 T3 indicates acute cor pulmonale but is seen in 0.9) has been shown to predict dou­ bling of mortality in the 30 days following diagnosis. 3 1 The major disadvantage of CTPA is the risk of contrast-induced nephropathy and radiation exposure.

CHAPTER 49 • VEN OUS THROMBOEMBOLISM Ventilation/Perfusion Sca n The V/Q scan was long considered one of the most useful aids to diagnose acute PE. The PIOPED trial (prospective investigation of pulmonary embolism diagnosis) combined low, intermediate, or high preclinical suspicion with a normal-, low-, intermediate-, or high-probability V/Q scan. 3 2 A normal V/Q scan effectively excluded the diagnosis of an acute PE, whereas if the clinical suspicion and the perfusion scan showed high probabilities, the diagnosis was very likely. Ventilation/perfusion scans interpreted as low or intermedi­ ate probability were considered nondiagnostic and required further testing to confirm or exclude an acute PE. In the PIOPED trial, 88% of patients with a high clini­ cal suspicion and high-probability V/Q scan had acute PE confirmed by pulmonary angiography. Among patients with a low-probability V/Q scan, angiographically proven PE was identified in 40% and 4% of patients with a high and low preclinical suspicion, respectively. Unfortunately, in as many as 75% to 80% of all PIOPED patients, no definitive diagnosis could be made because stud­ ies were interpreted either as low or intermediate probability. Ventilation/perfusion scanning has been replaced by multidetector CTPA and currently is considered a second-line modality in the diagnosis of PE. However, V/Q scan remains a valuable tool in the diagnosis of acute PE in patients with a normal chest x-ray or patients who cannot undergo CTPA (contrast allergy, renal insufficiency, or pregnancy) Pul monary Angiog raphy Pulmonary angiography remains the reference standard for which most studies are compared in the diagnosis of PE, despite the fact that it is not universally available, is invasive, and is costly. The definitive diagnosis of acute PE requires the presence of an intraluminal-filling defect in at least two views or demonstration of an occluded pulmonary artery. It is not without complications, and morbidity of 5 % and mor­ tality of 0 . 5 % were reported in the PIOPED trial.3 2 Echoca rdiog raphy (Tra nsthoracic and Transesophagea l) Abnormal TTE findings in acute PE include RV dilatation, RV hypokinesis, interventricular septal flattening or paradoxical motion, decreased inspiratory collapse of IVC, pulmonary artery hypertension and pulmonary artery dilatation, tricus­ pid regurgitation, patent foramen ovale (PFO) and rarely direct visualization of thrombus. The finding of akinesia of the mid-free RV wall with relative sparing of apex is referred to as McConnell sign. This sign was found in one study to have 94% specificity and 7 1 % positive predictive value for the diagnosis of acute PE.33 Hemodynamically unstable patients generally suffer severe RV dysfunction; however, RV hypokinesis presents in only 40% of patients with normal systemic pressure. A TTE alone cannot be used to diagnose PE but is a useful tool for risk stratification and identifying patients with acute PE

725

who may have a poor prognosis. RV dysfunction o n TTE has been associated with increased mortality among patients with acute PE. In a retrospective study, patients with an RV/ LV ratio �0.9 were 2 . 6 times more likely to die during hospi­ talization independently of other risk factors. 34 Transesophageal echocardiogram (TEE) can be used as a rapid bedside test to allow direct visualization of the main pulmonary arteries and should be considered in the hemodynamically unstable patients who cannot be moved to undergo a CTPA scan or pulmonary angiogram.35 In PIOPED Ill, magnetic resonance angiography (MRA) had insufficient sensitivity and a high rate of technically inadequate images. Addition of MRV to MRA improves sen­ sitivity; however, 52 % of patients in the study had a techni­ cally inadequate study. At the current time, an MRA/MRV should be considered only at those centers with experience with this modality and only for patients for whom standard tests are contraindicated.36 Ca rdiospecific Biomarkers in Pulmonary Embolism Cardiospecific biomarkers, including cardiac troponin and brain natriuretic peptide (BNP) , have become useful in the risk stratification strategy of patients with an acute PE. Eleva­ tion in troponin levels and BNP correlate with the presence of RV dysfunction and appear to be independent risk factors for poor or fatal outcome. In a meta-analysis of 1 985 acute PE patients, any elevation in troponin was found to confer a fivefold increase in short-term mortality. 3 7 A troponin level is considered the single most useful blood test for the risk stratification of patients with acute PE, while a normal BNP has a negative predictive value of 97% to 100% . 38•39

K E Y P O I N T S I N D I AG N O S I S O F VTE • Clinical prediction rules should b e used t o estimate the pretest probability for DVT and PE. • In patients with a low to moderate pretest probability for DVT or PE, a negative high sensitivity o-dimer has a very high nega­ tive predictive value (indicating a very low likelihood of VTE) . • Patients with a high pretest probability of VTE require addi­ tional diagnostic imaging studies. • Duplex ultrasonography is accepted as the first-line diagnostic imaging study for DVT. • Begin anticoagulation once suspect DVT (unless there is a con­ traindication for its use) . • Patients with a high pretest probability of PE require additional diagnostic imaging studies such CTPA. • Multidetector CTPA is accepted as the first-line diagnostic imag­ ing study for acute PE. • A V/Q scan is helpful if the chest x-ray is normal and a CTPA scan not possible or if the patient has renal insufficiency • Two D-echo or TEE is useful for the critically ill patient as a bedside test to help diagnose PE. • The presence of DVT on duplex ultrasonography is generally adequate for treatment of a PE .

726

SECTION VIII • AO RTA/PERIPHERAL VAS CULAR D ISEASE

• Pulmonary angiogram remains the gold standard diagnostic imaging for PE if diagnosis uncertain. • Troponins, BNP, and RV enlargement on echo or CT help pre­ dict outcome and are useful for risk stratification. • Begin anticoagulation once suspect PE (unless there is a con­ traindication for its use) using risk stratification tools including echocardiography and troponins or BNP.

T R E AT M E N T O F V E N O U S T H RO M B O E M B O L I S M The goals of treatment for VTE are to prevent extension, propagation, or embolization and recurrence of thrombosis. Treatment is also aimed at preserving valve function and pre­ venting the PTS in patients with DVT and to prevent CTPH and RV dysfunction in individuals with PE. Initial inpatient management should begin with weight-adjusted unfraction­ ated heparin (UFH) , a weight-based low-molecular-weight heparin (LMWH) preparation, or the anti-Xa inhibitor fonda­ parinux. A vitamin K antagonist (VKA) should be started as soon as possible, overlapping for a minimum of 5 days with one of the above-listed anticoagulants until the international normalized ratio (INR) is stable and ::'.'.2 .0 for at least 24 hours. U nfractionated Heparin UFH is generally administered intravenously, although it can also be effective when given subcutaneously Dosing is generally determined from a weight-based nomogram, and a bolus of 80 U/kg followed by 18 U/kg/h is commonly rec­ ommended for most adult patients. Subsequent dose adjust­ ments are made based on the results of either an activated partial thrombop!astin time (aPTT) or an anti-Xa assay using an amidolytic assay Heparin has a number of drawbacks. It has a vari­ able anticoagulant response among patients, a relatively short half-life, and adverse effects of bleeding, osteoporo­ sis, and heparin-induced thrombocytopenia (HIT) . HIT (an immune-mediated disorder that typically occurs 4 to 1 1 days after exposure to heparin products) is reported to occur in as many as 3% to 5% of all patients receiving UFH but occurs much less frequently in patients receiving any of the LMWH preparations. It can result in significant morbidity and mor­ tality, with life- and limb-threatening thrombotic complica­ tions including the loss of a limb , stroke, Ml, DVT, or PE. Treatment revolves around immediate cessation of UFH or LMWH and replacement with an alternative antithrombotic agent (direct thrombin inhibitor [DTI] ) . Currently, two DTls are approved by the U.S. Food and Drug Administration (FDA) for the treatment of HIT: argatroban, a small synthetic molecule, and a hirudin derivative (lepirudin) . Low-Molecu lar-Weight Heparin Depolymerization of UFH by chemical or enzymatic cleavage of its polysaccharide chains yields a mixture of heparin frag­ ments known as LMWH that have a mean molecular weight of

approximately 5 ,000 Da. This reduction in molecular weight leads to more predictable pharmacokinetics and a greater bioavailability than with UFH. A subcutaneous administered LMWH is the preferred anticoagulant for most hemodynami­ cally stable patients. Advantages of the LMWHs include: • • • • • • •

Once or twi.ce-daily subcutaneous injections Easy administration Dose by a weight-base adjusted regimen No monitoring necessary for most patients (see below) Outpatient administration Lower incidence of HIT Less osteoclast activation and lower incidence of osteoporosis

Although laboratory monitoring is generally not necessary, it is recommended in patients who are morbidly obese or who have significant renal disease, and in pediatric or preg­ nant patients. In these individuals, a 4-hour postinjection anti-Xa level using LMWH as the standard is recommended. Therapeutic levels are 0. 6 to 1 . 0 IU/mL for twice-daily injec­ tions and 1 . 0 to 2 . 0 IU/mL for once-a-day administration.40 LMWH preparations require dose adjustment for patients with a creatinine clearance :S;30 mUmin and are contraindi­ cated in patients on hemodialysis. The introduction of LMWH has dramatically altered the management of DVT in the United States. Two landmark clinical trials and a meta-analysis have demonstrated that subcutaneous injection of LMWH is as safe and effective in the outpatient treatment of acute DVT as UFH given in a hospital setting.4 1 Meta-analyses comparing LMWH to UFH found similar rates of maj or bleeding and recurrent VTE.42 Although the LMWHs are not approved in the United States for outpatient treatment of acute PE, several clini­ cal trials have demonstrated their safety A meta-analysis comparing UFH with LMWH in the inpatient treatment of hemodynamically stable PE patients demonstrated similar incidences of recurrent VTE, bleeding, and death.43 Heparin should still be first line of therapy in the following cases: • Persistent hypotension due to PE (massive PE) • Increased risk of bleeding • Concern about subcutaneous absorption (morbid obesity, anasarca) • When thrombolysis is being considered • Patients wi.th end-stage renal disease

Fondapari n ux Fondaparinux (Arixtra) is the only synthetic pentasaccharide that has been approved by the FDA for the treatment of acute DVT and PE. It is administered subcutaneously once daily and is almost 1 00% bioavailable. Dosing is weight based; 5 mg is recommended for individuals weighing 1 00 kg. Fondaparinux does not require dose adjustment or monitoring, but cau­ tion should be exercised while using this drug because of its long half-life and the lack of an antidote. Warfarin should be

CHAPTER 49 • VEN OUS THROMBOEMBOLISM started concurrently and continued for at least 5 days, until a therapeutic INR is attained. Fondaparinux is contraindicated in patients with renal insufficiency defined as a creatinine clearance 1 .4) h a s been associated

CHAPTER 5 0 • PERIPHERAL ARTERY DISEASE

with increased ca rdiovascu lar risk and a majority of these patients have significant PAD.The left leg ABI is abnormal and consistent with sig n ificant PAD, althoug h parti a l ly noncompressible vessels can also lead to over­ estimation of a n kle pressure and underestimation of severity of d isease in dia betic patients, and it is possible that d isease may be more severe than the ABI ind icates. In a l l patients with noncompressi ble vessels by ABI, the TBI should be measu red to establish the diagnosis of PAD. I n this case, seg mental PVR tracings or Doppler waveforms, including tracings at the a n kle, metatarsa l, and dig its wou l d be helpfu l along with TBls. 4. Answer C: This patient has known PAD that has

prog ressed to the point of ischemic rest pa i n and CLI. The history of pain worsening with l i m b elevation and improved with dependency (i.e., dangling the foot over the side of the bed) is very typica l for severe PAD/CU. I n this patient, repeat ABI measu rement and expedient evaluation for revascu la rization a re indicated. 5. Answer A: Su pervised exercise tra i n i n g is the most cost-effective fi rst-line treatment for ma nagement of claud ication in the PAD patient. A trial of pharmacother­ apy with cilostazol is also reasonable and recommend­ ed by the PAD guidelines (ref. ACC/AHA PAD H i rsch 2005). Th is approach would be fi rst-line thera py with revascularization ind icated (particu larly for suspected SFA disease) only after medical thera py has failed. There is l ittle evidence to support dual anti platelet therapy for the PAD patient without concomitant coronary a rtery disease or stenting, and a nticoagu lation with wa rfarin is not superior to aspirin alone and has hig her associated bleeding risk. Th is patient has reasonable dia betes mel­ l itus control (by H g B A 1 c) on ora l med ication and thus insulin is not ind icated. 6. Answer D: Th is patient has asym ptomatic PAD

but also has congestive heart fai l u re and i m pa i red LV systolic function. Cilostazol is approved for use i n sym ptomatic patients t o improve sym ptoms i n patients with lower extrem ity claud ication. It is contraindicated

747

i n this patient with CHF and LV dysfu nction and is also not ind icated g iven her lack of sym ptoms. There is no evidence that cilostazol i m p roves card iovascular outcomes among PAD patients.The most common side effects of cilostazol are headache, diarrhea, and pa lpitations. There is no ro le for revascu la rization i n t h i s asym ptomatic patient with m i l d ly red uced a n kle­ brachial ind ices. 7. Answer D: Not a l l patients with PAD a re ca ndidates for revascu larization. In this patient, additional imaging is req u i red given his CU and to assess candidacy for re­ vascula rization. Arterial d u plex or CTA of the extrem ities may be usefu l to diagnose anatomic location and de­ g ree of stenotic/occl usive lesions in the legs. Angioplas­ ty or bypass of the proximal a rteries (femoral, popl itea l, and tibial) alone may not be helpfu l if the d ista l runoff is com promised and a n imaging study ca n be used to fu rther assess ca ndidacy for revascula rization. While the patient may u ltimately req u i re toe a m p utation, deg ree of ischemia and candidacy for revascula rization m ust be considered fi rst. Cilostazol is not known to be beneficial for treatment of CLI. An a rterial flow p u m p m a y b e a n option for treating CU and ischemic wound i n patients who a re not ca ndidates for intervention . 8 . Answer D : Atherosclerotic PAD is u n l i kely in this

young woman with no risk factors. I n such cases, it is im portant to think of a lternative diagnoses. External iliac artery endofibrosis has been reported most com monly in bicyclists and leads to arterial narrowing and claudication or leg weakness, often at a high workload. Th is diagnosis should always be considered i n a young ath lete with claudication, especially a bicyclist. Th is entity has been trad itionally treated with open surgery but can also be managed by endovascular means. Aortic coarctation and vasculitis are u n l i kely based on the given history and the normal vascular exami nation. Arterial physio­ logica l testing (i.e., ABI at rest and after tread m i l l exercise reprod uces symptoms) and imaging studies can help to make this diagnosis.

C arotid Disease Amar Krishnaswamy and Mehdi Shishehbor

C

arotid atherosclerotic disease is an important cause of ischemic stroke and transient ischemic attack (TIA) . Diagnosis is made using ultrasound, computed tomog­ raphy, magnetic resonance angiography (MRA) , or angiogra­ phy Treatment consists of medical therapy to address the risk factors responsible for the formation and progression of ca­ rotid atherosclerosis, and carotid endarterectomy (CEA) or ca­ rotid artery stenting (CAS) in appropriately selected patients.

E P I D E M I O LO G Y Approximately 800,000 people in the United States experi­ ence a stroke each year, with a 30-day mortality of 1 0 % . 1-3 Up to 500,000 more individuals suffer a TIA annually 1 The male-to-female ratio for strokes is greater at ages 75 years (0. 76) ; overall, approxi­ mately 55 ,000 more women than men have a stroke each year. With respect to race, blacks have an almost twofold higher risk of first stroke than whites. PAT H O P H YS I O LO G Y O F CA ROT I D A RT E RY D I S E A S E An estimated 8 7 % o f strokes are ischemic in nature, about 1 5 % to 20% of which are attributed to complications of carotid atherosclerotic disease. 1 •4•5 Carotid stenosis can lead to ischemia via progressive narrowing and impairment of blood supply, or more commonly by providing a founda­ tion for the formation of local plaque rupture and athero­ thrombosis that embolizes distally. It is important to note that the severity of stenosis is predictive of ipsilateral stroke risk, with a 5-year risk of 7.8% in asymptomatic patients with 50 % , the 748

risk of ipsilateral stroke at 4 years was 1 0 . 3 % in those treated with medical therapy.7 Similar findings were noted by the asymptomatic carotid artery stenosis (ACAS) inves­ tigators who found a risk of ipsilateral stroke of 6 . 2 % at 2. 7 years of follow-up in patients with >60% stenosis. 7 The risk of stroke provided above is indicative of older studies with medical therapy that predates the current era. In more con­ temporary studies with optimal medical therapy, the risk of ipsilateral stroke has been shown to be as low as 0 . 34% per year with a risk of ipsilateral TIA of 1 . 78% per year.8 Risk factors for carotid stenosis are similar to those for coronary artery disease (CAD) , reflected in the fact that more than one-fifth of patients undergoing coronary artery bypass grafting (CABG) have >50% carotid stenosis.9 Conversely, in patients with cerebral infarction, more than 50% have some degree of coronary stenosis, and more than one-fourth have coronary stenoses >50%. 1 0 Material extracted from embolic protection devices (EPDs) after CAS reveal lipid vacuoles, foam cells, fibrin, and platelets, similar to that of coronary lesions. Increasing age results in greater risk of stroke or TIA in both men and women as well as across race. 1 The rela­ tive risk of stroke is doubled among smokers and returns to baseline levels after 5 years of abstinence. 11 Hypertension, diabetes mellitus, hyperlipidemia, and the metabolic syn­ drome all impart an increased risk. 12- 1 4 C A R OT I D A N D A R C H A N ATO M Y Defining aortic arch anatomy is an important part o f carotid and cerebrovascular angiography and is necessary in planning a carotid intervention. The aortic arch is defined as type I, II, or III depending on the relationship of the innominate artery to the inner and outer aortic arch curvature(s) (Fig. 5 1 . 1) . While the normal arch branching pattern consists of the innomi­ nate, left common carotid (LCC), and left subclavian (LSC) arteries (proximal to distal), there are a number of variants. In about one-fourth of patients, the LCC and innominate share a common origin, and in about one-sixth of patients, the LCC originates directly from the innominate artery. Both of these

CHAPTER 5 1 • CAROTID DISEASE Type I

Type I I

749

Type I l l

FIGURE 51 .1 Classification of aortic a rch type is

based upon the relationship of the innominate a rtery to the i n ner and outer cu rvatu re of the a rch. (Reprod uced from Krishnaswamy A, Klein J P, Ka padia SR. C l i n ical cerebrovascu lar a natomy. Cath Cardiovasc Int. 201 0;75:530-539, with permission from John Wiley and Sons.)

configurations are termed "bovine arch," though in reality bear no resemblance to the arch of cattle. 1 5 Other variations include an anomalous right subclavian artery that originates directly from the arch distal to the LSC artery, anomalous left vertebral artery originating directly from the arch, and a thyrocervical trunk originating from the aorta. The common carotid artery bifurcates into the external and internal carotid arteries (ICAs) at the C3-C4 interspace, and is the most common site of carotid atherosclerosis. The external carotid artery courses anteriorly and supplies numerous important branches to the face and neck, which also serve as collaterals in the setting of severe ICA disease. The ICA is divided into the cervical, petrous, cavernous, and supraclinoid portions, and no branches arise from the cer­ vical or petrous ICA. The supraclinoid portion of the ICA (carotid siphon) bifurcates into the middle cerebral artery (MCA) and anterior cerebral artery (ACA) . C L I N I C A L P R E S E N TAT I O N Carotid disease may b e diagnosed incidentally during physi­ cal examination or by diagnostic testing, or as part of the workup of stroke or TIA. While carotid auscultation is a common and important part of a comprehensive cardiovas­ cular physical examination, its sensitivity in patients with stenosis >60% is reported to be as low as 56% . 1 6 Therefore, in high-risk patients, carotid ultrasound is recommended for risk stratification. Population screening is not recommended by the United States Preventive Services Task Force (USP­ STF) , however. 1 7 TIA is defined as symptoms that resolve within 24 hours (most commonly within 30 minutes) and does not have associated acute imaging changes. Carotid-territory TIA or stroke can be reasonably elucidated from a detailed history and physical examination (Table 5 1 . 1) . Aphasia, dysarthria, or visual symptoms such as ipsilateral amaurosis fugax or contralateral homonymous hemianopia may be present. Sensory and motor symptoms are typically contralateral. D I AG N O S T I C T E S T I N G There are various imaging modalities used in the diagnosis of, and procedural planning for, carotid disease.

Ca rotid U ltrasound Ultrasonography is the standard noninvasive test for the evaluation of carotid disease. Large studies utilizing carotid ultrasound with angiography as a gold standard have reported sensitivity and specificity of >95 % in diagnosing angiographic stenosis > 50% . 1 8 .1 9 Standard criteria for the diagnosis of carotid stenosis (the modified Stradness crite­ ria) are listed in Table 5 1 .2 . It is important to note, however, that severe left ventricular (LV) dysfunction, severe aortic stenosis (AS) , and significant common carotid stenosis can all spuriously decrease carotid velocities and therefore mini­ mize the degree of stenosis calculated. Carotid l ntima Med ia Thickness Carotid intima media thickness (IMT) provides a measure of subclinical atherosclerosis and is usually measured in the common carotid artery. It is a measure of the thickness of the intima and media on two-dimensional (2-D) ultrasound imaging, normally increases with age, and is greater in men than women. 2 ° Carotid IMT may be used for risk stratifica­ tion purposes in patients without established atherosclerotic disease and intermediate probability as it has been associ­ ated with a higher risk of myocardial infarction (MI) , stroke, and cardiovascular death. 21-2 3 Computed Tomog raphy Angiog raphy Computed tomography angiography (CTA) is increasingly used in the evaluation of the coronary and peripheral arteries. In a recent evaluation of CTA, investigators reported a sensitiv­ ity and specificity of 77% and 95%, respectively, for diagnosing severe carotid stenosis H One significant limitation in the accu­ rate diagnosis of luminal narrowing is the presence of calcium blooming artifact. The diagnosis of moderate stenosis has lower accuracy (sensitivity 67%) , similar to MRA in this setting. Mag netic Resonance Ang iog raphy Carotid imaging using MRA can be performed after gado­ linium contrast administration or using time-of-flight (TOF) imaging without contrast. Unfortunately, noncontrast MRA may have poor sensitivity and specificity due to the lengthy time of acquisition ( 1 0 minutes) , which increases artifact. On the other hand, gadolinium MRA has reasonable sensitivity

750

SECTION VIII • AO RTA/PERIPHERAL VAS CULAR D ISEASE

Stroke Synd romes I nvolving the Major Vascular Territories Vascular Distri bution

Common Seq uelae

Ophthalmic artery Anterior choroidal artery MCA-origin (M l segment including lenticulostriate perforators)

Monocular blindness (amaurosis fugax) Contralateral hemiplegia and hemianesthesia, homonymous hemianopia Contralateral motor and sensory deficits predominantly affecting the face and arms, ipsilateral gaze deviation, homonymous hemianopia. Global aphasia if dominant hemisphere is involved Contralateral motor and sensory deficits predominantly affecting the face and arms. Broca's (nonfluent) aphasia if dominant hemisphere is involved Contralateral homonymous hemianopia. Wernicke's (fluent) aphasia if dominant hemisphere is involved Contralateral motor and sensory deficits predominantly involving the lower extremities, urinary incontinence, gait apraxia, abulia (implicates involvement of the recurrent artery of Heubner) Weakness, ataxia, oculomotor dysfunction and other cranial nerve deficits, dysphagia, vertigo, amnesia, altered consciousness, locked-in syndrome, "crossed" symptoms "Man-in-a-barrel" syndrome of proximal greater than distal arm and leg weakness Variable, may include visual field cuts, sensory deficits, weakness, apraxia, dysarthria.

MCA-superior division MCA-inferior division ACA Posterior circulation (vertebrobasilar system and its branches) ACA-MCA watershed infarction MCA-PCA watershed infarction

MCA, middle cerebral artery; ACA, anterior cerebral artery; PCA, posterior cerebral artery

(95 %) and specificity (92%) for diagnosing severe stenosis (> 70%) . 2 5 Similar to CTA, diagnosis of moderate stenosis is poor (sensitivity 66%) . Angiog raphy Established as the gold standard for evaluating carotid stenosis, angiography should be performed using digital subtraction

Doppler Criteria for Diagnosis of Carotid Stenosis Percent Stenosis

0%-19% 20%-39% 40%-59% 60%-79% 80%-99%

angiography (DSA) to "remove" the bones and soft tissues for better visualization of the arteries. Angiography provides high­ resolution images, allows an analysis of plaque quality (i.e. , cal­ cification, ulceration) , and enables the operator to evaluate the arch, neck vessels, intracerebral circulation, and collateral fill­ ing at the same time. This method is not usually a first-line test given the historically reported risk of transient (1 .3%) or per­ manent (0.6%) neurologic complications. 2 6•2 7 Notably, a more contemporary series of cerebrovascular angiography showed a much lower rate of complications, with 0.06% transient neu­ rologic deficits and 0.2% iatrogenic dissection, though this series included no arch aortograms and only a small percent­ age of patients with ischemic cerebrovascular disease. 2 8

Velocity Measurement

PSV 2 .0 PSV �150 emfs, EDY 4.0 PSV >240 emfs, EDY �135 emfs

PSV, proximal systolic velocity; !CA, internal carotid artery; CCA, common carotid artery.

M E D I C A L T R E AT M E N T Primary and secondary prevention o f carotid atherosclerosis includes risk factor modification and management. The use of CAS or CEA may be relevant in both situations, and are discussed further below. Anti platelet Therapy Historically, aspirin is the cornerstone of antiplatelet therapy In the Antithrombotic Trialist Collaboration's meta-analysis of 287 randomized trials enrolling >200,000 patients, they demonstrated a 25% reduction in nonfatal stroke and a 30% reduction in fatal or nonfatal ischemic stroke. 2 9 Additionally,

CHAPTER 5 1 • CAROTID DISEASE low-doses of aspirin (75 to 1 50 mg) were just as effective as higher doses, though with the caveat that this conclusion was based on less robust data. It should be noted, though, that the benefit of aspirin has been noted mostly in patients con­ sidered to be at high risk. Conversely, patients at low-risk for events have greater potential for harm due to bleeding. As a result, the American College of Cardiology/American Stroke Association (ACC/ASA) guidelines provide a Class I recom­ mendation for aspirin in patients for whom the benefits are likely to outweigh the risks (1 0-year risk 6% to 10%), though the recommendation is largely based on a reduction in all car­ diovascular events and not specifically stroke alone. 1 7 Use of aspirin in women specifically is noted as a Class Ila indica­ tion, again for those in whom ischemic stroke risk reduction outweighs the risk of gastrointestinal bleeding or hemorrhagic stroke u Recent trials have not convincingly shown that aspi­ rin decreases the risk of first stroke in patients with diabetes (and without CAD) , but have produced trends toward benefit. However, professional societies have found it difficult to com­ pletely abandon the use of aspirin in this setting. The ACC/ ASA guidelines therefore provide a Class Ilb recommendation for the use of aspirin in the primary prevention of stroke in diabetics at "high CV risk" with the caveat that the benefits "remain unclear." 1 7.3°.3 1 All patients with documented carotid disease should receive aspirin (or alternative regimen below) . Clopidogrel is an antiplatelet agent that irreversibly binds the P2Y 12 subunit of the ADP receptor, blocking GP Ilb/ IIIA-mediated platelet aggregation. In the Clopidogrel versus Aspirin in Patients at Risk of Ischemic Events ( CAPRIE) trial of patients with cardiovascular disease, clopidogrel demon­ strated an 8. 7% relative risk reduction (RRR) for the primary outcome of stroke, MI, or death.3 2 There was a trend toward benefit (RRR 7.3% , p 0.26) in patients with a history of stroke. On the basis of this and other studies, clopidogrel has been given an ACC/ASA Class I recommendation as an alternative agent to aspirin for secondary prevention in patients with a history of TIA or stroke. 33 While dual antiplatelet therapy (DAT) with aspirin and clopidogrel is often used in patients after coronary artery stenting, two large studies have not found a benefit in patients with stroke. In both the MATCH (Management of Atherothrombosis with Clopidogrel in High-Risk Patients with Recent Transient Ischaemic Attack or Ischemic Stroke) and CHARISMA ( Clopidogrel for High Atherothrombotic Risk and Ischemic Stabilization, Management, and Avoid­ ance) trials, DAT did not significantly decrease stroke out­ comes, but did result in a significantly increased bleeding. 34·35 The ACC/ASA guidelines therefore provide a Class III recommendation (not recommended) for the use of DAT for secondary prevention of stroke. 33 Dipyridamole (DP) is an adenosine deaminase (ADA) and phosphodiesterase (PDE) inhibitor, which results in an increased concentration of cyclic AMP, adenosine, and ade­ nine nucleotides; this inhibits platelet aggregation and causes vasodilation. Large studies using ASA and DP in patients with stroke have found a significant benefit to the use of combination therapy over monotherapy.36•37 Therefore, the =

751

ACC/ASA guidelines provide a Class I recommendation for ASA/DP combination therapy for patients with TINstroke, and suggest its use over aspirin monotherapy33 Anticoagu lant Therapy While anticoagulation with vitamin K antagonists is stand­ ard therapy for patients with cardioembolic source of TIA or stroke, there has been no suggestion of benefit over aspirin in patients with noncardioembolic stroke. The Stroke Prevention in Reversible Ischemia Trial (SPIRIT) randomized patients to warfarin (goal international normalized ratio (INR) 3.0 to 4.0) versus aspirin for secondary prevention. 38 Patients receiving warfarin had more than double the adverse events, mostly attributed to bleeding complications. The Warfarin-Aspirin Symptomatic Intracranial Disease (WASID) investigators made a similar randomization though with a more conservative INR goal (2.0 to 3 .0) but were forced to terminate the trial early due to an almost threefold increase in major hemorrhage. 39 Ultimately, due to complications of bleeding and a lack of ben­ efit for patients with atherosclerotic disease, anticoagulation is reserved for patients with a cardioembolic source of stroke or hypercoagulability disorder and is otherwise not indicated for the routine secondary prevention of TIA or stroke. Antihypertensive Therapy A large percentage of the US population has hypertension, and numerous studies have documented a 30% to 40% reduction in stroke with blood pressure control.40 While many studies using individual antihypertensive regimens (thiazides, beta-blockers, angiotensin-converting enzyme inhibitors, angiotensin receptor blockers, etc.) have shown a reduction in stroke, there is a paucity of data to recommend one specific therapy over another. The treatment of hyper­ tension for both primary and secondary prevention has been given a Class I recommendation by the ACC/ASA. 1 7 •33 For primary prevention in patients, treatment to a goal blood pres­ sure (BP) < 140/90 mm Hg is recommended, with a goal BP < 130/80 in patients with diabetes or renal disease (Class 1). 1 7 Secondary prevention guidelines support a goal BP of < 1 20/80 mm Hg with a Class Ila recommendation.33 All of these recommendations are made on the basis of the Sev­ enth Report of the ] oint National Committee on Prevention, Detection, Evaluation, and Treatment of High Blood Pres­ sure QN C 7) . While no specific agents are recommended for either group due to the small number of trials and lim­ ited comparisons, diuretics or the combination of diuretics and angiotensin-converting enzyme inhibitor (ACEi) are provided a Class Ila recommendation (for secondary pre­ vention) . 33 The use of ACEi or angiotensin receptor blockers (ARBs) is given a Class I recommendation for patients with diabetes. 1 7 •33 Antihyperlipidemic Therapy Statins (3-hydroxy-3-methylglutaryl-coenzyme A reduc­ tase inhibitors) have become the mainstay of antihyperlipi­ demic treatment for the primary and secondary prevention

752

SECTION VIII • AO RTA/PERIPHERAL VAS CULAR D ISEASE

NCEP Reco m mendations for Statin Therapy for Pri m a ry Stroke Prevention Factor

Goal

Recommendations

LDL-C 0-1 CHD risk factor

LDL-C < 1 60 mg/dL

Diet, weight management, and physical activity. Drug therapy recommended if LDL-C remains :2: 1 90 mg!dl. Drug therapy optional for LDL-C 1 60-1 89 mg/dL Diet, weight management, and physical activity. Drug therapy recommended if LDL-C remains :2: 1 60 mgldL Diet, weight management, and physical activity Drug therapy recommended if LDL-C remains :2:130 mg/dL (optionally :2: 1 00 mg/dL) . Diet, weight management, and physical activity. Drug therapy recommended if LDL-C :2:130 mg/dl. Drug therapy optional for LDL-C 70-129 mg/dL

2+ CHD risk factors and 1 0-year CHD risk l cm) . Normally, abnormalities in both glands represent adrenal hyperplasia. Hounsfield units > 1 0 usually indicate adrenal carcinoma, whereas Hounsfield units < 1 0 most likely suggest an adrenal adenoma. The difference in density results from a vascular tumor versus a lipid-rich adenoma. Adrenal vein sampling after administration of ACTH may be useful when no adrenal abnormality exists on CT scan or magnetic resonance imaging (MRI) or when there is an asymmetric abnormality in both glands. The sampling of the adrenal vein is technically difficult and should be restricted to experienced centers. It is important to assess both aldosterone and cortisol values at the time of sampling from the right and left adrenal glands and high and low inferior vena cava. To be certain the samples are from the

CHAPTER 5 2 • HALLMARKS OF ESSENTIAL AND SECONDARY HYPERTEN S I O N adrenal veins, cortisol should also b e measured in the same samples. Serum cortisol concentrations should be roughly the same in both adrenal veins and approximately 1 0-fold higher than in the peripheral vein. The aldosterone concen­ trations should be two times higher from the adrenal vein versus periphery An aldosterone ratio > 1 0 : 1 in the presence of a symmetric ACTH-induced cortisol response is diagnos­ tic of an aldosterone-producing adenoma. Medical therapy with eplerenone (selective aldosterone­ receptor antagonist) or spironolactone can be used in patients with bilateral adrenal adenomas, adenomas that cannot be excised surgically (poor surgical risk), in individuals with adrenal hyperplasia, and in those with significant responses to aldosterone-receptor antagonists who do not desire sur­ gery Surgical removal of an aldosterone-producing adenoma renders normotension and restoration of normal potassium homeostasis in most patients. Adrenal adenomas may be removed laparoscopically Patients may require drug treatment for 3 to 6 months prior to surgery Selective hypoaldosteronism usually occurs after aldosterone-producing adenoma removal. Cush ing Synd rome Clinical clues for Cushing syndrome include a history of recent change in facial appearance and considerable weight gain, together with complaints of weakness, muscle wasting, peripheral bruising, impotence, and, in women, amenorrhea and hirsutism.59 Typical physical features include truncal obesity, moon face, plethora, and purplish skin stria. Screening and laboratory studies may indicate glucose intolerance or frank diabetes mellitus, and occasionally neu­ trophilia with relative lymphocytopenia. Pathologic fractures of a rib are common. A dexamethasone suppression test may be helpful. For diagnosis and localization, a 24-hour uri­ nary free cortisol test, CT, and radioimmunoassay of plasma adrenocorticotropic hormone may be helpful. The standard of care for most cases of Cushing syn­ drome is surgical resection of a pituitary gland or an ectopic source of adrenocorticotropic hormone or removal of a cortisol-producing adrenal cortical tumor. Transsphenoidal pituitary adenomectomy or radiation therapy to the pituitary bed may be considered in selected cases. Pheochromocytoma Pheochromocytoma can present in many ways, reflecting var­ iation in the hormone it releases, the pattern of release, and differences in each individual's catecholamine sensitivities. Eighty percent of patients with pheochromocytoma present with headache, 57% with sweating, 48% with paroxysmal hypertension, 39% with persistent hypertension, 64% with palpitations; 13% of patients may be normotensive, and 8% may be completely asymptomatic. In approximately 10% of patients, tumors are discovered incidentally during CT/MRI of the abdomen for unrelated symptoms. Those individu­ als who warrant a workup for pheochromocytoma include patients with (a) episodic symptoms of headache, tachycardia,

769

Pheochromocytoma Suspect

� 1 .8

� 2,000

J

P H EO

1 ,000 to 2, 000

< 1 , 000

1 . 3 to 1 . 8

J

< 1 .3

S u p p res s i o n

Stimulation

Test

J

Test

FIGURE 52.S Pheoch romocytoma suspected. (Ada pted from Bravo EL. Pheochro mocytoma. Cordial Rev. 2002; 1 0:44-50.)

diaphoresis; (b) family history of pheochromocytoma or mul­ tiple endocrine neoplasia (MEN) syndrome; (c) unexplained paroxysms of tachy/brady dysrhythmias; and/or (d) hyper­ tension during intubation, induction of anesthesia, prolonged or unexplained hypotension after surgery, or adverse CV responses to ingestion or inhalation of certain drugs includ­ ing anesthetic agents, glucagon, ACTH, thyrotropin-releasing hormone, antidopaminergic agents, miloxane, phenothia­ zine, guanethidine, and tricyclic antibiotics. Currently available tests can establish the diagnosis of pheochromocytoma in >90% of cases. Figure 52.5 illus­ trates the approach to using plasma catecholamines and uri­ nary metanephrines in the evaluation of patients suspected of having pheochromocytoma.6° Fractionated plasma free metanephrines are the best test for familial (hereditary) pheochromocytoma, whereas 24-hour urinary metane­ phrines and catecholamines provide adequate sensitivity with low false-positive rates for sporadic pheochromocytoma. The combination of resting plasma catecholamines (NE plus E) at least 2 ,000 pg/ml and urinary metanephrines (normetanephrines plus metanephrines) at least 1 .8 mg in 24 hours has a diagnostic accuracy of approximately 98% in both sporadic and hereditary pheochromocytoma. A number of medications interfere with the biochemical diagnosis of pheochromocytoma. Methylglucamine results in a decrease in metanephrines, whereas sotalol increases metanephrine concentration. ARBs, ACE inhibitors, and bromocriptine decrease catecholamine values, whereas a1-blockers, BBs, and labetalol increase catecholamine values. Methyldopa and monamine oxidase inhibitors decrease vanillylmandelic acid (VMA) values, whereas nalidixic acid and anileridine increase VMA values. Phenothiazine, methyldopa, and tricyclic antibi­ otics have varying effects on these tests. A urinary metabolite of buspirone, an anxiolytic drug, is artificially measured as metanephrines, resulting in a marked increase in the measured metanephrine excretion. When blood specimens are drawn under standardized conditions, a total plasma catecholamine level ?:2 ,000 pg/ml is diagnostic of pheochromocytoma, whereas a value of 1 . 3 mg!dl) or the presence of albuminuria >300 mg!d or 200 mg of albumin per gram of creatinine has been associated with an increased risk for hypertension. The HOPE study data demonstrated a continuous rela­ tionship between serum creatinine levels and CV outcomes in hypertensive and normotensive patients. An additive risk exists with increased serum creatinine and microalbuminuria. 63 Hypertension is one of the main contributing factors to progressive renal injury, and lowering BP to

LPL

HL

IDL





ApoB- 1 0 0 Apo Cs ApoE

E

FFA

ApoCs

A

>

40 g!d) , sedentary lifestyle, or diets rich in saturated fats or carbohydrates

(Repri nted from Fuster V, Ross R, To pol EJ, eds. A therosclerosis and Coronary Artery Disease. 1 st ed. P h i ladelphia: Lippi ncott­ Raven; 1 996, with perm ission.)

• Physical exam .findings : body mass index (BMI) , waist circum­ ference, blood pressure, thyroid characteristics, xanthomas (cholesterol deposits of interdigital, tuberous, planar, or erup­ tive types) , xanthelasmas (xanthomas of the palpebral fissures) , arcus corneae, lipemia retinalis (pale-appearing retinal vessels) , peripheral pulses, vascular bruits, or hepatosplenomegaly: See Figure 53.3 for representative examples of xanthomas • Laboratory studies: fasting serum lipid panel, fasting glucose, thyroid-stimulating hormone (TSH) , creatinine, hepatic func­ tion panel, urinalysis, and a screen for microalbuminuria. Additional measurements such as lipoprotein subpopulation analysis, levels of serum Lp(a) , apoB, and high-sensitivity CRP (hsCRP) may help refine risk assessment, particularly in an intermediate-risk population or to help tailor intensity of thera­ pies, but would not be recommended as routine tests for all individuals.

C LA S S I F I C AT I O N O F DYS L I P I D E M I A S In 1967, Fredrickson e t al. 1 proposed a system to diagnose and classify lipid disorders based upon the specific lipopro­ tein or lipoproteins elevated in the patient's serum. While the Fredrickson classification system was useful shorthand for describing hyperlipidemias, it had two major shortcom­ ings: its phenotypes did not provide or even imply etiology, and HDL was not included. In a more clinically relevant

schema, lipid disorders can also be grouped into two broad categories. Primary dyslipidemias result from genetic variabil­ ity in one or more loci controlling the expression of proteins involved in lipoprotein synthesis, metabolism, or clearance. Secondary dyslipidemias are the consequence of a separate pathologic process. Primary Dyslipidemias The primary dyslipidemias of non-HDL-C have been sum­ marized in Table 53. 1 . Familial hypercholesterolemia (FH) is a common autosomal dominant disorder resulting from mutations in the LDL receptor (LDL-R) , or apoB receptor, leading to impaired hepatic clearance of LDL from the circu­ lation. Heterozygous FH (HeFH) occurs in 1 in 500 persons and is associated with serum LDL-C two to three times the average and a four- to sixfold increased risk for premature CHD . Without treatment, the average age for development of symptomatic CHD is 45 years in men, and 55 years in women. By age 39, 90% of FH heterozygotes exhibit detect­ able xanthomas on the extensor tendons of the hands, or on the Achilles tendons (Fig. 53.3). Several diagnostic criteria for FH exist, with that of the 1 5-year Simon Broome Register Group being the most commonly used.

TA B L E Primary Dysli pidemias of Non-H DL-C

Disorder

Molecular Basis

Familial hypercholesterolemia (FH)

LDL-R mutations Dysfunctional receptor j, Hepatic LDL clearance .

I n heritance/ Prevalence

Physica l Fi ndings

Sym ptomatic C H O Risk

Treatment (Nondieta ry)

TC 300-400 mgldl 4-6x average risk Statins, resins, Xanthomas of extensor LDL-C 200-300 mgldl hand tendons, Achilles Without treatment, apheresis fatal MI in nearly Apheresis, statins, tendons HoFH 1 : 1 million TC > 600 mgldl As in HeFH, plus resins 1 00% by age 20 LDL-C >600 mg/dl interdigital, tuberous (hands, elbows, buttocks, feet), planar (thighs, buttocks, knees) xanthomas. Similar to HeFH Similar to HeFH , Similar to HeFH Statins, resins, AD 1 :600 often with slightly apheresis more benign course.

AD HeFH 1 : 500

ApoB l OO mutation J, LDL affinity for LDL-R j, Hepatic LDL clearance Complex 1 : 20 Polygenic hypercholes- Presence of ApoE4 terolemia (PH) to 1: 1 00 allele, or Mildly defective LDLR, or Mildly defective apoBlOO. i apoB synthesis Complex 1 : 33 Familial combined Multiple implicated hyperlipidemia to 1 : 1 00 genes (FCH) Familial defective ApoB l OO

Lipid Profi le

No xanthomas TC 250-350 mg/dl LDL-C 150-250 mg/dl TG normal

l . 5-2x average risk

TC 250-350 mgldl Arcus cornea Xanthelasmas LDL-C 200-300 No xanthomas mgldl and/or TG > 130 mg/dl (often much higher)

2-5x average risk

Statins, niacin, resins

Statins (for phenotypes Ila, Ilb) Fibrates (for phenotypes Ilb, IV)

I I

00 �

"'

n f-j ....... 0 z

Vl [Tl

>< .......

n t:: z n � •

r

� z CJ

rg "O

<

z f-j

� [Tl

n � ?::! CJ ....... 0 r 0



Hyperapobetalipoproteinemia (HyperapoB)

i Synthesis apoB

Type III Hyperlipidemia Defective apoE 90% of patients are (Familial DysE2/E2 betalipoproteinemia)

Familial Endogenous and Familial Mixed Hypertriglyceridemias Familial Chylomicronemia

i hepatic VLDL, TG i hepatic VLDL, J, LPL activity

Inheritance and prevalence unclear

Usually AR, but can be AD 1 : 1 ,000 to 1 :5 ,000

LDL-C < 1 60 mg/dL (normal to borderline i) LDL-apoB > 1 30 mgldL TG normal or i TC 300-600 mg/dL TG 400-800 mg/dL and higher VLDL-C:TG > 0. 3

FEH: AD? 1 : 300 TG 200-500 mgldL FMH: unclear rare TG > 1 ,000 mgldL

LPL or apoC-II muta- AR 1 in 1 million tions � serum accumulation of chylomicrons

TG > 1 ,000 mg/dL

AD , autosomal dominant; AR, autosomal recessive; HeFH, heterozygous FH; HoFH , homozygous FH; RR, relative risk.

Arcus cornea Xanthelasmas No xanthomas

If TGi , > l 7x average risk If TG normal, 2-3x average risk

Planar xanthomas of the ii compared to average (RR palmar creases Tuberous xanthomas over unknown) knees and elbows, not attached to tendons No xanthomas CHD risk not consistently elevated. Some kindreds at higher risk than others. Eruptive xanthomas on Unclear association with CHD extensor limb surfaces, hepatosplenomegaly, pancreatitis, lipemia retinalis

Sta tins Resins

Fibrates Sta tins Niacin

Fibrates, niacin Fibrates

Drug therapy ineffective

n ::r: � '"" � tT1 "'

Ul VJ •

� ::r: tT1 tJ >-< r '""

Vl

6 tT1



s;:

Vl

" 00 U1

786

SECTION IX • CLINICAL AND PREVENTIVE CARDIOLOGY

Definite FH requires: (a) TC > 290 mg!dL in adults or TC > 260 mg/dL in children

under 1 6

O R LDL-C > 1 90 mg/dL in adults o r > 1 5 5 mg/dL in children PLUS (b) Tendon xanthomas in the patient, or first- or second-degree

relative (c) OR DNA-based evidence of LDL-R mutation or familial defec­

tive apoB l OO

Possible FH is defined as (a) above plus one of (d) or (e) : (d) MI before age 50 in second-degree relative, or before 60 in first

degree (e) Elevated cholesterol in first-degree relative, or >290 mg/dL in

second degree

Homozygous FH (HoFH) occurs in one in one million indi­ viduals. Patients with HoFH do not express any functional LDL-Rs, and consequently exhibit a more severe pheno­ type. TC levels are generally >600 mg/dL, with LDL-C levels six- to eightfold higher than average. Without treat­ ment, death from MI occurs in the first or second decades of life. In addition to the xanthomas observed in heterozy­ gotes, FH homozygotes are commonly affected by interdigi­ tal xanthomas, tuberous xanthomas on the hands, elbows, buttocks, and feet, and planar xanthomas on the posterior thighs, buttocks, and knees. Familia! Defective Apolipoprotein B l OO is associated with impaired LDL clearance due to reduced affinity of LDL for the LDL-R. The most common cause of this autosomal dom­ inant condition is a single base mutation in the apoB lOO gene. Although the prevalence of familial defective apoB lOO is unclear, and varies by ethnic background, it may be as common as 1 in 600. The lipoprotein concentrations, clini­ cal features, and treatment are similar to FH heterozygotes. Po!ygenic hypercho!estero!emia (PH) is the most common cause of an isolated elevation in TC or LDL-C, with preva­ lence in the United States estimated at between 1 in 20 and 1 in 100. TG levels are generally normal. Alterations in the function or expression of several key proteins involved in LDL metabolism have been associated with PH, including mildly defective LDL-R and apoB lOO, increased synthesis of apoB, and the presence of the apoE4 allele, which has a higher affinity for the LDL-R than the other apoE isoforms leading to downregulation of LDL-R synthesis and a second­ ary increase in serum LDL-C. Xanthomas are very rare or absent in patients with PH. Familia! combined hyperlipidemia (FCH) is the most com­ mon primary dyslipidemia in which multiple lipoprotein phenotypes exist, with a population prevalence of 1 % to 3 % . The three observed patterns o f elevated VLDL, elevated LDL, or both can be seen within a family or within a single patient over time. Transmission is complex, with multiple genes likely to be involved. It is associated with an estimated two­ to fivefold increased risk for CHD, accounting for one-third

to one-half of familial CHD , and up to 20% of all premature CHD. Traditionally, diagnostic criteria include: 1.

2.

TC and/or TG levels >90th percentile for age- and sex-matched controls. TC of 250 to 350 mg/dL and TG > 130 mg/dL (often higher, especially in diabetics) . At least one first-degree relative with elevated VLDL-C, LDL-C, or both

3. Strong family histories of hyperlipidemia and premature CHD

Other maj or characteristics include elevated apoB (> 120 mgldl) , a preponderance of sdLDL, and low HDL-C. FCH is usually diagnosed after age 20, and the patients are often hypertensive, overweight, insulin resistant, or diabetic. Arcus cornea and xanthelasmas are commonly seen, but tendon xanthomas are unusual. Hence, the absence of ten­ don xanthomas in a hypercholesterolemic patient is a useful feature to differentiate FCH from FH. Other less common dyslipidemias of apoB metabolism are frequently characterized by elevations in TG, VLDL or in LDL-apoB concentrations. These include hyperapobetalipo­ proteinemia (hyperapoB), type III hyperlipidemia (also known as familia! dysbeta!ipoproteinemia or remnant disease) , familia! endogenous hypertriglyceridemia (FEH) , familia! mixed hyper­ trig!yceridemia (FMH) , and familial chy!omicronemia and are

outlined in Table 53. 1 . There exist a heterogeneous group of rare familial dis­ orders of HDL-C, including Tangier Disease, Familia! LCAT Deficiency , and Partial LCAT Deficiency or Fish-Eye Disease which have not been consistently associated with premature CHD and are beyond the scope of this chapter. ApoAIMilano is a rare genetic variant of the apoAI protein resulting in low HDL-C levels. However, individuals bearing the mutation display longevity and an exceptionally low risk of CHD . In a controlled trial of patients with a history of acute coronary syndrome (ACS), infusions of purified ApoAIMHano induced plaque regression, as assessed by intravascular ultrasound (IVUS) . At present, such therapy is not available for clinical use but these studies suggest a future role for HDL-C modi­ fication to reduce cardiovascular risk. Secondary Dyslipidemias Evaluation of a dyslipidemia would not be complete without a thorough search for secondary and contributing causes. A care­ ful history and physical, accompanied by selected laboratory studies, including a fasting lipid panel, will frequently reveal the etiology of a patients dyslipidemia. Breakdown by the abnor­ mal lipid may simplify diagnosis of the dyslipidemia's cause: •



Elevated TC and LDL-C: diet rich in saturated fat, drugs (oral contraceptives or OCPs, HRT, HAART) , hypothyroidism, nephrotic syndrome, chronic liver disease, and chronic biliary tract disease (classically, primary biliary cirrhosis) Elevated TG: diet rich in carbohydrates, drugs (� blockers, thiazide diuretics, isotretinoin, glucocorticoids, HAART, OCP, HRT) , excessive alcohol consumption, obesity, DM, hypothy­ roidism, CRI, chronic pancreatitis, and pregnancy

CHAPTER 53 • THE DYSLIPIDEMIAS • Low HDL-C: very high carbohydrate, very low-fat diet, hypertriglyceridemia, obesity, sedentary lifestyle, smoking, DM

Often, these dyslipidemias can be at least partially con­ trolled by institution of lifestyle changes (including dietary improvement, weight loss, increased exercise, and smoking cessation) , withdrawal and replacement on an implicated medication, or recognition and treatment of an underlying disorder. A typical example is the hyperlipidemic patient who is resistant to lipid-lowering therapy, but later found to be hypothyroid. Treatment with levothyroxine then corrects the secondary dyslipidemia. M ETA B O L I C S Y N D R O M E Due to the growing recognition of its prevalence and the asso­ ciation of this syndrome with a particular dyslipidemic pat­ tern, the metabolic syndrome (MetS) is reviewed here. During the late 1 980s, Reaven observed that several CHD risk factors (namely dyslipidemia, hypertension, and hyperglycemia) fre­ quently cluster together. 2 Originally referred to as Syndrome X or the insulin-resistance syndrome, these descriptors have been supplanted by the more general term metabolic syndrome. The National Cholesterol Education Program's Adult Treat­ ment Panel Ill report (ATP Ill, discussed later) identified six components of the MetS which relate to CHD : abdominal obesity, atherogenic dyslipidemia (high TG, low HDL-C, and sdLDL) , hypertension (HTN) , insulin resistance, a prothrom­ botic state, and a proinflammatory state. 3 Given the ongo­ ing "epidemic of obesity," MetS is common and becoming more so, with approximately one-third of American adults fulfilling ATP III and global criteria,4 as listed in Table 53.2. Observational data have shown that such persons have a three to fivefold risk of CHD mortality, compared to those without MetS. A maj or health danger to persons with MetS appears to be a vastly increased risk of developing diabetes. MetS was highly predictive of new-onset DM in both men and women of the Framingham offspring cohort, as nearly half of the population-attributable risk of type 2 diabetes (DM2) could be explained by the presence of ATPIII-defined MetS.5 When overt DM develops, CHD risk increases sharply. T R E AT M E N T S T R AT E G I E S F O R DYS L I P I D E M I A S : T H E RA P E U T I C LI F ESTYLE CHANGES All dyslipidemic patients should be urged to adopt therapeutic lifestyle changes (TLC) , consisting of increased physical activ­ ity, ideal weight maintenance (often necessitating weight loss) , smoking cessation, and the pursuance of a low-saturated fat, low-cholesterol diet rich in fruits, vegetables, grains, and fiber (Table 53.3). In highly motivated individuals, TLC can result in an LDL-C reduction of nearly 30%, and should form the basis of all preventive treatment. Alcohol avoidance, smok­ ing cessation, physical activity, and diet are essential in the management of dyslipidemias characterized by very high TG.

787

TA B L E ATP I l l Clinical Identification of the Mets Risk Factor (Need 3)

Defi n in g Level

Abdominal obesity" Men Women TG HDL-C Men Women Blood Pressure Fasting glucose

Waist circumferenceh > 1 02 cm (>40 inches) >88 cm (>35 inches) � 1 50 mgldL l OK

M, 30-59

4,08 1

M, 40-55

Clofibrate Placebo Gemfibrozil Placebo

5.3

HHS

TC in upper tertile of those screened non-HDL-C >200 mg/dL

TC 290-380 SBP 50 mm Hg) significantly increases the maternal and fetal risks of pregnancy and women with uncorrected severe AS should be counseled against conception. Classic symptoms of AS such as dyspnea, angina pectoris, or syn­ cope usually become apparent late in the second or early third trimester. Ideally, women with known severe AS should undergo valve correction prior to conception. Although percutaneous aortic balloon valvuloplasty (PABV) prior to pregnancy can decrease the risk of pregnancy, labor, and delivery, it has lim­ ited durability and is unlikely to relieve AS in the long term. Therefore, surgical correction is the preferred approach. Bioprosthetic valves have limited durability; young patients will likely require reoperation within 10 to 1 5 years. However, implantation of a bioprosthetic valve avoids the need for anticoagulation during pregnancy Mechanical valves have greater durability but require antico­ agulation, which independently increases both the maternal and fetal complication risk during pregnancy. The decision to implant a bioprosthetic versus mechanical valve is com­ plex and should be made in consultation with both a cardi­ ologist and a cardiothoracic surgeon. Management of mild to moderate AS during pregnancy is largely conservative. When severe symptomatic AS is diagnosed during pregnancy, PABV should be performed prior to the demands of labor and delivery Aortic insuffi­ ciency that occurs as a postprocedural complication of PABV

822

S ECTION IX • CLINICAL A ND PREVENTIVE CARDIOLOGY

usually is well tolerated during labor and delivery. Vaginal or assisted vaginal delivery is usually well tolerated, although spinal and epidural anesthesia are discouraged during labor and delivery because of their vasodilatory effects. Invasive hemodynamic monitoring and is recommended during labor and delivery Coa rctation of the Aorta Coarctation of the aorta is a narrowing in the region of the ligamentum arteriosum, just distal to the origin of the left subclavian artery usu­ ally presenting with resistant hypertension in childhood. Coarctation is well tolerated during pregnancy, although hypertension, heart failure, angina, and aortic dissection are possible complications . Coarctation can be associ­ ated with intracerebral aneurysms , which may rupture during pregnancy. Hypotension in vascular beds distal to the coarctation can compromise uteroplacental blood flow, resulting in intrauterine growth retardation. Coarc­ tation of the aorta is often associated with a congenitally bicuspid aortic valve , which increases the risk of infective endocarditis. If at all possible, coarctation of the aorta should be corrected prior to pregnancy with standard surgical repair or balloon angioplasty with endovascular stent placement. Correction of coarctation during pregnancy is indicated in patients with severe uncontrollable hypertension, heart failure, or uterine hypoperfusion. The aortic wall adjacent to an area of coarctation has histologic features of cystic medial necrosis, which renders it vulnerable to dissection. Thus, women who underwent surgical repair of an aortic coarctation in childhood remain at risk for complications during pregnancy. Ma rfa n Synd rome Marfan syndrome is a connective tis­ sue disorder resulting from mutations in the fibrillin gene that are inherited in an autosomal dominant fashion (i.e. , 50% of offspring will inherit the disorder, regardless of gen­ der) . Thus, women with Marfan syndrome should receive genetic counseling well in advance of pregnancy consid­ eration. The clinical manifestations of Marfan syndrome include skeletal abnormalities, ectopia lentis, and cardio­ vascular abnormalities, such as aortic root dilatation with or without AR, aortic dissection, and MVP However, Marfan syndrome is a heterogeneous disorder with highly variable disease penetrance. It is estimated that pregnancy in patients with Marfan syndrome carries a 1 % risk of serious cardiac complica­ tions; this risk rises with increasing aortic root dimensions. Women with Marfan syndrome are more vulnerable to aortic dissection and/or rupture during pregnancy because of the additional weakness in the aortic wall imposed by hormonal changes. Interestingly, the occurrence of dissection appears to peak at 3 to 20 days postpartum, with a hypothesis being that oxytocin stimulation during breast-feeding may activate the extracellular signal-regulated kinase (ERK) pathway, which has been recently implicated in the pathophysiology

of this condition. In addition, women with Marfan syn­ drome may be more prone to spontaneous abortion and preterm labor. Screening echocardiography should be performed prior to pregnancy. Enlargement of the aortic root >4. 5 cm is associated with maternal mortality as high as 1 0 % . There is some evidence that pregnancy in women with Marfan syn­ drome may be relatively safe up to 4 . 5 cm, and therefore in women with an aortic diameter of 4.0 to 4 . 5 cm pertinent risk factors for dissection (family history of dissection, rapid growth) should be taken into account, and body surface area factored in too. The 20 1 1 European Society of Cardiol­ ogy guidelines strongly recommend preconception elective aortic root repair above 4. 5 cm and support individualiza­ tion of management in the range 4.0 to 4 . 5 cm; the 2 0 1 0 ACC/AHA guidelines o n thoracic aortic disease state that it is "reasonable" to prophylactically replace the aortic root and ascending aorta when the diameter exceeds 4.0 cm. The aortic root should be monitored by serial echocardiography throughout pregnancy, with progressive dilatation warrant­ ing termination of pregnancy and/or timely aortic repair or replacement. Medical management involves the use of beta-blockers throughout pregnancy to reduce the risk of aortic rupture, careful control of blood pressure, and consideration of gen­ eral anesthesia and cesarean section at the time of delivery to maximize hemodynamic control. Women with Marfan syn­ drome who do not manifest any cardiac abnormalities have a low rate of complications and can usually tolerate a normal vaginal delivery Spinal or epidural anesthesia is advised, to minimize the pain and stress of labor. H i g h-Risk Lesions Eisenmenger Synd rome The Eisenmenger syndrome is a consequence of uncorrected long-standing left-to-right shunts. Over time, pulmonary artery pressures approach and can exceed systemic pressures, resulting in reversal of the shunt flow direction from right to left and cyanosis. Eisenmenger syndrome is a possible common endpoint of multiple congenital lesions, including ASD, VSD, and PDA. Maternal mortality in women with Eisenmenger syndrome ranges from 30% to 50%, with a 50% risk of fetal loss if the mother survives. Mortality is frequently caused by compli­ cations of thromboembolic disease. Decompensation occurs most frequently during the first week after delivery Fetal risk due to maternal hypoxemia is substantial, with a high inci­ dence of fetal loss, premature delivery, intrauterine growth retardation, and perinatal death. Because of the considerable risk to both the mother and the fetus, pregnancy is not advised for women with Eisenmenger syndrome. If pregnancy should occur, thera­ peutic abortion is recommended. Women who choose to continue with pregnancy are advised to restrict physical activity, use continuous oxygen for at least the third trimester and consider use of pulmonary vasodilating drugs such as iloprost and prostacyclin. Anticoagulation is recommended

CHAPTER 5 5 • PREGNAN CY AND HEART DISEASE during the third trimester and for 4 weeks after delivery The most vulnerable period for the mother is labor and delivery and the first week postpartum. Vaginal delivery, facilitated by vacuum or low forceps extraction, is the delivery method of choice. Cesarean delivery is associated with a substantially higher mortality than the vaginal route. Anesthetic manage­ ment includes central venous and arterial pressure moni­ toring, with maintenance of adequate SVR and intravenous volume and prevention of sudden increases in pulmonary vascular resistance (PVR) . Complex Cya notic Congen ita l Heart Disease More women born with cyanotic congenital heart disease are surviving to childbearing age. In general, pregnancy is not recommended for women with uncorrected lesions. A low maternal oxygen saturation ( 1 60 to 180 mm Hg. Labetalol, nifedipine, and methyldopa are first-line agents in the treatment of hyperten­ sion in pregnant women. Hypertension due to preeclampsia typically improves within a few days of delivery and should return to baseline by 12 weeks following delivery. Coronary Artery Disease Manifestations of coronary artery disease (CAD) rarely arise during pregnancy. However the increasing prevalence of diabetes, smoking, and obesity, and the trend toward advanced maternal age, have all contributed to recent observations of increased mortality in pregnancy attrib­ utable to acute coronary syndromes. Acute myocardial infarction (MI) is estimated to have occurred in 6.2 per 100,000 deliveries in the USA between 2000 and 2002 . Pregnancy itself is an MI risk factor (risk three to four times above the nonpregnant state) , with most Mls in pregnancy occurring during the third trimester in women over age 33 years who have had multiple prior pregnancies. Coronary spasm, in situ coronary thrombosis, and coronary dissection are possible etiologies (in addition to classic obstructive ath­ erosclerosis) . Acute myocardial infarction may also be the ini­ tial clinical manifestation of an underlying hypercoagulable state, such as the antiphospholipid antibody syndrome. The diagnosis and management of acute myocardial infarction in the pregnant patient should follow established guidelines. Therapy for acute myocardial infarction must be modi­ fied in the pregnant patient. Thrombolytic agents are rela­ tively contraindicated in pregnancy due to the increased risk of maternal hemorrhage but remain a reasonable choice in the setting of an ST elevation Ml without timely percutane­ ous coronary intervention availability. Cardiac catheterization and percutaneous intervention must be performed with lead shielding of the patient's abdomen. Medications that are con­ sidered generally safe include low-dose aspirin, nitrates, beta­ blockers, and short-term heparin. ACE-inhibitors are strictly contraindicated; conversely statins have not been firmly established as teratogens, although given the scarcity of data, avoidance in pregnancy is generally recommended. There is no established data on the safety of clopidogrel and glycopro­ tein Ilb/IIIA inhibitors in pregnancy. In general, bare metal (rather than drug-eluting) stents should be used if required during pregnancy to lower the risk of in-stent thrombosis when clopidogrel is discontinued at the time of delivery: Arrhythmias in Preg na ncy The most frequent rhythm disturbances during pregnancy are premature atrial and/or ventricular complexes. They are not associated with adverse maternal or fetal outcomes and do not warrant antiarrhythmic drug therapy. Atrial fibrillation and atrial flutter are not common during pregnancy and can

TA B L E ••-I Drug Thera py for Mild to Moderate C h ro nic Hypertension i n Preg nancy Drug Class

Example

Fetal Risk

Breast-Feeding

Risk Class0

Safe

B oral form C intravenous

First-Line Agents

£Xz-Adrenergic blockers Methyldopa

Most commonly used Drug of choice Longest safety track record Avoid in women with prior history of depression Second-Li ne Agents

Calcium channel blockers

Nifedipine Amlodipine

Arteriolar vasodilators

Hydralazine

Beta-blockers

Atenolol Metoprolol Labetalol

a,

/3-Blockers

Thiazide diuretics

Hydrochlorothiazide

Avoid sublingual nifedipine (sudden hypotension) Good safety data Has been used effectively, but safety data are lacking Calcium channel blockers have tocolytic effects Possible associations: (a) first-trimester use and hypospadias, (b) thirdtrimester use and neonatal thrombocytopenia, (c) maternal and neonatal lupus-like syndrome, (d) more maternal hypotension and lower 1-min Apgar scores compared to labetalol or nifedipine ( 1 7) Associated with intrauterine growth retardation and newborn bradycardia; avoid atenolol Associated with intrauterine growth retardation and newborn bradycardia May induce fetal lung maturation Associated with neonatal hypoglycemia, thrombocytopenia, hemolytic anemia, and with maternal electrolyte disturbances

Safe No data; avoid Safe

Safe, but infants should be observed for bradycardia Safe, but infants should be observed for bradycardia

c

c c

D (atenolol) C (metoprolol) c

Safe, but may suppress lactation B

Drug Cl asses to Avoid

ACE inhibitors

Captopril Lisinopril

Angiotensin-receptor blockers (ARBs)

Losartan Valsartan

Second- and third-trimester use causes prematurity, intrauterine growth retardation, renal tubular dysplasia and neonatal anuria, severe oligohydramnios, lung and skull hypoplasia, limb contractures, neonatal hypotension No human data, but potentially associated with fetal renal and skull defects, neonatal hypotension and anuria

No data

D (C first trimester)

No data

D (losartan C first trimester)

"Risk Class C: Either studies in animals have revealed adverse effects on the fetus and there are no controlled studies in women , or studies in women are not available. Drug should be given only if the potential benefit justifies the potential risk to the fetus. Risk Class D: There is positive evidence of human fetal risk, but the benefits from use in pregnant women may be acceptable despite the risk.

826

SECTION IX • CLINICAL AND PREVENTIVE CARDIOLOGY

be treated with rate-controlling agents, such as beta-blockers or digitalis, or direct-current cardioversion, which can be performed safely during any stage of pregnancy Anticoagu­ lation is recommended for chronic atrial fibrillation in the setting of underlying structural heart disease. Atrioventric­ ular nodal reentrant tachycardia (AVNRT), the most com­ mon supraventricular arrhythmia in pregnant women, can be treated with adenosine and/or beta-blockers. Direct cur­ rent cardioversion is safe throughout pregnancy, and should be performed with the patient in the left lateral position for any arrhythmia compromising maternal and fetal perfusion. Catheter ablation is the strategy of choice for drug-refractory, hemodynamically significant supraventricular tachycardias and should preferably occur during the second trimester. Ventricular tachycardia (VT) is rare during pregnancy, but when it arises it most commonly originates in the right

ventricular outflow tract with a left bundle morphology and inferior axis. Other etiologies include PPCM, long QT syn­ drome (LQTS) , thyrotoxicosis, or hyperemesis gravidarum. VT associated with structural heart disease is associated with a significant risk of death, and should be promptly treated with cardioversion or medications. Most antiarrhythmic medications used to treat ventricular tachycardia are safe during pregnancy, although amiodarone's iodine component may cause neonatal goiter. Bradyarrhythmias are uncommon during pregnancy Pacemaker support is recommended only if the escape rhythm has an intraventricular conduction delay, the bradyarrhyth­ mia is symptomatic, or there is hemodynamic deterioration. Commonly used antiarrhythmic cardiovascular drugs during pregnancy and their potential side effects are shown in Table 5 5 . 5 .

TA B L E Antiarrhythmic Drugs i n Preg nancy Use During Breast-Feeding

Drug

Use

Fetal Risks

Adenosine Amiodarone

SVT SVT, VT

None reported IUGR, prematurity, hypothyroidism

Beta-blockers

AF, SVT, VT AF, SVT

Fetal bradycardia, hypoglycemia, IUGR None reported; tocolytic effect Possibly low birth weight, prematurity Teratogenic >2 mg!kg/d in animal studies Fetal death; limited data

C metoprolol and carvedilol; D atenolol C verapamil and diltiazem

c

No data; not recommended Safe

Neonatal CNS depression None reported; limited data Possibly embryotoxic at very high doses Neonatal thrombocytopenia, mild oxytocic effect Fetal bradycardia, IUGR

B C has been safely used in third trimester c

Safe Safe

Calcium-channel blockers Digoxin Dofetilide Flecainide Lidocaine Procainamide Propafenone Quinidine Sotalol

AF, atrial flutter AF AF, SVT, VT VT SVT, VT AF, SVT, VT SVT, VT AF, SVT, VT

Risk Class0

c D

C but has been used in pregnancy for decades c

No data Significant amiodarone and iodine secretion; not recommended Safe; observe for bradycardia Safe; observe for bradycardia Safe

c

No data; not recommended Relatively safe

B (all trimesters, previously D in second and third)

Not recommended

IUGR, intrauterine growth retardation; SVT, supraventricular tachycardia; VT, ventricular tachycardia; AF, atrial fibrillation; CNS, central nervous system. 'Risk Class B: Either animal reproduction studies have not demonstrated a fetal risk but there are no controlled studies in pregnant women, or animal reproduction studies have shown an adverse effect that was not confirmed in controlled studies in women in the first trimester (and there is no evidence of risk in later trimesters). Risk Class C: Either studies in animals have revealed adverse effects on the fetus and there are no controlled studies in women, or studies in women are not available. Drug should be given only if the potential benefit justifies the potential risk to the fetus. Risk Class D: There is positive evidence of human fetal risk, but the benefits from use in pregnant women may be acceptable despite the risk. Data retrieved from Micromedex 2 . 0 , correct as of October 1 st, 20 1 1 .

CHAPTER 5 5 • PREGNAN CY AND HEART DISEASE Abnormal Glucose Reg ulation During Preg na ncy In normal pregnancy, hormonally induced insulin resistance is countered by a compensatory increase in insulin secretion. Inadequate compensation caused by dysfunction of pancre­ atic f3 cells likely contributes to the development of gestational diabetes. Women who develop gestational diabetes prior to the 24th week of pregnancy face an 80% chance of develop­ ing type 2 diabetes mellitus within 5 years. Moreover, women who develop gestational diabetes have a greater risk of hyper­ tension, hyperlipidemia, electrocardiographic abnormalities, and overall mortality: Maternal hyperglycemia is associated with an increase in fetal and neonatal morbidity, including macrosomia, congenital malformations, fetal HCM, and neo­ natal hypoglycemia. Women diagnosed with gestational dia­ betes require close follow-up for primary prevention of type 2 diabetes, including dietary modification, maintenance of body weight control, and regular exercise. There is some evi­ dence that insulin-sensitizing agents may reduce the rate of conversion from gestational diabetes to type 2 diabetes. A N T I COAG U LAT I O N D U R I N G P R E G N A N CY Several conditions require the initiation or the maintenance of anticoagulation during pregnancy, including mechanical prosthetic valves, hypercoagulable states, prior or current deep venous thrombosis, and Eisenmenger syndrome. The three most common agents considered for use during preg­ nancy are unfractionated heparin (UFH) , low-molecular­ weight heparin (LMWH) , and warfarin. The Eighth American College of Chest Physicians (ACCP) Consensus Conference on Antithrombotic Therapy recommended three potential strategies for anticoagulation during pregnancy

827

(Table 55-6) : (a) adjusted-dose LMWH administered twice daily throughout pregnancy, (b) adjusted-dose UFH admin­ istered subcutaneously twice daily throughout pregnancy, or (c) UFH or LMWH until the 1 3th week, warfarin from week 1 3 to just before delivery and then UFH or LMWH until after delivery. There is the additional caveat that indi­ viduals with the highest risk metallic valves (such as older valve designs in the mitral position, or patients who have already suffered a stroke) may be managed with warfarin throughout pregnancy: Nevertheless, the choice of anticoag­ ulation regimens depends on the preferences of the patient and physician after consideration of the maternal and fetal risks associated with each drug. Warfarin crosses the placental barrier freely and can result in warfarin embryopathy (abnormalities of fetal bone and cartilage formation) . The risk of warfarin embryopathy has been estimated at 4% to 1 0 % , but is minimized when the daily dose is 60% of these individuals are women. However, there is a lower prevalence of hyper­ tension in women until the sixth decade at which point the prevalence in women begins to exceed that in men. Hyper­ tension is more likely to cause a cardiovascular event in women and is a risk factor for congestive heart failure in women. Nonetheless, women are more likely than men to be aware of their diagnosis, to receive treatment, and to reach target blood pressure. Etiology

The origin of hypertension is predominantly essential. Eti­ ologies that are seen exclusively or disproportionately in women include hypertension associated with pregnancy or use of oral contraceptives (especially older agents with higher doses of estrogens and progestins) and renovascular hypertension, which has a female:male ratio of 8: 1 . Treatment

Antihypertensive treatment recommendations are generally similar for women as compared to men except in certain situations. • Pregnancy: Methyldopa is generally preferred and ACE-inhibitors and angiotensin receptor blockers are contraindicated. • Elderly: Thiazide diuretics may be helpful as they have been shown to reduce hip fracture.

The following side effects have been demonstrated to be more frequent in women: • •

Hyponatremia Hypokalemia

• ACE-inhibitor induced cough • •

Peripheral edema from calcium channel blockers Hirsutism from minoxidil

Smoking Cigarette smoking is the leading preventable cause of death in men and women in the United States. Women who are heavy smokers (>20 cigarettes a day) have two- to fourfold increased risk of coronary disease compared with nonsmokers. Even light smokers (1 to 4 cigarettes a day) have a two- to three­ fold risk of fatal coronary heart disease (CHD) or nonfatal MI. Second-hand smoke may increase risk of CAD by 20%. The prevalence of smoking has declined in the United States since 1 965 , but more men have been successful at quitting than women. There are multiple factors in women's failure to quit, including fear of weight gain and lack of con­ fidence. Mech a n i s m s by Which Smoking May I n c rease Risk of CAD

Chronic smokers are insulin resistant, hyperinsulinemic, and dyslipidemic compared to nonsmokers. (There is a dose­ response relationship between number of cigarettes smoked and plasma cholesterol, possibly due to increased lipolysis.) Second-hand smoke may cause intimal wall damage and accelerate atherosclerotic plaque formation. In young women, the combination of smoking and use of oral contra­ ceptives promotes thrombogenesis. Smoki n g Cessation Decreases Risk

The Nurses' Health Study showed a 30% decrease in risk for CAD in 2 years following cessation of smoking. There is continued decline in risk over next 10 to 15 years, and then the risk is at the same level as that of a nonsmoker. Most studies show a 30% to 50% decrease in risk for CAD in the first 2 years following cessation. Obesity and Sedenta ry Lifestyle Obesity is an independent risk factor for CAD in women. In the Framingham Heart Study, relative weight in women was positively and independently associated with the develop­ ment of CAD as well as mortality from CAD and cardiovascu­ lar disease. In the Nurses' Health Study, women with a body mass index (BMI) of 25 to 29, had an age-adjusted relative risk for CAD of 1 .8, whereas those with BMI � 29 had a rela­ tive risk of 3 . 3 , compared with women of ideal BMI. Truncal obesity correlates with increased LDL-C and decreased HDL-C and also is associated with hyperinsulinemia and hypertension. Although obesity is associated with diabetes

CHAPTER 5 6 • WOMEN AND HEART DISEASE (insulin resistance) , hypertension, and hypercholesterolemia, the waist/hip ratio still correlates positively with CAD after controlling for smoking, hypertension, glucose intolerance, lipids, and BMI.

AHA 201 1 G U I DELI N ES FOR H E A RT D I S E A S E P R EV E N T I O N I N WO M E N The 20 1 1 American Heart Association/American College of Cardiology (AHNACC) guidelines for heart disease prevention in women are evidence based and provide individual recom­ mendations for women (see Tables 56. 1 and 56.2). In general, the recommendations differ very little from those for men. Women are grouped into high-risk (includes risk for CHD event of :2:10% over the next 1 0 years, women with CHD , cerebrovascular disease, diabetes mellitus, periph­ eral arterial disease, chronic kidney disease, or abdominal aortic aneurysm) , at-risk (includes women with systemic autoimmune collagen-vascular disease, history of preec­ lampsia, pregnancy-induced hypertension, or gestational diabetes along with other traditional risk factors) , and ideal cardiovascular health. As compared to the 2007 guidelines, the 20 1 1 guidelines have decreased the threshold at which women are considered to be high-risk (from risk for CHD event over the next 10 years of :2:20% to :2: 1 0%) .

H I S TO R I C A L P E R S P E CT I V E : E S T R O G E N T H E R A PY F O R P R E V E N T I O N O F C A R D I OVA S C U LA R D I S E A S E Physiologic Effects of Estrogen Benefits • Beneficial effects on lipid profile • Antioxidant effects • Reduction of serum fibrinogen • Inhibition of neointimal hyperplasia, smooth muscle cells, and collagen biosynthesis

835

Popu lation Data Observational studies (e.g. , the Nurses' Health Study) showed a significant reduction in MI or death among post­ menopausal estrogen users. Some randomized controlled trials of HRT for secondary prevention of cardiovascular disease have been undertaken. The HERS (Heart and Estrogen/progestin Replacement Study) was the first randomized controlled trial of HRT for prevention of CHD . It looked at estrogen and progestin ver­ sus placebo in postmenopausal women with prior MI, coro­ nary revascularization, or angiographic evidence of CAD. Results showed, over a mean follow-up period of 4. 1 years, no difference in the rates of nonfatal MI and coronary death, and a 52 % increase in cardiovascular events in the first year of HRT. The ERA (Estrogen Replacement and Atherosclerosis) trial was the first randomized angiographic-endpoint trial to test the effect of estrogen replacement therapy (ERT) and HRT on the progression of atherosclerosis in postmenopau­ sal women with documented CAD. Neither the HRT (estro­ gen and progestin) nor the ERT (estrogen only) showed any angiographic benefit on disease progression. In a randomized controlled trial of HRT for the primary prevention of CHD , the Women's Health Initiative (WHI) studied estrogen plus progestin in > 1 6 ,000 postmenopausal women aged 50 to 79 years. Primary outcomes were nonfatal MI or death from CHD. Mean follow-up was 5 . 2 years, but the trial was stopped early because the overall risks exceeded the benefits, with a hazard ratio for CHD of 1 .24. The study's conclusions stated: "Estrogen plus progestin does NOT con­ fer cardiac protection and may increase the risk of CHD among generally healthy postmenopausal women, especially during the first year of treatment. " The separate Estrogen Only arm of the WHI was stopped because the hormone increased the risk of CVA and did not reduce the risk of CHD. Thus, based on the randomized, placebo-controlled trials, hormone replacement therapy (estrogen and progestin or estrogen alone) is not indicated for either primary or sec­ ondary prevention of cardiovascular disease in women and may in fact increase risk.

• Potentiation of endothelium-derived relaxing factor • Calcium channel blocking effect • Increases prostacyclin biosynthesis • Decreases insulin resistance • May cause favorable distribution of body fat

Risks • Breast cancer: relative risk is 1 . 35 with > 1 0 years of hormone-replacement therapy (HRT) . • Endometrial cancer: relative risk is 8.22 with >8 years of HRT. • Risk of deep vein thrombosis (DVT)/pulmonary embolism (PE) is doubled with HRT. • Risk of gall bladder disease is doubled with HRT.

C L I N I C A L P R E S E N TAT I O N O F C A D I N WO M E N The most common presentation of CAD in women is angina pectoris (typical and atypical) , in contrast to men, who present most commonly with MI or sudden cardiac death. Women who present with acute coronary syn­ drome (ACS) are more likely to be older and to have more comorbidities than men, including hypertension, diabe­ tes , hyperlipidemia, and congestive heart failure . Women are less likely to have had a prior MI or revascularization. In MI, both men and women present with chest pain as the most common symptom; however, women are more likely

G u ideli nes for Prevention of CAD in Women: C l i n ica l Recommendations

Lifestyle I nterventions

Cigarette smoking

Women should be advised not to smoke and to avoid environmental tobacco smoke. Provide counseling at each encounter, nicotine replacement, and other pharmacotherapy as indicated in conjunction with a behavioral program or formal smoking cessation program (Class I, Level at Evidence B) . Physical activity

Women should be advised to accumulate at least 1 5 0 min/wk of moderate exercise, 75 min/wk of vigorous exercise, or an equivalent combination of moderate- and vigorous-intensity aerobic physical activity. Aerobic activity should be performed in episodes of at least 1 0 min, preferably spread throughout the week (Class I, Level of Evidence B) . Women should also be advised that additional cardiovascular benefits are provided by increasing moderate-intensity aerobic physical activity to 5 h 300 min/wk, 2 1/2 h/wk of vigorous-intensity physical activity, or an equivalent com­ bination of both (Class I, Level of Evidence B) . Women should be advised to engage in muscle-strengthening activities that involve all maj or muscle groups performed on :2'.2 d/wk (Class I, Level of Evidence B) . Women who need to lose weight or sustain weight loss should be advised to accumulate a minimum of 60-90 min of at least moderate-intensity physical activity (e.g. , brisk walking) on most, and preferably all, days of the week (Class I, Level of Evidence B) . Cardiac rehabilitation

A comprehensive CVD risk-reduction regimen such as cardiovascular or stroke rehabilitation or a physician-guided home- or community-based exercise training program should be recommended to women with a recent acute coronary syndrome or coronary revascularization, new-onset or chronic angina, recent cerebrovascular event, peripheral arterial disease (Gass I, Level of Evidence A) , or current/prior symptoms of heart failure and an LVEF �35% (Class I, Level of Evidence B). Dietary intake

Women should be advised to consume a diet rich in fruits and vegetables; to choose whole-grain, high-fiber foods; to consume fish, especially oily fish, at least twice a week; to limit intake of saturated fat, cholesterol, alcohol, sodium, and sugar; and avoid trans-fatty acids. See Appendix (Class I, Level of Evidence B) . Note: Pregnant women should be counseled to avoid eating fish with the potential for the highest level of mercury con­ tamination (e.g. , shark, swordfish. king mackerel, or tile fish) . Weight maintenance/reduction

Women should maintain or lose weight through an appropriate balance of physical activity; caloric intake, and formal behavioral programs when indicated to maintain or achieve an appropriate body weight (e.g. , BMI 90 % . Because it is cleared renally, it is contraindicated in patients with severe renal insufficiency (creatinine clearance 75 years ( 1 3 . 3 % vs. 4. 1 %) and in patients with diabetes (7.0% vs. 2.2%). EXTRACT.43 Enoxaparin was compared to UFH as adjunct therapy

with fibrinolytic therapy in patients with STEMI. With fibrino­ lytic therapy, patients received either UFH for at least 48 hours or enoxaparin. Enoxaparin was continued through the hospital stay, or a maximum of 8 days. Primary endpoint was composite of death from any cause, or nonfatal recurrent MI through 30 days that was 9 . 9 % for enoxaparin and 1 2 . 0 % for UFH (0.83 RR; 95% CI, 0.77 to 0.90, p < 0 . 0 0 1 ) . There was no difference in death between enoxaparin (6.9%) and UFH (7.5%) (p = 0 . 1 1) . There was a significant increase in the rate o f bleeding with enoxaparin (2 . 1 %) compared to 1 .4% for UFH ( 1 .53 RR; 95% CI, 1 . 23 to 1 .89, p < 0 00 1).

SYNERGY.44 Enoxaparin was compared to UFH for efficacy and

safety in high-risk patients with non-ST-segment elevation ACS managed with an early invasive strategy. Primary endpoint of death or nonfatal MI by 30 days was 14.0% (696/4,993) in the enoxaparin group and 1 4 . 5 % (722/4,985) in the UFH group (HR, 0.96; 95% CI, 0.86 to 1 .06). Enoxaparin was not superior but did meet noninferiority criteria. Enoxaparin had an increase in TIMI maj or bleeding (9 . 1 % vs. 7 . 6 % , p = 0 . 008) , but not in GUSTO (Global Utilization of Streptokinase and t-Pa for Occluded Arteries) severe bleeding (2 .7% vs. 2.2%).

depolarizes. Another is a sodium-calcium channel, which moves one calcium ion out in exchange for three sodium ions entering into the cell. The electrical balance within a cell is accomplished by the sodium/potassium ATP channel. Calcium is normally stored in the sarcoplasmic reticulum, which also helps control the level of intracellular calcium. There are three types of calcium channels, L, N, and T. They differ in distribution in various tissues, duration of opening, and voltage range. Only L and T are of interest in cardiology L-Type Channels L-type channels are found in heart muscle and in parts of the conducting system, smooth muscle, brain, adrenals, and kidneys. Although all calcium channel blockers bind to the calcium channel receptor, they have different binding sites. Blocking the L channels or receptors inhibits inward calcium currents into the cell, thus reducing the concentration of cal­ cium needed for muscle contraction, which leads to smooth muscle dilation, decreasing contractility of heart muscle, slowing of the sinoatrial (SA) node firing rate, and increas­ ing AV nodal conductance time. T-Type Channels T-type channels are found in blood vessels, adrenals, brain, kidneys, the heart conduction system, and in heart muscle under pathologic conditions such as cardiomyopathy: There is only one identified binding spot for medications, and blocking this receptor leads to dilation of peripheral and coronary blood vessels, thus decreasing SVR and increasing myocardial blood flow. Also , blockade of T-type channels produces a lowering of heart rate. Mech a n i s m of Action

The various classes of calcium channel blockers and the drugs in each class, as well as their mechanism of action, are shown in Table 6 1 . 7 . S i d e Effects

The side effects and contraindications for calcium channel blockers are shown in Table 6 1 .8. I n d ications a n d Preca utions

The various preparations and respective doses along with the labeled uses are shown in Table 6 1 . 9 . Major C l i n ica l Tri a l s ASCOT-BPLA.45 The calcium channel blocker amlodipine was

C A LC I U M C H A N N E L B LO C K E R S This class o f drugs is chemically and pharmacologically diverse. Before we classify the drugs, we will review some information on the various calcium channels. The main ports of entry for calcium into the cell are via voltage­ gated calcium channels, which open when the membrane

compared to atenolol as an antihypertensive regimen in patients with risk factors for coronary artery disease other than hyper­ tension. The study was stopped early because the all-cause mor­ tality rate was significantly lower with the amlodipine strategy than with the atenolol strategy. The amlodipine-based regimen was associated with reduced rates of strokes, cardiovascular death, and new-onset diabetes, and also total cardiovascular events and procedures.

CHAPTER 6 1 • CARD IOVASCULAR MEDICINE-ESSENTIAL PHARMACEUTI CALS

893

Calcium C h a n n e l Blockers Su bclass

Drugs Available i n Su bclass

Mechanism of Action

Dihydropyridines

Amlodipine (Norvasc) Felodipine (Plendil) Isradipine (Dynacirc) Nicardipine (Cardene) Nifedipine (Procardia, Adalat) Nimodipine (Nimotop) Nisoldipine (Sular) Diltiazem ( Cardizem, Dilacor, Tiazac) Verapamil ( Calan, Isoptin, Verelan)

Blocks Ca2• channel in smooth muscle, which leads to vasodilatation of peripheral and coronary arteries. Effect on sinus and AV nodes is negated by reflex increase in sympathetic tone.

N ondihydropyridines

Blocks Ca2 • channel in smooth muscle, which leads to vasodilatation of peripheral and coronary arteries as well as decrease in heart rate and prolonging conduction across the AV node.

AV, atrioventricular.

D I U RETICS By definition, diuretics (except osmotic diuretics) are drugs that lead to a net loss of sodium (Na•) and water from the body. These drugs (except spironolactone) work primarily in the kidneys, acting from within the tubular lumen. Hence, for these drugs to reach their target, they are secreted into

the proximal tubule. The sites of action for these drugs are shown in Figure 6 1 .3 . Loop Diuretics Loop diuretics are the most powerful diuretics, causing about 1 5 % to 20% of sodium in the tubule to be excreted, hence the name "high-ceiling diuretics. "

Side Effects, Contraindications, and Major Drug I nteractions o f Ca lcium Channel Blockers Su bclass

Side Effects

Contraindications

Drug I nteractions

Dihydropyridines

Headache, peripheral edema, flushing, reflex tachycardia (short-acting agents have higher inci­ dence) , rash, dizziness, hypotension, gingival hyperplasia (nifedipine)

Hypersensitivity to the medications; short­ acting agents should not be used for hyper­ tensive urgencies; AMI; acute stroke.

Diltiazem and verapamil

Negative inotropic effects, nausea, bradycardia, dizziness, peripheral edema, hypotension, heart block, constipation

AMI; heart block greater than first degree; heart failure; pulmo­ nary edema

Grapefruit juice with certain dihydropyridines. Fentanyl has been reported to cause severe hypotension when given with certain calcium channel blockers. This reaction may occur with all calcium channel blockers, but data are not available. H2 -receptor antagonists may increase the bioavailability of many of the dihydropyridine calcium channel blockers. H 2 -receptor antagonist may increase bioavailability of diltiazem. Beta-blockers may increase negative inotropic and chronotropic effects. Inhibit metabolism of carbamazepine, cyclosporin, digoxin, quinidine, and theophylline.

894

SECTION X • PHARMACOLOGY

Preparations, Dosi ng, and Labeled Uses for Ca lci u m Channel Blockers Drug

Ind ications

Amlodipine

Hypertension, stable and vasospastic angina Hypertension Hypertension Hypertension and stable angina Hypertension and vasospastic angina; unlabeled uses include prevention of migraine headaches, primary pulmonary hypertension Hypertension Hypertension, angina, and supraven­ tricular tachyarrhythmias Hypertension, angina, and supraventricular tachyarrhythmias; unlabeled uses are for migraine and cluster headaches, exercise­ induced asthma, and hypertrophic obstructive cardiomyopathy

Felodipine Isradipine Nicardipine Nifedipine

Nisoldipine Diltiazem" Verapamil"

"Diltiazem and verapamil are available as IV formulations that can used in the acute management of atrial fibrillation and supraventricular tachyarrhythmias.

acute renal failure, hyperkalemia, anion overdose, and acute pulmonary edema. Thiazide Diuretics Mech a n i s m of Action

Thiazide diuretics decrease active reabsorption of sodium and accompanying chloride by binding to the chloride site of the Na•/Cl- cotransport system in the distal convoluted tubule. Furthermore, they increase calcium reabsorption in the same region of the nephron. Drugs of this class are effective at low doses, for which reason they are called "low­ ceiling diuretics." Side Effects

Side effects of thiazide diuretics include hypokalemia, hypouricemia, glucose intolerance, hyperlipidemia, hypona­ tremia, allergic reaction, weakness, fatigue, and photosensitivity I n d ications a n d Preca utions

The various drugs in this subgroup are shown in Table 6 1 . 1 0 . Thiazides are not effective i n patients with clearance a) . Hence it leads to increased myocardial contractility and vasoconstriction. Side Effects Common side effects of epinephrine include tachycardia, flushing, hypertension, restlessness, exacerba­ tion of narrow-angle glaucoma, and ventricular arrhythmia. Indications and Preca utions

Epinephrine is indicated for use in ventricular standstill (cardioplegia or cardiac arrest) and shock (particularly anaphylaxis) . lsoproterenol ( l s u p rel) Mechanism o f Action Isoproterenol is a nonselective /3-receptor agonist (/31 > /32 ). It has the most potent inotropic effect of any inotrope. Side Effects Common side effects of isoproterenol include tachycardia and ventricular arrhythmia (the reason it is used in the electrophysiologic laboratory to stimulate tachycar­ dia) , hypotension, myocardial ischemia, mild tremor, nerv­ ousness, and flushing. Preparation, Dosing, Indications, and Preca utions Iso­ proterenol is labeled for use in shock, heart block, Adams­ Stokes attacks, and bronchospasm. Unlabeled uses include for bradycardia and for torsades de pointes until temporary pacing can be established.

M i l ri n o n e (Primacor) Mechanism of Action

Milrinone is a phosphodiester­ ase inhibitor that results in increased levels of intracellular cAMP by blocking its degradation, thus increasing contrac­ tility as well as vasodilatation because of its effect on arterial smooth muscle. Side Effects

Common side effects of milrinone include hypotension, ventricular arrhythmia, supraventricular tachy­ cardia, angina, chest pain, headache, and thrombocytopenia.

Ind ications and Preca utions Milrinone is labeled for short-term use in the management of CHF Similar to dobu­ tamine, it was found to increase mortality with long-term therapy, secondary to increasing ventricular arrhythmias.

Vasopressors Norepi n e p h ri n e (Levop hed) Mechanism of Action

agonist (/31

=

Norepinephrine is a mixed f3 and a a > /32) . Its primary effect is vasoconstriction.

Side Effects

Common side effects of norepinephrine include hyperten­ sion, headache, trembling, and ventricular arrhythmias. Ind ications and Precautions

The labeled use for norepinephrine is hypotension. It should be administered via a central line because of the risk of skin necrosis with extravasation. It should also be used with cau­ tion in patients with hepatic dysfunction or ischemic bowel, because it leads to splanchnic and hepatic vasculature con­ striction. Phenyl e p h ri n e (N eo-Syn eph rine) Mechanism of Action

Phenylephrine is a pure a-receptor agonist that has a vaso­ constrictor effect. Side Effects Common side effects include bradycardia, hypertension, and myocardial ischemia. I n d ications a n d Preca utions Phenylephrine is labeled for use in hypotension, particularly when associated with septic shock (vasodilatation) , and anesthetic hypotension. Furthermore, it may be used to counter hypotension due to vasodilatation in severe obstructive hypertrophic cardio­ myopathy.

N I T RAT E S Nitrates are a group o f drugs that are a source o f N O , which produces vasodilatation in the coronary circulation as well as arterioles and veins. NO mediates vasodilatation via cAMP. These effects contribute to their antianginal properties as well as their role in heart failure. Nitrates that are in use are summarized in Table 6 1 . 1 1 .

CHAPTER 6 1 • CARD IOVASCULAR MEDICINE-ESSENTIAL PHARMACEUTI CALS

897

N itrates: Mechanism o f Action, S i d e Effects, Dosing, and Ind ications Drug

Mechanism of Action

Side Effects and Preca utions

Dosing and Ind ications

ISDN (Isordil, Sorbitrate)

Biotransformation of nitrates releases NO, causing vasodilatation through cAMP. Venous dilation predominates. Same

Headache, hypotension (large doses) , flushing, dizziness, rash, nausea, methemoglobinemia, reflex tachycardia

Same

Same

Same; inconvenient for longterm therapy secondary to ointment formulation Same

Labeled: prevention of anginal attacks Unlabeled: heart failure, in combination with hydralazine) Labeled: prevention of anginal attacks Labeled: prevention of anginal attacks

Same

Same

Same

Same; methemoglobinemia may occur with high doses.

Isosorbide mononitrate (Imdur, Ismo, Monoket) Nitroglycerin paste (Nitrol) Nitroglycerin patch (various) Nitroglycerin SL (Nitrostat, Nitrolingual spray) Nitroglycerin IV (Tridil)

Same

May increase intracranial pressure IV formulation is poorly soluble

Same

Some of the maj or trials studying the role of nitrates in the treatment of heart failure are V-HeFT 1 1 The addition of hydralazine and isosorbide dinitrate

(ISDN) to standard therapy (digoxin and diuretics) improved mortality and LV function compared to placebo or prazosin in patients with heart failure . V-HeFT II.17 Hydralazine and ISDN were compared to enalapril

in men with chronic CHF receiving digoxin and diuretics. In the enalapril patients, mortality at 2 years was reduced ( 1 8%) compared to H-ISDN patients (25 % ) , p 0 . 0 1 6 . =

A-HeFT.47 The addition o f a fixed dose o f ISDN and hydralazine

to standard therapy for heart failure, including neurohormo­ nal blockers, is efficacious and increases survival among black patients with advanced heart failure.

P LAT E L ET I N H I B I TO R S Thrombus formation is a complex process that involves platelets, the vasculature including collagen and tissue fac­ tors (primary hemostasis) , as well as the coagulation path­ ways (secondary hemostasis) , as shown in Figure 6 1 .2 . The initiation o f thrombus formation in the arterial system

Labeled: prevention of anginal attacks Labeled: acute treatment or prophylaxis of anginal attacks Labeled: perioperative hypertension, CHF with acute MI, angina Unlabeled: hypertensive crisis, pulmonary hypertension

mainly involves platelet aggregation with a small amount of fibrin (white clot) , whereas in the venous system, it is com­ posed mainly of fibrin and red cells (red clot) . The primary functions of the platelets are to form a plug by adhesion and aggregation, as well as providing a phospholipid surface to facilitate procoagulant reaction. For the platelets to participate in coagulation, they have to be activated. This usually happens with exposure of the platelets to collagen in the vasculature (injury) . Platelets adhere to the collagen via GP lb-IX, leading to change in shape and granular release of ADP and thromboxane. Throm­ bin (from the coagulation pathway) , angiotensin II, norepi­ nephrine, and contents of granules released from platelets (including ADP and serotonin) stimulate the endothelium to release calcium and hence allow platelet activation. Factors that inhibit platelet activation are prostacyclin and NO. When platelets are activated, arachidonic acid in the endothelium is transformed to thromboxane A2 (TXA2) , by cyclooxygenase enzyme (COX). Thromboxane leads to more platelet activa­ tion. This step is the target for aspirin use as an antiplatelet therapy Serotonin, thrombin, and ADP bind to receptors on the endothelium, and via secondary messengers lead to release

898

SECTION X • PHARMACOLOGY

of calcium from the endoplastic reticulum. The binding of ADP to receptors on the endothelium is the site of action for clopidogrel, prasugrel and ticlopidine as antiplatelet therapy As mentioned earlier, when platelets are activated, TXA2 is released. TXA2 leads to the expression of GP Ilb/Illa recep­ tors, which allows linkage of adjacent platelets (aggregation) by fibrinogen and von Willebrand factor (vWF) to the GP Ilb/Illa. The GP Ilb/Illa receptors are the site of action of IV antiplatelet therapy (abciximab, tirofiban, and eptifibatide). Ora l Anti platelet Therapy Cyclooxygenase I n h i bito r/As pirin Mechanism o f Action Aspirin irreversibly acetylates plate­ let cyclooxygenase, decreasing the formation of TXA2 from arachidonic acid. Side Effects Side effects include bleeding, gastric ulcera­ tion, nausea/dyspepsia/heartburn, hemolytic anemia, and tinnitus (large doses or overdose) . I n d i cations a n d Preca utions Aspirin is labeled for analgesic, antipyretic, and anti-inflammatory use, as well as for MI, transient ischemic attacks, CVAs, and adjunct therapy for revascularization procedures (coronary artery bypass grafts, PTCA, and carotid endarterectomy) and stent implantation.

Clopidogrel (Plavix) Mechanism of Action Clopidogrel acts by inhibiting the P2Y 12 component of adenosine diphosphate receptors, which decreases the expression of the GP Ilb/Illa receptors on the platelet cell surface and thereby prevents platelet aggregation. Side Effects Common side effects of clopidogrel include bleeding, diarrhea, headache, dizziness, abdominal pain/ nausea/dyspepsia, purpura, rash, and sometimes thrombo­ cytopenia. Ind ications and Precautions

The drug is labeled for use in ACS and to reduce the risk of MI, stroke, and/or periph­ eral arterial disease in patients with a completed MI, stroke, and/or peripheral arterial disease. It reduces rates of athero­ thrombotic events in patients with UA or non-ST-elevation MI who are medically managed or with PCI (with or with­ out stent placement) . Most side effects when compared to aspirin in clinical trials were less in the clopidogrel-treated group. Maximal effects are seen 3 to 7 days after initiation of therapy Cases of thrombotic thrombocytopenic purpura (TTP) have been reported. Recent data demonstrate that clopidogrel should be continued for 12 months after coro­ nary artery stenting with a drug-eluting stent.64 Pra s u g rel (Effient) Mechanism of Action This is a prodrug which when con­ verted to its active metabolite, irreversibly inhibits the P2Y 12

portion of the ADP receptor on platelets to prevent activa­ tion of the GP Ilb/Illa receptor, reducing platelet activity Side Effects Common side effects include bleeding, hyper­ tension, headache, and nausea. Indications and Preca utions Prasugrel is indicated to reduce rates of thrombotic cardiovascular events in patients with UA, NSTEMI, or STEMI managed with PCI. Due to the increased concern for bleeding, use with other anticoagulants should be cautioned. Use is contraindicated in patients with history of transient ischemic attack or stroke. Use in patients > 75 years of age is not recommended due to increased risk of fatal and intracranial bleeding and uncertain benefit. For patients of low weight (300 mg in 6 1 % vs. 4% in ROW) of aspirin. Ticagrelor patients in the cohort on >300 mg doses of aspirin had worse primary efficacy outcomes compared to clopidogrel (HR 1 .45 , 95% CI, 1 . 0 1 to 2 . 09) . Clopidogrel in combination with high- or low-dose (< 1 00 mg daily) aspirin did not have any difference in outcomes. Ticagrelor compared to clopidogrel with low-dose aspirin had the lowest risk of car­ diovascular death, Ml, or stroke. =

=

=

900

SECTION X • PHARMACOLOGY

I ntravenous Anti platelet Therapy/ Glycoprotein l l b/l lla I n h i bitors Refer to Table 6 1 . 1 2 for details. Major C l i n ical Tri a l s PRISM.60 This study compared the effects of tirofiban with aspirin

versus heparin with aspirin on clinical outcomes in patients with UA. There was no difference between the two groups in terms of the combined endpoints at 30 days. Mortality alone was significantly reduced in the tirofiban group. RESTORE.61 The study focused on the effects of tirofiban on

adverse cardiac events in patients with UA who were undergo­ ing PCI. The study showed that tirofiban reduced the primary outcome, which was a composite of death, MI, PCI failure, and coronary artery bypass grafting (CABG) . However, the reduc­ tion in emergency revascularization seen early in the study was no longer significant at 30 days. TARGET.62 TARGET compared tirofiban and abciximab for pre­

vention of ischemic events with PCI. Abciximab was more effec­ tive than tirofiban in preventing nonfatal MI as well as in the composite endpoint of death, MI, or urgent target vessel revas­ cularization. TACTICS-TIMI-1 8 . 63 This trial compared early invasive to con­

servative therapy in patients with ACS treated with tirofiban. The study showed that the strategy of early catheterization and revascularization was associated with fewer maj or cardiac events than the conservative approach. PURSUIT.64 This study aimed to determine the effects of epti­

fibatide in patients with ACS between those undergoing PCI versus those being managed conservatively. The study found that eptifibatide reduced the composite endpoint of death or MI at 30 days with either strategy of management . ESPIRIT. 65 This study was aimed at assessing the efficacy and safety

of high-dose eptifibatide in elective coronary stent implanta­ tion. The study showed that eptifibatide reduced ischemic com­ plications after elective stent placement, as well as the combined endpoint of death and MI.

IMPACT-11.66 This study aimed at assessing eptifibatide impact on

the prevention of ischemic complications following PCI. The study showed that the use of eptifibatide was associated with reduced early abrupt closure and reduced the rates of 30 days ischemic events without increasing the risk of bleeding. How­ ever, there was no effect on reduction of 30-day mortality or MI, or 6-month cumulative ischemic event rate. EPI C . 67 This study showed that abciximab bolus and infusion at

the time of PTCA improved outcomes for as long as 3 years. It also showed that there was reduction in NQWMI and dis­ tal embolization in patients undergoing PCI on saphenous vein grafts. EPILOG.68 This trial was aimed at studying whether the clini­

cal benefit of abciximab on reducing ischemic complications in patients undergoing high-risk PCI can be extended to all patients undergoing PCI. Furthermore, the study looked at whether adjusting the heparin dose reduced the hemorrhagic complications associated with abciximab. The study showed that abciximab with low-dose heparin reduced ischemic com­ plications in patients undergoing PCI without increasing the risk of bleeding. EPISTENT.69 The purpose of this trial was to compare the out­

comes of stenting with or without the use of abciximab, and the outcomes of PTCA with abciximab. The study showed that abciximab significantly improved the outcome of PCI. Further­ more, PTCA with abciximab was better and safer than stenting without abciximab. CAPTURE.70 Abciximab infusion started 18 to 24 hours before PCI

and continued for 1 hour after PCI reduced the rates of peripro­ cedural MI and the need for revascularization in patients with UA having PTCA, without affecting the rates of MI. RAPPORT.71 This trial showed that abciximab in the setting of pri­

mary PCI for acute STEM! did not alter the primary endpoint at 6 months-a composite endpoint of revascularization (elec­ tive or urgent) , death reduction, or reinfarction. There was an increased risk of bleeding. ADMIRAL.72 This trial compared the effects of early administra­

tion of abciximab before primary stenting to stenting alone

G lycoprotei n l l b/l l la I n h i bitors: Prepa rations, Mechan ism of Action, and Side Effects Drug

Mechanism of Action

Side Effects

Abciximab (Reopro)

Murine-derived monoclonal antibody Fab fragment binds to the human GP Ilb/Illa receptor on the platelet surface, inhibiting platelet aggregation. Heptapeptide antagonist that reversibly inhibits GP Ilb/ Illa receptor on the platelet surface, inhibiting platelet aggregation Nonpeptide antagonist that reversibly inhibits GP Ilb/ Illa receptor on the platelet surface, inhibiting platelet aggregation

Bleeding, thrombocytopenia, hypersensitivity reactions

Eptifibatide (Integrilin) Tirofiban (Aggrastat)

Bleeding, thrombocytopenia Bleeding, thrombocytopenia

CHAPTER 6 1 • CARD IOVASCULAR MEDICINE-ESSENTIAL PHARMACEUTI CALS without Ilb/IIIa-inhibitor therapy. The results showed that early abciximab (before primary PCI for STEMI) improved vessel patency before and after stenting and at 6 months follow-up after the procedure. It was associated with improved clinical outcomes and LV function preservation compared to primary PCI alone .

T H R O M B O LY T I C S / F I B R I N O LYT I C S When the coagulation pathways are activated, there is a naturally occurring counter process of "clot dissolving" that begins spontaneously (Fig. 6 1 .2). This process involves plasminogen activators, tissue-type plasminogen activator (t-PA) , and urokinase-type plasminogen activator (u-PA) . t-PA is the major player in fibrinolysis, whereas u-PA is involved in cell migration and tissue remodeling. These activators break down plasminogen to plasmin, which in turn breaks down fibrin (thrombus) to fibrin degradation products. Fibrinolytics are drugs that mimic biologic activa­ tors and hence increase the level of plasmin, which in turn enhances thrombus breakdown. The goal of this therapy is to establish reperfusion to the myocardium, brain, or lung in acute MI (STEMI) , CVA, or PE, respectively Alteplase/t-PA (Activase) Mecha n i s m of Action

t-PA binds to clot-bound plasminogen to catalyze conver­ sion to plasmin. The specificity for clot-bound plasminogen decreases systemic fibrinolysis.

901

I n d icatio n s a n d Preca utions

The labeled use is for AMI. Combination of half-dose reteplase and full-dose abciximab was evaluated in the GUSTO-V trial and found not to be inferior to full-dose reteplase, but there was no mortality benefit, and risk of bleeding was increased. Streptokinase (Streptase) Mec h a n i s m of Action\

Streptokinase binds to clots and circulating plasminogen; this complex then catalyzes the conversion of plasminogen to plasmin. It is not specific for clot-bound plasminogen and therefore produces a systemic fibrinolytic state. Side Effects

Common side effects of streptokinase include bleeding, bronchospasm, periorbital swelling, angioedema, anaphy­ laxis, hypotension, rash, intracranial hemorrhage (0.2%), fever, and urticaria. I n d icatio n s and Preca utions

Labeled uses of streptokinase are AMI, PE, DVT, arte­ rial thrombosis or embolism, and occlusion of AV cannu­ lae. Patients should not receive streptokinase if they have received anisoylated plasminogen streptokinase activa­ tor complex (APSAC) or streptokinase within the last 12 months. Heparin is not given with streptokinase; if it is needed, the heparin is initiated 4 hours after streptokinase infusion.

Side Effects

Common side effects of t-PA include bleeding, intracranial hemorrhage (0.7%), hypotension, nausea/vomiting, and epistaxis. I n d ications a n d Preca utions

The labeled use of t-PA is for AMI, PE, and acute ischemic stroke. Acute MI patients should receive aspirin and heparin during t-PA infusion. In PE, heparin should be started at the end of the alteplase infusion. It is considered superior to streptokinase, but the risk of intracranial hemorrhage is greater than with streptokinase. Age >65 years and weight 75 years old. The advan­ tage is that it can be given as a single weight-adjusted bolus injection over 5 to 10 seconds. Major Clinic Trials GUSTO-I.73 This trial found that the mortality in patients with

acute STEMI was lower in patients who received t-PA and IV heparin than in those who received streptokinase with either IV heparin or subcutaneous heparin. Furthermore, early PCI when appropriate led to improved survival of patients with MI with cardiogenic shock at 30 days.

902

SECTION X • PHARMACOLOGY

GUSTO-IIl.74 Reteplase did not show superiority over alteplase in

terms of mortality benefit. The two therapies had comparable rates of hemorrhagic strokes. GUSTO-V.75 In this trial, half-dose reteplase was given with a

1 2 -hour infusion of abciximab. This combination was not supe­ rior to the standard dose of reteplase in terms of mortality The combination therapy was associated with an increased risk of bleeding complications.

VA S O D I LATO R S , M I S C E L LA N E O U S In this section, we discuss a more diverse group that does not fall into any specific pharmacologic class.

Side Effects

Common side effects of minoxidil include significant reflex tachycardia, sodium and water retention, weight gain, hir­ sutism, breast tenderness, gynecomastia, and headache. I n d ications a n d Preca utions

The labeled use for minoxidil is severe hypertension, and usually it is the last resort in treating hypertension that is not responsive to other medications. It should be used with beta-blockers to reduce the reflex tachycardia and diuretics to reduce sodium and water retention. N itroprusside ( N ipride)

Hyd ra lazine (Apresoline)

Mech a n i s m of Action

Mec h a n i s m of Action

Direct vasodilatation occurs secondary to the liberation of the nitroso group from the nitrosocyanide structure. Nitroprusside has a balanced effect on both veins and arteries.

Hydralazine works by direct relaxation of the arteriolar smooth muscle, causing a fall in blood pressure, reflex tachycardia, and an increase in cardiac output. The exact mechanism of action at a cellular level has not been determined completely: Side Effects

Common side effects of hydralazine include palpitation, tachycardia, flushing, myocardial ischemia, nausea, vomiting, anorexia, hypotension, and drug-induced lupus-like syn­ drome with prolonged use. I n d ications a n d Preca utions

Hydralazine has a labeled indication for moderate to severe hypertension and unlabeled use in heart failure. Fenoldopa m (Corlopa m) Mec h a n i s m of Action

Fenoldopam is a dopamine (DA 1 ) receptor agonist that causes smooth muscle relaxation, leading to vasodilatation and increased renal blood flow. Side Effects

Common side effects of fenoldopam include hypoten­ sion, headache, flushing, nausea, tachycardia, and, rarely; hypokalemia.

Side Effects

Common side effects of nitroprusside include hypotension, headache, nausea, confusion, and metabolic acidosis. Less common side effects are thiocyanate and cyanide toxicity: Prepa ration, Dosing, I n d ications, a n d Preca utions

The labeled use for nitroprusside is hypertensive urgencies and the management of acute CHF Patients with hepatic fail­ ure are at increased risk for developing cyanide toxicity, and this should be suspected in patients with metabolic acido­ sis, venous hyperoxemia, increased serum lactate levels, air hunger, confusion, seizures, and ataxia. Patients with sus­ pected cyanide toxicity should receive inhaled amyl nitrite while being given 300 mg of sodium nitrite IV, followed by 1 2 . 5 mg of sodium thiosulfate IV If symptoms reappear, then administer half the amounts of sodium nitrite and sodium thiosulfate again. These modalities shift cyanide conver­ sion to thiocyanate. Cyanide levels are not helpful acutely; because it may take up to 5 days to achieve results. Thiocy­ anate is a neurotoxin that causes confusion, psychosis, leth­ argy, tinnitus, convulsions, and hyperreflexia. Hemodialysis removes thiocyanate from the blood. Levels are not typically monitored unless infusion of >3 days or when high doses are used in patients with renal failure.

I n d ications and Preca utions

Fenoldopam is labeled for use as short-term therapy for severe hypertension. It is very expensive while having simi­ lar efficacy to nitroprusside in terms of hypotensive effect. Minoxid i l (Loniten)

Bosenta n (Tracleer) Mech a n i s m of Action

Bosentan is an endothelin-receptor antagonist (ET-A and ET-B) , which leads to vasodilatation (endothelin is a potent vasoconstrictor) .

Mec h a n i s m of Action

Minoxidil acts as a vasodilator, primarily affecting arterial smooth muscle. It antagonizes the effect of ATP on the ATP­ sensitive channel, which leads to hyperpolarization and muscle relaxation.

Side Effects

Common side effects of bosentan include severe hepatotox­ icity ( 1 1 % of patients) , teratogenicity, headache, flushing, hypotension, fatigue, pruritus, edema, and anemia.

CHAPTER 6 1 • CARD IOVASCULAR MEDICINE-ESSENTIAL PHARMACEUTI CALS I n d ications a n d Preca utions

Bosentan is labeled for management of pulmonary hyper­ tension in patients with Class III or IV symptoms. It is nec­ essary to monitor the liver function tests (LFTs) ; if there is three- to fivefold increase in these, dosage reduction or discontinuation is required. It is available only through the Tracleer Access Program (TAP) , a restricted distribution program. Am brisentan (Letai ris) Mecha n i s m of Action

Ambrisentan is an endothelin-receptor antagonist (ET-A and ET-B) , which leads to vasodilation. Affinity for ET-A is greater than ET-B.

placebo, p < 0 . 00 1 ; vs. nitroglycerin p 0 . 03). At 24 hours, nesiritide reduced PCWP significantly more than nitroglycerin (-8.2 mm Hg vs. -6.3 mm Hg; p 0 . 04) . Dyspnea at 3 hours was improved with nesiritide over placebo (p 0 . 03), but not compared to nitroglycerin (p 0 . 56). =

=

=

=

ASCEND-HF.77 Nesiritide was compared to placebo in addition to

standard care in hospitalized patients with acutely decompen­ sated heart failure. The primary endpoints included change in dyspnea at 6 and 24 hours, and composite endpoint of rehos­ pitalization for heart failure or death within 30 days. Nesiritide was not found to have a significant effect on dyspnea. Mortality and rehospitalization were not significantly different between the two groups. In regard to safety, hypotension occurred more frequently in the nesiritide group versus placebo (26.6% vs. 1 5 . 3 % ; p < 0.001) as did symptomatic hypotension (7.3% vs. 4.0%; p < 0 001).

Side Effects

Epoprostenol (Flolan)

Common side effects include peripheral edema, headache, decrease in hemoglobin/hematocrit, and palpitations.

Mech a n i s m of Action

I n d ications a n d Preca utions

Indicated for the treatment of pulmonary artery hyperten­ sion World Health Organization Group I for improving exercise ability and decrease rate of clinical deterioration. Ambrisentan is contraindicated in pregnancy. Nesi ritide (Natrecor)

903

Epoprostenol is prostaglandin I2 . It is a vasodilator of the systemic as well as pulmonary arteries. It also inhibits plate­ let aggregation. Side Effects

Common side effects of epoprostenol include jaw pain, hypotension, headache, rash, diarrhea, joint pain, and non­ cardiogenic pulmonary edema.

Mecha n i s m of Action

I n d icatio n s a n d Preca utions

Nesiritide is a recombinant human b type of BNP or rhBNP It binds to guanylate cyclase receptors in vascular smooth muscle and endothelial cells, increasing the intracellu­ lar level of cGMP and thereby causing venous and arterial vasodilatation, resulting in dose-dependent reduction in pulmonary capillary wedge pressure (PCWP) and systemic blood pressure. It also causes mild natriuresis.

The primary use for epoprostenol is for pulmonary hyper­ tension and pulmonary hypertension secondary to scle­ roderma in NYHA Class III and IV patients who have not responded to conventional therapy The half-life of the drug is very short, 3 to 5 minutes; hence abrupt cessation is not well tolerated by patients.

Side Effects

Common side effects of nesiritide include hypotension, headache, dizziness, and renal dysfunction. I n d ications a n d Preca utions

Nesiritide is labeled for use in acutely decompensated heart failure patients who have dyspnea at rest or with minimal activity. It should be avoided in patients with cardiogenic shock, aortic stenosis, and severe hypotension (systolic blood pressure ­ � CJ ...... 0 •



n c:::

(fl

��

Pharmacodynamic interaction with additive effects on slowing heart rate; also occurs with other rate-controlling medications such as diltiazem and verapamil (Continued)

CJ



(") ......

z

>-I ITI

� >­ n j 0 z (fl

...

\0 w

& ,

Comm o n ly Observed D rug-Drug I nteractions with Cardiovascu lar Medications0 (Continued)

Cardiovascu lar Drug

I nteracting Drug

Mechanism

Consid erations

Dofetilide

Cimetidine

Inhibition of renal tubular secretion and CYP 3A4 metabolism Inhibition of renal tubular secretion QT-prolonging effects possible; inhibition of CYP3A4

Contraindicated to give concurrently

Hydrochlorothiazide Itraconazole Ketoconazole Trimethoprim (alone or in combination with sulfamethoxazole) Triamterene Verapa mil Prochlorperazine QT-interval-prolonging drugs

Statins

Megesterol Amiodarone Bosentan Cyclosporine Amlodipine Diltiazem, verapamil Itraconazole , ketoconazole Gemfibrozil

Inhibition of renal cation transport system; additive effect on QT prolongation Inhibition of renal tubular secretion Inhibition of renal cation transport system Unknown; however, may increase absorption of dofetilide Inhibition of renal tubular secretion Additive effects on QT interval

Inhibition of renal tubular secretion CYP3A4 major metabolic pathway of statins listed on left CYP3A4 enzymes are induced by bosentan Cyclosporine inhibits CYP3A4 Inhibits CYP 3A4 Both verapamil and diltiazem are inhibitors of CYP3A4 Inhibits all CYP enzymes Increased risk of myopathy and rhabdomyolysis

Contraindicated to give concurrently Contraindicated to give concurrently; dofetilide has minimal metabolism through 3A4; itraconazole may prolong QT interval Contraindicated to give concurrently Contraindicated to give concurrently Contraindicated to give concurrently Contraindicated to give concurrently Contraindicated to give concurrently Contraindicated to give concurrently; many drugs, such as Class Ia and III antiarrhythmics, certain antipsychotics, certain quinolones , phenothiazine, many others Contraindicated to give concurrently Do not exceed 10 mg of simvastatin May require higher dose for pharmacologic response Contraindicated with simvastatin Do not exceed 20 mg of simvastatin. Reduce dose of listed statins. Do not exceed 1 0 mg of simvastatin Avoid statins during therapy with these antifungals Contraindicated with simvastatin

'° ...



n j 0 z ><

CJ) [Tl



::c >­ '"O

:::0

� >­ n 0 0 Cl r



Erythromycin, clar­ ithromycin Nefazodone HIV-protease inhibitors

Warfarin

Ranolazine Amiodarone Bile acid binders Antiplatelet medications Metronidazole Gemfibrozil

Certain macrolide antibiotics inhibit the CYP3A4 Nefazodone decreases CYP elimina­ tion of certain statins Certain protease inhibitors decrease metabolism of statins See Amiodarone Concomitant use decreases the absorption of warfarin Increased risk of bleeding Inhibition of warfarin metabolism

Rifampin

Inhibition of warfarin metabolism and protein-binding displace­ ment Inhibition of warfarin metabolism and protein-binding displace­ ment Increased metabolism of warfarin

Phenobarbital

Increased metabolism of warfarin

Phenytoin

Increased metabolism of warfarin

Trimethoprim and sulfamethoxazole

Azole antifungals

Decreased metabolism of warfarin

Macrolide antibiotics

Decreased metabolism of warfarin

Cyclosporine

Unknown mechanism, probably reduced warfarin metabolism

' This table provides a limited list of medications and interactions and is by no means complete . A review of the complete medication list for drug interactions should be performed frequently, especially when adding or discontinuing medications, to avoid unnecessary adverse drug events.

Avoid statin use during therapy with these antibiot­ ics. Contraindicated with simvastatin Avoid statin use. Contraindicated with simvastatin. Avoid statin use. Contraindicated with simvastatin. Do not exceed 20 mg of simvastatin See Amiodarone Take warfarin 2 h before or 2 h after a bile acid binder Monitor for signs and symptoms of bleeding Change antibiotic if possible; if not, consider reducing dose of warfarin and monitor INR more frequently Monitor INR more frequently while on therapy; consider alternative therapy; interaction may be seen with fenofibrate as well Avoid use if possible; if not, consider reducing warfarin dose and monitor INR more frequently while on therapy Monitor INR more frequently, as patients will need higher doses. Monitor INR more frequently, as patients will need higher doses. Monitor INR more frequently, as patients will need higher doses. Monitor INR more frequently, as patients will need lower doses. Monitor INR more frequently, as patients will need lower doses. Monitor INR more frequently, as patients will need lower doses.

n ::r:: >­ "rj >-I ITI



°' N

n >­ � tJ ...... 0 •



n c:::

(fl

�� tJ



CJ

z ......

>-I ITI

� >­ n j 0 z (fl \0 ...

U1

91 6

SECTION X • PHARMACOLOGY

Truncated List of Medications that l n d uce CYP Drug

CYP Enzyme(s) I n d uced

Phenytoin Phenobarbital Carbamazepine Rifampin Ritonavir Smoking

1A2 , 3A4, 2D6, 2C9 1A2 , 3A4, 2D6, 2C9 3A4, 2D6, 2C 1A2 , 3A4, 2D6, 2C9 1A2 , 2D6 1A2

possibly lasting hours after ingestion. Medications that interact with grapefruit juice have increased toxicities, as in the case of felodipine, for which there can be two times the amount of drug absorbed, increasing the hypotensive risk. Other calcium channel blockers, such as amlodipine, are less affected by grapefruit juice. A similar case can be made for coadministration of certain statins with grapefruit juice. Simvastatin, lovastatin, and to a lesser extent atorv­ astatin will have increased levels with the coadministration of grapefruit juice and have an increased potential for the development of adverse events. S U G G ESTE D READ I NGS

lipophilic compounds from the enterocyte back to the intes­ tinal lumen (reverse transport) . Hence these two enzymes work together to change the amount of medication that reaches the systemic circulation. Grapefruit juice is the classic example of a drug-food interaction, with inhibition of gastrointestinal CYP3A4 and P-glycoprotein leading to an increase in medication bio­ availability. Grapefruit juice may increase the levels of cer­ tain dihydropyridine calcium channel blockers, statins, cyclosporine, and other medications. This interaction may be observed with as little as 200 ml daily, with an effect

j TA B L E li7...C I Truncated List of Medications that Prolong the QT I nterva l

Amiodarone Disopyramide Dofetilide Droperidol Erythromycin Haloperidol Ibutilide Itraconazole

Ketoconazole Procainamide Phenothiazine antipsychotics Quinidine Quinolone antibiotics Sotalol Tricyclic antidepressants Ziprasidone

Asberg A. Interactions between cyclosporin and lipid-low­ ering drugs: implications for organ transplant recipients. Drugs. 2003;63(4) :367-378. Ferrari P, Bianchi G . The genomics of cardiovascular disor­ ders: therapeutic implications. Drugs. 2000;59(5) : 1 025- 1 042 . Haddad PM, Anderson IM. Antipsychotic-related QTC prolongation, torsade de pointes and sudden death. Drugs. 2002;62( 1 l) : 1 649- 1 67 1 . Hunter AL, Cruz RP, Cheyne BM, e t al. Cytochrome P450 enzymes and cardiovascular disease. Can ] Physiol Pharmacol. 2004;82 : 1 053- 1 060. Kane G C , Lipsky JJ . Drug-grapefruit juice interactions. Mayo Clin Proc. 2000;75:933-942 . Kuhlmann ] , Muck W Clinical-pharmacological strategies to assess drug interaction potential during drug development. Drug Safety . 200 1 ;24(1 0) : 7 1 5-72 5 . Michalets E L Update: clinically significant cytochrome P-450 drug interactions. Pharmacotherapy. 1998 ; 1 8( 1 ) : 84-1 1 2 . Ozdemir V, Shear NH, Kalow W What will b e the role of pharmacogenetics in evaluating drug safety and minimising adverse effects? Drug Safety. 200 1 ;24(2) : 75-85. Schelleman H, Stricker B H , d e Boer A, e t al. Drug-gene interactions between genetic polymorphisms and antihypertensive therapy Drugs. 2004;64( 1 6) : 1801-1 8 1 6 . Schmidt LE, Dalhoff K . Food-drug interactions. Drugs. 2002 ; 6 1 ( 1 0) : 148 1-1502. Stone SM, Rai N , Nei J . Problems and pitfalls in cardiac drug therapy Rev Cardiovasc Med. 200 1 ;2(3) : 1 26-142. Trujillo TC, Nolan P E . Antiarrhythmic agents: durg interac­ tions of clinical significance. Drug Safety. 2000;23(6) :509-532. Zhang Y, Benet L Z . The gut as a barrier to drug absorption: combined role of cytochrome P450 3A and P-glycoprotein. Clin Pharmacokinet. 200 1 ;40(3) : 1 59-1 68 .

CHAPTER 62 • CARD IOVASCULAR DRU G INTERACTI ONS

91 7

QU ESTIONS AND ANSWERS Questions

1 . Simvastatin levels ca n be increased by which of the fol lowi ng? a. Water b. G rape j uice c. G rapefruit ju ice d. Orange j uice

5 . T h e i nteraction between Sotalol and Di ltiazem would be considered a: a. Pha rmacogenomic interaction b. Pharmacodynamic i nteraction c. Pharmacoki netic i nteraction d. None of the choices Answers

2. A 50 yea r old man was i nitiated on Amiodarone for pa roxysmal atria l fibrillation. He com plains of severe nausea after each dose and would l i ke to try a d ifferent antiarrhythmic. You decide that sotalol wou l d be a good alternative agent. He is cu rrently ta king wa rfa rin, aspirin, hyd rochlorothiazide, and lisinopril. Which of his cu rrent therapies will need to be adj usted after Amiodarone is discontinued? a. Aspirin b. Hyd rochlorothiazide c. Lisinopril d. Warfarin

1 . Answer C: Gra pefruit juice wi l l i n hibit p-g lycoprotein and the CYP 3A4 enzyme system. I n June of 201 1 the FDA safety alert reiterated the product label for simvastati n stating that patients should d r i n k < 1 quart o f g rapefruit juice daily to prevent adverse effects of simvastatin.

3. A 65 yea r old man has a history of hypercholester­ olemia and is cu rrently on si mvastatin 40 mg dai ly. At his most recent appoi ntment, he is now noted to have hypertension and you wa nt to i nitiate a m lo­ dipine 5 mg dai ly. How should you a lter the dose of Simvastatin? a. No change needs to be done. b. I ncrease the dose to 80 mg dai ly. c. Decrease the dose to 20 mg daily. d. Decrease the dose to 1 0 mg daily.

3. Answer C: Based on the new dosing recom menda­ tions the maxi m u m dose of si mvastati n when taken with a m lodipine is 20 mg dai ly. The exact mecha nism of this interaction is u nclear, however, most l i kely is related to i nteractions with the CYP 450 enzyme system . The dose of 80 mg should not be i n itiated on a ny patients at this time as there is a n increased risk of rhabdomyolysis.

4. A 42 yea r old woman has a mechanical m itra l va lve and is on wa rfarin to prevent va lve throm bosis. She is now diagnosed with tuberculosis and wi l l need to receive rifampin as part of her thera py. Which of the fol lowi ng statements is correct? a. Rifa m p i n wi l l i ncrease the e l i m i nation of wa rfarin. b. Rifa m pi n wi l l decrease the elimination of warfarin. c. There is no concern regarding rifampin and warfarin. d. Change wa rfarin to Dabigatra n .

2. Answer D: Amiodarone i n h i bits the meta bolism of wa rfarin and u pon d iscontin u i ng thera py you will need to mon itor his i nternational norma l izing ratio (I N R) more frequently and be ready to increase the dose of wa rfarin as the I N R starts to decrease. This may not be i mmed iately recognized as the half-life of Amiodarone is 25 to 1 00 days.

4. Answer A: Rifampin is a powerfu l inducer of the CYP 450 enzyme system. The addition of rifampin wil l in­ crease the elimination of wa rfarin and occu rs rapid ly.This combination wou ld require increased monitoring of the I N R. Doses of warfarin wi l l need to be g reatly i ncreased while the combination is continued. Dabigatran does not have an ind ication for the prevention of valve throm bo­ sis and is not an appropriate therapy at this time. S. Answer B: The addition of diltiazem to sotalol would cause a n additive effect on slowing of the atrioventricu­ lar (AV) node.This i nteraction is a synergistic effect on prolongation of the AV node refractory period. This is not a pharmacokinetic effect as there is no alteration i n the levels of either drug.

Page numbers followed by t indicate table; those followed by f indicate figure. A AAA . See Abdominal aortic aneurysm

Abciximab (Reopro), 583, 694, 876, 900t Abdominal aortic aneurysm (AAA) , 7 1 6-7 1 7 Ablation, 370 atrial flutter, 393 atrioventricular junction, 386 catheter, 394 Absolute risk reduction (ARR) , 57 ACOAHA. See American College of Cardiology/ American Heart Association Accelerated AV junctional rhythm, 296 Accelerated idioventricular rhythm (AIVR), 297 ACCP See American College of Chest Physicians ACE. See Angiotensin-converting enzyme ACIP trial, 690 ACP Journal Club Criteria, 6 1 Acromegaly, 858 ACS. See Acute coronary syndrome Action potential (AP), membrane calcium-dependent AP myocardial tissues vs. sodium channel tissues, 44, 44f conduction system anatomy and physiology, 45-46 electrical coupling cells, 45 Na• channel activity regulation, 43-44, 44f sodium-dependent cells, 4 3, 44f ACTION trial, 560 Activase. See Alteplase Acute chest pain syndromes, 1 2 1-124, 122t-123t Acute cor pulmonale, 30 1-302 Acute coronary syndrome (ACS), 532. See also Non-ST-elevation acute coronary syndrome ACOAHA classification for, 575, 576t angiotensin-converting enzyme inhibitors (ACE-I) , 585 clinical presentation for, 576 discharge planning and noninvasive stress testing, 585-586, 585t epidemiology and prognosis, 575-576 management of anti-ischemic agents, 584-585 antiplatelet agents, 580-583, 5 8 lf antithrombin therapy, 583-584 aspirin for, 580-583, 582f DT!s, 584 glycoprotein !lb/Ila inhibitors for, 583 heparins low molecular weight, 583 unfractionated, 583-584 initial approach, 578 invasive therapy, 578-5 8 1 , 579t miscellaneous agents, 585 thienopyridines, 58 1-583 pathophysiology, 576-577 risk stratification biomarkers as, 577 ECG and, 577 TIMI risk score, 577-578, 578f summary on, 586 terminology and overview, 575 Acute mitral regurgitation, 479-480. See also Mitra! regurgitation diagnosis for, 6 1 3 , 6 1 3f symptoms and signs of, 6 1 3 treatment for, 6 1 3-6 1 4 Acute myocardial infarction (AMI) , 839 dobutamine for, 608 dopamine for, 608

epinephrine for, 608 management of, 608 norepinephrine for, 608 phenylephrine for, 608 Acute pericarditis, 30 1 , 844-846, 845f, 845t, 846f. See also Pericarditis Adalat. See Nifedipine Adenosine, 1 19-1 2 1 , 120t Adenosine diphosphate (ADP), 1 1 9 , 120t Adenosine triphosphate (ATP) for cardiac pacing, 4 3 7 hydrolysis of, 48 myocardial perfusion imaging, 1 19 , 120t ADME (drug absorption, distribution, metabolism, and elimination), 876 ADMIRAL trial, 599 ADP See Adenosine diphosphate Adrenal insufficiency, 858 Adrenaline. See Epinephrine Adrenergic agonists, 88 1-882, 882t Adrenergic antagonists, 882-884, 883t a-Adrenergics, 4 3 ,8-Adrenoreceptor kinase (,BARK!) , 48-49, 49f AF See Atrial fibrillation AFCAPS!TexCAPS. See Air Force!Texas Coronary Atherosclerosis Prevention Study AFFIRM (Atrial Fibrillation Follow-Up Investigation of Rhythm Management), 384-385, 385t AFRP See Atrial Functional Refractory Period Aggrastat. See Tirifiban a-agonists, 45 1 , 45 l t AHA. See American Heart Association Air Force!Texas Coronary Atherosclerosis Prevention Study, 79 1 , 793t AIREX, 885 AIVR. See Accelerated idioventricular rhythm Aldomet. See Methyldopa Aldosterone, 4 3 antagonists, 168 receptor, 886-887 ALLHAT (Antihypertensive and Lipid-Lowering Treatment to Prevent Heart Attack Trial) , 637 Alteplase/t-PA (Activase) , 901 Ambrisentan (Letairis), 903 American College of Cardiology/American Heart Association (ACOAHA) , 608 American College of Chest Physicians (ACCP), 727, 827 American Heart Association (AHA) , 5 1 4 American Society o f Nuclear Cardiology, 1 2 1 AMI . See Acute myocardial infarction Aminoglycosides, 5 14 Amiodarone, 389 Amlodipine, 169, 560t Ampicillin, 5 14 Amyl nitrite, 14 Amyloidosis, 141 Anemia, 861 Angina classifications, 575 refractory enhanced external counterpulsation, 56 7-568 miscellaneous proposed therapies, 568 spinal cord stimulation, 568 stable definition of, 550-5 5 1 diagnostic testing/risk stratification conclusion on, 554-555

coronary angiography, 553-554 echocardiography, 553 electrocardiography, 5 5 1 exercise ECG testing, 55 1-552 , 553t stress testing, 553, 554t medical treatment for antianginal therapies, 559-562 conclusion on, 562 medical therapies, 556-559 pathophysiology of, 5 5 1 revascularization conclusion on, 566 coronary artery bypass grafting vs. medical therapy, 562-564 PC! vs. CABG, 565-566 PC! vs. medical therapy, 564-565 unstable, 575 variant/prinzmetal, 535 Angiography indications for, 809 pulmonary, 725 Angiomax. See Bivalirudin Angiotensin 1-9, 50 I, 50 11, 43, 50, 158 Angiotensin-converting enzyme (ACE), 610, 884-886. 884f, 885t for stable angina, 558-559 for STEM!, 60 1 Angiotensin-receptor blockers (ARBs) , 6 1 0 for HTN, 764 II, 886, 886t Anglo-Scandinavian Cardiac Outcomes Trial-Lipid Lowering Arm (ASCOT-LLA) , 793t, 794, 798, 803 Annuloplasty, 497 Antianginals, stable angina beta-blockers, 559-560, 559t calcium channel blockers, 560, 560t combination therapy, 561 nitroglycerin and nitrates, 560-5 6 1 , 5 6 l t ranolazine, 56 1-562 Antiarrhythmic drugs, AF, 384 CHAOS 11 score, 39 1-393, 392t class IA, 387-388, 388t class IC, 389 class III, 389-390 inpatient vs. outpatient initiation, 39 1 proarrhyrhmia risks associated with, 390-3 9 1 selection, 390-3 9 1 Antibiotics, 466 Anticoagulants, 635 , 887, 888f during pregnancy, 827-828, 827t for valves, 524-525, 524t Anticoagulation, 526 chronic, 391 recommendations for, 392-393 Antihyperlipidemic therapy, 75 1-752, 752t Anti-ischemic agents, 584-585 for ACS, 584-585 for STEM!, 600 Antiplatelet agents for ACS aspirin, 580 glycoprotein !lb/Illa inhibitors, 583 thienopyridines and ADP inhibitors, 580-583 for STEM!, 598 Antiplatelet medications, 899

91 9

920

INDEX

Antiplatelet therapy, 750-75 1 Antiproliferative agents, 192-193 Antithrombin therapy, 583-584 Antithrombotic therapies for stable angina, 557 for STEM!, 599 Antithrombotic Trialists Collaboration, 557 Aorta. See also Coarctation of aorta anatomy of, 709 diseases of, 142, 146 pathologic processes for, 709-7 1 0 Aortic aneurysms, 1 4 2 , 7 1 5-7 1 6 , 7 1 8 Aortic dissection, 1 4 2 , 7 1 0 , 7 1 0J, 7 1 8 chronic, 7 1 3 classifications of, 7 1 0 , 7 1 1f, 7 l l t clinical presentations of, 7 1 1 diagnostic testing, 7 1 1-7 1 2 , 7 1 2 t iatrogenic, 7 1 3 , 7 1 3f management of, 7 1 2-7 1 3 i n young, 7 1 3 Aortic insufficiency chest X-ray for, 469 clinical findings on, 469 ECG for, 469 echocardiography for, 469-470, 469f, 470f etiology of, 468 pathophysiology of, 468 physical exam, 469 treatment for, 470-47 1 , 470t, 4 7 l t vasodilators for, 470-47 1 , 470t Aortic regurgitation (AR), 662 pregnancy and, 820 severe, 36 Aortic stenosis (AS), 662-663, 662t, 82 1-822 chest radiography, 70, ?Of classifications of, 465 clinical findings in, 462-464, 464f ECG for, 463 etiologies of, 46 1-462, 462f invasive assessment of, 464-465 , 465f pathophysiology of, 461 pseudostenosis vs . low gradient, 468 subvalvular, 462 supravalvular, 462 treatment of, 465-468, 466t, 467t TIE for, 463 vasodilators for, 465 Aortic valve(s) anatomy, 46 1 , 462f morphology of, 464 resistance, 660 Aortic valve area (AVA) , 464 AP See Action potential Apresoline. See Hydralazine AR See Aortic regurgitation ARBITER-2. See Arterial Biology for the Investigation of the Treatment Effects of Reducing Cholesterol ARBs. See Angiotensin-receptor blockers Arginine, 158 Arixtra. See Fondaparinux ARR. See Absolute risk reduction Arrhythmias, 394, 407-4 1 0 , 454 Arrhythmic complications bradyarrhythmias, 620-62 1 , 622t tachyarrhythmias, 62 1 Arrhythmogenic right ventricular cardiomyopathy (ARVC) , 1 4 1-142, l 42t, 222 Arrhythmogenic right ventricular dysplasia (ARVD) , 378 Arterial Biology for the Investigation of the Treatment Effects of Reducing Cholesterol (ARBITER-2) , 797 Arterial pulse(s) . See also Pulse(s) anacrotic pulse, 7 Corrigan, 7 dicrotic pulse, 7 disease states, 4, 6-7, 6f double-peaked pulse, 6

Duroziez sign, 7 Hill sign, 7 loud systolic sound, 7 Mayen sign, 7 normal, 4, 6f osier sign, 7 pressure/pulse difference in two arms, 7 principles, 4 pulse deficit, 7 pulsus alternans, 4 pulsus paradoxus, 4, 6 pulsus tardus and parvus, 7 radial-to-femoral delay, 7 supravafvular AS and, 7 traube sign and, 7 water-hammer pulse, 7 ARTS (Arterial Revascularization Therapies Study) I, 692 11, 692 ARVC. See Arrhythmogenic right ventricular cardiomyopathy ARVD . See Arrhythmogenic right ventricular dysplasia AS . See Aortic stenosis ASCOT-LLA. See Anglo-Scandinavian Cardiac Outcomes Trial-Lipid Lowering Arm ASD . See Atrial septa! defect Aspirin, 635, 693-694, 898 for ACS, 580 for PC!, 693-694 for stable angina, 562 for STEM!, 598 ASSENT (Assessment of Safety and Efficacy of a New Thrombolytic) 2 , 629 -3 trial, 600 Atheromatous aortic disease, 7 1 7, 71 ?f, 7 1 8 , 7 1 9 Atherosclerosis, 709 Atorvastatin for NSTEMI, 585 for STEM!, 601 Atorvastatin Versus Revascularization Treatments (AVERT), 794 ATP See Adenosine triphosphate Atrial arrhythmias ablation of maze procedure, 394 pacemaker therapy, 394 pulmonary vein isolation, 394 catheter ablation of, 394 Atrial bradycardia, ectopic, 295 Atrial extrastimulus testing, 3 6 1 Atrial fibrillation (AF) , 2 5 8 , 2 9 6 antiarrhythmic drugs for class lA, 387-388 class IC, 389 class III, 389-390 inpatient vs. outpatient initiation of, 391 proarrhythmia risks associated with, 390-391 selection of, 39 1 , 39 l t, 392t AV node ablation, 393-394 catheter ablation, 394 definitions, 383 epidemiology, 382 evaluation, 383-384 factors predisposing, 382 management, 384-385, 385t, 386t morbidity and mortality, 382-383 nonpharmacologic rate control, 386 onset of, 493 pathogenesis, 383 pharmacologic conversion, 387, 387t sinus rhythm maintenance of, 387, 388t restoration of, 386-387, 387t Atrial flutter, 296 ablation of, 396 cavotricuspid isthmus-dependent atrial flutter, 383

clinical presentation and diagnosis, 408, 408f differential diagnosis, 4 1 5 , 4 1 5f mechanism, 408-409 noncavotricuspid isthmus-dependent atrial flutter, 383 nonpharmacologic management, 393 reentrant mechanism, 383 treatment, 409 Atrial Functional Refractory Period (AFRP) , 367 Atrial pacing, 98, 99 EP and, 366, 367f incremental, 358-359 Atrial rhythm, ectopic, 295 Atrial septa! defect (ASD) , 820 coronary sinus ASD, 2 79 primum, 302 primum ASD , 277, 278t, 279f secundum, 302 secundum ASD, 278, 280f sinus venosus ASD , 2 77, 280f types of, 277, 278f Atrial tachycardia, 296 clinical presentation and diagnosis, 40 l t, 407 mechanisms, 407 treatment, 408 Atrial-dependent arrhythmias, 407-409, 408f atrial flutter, 408-409 atrial tachycardia, 407-408 Atrioventricular block (AVB) first degree, 298 second degree Mobitz Type I (Wenckebach), 298 Mobitz Type II, 298 third degree (complete heart block), 298 Atrioventricular junction ablation, 386 Atrioventricular junctional bradycardia, 296 Atrioventricular junctional rhythm, accelerated, 296 Atrioventricular junctional tachycardia, 296 Atrioventricular nodal reentrant tachycardia (AVNRT), 296 Atrioventricular reentrant tachycardia (AVRT) , 296 AV junctional bradycardia, 296 AV junctional tachycardia, 296 AV nodal refractory periods, 362, 363f AV node ablation, 393-394 AV node-dependent tachycardias (AVNRT) clinical presentation and diagnosis, 40 1 , 40 l t mechanism, 401-402 , 402f treatment, 402 AV reciprocating tachycardia (AVRT) . See also Wolff-Parkinson-White syndrome accessory pathways, 403, 403f, 403t antidromic tachycardia, 404 clinical presentation and diagnosis, 402-403 electrophysiologic characteristics and diagnostic maneuvers, 403-404 mechanism, 403, 404f treatment, 405-406 Wolff-Parkinson-White syndrome and sudden cardiac death, 406-407, 407t AV reentrant tachycardia differential diagnosis, 4 1 4 , 4 l 4f AVERT. See Atorvastatin Versus Revascularization Treatments AVNRT. See Atrioventricular nodal reentrant tachycardia AVRT. See Atrioventricular reentrant tachycardia Azathioprine (Imuran), 1 92-193 Azole antifungals, 9 1 5 t Aztreonam, 5 1 4 B Ball-in-cage valve, 523f Balloon valvotomy, 494 Balloon valvuloplasty, 46 7, 46 7 t

INDEX BARI (Bypass Angioplasty Revascularization Investigation) , 565 Baseline intervals, 353-354, 354f Basiliximab (Simulect) , 190 Bayes theorem, 58 BBB. See Bundle branch block Beh�et disease, 861 Benzathine penicillin, 466, 466t Bernoulli principle, 85 Beta-adrenergic blockers, 385 Beta-blockers, 559-560, 559t, 584, 586, 600-601 for MI, 636, 636f for syncope, 45 1-452 Bezafibrate Infarction Prevention (BIP) , 796 BHAT (Beta-Blocker Heat Attack Trial), 636 Bias in biostatistics, 60 observation, 60 recall, 60 selection, 60 verification, 60 Bicuspid aortic valve, 2 8 1 , 282f, 283, 283f Bileaflet mechanical valve, 523f Biomarkers, 577, 629, 63 1 , 63 lf Biostatistics bias in, 60 causality and validity in, 60-6 1 confounding and interaction in, 59-60, 59f current controversies associated with, 6 1 data presentation/outcomes reporting in, 5 7-59 , 59f exposures and outcomes, 5 5 multivariable regression in, 60 specific types issues in, 6 1 statistical tests for, 56-5 7 types of clinical studies in case reports as , 55 case series as, 55 case-control study as, 56, 56f cohort study as, 55-56 cost-effectiveness studies on, 56 GWAS, 56 meta-analysis, 56 prospective vs. retrospective, 56 randomized controlled trials as, 56, 56f studies of diagnostic tests, 56 BIP See Bezafibrate Infarction Prevention Bivalirudin, 599, 697 Bjork-Shiley valves, 522 Blood flow definition of, 85-87 pulmonary, 65f microbiology, 507-509, 508t BMD. See Bone mineral density BML See Body mass index Body mass index (BM!) , 200 Bone mineral density (BMD) , 196 Bosentan (Tracleer) , 902-903 Bradbury- Eggleston syndrome. See Primary autonomic failure Bradyarrhythmias, 356, 620-62 1 , 622t Bradykinins, 50 Brain natriuretic peptide (BNP) , 577, 627, 63 1 , 725 Braunwald classification of unstable angina, 575 BRAVE trial, 597 Brockenbrough response, 253, 253f Brucella, 508, 508t Brugada syndrome, 3 77-378, 377f B-type natriuretic peptide (BNP) , 158 Bundle branch block (BBB) , 62 1 c

CABG. See Coronary artery bypass graft CAD. See Coronary artery disease CADILLAC trial, 597 Calan. See Verapamil

Calcification coronary artery, 75, 75f left atrial wall, 73, 73f mitral annular, 74, 74f patterns, 73, 73f Calcineurin inhibitors, 1 9 1 , 192t Calcium blockers, dihydropyridine, 763 dependent ap myocardial tissues vs. sodium channels tissues, 44, 44f elemental, 196 homeostasis, in cardiac biochemistry, 48 Calcium channel blockers (CCBs), 385, 560, 560t, 892-894, 893t, 894t class of, 892 , 893t, 894t for HF, 1 69 for stable angina, 560, 560t for syncope, 45 l t, 453 CAMP See Cyclic adenosine 3'5'-monophosphate Canadian Cardiovascular Society ( CCS) , 550, 5 5 l t Candida, 1 94 CAPITAL-AMI (Combined Angioplasty and Pharmacological Interventions vs. Thrombolytics Alone in Acute MI) , 686 CAPRICORN (Carvedilol Post-Infarct Survival Control in Left Ventricular Dysfunction), 636 CAPRIE trial, 580 Captopril, 774t, 9 1 2t CAPTURE trial, 900 CAR See Coxsackie adenovirus receptor Carbamazepine, 9 l 6t Carbomedic valves, 522 Carcinoid heart disease, 870-872 Carcinomas, 768, 868 Cardene. See Nicardipine Cardiac allograft vasculopathy (CAV), 187-189 Cardiac amyloidosis, 1 4 1 Cardiac anatomy See also Heart cardiac chambers left atrium, 26, 26f left ventricle, 26-2 7 right atrium, 25, 25f right ventricle, 25-26, 26f cardiac valves aortic valve, 27f, 28, 28f mitral valve, 27-28, 27f pulmonic valve, 27, 27f tricuspid valve, 2 7 conducting system, 30-3 1 , 3 lf great vessels aorta, 28-29 cardiac lymphatic system, 30 cardiac veins, 30, 30f coronary arteries, 29-30, 29f-30f pulmonary arteries, 28 pulmonary veins, 28 vena cavae, 28 heart and great vessels, 24, 25f innervation, 3 1 pericardium, 24, 25f Cardiac biochemistry ,B-adrenergic signaling (/3-AR) , 48-49, 49f cardiac contractility and calcium homeostasis, 48 digitalis AND Na'-K' ATPase, 49 nitric oxide, 49f, 50 phosphodiesterase inhibitors (PD!s) , 49 renin-angiotensin system (RAS), 49-50, 50f Cardiac catheterization, 253, 253f, 64 1-65 1 , 642f, 644t, 645f, 646f, 647t, 648f, 649t, 650t Cardiac chamber enlargement, 299-300 Cardiac CT aortic disease, 146 coronary artery disease, 146-14 7, 14 7f coronary calcium scoring, 146 indications, 144 limitations, 146-148, 147t

921

pulmonary vein assessment, 146 technical considerations, 144-145 Cardiac diseases, 376. See also specific types Cardiac free wall rupture diagnosis of, 6 1 4-6 1 5 , 6 1 5f signs and symptoms of, 6 1 4 treatment for, 6 1 5 Cardiac hemangiomas, 868 Cardiac intervals, measurement of, 297-299 Cardiac lipomas, 867-868 Cardiac masses, 148 Cardiac MRI aortic disease, 142 constrictive pericarditis, 14 1-14 2 coronary artery disease, 140, l 40f heart failure, 140 limitations, 143, l 43t myocardial disease arrhythmogenic right ventricular cardiomyopathy (ARVC), 1 41-142, 142t dilated cardiomyopathy, 140 hypertrophic cardiomyopathy, 140-141 restrictive cardiomyopathy, 1 4 1 technical considerations MRI physics, 1 3 7 pulse sequences, 1 3 7-140, 138f, 139f viability assessment, 140 Cardiac output (CO), 656 low, 7 measurement, 656 Cardiac pacing. See also Implantable cardioverter defibrillators basic concepts of impulses and timing, 429-430, 429f device features, 427-429 , 428f indications, 4 25 permanent pacing, 426, 426t-427t programming, 430-43 1 , 430t, 43 lf summary on, 4 38 temporary pacing, 4 25 troubleshooting and complications, 43 1-432 Cardiac papillary fibroelastoma, 867, 868f Cardiac physical examination arterial pulse anacrotic pulse, 7 Corrigan, 7 dicrotic pulse, 7 disease states, 4, 6-7, 6f double-peaked pulse, 6 Duroziez sign, 7 Hill sign, 7 loud systolic sound, 7 Mayen sign, 7 normal, 4, 6f osier sign, 7 pressure/pulse difference in two arms, 7 principles, 4 pulse deficit, 7 pulsus alternans, 4 pulsus paradoxus, 4, 6 pulsus tardus and parvus, 7 radial-to-femoral delay, 7 supravafvular AS and, 7 traube sign and, 7 water-hammer pulse, 7 atrial septa\ defect, 1 9-20 continuous heart sounds, 1 7 , l 7f diastole, 1 1 , l lf diastolic murmurs, 1 6- 1 7 , 1 6f dynamic auscultation, 1 7-18, 1 8f extra heart sounds diastole, 1 1 , l lf ES as, 1 1 nonejection clicks, 1 1-12, 1 2f OS as, 1 1 pacemaker sounds as, 1 2 pericardia! friction rubs, 1 2 prosthetic heart sounds, 1 2 systole, 1 1 , l lf

922

INDEX

Cardiac physical examination (Continued) first heart sound intensity, 9 principles, 9 splitting, 9-1 0 fourth heart sound, 1 0-1 1 heart murmurs, 12-13 inspection, 4, 5t jugular venous pulse disease states, 8-9 , 9f jugular venous pressure, 7 jugular venous waveforms, Sf Kussmaul sign, 9 principles, 7 physical examination, 4 precordial motion dilation, 9 disease states, 9 hypertrophy, 9 principles, 9 pregnancy, 20 second heart sound intensity, 1 0 principles, 1 0 single S, , 1 0 splitting, 1 0 , 1 Of specific diseases and acute myocardial infarction, 1 8-19 atrial septa] defect, 1 9-20 cardiac tamponade, 1 9 constrictive pericarditis, 1 9 , 1 4 1-14 2 dilated cardiomyopathy, 1 9 pulmonary hypertension, 1 9 restrictive CM, 1 9 RV infarction, 1 9 type A aortic dissection, 1 9 ventricular septa! defect, 2 0 systole, 1 1- 1 2 , 1 2f systolic murmurs, 13-16, 1 3f-1 5f third heart sound, 1 0 Cardiac rhabdomyomas, 868 Cardiac sarcoidosis, 1 4 1 Cardiac tamponade, 1 9 Cardiac tumors benign, 86(Hl68, 867f-869f malignant, 868-870, 8 7 lf secondary, 870, 872f Cardiac valves aortic valve, 27f, 28, 28f mitral valve, 27-28, 27f pulmonic valve, 27, 27f tricuspid valve, 27 Cardiac veins, 30, 30f Cardiomediastinal silhouette patterns, 67-72 , 68f-72f Cardiomyopathy (CM) , 823. See also Dilated cardiomyopathy; Hypertrophic cardiomyopathy ischemic vs. nonischemic, 1 29-130 myocarditis and forms of, 220 restrictive, 141 Cardionet system, 445 , 446f Cardiovascular diseases (CVDs) , 88 1 , 882t after pregnancy, 823-827, 825t, 826t board exam first-time taker pass rates, 1 , 2t format, 1 , 2t general information, 1 pitfalls, 3 tips, 2-3 risk factors novel, 547 traditional modifiable diabetes mellitus, 546 hypertension, 546 lipids, 546 obesity and metabolic syndrome, 546-547 tobacco, 546

in United States age and gender variation of, 539f, 54 1 , 54 lf cost and health care resources for, 544-545, 545f prognosis and risks associated with, 54 3-544, 544f-545f racial and socioeconomic disparities of, 540f, 54 1-542 , 542f silent myocardial ischemia/infarction, 539-54 1 , 540f, 54lf temporal trends of, 542-543, 543f young people and, 542 Cardiovascular drug interactions food-drug interactions, 9 1 2 , 9 1 6 pharmacodynamic drug interactions, 9 1 2 , 9 1 6t pharmacokinetic interactions of, 9 1 1-9 1 2 , 9 12t-9 1 6t Cardiovascular Medicine Board Examination, 4 3 Cardiovascular physiology, myocardial performance autonomic nervous system, 38 cardiomyopathy, 38, 38f, 38t determinants of afterload, 34-35 contractility, 35, 35t heart rate, 35-36 myocardial performance, 36 preload, 34 graphic illustration Force-Tension curves, 36, 36f pressure-volume loops, 36-37, 3 7f starling law and Frank-Starling curves, 36, 36f pressure-volume loop compliance, 37 elastance and contractility, 38 intracardiac pressures, 3 7, 3 7f stroke work, 38 ventricular volumes, 3 7 valvular heart disease, 37t, 38, 38f Cardizem. See Diltiazem CARDS. See Collaborative Atorvastatin Diabetes Study Cardura. See Doxazosin CARE. See Cholesterol and Recurrent Events Camey syndrome, 866 Carotid aortic arch anatomy, 7 48-7 49, 7 49f artery stenting, 753-755 clinical presentation, 749, 750t diagnostic testing, 749-750, 750t endarterectomy, 752-753, 753f, 754t epidemiology, 7 48 medical treatment anticoagulant therapy, 75 1 antihyperlipidemic therapy, 75 1-752, 752t antihypertensive therapy, 75 1 antiplatelet therapy, 750-75 1 pathophysiology of, 748 recommendation, 752 Carotid artery stenting, 753-755 Carotid endarterectomy, 752-753, 753f, 754t Carotid intima-media thickness (CIMT) , 797 Carvedilol, 636 Carvedilol Prospective Randomized Cumulative Survival (COPERNICUS) , 884 CASS (Coronary Artery Surgery Study), 562 Catapres. See Clonidine Catecholamines, 48 Catheter(s), 700 ablation of AF, 394 maze , 394 Causality, 60-6 1 CAV See Cardiac allograft vasculopathy CBC. See Complete blood count CCBs. See Calcium channel blockers CDC. See Centers for Disease Control CDP See Coronary Drug Project Cellcept. See Mycophenolate mofetil Centers for Disease Control (CDC) , 541

Certican. See Everolimus CETP See Cholesteryl ester transfer protein CGMP See Cyclic guanosine 3'5'-monophosphate Chagas heart disease, 220 CHARM trials (Candesartan in Heart Failure Assessment in Reduction of Mortality), 635 , 886 CHO. See Congenital heart disease; Coronary heart disease Chemotherapy, 862 Chest discomfort. See also Acute chest pain syndromes associated symptoms, 534 atypical, 535 causes of, 533t cocaine-associated, 535 context of, 534 diagnostic workup, 535 differential diagnosis, 532, 533t family/social/medication history and, 534 history of, 532 location of, 532-533 medical history, 534 physical examination, 534-535 plaintive and palliative factors, 534 presyncopal/syncopal symptoms, 534 quality of, 533 radiation of, 533 severity of, 533 special circumstances, 535 timing of, 534 variant/prinzmetal angina, 535 in women, 535 work-up for, 535 Chest radiography (CR) approach and projections, 64 atrial fibrillation, 75, 75f calcification patterns calcific constrictive pericarditis, 73, 73f calcified post-Ml left ventricular true aneursym, 74, 74f coronary artery calcification, 75, 75f left atrial wall calcification, 73, 73f mitral annular calcification, 74, 74f cardiomediastinal silhouette patterns aortic stenosis, 70, 70f coarctation, 7 1 , 7 1f mitral regurgitation, 72, 72f mitral stenosis, 72, 72f normal cardiac silhouette, 68, 68f pectus excavatum, 69, 69f pericardia! cyst, 69, 69f pseudocoarctation, 70, 70f valvar pulmonic stenosis, 7 1 , 7 1f Kussmaul sign, 76, 76f pulmonary vascular patterns cardiovascular disease states, 64-66 , 64f-66f pulmonary arterial hypertension (PAH) , 66, 66f pulmonary edema, 67, 67t pulmonary venous hypertension, 6 7 Chest x-ray (CXR), 463, 535 CHF. See Congestive heart failure Cholesterol absorption inhibitors for dyslipidemias, 788-790, 789t embolization syndrome, 7 1 7 Cholesterol and Recurrent Events (CARE), 791 Cholesteryl ester transfer protein (CETP) , 782 Cholestyramine, 789t Chronic mitral regurgitation, 480-482 , 480f, 48 l t, 482t Chronic renal dysfunction, 195 Churg-Strauss syndrome, 860 Chylomicrons, 78 1 , 785t CI. See Confidence interval CIBIS-II, 884 Cilostazol (Pletal), 899

INDEX Cimetidine, 192t, 9 14t CIMT. See Carotid intima-media thickness CIN. See Contrast induced nephropathy Ciprofibrate, 789t Circle of Willis, 268 Circulatory-assist devices, 608-6 1 0 C K . See Creatine kinase Clarithromycin, 9 1 3t, 9 1 S t CLARITY trial, S98 CLASSICS, 899 Clearance, 878 Clofibrate, 789t, 792t Clonidine (Catapres), 774t, 881 Clopidogrel (Plavix), S80, S82, 898 CM. See Cardiomyopathy CO. See Cardiac output COA. See Coarctation of aorta Coagulopathies, S94 Coanda effect, 87 Coarctation, Chest radiography, 7 1 , 7 1f Coarctation of aorta (CoA) , 28S, 28Sf, 286f, 822 HTN and, 773 pregnancy and, 822 Colesevelam, 789t Colestipol, 789t Collaborative Atorvastatin Diabetes Study (CARDS), 794-79S Collimators, 104 COMET (Carvedilol or Metoprolol European Trial) , 636 Complete blood count (CBC), S77 Complex cyanotic congenital heart disease, 823 Complex lesions acyanotic, 268-269 cyanotic, 269-270 Computed tomography pulmonary angiography (CT.PA) , 724 Computerized tomography (CT) cardiac, 144 clinical applications for, l 4S-l 46, l 4Sf evaluation of, 148, l 49t indications of, 144 limitations of, 146-148, 147t technical considerations for, l 44-l 4S Confidence interval (Cl) , S7 Congenital heart disease ( CHD) in adult complex lesions (acyanotic), 268-269 complex lesions (cyanotic), 269-270 concepts, 264-26S general management strategies, 270 hereditary and chromosomal defects, 277, 278t shunt lesions, 26S-267, 266t stenotic lesions, 267-268 operations for, 29 1 , 2 9 l t Congenital valve disease, 4 6 1 Congenitally corrected D-TGA, 287, 2 8 8 , 290, 290f Congestive heart failure (CHF) abnormal cellular mechanisms, 1 S9-160 adaptive responses, 1S6-1 S7, 1S6f autonomic nervous system dysfunction, 1S7, 1S7f biomarkers, 1S8 B-type natriuretic peptide and, l S8 counterregulatory systems, l S8 definition of, lSS demographics of, lSS index event and, 1SS-1S6, 1 S 6f inflammation and oxidative stress, l S8-l S9 left ventricular remodeling during, 1 S 9 , 1S9f peripheral vascular adaptations in, 160 prognosis, 160- 1 6 1 , 1 60t- 1 6 l t renin-angiotensin-aldosterone system and, 1 S7-1S8 salt/water retention during, 1S9 skeletal muscular adaptations in, 160 SNS and, 1S7 stage of, 1 SS-1 S 6 , 1 S 6f

summary on, 1 6 1 Conn syndrome, 8S8 Connective tissue disorders, 8S9-86 l CONSENSUS . See Cooperative North Scandinavian Enalapril Survival Study Constrictive pericarditis, 66Sf, 848-8S2, 848f, 849f, 8SOt, 8S lf, 8S2f Constrictive physiology, 664-66S , 664f Continuity equation, 86 Contrast induced nephropathy (CIN) , 697-698 Contrast materials, 70 1 , 70 l t Cooperative North Scandinavian Enalapril Survival Study (CONSENSUS) , 88S COPERNICUS. See Carvedilol Prospective Randomized Cumulative Survival Corlopam. See Fenoldopam Coronary angiography, SS3-SS4 Coronary arteries, 29-30, 29f, 30f calcification of, 7S, ?Sf Coronary artery bypass graft (CABG), 8 1 1-8 1 2 MP! after, 1 2 7 PC! vs. , S6S-S66 PCI/post, 692-693 Coronary artery disease (CAD), 140, l 4S-l 46, 14Sf, 379, 824 global burden of, S38, S39t risk factors novel, S47 traditional modifiable diabetes mellitus, S46 hypertension, S46 lipids, S46 obesity and metabolic syndrome, S46-S47 tobacco, S46 SE for, 1 0 1 , 1 02t in United States age and gender variation of, S39f, S4 l, S4 lf cost and health care resources for, S44-S4S , S4Sf prognosis and risks associated with, S4 3-S44, S44f-S4Sf racial and socioeconomic disparities of, S40f, S4 l-S42 , S42f silent myocardial ischemia/infarction, S39-S4 l , S40f, S4 lf temporal trends of, S42-S43, S43f young people and, S42 Coronary calcification on CT, 1 28-1 2 9 Coronary calcium scoring, 146 Coronary Drug Project (CDP) , 797 Coronary heart disease (CHD) morbidity/mortality and, 78 1 prevention of early trials for primary and secondary, 79 1 , 792t lipid-regulating trials for, 790-79 1 primary, landmark statin trials, 79 1 , 794 secondary, landmark statin trials, 79 1 , 794 subsequent statin trials for, 794-79S , 79St Coronary stenoses, 1 1 7-1 1 8 Corrigan pulse, 7 Corticosteroids, 193 Coumadin, 6 1 8 Coxiella burnetti, SO?, S08t Coxsackie adenovirus receptor (CAR) , 2 1 4 Creatine kinase (CK), 629 CREDO ( Clopidogrel for the Reduction of Events during Observation), 634, 693, 899 CREST syndrome, 8S9 CsA. See Cyclosporine CT. See Computerized tomography CT.PA. See Computed tomography pulmonary angiography CURE (Clopidogrel in Unstable Angina to Prevent Recurrent Events), S80-S8 1 , 634 CURRENT-OASIS 7 trial, S80 Cushing syndrome, 769, 8S8 CVDs. See Cardiovascular diseases CXR. See Chest x-ray Cyclic adenosine 3'S'-monophosphate (CAMP) , 1 19

923

Cyclic guanosine 3'S'-monophosphate (CGMP) , so Cyclosporine (CsA), 191, 218 Cystic medial degeneration, 709 D

Daclizumab (Zenapax), 190 Dallas criteria, 213, 220 Dalteparin (Fragmin), 89 1-892 dB. See Decibels DCM. See Dilated cardiomyopathy D-Dimer, 722 Decibels (dB), 78 Deep vein thrombosis (DVT) clinical presentation and diagnosis, 72 1-722, 722t D-Dimer, 722 diagnostic testing, 723 duplex ultrasonography, 723, 723t venography, 723 DEFINITE trials, 364 Dextrocardia, 302 Diabetes mellitus (DM) CAD and, S46 occurrence, S3S Diazoxide, 774t Digitalis effect of, 301 NA'-K' AT.Pase, 49 toxicity, 301 Digitalis Investigation Group (DIG) trial, 1 69 Digman (Lanoxin) , 38S, 904, 9 1 3 t Dihydropyridines, 893t calcium blockers, 763 Dilacor. See Diltiazem Dilated cardiomyopathy (DCM), 376, 8 1 1 definition of, 2 2 1 idiopathic, 22 1-222 inherited forms of, 222 SCD and, 376 VT and, 379 Diltiazem (Cardizem, Dilacor, Tiazac), 2S4, 893t, 894t, 9 l 3t-9 14t Dimensionless index, 464 DINAMIT trials, 602 Dipyridamole myocardial perfusion imaging and, 120, 1 20t for stress echocardiography, 99 Direct thrombin inhibitors (OT.Is) , S84, 727-728, 889-89 1 890t Disability-adjusted life year (DALY) , S38 Disopyramide, 388 Diuretics, 6 1 0 , 89Sf loop, 893 thiazide, 894, 89St DM. See Diabetes mellitus Dobutamine (Dobutrex) physiologic effects, 3S SE, 98-100 Dofetilide, 389 Dopamine (Inotropic), 89S-896 for AMI, 608 physiologic effects of, 60 1 , 608 Doppler echocardiography flow principles of, 8S-87, 86f imaging artifacts, 84-8S, 8Sf principles of jets, 87, 87f strain imaging, 83-84, 84f three-dimensional chocardiography, 8 1-82 ultrasound basic principles of, 78, 79f, 79t harmonic imaging, 8 1 , 8 lf imaging modalities, 80-8 1 , 8 lf tissue interaction, 78-79 transducers, 78-80 Doppler shift, 82-83 Doxazosin (Cardura), 882 Dressler syndrome, 603 >

924

INDEX

Dronedarone, 389-390 Droperidol, 9 1 6t Drug(s). See also specific drugs antiarrhythmic for AF, 384 class lA, 387-388 class IC, 389 class III, 389-390 inpatient vs. outpatient initiation of, 39 1 selection of, 390-3 9 1 cardiovascular pharrnacodynamic interactions of, 9 1 2 , 9 16t pharrnacokinetic interactions of, 9 1 1-9 1 2 , 9 12t-9 16t DT!s. See Direct Thrombin Inhibitors DTIT See Thrombin Inhibitors Trialist Collaborative Group Duke criteria, 5 1 0 Duke Treadmill Scores, 552, 553t Duplex ultrasonography, 723 Duroziez sign, 7 DVT See Deep vein thrombosis Dynacirc. See Isradipine Dynamic range, 8 1 Dyslipidemias apheresis therapies for, 790 classification of, 783, 783f primary, 783-786, 783f, 784-785t secondary, 786-787 diabetes and lipid management, 798 diagnosis of, 783, 783f fibrates monotherapy trials, 796-797 future directions for, 802 lipid lipoproteins and, 78 1-783 , 782f other, 795-796, 796t regulating trials, 790-79 1 metabolic syndrome (metS) , 787, 787t metS and, 787, 787t niacin monotherapy trials, 797 pharmacologic therapies for, 788-790, 789t regulating trials, 790-79 1 , 792t TLC for, 787-788, 788t treatment guidelines, 798-202, 799-80 l t treatments for, 787-788, 788t E EBCT See Electron-beam CT Ebstein anomaly, 286-287, 288f, 289f ECG. See Electrocardiogram Echocardiography, 25 1-253, 25 lf-252f, 553. See also Stress Echocardiography; Transthoracic echocardiography doppler flow principles of, 85-87, 86f imaging artifacts, 84-85, 85f principles of jets, 87, 87f strain imaging, 83-84, 84f three-dimensional chocardiography, 8 1 -82 ultrasound basic principles of, 78, 79f, 79t harmonic imaging, 8 1 , 8 lf imaging modalities, 80-8 1 , 8 lf tissue interaction, 78-79 transducers, 78-80 stress, 840 three-dimensional, 8 1-82 two-dimensional, 80 valve prostheses, complications of, 525-527, 525f-527f ECSS. See European Coronary Surgery Study Ectopic atrial bradycardia, 295 Ectopic atrial rhythm, 295 EDP See End-diastolic pressure EDPVR See End-diastolic pressure-volume relationship Edrophonium chloride (Tensilon) , 399 EDV See End-diastolic volume EECP See Enhanced external counterpulsation

EF See Ejection fraction Effective refractory period (ERP), 354 accessory pathway anterograde/retrograde AVN, 367 fast/slow AVN, 367 Ehlers-Danlos syndrome, 1-2 Eisenmenger syndrome, 822-823 Ejection fraction (EF) , 35 Ejection sounds (ES), 1 1 ELATE. See Extended Low-Intensity Anticoagulation for Thromboembolism Electrical coupling cells. See GAP junction Electrocardiogram (ECG), 628f, 630-63 1 for AS, 463 for ACSs, 577 board preparation, 294 exercise stress test current guidelines/essential evidence, 90-94, 92t, 93t, 94t diagnostic interpretation, 9 1 essential facts, 90 indications and contraindications, 90-9 1 , 92t, 93t methods, 9 1 prognostic interpretation, 9 1-94 summary on, 94 interpretation of assessment of standardization/recorded leads and, 294-295 atrial/ventricular rates/rhythms in, 295-297 measurement of cardiac intervals in, 297-299 P-wave/QRS-complex axes in, 297 P-wave/QRS-complex/T-wave morphologies in, 300-302 intracardiac, 353 twelve-lead, 294 Electrocardiography, 628-629, 628f Electron-beam CT (EBCT) , 1 44 Electrophysiology (EP), 447-448, 447t, 448t atrioventricular block and, 359-36 1 , 359f, 360f, 36lf atrioventricular conduction, 358-359 atrioventricular nodes and, 36 1-363, 36 lf, 362f, 363f basis of, 352-356, 353f, 354f, 355f, 356f bradyarrhythmia evaluation by, 356 indications for, 352 sinus node function and, 356-358, 356f, 357f, 358f summary, 370-3 72 tachyarrhythmia evaluation by, 363-370, 364f, 365f, 366f, 367f, 368f, 369f, 370f ELISA. See Enzyme-linked immunosorbent assay ELITE 1 . See Evaluation of Losartan in the Elderly study ELITE-II, 886 Embolic protection devices, 7 48 Enalapril, 859, 885t Enalaprilat, 77 4t End-diastolic pressure (EDP) , 34 End-diastolic pressure-volume relationship (EDPVR), 37 End-diastolic volume (EDV) , 34 Endless loop tachycardia, 4 31 Endocarditis Loffler, 242 prophylaxis, 465 Endomyocardial fibrosis, 242 Endothelin-1 (ET- 1 ) , 228 End-systolic pressure-volume relationship (ESPVR), 37 Enhanced external counterpulsation (EECP) , 567-568 Enoxaparin (Lovenox), 891-892 Enzyme-linked immunosorbent assay (ELISA), 722 EP See Electrophysiology EPHESUS trial (Eplerenone Post-Acute Myocardial Infarction Heart Failure Efficacy and Survival Study), 635

Epidemiology, clinical bias, 60 causality and validity, 60-6 1 confounding and interaction, 59-60, 59f data presentation/outcomes reporting, 5 7-59 exposures and outcomes, 55 multivariable regression, uses and pitfalls, 60 statistical tests, 56-5 7 types of clinical studies case reports, 55 case series, 55 case-control study, 56, 56f cohort study, 55-56 cost-effectiveness studies, 56 diagnostic tests, 56 genome-wide association studies (GWAS) , 56 meta-analysis, 56 prospective-versus-retrospective studies, 56 randomized controlled trials, 56, 56f EPILOG trial, 900 Epinephrine (Adrenaline) , 896 for AMI , 608 physiologic effects of, 35 EPISTENT trial, 900 Epoprostenol (Flolan), 903 Eptifibatide (Integrilin), 583, 900t Ergonovine, 535 ERP See Effective refractory period Erythromycin, 9 1 3t, 9 l 5t, 9 l 6t ES. See Ejection sounds ESETCID . See European Study on the Epidemiology and Treatment of Cardiac Inf! ammatory Disease Esmolol, 386t ESPIRIT study, 900 ESPVR. See End-systolic pressure-volume relationship Estrogen therapy, 835 European Coronary Surgery Study (ECSS) , 562 European Study Group on Diastolic Heart Failure, 245 European Study on the Epidemiology and Treatment of Cardiac Inf! ammatory Disease (ESETCID), 2 1 8 , 220 Evaluation of Losartan in the Elderly study (ELITE) , 166 Event monitoring, 43 1 , 43lf Everolimus (Certican) , 188, 1 92t, 193 Extended Low-Intensity Anticoagulation for Thromboembolism (ELATE), 727 External-beam radiation therapy (XRT) , 862 Extrastimulus, 354 Ezetimibe, 788, 789t F FA. See Fatty acids Fallot, tetralogy of, 286, 287f Familial Atherosclerosis Treatment Study (FATS), 797 Familial combined hyperlipidemia (FCH), 784t, 786 Familial Defective Apolipoprotein B lOO, 784t, 786 Familial hypercholesterolemia (FH) , 783 , 784t FATS. See Familial Atherosclerosis Treatment Study Fatty acids (FAs), 781 FC. See Free cholesterol FCH. See Familial combined hyperlipidemia FDA. See U .5. Food and Drug Administration FDG. See 18F-2-fluoro-2-deoxyglucose Felodipine, 560t, 893t, 894t, 9 1 2 t, 9 1 6 Fenofibrate, 789t, 796t, 797 Fenoldopam (Corlopam), 902 FFR See Fractional flow reserve FH. See Familial hypercholesterolemia Fibrates, 789t, 796-797 Fibrinolysis, 594t, 595, 603, 688t, 689t Fibrinolytic therapy for PVE, 526 for STEM!, 596, 596t

INDEX Fibroelastomas, 867, 868f Fibromas, 869f, 8689 Fibromuscular dysplasia (FMD), 860 Fick technique, 656, 658 Flecainide, 389 Flolan. See Epoprostenol Florinef, 45 1 , 45 l t Flow reserves, 1 1 7 18F-2-fluoro-2-deoxyglucose (FOG) , 106 Fluoroscopy, 676, 677f, 678f, 679f Fluvastatin, 789t FMD. See Fibromuscular dysplasia Fondaparinux (Am
The Cleveland Clinic Cardiology Board Review, 2nd Edition

Related documents

954 Pages • 654,509 Words • PDF • 68.9 MB

540 Pages • 178,753 Words • PDF • 130.2 MB

476 Pages • 304,757 Words • PDF • 7.6 MB

199 Pages • 95,987 Words • PDF • 7.9 MB

658 Pages • 155,366 Words • PDF • 6 MB

219 Pages • 104,115 Words • PDF • 1.3 MB

216 Pages • 73,410 Words • PDF • 557.2 KB

210 Pages • 52,688 Words • PDF • 750.8 KB

218 Pages • 71,680 Words • PDF • 1.2 MB

546 Pages • 158,218 Words • PDF • 143.6 MB

492 Pages • 160,274 Words • PDF • 4 MB